• Shuffle
    Toggle On
    Toggle Off
  • Alphabetize
    Toggle On
    Toggle Off
  • Front First
    Toggle On
    Toggle Off
  • Both Sides
    Toggle On
    Toggle Off
  • Read
    Toggle On
    Toggle Off
Reading...
Front

Card Range To Study

through

image

Play button

image

Play button

image

Progress

1/1199

Click to flip

Use LEFT and RIGHT arrow keys to navigate between flashcards;

Use UP and DOWN arrow keys to flip the card;

H to show hint;

A reads text to speech;

1199 Cards in this Set

  • Front
  • Back

1 Bone marTow cells from an organ donor are cultured in vitro at 3JOC in the presence of recombinant erythropoietin. A photomicrograph of a typical "burst-fonning unit" is shown in the image. This colony, commiued to the eryuu ocyre pathway of differentiation, represents an example of which of the following physiologic adaptations Lo transmembrane signalin g? ***(A) An·ophy ***(B) Dysplasia ***(C) Hyperplasia ***(D) Hypertrophy ***(E) Metaplasia

1 The answer is C: Hyperplasia. Hyperplasia is defined as an increase m the number of cells in an organ or dssue. Uke hypertrophy (choice D}. it is often a response to trophic signals or increased functional demand and is commonly a normal process. E1ylhroid hyperplasia is typically seen in people living at high aldtude. Low oxygen tension evokes the producdon of erythropoietin , which promotes the survival and proliferation of erylhwid precursors in the bone marrow. The cellular and molecular mechanisms that are responsible for hyperplasia clearly rela te to the control of cell p roliferation (i.e., cell cycle). None of the other choices describe increased numbers of cells. ***Diagnosis: Erythropoiesis, hyperp lasia
2 A 50-year-old clu-onic alcoholic presents to the emergency room with 12 hours of severe abdominal pain. The pain radiates to the back and is associated \ViLl1 an urge to vomit. Physical examination discloses exquisi te abdominal tenderness. LaboraLOJY studies show elev;ued semm amylase. vVhich of the following morphologic changes would be expected in the peripancreatic tissue of this patient? ***(A) Coagulative necrosis ***(B) Casem1s necrosis ***(C) Fat necrosis ***(D) Fibrinoid n ecrosis ***(E) Liquefactive necrosis

2 The answer is C: Fat necrosis. Saponification of fat detived from peripancreatic fat cells exposed to pancreatic enzymes is a typical feature of fat necrosis. Lipase, released from pancreatic acinar cells during an attack of acute pancreatitis, hydrolyzes fat into fatty acids and glycerol. Free fatty acids bind with calcium w fonn soaps, which is a process known as saponification Entry of calcium ions into the injured dssue reduces the level of calcium in blood. Hypocalcemia is, therefore, a typical finding in patients who had a recem bout of acll[e pancreatitis. Patients with acute pancreatitis experience sudden-m1Set abdominal pain, distention, and vomiting. The other choices are not typically seen in petipancreatic tissue following acute pancreatitis, although bquefactive necrosis (choice E) may be observed. ***Diagnosis: Acute pancreatitis

3 A 68-year-old man with a history of gastroesophageal reflu.x disease suffers a massive stroke and expires. The esophagus at autopsy is shown in the image. Histologic examination of lhe abnom1al tissue shows imestine-Uke epithehum composed of goblet cells and surface cells simHar to those of incompletely imestinalized gastric mucosa. There is no evidence of nuclear atypia. Which of lhe roUowing tenns best desc1ibes this morphologic response to persistent injury in the esophagus of this patient? ***(A) Acyplcal hyperplasia ***(B) Complex hyperplasia ***(C) Glandular metaplasia ***(D) Simple hyperplasia ***(E) Squamous metaplasia
3 The answer is C: Glandular metaplasia. The major adaptive responses of cells w sublethal injury are atrophy, hypertrophy, hyperplasia, metaplasia, dysplasia, and intracellular storage. Metaplasia is defined as the conversion of one differentiated cell pathway to another. ln this case, the esophageal squamous epithelium is replaced by columnar epith elium as a result of chronic gastroesophageal reflux. The lesion is characte1ized histologically by intestine-like epithelium composed of goblet cells and celts similar to those of incompletely intestinalized gastric mucosa. Squamous metaplasia (choice E) occurs in Lhe bronchial epithelium of smokers, among other examples. Choices A, B, and D are preneoplastic changes that are most often described in. the uterine endometrium of postmenopausal women. ***Diagnosis: Barrett esophagus, metaplasia
4 ACT scan of a 43-year-old \voman with a parathyroid adenoma and hyperparathyroidism reveals extensive caldum deposits in the lungs and kidney parenchyma. These radiologic findings are besL explained by which of the following mechanisms of disease? ***(A) Aneriosclerosis ***(B) DysLrophic calcificaLion ***(C) Granulomatous inflammation ***(D) Metastatic calcification ***(E) Tumor embolism
4 The answer is D: Metastatic calcification. Metastatic calcification is associated \.Vith an increased sennn calcium concentration (hypercalcemia). AbnosL any disorder that increases serum calcium levels can lead to calcification in the alveo lar septa of the lung, renal tu bules , and blood vessels. 111e patiem in this case had a parathyroid adenoma that produced large quantities of pa rathyroid honnone. Other examples of metasmtic calcificmion include multiple opacities in the cornea of a child given large amounts of vitamin D and pan ially calcified alveolar septa ln the lungs of a patient with breast cancer metastatic to bone. Breast can cer metastaSes to bone are often osteolytic and, therefore, accompanied by h ypercalcemia. Dystrophic calcification (choice B) has us migin in direct cell injury. Aneriosclerosis (choice A) is an example of dystrophic calcification. ***Diagnosis: Hyperparathyroidism, meLastatic calci.ficaLion
5 A 75-year-old woman wiLh Alzheimer disease dies of congesLive heart failure. The brain ar autopsy is shown in Lhe image. This patients brain exemplifies which of the foUowing responses to chronic injury? ***(A) Anaplasia ***(B) Atrophy ***(C) Dysplasia ***(D) HypeqJlasia ***(E) Hypertrophy
5 The answer is 8: Atrophy. Clinically, atrophy is recognized as diminution in the size or function of an organ. lt is often seen in areas of vascular insufficiency or chronic inflammation and may result from disuse. An·ophy may be thought of as an adaptive response to stress, in \.Vhicl1 the cell shuts down its di(fe rentiated functions. Reduction in the size of an organ may re flect reversible cell atrophy or may be caused by irreversible loss of cells. For example, atrophy of the brain in Lhis patient with Alzheimer disease is secondary to extensive cell death, and the size of the organ cannot be restored. This patients brain shows marked atrophy of the frontal lobe. The gyri are thilmed, and sulci are widened . Anaplasia (choice A) represems Jack of differentiated features in a neoplasm. ***Diagnosis: Alzheimer disease, atrophy
6 A 68-year-old woman with a history of heavy smoking and repeated bouts of pneumonia presents with a 2-\veek history of fever and productive cough A chest X-ray reveals a right lower lobe infiltrate A transbronchial biopsy confirms pneumonia and further demonstrates preneoplastic changes within the bronchial mucosa. Which of the following best characterizes the morphology of this bronchial mucosal lesion? ***(A) Abnormal pauern of cellular maturation ***(B) Increased numbers of otherwise normal cells ***(C) Invasiveness through Lhe basement membrane ***(D) Transfonnation of one differentiated cel1 type to another ***(E) Ulceration and necrosis of epithelial cell!s
6 The an-swer is A: Abnormal pattern of (ellular maturation. Cells that compose an epithelium exhibit uniformity of size and shape, and they undergo maturation in an orderly fashion (e.g., from plump basal cells to flat superficial cells in a squamous epithelium). When we speak of dysplasia, we mean ucat this regular appearance i.s disturbed by ( l ) variations in the size and shape o[ the cells: (2) enlargement, irregularity, and hyperchromatism of the nuclei; and (3) disorderly arrangemem of the cells within the epitheli1.1m. Dysplasia of dee bronchial epiucellum is a reaction of respiratory epithelium to carcinogens in wbacco smoke. It is pmemially reversible if the patiem swps smoking bm is considered preneoplastic and may progress to carcinoma. Choices B, D, and E are not preneoplastic changes. Invasiveness (choice C) connotes malignam behavior. ***Diagnosis: Pneumonia, dysplasia
7 A 64-year-old man with long-standing angina pectoris and arterial hypertension dies of spontaneous intracerebral hemorrhage. At autopsy, the heart appears globoid. The lefL vennide measures 2.6 em on cross section (shown in the image). This adaptation to chronic injmy was mediated p1imarily by changes in the intracellular concentration of which of the foliO\vi.ng components? ***(A) DNA ***(B) Glycogen ***(C) Lipid ***(D) mRNA ***(E) \Va ter
7_ The answer is D: mRNA. Hypenrophic cardiac myocytes have more cytoplasm and larger nuclet than normal celts. Although the elucidation o[ the ceUular and molecular mechanisms underlying the hypertrophic response is still actively pursued, it is clear that the final steps include increases in mRNA, rRNA, and protein. Hypenrophy resuli:S from r.ranscriprional regulation. Aneuploidy (chmce A) is nm a featu re of myofiber hypertrophy. 'vVater influx (choice E), which is typical of hydropic swelling in acme injury, is not a common feature of hypertrophy ***Diagnosis: Hypertrophic heart disease, hypertrophy
8 A 24-year-old woman contracts toxoplasmosis during her pregnancy and delivers a neonate at 3 7 weeks of gestation with a severe malfomtation of the central nervous system. MRI studies of the neonate reveal porencephaly and hydrocephalus. An X-ray fi lm of the head shows irregular densities in the basal ganglia. These X-ray findings are best explained by which of lhe following mechanisms of disease? ***(A) Amniotic fluid embolism ***(B) Dystrophic calcification ***(C) Granulomawus inflammation ***(D) Metastatic calcification ***(E) Organ immaturity
8 The answer is B: Dystrophic calcification. Dystrophic calci6cadon refl.ects underlying cell injury. Serum levels of calcium are normal, and the calcium deposits are located in previously damaged tissue. lnnautetine Toxoplasma infection affects approximately 0.1% of all pregnancies. Acute encephalitis in the fetus afflicted with TORCH syndrome may be associated with foci of necrosis that become calcified. Microcephaly, hydrocephalus, and microgyria are frequent complications of these inr.rau terine infections. Metastatic calcification (choice D) reflects an underlying disorder in cakium metabolism. ***Diagnosis: Dystrophic calcification
9 A 30-year-old man with AIDS-dementia complex develops acute pneumonia and dies of respiratory insufficiency. At autopsy. rnany central nervous system neurons display hydropic degeneration. This manifestation of sublethal neuronal injury was most likely mediated by impairment of which of the following cel1ular processes? ***(A) DNA synthesis ***(B) Upid peroxidation ***(C) Miwtic spindle assembly ***(D) Plasma membrane sodium transpon ***(E) Ribosome biosynthesis
9 The answer is D: Plasma membranesodium transport. Hydropic swelling reflecrs acute, reversible (sublethal) cell injury It results fTom impairment of cellular volume regulation, a process that controls ionic concentrations in the cytoplasm. This regulation, particularly for sodium, involves ( 1) the plasma membrane, (2) the plasnca membrane sodium pump, and (3) the supply of ATP. Injurious agents may imerfere with these membrane-regulated processes. Accumulation of sodium in the cell leads to an increase in water content to malmain isosmotic conditions, and the cell then swells. Lipid peroxidadon (choice B) is often a feature of irreversible cell injury. The other choices are unrelated to volume control. ***Diagnosis: Acute reversible injury
10 A 62-year-old man is brought to the emergency room in a disotiemed state. Physical examination reveals jaundice, splenomegaly, and ascites. Serum levels of ALT, AST, alkaline phosphatase, and bi lirubin are all elevated. A liver biopsy demonstrates alcoholic hepatitis with Mallory bodies. These cytoplasmic structures are composed of interwoven bundles of which of the following protelns' ***(A) 0.1-Antitrypsin ***(B) (3-Amyloid (A(3) ***(C) Intermediate filamenLS ***(D) Prion protein (PrP) ***(E) a.-Synuclein
10 The answer is C: Intermediate filaments. Hyaline is a term that refers to any material that exhibits a reddish, homogeneous appearance when stained with hematoxylin and eosin (H&E). Standard tem1inology includes hyaline anenolosclerosis, alcoholic hyali11e in the liver, hyaline membranes in the lung, and hyaline droplets in various cells. Alcoholic (Mallory) hyaline is composed of cytoskeletal imennediate filamenrs (cywkeratins), whereas pulmonary hyaline membranes consist of plasma proteins deposited in alveoli. Stmcturally abnonnal a.,-andrrypsln molecules (choice A) accumulate in the liver of patienrs with fJ.1-amitrypsin defidency. a. -Synuclein (choice E) accumulates in neurons in the substantia nigra of patients with Pa rkinson disease. ***Diagnosis: Alcoholic liver disease
11 A 65-year-old man suffers a heart attack and expires. Examination of the lungs at auwpsy reveals numerous pigmented nodules scatte red throughout the parenchyma (shO\vn in the image). What is the appropriate diagnosis? ***(A) Anthracosis ***(B) Asbestosis ***(C) Hemosiderosis ***(D) Sarcoidosis ***(E) Sillcosis
11 The answer is A: Anthracosis. Amhrncosis refers to the storage of carbon particles in the ltmg and regional lymph nodes. These panicles accumulate in alveolar macrophages and are also o·ru1sponed to hilar and mediastinal lymph nodes, where the indigestible matenal is stored indefinitely within Lissue macrophages. Although the gross appearance of the lungs of persons with anthracosis may be alarming, the condition is innocuous. Workers \Vho mine hard coal (anthracite) develop pulmonary fibrosis, owing to the presence of toxic/fibrogenic dusts such as silica. This type of pneumoconiosis is more properly classified as anthracosilicosis. Hemosiderosis (choke C) represents intracellular storage of iron (hemosiderin). The other choices are not associated with dark pigmentation in Lhe lung ***Diagnosis: Pneumoconiosis, anthracosis
12 A 32-year-old woman whh poorly controlled diabetes mellims delivers a healthy boy at 38 weeks of gestation As a result of maternal hyperglycenlia during pregnancy, pancreatic islets in the neonate would be expected co show which of Lhe following morphologic responses to injury? ***(A) Alrophy ***(B) Dysplasia ***(C) Hyperplasia ***(D) Metaplasia ***(E) Necrosis
12 The answer is C: Hyperplasia. lnfanrs of diabetic mothers show a 5% to 10% incidence of major developmen tal abnormalities, including anomalies of the heart and great vessels and neural tube defects. The frequency of these lesions relates to the control of maternal diabetes during early gestation. Dming fetal developme11t , the islet cells of the pancreas have proliferative capacity and respond to increased demand for insulin by undergoing physiologic hyperplasia_ Femses exposed LO hyperglycemia in utero may develop hyperplasia of the pancreatic ~ cells, which may secrete insulin autonomously and cause hypoglycemia at binh. Metaplasia (choice D) is defined as the conversion of one differentJaLed cell pathway to another. ***Diagnosis: Diabetes mellitus
13 A 59-year-old female alcoholic is brought to the emergency room with a fever (38. f 0 C/l03°F) and foul-smelling breath. The patiem subsequently develops acULe bronchopneumonia and dies of respiratory msufficiency. A pulmonary abscess is identified at aULopsy (shown in the image). Histologic examination of the wall of this lesion would most likely demonstrate which of the following pathologic changes? ***(A) Caseous necrosis ***(B) Coagulative necrosis ***(C) fat necrosis ***(D) Fibrinoid necrosis ***(E) UquefaCLive necrosis
13 The answer is E: Liquefactive necrosis. When the rate of dissolmion of the necrotic cells is faster than the rate of repair, the resulting morphologic appearance is te rmed liquefactive necrosis. The polymorphonuclear leukocytes of the acme inflammatory reaction are endowed with potent hydrolases that are capable o [ digesting dead cells. A sharply localized collection or these acute inflammatory cells in response [0 a bacterial infection produces rapid death and dissolution of tissue. The resu It is often an abscess defined as a cavity formed by liquefactive necrosis in a solid tissue. Caseous necrosis (choice A) is seen in necrotizing granulomas. In coagulative necrosis (choice B), the outline of the cell is retained. Fat (choice C) is not present in the lung parenchyma. Fibrinoid necrosis (choi ce D) is seen in patien ts \Vith necrotizing vasculitis. ***Diagnosis: Puhnonary abscess, liquefactive necrosis
14 A 20-year-old man from China is evaluated for persistent cough, night S\"leaLS, low-grade fever, and general malaise. A chest X-ray reveals findings "consistent wilh a Ghon complex." Sputum culLUres grow acid-fast bacilli. Examination of hilar lymph nodes in Lhis patient would most likely demonstrate which of the follov.ring pathologic changes? ***(A) Caseous necrosis ***(B) Coagulative necrosis ***(C) Fat necrosis ***(D) Fibrinoid necrosis ***(E) Liquefactive necrosis
t4 The answer is A: Caseous necrosis. Caseous necrosis is a characteristic of primary tuberculosis, in which the necrotic cells fail to retain their cellu lar outlines. They do not disappear by lysis , as in liquefactive necrosis (ch oice E), but persist indefinitely as amorphous, coarsely granular, eosinophilic debris. Grossly, this debtis resembles clumpy cheese, hence the name caseous necrosis. Primaty tuberculosis is often asymptomatic or presents with nonspecific symptoms, such as low-grade fever, loss of appetite, and occasional spells or coughing. The Ghon complex includes parenchymal consolidation and ipsilateral enlargement o[ hilar lymph nodes and is often accompanied by a pleural effusion. Fibnnoid necrosis (choice D) is seen in patients with necrouzing vasculitis. ***Diagnosis: Tuberculosis, Mycobacterium tuberculom
15 A 31-year-old woman complains of increased vaginal discharge of 1-month duration. A cervical Pap smear is shown in the image. Superficial epithelial cells are identified witl1 anows. When compared to cells from the deeper intermediate layer (top), the nuclei of Lhese superficial cells exhibit which of the following cytologic feam res? ***(A) Karyolysis ***(B) Karyorrhexis ***(C) Pyknosis ***(D) Segmentation ***(E) Vi ral inclusion bodies
15 The answer is C: Pyknosis. Coagulative necrosis reters to light microscopic alterations in dying celJs. When stained with the usual combination of hematoxylin and eosin, the cytoplasm of a necrotic cell is eosinophilic. The nucleus displays an initial dumping of chromatin followed by its redistribution along the nuclear membrane. In pyknosis, the nucleus becomes smaller and stains deeply basophilic as chromatin clumping continues. Karyorrhexis (choice B) and karyolysis (choice A) represent further steps in the fragmentation and dissolution of the nucleus. These steps are not evident in the necrotic cells shown in this Pap smear. ***Diagnosis: Cervical intraepithelial neoplasia, pyknosis
16 A 30-year-old woman suffers a tonic-clonic seizure and presents with deli1ium and hydrophobia. The patient stales that she was bitten on the hand by a bat about l momh ago. The patient subsequently dies of respiratory failure. VLral particles are found throughout the brainstem and cerebellum at autopsy. ln addition to direct vi ral cytotoxicity, the necrosis or virally infected neurons in this patient was mediated primarlly by which of the following mechanisms? ***(A) Histamine release from mast cells ***(B) Humoral and cellular immunity ***(C) Neutrophil-mediated phagocytosis ***(D) Release of oxygen radicals from macrophages ***(E) Vasoconstriction and ischeml8
16 The answer is 8: Humoral and cellular immunity. Both humoraJ and cellu lar anns of the immune S)1Stem protect against the hamuul effects of viral infections. Thus, the presentation of viral proteins to the immune system immunizes the body against the invader and elicits both killer cells and the production of antiviral antibodies. These anns of the immune system eliminate virus-infected cells by either inducing apoprosis or directing complement-mediated cytolysis_ In this patiem, the rabies virus entered a pe1ipheral nerve and was transported by retrograde axoplasmic flow to the spinal cord and b rain. The infl ammation is centered in the brainstem and spills into the cerebellum and hypothalamus. The other choices are seen in acme inflammation, but they do nm represent antigen-specific responses tO viral infections. ***Diagnosis: Rabies
17 A 52-year-old woman loses her right kidney following an auto· mobile accident. ACT scan of the abdomen 2 years later shows marked enlargement of the left kidney. The renal enlargement is an example o[ which of the following adaptations? ***(A) Atrophy ***(B) Dysplasia ***(C) Hyperplasia ***(D) Hypemophy ***(E) Metaplasia
17 The answer is D: Hypertrophy. Hypertrophy is a response to trophic signals or increased functional demand and is commonly a normal process. For example, if one kidney is rendered inoperative because of vascular occlusion, the contralateral kidney hypertrophjes to accommodate increased demand. The molecular basis of hypertrophy reflects increased expression of growth-promoting genes (protooncogenes) such as myc,Jos , and ra.s . Hyperplasia (choice C) of renal tubular cells may occur, but enlargement of the kidney in this patient is best referred to as hypertrophy (Le., increased organ size and function). ***Diagnosis: Hypertrophy
18 An 82-year-old man has profound bleeding fTom a peptic ulcer and dies of hypovolemic shoe k The liver at autopsy displays centrilobular necrosis. Compared to viable hepatocytE$, the necrotic cells contain higher intracellular concemrations of which of [he following? ***(A) Calcium ***(B) Cobal[ ***(C) Copper ***(D) lron ***(E) Selemum
18 The answer is A: Calcium. Coagulative necrosis is characterized by a massive influx of calcium imo the ceU. Under nonnal circumstances, the plasma membrane maintains a s teep gradiem of calcium ions, whose concentration in inters titial fluids is 10,000 times higher than thm inside the cell. Irreversible cell injury damages the plasma membrane, which then fails w maintain this gradient, allowing the influx of calcium into the cell. The other chokes would most likely be released upon cell death. ***Diagnosis: Coagulative necrosis
19 A 2.8-year-old woman is pinned by falling deb1ts during a hurricane. An X-ray film of the leg reveals a compound fracmre of the right tibia. The leg is immobilized in a casL for 6 ·weeks. vVhen the cast is removed, the patiem nQ[ices that her rig]1[ leg is weak and visibly smaller in circumference than the left leg. Which of the following terms best describes tllis change in the patients leg muscle? ***(A) Atrophy ***(B) Hyperplasia ***(C) Metaplasia ***(D) Ischemic necrosis ***(E) lneversible cell injury
19 The answer is A: Atrophy. The most common fonn of atrophy follows reduced functional demand. for example, after immobilization of a limb in a cast as treatmem for a bone fracru re, muscle cells atrophy. and muscular s trength is reduced. The expresston of differemimion genes is repressed. On reswration of normal conditions, atrophic cells are fully capable of resuming their differentiated functions; size increases to nonnal, and specialized hmcdons, such as p rotein synthesis or comractile force, rerum to their original levels. Ischemic necrosis (ch oice D) is typically a complication of vascular insufficiency. In-eversible inJury to skeletal muscle (choice E) would be an unlikely complication of bone fracture. ***Diagnosis: Atrophy, bone fracmre
20 A 70-year-old man is hospitalized after suffering a mild stroke. While in the hospital, he suddenly develops crushing substernal chest pain. Analysis of serum proteins and ECG con.finn a diagnosis of acute myocardial infarction. The patient subsequemly develops an anhythmia and expires. A cross section of the left vemricle at autopsy is shown in the image Histologic examination of the affected heart muscle would demonstrate whkh of the following morphologic changes7 ***(A) Caseous necrosis ***(B) Coagulative necrosis ***(C) Fat necrosis ***(D) Fibrinoid necrosis ***(E) Uquefactive necrosis
20 The answer is B: Coagulative necrosis. Ischemic necrosis of cardiac myocytes is the leading cause of death in the Western world. In brief, the interruption of blood supply to the bean decreases the delivery of 0 2 and glucose. l.ack of 0 2 impairs mitochondtial electron transport, thereby decreasing ATP synthesis and facilitating the production of reactive oxygen species. Mitochondrial damage promotes the release of cytochrome c to the cytosol, and the ce ll dies. The moq:>hologic appearance of the necrotic cell has traditionally been tem1ed coagulative necrosis because of its similatity w the coagulation of proteins that occurs upon heating. ***Diagnosis: Myocardial infarction, coagulative necrosis
21 Which of the following hiswlogic featu res would provide definitive evidence of necrosis in the myocardium of the patiem described in Question 20? ***(A) Disaggregation of polyribosomes ***(B) Increased imracellular volume ***(C) Inllux of lymphocytes ***(D) Mitochondrial swelling and calcification ***(E) Nuclear fTagmemation
21 The answer is E: Nuclear fragmentation. Nuclear fragmentation (karyorrhexis and karyolysis) is a hallmark of coagulative necrosis. Choices A, B, and 0 are incorrect because they are fearures of both reversibly and ineversibly injured cells. Lymphocytes (choice C) are a hallmark of chronic inflammation. ***Diagnosis: Myocardial infarction
22 A 90-year-old woman with mild diabetes and Alzheimer dis· ease dies in her sleep. At autopsy, hepatocytes are noted to contain golden cytoplasmic granules that do not stain with Prussian blue. Which of the following best accounts for pigment accumulation in the liver of this patient? ***(A) Advanced age ***(B) Alzheimer disease ***(C) Congestive heart fai]ure ***(D) Diabetic keLOacidosis ***(E) Hereditary hemochromawsis
22 The answer is A: Advanced age. Substances that cannot be metabolized accumulate in cells. Examples include (I) endogenous substrates that are not processed because a key enzyme is missing (lysosomal storage diseases), (2) insoluble endogenous pigments (Lipofuscin and melanin), and (3) exogenous paniculates (silica and carbon}. lipofuscin is a "wear and tear" pigment of aging that accumulates in organs such as the brain, hean, and liver. None of the other choices are associated with lipofuscin accumulation. ***Diagnosis: Aging, hpofuscin
23 Which of the following mechanisms of rusease best describes the pathogenesis of pigment accumulation in hepatocytes in the patient described in Question 22? ***(A) Degradation of melanin pigments ***(B) Inhibition of glycogen biosynthesis ***(C) Malabsorption and enhanced deposition of iron ***(D) Peroxidation of membrane lipids ***(E) Progressive oxidation of bilirubin
23 The answer is 0: Peroxidation of membrane lipids. Lipofuscin is found in lysosomes and contains peroxidation products of unsaturated fatty acids. The presence of this pigment is thought w reflect continuing llpid perm.'. i dation of cellular membranes as a result of inadequate defenses against activated oxygen radicals. None of the other mechanisms of disease leads to the formation and accumulation of li pofuscin granules. ***Diagnosis: Lipofuscin, intracellular swrage disorder
24 A 45-year-old man presents with increasing abdominal girth and yellow discoloration of his skin and sclera. Physical examination reveals hepatomegaly and jaundice. A Pntssian blue stain of a liver biopsy is shown in the image What is the major intracellular iron storage protein in this patiem's hepawcytes? ***(A) Bilirubin ***(B) Haptoglobin ***(C) Hemoglobin ***(D) Hemosiderin ***(E) Transferrin
24 The answer is D: Hemosiderin. Hemosiderin is a partiaJly denatured form of ferritin that aggregates easily and Ls recognized microscopically as yellow-brmvn granules in the cytoplasm, which turn blue with the Prussian blue reaction. In hereditary hemochromatosis, a genetic abnormality of iron absorption in the small intestine, excess iron is s tored mostly in the form of hemosiderin, primarily in the liver. Hemoglobin (choice C) is the iron-containing pigmem of RBCs. Bilirubin (choice A) is a product of heme catabolism that may accumulate in liver cells but does not srain wi.lh Prussian blue_ Transferrin (choice E) binds serum iron. ***Diagnosis: Hereditary hemochromatosis
25 A 60-year-old man \Vith ch ronic cystitis complains of urinary fTequency and pelvic discomfort. Digital rectal examination is unremarkable. Biopsy of the bladder mucosa reveals foci of glandular epithelium and chronic Jnnammatory cells_ No cytologic signs of atypia or malignancy are observed. Whkh of the follo.,ving terms best describes the morphologic response to chronic injury in this patient? ***(A) Atrophy ***(B) Dysplasia ***(C) Hyperplasia ***(D) Hypemophy ***(E) Metaplasia
25 The answer is E: Metaplasia. Metaplasia of transitional epithelium to glandular epithelium is seen in patients with chronic innammation of the bladder (cystitis glandularis). Metaplasia is considered to be a protective mechanism, but it is not necessarily a ham1less process. For example, squamous metaplasia in a bronchus may protect against injmy produced by tobacco smoke, but it also impairs the production of mucus and ciliary clearance of debris. Funhermore, neoplastic transfo rmation may occur in metaplastic epithelium Lack of cytologic eVIdence for atypia and neoplasia rules out dysplasia (choice B) ***Diagnosis: Chronic cystitis, metaplasia
26 A 60-year-old man is rushed to the hospitaJ with acme liver failure. He undergoes successful orthotopic liver transplamalion: however, the transplanted Liver does not produce much bile for the first 3 days. Poor graft f1.mc tion in this patient is thought LO be the resuh of "reperfusion injury." Which of the following substances was the most likely cause of reperfusion injury in this patients transplanted liver? ***(A) Cationic proteins ***(B) Free ferric iron ***(C) Hydrochlorous acid ***(D) Lysosomal acid hydrolases ***(E) Reactive oxygen species
26 The answer is E: Reactive oxygen species. lschetnialreperfusion (l!R) injury is a common clinical problem that atises in the setting of occlusive cardiovascular disease, infection, transplantation, shock, and many other circumstances. The genesis of liR injury relates w the inte1play between transient ischemia and the re-establishment of blood flow (reperfusion)_ Initially. ischemia produces a type of cellular damage that leads w the generation of free radical species. Subsequently, reperfusion provides abundant molecular oxygen (0 ,) w combine with fTee radicals to fonn reactive oxygen specles. 0>..-ygen radicals are fmmed inside cells through the xanthine oxidase pathway and released from activaled neutrophils. ***Diagnosis: Myocardial infarction
27 A 68-year-otd woman with a history of hyperlipidemia dies of cardiac arrhythnua following a massive hearr attack. Peroxidation of which of the following molecules was primati ly responsible for causing the loss of membrane imegrity in cardiac myocytes in this patiem? ***(A) Cholesterol ***(B) Glucose transport proteins ***(C) Glycosphingolipids ***(D) Phospholipids ***(E) Sodium-potassium ATPase
27 The answer is D: Phospholipids . During lipid peroxidation, hydro>..'yl radicals remove a hydrogen atom from the unsaturated fatty acids of membrane phospholipids. The lipid radicals so f01med reacL with molecular o:-... ygen and form a hpid peroxide radical. A chain reaction is initiated. lipid peroxides are unstable and break dow·n into smaller molecules. The destruc· tion of the unsaturated fatty acids of phospholipids results in a loss of membrane integrity. The other choices represent targets for reactive oxygen species, but protein cross-linking (choices B and E) does not lead to rapid loss of membrane integrity il1 patients with myocardia] infarction. ***Diagnosis: Myocardial infarction
28 A 22-year-old construction worker sticks himself with a sharp , rusty nail. Within 24 hours, the wound has enlarged to become a 1-cm sore that drains thick, purulent matetial. This skin wound illustrates ·which of the following morphologic types of necrosi5? ***(A) Caseous necrosis ***(B) Coagulative n ecrosis ***(C) Fat n ecrosis ***(D) Fibnnoid necrosis ***(E) Liquefactive necrosis
28 The answer is E: Liquefactive necrosis. Polymorphonuclear leukocytes (segmented nemrophils) rapidly accumulate at sites o[ injury They are loaded with acid hydrolases and are capable o[ digesting dead cells. A localized collection of these inOammatory cells may create an abscess with central liquefaction (pus). Uquefacdve necrosis is also commonly seen in the brain. Caseous necrosis (choice A) is seen in necrotizing granulomas. Fat necrosis (choice C) is typically encountered in patients with acute pancreatitis. Fibrinoid necrosis (choice D) is seen in patients with necrotizing vasculitis. ***Diagnosis: Abscess, acme inflammation
29 A 4 2-year-old man undergoes liver biopsy for evaluation of the grade and stage of his h epatitis C virus infection. The biopsy reveals swollen (ballooned) hepatocytes and moderate lobular innammatory activity (shown in the image). The arrow identifies an acidophilic (Counci1man) body. Which of the following cellular processes best accounts for the presence or scattered acidophilic bodies in this liver biopsy? ***(A) Aggregation of intermediale filament proLeins ***(B) Apopwtic cell dealh ***(C) Coagulative necrosis ***(D) Collagen deposition ***(E) Intracellular viral inclusions
29 The answer is B: Apoptotic cell death. Apoptosis is a programmed pathway o[ cell death that is triggered by a variety of exn·acelJular and intracellular signals. lt is often a selfdefense mechanism, desu-oying cells that have been infected with pathogens or those in which genomic alterations have occun ed. Arter staining with hematoxylin and eosin, apopLOtic cells are visible under the light microscope as acidophilic (CounCilman) bodies. These deeply eosinophilic su-ucrures represent membrane-bound cellular remnants that are extruded into the hepatic sinusoids. The other choices do not appear as acidophilic bodies. ***Diagnosis: Viral hepatitis
30 Which of the following biochemical changes characterizes the fo rmation of acidophilic bodies in the patient described in Question 29? ***(A) Fragmenration of DNA ***(B) Loss of tumor suprnessor protein p53 ***(C) Mitochondrial swelling ***(D) Synthesis of arachidonic acid ***(E) Triglyceride accumulation
30 The answer is A: Fragmentation of DNA. Fragmentation of DNA is a hallmark of cells undergoing both necrosis and apoptosis, but apoptotic cells can be detected by demonstrating nucleosomal "]addering. n This pattern of DNA degradation is characteristic of apoptotic cell death. It results from the cleavage or chromosomal DNA at nucleosomes by endonucleao:;es. Since nucleosomes are regularly spaced along the genome, a pauem of regular bands can be seen when fragments of cellular DNA are separated by electrophoresis. The other choices are associated with cell injmy, but they do nm serve as distinctive markers of programmed cell death. ***Diagnosis: Viral hepatitis
31 A 56-year-old woman with a history of hyperlipidemia and hypenension develops progTessive, right renal artery stenosis. Over time, this patients right kidney is Likety LO demonstrate which of the following morphologic adaptations to partial ischemia? ***(A) Atrophy ***(B) Dysplasia ***(C) Hyperplasia ***(D) Hypertrophy ***(E) Neoplasia
31 The answer is A: Atrophy. Inter fe rence with blood supply to tissues is known as ischemia. Total ischemla results in cell death. Panial ischemia occurs after incomplete occlusion of a blood vessel or in areas of inadequate collateral circulation. This results in a chronically reduced oxygen supply, a conclition often compatible with cominued cell viability Under such circumstances, cell atrophy is common. For e..'<ample, it is frequently seen around !he inadequately perfused margins of infarcrs in the hean, braiJ1, and kidneys. None of the other choices desc1ibe decreased organ size and funccion. ***Diagnosis: Renal artery stenosis
32 A 5-year-old boy suffers blum trauma to the leg in an automobile acndem. Six months later, bone u·abeculae have formed v.<ithin the striated skeletal muscle at 1.he site of tissue injury. This pathologic condition is an example of which of the fol lowing morphologic adaptations lo injury? ***(A) An·ophy ***(B) Dysplasia ***(C) Metaplasia ***(D) Metastanc calcification ***(E) Dystrophic calcification
32 The answer is C: Metaplasia. Myositis ossificans is a disease characterized by formation of bony trabeculae within sui ated muscle. lt represents a form of osseous metaplasia (i.e., replacement of one differentiated tissue with another type of nonnal differentiated tissue) Although dystrophic calci fica tion (choice E) frequently occurs at sites of prior injUJy, it does not lead lO the formation of bone trabeculae. ***Diagnosis: Myositis ossificans, metaplasia
33 A +3-year-old man presents with a scaly, erythematous lesion on. the dorsal smface of his left hand. A skin biopsy reveals atypical keratinocytes filling the entire thickness of the epiderntis (shown in the image) The arrows point to apoptotic bodies. 'Which of the following proteins plays the most in1ponam role in mediating programmed cell death in this patient's skin cancer? ***(A) Catalase ***(B) Cymchrome c ***(C) Cytokeratins ***(D) Myeloperoxi dase ***(E) Superoxide dismutase
33 The answer is B: Cytochrome c. The miwchondrial membrane is a key regulator of apopwsis. When mitochondrial pores open, cytochrome c leaks out and activates Apaf-1 , which convens procaspase-9 to caspase-9, resulting in the activation of dmvns tream caspases {cysteine proteases). These effector caspases cleave target proteins, including endonucleases nuclear proteins, and cytoskeletal proteins to mediate the va1ied morphological and biochemical changes that accompany apoptosis . Reactive oxygen species (related to choices A, D, and E) are uiggers of apoptosis, bUl they do not mediate programmed cell death. ***Diagnosis: Apopmsis, squamous cell carcinoma o£ skin
34 A 16-year-old girl with a history of suicidal depression swallows a commercial soiYent. A liver biopsy is performed w assess !.he degree of damage to the hepatic parenchyma. Histologic examinarion demonstrates severe swelling of the cemrilobular hepatocytes (shown in the image). Which of the follmving mechanisms of disease best accounts for the reversible changes nored in this liver biopsy? ***(A) Decreased stores of imraceUular ATP ***(B) Increased storage of triglycerides and free fatty acids ***(C) lmracytoplasmic rupture oflysosomes ***(D) Mitochondrial membrane permeability tTansition ***(E) Protein aggregation due to increased cytosolic pH
34 TheanswerisA:DecreasedstoresofintracellularATP. Hydropic swelling may result from many causes, including chemicaJ and biologic wxjns, infections. and ischemia. lnjmious agen ts cause hydropic swelling by (1) increasing the penneabiliry of the plasma membrane w sodium; (2) damaging the membrane sodium-potassium ATPase (pump); or (3) interfering with the synthesis of ATP, thereby depriving the pump of its fuel. The mher choices are inconect because they do not regulate concentrations of intracellu lar sodium. ***Diagnosis: Hydropic swelling, hepatotoxicily
35 A 40-year-old man is pulled from the ocean after a boadng accident and resuscitated. Six hours later, the patiem develops acme renal failure. Kidney biopsy reveals evidence of kaiyorrhexis and karyolysis in renal tubular epithelial cells. Whkh of the following biochemical evems preceded Lhese pathologic changes? ***(A) Activation of Na•fl<+ ATPase ***(B) Decrease in intracellular calcium ***(C) Decrease in intracellular pH ***(D) Increase in ATP production ***(E) In crease in intracellular pH
35 The answer is(: Decrease in intracellular pH. During periods of ischemia, anaerobic glycolysis leads w the overproduction of lactate and a decrease in intracellular pH. Lack of 0 , during myocardial ischemia blocks the production of ATP Pyruvate is reduced to lactate in the cytosol and lowers intracellular pH. The acidification of the cytosol initiates a downward spiral of events that propels the cell toward necrosis. The other choices point to changes in the opposite direction of what would be expected in irreversible cell injury. ***Diagnosis: Acute mb ular necrosis
36 A 58-year-old man presents \Vith symptoms of acme renal failure. His blood pressure is 220/130 mm Hg (malignam hypertension). While in the emergency room, the patient suiiers a stroke and expires. Microscopic e:<amination of the kidney a£ autopsy is shown in the image Which of the following morphologic changes accoums for the red material in the wall of Lhe anery? ***(A) Apoprosis ***(B) Caseous necrosis ***(C) Fal necrosis ***(D) Fibrinoid necrosis ***(E) liquefactive necrosis
36 The answer is D: Fibrinoid necrosis. FibJinoid necrosis is an alteration of injured blood vessels, in which the insudation and accumulation of plasma proteins cause the wall to stain lntensely with eosin. The other choices are not typically asso· dated directly with vascular injury. ***Diagnosis: Malignant hypertension, fibrinoid necrosis
37 A 10-year-old girl presents with advanced features of progeria (patient shown in the image). This child has inherited mutations in the gene that encodes which of the following types of intracellular proteins? ***(A) Helicase ***(B) lanlin ***(C) Oxidase ***(D) Polymerase ***(E) Topoisomerase
37 The answer is B: Lamin. Hutchinson-Gilford progeria is a rare genetic disease characterized by early cataracts, hair loss, arro· phy of the skin, osteoporosis, and atherosclerosis. This phe· notype gives the impression of premature aging in ch ildren. Progeria is one of many diseases caused by mulations in the human lamin A gene (LMNA). Lamins are intennediate filament proteins that fom1 a fibrous meshwork beneath the nuclear envelope. Defective lamin A is thought to make the nucleus unstable, leading to cell injury and death. Mutations in the other genes are not linked to Hutchinson-Gilford progeria syndrome. ***Diagnosis: Progeria
38 A 32-year-old woman develops an Addisonian crisis (acme adrenal insufficiency) 3 months after suffering mas.sive hemorrhage during the delivery of her baby. A CT scan of the abdomen shows small adrenal glands. Which of the following mechanisms of disease best accoums for adrenal atrophy in this patient? ***(A) Chronic inflammation ***(B) Chronic ischemia ***(C) HemotThagic necrosis ***(D) Lack of trophic signals ***(E) Tuberculosis
38 The answer is D: Lack of trophic signals. Atrophy of an organ may be caused by intenuption of key trophic signals. Postpartum in farction of the anterior pitui.tary in this patient resulted in decreased production of adrenocorticotropic hormone (ACTH, also termed corticotropin). Lack of corticotropin results in atrophy of the adrenal cortex, which leads to adrenal insufficiency: Symptoms of acute ad renal insufficiency (Addisonian crisis) include hypotension and shock, as well as weakness, vomiting, abdominal pai11, and lethargy. The otJ1er choices are unlikely causes of postpartum adrenal insufficiency: ***Diagnosis: Sheehan syndrome, adrenal insufficiency
39 A +7-year-old man vvith a history of heavy smoking complains of chronic cough. A "coin lesion" is discovered in his right upper lobe on chest X-ray. Bronchoscopy and biopsy fall to identify a mass, but the bronchial mucosa displays squamous metaplasLa. What is the most likely outcome o[ this morphologic adaptation if the patient stops smoking? ***(A) Atrophy ***(B) Malignant transfonnadon ***(C) Necrosis and scarring ***(D) Persistence throughout life ***(E) Reversion w nonnal
39 The answer is E: Reversion to normal. Metaplasia is almost invariably a response to persistent inj ury and can be thought of as an adaptive mechanism. Prolonged exposure of the bronchi to tobacco smoke leads w squamous metaplasia of the bronchial epithelium. Unlike malignancy (choice B) and necrosis with scarring (choice C), metaplasia is usually fully reversible. lf the source of injury in this patient is removed (the patient stops smoking), then the merap11sdc epithelium \vil1 eventually return 1.0 nonnaL ***Diagnosis: Chronic bronchitis, metaplasia
40 A <50-year-old fanner presems "''llh multiple patches of discoloration on his face Bjopsy oflesional skin reveals actinic kerarosis. Which of the following terms best describes this response of the skin to chronic sunlight exposure? ***(A) Atrophy ***(B) Dysplasia ***(C) Hyperplasia ***(D) Hyperu-ophy ***(E) Metaplasia
40 The answer is 8: Dysplasia. Actinic keraLOsis is a fom1 of dysplasia in sun-exposed skin. HislOiogically, such lesions are composed of atypical squamous cells, which vary in size and shape. They show no signs of regular maturation as the cells move from the basal layer of the epidermis to the surface. Dysplasta is a preneoplastic lesion, in the sense that it is a necessa1y stage in the multistep evolution to cancer. However, unlike cancer cells, dysplastic cells are not entirely amonomous, and the histologic appearance of the tissue may still revert to normal. None of the other choices represent preneoplastic changes in sun-e}..-posed skin. ***Diagnosis: Actinic keratosis, dysplasia
41 A 59-year-old \VOmaJ1 smoker complains of intermittent blood in her urine. U1inalysis confirms 4+ hematuria. A CBC reveals increased red cell mass (hematocrit). ACT scan demonstrates a 3-cm renal mass, and aCT-guided biopsy displays renal cell carcinoma. Which of the following cellular adaptations in the bone marrow best e>..-plains the increased hematocrit in this patient? ***(A) Atrophy ***(B) Dysplasia ***(C) Hyperplasia ***(D) HyperLTophy ***(E) Metaplasia
41 The answer is C: Hyperplasia . Renal cell carcinomas often secrete erythropoietin. This hormone s timulates the growth of erythrocyte precursors in the bone manow by inhibiting programmed cell death. Increased hematocrit in this patient is the result of bone marrow hyperplasia affecting the erythroid lineage. The other choices do not represem physiologic responses to e1ythropoietin. ***Diagnosis: Renal cell carcinoma, hypellJiasia
42 A 33-year-old woman has an abnormal cervical Pap smear. A cervical biopsy reveals that the epithelium lacks nom1al pohuicy (shown in the image). Individual cells display hyperchwmatic nuclei, a larger nucleus-m-cytoplasm ratio, and disorderly tissue anangement. Whkh of the following adaptadons to chronic injury best describes these changes in the patients cervical epithelium? ***(A) Alrophy ***(B) Dysplasia ***(C) Hyperplasia ***(D) Hypertrophy ***(E) Metaplasia
42 The answer is 8: Dysplasia. The distinction between severe dysplasia and early cancer of the cervix is a common diagnostic problem for the pathologist. Both are associated wuh disordered growth and maruranon of the tissue. Sintilar ro the development of cancer, dysplasia is believed to result from mut.arions in a proliferating cell population. When a particular mmation confers a growth or survival advantage. the progeny of the affected cell will tend to predominate. In turn, their continued proliferation provides the opportunity for funher mut.ations. The accumulation of such mutations progressively distances the cell from nonnal regulawry constralnts and may lead to neoplasia. None of the other choices are associated 'Yvilh lack 0 r normal tissue polarity. ***Diagnosis: Cervical imraepithehal neoplasia, dysplasia
43 A 24-year-old woman accidentally ingests carbon tetrachlolide (CCl_.) iJ1. the laboratory and develops acute Liver failure. 'vVhich of the following cellular proteins was d irectly involved in the development of hepatotoxicity in this patient? ***(A) Acetaldehyde dehydrogenase ***(B) Alcohol dehydrogenase ***(C) Glucose-6-phosphate dehydrogenase ***(D) Mtxed function oxygenase ***(E) Superoxi de dismutase
43 The answer i.s D: Mixed function oxygenase. The membolism of CCI4 is a model system for wxicologic studies. CCl4 is first metabolized via the mixed fl.mCLion oxygenase system (P450) of the liver to a chloride ion and a hlghly reactive nichloromethyl free radical. Like the hydroxyl radical, this radical is a potent initiator of lipid peroxidation, which damages the plasma membrane and leads to cell death. The other choices are not involved in the fom1ation of the trichloromethyl free radical in liver cells. ***Diagnosis: Hepatic failure, hepatotoxicicy
44 A 30-year-old woman presents v.rith a 2-month hiswry of fatigue, mild fever, and an erylhematous scaling rash. She also notes joint pain and swelling, primarily involving the small bones of her fingers. Physical examination reveals erythema LOLlS plaques with adherem silvery scales that induce punctate. bleeding points when removed. Biopsy of lesionaJ skin reveals markedly increased thickness of the. epidermis (shown in the image). Which of the following ten11s best describes this adaptation to chronic injury Ln this patient witl1 psoriasis? ***(A) Atrophy ***(B) Dysplasia ***(C) Hyperplasia ***(D) Hypenrophy ***(E) Metaplasia
44 The answer is C: Hyperplasia. Psoriasis is a disease of the dernlis and epidermis that is characterized by persistem epidem1al hyperplasia. It is a chronic, frequemly famHial disorder d1at fearures large, etythemawus, scaly plaques, commonly on the dorsal extensor cutaneous surfaces. There is evidence to suggest tha t de regulation of epidem1al proliferation and an abnormality in the microcirculation of the dem1is are responsible for the development of psmiatic lesion s. Abnormal proliferation ofkeratinocytes is thought w be related ro defective epidennaJ cell surface receptors and aiLered intracellular signaling. The oth er choices do not describe increased numbers of othenl\rJse normal epidem1al cells. ***Diagnosis: Psoriasis, hyperplasia
45 A 2-+-year-old woman with chronk depression ingests a boule of acetaminophen tablets. Two days later, she is jaundiced (elevated serum biHrubin) and displays symptoms of encephalopathy, including impairmem in spatial perception. ln the liver, toxic metabolites of acetaminophen are generated by which of the following organelles? ***(A) Golgi apparatus ***(B) Mitochondria ***(C) Nucleus ***(D) Peroxisomes ***(E) Smooth endoplasmic reticulum
45 The answer is E: Smooth endoplasmic reticulum. Carbon reuachloride and acetaminophen are well-studied heparotoxins. Each is metabolized by cymchrome P450 of the mL'<edfunction oxidase system, located in the smooth endoplasmic reticulum. These hepatotoxins are metabolized diffe rently, and it is possible to relate the subsequent evolution of lethal cell injury w the specific features of this metabolism. Acetaminophen, an important constituent of many analgesics, is innocuous in recommended doses, but when consumed to excess it is highly toxi.c Lo the liver. The metabolism of acetaminophen to yield highly reactive quinones is accelerated by alcohol consumption , an effect mediated by an ethanol-induced increase in cytochrome P450. ***Diagnosis: Hepatotmncity, necrosis
46 A 45-year-old woman presents with a 2-momh history of fatigue and recunent fever. She also complains o[ tenderness below the right costal margin and dark urine. Physical examination reveals jaundice and mild hepatomegaly. The serum is positive for hepatitis B virus amigen. Which of the following best describes the meclumism of indirect virus-mediated hepatocyte cell death in this patient? ***(A) Accumulation of abnormal cytoplasmic proteins ***(B) Immune recognition of viral antigens on the cell surface ***(C) Generation of cytoplasmic free radicals ***(D) Impaired plasma membrane Na•tK• ATPase activity ***(E) Interference with cellular energy generalion
46 The answer is B: Immune recognition of viral antigens on the cell surface. Viral cytotoxicity is either direct or indirect (immunologically mediated). Viruses may injure cells directly by subverting cellular en?ymes and depleting the cell's nutriems, thereby dismpting the nonnal homeostatic mechanisms. Some viruses also encode proteins that induce apopwsis once daughter virions are maL ure. Vmtses may also injure cells indtrectly through activation o[ the immune system. Both humoral and cellular arms of the inmmne system prmecL against the ha.rmful e[fects of viral infections by eliminating infected cells. In brief, the presemation of viral proteins to the immune system in the comext of a self major histocompatibility complex on the cell surface immunizes the body against Lhe invader and elicits both killer cells and amhrJral amibodies. These arms or the immune system eliminate virus-infected cells by inducing apoptosis or by lysing the virally infected target cell \Vith complement. None of the other choices describe mechanisms of indirect viral cytotoxic it)~ ***Diagnosis: Hepalitis, viral
47 You are asked to presem a grand rounds seminar on the role of abnormal proteins in d isease. ln this connection , intracellular accumulation o[ an abnormally folded p rotein plays a role in the pathogenesis of which of the followi ng diseases? ***(A) AA amyloidosis ***(B) Al amyloidosis ***(C) a 1-Antitrypsin deficiency ***(D) Gaucl1er disease ***(E) Tay-Sachs disease
47 The answer is C: a.1-Antitrypsin deficiency. Several acquired and inherited diseases are characterized by imracellular accumulation o£ abnormal proteins. The deviant tertiary structure of the protein may result from an inhe1ited mmation that alters the nonnal primary amino acid sequence, or may reflect an acquired defect in protein folding. a 1 -Amiuypsi11 deficiency is a heritable disorder in whkh mmadons in the gene for o:1-antitrypsin yield an insoluble protein. The mutant protein is not easily exponed_ lt accumulates in liver cells, causing ceiL injury and cin-hosis_ Pulmonary emphysema is another complication of a.1-ml titrypsin deficiency Choices A and B are amyloidoses that represent extracellular deposiLS of fibrillar proteins arranged in ~-pleated sheet. Choices D and E are lysosomal storage diseases that represent intracellular deposits of unmetabolized sphingolipids. ***Diagnosis: n 1-Amitrypsin deficiency
48 A 38-year-old woman shows evidence of early cataracts, hair loss, atrophy of skin, osteoporosis, and accelerated atherosclerosis. This patient has most likely inherited mutations in both aUeles of a gene that encodes which of the follmving types of intracellular proteins? ***(A) Deaminase ***(B) Helicase ***(C) Oxidase ***(D) Polymerase ***(E) Topo1somerase
48 The answer is B: Helicase. \.Verner syndrome is a rare autosomal recessive disease characte1ized by early cataracts, hair loss, atrophy of the skin, osteoporosis, and accelerated atherosclerosis. Affected persons are also at risk For development of a variety of cancers. Unlike Hutchinson -Gilford progeria, patiems with Werner syndrome typically die in the fifth decade from either cancer or cardiovascular disease. Werner syndrome is caused by mutations in the WRN gene, which encodes a protein with multiple DNA-dependent enzymatic functions, including proteins v .. ith ATPase, helicase, and exonuclease activity. Hutchinson-Gilford proge1ia is caused by mutations in the human lamin A gene, which encodes an imermediate filamem protein that form a fibrous meshwork beneath the nuclear envelope. Mutations in the other choices are noL associated \"'ith \>Verner syndrome_ ***Diagnosis: Werner syndrome
49 A 28-year-old man with a history of radiation/bone man-ow transplantation for leukemia presents with severe dianbea~ He subsequently develops septic shock and expires. Microscopic e.,'(amination of the colon epithelium at autopsy reveals 11umerous acidophmc bodies and small cells with pyknotic nuclei. Which of the following proteins most likely played a key role in triggering radiation-induced cell deaLh in this patients colonk mucosa7 ***(A) Cywchrome P450 ***(B) P-Catenin ***(C) E-Cadherin ***(D) P-Select in ***(E) p53
49 The answer is E: p53. ApopLOsis detect.s and destroys cells t.hat harbor dangerous mut.at.ions, thereby maintaining genet.ic consistency and preventing the development of cancer. There are several means, t.he most. imponant of which is probably p53, by which the cell recognizes genomic abnonnalities and assesses whe1:her they can be repaired. lf the damage to DNA is so severe that it ca1m01: be repaired, the cascade of even ts leading w apoptosis is activated, and the cell dies. This process protects an organism from the consequences of a nonf- t.mctional cell or one that. cannot control its own proliferation (e.g., a cancer cell). After it binds to areas of DNA damage, p53 activat.es prot.eins that arrest the cell in G 1 of t.he cell cycle, allowing time for DNA repair to proceed. It also directs DNA repair enzymes lO the site of injury. If t.he DNA damage cannot be repaired, p53 activates mechanisms that terminat.e in apoptosis. There are several pathways by which p5.3 induce apoptosis. This molecule dowmegulates transCJiption of t.he antiapoplotic prot.ein Bcl-2, while il upregulates lransCJiption of the proapoptotic genes bax and b ak. Cytochrome P450 (choice A) is a member of the m1xed function oxidase system. P-Catenin (choke B) is a membrane protein associated with cell adhesion molecules. Seleclins (choices C and D) are celJ adhesion molecules involved in leukocvte recirculation. ' ***Diagnosis: Apoptosis
f A 22-year-old woman nursing her newborn develops a tender erythematous area around the nipple of her left b reast. A thick, yellow fl uid is observed to drain from an open fissure. Examination of this breast fluid under the light microscope will most likely reveal an abundance of which of the following inflammatory cells? ***(A) B lymphocytes ***(B) Eosinophils ***(C) Mast cells ***(D) Neurropluls ***(E) Plasma cells
1 The answer is D: Neutrophils. The thick, yellow fiuid draining from the breast fissure in this patiem represents a purulent exudate. Purulent n.'Udates and effusions are associated with pathologic conditions such as pyogenic bacte1ial infection.c:;, in which the p redominant cell type is the segmemed neutrophil (polyn10111honuclear leukocyte). Mast cells (choice C) are granulated cells that contain recepwrs for lgE on the1r celJ surface. They are additional cellular sources of vasoactive mediators, panicularly in response to allergens. B lymphocytes (choice A) and plasma cells (choke E) are mediators of chTOnic inflammation and provide antigen-specific immunily to infectious diseases. ***Diagnosis: Acme mastitis
2 Which of the following mediators of in flammation facilitates chemota..xis, cytolysis, and opsonization at the site of irillammation in the patient d escribed in Question 1? ***(A) Complement proteins ***(B) Dden.sins ***(C) Kallikrein ***(D) Kinins ***(E) Prostaglandins
2 The answer is A: Complement proteins. Complement proteins act upon one another in a cascade, generating biologically active fmgments (e.g., C5a, C3b) or complexes (e .g., C567). These products of complement activation cause local edema by increasing the pem1eabllity of blood vessels. They also promme chemotaxis of leukocytes and lyse cells (membrane auack complex) and act as opsonlns by coating bacteria. Although the other choices are mediators of inflammation, they have a more restricted set of functions. Kinins (choice D) are rom1ed foUowing tissue trauma and mediate pain transmission. None of the other choices are involved in opsoruzarion or cytolysis. ***Diagnosis: Acute mastitis
3 A 63-year-old man becomes febrile and begins expectorating large amounts of mucopurulent sputum. Sputum cultures are positive for Gran1-positive diplococci . Which of the fol1owiJ1g mediatOTs of inflammation provides potent chemotactic factors [or the directed migration or inflammatory cells into the alveo 1 ar air spaces of this patient? ***(A) Bradykinin ***(B) Histamine ***(C) Myeloperoxidase ***(D) N-formylated peprides ***(E) Plasmin
3 The answer is D: N·formylated peptides. The most potent chemotactic factors for leukocytes at the site or injury are (l) complement proteins (e.g., C5a); (2) bacte1ial and mitochondtial products, particularly low molecular we1ght N-formylated peptides; (3) products of arachidonic acid metabolism (especially LTB4); and (4) chemolcines (e.g., imerleukin-1 and interferon-)'). Plasmin (choice E) is a fibrinolytic enzyme generated by activated Hageman factor (clotting factor Xll). Histamine {choice B) is one of the primary mediators of increased vascular permeabillty. None of the other choices are chemotactic agents. ***Diagnosis: Pneumonla
4 A 59-year-old man suffers a massive hean attack and e>.:pires .....__ 24 hours later due to ventricular arrhythmia. HislOloglc examination of the affected heart muscle at autopsy would show an abundance of '"lhich of the follo\llfing mflammatory cells? ***(A) Fibroblasts ***(B) Lymphocytes ***(C) Macrophages ***(D) NeULrophils ***(E) Plasma cells
4 The answer is D: Neutrophils. During acute inllammarion, neurrophils (PMNs) adhere w the vascular endmhelium. TI1ey flatten and migmte from the vasculature, through the endothelial cell layer, and into the surrounding tissue. About 2-f hours after the onset of infarction, PMNs are observed to infi ltrate necrotic tissue at the periphery of the lnfarct. Their function is to clear debris and begin the process of wound healing. Lymphocytes (choice B) and plasma cells (choice E) are mediators of chronic in Fl ammation and provide antigenspecific immunity to in reclious diseases. Fibroblasts {choKe A) and macrophages (choice C) regulate scar tissue fom1ation at the site of infarction. ***Diagnosis: Acute myocardial infarction
5 A 5-year-old boy punctures his thumb with a rusty nail. Four hours later, the thumb appears red and swollen. Initial S\VeUing of the boy's thumb is primarily due to which of the following mechanisms? ***(A) Decreased intravascular hydrostatic pressure ***(B) Decreased intravascular oncotic pressure (() Increased capillary permeability ***(D) Increased intravascular oncotic pressure ***(E) Vasoconsniction of anerioles
5 The answer is(: Increased capillary permeability. Forces that .____ regulate the balance of vascular and tissue fluids include (l) hydrostatic pressure, (2) oncolic pressure, (3) osmotic pressure, and ( 4) lymph fiow. During inilanunation, an increase in the permeability of the endothelial cell barrier results in local edema. Vasodilation of arterioles exacerbates fluid leakage, and vasoconstriction of postcapillary venules increases the hydrostatic p ressure in the capilla1y bed (thus, not choice A), potentiating the fonnation of edema. Vasodilation of venules decreases capillary hydrostatic pressure and inhibits the movemem of fluid into the extravascular spaces. Acme inflammation is not associated with changes in plasma oncotic pressure (choices Band D). ***Diagnosis: Inflammatory edema
6 \Nhich of rhe following serum proteins activates the complement, coagulation, and fib rinolytic systems at t.he site of injury in the patient described in Question 5? ***(A) Bradykinin ***(B) Hageman factor (() Kallikrein ***(D) Plasmin ***(E) Thrombin
6 The answer is B: Hageman factor. Hageman factor (clouing factor XU) provides a key source of vasoactive mediators. Activation of this plasma protein at the site of tissue injury stimulates (l) conversion of plasminogen to plasmin, which induces 6btinotysis; (2) conversion of p rekallikrein to kallikrein, which generates vasoactive peptides of low molecular weight referred w as kinlns; (3) activation of the alternative complemem pathway; and ( 4) activation of the coagulation system. Although the other choices are mediators o[ inflammation, they have a more restricted set of func tions. ***Diagnosis: Jnflammadon
7 An 80-year-old woman presems with a 4-hour history of fever, shaking chills, and disorientation. Her blood pressure is 80/40 mm Hg. Physical examination shows diffuse purpura on her upper arms and chest Blood cultures are positive for Gramnegative organisms. Which of the following cytokines is primalily involved in the pathogenesis of direct vascular injury in this patient with septic shock? ***(A) lnterferon-y ***(B) lnterleukln-1 (() Platelet-derived growth factor ***(D) Transforming growth facto r-~ ***(E) Tumor necrosis factor-a
7 The answer is E: Tumor necrosis factor-a. (TN F-a) . Septicemia (bacteremia) denotes the clinical condition in which bacteria are found in the circulation. It can be suspeCLed clinically, but the final diagnosis is made by culmring Lhe organisms from the blood. In patients with endowxic shock, lipopolysaccharide released from Gram-negative bacteria stimulates monocytes/ macrophages to secrete large quamities of TNF-a.. This glycoprotein causes direct cytotoxic damage to capillary endothelial ceTis. The other choices do nor cause direct vascular i njmy ***Diagnosis: Septic shock
8 A 24-year-old intravenous d rug abuser develops a 2-day hisLO t)' of severe headache and feve r. His temperature is 38.JOC (103"'F). Blood cultures are positive for Gram-positive cocci. The patient is given intravenous antibiotics, but he deteliorates rapidly and dies. A cross section of the brain a[ amopsy (shown in [he image) reveals rwo encapsulated cavities. Which of the following te rms best characterizes this pathologic finding? ***(A) Cnrordc inflammation ***(B) Fibrinoid necrosis ***(C) Granulomatous inflammation ***(D) Reactive gliosis ***(E) Suppurative inflammation
8 The answer is E: Suppurative inflammation. Suppurative inflammation describes a condition in which a purulemexudate is accompanied by significant liquefactive necrosis. It is the equivalent of pus. The photograph shows two encapsulated cavi£ies in the brain. These abscesses are composed of a central cavity filled with pus, surrounded by a layer of granulation tissue. Chronic inflammation {choice A) is nonsuppurative. Fibrinoid necrosis (choice B) is observed in areas of necrotizing vasculitis. Granulomatous inflammation (choice C) is seen in patients with tuberculosis. Reactive gliosis (choke D) is a nonnal response of the brain to injury and infection but is not visible on the cut surface of the brain at autopsy ***Diagnosis: Cerebral abscess
9 A 36-year-old woman with pneumococcal pneumonia develops ....__ a right pleural effusion. The pleural fluid displays a high specific gravity and contains large numbers of polymorphonuclear (PMN) leukocytes. Which of the following best characteiizes this pleural effusion? ***(A) Fibrinous exudate ***(B) Lymphedema ***(C) Purulem exudate ***(D) Serosanguineous exudate ***(E) Transudate
The answer is C: Purulent exudate. The pleural effusion encountered in this patiem represents excess O.uid in a body cavicy. A Lransudate denotes edema fluid with low protein comem, whereas an CA'1.tdate denotes edema fluid with high protein content. A pumlent e.~:udate or efl'usion contains a prominent cellular componem (PMNs). A serous exudate or effusion is characterized by the absence of a prominem cellular response and has a yellow, strawli.ke color. Fib1inous exudme (choice A) does not contain leukocytes. Serosanguineous e>..'Udate (choice D) comains RBCs and has a red tinge. ***Diagnosis: Bactetial pneumonia, pleural effusion
10 A 33-year-old man presents with a 5-week history of calf pain and swelling and low-grade fever. Serum levels of creatine kinase are elevated. A muscle biopsy reveals numerous eosinophils. What ls the most likely etiology of this patients myalgia? ***(A) Autoimmune disease ***(B) Bacterial infection ***(C) Muscular dystrophy ***(D) Parasitic infeCLion ***(E) Viral infection
10 The answer is D: Parasitic infection. Eosinophils are particularly evident during allergic-type reactions and parasitic infestations. Infections with Tnchinclla are accompanied by eosinophilia, and skeletal muscle is typically infiltrated by eosinophils. Patients with muscular dystrophy (choice C) show elevated serum levels of creatine kinase, but eosinophils are not seen on muscle biopsy Bacterial infections (choice B) are associated with neutrophilia, and affected tissues are infiltrated with PMNs. Viral infections (choice E) are associated with lymphocytosis, and affected tissues are infiltrated \Vith B and T IymphocyLes. Polymyositis, an autoimmune disease (choice A), does not featu re eosinophils. ***Diagnosis: Trichinosis
11 A 10-year-old boy \Vith a history o[ recurrent bacterial infecrions presents with fever and a productive cough. Biochemical analysis of his nemrophils demonstrates that he has an impaired ability to generate Teactive oxygen species. This patient most likely has inherited mutations in the gene that encodes which of the following proteins? ***(A) Catalase ***(B) Cytochrome P450 ***(C) Myeloperoxidase ***(D) NADPH oxidase ***(E) Superoxide dismutase
11 The answer is D: NAPDH oxidase. The imponance of oxygendependent mechanisms in the bacterial killing by phagocytic cells is exemplifted in chronic granulomatous d isease of childhood. Children with this disease suffer from a hereditary deficiency of NADPH oxidase, resulting in a failure to produce superoxide anion and hydrogen peroxide during phagocytOsis Persons with this disorder are susceptible ro recurrent bacterial infections. Patients deficient in myeloperoxidase (choice C:) cannot produce hypochlorous acid {HOCl) and e..'.'perience ru1 increased susceptibility m infections with the fun gal pathogen Ca11dida. Catalase (choice A) converts hydrogen perox1de to water and molecular oxygen. ***Diagnosis: Chronic granulomatous disease
12 A 25-year-old woman present'> \Vith a history of recu rrent shortness of b reath and severe wheezin g. Laboratory studies demonstrate that she has a deficiency of Cl inhibitor, an esterase inllibiwr that regulates the activation or the classicaJ complemem pathway. What is the diagnosis? ***(A) Chronic granulomatous disease ***(B) Hereditary angioedema ***(C) Myeloperoxidase deficiency ***(D) Selective lgA deficiency ***(E) vViskmt-Aldrich syndrome
12 The answer is 8: Hereditary angioedema. Deficiency of Cl inhibitor, with excessive cleavage of C4 and C2 by Cls, is associated with the syndrome of hereditary angioedema. This disease is characterized by episodic, painless, nonpitting edema of soft tissues. lt is the result of chronic complement activation, with the generation of a vasoactive peptide from C2, and may be life threatening b ecause of the occurrence of laryngeal edema. Chronic granulomatous disease (choice A) is due to a hereditary deficiency of NADPH oxidase. Myeloperoxidase deficiency (choice C) increases susceptibility to infections with Ca11 dida. Selective lgA deficiency (choice D) and Wiskott· Aldrich syndrome (choice E) are congenital immunodeficiency disorders associated ·...vith defects in lymphocYle function. ***Diagnosis: Hereditary angioedema
13 A 40-year-old man complains of a 2-week history of increasing abdominal pain and yellow discoloration o[ his s-clera. Physical examination reveals right upper quadrant pain. Laboratory s tudies show elevated serum levels of alkaline phosphatase (520 U/dl ) and bilirubin (3.0 mgldl). A liver biopsy shows ponal fibrosis, with scattered foreign bodies consistent with schtstosome eggs. Which of the following inflammatory cells ts most likely to predominate in the portal tracts in the liver of this patiem? ***(A) Basophils ***(B) Eosinophils ***(C) Macrophages ***(D) Monocytes ***(E) Plasma cells
13 The an:swe-r is 8: Eosinophils. Eosinophils are recruited in parasitic infestations and would be expected LO predominate in the ponal tracts or the liver in patients with schistosomiasis. Eosinophils contain leukotrienes and platelet -activating factor, as well as acid phosphatase and eosinophil major basic protein. Plasma cells (choice E) are differentiated B lymphocytes that secrete large amounts of monospecific immunoglobulin. ***Diagnosis: Schistosomiasis, eosinoph1ls
14 A 41-year-old woman complains of excessive menstrual bleeding and pelvic pain o[ 4 momhs. $he uses an intrauterine device for contraception. Endometrial biopsy (shown in the image) reveals an excess of plasma cells {arrows) and macrophages within the stroma. The presence of these cells and scattered lymphoid follicles within the endometrial stroma is evidence of which of the follmvmg conditions? ***(A) Acute inflammation ***(B) Chronic inflammation ***(C) Granulation tissue ***(D) Granulomatous inflammation ***(E) Mensnu ation
14 The answer is B: Chronic inflammation. Inflammation has historically been referred to as either acme or chronic, depending on the persistence of the injury, clinical symptoms, and the nature of the inflammatory response. The cellular componems of chronic inflammation are lymphocytes, antibodyproducing plasma cells (see arrows on photomicrograph), and macrophages. The chron ic inflammatory response is o ften prolonged and may be associated with aberrant repair (i.e , fibrosis) Neutroph ils are featured in acute inflammation (choice A) and menstruation (choice E). Choices C and D do not exhibit the histopathology shown in the image. ***Diagnosis: Chronic endometritis
15 A 62-year-old woman undergoing chemotherapy for breast cancer presents ·with a 3-day history of fever and chest pain. Cardiac catheterization reveals a markedly reduced ejecrion fTaction with nonnal coromny blood now. A myocardial biopsy is obtained, and a PCR test for coxsackievims is positive. Hiswlogic examination of this patients myocardium will most likely reveal an abundance of which of the following inflammatory cells? ***(A) EosinophHs ***(B) Lymphocytes ***(C) Macrophages ***(D) Mast cells ***(E) Neutrophils
15 The answer is 8: Lymphocytes. This patient with viral myocarditis will show an accumulation of lymphocytes in the affected heart muscle. NaJve lymphocytes encounter antigen-presenting cells (macrophages and dendritic cells) in the secondary lymphoid organs. ln response to this cell-cell interaction, they become activated, circulate in the vascular system, and are recruited to petipheral tissues (e.g., heart). The other choices are not characteJistic responders to viral infections, although acute inflanunation may be observed in lytic infections. ***Diagnosis: Viral myocarditis
16 A 58-year-old woman with long-standmg diabetes and hypertension develops end-stage renal disease and dies in uremia. A shaggy fibtin-tich exudate is noted on the visceral pericardium at autopsy (shown in the image). 'vVhich of the following best explains the pathogenesis of this fibtinous exudate? ***(A) Antibody binding and complement activation ***(B) Chronic passive congestion ***(C) Injury and increased vascular permeability ***(D) Margination of segmented nemrophils ***(E) Thrombosis of penetrating corona1y arteries
16 The answer is (: Injury and increased vascular penneability. Binding of vasoactive mediators to specific recepwrs on endothelia] cells results in contraction and gap formation. This break in the endothelial ban ier leads w the leakage of intravascular Quid into the extravascular space. Direct injury to endothelial cells also leads w leakage of imravascular fluid. A fibrinous exudate contains large amoums of fibrin as a result of activation of the coagulation system. \ 71/hen a fibrinous exudate occurs on a serosal surface, such as the pleura or pericardium, iL is referred to as fibrinous pleuriLis or fibrinous pericarditis. Although the other choices describe aspects of inflammation, they do not address the pathogenesis o( edema formation with activation of the coagulation system. ***Diagnosis: End-s~.age kidney disease , fibrinous pericarditis
17 A 68-year-old man presents with feve r, shaking chills, and shortness of breath. Physical examination shows rates and decreased breath sounds over both lung fields. The patient exhibits grunting respirations, 30 to 35 breaths per minute, ~rith flaring of the nares. The sputum is rusty yellow and displays numerous polymorphonuclear leukocytes. Which of the following mediawrs of inflammation is chiefly responsible for the development of fever in this p<Hient? ***(A) Arachidonic acid ***(B) lmerleukin-1 ***(C) Leukmriene B 4 ***(D) Prosracyclin (PGI) ***(E) Th romboxane A!
17 The answer is B: lnterleukin-1. Release of exogenous pyrogens by bacteria, viruses, or injured cells stimulates the production of endogenous pyrogenssuch as IL-l a, IL-l~. and TNF-a. IL-l is a 15-kDa protein that stimulates prostaglandin synthesis in the hypothalamic the rmoregulatory centers, thereby altering the "thermostat" that conu·ols body temperarure. l nhlb1tors of cyclom.:ygenase (e.g, aspirin) block the fever response by inhibiting PGE2 synthesis in the hypothalamus. Chills, rigor (profound chills with shivering and piloerectton), and sweats (mallow hem dissipaUon) are sympLoms assoctated with fever. The other choices are mediawrs of inflammation, but they do not directly conuol body temperature ***Diagnosis: Bacterial pneumonia
18 Spmum culLUres obtained from the patient described in Question 17 are positive for Strepwcoccus pneu moniae. Removal of bacteria from the alveolar air spaces in this patiem involves opsonization by complemem, an imponanr su~p in mediating which of the following leukocyte functions? ***(A) Chemotaxis ***(B) Diapedesis ***(C) Haptotaxis ***(D) Margination ***(E) Phagocymsis
18 The answer is E: Phagocytosis . Many inOammatory cells are able to recognize, intemalize, and digest foreign materials, microorganisms, and cellular debris. This process is lermed phagocytosis, and Lhe effector cells are knovm as phagocytes. PhagocytosisofmostbiologicagentsisenhancedbyLheircoaLing \Vith specific plasma components (opsonlns), particularly immunoglobulins or the C3b fragment of complement. The other functions are not enhanced by opsonization. ***Diagnosis: Bacterial pneumonia
19 Which of the following mediators of in flammation is primarily responsible for secondary lnjUJy to alveolar basement membranes and lung parenchyma in the patiem described in Questions l 7 and 18? ***(A) Complemem proteins ***(B) Fibrin sp1it products ***(C) Immunoglobulins ***(D) lnterleukin-1 ***(E) Lysosomal enzymes
19 The answer is E: Lysosomal enzymes. The primary role of neutrophHs in inflammation is host defense and debridement of damaged tissue. However, when the response is extensive or unregulated, the chemical mediators of inOammation may prolong tissue damage. Thus, the same neutrophil-de!ived lysosomal enzymes that are benefidal when active imracellularly can be harmful when released w the extraceUular environment. The other choices are less likely to cause direct injury w the lung in a patient \Vith pneumonia. ***Diagnosis: Bacterial pneumonia
20 Which of the following proteins inhibits fibtinolysis, activation of the complemem system, and protease-mediated damage in the lungs of the patiem described in the previottS questions? ***(A) Acid phosphatase ***(B) Lactofenin ***(C) Lysozyme ***(D) 0: 1-Macroglobulin ***(E) Myeloperoxidase
20 The answer is D: ~-Macrog lobulin . Proteolytic enzymes that are released by phagocytic ceUs during inflammation arc regulated by a family of protease inhibitors, including a 1-antitrypsin and a 2-macroglobulin. These plasma-derived p roteins inhibit plasmin-activated fibrinolysis and activation of the complement system and help protect against nonspcci fie tissue injmy during acute inflammation. Lysozyme (choice C) is a glycosidase that degrades the peptidoglycans of Grampositive bacterial cell walls. Myeloperoxidase (choice E) is contained within nemrophil granules. ***Diagnosis: Bacterial pneumonia
21 A 35-year-old woman presents with a 5-day history o [ a painful sore on her back Physical examinauon reveals a 1-cm abscess over her left shoulder. Biopsy of the lesion shows vasodilation and leukocyte margination (shown ln the image). What glycoprote]n mediates initial tethering of segmented neulrophHs lo endothelLal cells in this skin lesion? ***(A) Cadherin ***(B) Ent.actin ***(C) I me grin ***(D) Laminin ***(E) Select in
21 The answer is E: Selectin. Setectins are sugar-binding glycoprmeins that mediate the initial adhesion of leukocytes to endothe!Jal cells at snes of mllammarion. E-se lectins are found on endothelial cells, P-selectins are found on platelets, and L-selectins are found on leukocytes. E-seleClins are stored in Weibel-Palade bodies of resting endothelial cells_ Upon aCLivation, E-selectins are redisuibuted along the luminal surface of the endothelial cells, where they mediate the initial adhesion (tetheling) and rolling of leukocytes. After leukocyte..'> have come to a rest, imegJ.ins (choice C) mediate transendothelial cell migration and chemotaxis. Cadhetins (choice A) medhne cell-cell adhesion, but they are not involved in neutrophil adhesion LO vascular endmhelium. Emactin (choice B) and laminin (choice D) are basement membmne proteins. ***Diagnosis: Carbuncle
22 A 14-year-old boy receives a laceration on his forehead during an ice hockey game. vVhen he is first attended to by the medic, th ere is blanching of the skin around the wound. Which ofLhe following mech anisms accounts for this transient reaction to neurogenic and chemical stimuli at the site of injury? ***(A) Constriction of postcapillary venules ***(B) Constriction of precapUlary arterioles ***(C) Dilation of postcapillary venules ***(D) Dilation of precapillaty anetioles ***(E) Ischemic necrosis
22 The answer is 8: Constriction of pre(apillary arterioles. The initial response of anetioles to neurogenic and chemical stimuli is transient vasoconstriction. However, shonly thereafter, vasodilation (choice D) occurs, with an increase in blood flow to the inflamed area. nus process is referred to as active hyperemia_ None of the other choices cause transient skin blanching. ***Diagnosis: Laceration
23 An 8-year-old girl with asthma presents with respiratmy disn·ess. $he has a history of allergies and upper respiratory tract infections. She also has history of wheezes associated with exercise. Which of the following mediators of inflammation is rhe most powerful stimulator of bronchoconstriction and vasoconsuiction in this patient? ***(A) Bradykinin ***(B) Complement proteins ***(C) Interleukin-1 ***(D) Leukorrienes ***(E) Tumor necrosis factor-a.
23 The answer is D: Leukotrienes. Asthma is a chronic lung disease caused by increased responsiveness of the airways to a vatiety of stimuli. Chemical mediators released by chronic lnflam.matory cells in the lungs of these patients stimulate bronchial mucus production and bronchoconsttiction. Among these mediators are leukoo-ienes, also 1<nown as slow-reacting substances of anaphylaxis. They are derived from arachidonic acid through the lipoxygenase pathway Leukorrienes stimulate contraction of smooth muscle and enhance vascular permeability They are responsible for the development of many of the clinical symptoms associated with asthma and other allergic reactions. Although the other choices are imponant mediators of inflammation, they do not play a leading role in the development of bronchoconstriction ln patients with bronchial asthma. ***Diagnosis: Asthma
24 Which of the following preformed substances is released from mast cells and platelets, resulting in increased vascular permeability in the lungs of the pat1cnt described in Question 23? ***(A) Bradykinin ***(B) Hageman factor ***(C) Histamine ***(D) Leukmrienes (SRS-A) ***(E) Th romboxane A2
24 The answer is(: Histamine. When lgE-sensitized mast cells are stimulated by antigen, prefom1ed mediators of inflammation are secreted into the extracelJular tissues. Histamine binds to specific H1 receptors in the vascular wall, inducing enclothellal cell contraction, gap formation, and edema. Massive release of histamine may cause circulatory collapse (anaphylactic shock). Bradykinin (choice A) and Hageman facto r (choice B) are plasma-dertved mediators. The other choices are not preformed molecules bm are symhesized de novo following edt activacion. ***Diagnosis: Asthma
25 A 75-year-old woman complains of recent onset o[ chest pain, fever, and productive cough with rust-colored spumm. A chest X-ray reveals an infiltrate in the right middle lobe. Sputum cultures are positive for Streptococcus pneumoniae. Phagocytic cells m this patient's affected lung tis..c;ue generate bacteriocidal hypochlorous acid using which of the follml\ing enzymes? ***(A) Catalase ***(B) C)'clooxygenase ***(C) Mydoperoxi dase ***(D) NADPH oxidase ***(E) Superoxide dismmase
ZS The answer is C: Myeloperoxidase. Myeloperoxidase camlyzes the conversion of Hp2, in che presence of a halide (e.g., chlmide ion), to form hypochlorous acid. This powerful oxidant is a major bactericidal agent produced by phagocytic cells. Patients deficient in myeloperoxidase cannot produce hypochlorous acid and have an increased susceptibility to recurrent infections. Catalase (choice A) cacabolizes H20r Cyclooxygenase (choice B) mediates the conversion of arachidonic acid to prostaglandins. NADPH oxidase (choice D) is involved in oxygen-fl-ee radical formation during the neurrophil respiratory burst. Superoll.ide dismutase (choice E) reduces the superO}I.'lde radical to H20 r ***Diagnosis: Bacterial pneumonia
26 A 28-year-old woman cuts her hand while dicing vegetables i11 the kitchen. The wound is cleaned and sutured. Five days later, the site of injury contains an abundance or chronic inOammawry cells that actively secrete ilu erleukin-1 , tumor necrosis facwr-a , interferon-a., numerous arachidonic add derivatives , and va tious enzymes. Name these cells. ***(A) B lympboc)'tes ***(B) Macrophages ***(C) Plasma cells ***(D) Smoolh muscle cells ***(E) T lymphoc)'tes
26 The answer is 8: Macrophages. The macrophage is the pivotaJ cell in regulating chronic Ln.flammation. Macrophages, which are derived from cLrculating monocytes, regulate lymphocyte responses to antigens and secrete a variety of media~ors that modulate the proliferation and function of fibroblasts and endothelial cells. None of the other ceUs have this wide spectnnn of regulatory functions . ***Diagnosis: Laceration , wound healing
27 A 68-year-old man with prostate cancer and bone metastases presents with shaking chills and fever. The peripheral \VBC coum is 1,000/J.!L (normal = 4,000 w 11,000/!ll-). Which of the following terms best desctibes this hematologic finding? ***(A) Leukocytosis ***(B) Leukopenia ***(C) Neutrophilia ***(D) Pancytopenia ***(E) Leukemoid reaction
27 The answer is B: Leukopenia. Leukopenia is defined as an absolute decrease in the circulating WBC count. lt Ls occasionally encountered under conditions of chronic inOammation, especially in patients who are malnourLshed o r who su ffer from a chronic debilitating disease. Leukopenia may also be caused by typhoid fever and certain viral and ticketlSial infections. LeukocytosLs (choice A) is defined as an absolute increase in the circulating WBC count. Neutrophilia (choice C) is defined as an absolute increase in the circulating neutrophil coum. Pancytopenia (choice D) refers ro decreased circulating levels of all formed elements in the blood. ***Diagnosis: Prostate cancer
28 A 25-year-olcl machinist i.5 injured by a metal sliver in his left hand. Over the next few days, the wounded area becomes reddened, tender, swoUen, and fee ls wann to the touch. Redness at the site of injury in this patient is caused primatily by which of the following mechanisms? ***(A) Hemon hage ***(B) Hemostasis ***(C) Nemrophilmargination ***(D) Vasoconstriction ***(E) Vasodilation
28 The answer is E: Vasodilation. Vasodilation of precapillary anerioles increases blood flow at the site of tissue injury. This condition (active hyperemia) is caused by the release of specific mediators. Vasodilalion and hyperemia are ptimaril y responsible for the redness and warmth (rubor and calor) at sites of injury. The other choices do nor regulate active hyperemia. ***Diagnosis: Acute in n.ammation
29 The patiem described in Question 28 goes w the emergency room to have the sliver removed. Which of the following mediators of inflammation plays the most imponam role 111 stimulating platelet aggregation at the site of injury fo ll ovving this minor surgical procedure? ***(A) Leukotriene C4 ***(B) Leukorriene D + ***(C) Prostaglandin E, ***(D) Prostaglandin 12 ***(E) TI1romboxane A2
29 The answer is E: Thromboxane A2 . Platelet adheren ce , aggregation, and degranulation occur wh en platelets come in contact with fibrillar collagen or thrombin {after activation of the coagulation system). Platelet degranulation is associated with the release of serotonj n, which directly increases vascular permeability. ln addition, the arachidonic acid metabo11te thromboxane A2 plays a key role in the second wave of platelet aggregation and med iates smooth muscle constriction. Prostaglandins E2 and 12 (choices C and D) inhibit inflammatory cell fu nctions. Leukotrienes C~ and D~ (choices A and B) induce smoorh muscle contraction. ***Diagnosis: Acute in nammation
30 Twenty-fou r hours later, endothelial cells at the site of i.njuty in the patient described in Questions 28 and 29 release a chemical mediator that inhtbits furtl1er platelet aggregation. Name this mediator of inflammation. ***(A) Plasmin ***(B) Prostaglandin (PGI2 ) ***(C) Serownin ***(D) Thrombin ***(E) Thromboxane A2
30 The answer is B: Prostaglandin (PGI2 ) . PG12 is a derivative of arachidon ic acid mat is formed in the cyclooxygenase enzyme pathway. It promotes vasodiJation and bronchodilation and also in hibits platelet aggregation. It activates adenylyl cyclase and increases intracellular levels of cAMP. Its action is diametrically opposite to that of thromboxane A2 (ch oice E), wh ich activates guanylyl cyclase and increases intracellular levels of cGMP. Plasmin {choice A) degrades fibrin. Serotonin (choice C) is a vasoactive amine. Thrombin (choice D) is a protease that mediates the con version of fibrinogen ~o fibrin. ***Diagnosis: Acute inOammation
31 A 37 -year-old man >vixh AIDS is admitted w the hospilal with a 3-week hiswry of chest pain and shortness of breath. An X-ray film of the chest shows bilateral nodularities of the lungs. A CT-guided lung biopsy is shown in the image. The multinucleated cell in the center of this field is most likely derived from which of the foUowing inflammalOry cells? ***(A) Basophils ***(B) Capillary endothelial cells ***(C) Macrophages ***(D) M}rofibroblasts ***(E) Smooth muscle cells
31 The answer is C: Macrophages. Granulomas are collections of epithelloid cells and multinucleated giam cells that are fanned by cytoplasmic fusion of macrophages. ·when the nuclei are arranged around Lhe periphery of the cell in a horseshoe pauem (see photomicrograph), the ce ll is termed a Langhans giant cell. Frequently, a foreign pathogenic agent is identified \vjthin the cytoplasm of a multinucleated giant cell, in which case the label foreign body giam cell is used. The other cells do not fonn multinucleated giam cells in granulomas ***Diagnosis: AlDS, granulomatous inflammation
32 A 45-year-old woman with autoimmune hemolytic anemia presents \\'ith increasing fatigue. Which of the following mediators of inflammation is primarily responsible for antibody-mediated hemolysis in this patien t? ***(A) Arachidonic acid metabolites ***(B) Coagulation proteins ***(C) Complement proteins ***(D) Kallikrein and kinins ***(E) Lysophospholipids
32 The answer is C: Complement proteins. Activation of the complement cascade by me classical or alternative pathway leads to the cleavage of complement fragments and the formation of biologically active complexes. The C5b fragmem aggregates with complement proteins C6, C7, C8, and C9, resulting in the polymerization of the membrane attack complex (MAC). 1v1AC lyses cells by inserting into the lipid bilayer, fanning a pore, and destroying me pem1eability barrier of the plasma membrane. Kallikrein and kinins (choice D) are formed following tissue trauma and mediate pain transmission. None of the other choices mediate hemolysis . ***Diagnosis: Hemolytic anemia, amoinmmne disease
33 A 59-year-old alcoholic man is brougl1t to the emergency room with a fever (38.rCil03°F) and foul-smelling breath. A chest X-ray reveals a pulmonary abscess in the right lower lobe. The patient subsequently develops acute bronchopneumonia and dies. Microscopic examination of tl1e lungs at autopsy is shown in the image. Activation of phospholipase ~ in these intra-alveolar cells resulted in the fom1ation of which of the follo\ving mediators of inflammation? ***(A) Arachidonic acid ***(B) cAMP ***(C) cGMP ***(D) Diacylglycerol ***(E) Inositol trisphosphate
33 The answer is A: Arachidonic acid. Cellular sources of vasoactive mediawrs are (1) derived fTom the metabolism of arachidonic acid (prostaglandins, thromboxanes, leukotrienes, and platelet-activating facto r), (2) prefom1ed and srored in cyroplasmic granules (histamine, serotonin, and lysosomal hydrolases), or (3) generated as normal regulators of vascular f1.mction (nitric oxide and neurokinins). The photomicrograph shows polymorphonuclear leukocytes responding to a bacterial pneumonia. Free arachidonic acid in these acute inflammatory cells is derived from membrane phospholipids (prin1arity phosphalidylcholine) by stimulus-induced activation of phospholipase A1 . Phospholipase A2 activation does not generate the other inflammatory medialOrs listed. ***Diagnosis: BaC(erial pneumoma
34 A lO~year-old girl presents with a 2-week hismry of puffiness around her eyes and swelling of the legs and ankles. Laboratory s tudies show hypoalbuminemia and proteinuria The u rinary sedimem contains no inflammatory cells or red blood cells. Which of the following tenus describes this pad em's peripheral edema? ***(A) Effusion ***(B) Exudate ***(C) Hydropic change ***(D) Lymphedema ***(E) Transudate
34 The answer is E: Transudate. According to the Starling principle, the interchange of Ouid between vascular and extravascular compartments resullc; from a balance of forces that draw fluid into the vascular space or out into tissues. These forces include (l) hydrostatic pressure, (2) oncotic pressure (reOects plasma protein concentration), (3) osmotic pressu re, and (4) lymph now \..Vhen the balance of these forces is altered, the net result is rluid accumulation in the interstitial spaces (i.e, edema). Although edema accompanies acute inOammation, a variety of noninflammatory conditions also lead to the fom1arion of edema. for example, obsuuction of venous outflow or decreased light ventricular function results in a back pressure in the vasculature, thereby increasing hydrostatic pressure. l oss of albumin (kidney disorders, this case) or decreased synthesis of plasma proteins (liver disease, malnutrition) reduces plasma oncOlic pressure. Noninfla1mnatmy edema is refe1Ted w as a transudate. A transudate is edema fluid with a low protein comem. An exudate (choice B) is edema fluid \Vilh a high protein and lipid concentration that frequently contains inflammatmy cells. An effusion (choice A) represems excess Ouid in a body cavity such as the peritoneum or pleura. Lymphedema (choice D) is u.c;ually associated with obstruction of lymphatic flow (e.g., surgery or infection). ***Diagnosis: Nephrotic syndrome, noninflammatory edema
35 A 25-year-old woman develops a sore, red, hot, swollen left knee. She has no history of trauma and no famihal history of joint disease . Fluid aspirated from the joint space shows an abundance of segmented neutrophils. Transendothelial migration of acute inOammatmy cells imo this patiem:S joint space was mediated ptimarily by which of the following families of prO[eins? ***(A) Entactins ***(B) Fibrillins ***(C) Fibronectins ***(D) I me grins ***(E) Laminins
35 The answer is D: 'lntegrins. Chemokines and other proinflammatory molecules activate a family of cell adhesion molecules, namely the integrins. Molecules in this fami ly participate in cell-cell and cell-substrate adbesions and eel] signaling. lntegrins are involved in leukocyte recruitmem to sites of injury in acute inflammation. The other choices are extracellular matrix molecules that maintain LL-;sue architecture and facilitate wound healing. ***Diagnosis: Gonococcal anhritis
36 Aspirin is effem ve in relieving symptoms of acute inflammation in Lhe patient described in Question 35 because it inrubits wruch of the following enzymes? ***(A) Cyclooxygenase ***(B) MyeloperoxMase ***(C) Phospholip ase~ ***(D) Protein kinase C ***(E) Superoxide dismmase
36 The an.swer is A: Cyclooxygenase. Arachidonic acid is metabolized by cycloOA'Ygenases (COX-1 , COX-2) and lipoxygenases(5-LOX)mgenerateprostanoidsandleukoLrienes, respectively. The early inflammatmy prostanoid response is COX-1 dependent. COX-2 becomes d1e major source of prostanoicls as inflammation progresses. Inhibition of COX is one mechanism by which nonsteroidalami-mflammatorydrugs (NSAIDs), includlng aspirin, indomethacin, and ibuprofen. exen Lheir potent analgesic and anti-inflammatory effects. NSAlDs block COX-2- induced fom1ation of prostaglandins, thereby mitigating pain and inflammation. Myeloperoxidase (choice B) catalyzes the conversion of H10 2 , in the presence of a halide (e.g. , chloride ion) to form hypochlorous acid. This poweJ{ul oxidant is a major bactericidal agent produced by phagocytic cells. Superoxide dismutase (choice E) reduces the superoxide radical to H 20 2 . ***Diagnosis: Gonococcal anhntis
37 A 50-year-old woman is ruscovered w have metastatic breast cancer. One week after receiving her first dose of chemotherapy, she develops bactetial pneumonia. Which of the fol lowing best explains this patiems susceptibility to bactetial infection? ***(A) Depletion of serum complemem ***(B) lmpai red neutrophil respiratory burst ***(C) Inhibition of clQ[ting factor activation ***(D) Lymphocytosis ***(E) Neutropen ia
37 The answer is E: Neutropenia. The importance of protection afforded by acme inflammatmy cells is ernphas1zed by the frequency and severity of infections in persons with defective phagocytic cells. The most cmmnon defect is iatrogenic nemropema secondaq to cancer chemotherapy Chemotherapy would not be expected Lo deplete semm levels of complement (choice A) or alter the respiratory burst withil1 activated neutrophils (choice B). ***Diagnosis: Bactenal pneumonia
38 A 53-year-old man develops weakness, malaise, cough with bloody sputum, and night sweats. A chest X-ray reveals numerous apical densities bilaterally. Exposure to Myco bacterium t~.tberculos is was documemed 20 years ago, and M. tuberculosis is identified in the sputum. The patient subsequently dies of respiratory insufficiency. The lungs are e.xammed at autopsy (shown in the image). Wl1ich of the following best characterizes th e histOpathologic features of this pulmona1y lesion7 ***(A) Acute suppurative inflammatlon ***(B) Chronic inflammation ***(C) Fat necrosis ***(D) Fibrinoid necrosis ***(E) Granulomatous inflammation
38 TheanswerisE: Granulomatous inflammation. The photograph shows a necrotizing granuloma due to M. LUberwlosis. The necrotic center is surrounded by histiocytes, giant cells, and fibrous tL<;sue. Granulomatous inflammation is elicited by fungal infections, tuberculosis, leprosy, schistosomiasis, and the presence of foreign material. ll is characteristically associated with caseous necrosis produced by M. i1!berculosis. The other choices ffia}' be seen as secondary features in granulomatous inflammation. ***Diagnosis: Pulmonary tuberculosis
39 A 59-year-old man experiences acute chest pain and is rushed to the emergency room. Laborawry scudies and ECG demonsmue an acute myocardial infarction; however, coronary anety angiography perfom1ed 2. hours later does not sho>v evidence of thrombosis. Intravascular thrombolysis that occurred in this padem was mediated by plasminogen activators that were released by which of the following cells? ***(A) Cardiac myocyte..<; ***(B) Endmhelial ce1Js ***(C) Macrophages ***(D) Segmemed neutrophils ***(E) Vascular smooth muscle cells
39 The answer is B: Endothelial cells . The vascular endothelium has the ability to promote or inhibit tissue perfusion and inflammatory ceU infltLx through multiple med1anisms. For example, endothelial cells in the vicinity of the thrombus produce tissue-type plasminogen activators, which activate plasmin and initiate thrombolysis (fibtinolysis). None of the other ceUs produce significant quantities of plasminogen activators. ***Diagnosis: Myocardial infarction, hemostasis
40 Which of the following mediators of inOammation causes relaxation of vascular smooth muscle ceUs and vasodilation of arterioles at the site of myocardial infarction in the patient described in Question 39? ***(A) Bradykinin ***(B) Histamine ***(C) Leukmrienes ***(D) Ninic oxide ***(E) Thromboxane A,_
40 The answer is D: Nitric oxide. Nitric oxide (NO), which was previously knm.vn as endothelium-de rived relaxing factor, leads to relaxation of vascular smooth muscle cells and vasodilation of arterioles. NO also inhibits plateleL aggregation and mediates the kUling of bactetia and tumor cells by macrophages. Histami.ne (choice B), leukotrienes (choice C), and thromboxane A2 (choke E) stimulate Lhe contraction of smooth muscle cells. ***Diagnosis: Acute myocardial infarction
41 A 68-year-old coal miner \Vith a history of smoking and emphysema develops severe air-Gow obstruction and expires. Autopsy reveals a "black lung,n \Vilh coal-dust nodules scattered throughout the parenchyma and a central area of dense fibrosis. The coal dust entrapped \\lithin this miners lung 'vas sequestered primarily by which of the folio•ving cells7 ***(A) Endothelial ceUs ***(B) Fibroblasts ***(C) Lymphocytes ***(D) Macrophages ***(E) Plasma cells
41 TheanswerisD: Macrophages. Coal workers'pneumoconiosis reflects the inhalation of carbon particles. The character istic pulmonary lesions of simple coal workers pneumoconiosis include non palpable coal-dusl macules and palpable coaldust nodules, bO£h of which are typically multi ple and scatt.ered throughout the lung as 1- lO 4-mm black foci. Nodules consisL of dust-laden macrophages associated with a fibrotic stroma. NoduJes occur when coal is admixed with fibrogenic dusLs such as silica and are more properly classified as anthracosilicosis. Coal-dust macules and nodules appear on a chest radiograph as small nodular densities. The other choices are not phagocytic cells. ***Diagnosis: Anthracosillcosis, coal workers' pneumoconiosis
42 A 40-year-old man presents with 5 clays of productive cough and fever. Psettdomona.s aeruginosa is isolated from a pulmonmy abscess. The CBC shows an acute effect characterized by marked letlkocytosis (50,000 \NBCJ~LL) , and the differential count reveals numerous immature cells (band forms). Which of the follm.ving terms best describes these hematologic findings? ***(A) Leukemoid reaction ***(B) Leukopenia ***(C) Myeloid metaplasia ***(D) Myeloproliferadve disease ***(E) Neutrophilia
42 The answer is A: Leukemoid reaction. Circu laring levels of leukocytes and Lheir precursors may occasionally reach ve1y high levels (>50,000 WBOp L). Such a situation, referred w as a leukemoid reaction , is somedmes difficuh to difreremiate from leukem]a. 111 contrast to bacterial infecdons, viral infections (including infectious mononucleosis) are characterized by lymphocytosis, an absolute increase in the number of circulating lymphocytes. Parasitic infestations and certain allergic reactions cause eosinophilia, an increase in the number of circulating eosinoph ils. Leukopenia is defined as an absolu te decrease in the circulating WBC count. Myloid metaplasia (choice C) and myeloproliferative disease (choice D) are chronic disorders of the hemaLOpoietic system. Although technically correct, neutrophiUa (choice E) by itself does not demonstrate immaLure cells (band forms) and usually refers to lower levels of increased nemrophi.ls. ***Diagnosis: Pulmonary abscess
43 A 19-year-old woman presents with 5 days of fever (38°0101 "F) and sore throat. She repons that she has felt fatigued for the past week and has difficulty swallowing A physical examination reveals generalized lymphadenopathy If this patiem has a viral infection. a CBC will most likely show which of the following hemawlogic findings? ***(A) Eosinophilia ***(B) Leukopenia ***(C) Lymphocytosis ***(D) Neutrophilia ***(E) Tl1 rombocythemia
43 The answer is C: Lymphocytosis. Peripheral blood lymphocytosis is defined as an increase in the absolute peripheral blood lymphocyte count above the normal range (<4,000/j..LL in children and 9,000/j..LL in infants) The principal causes of absolute peripheral blood lymphocytosis are (l) acute vi ral infections (infectious mononucleosis, whooping cough, and acute infection lymphocytosis), (2) chronic bacterial infections (tuberculosis, brucellosis), and (3) lymphoproltferative diseases. The mher choices are not features of acute viral infections. ***Diagnosis: Infectious mononucleosis
44 A 40-year-old woman presents with an 8-month history of progressive gene ralized itching, weight loss, fa tigue, and yellow sclerae. Physical examination reveals mUd jaundice The anti mitochondrial antibody test is positive. A liver biopsy discloses p ericluctal inflammation and b lle duc t injury (shown in the image). Which of the follO\ving inflammato ry ce lls is the principal mediator of destntClive cholangitis in thls pariem? ***(A) Eosinophils ***(B) B lymphocytes ***(C) T lymphocytes ***(D) Mast cells ***(E) Neurrophils
44 The answer is C: T lymphocytes. Primary biliary cirrhosis (PBC) is a chronic progressive cholestatic liver disease characterized by destruction of intrahepatic bile ducts {nonsuppurative desmtctive cholangitis). PBC occurs principally in middle-aged women and is an amoimmune disease. Most paLiems with PBC have at least one other disease usually classed as autoimmune (e.g., thyroiditis, rheumawid arthritis, sclerodetma, Sjogren syndrome, or systemic lupus erythematosus). More than 95% of patients with PBC have circulating amimitochondrtaJ antibodies. The cells sunounding and infiltrating the sites ofbile duct damage are predominantly suppressor/cytotoxic (CDS•) T lymphocytes, suggesting that they mediate the destruction of the ductal epithellum. Macrophages and B lymphocytes (choice B) are associated with periductal inflammauon but do not mediate epithelial cytotoxicity. Eosinoph ils (choice A) have no role in primaty immune-related mechanisms. The other inflammatory cells (choices D and E) do not participate in the pathogenesis o[ PBC. ***Diagnosis: Primary biliary cirrhosis, chronic inflammation
45 A 25-year-old woman presents with a 2-week history of febri le illness and chest pain. She has an erythemarous, macular facial rash and tender joints, particularly in her left wrist and elbow A CBC shows mild anemia and thrombocytopenia. Conicosteroids are prescribed for the paliem. This medication induces the synthesis of an inhibitor of which of the following enzymes in inOammatory cells? ***(A) Upo>-'Ygenase ***(B) Myeloperoxidase ***(C) Phospholipase~ ***(D) Phospholipase C ***(E) Superoxide dismmase
45 The answer is C: Phospholipase A1. Corticosteroids are widely used to suppress the tissue destrunion associated with many chronic inOammatory diseases, including rheumatoid arthritis and systemic lupus erythematosus. Corticosteroids induce the synthesis of an inhibitor of phospholipase A2 and block Lhe release of arach1donk acid from the plasma membranes of inflammatory cells. Although corticosteroids are \~o,ridely used to suppress inflammatory responses, the prolonged administration of these compounds can have deleteriouseffens, including atrophy of the adrenal glands. Myeloperoxidase (choice B) catalyzes the con version or H20r in the presence of a halide {e.g., chloride ion) to form hypochlorous acid. This powerful oxidam is a major bactericidal agent produced by phagocytic ceUs. Superoxide dismUlase (choice E) reduces the superoxide radical to Hl02 . ***Diagnosis: Systemic lupus erythemawsus
46 The pad em described in Question 45 is noted to have increased serum levels of ceruloplasmin, fibrinogen, a.1-rnacroglobulin, serum amyloid A protein, and C-reactive protein. Together, these markers belong to which of the following families of proteins? ***(A) Acute phase proteins ***(B) Anaphylato>.ins ***(C) Inhibitors of platelet activation ***(D) Protease inhibito rs ***(E) Regulawrs of coagulation
46 The answer is A: Acute phase proteins. These proteins are synthesized primarily by the liver and are released imo the circuladon in response to an acme inflammawry challenge. Changes in the plasma levels of acute phase protelns are mediated p1imatily by cytokines (IL-l, ll-6 , and TNF-a). Increased plasma levels of some acute phase prmeins are reflected in an accelerated erythrocyte sedimentation rate , which is an index used clinically to moniwr the activity of many inflammatory diseases. None of the other choices describe the set of serum markers listed in tills question. ***Diagnosis: Systemic lupus erythematosus
1 A 74-year-old woman presents with acute chest pain and shortness of breath. Cardiac catheterization demonstrates occlusion of the left anterior descending coronary anery. Laboratory studies and ECG are consistent with acute myocardial infarCLion. Which of the following is the most likely pathologic finding in the affected heart muscle 4 weeks later7 ***(A) Capillary-rich granulation tissue ***(B) Collagen-rich scar tissue ***(C) Granulomawus inflammation ***(D) Neurrophils and necrotic debris ***(E) Vascular congestion and edema
1 The answer is B: Collagen-rich scar tissue. Pathologic findings in congestive hean failure include microscopic signs of coagulative necrosis approximately 24 hours after the on..set of vascular occlusion. Polymorphonuclear leukocytes and macrophages predominate during the next 2 w 5 days (choice D). Toward the end of the first week, the infarct is invaded by capillaryJicll granulation tissue (cl1oice A). Ultimately, the necrotic myocardium is replaced by collagen-rich scar tissue (weeks to months). Granulomatous inOammation (choice C) does not occur after an ischemic myocardial infarct. Vascular congestion and edema (choice E) are features of acme inflammation. ***Diagnosis: Myocardial infarction
2A 4-year-old boy falls on a rusty nail and punctures his skin. The wound is cleaned and covered with sterile gauze. Which of the following is the initial evenr in the healing process? ***(A) Accumulation of acute inflammatory cells ***(B) Deposition of proteoglycans and collagen ***(C) Differentiation and migration of myofibroblasts ***(D) Formation of a fibrin clot ***(E) Macrophage-mediated phagocytosis of cellular debris
2 The answer is D: Formation of a fibrin clot. The initial phase of the repair reaction , which typically begins wi th hemon-hage, involves the fomlati on 0 r a fibrin clot that fills the gap created by the wound. A thrombus (clot), referred to as a scab after dqing om, forms on the wounded skin as a barrier to invading microorganisms. lt also prevents the loss of plasma and tissue fluid . Formed p1imarily from plasma fibrin, the thrombus is rich in fibronectin. The thrombus also contains contracting platelets, whkh are an initial source of growth facwrs. Much later, the thrombus undergoes proteolysis, after which it is penetrated by regenerating epithelium. The scab then detaches. Accumulation of acute i11 ilammawry cells (ch oice A) might occur after fo rmation of the initial fibrin clot. Collagen fonnation (choice B) and macrophage activity (choice E) occur much later. Myofrbroblasrs (choice C) begin to accumulate in the wound around the 3rd day. ***Diagnosis: Wound healing
3 An 82-year-old man dies 4 years after developing congestive heart failure. He had a history of multiple myocardial infarcts over the past 10 years. A trichrome stain of heart muscle at autopsy is shown in the image. What is the predominant type of collagen found in this mature scar tissue? ***(A) l)rpe 1 ***(B) Type n ***(C) Type TV ***(D) Type V ***(E) Type Vl
3 The answer is A: Type I collagen. A mature scar is composed primarily of Lype 1 collagen. By comrast, the early matrix of granulation tissue contains prmeoglycans, glycoproteins, and type lll collagen. Eventually, the temporary matrix is removed by a combination o[ extracellular and intracellular digestion, and the definitive matrix is deposited. Extracellular crossJill king of the newly synthesized type 1 collagen progressively increases wound strength. Collagen type II (choice B) is found in cartilage. Collagen type IV (choice C) is found in basement membranes. Collagen types V and Vl (choices D and E) are found in various organs. ***Diagnosis: Myocardial infarction
4 A 25-year-old woman sustains a deep, open laceration over her right foreann in a motorcycle accident. The wound is cleaned and sutured . Which of the following cell types mediates contraction of the wound to facilitate heallng? ***(A) Endothelial cells ***(B) Fibroblasts ***(C) Macrophages ***(D) Myofibroblasts ***(E) Smooth muscle cells
4 The answer is D: Myofibroblasts. The myofibroblast is the cell responsible for wound contraction as well as the defomling pathologic process tenned wound contracture. These cells ex1)ress a.-smooth muscle actin, desmin, and vimentin, and they respond to pharmacologic agents thal cause smooth muscle to contract or relax. Myofibroblasts exert their contractile effects by forming syncytia, in whkh the myofibroblasts are bound together by tight junctions. By contrast, fibroblasts (choice B) tend to be solitary cells, surrounded by collagen fibers. Endothelial cells (choice A) respond to growth factors and form capillaries, which are necessary for the delivery of nutrienrs and infiammatOJy cells. Neither macmphages (choice C) nor smooth muscle cells (choice E) mediate wound contraction. ***Diagnosis: Wound contraction
5 Duling the next 3 months, the wound heats with fonnation ,___ of a linear scar. Which of the following nutritional factors is required for proper collagen assembly in the scar tissue of the patient described in Question 47 ***(A) Folic acid ***(B) Thiamine ***(C) Vitamin A ***(D) Vitamin C ***(E) Vitamin E
5 The answer is D: Vitamin C. Vitamin C (ascorbic acid) is a "-- powerful, biologic reducing agent that is necessary for the hydroxytaoon of protine residues in coUagen. Most of the clinical features associated with vitamin C deficiency (scurvy) are caused by the formal:ion of an abnormal collagen that lacks tensile strength Patients with vitamin C deficiency exhibit poor >vound healing. Dehiscence (bursl:ing open) of previously healed wounds may also occur. None of the other choices are required for collagen assembly. ***Diagnosis: Wound healing
6 A 70-year-old woman with diabetes develops an ulcer on her ,___ right leg (shown in the image). The ulcer bed is covered with granuLation tissue. Whkh of the fo11owing are 1:he p1inciple cellular compon ents found in the bed of this wound? ***(A) Fibroblasrs and endothelial cells ***(B) Myofibroblasts and eosinophils ***(C) NeU£rophils and lymphocytes ***(D) Plasma cells and macrophages ***(E) Smooth muscle cells and Merkel cells
6 The answer is A: Fibroblasts and endothelial (ells. Granulation tissue has two major components: cells and proliferating capillaries. The cells are mostly fibroblasts, myofibroblasts, and macrophages. fibroblasts and myofibroblasts derive from mesenchymal stem cells. Capillaries arise fmm adjacent blood vessels by division of endothelial cells ln a process termed angiogenesis. Macrophages are a principal source of growth factors and ~ue recognized for their phagocytic functions. Granulal:ion tissue is fluid laden, and its cellular consl:ituems supply antibacterial antibodies and gro>vth factors. Once repaiT has been achieved, most of the newly formed capillaries are obliterated and then reabsorbed, leaving a pale avascular scar. Although the other inOammatOJ)' cells listed may be found in this healing wound, they do not constitute the principal components of granulation tissue. ***Diagnosis: Diabetic ulcer, granulation tissue
7 Which of the fo llowing proteins helps stimulate healing and angiogenesis in the wound of the patiem described in Question 6? ***(A) o.1-Antitrypsin ***(B) Caspase-9 ***(C) Lysozyme ***(D) o.2-Macroglobulin ***(E) Metalloproteinase
7 The answer is E: Metalloproteinase. Matrix metaHoproteinases (M1v!Ps) are crucial components in wound heaJjng because they enable cells to migrate by degrading matrix proteins. Members of t.hts protein ramHy include collagenase, srromelysin, and gdatinase. 1n addition to enhancing ceU migration, MMPs can cUsn.tpt cell-cell adhesions and release bioactive molecules stored in the matrix. MMP activity can be minimized by binding LO specific proteinase inhibi tors such as a 1-antiuypsin (choice A) and a 1-macroglobulin (choice D). Lysozyme (choice C) is a secretmy product of neutrophils Lhat degrades bacte1ial cell walls. ***Diagnosis: Diabetes mellitus
a A 68-year-old man pTesents for repair of an abdominal aortic aneurysm. Severe complicated atherosclerosis is noted at surgery, prompting concern for embolism of atheromatous matelial to the kidneys and Dlher organs. lf the patie!1l were to develop a renal conical infarct as a result of surgery, which of the following would be the most likely outcome7 ***(A) Chronic in l1ammation ***(B) Granulomamus inflammation ***(C) Hemangioma fonnation ***(D) Repair and regeneration ***(E) Scar formation
8 The answer is E: Scar formation. A large infarCL of the k1dney will heal by fibrosis (scar fonnarion) In most renal diseases, there is destruction of the exnacellular matrix framework. Repair and regeneration (choice D) ts then incomplete, and scar formation is the expected outcome. The regenerative capacity o[ renal Lissue is m<udmal in cortical tubules, less in medullruy mbules, and none.x:tsLent in glomerulL Recent data suggest that renal mbule repair occurs due to the proliferation of endogenous renal progenitor (stem) cells. Chronic inflammation (choice A) precedes scar fonnation. Granulomawus inOammanon (choice B) is not a compUcation of renal cortical in farction. Hemangiomas (choice C) are common benign tumors of endotheUal cells that usually occur in the skin. ***Diagnosis: Infarction; embolism, atheroembolus
9 A 40-year-old woman presents with a painless lesion on her ligh t ear lobe (shown in the image)_ She reports that her ea rs were pierced 4 months ago. Which of the following best e.'<plains the pathogenesis of this lesion7 ***(A) Clonal expansion o[ smooth muscle cells ***(B) Exuberam fom1ation of granulation tissue ***(C) Increased growth of capillary endothelial cells ***(D) Increased turnover of extracellular matrix proteoglycans ***(E) Maturation arrest of coUagen assembly
._9_ The answer is E: Maturation arrest of collagen assembly. Keloid is an e>.'Uberam scar that tends w p rogress beyond the she of initial injury and recurs after excision. Dark-skinned persons are more fTequemly affected by keloi.ds than light~skinned people. Keloids are characterized by changes in the ratio of type Ill to type l collagen, suggesting a maturation arrest""" in the h ealing process. Funher support for maturation arrest as an explanation for keloids and hypertrophic scars is the overexpression of fibronectin in these lesions. Keloids are unsightly, and attempts at surgical repair are always problematic. The other choices do not address the pathogenesis of keloids. ***Diagnosis: Keloid
10 A 58-year-old woman undergoes lumpecwmy for breast cancer. One month following surgery, she notices a fi rm 0.3-cm nodule along one edge of the surgical incision. Biopsy of this nodule reveals chronk inflammatory cells, multinucleated giant cells, and extensive fibrosis. The multinucleated cells in this nodule most likely fmmed in response to which of the following pathogenic stimuli? ***(A) Bacterial infection ***(B) Foreign material ***(C) Lymphatic obstruction ***(D) Neoplastic cells ***(E) VLral infection
10 The answer is B: Foreign material. Granulomatous inOammation is a subtype of chronic inflammation, which develops when acute inflammatory cells are unable to digest the injutious a gem (e.g .• suture or talc). Fusion of macro phages within the lesion results in the formation of multinucleated giant cells. None of the other choices elicit this type of granulomatous reaction. ***Diagnosis: Granulomatous inflammation
11 A 57-year-old man with a history of alcoholism presents with yellow discoloraLion of his skin and sclerae. Laboratory studies show elevated serum levels of liver enzymes (AST and ALT) A Lrichrome stain of a liver biopsy is shown in the image. A similar pattern of regeneration and fibrosis would be expecLed in the liver of a patiem with which of the following conditions? ***(A) Actne toxic liver injury ***(B) Chronic viral h epatitis ***(C) Fulminant hepatic necrosis ***(D) Hepawcellular carcinoma ***(E) Th ron1bosis of the portal vein
11 The answer is B: Chronit: viral hepatitis. Chronic liver injuty (e.g., chronic viral hepatitis) is associated with the development ofbroad col1agenous scars within the hepatic parenchyma. This is termed cirrhosis. Hepatocytes fom1 regenerative nodules that lack central veins and expand to obsnuct blood vessels and bile flow Ponal hypenension and jaundice ensue, despite adequate numbers of regenerated but disconnected hepatocytes. Acme toxic liver injury (choice A) is genera11y reversible. Fulminant hepatic necrosis (choice C), if the patiem survives, usually regenerates. Hepatocellular carcinoma (choice D) may be associated with rumor fibrosis but nm with regeneration. Portal vein tlu·ombosis (choice E) does not cause hepatic fibrosis but may be a complication of embolism. ***Diagnosis: Alcoho1ic 1iver disease, cirrhosis
12 A 10-year-old boy trips at school and scrapes the palms of his hands. TI1e wounds are cleaned and coveTed with s terile gauze. \~ich of the following Lerms best characterizes the healing of these superficial abrasions? ***(A) Fibrosis ***(B) Grrumlation tissue ***(C) Primary intention ***(D) Regeneration ***(E) Secondary intention
12 The answer is D: Regeneration. Su perficial abrasions of the skin heal by a process of regeneration. It is mediated by stem cells or stabile cells that are able to progress through the cell cyde and fully restore normal tissue organization and function. Cellular migration is the predominant means by which the wound surface is reepithelialized. Fibrosis (choice A) refers to abenant healing with deposition of collagen-rich scar tissue. Granulation tissue (choice B) forms during the repair of deep wounds. Primary and secondmy intennons (choices C and E) arc fearures of healing in deeper wounds. ***Diagnosis: Superfi cial abrasion
13 'vVhich of the follmving cellular processes helps reswre nom1al epith elial structure and funccion in the patiem described in Qu estion 12? ***(A) Collagen and fibronectin-rich extracellular matrL-..: deposition ***(B) Comact inhibition of epithelial cell growth and motility ***(C) Myofibroblast differentiation and syncytia forma tion ***(D) Platelet activation and intravascular coagulation ***(E) Prolife ration of capillary endothelial cells (angiogenesis)
13 The an-swer is B: Contact inhibition of epithelial growth and motility. Mamration of the epidermis requires an intact layer of basal cells that are in diJ·ect contact with one anod1er. lf this contact is disrupted, basal epithelial cells at the wound margin become actiYated and evemua1ly reestablish contact with oilier basal cells through extensive cell migration and mitosis. \Vhen epithelial continuity is reestablished, migration and cell division cease, and the epidermis resumes its nom1al cycle of maturation and shedding. This process of epithelial grovvth regulation is referred to as "contact inhibition of growth and motility" The other choices describe responses to deep wound healing ***Diagnosis: Superficial abrasion, regeneralion
14 A 34-year-old woman has a benign nevus removed from her back under local anesthesia. \Vhich of the following families of cell adhesion molecules is the principal component of the provisional marrL-x that fonns dming early wound healing' ***(A) Cadherins ***(B) fibronectins ***(C) lntegrins ***(D) Lam1nins ***(E) SeleCLins
14 The answer is 8: Fibronedins. Fibronectins. are adhesive glycoprmeins that are widely distribmed in stromal connective tissue and deposited at the si te of tissue mjury During the initial phase of healing, fibronectin in the extravasated plasma is cross-linked to fibrin, collagen, and other extracellular matrix componems by the action of nansglmaminases. This cross-linking proVIdes a provisional stabilization of the wound during the first several hours. Fibronectin, cell debTis, and bacterial products are chemoam actams for a variety of cells that are recruited to the wound site over the next several days. Selectins (choice E) are sugar-binding glycoproteins that mediate the initial adhesion or leukocytes to endothelial cells at si tes of innammation. They are found at the cell surface and are not pan of the extracellular matrix. Cadherins (choice A) and integrins (choice C) are cell adhesion molecules. Like the selectin family of cell adhesion proteins, they are found at the cell surface and are not part of the extracellular matr Lx. ***Diagnosis: Wound heallng
15 Which of the following fammes of glycoproteins plays the most important role in regulating the migration and differentiation of leukocytes and connective tissue celts during wound healing in Lhe patient described in Question 14? ***(A) Cadherins ***(B) FibriUins ***(C) lmegrins ***(D) Laminins ***(E) Selectins
15 The answer is C: lntegrins . The locomotion of leukocytes i.s powered by membrane extensions called lamellipodia. Slower moving cells, such as fibroblasts, extend fingerlike membrane pronusions called filopodia. The leading edge of the cell membran e adheres lO the extracellular mauLx through tra115membrane adhesion receptors termed integJins. These cell surface glycoproteins transmit mechanical and chemical signals, thereby regulating cellular survival, proliferation, differemiadon, and migration. The motility or epithelial cells is also regtllated by imegrin receptors. Cadherins (choice A) are ceU-celJ adhesion molecules. Fibrillins (choice B) are smtcrural molecules that interact with elastic fib r ils. Lamil1ins (choice D) are basement membrane glycoproteins Selec tins (choice E) mecliate the recruitment of neutrophils in acme inflammation bm do not mediate directed cell migration at the site of tissue injury. ***Diagnosis: Wound healing
16 A 29-year-old cm·pemer receives a traumatic laceration w her ldt arm. vVhich of the following is the most important facror that determines whether this wound will heal by primary or secondary intention? ***(A) Apposition of edges ***(B) Depth of wound ***(C) Metabolic sLams ***(D) Skin site affected ***(E) Vascular supply
16 The answer is A: Apposition of edges. Healing by primary inten tion occurs in wounds wi th closely apposed edges and rn]nimal tissue loss. Such a wound requires only nunimal ceJJ proliferation and neovascularization to heal, and the result is a small scar. Healing by secondary intention occurs in a gouged wound, in which the edges are far apart and in which there is substantial tissue loss. This wound requires wound contraction, extensive cell proliferation, and neovascularization (granulation tissue) to heaL Granulation tissue is eventually resorbed and replaced by a large sear that is functionally and esthetically unsatisfactmy. The other choices are important detern1Jnants of the oULcome of wound healing, btu they do not provide a point of distinction between primary and secondary intentions healing. ***Diagnosis: Healing by primary intention
17 Activated fibroblasts, myofibroblasts, and capillary sprouts are most abundam in the wound of the patient desc1ibed in Question 16 at which of the fo llowing times after injury? ***(A) 3 to 6 hours ***(B) 12 to 2+ hours ***(C) 3 to 5 days ***(D) 8 to 10 days ***(E) 2 \<Veeks
17 The answer is C: 3 to 5 days. Activated fibroblasts, myofibroblasts, and capillary sprouts are abundant in healing wounds 3 to 5 days following injury Activated fibroblasts change shape from oval to bipolar as they begin to fonn collagen and synthesize a variery of extracellular matrix proteins. Neur.rophils accumulate In the wound 12 lO 24 hours after injury (choice B) . Mature scar tissue would be visible 2 weeks following injury (choice E). ***Diagnosis: Healing by primal) ' intention
18 A 9-year-old boy receives a deep laceration over his right eyebrow playing ice hockey. The wound is cleaned and sutured. Which of the following desctibes the principal function of macropha.ges that are present in the wound 24 to 48 hours after injury? ***(A) Antibody production ***(B) Deposition of collagen ***(C) Histamine release ***(D) Phagocytosis ***(E) ·wound contraction
18 The answer is D: Phagocytosis. Macrophages an ive at the site of injmy shorLly after neutrophils, bm they persist in the wound for days longer. Macrophages remove debris and orchestrate the fom1ation of granulation tissue by releasing cywkines and chemoattractants. None of the other choices are functions of tissue macrophages. For example, plasma celts produce antibodies (choice A), and myofibroblasts mediate wound contraction (choice E). ***Diagnosis: Laceration
19 Which of the followin g collagens is deposited fiTSl dming wound healing in the padem described in Question 18? ***(A) Type 1 ***(B) Type II ***(C) Type Ill ***(D) Type N ***(E) Type V
19 The answer is C: Type Ill. Concurrent with fibrinolysis , a temporary matrix composed of prOLeoglycans, glycoproteins, and type HI collagen is deposited. The secretion of type lll collagen is a forerunner to the fo rmation of t)rpe l collagen {choice A), which will impan greater tensUe strength to the wound. TGF-~ enhances the synthesis of collagen and 6bronectin and decreases metalloprOLeinase transcription and matrix degradation. Extracellular cross-linking of newly synthesized collagen further increases the mechankal s trength of the wound. Type 11 collagen (choice B) is found in canilage. Type lV collagen (choice D) is found in basement membranes. ***Diagnosis: Laceration
20 A 16-year-old boy suffers a concussion during an ice hockey game and is rushed to the emergency mom. A CT scan of the brain reveals a cerebral contusion of the left fromal lobe. The boy lies comatose for 3 days but eventually regains consciousness. Which of the following cells is the principal mediator of scar formation in the central nervous system of this patient? ***(A) Fibroblasts ***(B) Glial cells ***(C) Nemons ***(D) Oligodendrocytes ***(E) Schwann cells
20 The answer is B: Glial cells . Damage to the brain or spinal cord is followed by growth of capillaries and gliosis (i.e., the proliferation of asLrocytes and microglia). Gliosis in the cemral ne1vous system is the equivalem of scar formation elsewhere; once established, iL remams permanemly. In spinal cord injulies, a.'<onal regeneration can be seen up to 2 weeks after injury. After 2 weeks, gliosis has taken place and auempts at axonal regeneration end. ln the central nervous system, axonal regeneration occurs only in the hypOlhalamohypophysial region, where glial and capillary ban iers do not interfere with axonal regeneration. Axonal regeneration seems to require contact with exu·acellular fluid containing plasma proteins. The other cells listed do not prollferate significantly in response Lo brain or spinal cord injury. ***Diagnosis: Cerebral contusion, gliosis
21 A 30-year-old firefighter suffers e}..'tensive third-degree burns over his arms and hands. This patient is at high risk for developing which of the following complications of wound healing? ***(A) Contracture ***(B) Dehiscence ***(C) Incisional hernia ***(D) Keloid ***(E) Traumatic neuroma
21 The answer is A: Contracture. A mechanical reduction in the size of a wound depends on the presence of myofibroblasts and sustained cell contraction. An exaggeration of these processes is te rmed contracture and results in severe defonnity of the wound and surrounding tissues. Comracrures are particularly conspicuous in the healing of se1ious burns and can be severe enough to compromise the movement of joints. ***Diagnosis: Comracmre
22 A 23-year-old man suffers a cmsh injury of his foot, which becomes secondarily infected. He undergoes a below-the-knee amputation. Six momhs later, the patient complains of chronic pain at the site of amputation. A firm nodule is identified at the scar site. A biopsy of the nodule demonstrates haphazard growth of nerves (shown in the image). Which of the following is the most likely diagnosis? ***(A) Ganglloma ***(B) Ga nglioneu rom a ***(C) Hamanoma ***(D) Neural nevus ***(E) Neuroma
22 The answer is E: Neuroma. Neurons in the peripheral nervous system can regenerate their a;'{ons, and under ideal circumstances, interruption in the continuity of a peripheral ne1ve results in complete functional recovery However, if the cut ends are not in perfect alignment or are prevented from establishing continuity by inflammation, a rraumatic neuTOma results. This bulbous lesion consists of disorganized axons and proliferating Schwann cells and fibroblasts. In this patients biopsy, the original nerve (lower left) enters the neuroma. The nerve is surrounded by dense collagenous tissue , which appears dark blue in this trichrome stain. Ganglioma (choice A), ganglioneuroma (choice B), and hamartoma (choice C) are benign neoplasms. ***Diagnosis: Tmuma£ic neuroma
23 A 34-year-old man presems with a 5-day history of a painful sore on his hand. Physical examination reveals a 0.5-cm abscess on the extensor surface of the left hand that drains a thick, purulem material. Diapedesis of leukocytes into and around this patients infected wound occurs primarily at which of the following anatomic locations? ***(A) Lymphatic capillaries ***(B) Postcapillary venules ***(C) Precapillary anerioles ***(D) SmaU dermal arteries ***(E) Small dennal veins
23 The answer is B: Postcapillary venules . One of the earliest responses following tissue injury occurs within the microvasculature at the level of the capillary and postcaplllary venule. 'vVithin this vascular network are the major componen ts of the inOammatory response, including plasma, platelets, erythrocytes, and circulating l eukocytes. Following injury, changes in the structme o[ the. vascular wall lead tO activation of endothelial ce.U.s, loss of vascular integtity, leakage of fluid and plasma components from the intravascular compartment, and emigration of erythrocytes and leukocytes from the vascular space into the extravascular tissue. (diapedesis). Leukocyte recmitment in the postcapillary venule is initiated by i.meraction of leukocytes with endothelial cell su rface selectin molecules. Leuko<::ytes do not typically undergo diapedesis at the other anawmic locations listed. ***Diagnosis: Carbuncle, margination
24 A 35-year-old pregnam woman with a history of chronic gasn·itis presents to the emergency room complaining of acute abdominal pain. Physical examjnation reveals hepatomegaly, ascites, and mild jaundice. The patient subsequently develops acute hepatic failure and expires. Autopsy reveals thrombosis of the hepatic veins (Budd-Ch1ari syndrome). During t]le autopsy, a lesion is identified in the distal stomach and examined by light microscopy (shov.rn in tl1e image). Which of the following best describes this incidental finding at autopsy? ***(A) Carcinoma ***(B) Contracture ***(C) Diverticulum ***(D) Granuloma ***(E) Ulcer
24 The answer is E: Ulcer. lncidemal findi ngs are frequently encountered at autopsy. In this case, a pep tic ulcer is identified in the distal stomach. Histologic examination shows focal destrunion of the mucosa and full- thickness replacement of the muscularis with collagen-rich connective tissue (see photomicrograph) Gasrric ulcers are usually single and less than 2 em in diameter. Ulcers on the lesser curvature are commonly associated whh chronic gasuilis (this patient), whereas those on the greater curvature are often related to NSAIDs. Grossly, chronic peptic ulcers may closely resemble gastric carcinoma (choice A). Thus, the endoscopist must take multiple biopsies from the edges and bed of any gasnic ulcer. Although contraccion and scan ing of gastric u1cers (choice B) may occur and may cause pyloric stenosis, the hiswpathologic findings do not suggest this complication. Divenicula of the stomach (choice C) are rare and, if present, are usually Uned by a normal gasuic mucosa. Granuloma (choice D) features inflammawry cells tha t are not observed. ***Diagnosis: Gastric ulcer, peptic ulcer disease
1 A 35-year-old man a5ks for advice regarding seasonal eye itching and runny nose. Recurrent conjunctivitis in thJs patient is most likely caused by which of the following mechanisms of disease? ***(A) Auroinununity ***(B) Bacterial infection ***(C) Chemical toxicity ***(D) Hypersensitivity ***(E) Viral infection
1 The answer is D: Hypersensitivity. Although rile incorrect choices may cause eye irritation, seasonal conjunctivitis is typically caused by allergies to pollens that are released during a particular Lime of the year. Allergic rhinitis (hay fever) is the most common type I bypersensitivily disease in adulLS. It may be caused by pollen, house dust, animal dandmff, and many other al lergens. Antigens inhaled react with lhe lgE atlached to basophils in the nasal mucosa, thereby uiggerlng the release of vasoactive substances sLOred in cytoplasmic granules. Histam. ine , the main mediawr released fmm mast cells, increases the permeabil.ity of mucosal vessels, causing edema and sneezing. ***Diagnosis: Conjunctivitis, hypersensltivity reaction
2An 8-year-old boy presen£S with periorbitaJ edema and throbbing headaches. His parents repon that the boy had a "sn·ep throatM 2 weeks ago. Urinalysis shows 3+ hemarmia. A renal biopsy shows hypercellular glomeruli, and electron microscopic examination of glomemli discloses subepithelial humps. ""Which o[the following best explains the pathogenesis" of glomemlonephtitis in this patiem? ***(A) Amineutrophil cytoplasmic amoamibodies ***(B) Deposition of circulating immune complexes ***(C) Directly cytotoxic IgG and lgM antibodies ***(D) lgE-mediated mast cell degranulation ***(E) T cell-mediated delayed hypersensidvity reaction
2 The answer is 8: Deposition of circulating immune c>Omplexes. Type lll hypersensitivity reactions are characterized by immune complex deposition, complemem fi"lffition, and localized inO.ammation. Antibody d.ire~cred against either a circulating amigen or an antigen that is deposaed in a tissue can give rise to a type m response. Diseases that seem w be most clearly anribmable to the deposition of immune complexes are systemic lupus erythematosus, rheumawid arthritis, and varieties of glomemlonephritis. Strepwccocal infection in this case led to the deposition of amigens and amibod.ies in glomerular basemem membranes, resulting in clinical features of nephritic syndrome (e.g., hematuria, oliguria, and hypenension). Poststreptococcal illnesses do not include any of the other choices. ***Diagnosis: Postinfectious glomerulonephritis
3 A 21-year-old woman p resents with a J -mon th history of malaise, joint pain, weight loss, and sporadic fever. The patient appears agitated. Her temperature is 38°C (101 °F). Other physical findings include malar rash , erythematouspink plaques with telangiectatic vessels, oral ulcers, and nonhlanching purpunc papules on her legs. Laboratory studies show elevated levels of blood urea nitrogen and creatinine. Antibodies directed to which of the following ami.gens would be expected in the semm of this padem7 ***(A) C-ANCA (ami-protelnase-3) ***(B) Double-stranded DNA ***(C) P-ANCA (an ti-myeloperoxidase) ***(D) Rheumatoid factor ***(E) Scl-70 (anti-topoisomerase I)
3 The answer is 8: Double-stranded DNA. Systemic lupus erythematosus (SLE) is an autoimmune, inflammatory disease that may involve almost any organ but characteiistically affects the kidneys, joints, serous membranes, and skin. Autoantibodies are formed against a variery of self-antigens. The most important diagnostic autoantibodies are those against nuclear amigens-in panicular, antibody to double-stranded DNA and to a soluble nuclear antigen complex that is pan of the spliceosome and is tenned Sm (Smith) antigen. High titers of these nvo autoantibodies (tenned antinuclear antibodies) are nearly pathognomonic for SLE. Antibodies to rheumatoid factor (choice D) are seen in patients wilh rhemnawid arthritis. Antineutrophil cytoplasmic antibodies (choices A and C) are seen in patients wiLh small vessel vasculitis (e.g., \Vegener granulomalosis). ***Diagnosis: Systemic lupus erythematosus
4 Serum levels of complement proteins may be reduced dming the active phase of disease Ln the patient desc1ibed in Quesrion 3 due to which of rhe foUo\ving mechanisms of disease? - ***(A) Binding of complement LO immune complexes ***(B) Decreased complement protein biosynthesis ***(C) Defective activation of the complement cascade ***(D) Increased minary excretion of immunoglobulins ***(E) Stimulation of t11e acme phase response
4 The answer is A: Binding of complement to immune complexes. Acquired deficiencies of early complement components occur in patients with autoimmune diseases, especially those associated with circuJating immune complexes kg., systemic lupus erythemawsus [$LEI). Antigen-antibody complexes fom1ed in the circulation during the active stage or these diseases lead to a marked reduction in circulating levels of complemem proteins (hypocomplementemia). None of the other choices mediates hypocomplementemia in patients with SLE. ***Diagnosis: Systemic lupus erythematosus
5 A 45-year-old woman complains of severe headaches and difficulty swallowing. Over the past 6 months, she has noticed small, red lesions around her moULh as well as thickening of her skin. The pat.lem has "stone facies.. on physical examination. Which of the following antigens is the most common and most specific target of autoantibody in patie:ms with this disease? ***(A) C-ANCA (anti-proteinase-3) ***(B) Double-stranded DNA ***(C) P-ANCA (ami-myeloperoxidase) ***(D) Scl-70 (anti-ropoisomerase I) ***(E) 5$-NSS-B
5 The answer is D: Scl-70 {anti-topoisom~rase I). Sclerodenna is an amoimmune disease of connective tissue. Circulating male fetal cells have been demonstrated in blood and blood vessel walls of many women with scleroderma \Vho bore male children many years before the disease began Accordingly, it has been suggested that sclerodem1a in these patients is stmtlar to graft-versus-host disease. Antinuclear antibodies are common but are usually present in a lower tiler than in patients with SLE. Antibodies virtually specific for sclerodenna include (l} nucleolar autoantibodies (primarily against RNA polymerase); (2) amibodies to Scl-70, a nonhistone nuclear protein topoisomerase; and (3) amicemromere antibodies, which are associated with the "CREST" variant of the disease. The Scl-70 autoantibody is most common and specific for the diffuse form of scleroderma and is seen in rO% of patients. Autoantibodies to double-stranded DNA (choice B) are seen in patients with SLE. Autoantibodies to SS-NSS-B (choice E) are seen in patients wiLh SjOgren syndrome. ***Diagnosis: Scleroderma
6 A skin biopsy in the patiem described in Question 5 would most likely show a pe rivascular accumulation of which of the following extracellular mallix proteins? ***(A) Collagen ***(B) Elastin ***(C) Entactin ***(D) Fibronectin ***(E) laminin
6 The answer is A: Collagen . Scleroderma is characterized by vasculopathy and excessive collagen deposition in the skin and internal organs, such as the lung, gastrointestinal tract, healt, and kidney. The disease occurs four times as often in women as in men and mostly in persons aged 25 to 50 years. Progressive systemic sclerosis i.s characterized by widespread excessive collagen deposition. There is emerging evidence for the e>..'J)ansion of fibrogenic clones of fibroblasts. These clones display augmented procollagen synthesis, including increased circulating levels of Lype lU collagen aminopropeptide. Ti.ssue levels of the other proteins are not significantly altered in patients with scleroderma. ***Diagnosis: Scleroderma
7 Dming the physical examination of a 22.-year-old man, a purified protein derivative isolated from Mycobaaeritm1 wberculosL~ is injected imo Lhe skin. Three days later, the injection site appears raised and indurated. Which of the following glycoproteiJ1s was directly involved in antigen presentation during the init]ation phase of delayed hypersensitivity in this patient? ***(A) CD4 ***(B) CD8 ***(C) Class [ HLA molecules ***(D) Class II HLA molecules ***(E) GlyCAM-l
7 The answer is D: Class II HLA molecules. Delayed-type hypersensitivicy is de fined as a tissue reaction involving lymphocytes and mononuclear phagocytes, which occurs in response to a soluble protein antigen and reaches greatest intensity 24 to 48 hours after initiation. ln the initial phase, foreign protein antigens or chemical ligands interact with accessory cells beating class li HI..A molecules. Protein amigens are actively p rocessed into shon peptides within phagotysosomes and are presented on the cell surface in conjunction with the class 11 HLA molecules. The latter are recognized by CD4. T cells (choice A), which become activated to synthesize an array of cytokines. The cytokines recruit and activate lymphocytes, monocytes, fibroblasts, and other iJ1flammaLOry cells. Suppressor T celts are CD8+ (choice B). Class I HI..A molecules {choice C) provide targets for cell-mediated cytotoll."icity. GlyCAM-1 (choice E) is a cell adhesion molecule involved in lymphocyte trafficking. ***Diagnosis: Delayed-type hypersensitivity
8 A 54-year-old woman is involved in an automobile accident and requires a blood transfusion. Five hours later, she becomes febrile and has severe back pain. Laboratory sllldies show evidence of imravascular hemolysis. h is discovered that type A Rh+ blood was given by mistake w this type B Rh+ patient. Which of rhe follo•ving best e-xplains the development of intravascular hemolysis in this patient? ***(A) Antibody-dependent cellular cytotoxicity ***(B) Antibody-mediated complement fi:xadon ***(C) Delayed-type hypersensitivity ***(D) Immune complex disease ***(E) Immediate hypersensitivity
8 The answer is B: Antibody-mediated complement fixation. Type 11 hypersensitivity reactions are mediated by antibodies directed against fixed an tigens. ln this case, prefonned antibodies in the patients blood auached to foreign antigens (oligosacchalides) on the membranes of the transfused erythrocytes. At sufficiem densit)~ bound immunoglobulms fix complemem. Once aCLivated, the complement cascade leads to the desrruction of the target cell through [ormation of a membrane ;;mack complex. This type of complemem-medimed cell lysis occurs in autoimmune hemolytic anemia. Antibody-dependent cellmediated cytotoxicity (ADCC, choice A) involves cytolytic leukocytes that attack antibody-coated target cells. ADCC may be involved in the pathogenesis of some autoimmune diseases (e.g., autoimmune thyroiditis). Delayed-Lype hypersensitivity (choice C) occurs over a period of days and does not involve preformed antibodies. ***Diagnosis: Hemolytic anemia, jaundice
9 A 40-year~old man complains of having yellow skin and sclerae , abdominal tenderness, and dark urine. Physical examination reveals jaundice and mild hepatomegaly. Laboratory stu dies demonstrate elevated serum bilintbin (3.1 mgfdl), decreased serum albumin (2.5 gfdL), and prolonged prothrombin Lime (17 seconds). Serologic tests reveal amibodies to hepatitis B core antigen (IgG anti-HBc.Ag). The senun is also posiuve for HBsAg and HBeAg Which of the fo llo'Ning glycoprO[eins serves as the principal cell surface receptor for viral antigens on B lymphocytes in this patient' ***(A) CD4 ***(B) CDS ***(C) HLA class 1 molecules ***(D) HLA class 1I molecules ***(E) Membrane immunoglobulm
9 The answer is E: Membrane immunoglobulin {mig) . The clinicopathologic findings presemed here indicate that rhis patient is a chrome HBV carrier with active hepatitis. Humoral immune responses to specific viral antigens tn this patient involve the activation and differen tiation of B lymphocytes into antibody-secreting plasma cells. Analogous to T cells, B cells express an antigen-binding receptor, namely mtg. This immunoglobulin bears the same antigen specificity as the soluble immunoglobulin that is ultimately secreted. Class I Hl.A molecules (choice C) provide targets for CD8• T cells in cell-meruated cytotoxiclty. Class ll HLA molecules (choice D) are recognized by CD4'" T cells, which become activated to synthesize an array of cytoklnes. ***Diagnosis: Humoral immunity, chronic hepatitis
10 What glycoprotein on virally infected hepatocytes provides a target for cell-mediated cytotoxicity in the patient described in Question 9? ***(A) CD4 ***(B) CD8 (C} Class l HLA molecules ***(D) Class II HLA molecules ***(E) GlyCAM-1
10 The answer is C: Class I HLA molecules. Class I molecules of the major histocompatibility complex present foreign peptides and are recognized by cytotoxic T lymphocytes dming graft reiection or d uring cell-mediated killing of virus-infected cells. All tissues e.,xpre.ss class [ molecules, whereas class Il molecules (choice D) are displayed ptimati ly on macrophages and B lymphocytes. CD4 and CDS (choices A and B) are cell surface markers of helper and killer T lymphocytes, respectively. GlyCA.M-1 (choice E) facilitates lymphocyte recirculation by provtding a recepmr for leukocyte attachment to high endothelial venules. ***Diagnosis: Chronic hepatitis B
11 A 45-year-old woman presents with a 1-year history of dry mouth and eyes_ A biopsy of a minor salivary gland reveals infiltrates of lymphocytes forming focal genninal centers. 'vVhich of the following cellular organelles is a target for autoantibodies in this patient? ***(A) Centromere ***(B) Lysosome ***(C) Nucleus ***(D) Peroxisome ***(E) Plasma membrane
11 The answer is C: Nucleus. Sjogren syndrome (SS) is an autoimmune disorder characterized by keratoconjun ctivitis sicca and xerostomia in the absence of other connective tissue disease. The production of autoantibodies, particularly aminuclear antibodies directed against DNA or nonhistone proteins, typically occurs in patients with SS. Autoantibodies to soluble nuclear nonhistone proteins, especially tl1e antigens SS-A and SS-B, are found in half of patients with primary $Sand are associated with more severe glandular and extraglandular manifestations. Autoantibodies to DNA or hjstones are rare. Organ -specific autoantibodies, such as those directed against salivary gland antigens, are distinctly uncommon. Autoantibodies Lo centromere proteins (choice A) are seen in the CREST variant of progressive systemic sclerosis. ***Diagnosis: Sjogren syndrome
12 An 8-month-old boy with a history of recurrent pneumonia is found to have almost no circulating lgG. Cellular immunity is normal. His brother had this same disease and died of echovims encephalitis. His parents and sisters have normal serum levels of lgG. What is the appropriate diagnosis? ***(A) DiGeorge syndrome ***(B) Isolated lgA deficiency ***(C) Severe combined immunodeficiency ***(D) WiskoLt-Aldrich syndrome ***(E) X-lin ked agammaglobulinemia of Bruton
12 The answer is E: X-linked agammaglobulinemia of Bruton. The congenital disorder Bruton X-linked agammaglobulinemia appears in male infams at 5 to 8 momhs of age, the period during which maternal antibody levels begin to decline. The infant suffers from recunem pyogenic infections and severe hypogammaglobulinemia. There is an absence of both mature B cells in peripheraJ blood and plasma cells in lymphoid tissues. The genetic defect, located on the long arm of the X cl1rornosome, is an inactivating mutarion of the gene for B-cell ryrosine ldnase, an enzyme critical to B-lymphocyte maturation. Vviskott-Aldrich syndrome (choice D) is also an X-lin ked genetic disease but is characterized by defects in both B-cell and T-cell functions (i .e. , humeral and cellular immunity). DiGeorge syndrome (choice A) is a developmental disorder characte1ized by thymic and parathyroid aplasia. ***Diagnosis: X-linked agammaglobulinemia of Bmwn
13 A 52-year-old woman with a history of systemic hypenension and chronic renal failure undergoes kidney transplantation, but the graft fails to produce urine. A renal biopsy is diagnosed as "hyperacute transplant rejection." Graft rejection in this padem is caused primarily by which of the following mediators of immunity and inflammation? ***(A) Cywroxic T lymphocytes ***(B) Helper T lymphocytes ***(C) Mononuclear phagocytes ***(D) Natural killer cells ***(E) Prefo rmed antibodies
13 The answer is E: Preformed antibodies. Hyperacute rejection occurs within minutes to hours after transplantation. It is manifested clinically as a sudden cessation of u rine output, along with fever and pain in the area of Lhe graft site. This immediate rejection is mediated by preformed antibodies and complement activation products. Lymphocytes and macrophages (choices A, B, and C) are associated with acute and chronic graft rejection. ***Diagnosis: Hyperacme grafr rejection
14 A 30-year-old woman complalll5 of impaired speech and frequent aspiration of food. Physical examinadon reveals diplopia and drooping eyelids. A mediastinal mass is removed and diagnosed as thymoma. The symptoms of muscle weakness in Lhis padem are caused by antibodies directed against which of the following cellular componems? ***(A) Acetylcholine receptor ***(B) Calcium channel ***(C) Desmogkin-3 ***(D) Rheumatoid factor ***(E) Thyroid-stimulating honnone (ISH) receptor
14 The answer is A: Acetylcholine receptor. Myasthenia gravis is a type li hypersens1tivity disorder caused by antibodies thar bind to the acerylcholine recepwr. These antibodies interfere with the transmission of neural impulses at the neuromuscular junction, causing muscle weakness and easy fatigabilh:y External ocular and eyelid muscles are most often affected, but the disease is often progressive and may cause death by respiratory muscle paralysis. Autoantibodies to desmoglein-3 (choice C) are found ln patients w'ilh pemphigus vulgaris, an autoimmune bl istering skin disorder. Antibodies to the TSH receptor (choice E) are seen in patients with Graves hyperthyroidism. Antibodies to calcium channels (choice B) are found in patients with Eaton-Lambert syndrome. This paraneoplastic syndrome also manifests as muscle weakness but is usually associated with smaU cell carcinoma of the lung. Rheumatoid factor (choice D) represents multiple antibodies directed against the Fe ponion of IgG and is seen in pariems \Vith rheumatoid arthritis and many other collagen vascular diseases. ***Diagnosis: Myasthen ia gravis, thymoma
15 A 31-year-old man wlth AIDS complains of difficulty swallowing. Examination of his oral cavity demonsu-ates whitish membranes covering much of his tongue and palate. Endoscopy also reveals several whitish, ulcerated lesions in the esophagus. These pathologic find ings are fundamentally caused by loss of which of the following immune cell"> in this patient? ***(A) B lymphocytes ***(B) Helper T lymphocytes ***(C) Killer T lymphocytes ***(D) Monocytes/macrophages ***(E) Natural killer (NK) cells
15 The answer is B: Helper T lymphocytes. The relentless progression of HlV infection is now recognized as a continuum that extends from an initial asymptomatic state to the immune depletion that characterizes patients with oven AID$. The fundamental lesion is infection of CD4+ (helper) T lymphocytes, which leads to the depletion of this cell population and impaired immune function. As a result, patients with AID$ usually die of opportunistic infections. HN does infect the monoc)'le/macrophage Uneage (choice D), but infected cells exhibit little if any cytotoxicity. NK cell activity (choice E) is also decreased in AIDS. This defect may contribute to the appearance of malignant mmors and the viral infections that plague these padems. The suppression of NK cell activity has been related to a decrease in the number of NK cells and to a reduction in lL-2 levels due to the loss of Co 4· cells. ***Diagnosis: AIDS
16 Which of the following en zymes converts the HIV genome imo double-stranded DNA in h ost cells in the patient described in Question 15? ***(A) DNA polymerase (Pol- l ) ***(B) DNA polymerase (Pol-2) ***(C) lmegrase ***(D) Reverse transcriptase ***(E) Topoisomerase
16 The answer is D: Reverse transcriptase. The primary etiologic agent of AIDS is HIV- 1, an enveloped RNA retrovirus that contains a reverse mmscrtptase (RNA-dependent DNA polymerase). After it enters into the cytoplasm of a T lymphocyt e, the virus is uncoated, and its RNA is copied into double-stranded DNA by retroyjral reverse trat15Cliptase. The DNA derived from the virus is imegrated imo the host genome by the vinl imegrase protein (choice C), thereby producing the latent proviral fom1 of HIV-1. Viral genes are replicated along with host chromosomes and, therefore, persist for the life of the cell. ***Diagnosis: ArDS
17 A 20-year-old woman with a history of asthma and aller· gies undergoes skin testing to identify potential allergens in her environmem. A positive skin reaction to ragweed in this patient would be mecUated by which of the following classes of immunoglobulin? ***(A) lgA ***(B) IgD ***(C) IgE ***(D) IgG ***(E) IgM
t7 The answer is C: lgE. Immediate-type hypersensitivity is manifested by a localized or generalized reaction that occurs within minutes after exposure to an antigen or "allergen" to which th e person has previously been sensitized In its generalized and most severe Form, immediate hypersensitivit}' reactions are associated \vi th bronchoconstriction, airway obstruction, and circulatory collapse, as seen in anaphylactic shock. Type I hypersensitivity reactions feature the formation oflgE antibodies that bind avidly to Fc-epsilon (Fe-E) receptors on mast cells and basophils The high-avidity bLnding of tgE accounts for the term cytophilic antibody. Once exposed to a specific allergen that has resulted in the formation of IgE, a person is sensitized. Subsequent responses to the allergen induce an immediate release of a cascade o[ proinilammatmy mediators. These meciJators are responsible for smooth muscle contraction, edema formation, and the recruitment of eosinophils. None of the other immunoglobulin classes mediates immediate hypersensitivity. ***Diagnosis: Asthma
18 A 53-year-old woman complains of progressive weight loss, nervousness, and sweating (patiem shown in me image). Physical examinmion reveals tachycardia and exophthalmos. Her thyroid is diffusely enlarged and warm on palpadon. Se rum levels of Lhymid-stimulaling hormone (TSH) are low, and levels of thyroid hom1ones (T3 and T} are markedly elevaled. Which of me following mechanisms of disease best e}qJlains the pathogenesis of this patiems thyroid candid on? ***(A) Antibody-dependent ceUular cytotoxicity ***(B) Cytopathic autoantibodies ***(C) Delayed-type hypersensitivity ***(D) Immedtate hypersensitivity ***(E) Immune complex disease
18 The answer is B: Cytopathic autoantibodies. Graves disease is a rype 11 hypersensitivity disorder caused by antibodies to the T$H receptor on follicular cells of the thyroid. Antibody binding to the T$H receptorstimulates a release of teu·aiodothyronine (T) and triiodothyronine (T) from the thyroid imo t:he circulation. Circulating T~ and T 3 suppress TSH production in the pi tuitary Sweating, weight loss, and tachycardia are evidence of the hypem1etabolism typical of hypenhyroidism. Graves disease also causes exophthalmos. Delayed-type hypersensitivity (choice C) is seen in patients with poison ivy and graft rejection. Immune complex disease {choice E) is caused by deposition of immune complexes and complement activation. ***Diagnosis: Graves disease
19 A 12-month-old infant with a history of recurrent infections, eczema, generalized edema, and easy bruising is diagnosed with an X-hnked, recessive , congenital immunodeficiency. The CBC shows thrombocytopenia. What is the most likely diagnosis? ***(A) DiGeorge syndrome ***(B) Isolated lgA deficiency ***(C) Severe combined immunodeficiency ***(D) Wiskott-Aldrich syndrome ( E) X-linked agammaglobulinemia of Bruton
19 The answer is D: Wiskott-Aidrich syndrome. This rare syndrome is characterized by (1) recurrent infections, (2) hemorrhages secondary to thrombocytopenia , and (3) eczema. It typically manifests in boys within the first Cewmomhs of life as petechiae and recurrent infections (e.g., dianhea). It is caused by numerous distinct mmarions in a gene on the X chromosome that encodes a protein called WASP (WiskottAldlich syndrome protein), which is ex1Jressed at high levels in lymphocytes and megakaryocyres. WASP binds members of the Rho family of GTPases. WASP itself comrols the assembly of actin filaments that are required to form microvesicles. X-linked agammaglobulinemia of Bruton (choice E) is not associated with thrombocytopenia and eczema. Choices A, B, and Care not X-linked genetic diseases. ***Diagnosis: Wiskon-Aldrich syndrome
20 A 24-year-old woman with leukemia receives an allogeneic bone marrow transplant. Three weeks later, she develops a skin rash and diarrhea. Liver function tests show elevated serum levels of AST and ALT. A skin biopsy discloses a sparse lymphocytic infiluate in the dermis and epidermis, as well as apoptotlc cells in the epidermal basal cell layer. Skin rash and dianhea in this patient are caused ptimalily by which of the follm"~.•ing cells? ***(A) Donor lymphocytes ***(B) Donor plasma cells ***(C) Fixed tissue macrophages ***(D) Recipient lymphocytes ***(E) Recipient plasma cells
20 The answer is A: Donor lymphocytes. The advem of transplantation of bone man-ow into patients whose immune system has been ablated or into otherwise immunodeficient patients has resulted in the complication of graft-versus-host disease (GVHD). GVHD occurs when lymphocytes in the grafted tissue recognize and react to the recipient. GVHD can also occur when an ilmmmodefiCiem patient is u·ansfused with blood comailung HlA-incomparible lymphocytes. The major organs affected in GVHD include the skin, gastroimesrinaltTact, and liver. Cli11icall>; GVHD manifesl5 as rash, dianhea, abdominal cramps, anemia, and liver dysfunction. None of the other cells mediates GVHD. ***Diagnosis: Graft-versus-host disease
21 A 20-year-old gardener presents to his family physician for treatment of what he describes as "poison ivy.·· The patients hands and anns appear red and are covered with oozing blisters and crusts. Which of the following best describes the pathogenesis of these skin lesions? ***(A) Cytotm.:ic antibody production ***(B) Delayed-type hypersensitivity ***(C) Deposition of amiglmen antibodies ***(D) Deposition of circulating immune complexes ***(E) IgE-mediated mast cell degranulation
21 The answer is B: Delayed-type hypersensitivity. "Poison ivy" is a type IV hypersensitivity reaction to plants of the RJut~ genus. This I -lymphocyte-mediated allergic contact dc1matitls presents as urticaria and bullous eruption. Blisters rupture and heal with CJusts, usually without scarring. Deposition of antigluten antibodies (choice C) occurs in patients with dennatitis herpettform.is. IgE-mediated mast cell degranulation (choice E) is pan of the response to poison ivy (hypersensitivity reactions overlap), but tllis immediate respon.r;e does not explain the pathogenesis of delayed hypersensitivity in this patient. ***Diagnosis: AJlergic contact dennatitis
22 A 9-month-old girl ·with a histmy of recurrem pulmona1y infections is found to have a congenital deficiency of adenosine deaminase, which is associated with a vinual absence of lymphocytes in her peripheral lymphoid organs. What is the appropriate diagnosis? ***(A) Bruton X-linked agammaglobulinemia ***(B) DiGeorge syndrome ***(C) Isolated lgA deficiency ***(D) Severe combined immunodeficiency ***(E) Wiskott-Aldrich syndrome
22 The answer is D: Severe combined immunodeficiency (SCID ). SOD is a group of disorders of I and B lymphocytes that are characterized by recunent viral, bacterial, fungal, and protozoal infecLions. Many infants with SCID have severely reduced volumes oflymphoid Lissue and an immature thymus that lacks lymphocytes. In some patienlS, lymphocytes fail to deve lop beyond p re-B cells and pre-T cells. About one half of these severely immunodeficient children lack adenosine deaminase (ADA) ADA deficiency causes the accumulation of intermediate products that a re toxic to lymphocytes. These children cannot survive beyond early infancy unless they are raised in a sterile environmem ("'bubble children"). None of the other chmces a re associated with ADA deficiency ***Diagnosis: Severe combined immunodeficiency
23 A 50-year-old man complains of fever, weight loss, abdominal pain, and bloody mine. Physical examination reveals red-purple discoloration or Lhe skin. Serologic findings are inconclusive, but a positive P-ANCA test suggests an aULoimmune disease. Biopsy oflesional skin discloses fibrinoid necrosis of a small muscular arrery (shown in the image). Which of Lhe following immune responses best explains the pathogenesis of inflammation and necrotizing vasculitis in this patiem? ***(A) Antibody-dependem cellular cywtm.:icity ***(B) Cytopathic autoantibodies ***(C) Delayed-type hypersensitivity ***(D) Immediate hypersensitivity ***(E) Immune complex disease
23 The answer is E: Immune complex disease. Immune comple..x (type Ill) hypersensitivity reactions cause vasculitis. Antigenantibody complexes are eith er formed in the circulation and deposited in tJ1e tissues or formed in situ. lmmune complexes induce a localized inflammatory response by fixing complement, which leads to Lhe recmitment of n eutrophils and monocytes_ The vasculitis in patients •Nith polyarteritis nodosa involves small 1.0 medium-sized muscular arteries. The diagnosis is usually made by b iopsy of the skin, muscle, pe1iphe ral nerves, or the most affected imemal organ (the kidney tn this case). The most prominent morphologic feamre of the affected anery is an a rea of fibrinoid necrosis (see photomicrograph). Oilier examples of type Ill hypersensitivity reactions include Henoch-Schonlein purpura (vascular lgA deposiLS) and vasculitis associated wiLh hepatitis C infection_ The otJ1er choices are uncommon mediarors of vasculitis in patients with polyaneriLis nodosa. ***Diagnosis: Polyaneritis nodosa
Z4 A neonate develops spastic contractions on the second postpartum day Laboratory studies show hypocalcemia MRI studies demonstrate aplasia of the thymus and parathyroid glands. 'vVhat is the appropriate diagnosis? ***(A) Adenosine deaminase deficiency ***(B) Common variable immunodeficiency ***(C) DiGeorge syndrome ***(D) Transient hypogammaglobulinemia of infancy ***(E) Wiskott-Aldri.ch syndrome
24 The answer is C: DiGeorge syndrome. DiGeorge syndrome is a chromosomal defect that results in developmemal anomalies of the bran chial (pharyngeal) pouches and organs that develop from these embryonic structures (thymus, parathyroids, and aonic arch). These children present with tetany caused by hypoparathyroidism and deficiency of cellular immunity TI1ey also have characteristic facial features ("angry took"). In th e absence of a thymus, T-cell maruration is imenupted at me pre-I stage. DiGeorge syndrome has been corrected by transplanting thymic tissue. None of the other choices are associated v.rixh thymic aplasia. ***Diagnosis: DiGeorge syndrome
25 A 50-year-old woman complains of imenniltem tingling and pain in the tips of her fingers. She also reports joi.m and muscle pain. Physical examination reveals lymphadenopathy. Laborawry studies show hypergammaglobulinemia. The antinuclear amibody test is positive, bm there is no evidence of antibodies against double-stranded DNA. Urinalysis ls normal. The patient responds wellw steroids. vVhich of the following is the most likely diagnosis? ***(A) Graves disease ***(B) MLxed connective tissue disease ***(C) Myaslhenia gravis ***(D) Sclerodenua ***(E) Sjogren syndrome
25 The answer is B: Mixed connective tissue disease (MCTD). MCm has features of other common autoimmune diseases (e.g. , SLE and sclerodem1a) but appears to be distinct. Patients typically have au loami bodies to ribonucleo proteins, but unlike SLE, they do not have antibodies to Sm amigen or double-stranded DNA. Some patients with MCTD develop symptoms of scleroderma or rheumatoid arthritis, suggesting that MCTD may be an intennediate stage in a gen etically determined progresswn. Whether MCTD rep resents a distinct entity or simply an overlap of symptoms in patients with otl1er types of collagen vascular diseases remains an open question. lmermiuem episodes of ischemia of tJ1e fingers , marked by pallor, paresthesias, and pain, are refened to as Raynaud phenomenon. None of the other choices feature this constellation of signs and symptoms. ***Diagnosis: MLxed connective tissue disease
26 A 25-year-old woman complains oflow-grade fever, fatigue, and persistem rash over her nose and upper chest. She also notes pain in her knees and elbows. A skin biopsy shows dermal innammati.on and granular deposits oflgG and C3 complement along the basement membrane at the epidermal/dermal junedon. Urinalysis reveals microscopic hematuria and proteinuria. The aminuclear antibody test is positive. The development of thromboembolic complications (e.g., deep venous thrombosis) in this patient is commonly associated v.rilh elevated serum levels .of antibodies to which of the follo\ving antigens? ***(A) ABO blood group antigens ***(B) Class U HLA molecules ***(C) Clotting facto rs ***(D) Fibrinolytic enzymes ***(E) Phospholipids
26 The answer is E: Phospholipids. One thi rd of patients \<Vith systemic lupus erythematosus (SLE) possess elevated concentrations of amiphospholipid antibodies. This phenomenon predisposes these patients to thromboembolic complications, including stroke, pulmonary embolism, deep venous thrombosis, and portal vein thrombosis. The clinical course of SLE is highly variable and typically exhibits exacerbations and remissions. Wllh the recognition of mild fo nns of the disease , improved amihypenensive medications, and the use of immunosuppressive agents, the overall 10-year survival rate approaches 90%. Antibodies against clotting factors (choice C) or fibrinolytic enzymes (choice D) are not involved in the clotting tendency associated with SLE. ***Diagnosis: Systemic lupus erythematosus
27 A 30-year-old woman is found to have a congenital immunodeficiency that has remained largely asymptomatic throughout her life. \Vhich of the following is the most llkely diagnosis? ***(A) Adenosine deaminase deficiency ***(B) Chronic mucocmaneous candidiasis ***(C) PUJine nucleoside phosphorylase deficiency ***(D) Selective IgA deficiency ***(E) vVL-;kott-Aldrich syndrome
27 The answer is D: Selective lgA deficiency. Selective lgA deficiency is the most common primaty immunodeficiency syndrome, with an incidence of 1:700 among Europeans. Although patients are often asympwmatic, they occasionally present with respiratory or gastrointestinal infections of varying severity. They also display a strong predilection for allergies and collagen vascular diseases. Patients with lgA deficiency have normal numbers of lgA·bearing B cells, and th eir varied defects result in an inability to symhesizc and secrete lgA subclasses. Patients with chronic mucocutaneous candidiasis (choice B) show an increased susceptibility to Cm1dida infections an.d also may exhibit various endocrine disorders (e.g., hypoparathyroidism and Addison disease). The other choices are associated with severe immunodeficiency ***Diagnosis: Selective lgA deficiency
28 A 60-year-old woman with type 2 diabetes and end-stage renal disease receives a kidney transplanl. Three weel<s later, the patient presents with azotemia and oliguria. lf this patient has developed acute renal fa ilure, which of the following pathologic find ings would be expected on renal biopsy? ***(A) Arterial intimal thickening and vascular stenosis ***(B) Glomerulosclerosis ***(C) Interstitial infiltrates of lymphocytes and macro phages ***(D) Neurrophilic vasculitis and fibrinoid necrosis ***(E) Tubular atrophy and i.mersLitial fibrosis
28 The answer is C: Interstitial infiltrates of lymphocytes and macrophages. Transplant rejection reactions have been traditionally categorized imo hyperacute, acute, and chronic rejection based on the clinical tempo of the response and on the mechanisms involved. Acme rejenion is characte1ized by an abrupt onset of azotemia and oliguria, which may be associated with fever and graft tenderness. A needle biopsy would be expected to show (l) interstitial infiltrates of lymphocytes :md macrophages, (2) edema, (3) lymphocytic rubulitis, and ( 4) tubular necrosis. Neutrophilic vasculitis and fibrinoid necrosis (choice D) are seen in hyperacute rejection. Arterial intimal thickening (choice A), glomerulosclerosis (choice B), and tubular atrophy (choice E) are seen in chronic graft rejection. ***Diagnosis: Acme graft rejection
29 A 12-year-old boy presents with a 5-day history of sore throaL. His temperature is 38. 7"C (103°F) Physical examination reveals in flamed tonsils and swollen cervical lymph nodes. Trafficking and recirc ulation of blood-borne lymphocytes through the cervical lymph nodes in this patient occurs prinunily at which of the fo llov.ling locations? ***(A) Afferent l)'mphatic vessel ***(B) EITerent lymphatic vessel ***(C) Hassan corpuscles ***(D) High endothelial venules ***(E) Peyer patches
29 The answer is D: High endothelial venules (HEVs) . B and T lyrnphocytes circulate via Lhe vascular system to secondary lymphoid organs and tissues. Included among these tissues are lymph nodes, mucosa-associated lymphoid tissues, and spleen. ln the case or lymph nodes, lymphocyte trafficklng occurs through specialized postcapiUary venules termed high endothelial venules (HEVs). HEVs express an array of specific cell .adhesion molecules (e.g., CD3l) that allow lymphocyte binding and diapedesis. The cuboidal shape of HEY cells reduces now-mediated shear forces and specialized imercellular connections facilitate egress of lymphocytes out of the vascular space. Afferent and effe rent lymphatic channels (choices A and B) do not possess HEVs. Hassall corpuscles (choice C) are found in the meduUa of the Lhymus. Peyer patches (choice E) are organized lymphoid tissues found in the small imestine. ***Diagnosis: Lymphadenopathy, streptococcal pharyngiLis
30 A 28-year-old woman with a history of drug abuse presents with an infectious mononucleosis-like syndrome and lymphadenopathy. Blood tests subsequently indicate that she is HN-positive. Which of the following lymphocyte-associated proteins mediates the entry of HlV imo host cells in this patient? ***(A) CD4 ***(B) CD8 ***(C) GP4l ***(D) GP120 ***(E) LFA-1
30 The answer is A: CD4. The HIV-1 genome consists of two identical 9. 7 -kb single su·ands of RNA enclosed within a core of viral proteins. The core i.s enveloped by a phospholipid bilayer derived from the host cell membrane, in which are found vi.rally encoded glycoproteins (gpl20 and gp41). In addition to the gag. pol. and env genes--characteristic of all replication-competent RNA viruses-HIV-1 contains six Olher genes that code for proteins involved in replication . The specific target cells for HJV-1 are CD4+ helper T lymphocytes and mononuclear phagocytes, although infection of other cells occurs. The HIV envelope glycoprotein gpl20 (either on the free virus or on the surface of an infected cell) binds CD4 on the surface of helper T lymphocytes. The binding of gpl20 to CD4 allows gp4lto insert into the cell membrane of the lymphocyte, thereby promoting fusion of the viral envelope with the lymphocyLe. En try of HIV-1 in to a target cell in vivo also requires viral binding to a coreceptor, ~-chemokine receptor 5 (CCR-5). Choices C and D (gp4l and gp 120) are involved in viral replication, but they are present on the viral envelope. Choice E {l fA-1) i5 a member of the leukocyte imegrin family that is involved m cell-cell adhesion. ***Diagnosis: Acquired immunodeficiency
1 A 25-year-old man presems 1 week after discovering that his left testicle is twice the normal size. Physical examination reveals a nomender, testicular mass that cannot be transilluminated. Serum levels of alpha-fetoprotein and human chorionic gonadotropin are normaL A hemiorchiectomy is perfonned, and histologic examination of the surgical specimen shows embryonal carcinoma. Compared to normal adult somatic cells, this genn cell neoplasm would most likely show high levels of expression of which of the following proteins? ***(A) Desmin ***(B) Dystrophin ***(C) Cytochrome c ***(D) P sel eC£in ***(E) Telomerase
....._1_ The answer is E: Telomerase. Somatic cells do not normally express telomerase, which is an ~nzyme that adds repetitive sequences w maintain the length of the telomere. Thus, \Vith each round of somatic cell replication, the telomere shortens. The length of relomeres may act as a "molecular clock" and govern the lifespan of replicating cells. Because cancer cells and embryonic cells ex1)ress high levels of telomerase, the reactivation of this enzyme may be imponam for maintaining stem cell proliferation. Most human cancers show activation of the gene for the catalytic subunit or telomerase: huma11 telomerase reverse tTanscriptase. P selectin {choice D) is a cell adhesion molecule lhat mediates the margination of neutrophils during acute inflammation. The other choices are not involved in malignant transformation. ***Diagnosis: Embryonal carcinoma
2 A 25-year-old woman presents for a gynecologic examination. The cervical Pap smear shows "kollocytic atypia" characterized by perinuclear halos and w1inkled nuclei (shown in the image). A cervical biopsy reveals invasive squamous cell carcinoma. Molecular tests for human papil1omavirus (HPV) in the tumor cells are positive. Which of the follo·wing mechanjsms of disease best explains the role ofHPV in the pathogen· esis of neoplasia in this patient? ***(A) Activation of cellu lar oncogenes ***(B) Enhanced transcription of relomerase gene ***(C) Episomal viral replicatiorr ***(D) Inactivation of tumor suppressor proteins ***(E) Insertional mmagenesis
2 The answer is D: Inactivation of tumor suppressor pro· t.eins. Unlike RNA Lmnor viruses, whose oncogenes have normal ce llular counterparts, rhe transfo rming genes of DNA viruses are nm homologoU5 wirh any cellular genes. This conundrum was resolved wilh the discovery that the gene products of oncogenic DNA viruses inactivate rumor suppressor proteins. For example, prO[eins encoded by the E6 and E7 genes of HPV16 bind p53 and pRb. The other choices are involved in Lhe pathogenesis of neoplasia, bm they are not specific for HPV. ***Diagnosis: Cervical intraeptlhelial neoplasia, HPV infection
3 The patiem desctibed in Question 2 undergoes a hysterecwmy. ln addition lOa focus or invasive carcb1oma, the pathologisL idemtfies dysplastic squamous ce lls occupying the emire thickness of the cervical epithelium, with no evidence of epithelial maturation. The basal membrane in these areas appears imact. Which of the following tem1s best describes this cerncal lesion? ***(A) Alypical hype rplasia ***(B) Carcinoma in siru ***(C) Carcinomatosis ***(D) Complex hyperplasia ***(E) Koilocylic atypia
3 The answer is B: Carcinoma in situ. Most carcinomas begin as localized growths confined to rhe epithelium in which rhey arise. As long as these early cancers do not penetrate the basemem membrane on which the epithellum rests, such tumors are labeled carcinoma in situ. When the in situ tumor acquires invasive potential and extends directly through the underlying basement membrane, it is in a position to compromise neighboring tissues and metastasize. Carcinomatosis (choice C) is a clinical term used to desc1ibe widespread dissemination of cancer. Koilocytosis (choice E) implies the presence of squamous cells with pe1inuc lear halos and nuclear changes. lt is indicative of human papillomavims infection and canies an increased risk of carcinoma. Atypical and complex hyperplasia (choices A and D) refer to proliferative lesions of the glands within the uterine endomeuium. ***Diagnosis: Cervical carcinoma, carcinoma in situ
4 A 62-year-old woman presents with a breast lump that she discovered 6 days ago. A breast biopsy shows lobular carcinoma in situ. Compared to normal epithelial cells of the breast lobule, these malignant cells 'vould most likely show decreased expression of which of the follov.ring proteins? ***(A) Desmin ***(B) E-cadherin ***(C) Lysyl hydroxylase ***(D) P sel ectin ***(E) Telomerase
4 The answer is B: E-cadherin. Cadherlns are Ca2+-dependem transmembrane glycoproteins that mediated cell-cell adhesion. E-cadherin is expressed on the surface of all epithelia and mediates ceU adhesion by "zipper-like" interactions. Overall, cadherins suppress invasion and metastasis. Thus, it is perhaps not surprising that the expression of E-cadherin is reduced in most carcinomas. Desmin (choice A) is an intermediate fi lament protein found in cells of mesenchymal origin. Lysyl hydroxylase (choice C) is involved in the posttranslational modifLcation of collagen. P selectin is a cell adhesion molecule that mediates the margination of neutrophils dming acute inflammation. Telomerase (choice E) is increased in certain malignancies ***Diagnosis: Breast cancer
5 An 80-year-olcl man complains of lower abdominal pain, increasing weakness, and fatigue. He has lost 16 lb (7 3 kg) in the past 6 momhs. The prostate-specLfic antigen test is elevated (8.5 nglml). Rectal examination re,·ea ls an enlarged and nodular prostate. A needle biopsy of the prostate disci oses invasive prostatic adenocarcinoma. Histologic grading of this parienl's carcinoma is based primarily on which of the following Clitetia' ***(A) Capsular involvement ***(B) E.xtem of regional lymph nodes involvemem ***(C) Pulmonary metastases ***(D) Resemblance to nom1altissue of origin ***(E) Volume of prosta£e involved by mmor
5 The answer is D: Resemblance to normal tissue of origin. To esmblish cmeria for Lherapy, many cancers are classified according lO hiswlogic gradmg schemes or by staging protocols Lhat dese1ibe the extent of spread. Cancer grading reflects cellular characteristics. Low-grade tumors are well differentiated, whereas high-grade rumors lack differendated features (anaplasia). The general correlation between cywlogic grade and the behavior of a neoplasm is not invariable. Ind eed, there are many examples of rumors of low cytologic grades that exhibit substantial malignant properties. The other chokes pertain to cancer staging. ***Diagnosis: Prostate cancer
& A 50-year-old woman presents with a lump in her breast. A 4-cm firm and fixed mass is noted on breast examination. Excisional biopsy reveals malignant cells lhat fonn glandlike stmcmres and solid nests, surrounded by a dense collagenous stroma. A connective tissue stain (trichrome) of the biopsy is shown in the image. Which of the following descriptive terms best describes the blue areas observed in this specimen? ***(A) Colloid carcinoma ***(B) Comedocarcinoma ***(C) Desmoplastic change ***(D) Medullary carcinoma ***(E) Papillomatosis
6 The answer is C: Desmoplastic change. Seconda1y descr iptors are used to refer to a tumors morphologic and functional characteristics. Papillomatosis (ch oice E) describes frond-like structures. Medullary (choice D) signifies a soft cellu1ar tumor, whereas scirrhous or desmoplastic implies dense fibrous stroma. Colloid carcinomas (choice A) secrete abundant mucus. Comedocarcinoma (choice B) is an intraduCLal neoplasm in which necrotic material can be e>.'J)ressed from the ducts. ***Diagnosis: Breast cancer
7 A 65-year-old man complains of muscle weakness and a dry cough for 4 months. He has smoked two packs of cigarettes daily for 45 years. A chest X-ray shows a 4-cm cemral, left lung mass. Laboratory srudies reveal hyperglycemia and hypertension. A transbronchial biopsy is diagnosed as sma!J cell carcinoma. Metastases w the liver are detected by CT scan. 'vVhich of the following might account for the development of hyperglycemia and hypertension in this patient? ***(A) Adrenal metastases ***(B) Paraneoplasdc syndrome ***(C) Piruitary adenoma ***(D) Pi tuitary metastases ***(E) Thrombosis of the renal anery
7 The answer is 8: Paraneoplastic syndrome. Cancers may pro- ,__ duce remote e!Tects, collectively termed paraneoplastic syndromes. For example, rhe secrnion of corticotropin (ACTH) by a mmor leads to clinical featu res of Cushing syndrome, including hyperglycemia and hypenension. ConicOLropin production is most commonly seen \Vith cancers of the lung, particularly small cell carcinoma. Adrenal and piruitary metastases (choices A and D) would lead to loss of adrenal function (Addison disease). Although pituitary adenoma (choice C) is a possible cause of Cushing syndrome, this choice would be unlikely in a patiem with lung cancer. ***Diagnosis: Small ceO carcinoma of lung, paraneoplastic syndrome
8 A 60-year-old man presents with a 4-momh history of increasing weight loss, wheezing, and shormess of breath. He has smoked two packs of cigarettes a day for 40 years. His past medical history is significant for emphysema and chronic bronchitis. A chest X-ray shows a 10-cm mass in the left lung. Bronchoscopy discloses obstruction of the left main stem bronchus. A biopsy is obtained (shown in the image). lmmunohistochemkal studies of Ll1is biopsy specimen would most likely show strong expression of which of the following tumor markers? ***(A) Alpha-fetoprotein ***(B) Calretinin ***(C) Carcinoembryonic antigen ***(D) Cytokeratins ***(E) Synapwphysin
8 The answer is D: Cytokeratins. Tumor markers are products of malignant neoplasms that can be detected in cells or body fh.lids. Useful rumor markers include immunoglobulins, fetal proteins, enzymes, hormones, and cytoskeletal proteins. Car· cinomas uniformly express cytokeratins, which are intermedjate filaments. Alpha-fetoprotein (choice A) is a marker for yolk sac carcinoma and hepalocellular carcinoma. Calretinin (choice B) provides a marker for mesothelioma. Carcinoembryonic antigen (choice C) is a marker for colon carcinoma and many other malignancies. Synaptophysin (choice E) is a marker for neuroendocrine tumors, including small cell carcinoma of the lung. ***Diagnosis: Squamous cell carcinoma of lung
9 vVl1ich of the fo llowing potent carcinogens was most likely involved in the pathogenesis of lung cancer in the patient desCJibed in Question 8? ***(A) Aflawxin B1 ***(B) Asbestos ***(C) Azo dyes ***(D) Polycyclic aromatic hydrocarbons ***(E) Vinyl chloride
9 The answer is D: Polycyclic aromatic hydrocarbons. Polycyclic aromatic hydrocarbons, originally derived from coal tar, are among Lhe most extensively studied carcinogens. These compounds produce cancers at tl1e site of application. Since polycyclic hydrocarbons have been identified in cigareue smoke, it has been suggested (bm not proved) that they are involved in the pathogenesis of lung cancer. Aflatoxin B1 (choice A), a natural product of the fungus Aspergillus flavus, is among the most potent liver carcinogens. Asbestos (choice B), a mineral , is associated with mesothelioma and adenocarcinoma of lung. Industrial workers exposed to high levels of vinyl chloride (choice E) in the ambient atmosphere developed angiosarcomas of the liver. ***Diagnosis: Squamous cell carcinoma of lung
10 A 33-year-old woman discovers a lump in her left breast on self-examination. Her mother and sister bOlh had breast cancer. A mammogram demonstrates an ill-defined density in the omer quad rant of th e left breast, \Vith microcalcifications. Needle aspiration reveals the presence of malignant, ductal epithelial cells. Genetic screening identifies a mutation in BRCAl. In addition to cell cycle control, BRCAl protein p romotes which of the following cellular functions? ***(A) Apoptosis ***(B) Cell adhesion (C} DNA repair ***(D) Gene transcription ***(E) Transmembrane signaling
10 The answer is C: DNA repair. Breast (BR) cancer (CA) suscepLibility genes (BRCAl and BRCA2) encode tumor suppressor proteins involved in checkpoint functions related to progression of the cell cycle into S phase. BRCAl and BRCA2 proteins also promote DNA repair by binding to RAD51, a molecule that mediates DNA double-strand repajr breaks. The other choices may be abnormal in neoplasia, bm they are not primarily affected by BRCAl. ***Diagnosis: Breast cancer
11 A 60-year-old man who worked for 30 years in a chemical facto ry complains of blood in his urine. Urine cytology discloses dysplastic cells. A bladder biopsy demonstrates transitional ceU carcinoma. Which of the fo llowing carcinogens was most likely involved in the pathogenesis of bladder cancer in this patiem? ***(A) Aniline dyes ***(B) Arsenic ***(C) Benzene ***(D) Ci.splatinum ***(E) Vinyl ch loride
11 The answer is A: Aniline dyes. Transitional cell carcinoma is the most common malignant tumor of the urinary bladder, and the incidence of bladder cancer is increased in aniline dye workeTs. These azo dyes are converted to water-soluble carcinogens in the liver. They are excreted in the urine, where they pnmarily affect the transitional epithelium of the bladder. Benzene exposure (choice C) is associated with leukemia. Vinyl chloride exposure (choice E) has been associated with hepatic angiosarcomas. ***Diagnosis: Transitional celJ carcinoma of bladder
12A 60-year-old man presems with an ulcerated, encrusted, and tnfilnating lesion on the sun-exposed dorsal aspect of a finger (shown in the image). A biopsy reveals squamous cell carcinoma. The metast:acic potential of this neoplasm would be enhanced by upregulation of the gene for which of the following proteins? ***(A) Collagen type N ***(B) Desmin ***(C) E-cadherin ***(D) Glmathione peroxidase ***(E) Plasminogen activator
12 The answer is E: Plasminogen activator. Malignam cells and stromal cells associated with cancers elaborate a variety of proteases that degrade basemem membrane components. Such enzymes include the urokinase-type plasminogen activator (u-PA) and matrix metalloproreinases. u-PA converts serum plasminogen to plasmin, a serine p rotease that degrades laminin and activates type IV procotlagenase. Changes in the expression of u-PA, the u-PA receptor, and PA inhibi tors have been reported in differenl cancers. Metastatic cells would be e>q)ected to show reduced expression of collagens (choice A) and cadherins (choice C). Desmin (choice B) is found in cells of mesenchymal origin. ***Diagnosis: Squamous cell carcinoma of skin
13 A 45-year-old man presems wi th a 9-momh history of a reddish nodule on his foot. Biopsy of the nodule d1scloses a poorly demarcated lesion composed of fibroblasts and endothelial-Like cells lining vascular spaces. Funher work-up identifies similar lesions in the lymph nodes and liver. The tumor cells comain sequences of human h erpesvirus-8 (HHV-8). This patiem most likely has which of the following diseases? ***(A) Acquired inununodeficiency ***(B) Ataxia telangiectasia ***(C) Li-f raumeni syndrome ***(D) Neuronbromatosis t)•pe l ***(E) Xerodenna pigmemosum
13 The answer is A: Acquired immunodeficiency. Kaposi sarcoma is the mosL conunon neoplasm associaLed with acquired immunodeficiency syndrome (AID$). The neoplastic celJs contain sequences of a novel virus, HHV-8, which is also known as Kaposi sarcoma-associated herpesvirus. ln addition w infecting the spindle cells of Kaposi sarcoma, HHV-8 is lymphotropic and has been implicaLed in two uncommon B-cell lymphoid malignancies, namely, primary effusion lymphoma and mullicentric Castleman disease. Like other DNA viruses, Lhe HHY-8 genome encodes proteins thm imerfere wiLh the p53 and pRb tumor suppressor pathways. The oth er choices are hereditary conditions associated with can cer; however, these parient.c; do not Lypically acquire Kaposi sarcoma. The predominant malignancy seen in patiems with ataxia telangiectasia (choice B) is lymphoma/ leukemia. ***Diagnosis: Kaposi sarcoma, AIDS
14 Dming a routine checkup, a 50-year-old man is found to have blood in h ls urine. He is otherwise in excellent health. An abdominal CT scan reveals a 2-cm right renal mass. You inform the patient that staging of this tumor is key to selecting treatment and evaluating prognosis. Which of the following ls the most imponam staging factor for this patient? ***(A) Histologic grade of the rumor ***(B) Metastases to regional lymph nodes ***(C) Proliferative capacity of the tumor cells ***(D) Somatic mutations in the p53 tumor suppressor gene ***(E) Tumor cell karyotype (aneuploidy)
14 The answer is B: Metastases to regional lymph nodes. The choice of surgical approach or treatment modabties is influenced more by the stage of a cancer than by its cytologic grade. The significant criteria used for sLaging vary with differem organs. Commonly used criteria include (l) tumor size, (2) extent of local growth, (3) presence of lymph node metastases, and (4) presence of distam metastases. The mher choices reflect grade of the tumor. ***Diagnosis: Renal cell carcinoma
15 A 68-year-old man who has worked in a shipyard and manufacturing plam all his adult life con1plains of a 4-momh history of chest discomfort, malaise, fever, night sweats, and weight loss. A ch est X-ray reveals a large pteural eiTc1sion. The patiem dies 5 months later of cardiorespiralory failure. The lung at auwpsy is shown in the image. This malignant neoplasm is associaLed ·wilh environmemal e>q)osure Lo which o[ the following carcinogens? ***(A) Mlawxin B 1 ***(B) Asbesws ***(C) Beryllium ***(D) Ionizing radiation ***(E) Silica
15 The answe-r is B: Asbestos. The characteristic tumor associated \vith asbestos e>q)osure is mesothelioma of the pleural and petiwneal cavities. This cancer has been reponed to occur in 2% to 3% of heavily exposed workers. The pipe fitters ln shipyards were the most exposed workers. Many of these workers developed mesotheliomas 20 to 40 years after e}..rposure. lt is reasonable to sum1ise that mesotheliomas of both the pleura and the peritoneum reflect the close contact of these membranes with asbestos fibe rs transponed w them by l)mphatic channels. Like the polycyclic aromatic hydrocarbons, aflawxin B1 (choice A) can bind covalemly w DNA and is among the most potem liver carcinogens recognized. Betyllium (choice C) and silica (choice E) cause lung disease, bU[ they are not carcinogenic. ***Diagnosis: MesotheHoma
16 A 58-year-old woman with colon cancer presents with 3 months of increasing shortness of breath. A chest X-ray reveals numerous, bilateml, round masses in both lungs. Histologic examination of an open-lung biopsy discloses malignant gland-like strucrures, which are near!)' identical to the colon primary. Which of the following changes in cell behavior was the first step in the process leading to tumor metastasis fTom the colon to the lung in this patient? ***(A) AlTest within the circulating blood or lymph ***(B) Exit from the circulation into a new tissue ***(C) Invasion of the underlying basement membrane ***(D) Penetration of vascular or lymphatic channels ***(E) Stimulation of angiogenesis ·within the pulmonary metastases
16 The answer is C: Invasion of the underlying basement membrane. The 6rsr event in tumor cell invasion is breach of the basement membrane that separates an epithelium from the underlying mesenchyme. Afu:r invading the imerstidal tissue, malignant cells penen·are lymphatlc or vascular channels (choice D). ln the lymph nodes, communications between the lymphatics and venous tributaries allow malignant cells access to the systemic circulation. The other choices are imponam f OT tumor metastases, but the}' OCCur JateT than basement membrane invasion. ***Diagnosis: Adenocarcinoma of colon
17 A 68-year-old man complains of recem changes in bowel habits and blood-tinged stools. Colonoscopy reveals a 3-cm mass in the sigmoid colon. Biopsy of the mass shows infiltrating malignant glands. These neoplasdc cells have most likely acquired a set of mutations that cause whkh of the following changes in cell behavior? ***(A) Decreased cellular motility ***(B) Enhanced stem cell differentiation ***(C) Increased cell-cell adhesion ***(D) lncreased susceptibility to apoptosis ***(E) Loss of cell cycle resuiction point control
17 The answer is E: Loss of cell cycle restriction point con· trot. Cancer ce lls often d isplay loss of cell cycle restriction point control through mechanisms such as overe>..'Pression of cyclin Dl, loss of Cdk inhibitors, or inactivation of the pRb or p53 proteins. The p53 gene is deleted or mutated in 75% of cases of colorectal cancer and frequently mutated in numerous other tumors. The p53 protein is a negative regulator of cell division. Inactivating mutations of p53 cause loss of ceU cycle restriction point control and allow cells ,:vilh damaged DNA to progress through the cell cycle. Malignant cells have increased cellular motility (see choice A), reduced stem cell differenriation (see choice B), decreased cell adhesion (see choice C), and decreased susceptibility to apoptosis (see choice D). ***Diagnosis: Adenocarcinoma of colon
18 A35-year-old woman complains of nipple disd1arge and iiTegular menses of 5 months duration. Physical eo'Xamination reveals a milky discharge from both nipples. MRl shows an enlargement of the anterior pituiLalY Which of the following is the most likely hismlogic diagnosis of Lhis patient's pimi Lary tumor? ***(A) Adenoma ***(B) Choristoma ***(C) Hamartoma ***(D) Papilloma ***(E) Teramma
18 The answer is A: Adenoma. Benign tumors mising from a glandular epithelium are termed adenomas. Patients with a prolactin-secreting pituitary adenoma present with amenorrhea and galactonhea. Ectopic islands of nom1al tissue are called choristomas (choke B). Localized, disordered differentiation during development results in a hamartoma (choice C). Papillomas (choice D) do not occur in the pituitary Benign tumors that arise from germ cells and contain all three germ layers are termed teratomas (choice E). ***Diagnosis: Pituitary adenoma, prolactinoma
19 A 52-year-old woman presents with a 1-year histOt)' of upper truncal obesity and moderate depression. Physical examination shows hirsutism and moon facies. A CT scan of the thorax displays a hJiar mass. A transbronchial lung biopsy discloses small ceO carcinoma. Electron microscopy of this patiems lung tumor will most likely reveal which of the following cytologic features? ***(A) Councilman bodies ***(B) Hyperplasia of endoplasmic reticulum ***(C) Miwchondrial calcification ***(D) Myelin figures in lysosomes ***(E) Neuroendoc1ine granules
19 The answer is E: Neuroendocrine granules. Neuroendocrine tumors may synthesize a number of hormones. The presence of small, membrane-bound granules with a dense core is a feature of these neoplasms. Dense granules are visible by electron microscopy. ln this way; electron microscopy may aid in the diagnosis of poorly differentiated cancers, whose classification is problematic by light microscopy. Carcinomas often exhibit desmosomes and specialized junctional complexes, which are structures that are not typical of sarcomas or lymphomas. Myelin figures (choice D) are seen in patients with inherited lysosomal swrage disease. Councilman bodies {choice A) arc apoptotic hepatocytes (acidophilic bodies). ***Diagnosis: SmalJ cell carcinoma of lung, paraneoplastic syndrome
20 Cytogenetic s tudies in a 40-year-old woman with follicular lymphoma clemonslrate a t(l4;18) chromosomal translocation involving the bd-2 gene. Consmurive expression of the protein encoded by the bcl-2 gene inhibits which of the follo\ving processes in this patiem5 transformed lymphocytes? ***(A) Apoptosis ***(B) DNA excision repair ***(C) Gl-to-5 cell cycle p rogression ***(D) Oxidative phosphorylaLion ***(E) Protein (N-linked) glycosylation
20 The answer is A: Apoptosis. Many human cancers show abnormalities in the comrol of apoptosis. For example, follicular B-ceU lymphomas display a charaCLeristic chromosomal translocation in wh.ich the bcl-2 gene is brought under the transctiptional control of the immunoglobulin light-cl1ain gene prommer, thereby causing overexpression of bcl-2. As a result of Lhe antiapopwtic properties of bd-2, the neoplastic clone accumulates in lymph nodes. Since its demonsu·arion in follicular tymphomas, bc1-2 expression has been observed in a variety of other human cancers. None of the other choices describes the fcmction of bcl-2. ***Diagnosis: Follicular ly"D1phoma
21 A 60-year-old man presems with a 6-month history ofincreasing weight loss and fatigue. Physical examination reveals conspicuous hepatomegaly. An abdominal CT scan reveals multiple canon baiJ nodules in the liver (shown in the image). A CTguided biopsy revea1s a mucous-secreting adenocarcinoma. This patients metastatic liver cancer most hkely originated in which of the following anatomic locations? ***(A) Adrenal medulla ***(B) Bone marrow ***(C) Brain ***(D) Pancreas ***(E) Urinary bladder
21 The answer is D: Pancreas. Radiologic evidence of "canon ball" lesions in the liver or lung suggests metastatic cancer. The liver is involved in a thi rd of all metastatic cancers, including halr of those of the gastrointestinal tract, breast, and lung. Other tumors that characteristically metastasize to the liver are pancreatic carcinoma and malignant melanoma. Liver metastases are the most common cause of massive hepatomegaly. Vlsible secre tions of tumor cells, such as mucin or serous Quid, provide important clues for tumor diagnosis. Mucin-secreting glandular epithelium and mucin-secreting adenocarcinoma are e::\.-pected in the pancreas. None of the OLh.er organs are composed of glandular epithelial cells or produce mucin. ***Diagnosis: Metastatic cancer
22 A 59-year-old woman presents with increasing pigmemation of the skin. PhysicaJ examination shows hyperkeratosis and hyperpigmemation of the axilla, neck. flexures, and anogenital region. Endocrinologic studies reveal nonnal semm levels of adrenal conkosteroids and glucoconicoids. If this patients skin pigmemation represents a paraneoplastic syndrome, the primary tumor would most likely be found in which of the following anatomic locations? ***(A) Bladder ***(B) Ce rvi..~ ***(C) Esophagus ***(D) Pleura ***(E) Stomach
22 The answe.r is E: Stomach. Acamhosis mgncans is a cutaneous disorder marked by hyperkeratosis and pigmentation of the axilla, neck, flexures, and anogenital region. lt is of particular imerest because more than half of patients with acanthosis nigricans have cancer. Over 90% of cases occur in association with gastrointestinal carcinomas (primarily stomach cancer). The other mmors are uncommon causes of acanth osis nigricans. ***Diagnosis: Paraneoplastic syndrome, acanthosis nigricans
23 A 65-year-old man dies after a protracted baule with metastatic colon carcinoma. At autopsy, the liver is filled 'Nith multiple nodules of cancer, many of which display central necrosis (umbilication). Which of the following best ex:plains the pathogenesis of rumor umbilication in this patient? ***(A) Biphasic tumor ***(B) Chronic inflammation ***(C) Granulomatous inflammation ***(D) Ischemia and infarcdo11 ***(E) Snmulauon of angiogenesis
23 The answer is D: Ischemia and infarction. Angiogenesis is a requirement for the continued growth of cancers, whether plimary or metastatlc. ln the absence of new vessels to supply the nutrients and remove waste products, malignant tumors do not grow larger than llo 2 mm in diameter. ln general, causes of tumor cell death in situ include (l) programmed cell death (apoptosis); (2.) inadequate blood supply, >vith consequent ischemi.a; (3) a paucity o[ nutrients; and (4) vulnerability to specific and nonspecific host defenses. The CT scan provided [or Question 2.1 shows central necrosis (umbtlicatlon) in most of the metastatic rumor nodules. None of the other choices are likely causes of tumor necrosis. ***Diagnosis: Metastatic cancer
24 A 59-year-old man complains of progressive weakness. He reports that his stools are very dark. Physical examination demonstrates fullness in the right lower quadrant. Labo ratory studies show iron deficiency anemia, with a serum hemoglobin level of 7.4 gldl. Stool specimens are positive for occult blood. Colonoscopy discloses an ulcerating lesion of the cecum. Which of the following serum mmor tnarkers is most likely to be useful for following this patient after surgery? ***(A) Alpha-fetopmtein ***(B) Carcinoembryonic antigen (C} Chorlonic gonadotropin ***(D) Chromogranin ***(E) Coagula non factOr VIII
24 The answer is 8: Carcinoemhryonic antigen {CEA) . Colorectal cancer is asymptomatic in its initial stages. As the mmor grows, the most common sign is occult blood in feces, especially when the tumor is in the proximal ponion of the colon. Chronic, asymptomatic bleeding typically cau ses iron-deficiency anemia. Adenocarcinomas of the colon usually e":press CEA, a glycoprotein that is released mto the circulation and serves as a serologic marker for these tumors. CEA is also found in association with malignant rumors of the pancreas, lung, and ova1y AFP (choice A) is expressed by hepatocellular carcmoma and yolk sac tumors. Chromogranin (choice D) is e."<pressed by neuroendoaine rumors. Chmionic gonadmropin (choice C) is secreted by chmiocarcinoma. ***Diagnosis: Colon cancer
25 l aboratory studies of Lhe surgical specimen obtained from the patient described in Question 24 demonstrate hypem1ethylation of the p53 gene. Which of the following best charactelizes this biochemical change in Lhe neoplastic cells? ***(A) Epigenetic modification ***(B) Gene amplificaLion ***(C) Insertional mutagenesis ***(D) Nonreciprocal translocation ***(E) Protooncogene mutation
25 The answer is A: Epigenetic modification. Hypermethylation of many rumor suppressor and DNA repair genes has been demonso·ared in human rumors. The pa1:hways controlled by these genes are, therefore, suppressed. For example, the normal p53 gene can be inactivated by hypennethylation. Thus, aberrant methylation of tumor suppressor genes may be an epigenetic mechanism for a "second hit," leading to loss of he1:erozygosi1:y. Unlike genetic ch<mges in cancer, ep1genetic changes are reversible, and a search for drugs that influence DNA methylation is under way. The other choices are unrelal: cd to DNA methylation. ***Diagnosis: Colon cancer
26 A 20-year-old woman has an ovarian tumor removed. The surgical specimen is 10 em in diameter and cystic. The cystic cavity is found to contain black h air and sebaceous material. Histologic examination of the cyst wall reveals a variety of benign differentiated tissues, including skin, cartilage. brain, and mucinous glandular epithelium. Wh at is the diagnosis? ***(A) Adenoma ***(B) Chondroma ***(C) Hamartoma ***(D) Teratocarcinoma ***(E) Teratoma
26 The answer is E: Teratoma. Teratomas are benign tumors composed of tissues derived from all three primary gem1 layers: ectoderm, mesoderm, and endodem1. They are most common in the ovary but also occur in the testis and extragonadal sites. Teratocartlnomas (choice D) are malignant tumors that harbor emb ryonal carcinoma stem cells. Adenoma (choice A) is a benign tumor of epithelial origin. Chondroma (choice B) is a benign cartilaginous tumor. Hamartoma (choice C) is disorganjzed normal tissue. ***Diagnosis: Mature teratoma
27 A +2-year-old man preseniS with upper gastroimestinal bleeding. Upper endoscopy and biopsy reveal gastric adenocarcinoma. Which country of the world has the highest incidence of this malignant neoplasm' ***(A) Argentina ***(B) Canada ***(C) Japan ***(D) Mexico ***(E) United States
27 The answer is C: Japan. The highest incidence of stomad1 cancer occurs in Japan, where the disease is almost ten times as frequent as it is among American whites. A study of Japanese residents o [ Hawaii found that emigrants from Japanese regions with the highest risk of stomach cancer continued to exhibit an excess risk in Hawaii. By contrast , their offspring who were born in Hawaii had the same incidence of this cancer as American whites. The highest incidence of colorectaJ cancer is found in the United States (choice E). ***Diagnosis: Gastric cancer
28 An 8-year-old girl with numerous hypopigmcnted, ulcerated, and crusted patches on her face and forearms develops an indurated, crater-like, skin nodule on the back of her left hand. Biopsy of this skin nodule discloses a squamous cell carcinoma. Molecular biology sn1dies reveal that this patient has gennline mutations in the gene encoding a nucleotide excision repair enzyme. What is the appropriate diagnosis? ***(A) Ataxia relangieClasia ***(B) Hereditary albinism ***(C) Li -Fraumeni syndrome ***(D) Neurofibromawsis. type 1 ***(E) Xerodenna pigmemosum
28 The answer is E: Xeroderma pigmentosum. Xeroderma pigmentosum is an aULosomal recessive disease in which increased sensitivity Lo sunlight is accompanied by a high incidence of skin cancers, including basal ceU carcinoma, squamous cell carcinoma, and malignant melanoma. Several xeroderma pigmemosum genes arc involved in nucleodde excision of ultraviolet-damaged DNA. Li-Fraumcni syndrome (choice C) refe rs to an inherited predisposition to develop cancers in many organs due to gerrnline mutations of p5.3 Ataxia telangiectasia (choice A) fe.arures cerebellar degeneration, immunologic abnormalities, and a predisposition to cancer. The mULated gene codes for a nuclear phosphoprotein involved in regulation of the cell cycle and DNA repair. Patients with hereditary albinism (choice B) are also at high risk for development of squamous cell carcinoma of the skin, but they do not have a defect in DNA excision repair. Patients with neurofibromatosis (choice D) develop benign cutaneous neu rofibromas. ***Diagnosis: Xeroderma pigmemosum
29 A 59-year-old woman complah1S of"feeling light-headed" and losing 5 kg (11 lb) in the last month. A CBC Teveals a normocytic, nom1ochromic anemia. The patient subsequently dies of metastatic cancer. Based on current epidemiologic data for cancer-assoctated monality in women. which of the following ts the most lD<ely primaty sire for this patiems malignant neoplasm? ***(A) Brain ***(B) Breast ***(C) Colon ***(D) Lung ***(E) Urinary bladder
29 The answer is D: Lung. lung carcinoma is Lhe caLLSe of most cancer-related deaths in the United States and \>\!estern Europ-e in men and women. The second most common cause of death from cancer in women is breast cancer (choice B) One of the most conm1on findings in patients with cancer is anemia, but the mechanism for this paraneoplastic syndrome is nor dear. The anemia is usually normocydc and normochromic, although iron deficiency anemia is common in cancers 1.hat bleed imo the gastrointestin al tract. ***Diagnosis: lung cancer
30 The parents of a 6-month-old girl palpate a mass on the left side of the childs abdomen. Urinalysis shows high levels of vanillylmandelic acid. A CT scan reveals an abdominal tumor and bony metastases The primary tumor is surgically resected. Histologic examination of the surgical specimen disdoses neuroblastoma. Evaluation of the N-myc prowoncogene in this childs tumor will most hkely demonstrate which of the following genetic changes? ***(A) Chromosomal translocation ***(B) Exon deletion ***(C) Expansion of a trinucleotide repeat ***(D) Frameshift mutation ***(E) Gene amplification
30 The answer is E: Gene amplification. Chromosomal alterations that result in an increased number of copies of a gene have been found p1imarily in solid tumors. Such aben adons are recognized as (l ) homogeneous sLaining regions (HSRs); (2) abnonnal banding regions on chromosomes; or {3) double minutes, which are visualized as small, paired cytoplasmic bodies. In some cases, gene amplification has been shown to involve prmooncogenes. f or example, HSRs may be seen in neuroblastomas and are all de1ived from the N-myc protooncogen e. The presence of N-myc H$Rs is associated with up to 700-fold ampli.ficanon of this gene and is a marker of advanced disease with a poor prognosis. Although Lhe other choices are mechanisms for protooncogene activation, they do nor cause up regulation of N -myc in patients with neuroblastoma. ***Diagnosis: Neuroblastoma
31 An 8-year-old African boy presents with swelling in his jaw and massive facial disfiguration. Biopsy reveals a tumor invading u1e bone marrow of the jaw. The pathogenesis of this malignant neoplasm is associated with a virus that exhibits a lTopism for which of the follo\ving cells? ***(A) Chondrocytes ***(B) Fibroblast5 ***(C) Lymphocytes ***(D) Macrophages ***(E) Osteocyt.es
31 The answer is C: Lymphocytes. Four DNA viruses Omman papillomavirus, Epstein-Barr vi rus I EBVI , hepatitis 13 virus, and herpesvirus-B) are incriminated in the development of human cancers. EBV was Lhe first v1ms to be uneguivocally lin ked to the development of a human tumor. ln 1958, Burkiu described a fonn of childhood lymphoma in a geogTaphicaJ belt across eguatorial Africa, which he suggesled might have a viral etiology A few years later, EpsLein and Barr discovered viral particles in cell lines cultured from patients wi th Burkitt lymphoma. African Burkitt lymphoma is a B-cell mmor, in \'.'hich the neoplastic lymphocytes invariably contain EBV in theiT DNA and manifest EBV-related antigens. EBV does not infect the other choices. ***Diagnosis: Burkitt lymphoma, EBV
32 A 58-year-old woman undergoes rou tine colonoscopy. A 2-cm submucosal nodule is idemified in the appendix. Biopsy of the nodule shows nests of cells with round, urn form nuclei. El ectron microscopy reveals numerous n euroendocrine granules in the cywplasm. This patients neoplastic disease is associated \vith which of the following clinical fea tures? ***(A) Congestive heart failure ***(B) Flushing and wheezing ***(C) Muscular dystrophy ***(D) Progressive systemic sclerosis ***(E) Pulmonary embolism
32 The an:swer is B: Flushing and wheezing. Carcinoid synd rome is a systemk paraneoplastic disease caused by the release of hom10nes fTom carcinoid tumors (via neuroendoctine granules) into venous blood. Sympwms of Oushing, bronchial wheez]ng, watery diarrhea, and abdominal colic are caused by the release of serotonin, b radykinin , and histamine. Carcin aids are neuroendocrine tumors of low malignancy that are most commonly located in the submucosa of the intestines (e.g., appendix, tem1inal ileum, and rectum). The other choices are not associated with this paraneoplastic syndrome. ***Diagnosis: Carcinoid tumor, paraneoplastic syndrome
33 A 55-year-old woman presents with increasing weight loss and fatigue and su bsequently dies of metastatic cancer. The vertebral column at autopsy is shown in the image. \.Vhat is the diagnosis? ***(A) Chondrosarcoma ***(B) Melanoma ***(C) Multiple myeloma ***(D) Qs[eosarcoma ***(E) Rhabdomyosarcoma
33 The answer is B: Melanoma. The photograph shows pigmented cells in the vertebral bodies of a person who died of malignant melanoma. This autopsy finding illusmues the point that accurate mmor identification depends on morphologk resemblance to nom1al tissue. Tumor emboli in this case probably reached bone after surviving passage through the pulmonary microcirculation. Non e of the other tumors show pigmentation. ***Diagnosis: Melanoma
34 A 45~year-old woman presents with abdominal pain and vaginal bleeding. A hysterectomy is performed and shows a benign tumor of the uterus derived from a smooth muscle celL What is the appropriate diagnosis' ***(A) Angiomyolipoma ***(B) Leiomyoma ***(C) Leiomyosarcoma ***(D) Myxoma ***(E) Rhabdomyoma
34 The answer is B: leiomyoma. Leiomyoma is the most common benign tumor of the uterus, usually arising in women of reproductive age. lt originates from smooth muscle cells of the myometrium. None of the other choices are benign tumors of smooth muscle. ***Diagnosis: Leiomyoma of uterus
35 Cytogenetic studies in a 70-year-old woman v•.rith chronic myelogenous leukemia {CML) demonstrate a L(9;22) chromosomal translocation. Which of the following best explains the role of this u·anslocation in the pathogenesis of leukemia in this patient? ***(A) Altered DNA methylation status ***(B) Enhanced expression of telomerase gene ***(C) Expansion of a trinucleotide repeat ***(D) Inactivation o[ mmor suppressor protein ***(E) Protooncogene activation
35 The answer is E: Protooncogene activation. The best-known example of an acquired chromosomal nanslocadon in a human cancer is the Philadelphia chromosome, which is found in 95% of patients with CML. The c-abl prowoncogene on chromosome 9 is translocated w chromosome 22, it is placed in juxtaposition w the breakpoint cluster region (ho} The c-ab[ gene and bcr region unite to produce a hybrid oncogene that codes for an aberrant protein \vi th very high levels of tyrosine kinase activity, which generates nliLOgenic and amiapopLOtic signals. ***Diagnosis: Chronic myelogenous leukemia, Philadelphia chromosome
36 A 33-year-old woman presents with a diff·use scaly skin rash of 4 weeks duration. Biopsy of lesional skin reveals a cutaneous T-cell lymphoma (mycosis fungoides). Which or t.he following immunohistochemical markers would be most useful for identifying malignant cells in the skin of this patient? ***(A) Calcitonin ***(B) CD4 ***(C) Desmin ***(D) HMB-45 ***(E) S-100
36 The answer is B: CD4. CD4 is a cluster-differentiation antigen of helper T lymphocytes. ffivtB-45 and S-100 (choices D and E) are markers for malignanr melanoma, among other mmors. Calcitonin (choice A) is a peptide hom1one. Desmin (choice C) is an intermediate filament protein found in cells of mesenchymal mi gin. ***Diagnosis: Mycosis fungoides
37 A 63-year-old woman ·with chronic bronchitis presents with shortness of breath. A chest X-ray reveals a 2-cm "coin lesion" in the upper lobe of the left lung. A CT-guided lung biopsy is obtained. Which of the followmg describes the histologic features of this lesion if the diagnosis is hamartoma? ***(A) Benign neoplasm of epithelial origin ***(B) Disorganized nom1al tissue ***(C) Ectopic islands of normal tissue ***(D) Granulation tissue ***(E) Granulomawus inflammation
37 The answer is 8: Disorganized normal tissue. Localized, disordered differentiation during embryonic development resul lS in a hamartoma, a disorganized caricature of normal tLc;sue components. Such tumors, which are not strictly neoplasms, contain varying combinations of cartilage, ducts or bronchi, cmmecrive tlSSue, blood vessels, and lymphoid tissue. Ectopic islands of normal tissue (choice C), called chorisroma, may also be mistaken for true neoplasms. These small lesions are represemed by pancreatic tissue in the wall of the stomach or intestine, adrenal rests under the renal capsule, and nodules of splenic tissue in the periwneal cavity ***Diagnosis: Hamartoma
38 A 67 -year-old woman presents with a massively swollen abdomen. The patient was diagnosed with papillary, serous cystadenocarcinoma of the ovary 3 years ago. She dies in a hospice 1 month later. At auwpsy, the pe1iwneum is studded wilh smallmmors (shown in the image), and there are 4 L of ascites. Which of the following routes of tumor metastasis accounts for these auwpsy findings? ***(A) Direct tumor extension ***(B) Hematogenous spread ***(C) Lymphatic spread ***(D) Seeding of body cavity ***(E) Venous spread
38 The answer is D: Seeding of body cavity. The photograph shows a loop of small bowel and mesentery studded with small nodules of metastatic cancer. Malignant tumors that arise in organs adjacem to body cavities (e.g., ovaries, gastrointestinal tract, or lung) may shed malignant cells into these spaces. Such body cavities include principally the petitoneal and pleural cavities, although occasional seeding o[ the pericardia! cavity, joint space, and subarachnoid space are observed. Tumor cells in these si tes grow in masses and often produce fluid (e.g., ascites or pleural Ouid), sometimes in massive quantities. Although the other choices provide routes for rumor metastasis, they do not lead to peritoneal carcinomatosis in patienlS with ovatian cancer. ***Diagnosis: Ovarian cancer, carcinomatosis
39 A 2-year-old boy is found to have bilateral retinal tumors. Molecular studies demonstrate a germline mutation tn one allde of the Rb gene. Which of the follmvinggenetic events best explains the mechanism of carcinogenesis in this patient? ***(A) Balanced translocation ***(B) Expansion of trinucleotide repeat ***(C) Gene amplification ***(D) Loss of heterozygosity ***(E) Maternal nondisjunction
39 The answer is D: Loss of heterozygosity. Retinoblastomas are 1nalignam ocular tumors of young children. In cases of hereditmy retinoblastoma, an affected child inherits one defective Rb aUele mgether wi£h one nom1al gene. This heterozygous state is not associated with any observable changes in t.he retina because 50% of the Rb gene product. is sufficient. to prevent the development of retinoblastoma. However, if the remaining nmmal Rb allele is inactivated by deletion or mutation, l11e loss of its suppressor function leads to the appearance of a neoplasm. This genetic process is referred to as loss of heterozygosicy: The other choices have not been associated 'Nilh the loss of £Umor suppressor genes in somatic cells. ***Diagnosis: Retinoblastoma
40 A 48-year-old nulliparous woman complains that her menstrual blood fiow is more abundant Lhan usual. An ultrasound examination reveals a polypoid mass in the ute1ine fundus. The patient subsequemly, undergoes a hysterectomy, which reveals a poorly differentiated endometrial adenocarcinoma_ The development o [this neoplasm was preceded by which of the following histopathologic changes in the glandular epithelium? ***(A) Atrophy ***(B) Hydropic swelling ***(C) Hyperplasia ***(D) Hypemophy ***(E) Metaplasia
40 The answer is C: Hyperplasia. The cellular and molecular mechanisms of hyperplasia are related to the control of eel] proliferation and provide a basis for further genetic changes that can lead to neoplasia. Endometrial hyperplasia refers to a spectrum that ranges from simple glandular crowding to conspicuous proliferation of atypical glands. These changes are often difficult to distinguish from carcinoma. The risk of developing endometrial cancer increases with higher degrees of endometrial hyperplasia. Estrogen exposure is thought to be a risk factor for both endometrial hyperplasia and endometrial carcinoma. Neoplas tic transfonnation may occur in the se tting of a metaplastic epithelium (e.g., cancers of the lung, cervix, stomach, and bladder); however, metapLasia (choice E) does not precede Lhe development of merine adenocarcinoma. The other cholces do not represem risk factors for cancer. ***Diagnosis: Endometrial adenocarcinoma
41 A 53-year-old womru1 with a longstanding history of ulcerative colitis presents with increasing chest pain and shortness oJ breath of 2 mom.hs duration. She reports four recent episodes of hemoptysis_ The patiem subsequently develops overwhelming sepsis and expires. A section through the tight lung is examined at autopsy (shown m the image) What is the appmpriate diagnosis? ***(A) Carcinoid tumor of lhe lung ***(B) P1imary adenocarcinoma of the lung ***(C) Metastatic carcinoma of the lung ***(D) tvHJiary tuberculosis ***(E) Sarcoidosis
41 The answer is C: Metastatic carcinoma of the lung. This patient's lung shows numerous nodules of metastatic carcinoma conesponding to "cannon ball" meLastases seen radiologically Pulmonary metastases are more common than primary lung rumors, and the histologic appearance of most metastases resembles that of the prima1y tumor. Persons with ulcerative colitis (such as this patient) have a higher risk of colorectal cancer than the general population. The risk is related to the extent of colorectal involvement and the duradon of the inflammatory disease. Carcinoid rumor of the lung (choice A) and primmy lung cancer (choke B) would not typically show multiple, circumscribed nodules. Milia1y tuberculosis (choice D) and sarcoidosis (choice E) feamre 111m-sized inl1ammawry nodules (minute granulomas). ***Diagnosis: Metastatic cancer, metastatic carcinoma of the lung
42 A 50-year-old woman presents with a 2-year histOI)' of upper truncal obesity and depression. Serum levels of glucose and cortisol are elevated. ACT scan of the abdomen reveals a 2-cm suprarenal mass. The surgical specimen is shown in the image. If this neoplasm is benign, which of the following is the most appropriate diagnosis? ***(A) Adenoma ***(B) Chondroma ***(C) Upoma ***(D) Papilloma ***(E) Teratoma
42 The answer is A: Adenoma. The patient shows signs and sympwms of Cushing syndrome (upper truncal obesity and hyperconisolism). The surgical specimen reveals a circumscribed tumor of the adrenal cortex that produces cortisol. Hismlogic examination of this tumor reveals nests of clear, lipid-laden epithelial cells. None of the other choices desCiibe a benign rumor of glandular epithelial origin. ***Diagnosis: Adrenal adenoma, Cushing syndrome
43 A 65-year-old man presents with a pearly papule on his upper lip (pa tient shown in the image). A biopsy reveals buds of atypical, deeply basophilic kerannocytes extending from the overlying epidermis into the papillary dermis. Which of the following carcinogenic stimuli was the most imponant risk factor for developmem of this patients skin cancer? ***(A) Aflatoxin B1 ***(B) Divalem metal cations ***(C) Aromatic ammes and azo dyes ***(D) Vinyl chloride ***(E) Sunlight
43 The answer is E: Sunlight. Basal cell carcinoma {BCC} is the most common malignam rumor in persons with pale skin. 13CC usually develops on the sun-damaged skin of people with fair skin and freckles. There is a direct correlation betwee11 total exposure lO sunlight and t.h.e incidence of 13CC, as well as squamous cell carcinoma and melanoma. The deleterious effects of sunlight (l)V radiation) include enzyme inactivation, mutagenesis, and cell death. Divalent metal cations such as nickel, lead, cadmhtm, cobalt, and beryllium (choice 13) can react v.rith biomolecules and induce cancer. Most metalinduced cancers occur in an occupational setting; however the carcinogenic mechanis1ns are unknown. ***Diagnosis: Basal cell carcinoma
44 A 28-year-old man with a familial disease affecting the gastrointestinal tract undergoes a colectomy. The surgical specimen is shown in the image. Molecular studies demonstrate a germline mutation in the APC gene. The normal product of this gene (protaoncogene) primatil y regulates which of the following cell behaviors? ***(A) Apoptosis ***(B) Autophagy ***(C) Ce 11 cycle ***(D) Differemiation ***(E) Motility
44 The ait.swer is C: Cell cycle. The surgical specimen reveals thousands of small adenomatous polyps on the mucosal surface of the colon. Patients with adenomatous polyposis coU have mutations in the APC rumor suppressor gene. Most cases are familial, bm 30% to '50% represem new mutations. The mean age for occurrence of symptoms is 36 years. 'Without the APC protooncogene, cells are unable to downregulate signals from E-cadherin to f3-catenin to nuclear transcription facwrs (myc and cyclin D) LhaL regulate cell cycle progression. Autophagy (choice B) is a normal catabolic process in which cellular components and organelles are degraded in lysosomes. Autophagy is often a response to cell injury It is also believed lO protect cells from intracellular pathogens and slow the p rogression of valious chronic diseases, including cancer. ***Diagnosis: Ade11omawus polyposis coli
1 A 4-year-old gi rJ presents for a preschool physical examination. The child has a small head circumference, thin upper Hp, and low-bridge nose. She shows evidence of mild mental retardation. Her paren ts slate that sl1e is often "emotional." Which of the following maternal causes of birth defects most likely accounts for these clinicopathologic findings? ***(A) Alcohol abuse ***(B) Cigarette smoking ***(C) Congenital syphilis ***(D) Inadequate nmrition ***(E) Poorly controlled diabetes mellitus
1 The answer is A: Alcohol abuse. Fetal alcohol syndrome refers to a complex of abnormalities induced by the maternal consumption of alcoholic beverages while pregnant that includes (l) growth retardation, (2) dysfunction of the cemral nervous system, and (3) characteristic facial dysmorphology (e.g., small head circumference and thin upper lip). One fifth of children with fetal alcohol syndrome have IQs below 70, and 40% have lQs between 70 and 85. Even with a nom1al JQ, these children tend to have short memory spans, impulsiveness, and emotional instability. The children of mothers who smoke (choice B) or who have inadequate nutrition (choice D) may also exhibit deficiencies in physical growth and intellectual development; however, Lhere is no associaLion with the pattern of facial dysmorphology and emotional instability seen in this case. Congenital syphilis (choice C) may also cause mental retardation, but it would show protean manifestations not illustrated in Lhis case. Gestational diabetes (choice E) does noL cause mental retardation. ***Diagnosis: Fetal alcohol syndrome
2 A 12-month-old boy is brought LO the emergency room for examination of his right arm following a rumble at home. Radiologic examination of the Umb reveals a recent fracrure of the right ulna and evidence of additional healing fTactures. The child is noted to have blue sclerae. This patient most likely carries a mmation in a gene that encodes which of the follO\ving proteins? ***(A) Collagen ***(B) fibrillin ***(C) Keratin ***(D) Myosin ***(E) Tubulin
2 The answer is A: Collagen. Osteogenesis imperfecta (01), or briLtle bone disease, is a group of inherited disorders expressed principally as fragility of bone. The genetic defects in the four types of OI are heterogeneous, but all affect the synthesis of type I collagen. Type l Ol is characterized by a nonnal appearance at birth, bm fractures of many bones occur during infancy and at the time the child leams to walk. Such patients have been desCJibed as being as "fragile as a China doll." Children with type I 0 1 typically have blue sclerae as a result of the deficiency in collagen fibers, which imparts translucence to the sclera. A high incidence of healing loss occurs because fractures and fusion of bones of the middle ear restr icL their mobility. Fibrillin gene mutations (choice B) are found in patients with Marfan syndrome. Ke ratin gene mutations (choice C) are found in patients with epidermolytic hyperkeratosis. ***Diagnosis: Osteogenesis imperfecta
_3_ A 28-year-old woman gives binh to a stillborn with a severe neural tube defect (neonate shown in the image). This birth defect was caused by an error of morphogenesis that occurred at \Vhich of the following slages of development after fertilization? ***(A) lto 10 days ***(B) 20 to 40 days ***(C) 90 to 120 days ***(D) 6w 9 momhs ***(E) Birth trauma
3 The answer is B: 20 to 40 days. Anencephaly refers to the congenital absence of the cranial vault, with cerebral hemispheres either missing or reduced w small masses. It is a dysraphlc anomaly of neural mbe closure that results from an injury to the fetus be tween the 23rd and 26th day of gestation. Dming feLal developmem, the neural plate is transformed into the neural tube by fusion of the posterior surfaces. Failure of the neural tube to close results in the lack of closure of the overlying bony so·ucrures of the cranium and an absence of the calvarium, skin, and subcutaneous tissues of this region. The exposed brain is incompletely formed or absem. Blastocyst fonnation and implantation occur on days 1 to 10 after fertilization (cholce A). ***Diagnosis: Acrania, neural LUbe defect
4 A 20·year-old man is examined by a new family physician who discovers numerous pigmemed patches and pedunculated skin tumors on his chest. Biopsy of a tumor discloses a berugn neoplasm detived from Schwann cells. Neither the patients father nor mot.her shows signs of this djsease. This patient most likely carries a mutation in a gene that encodes which of the fo llowing proteins? ***(A) Epidennal growth factor receptor ***(B) GTPase activating prQ[ein ***(C) NF·KB transcripLion fanor ***(D) Protein kinase C ***(E) Ras protein p21
4 The answer is 8: GTPase activating protein. Neurofibroma· . ..__ tosis type l (NFl) is characterized by (l) disfigming neurofibromas , (2) areas or dark pigmentation of the skin (cafe au lait spots), and (3) pigmented lesions of the iris (Lisch nodules). lt is one of the more common autosomal dominam disorders. The NFl gene has a high rate of mutation, and half of cases are sporadk rather than familial. The protein product, tem1ed neurofibromin, is expressed in many tissues and belongs to a family of GTPase-act.ivating proteins (GAPs), which inactivate ras protein (choice E). Thus, NFl is a classic tumor suppressor gene. Loss of GAP activity (in cells acquiring a second hit mutation) pennits uncomrolled ras p21 activation, an effect that p redisposes ro the formation of benign neurofibromas. None of the other choices (A, C, and D) are associated with the pathogenesis of neurofibromatosis. ***Diagnosis: Neurofibromatosis, type 1
5 The patient described in Question 4 is at increased risk of developing which of the following malignant neoplasms? ***(A) Ganglioneuroma ***(B) Glioblastoma multifom1e ***(C) Neurofibrosarcoma ***(D) Serous cystadenocarcinoma ***(E) Squamous cell carcinoma
5 The answer is C: Neurofibrosarcoma. One of the major compll cations of neuro6bromawsis type 1 (NF1), occurring in 3% w 5% of patients, is the appearance of a neuro6brosarcoma in a neurofibroma. Nfl is also associated with an increased incidence of other neurogenic tumors, including meningioma, optic glioma, and pheoch romocytoma. The other tumors listed are not associated with NFl. ***Diagnosis: Nemofibromatosis, type 1
6 A 25-ye.ar-old pregnant woman, at 16 weeks of gestation, visits her obstetrician. A screening test suggests the possibility of a neural rube defect iJ1 her ferus. An uJr rasound e.:xamination shows a 3--em neural tube defect m the thoracic spine. The screening test that was administered to Lhe mother measured serum levels of which of the follo\ving proteins? ***(A) Albumin ***(B) Alpha.fetoprotein ***(C) Bilirubin ***(D) Chromogranln ***(E) Human chorlonic gonadotropin
6 The answer is 8: Alpha·fetoprotein (AFP) . Scree11ing of preg· nam women for serum AFP and examination by ulnasonography allow detection of virtually all anencephalic fetuses. Levels of the mher prmeins are nor significamly affected by a neural mbe defect in the fetus. ***Diagnosis: Neural tube defect, spina bifida
7 A 25-year-old man presems for a routine physical examination. TI1e paliem is 1.all (6 fl, 5 in) and has long fingers (shown in the image). One year later, he suiTe rs a dissecting aonic aneurysm. This patient most likely cani es a mutation in a gene that encodes which of the following proteins? ***(A) Collagen ***(B) Dystrophin ***(C) Elasdn ***(D) Fibrillin ***(E) Myosin
7 The answer is D: Fibrillin. The cause ofMarfan syndrome has been established as missense mUlations in the gene coding for fiblillin-1 (FBNl). Fibrillin is a famUy of connective tissue proteins analogous m the collagens. It is widely disuibmed in many tissues in the fom1 of a fiber system H~Imed microfib1ils. For example, the deposition of elastin on microfibnllar fibers produces the concentric rings of elastin in the aonic walL Collagen gene mutations (choice A) are found in patien ts with Ehlers-Danlos syndrome and osteogenesis imperfecta. Dystrophin gene mutations (choice B) are found in patient.<; ''lith muscular dystrophy. ***Diagnosis: Marfan syndrome
8 The genetic disease encountered in the patient described in Question 7 follows which of the following patterns of inheritance? ***(A) Autosomal dominam ***(B) Autosomal recessive ***(C) Multifactorial ***(D) X-linked dominant ***(E) X-linked recessive
8 The answer is A: Autosomal dominant. Mar fan syndrome is an autosomal dominant, inherited diso rder of connective tissue characterized by a variety of abnormalities in many organs, including the heart, aona, skeleton, eyes, and skin. One third of cases represent sporadic mmations. The incidence in the United States is l per 10,000. ***Diagnosis: Marfan syndrome
9 A 12-momh-old boy shmvs progressive weakness, mental deterioration, and loss of vision. Laboratory smdies demonsnate decreased activity ofhexosaminidase A. The child evemually becomes blind and dies at 3 years of age. Which of the fo llowing best describes the pathogenesis of neuronal degeneration in this patient? ***(A) Accumulation of unmetabolized substrate ***(B) Decreased utilization of metabolic end-product ***(C) Fonnauon of an abnormal metabolic end-product ***(D) Opening of mitochondr ial membrane pore ***(E) Synthesis o[ a novel glycosphingolipid
9 The answer is A: Accumulation of unmetabolized substrate. Tay-Sachs disease is the catastrophic infantile variant of a class of lysosomal storage diseases known as Gtv~ gangliosidoses. This ganglioside is deposited in neurons of the central nervous system due to a failure oflysosomal degradaLion and accumulation of an unmetabolized substrate. Gangliosides are glycosphingolipids that are presenL in the outer leaflet of the plasma membrane, particularly in neurons. The lysosomal catabolism of ganglioside GM2 is accomplished through the activit)' of the ~-hexosarninidases (A and B), which are composed of a and p subunits and require the participation of the GM2-activator protein. A deficiency in any of these components results in clinical disease. None of the other choices e~rplains the pathogenesis of tlus disease. ***Diagnosis: Tay-Sacbs disease
10 lf the parenl5 of the child described in Question 9 have a total of four sotlS and t\VO daughters, then, on average, how many of their children may be expected to be asymptomaLlc (ie., silent) carriers of tl1is gene mmation? ***(A) One child ***(B) Two children ***(C) Three children ***(D) Four children ***(E) Five children
10 The answer is C: Three <hildren. Tay-Sachs disease is inherited as an amosomal recessive trait and is predominantly a disorder of Ashkenazi jews, in whom the carrier rate is lin 30, and the natural incidence of homozygotes is l in 4,000 live newborns. In amosomal recessive diseases, on average, half of the o(fspring are expected to be heterozygmes and silem carriers of the gene mmation. ***Diagnosis: Tay-Sachs disease
11 A 4-year-old boy is admitted to the hospital with pneumon ia and respiratory distress. The nurses repon that the childs bowel movements are greasy and have a pungent odor. A sweatchiOJide test is posi tive. Which of the following mechanisms of disease is the most likely cause of steatorrhea in this child? ***(A) Abnmmal dietary intake ***(B) Bacterial overgrowth ***(C) Hyperbilirubinemia with kernlcterus ***(D) L1ck of pancreatic enzyme secretion ***(E) Obstruction caused by meconium ileus
11 The answer is D: Lack of pancreatic enzyme secretion. Cystic fibrosis (CF) is an autosomal recessive disorder affecting children, which is characterized by (l) chronic pulmonary disease, (2) deficient exocrine pancreatic function , and (3) mher complications of inspissated mucus in a number of organs, including the small intestine, the liver, and the reproductive n·act. The diagnosis of CF is most reliably made by the dem- 011Stration of increased concentrations o[ electrolytes in the sweat. The decreased chloride conductance characteristic of CF resulls in a failure of chloride reabsorption by the cells of the sweat gland ducts and, hence, w the accumulation of sodium chloride in !.he sweat. AU of the pathologic consequences of CF can be amibULed to the presence of abnormal ly thick mucus. Lack of pancreatic enzyme secretion in patients ''lith CF causes malabsorption and foul-smelling fatty stools (steawrrhea). The other choices do not address the underlying cause of malabsorption in patients with CF. ***Diagnosis: Cystic fibrosis
12 The patiem described in Question 11 carries mutations in the gene that encodes which of the following types of protein? ***(A) Membrane ion channel ***(B) Mitochondrial transport protein ***(C) NaT/K' A:rPase ***(D) Nuclear transport protein ***(E) Recepwr tyrosine kinase
1 Z The answer is A: Membrane ion channel. The gene responsible for cystic fibrosis (CF) encodes a large protein termed the cystic fibrosis transmembrane conductance regulator (CfTR). CFTR is a member of the AIP-binding family of membrane transporter proteins that constitutes a chloride chmmel in most epithelia. The secretion of chloride anions by mucoussecreting epithelial cells controls the parallel secretion of fluid and, consequently, the viscosity of the mucus. h is estimated th at 1 in 25 whiLes is a heterozygous car rier of the CF gene, and the incidence is lin 2,500 newborns. The most common cause of morbidity and mortality in patients with CF is pulmonary disease, secondary to chronic infections. Receptor tyrosine kinase gene mutations often lead to uncontrolled cell growth. ***Diagnosis: Cystic fibrosis
13 A lO~year· ol d child presents with xanthomas on the ex'tensor surfaces of his forearms. laboratory studies demonstrate a total serum cholesterol of 820 mgldl. The childs mother and maternal gran dfather also have elevated serum cholesterol. This patient most likely has murations in the gene that encodes which of the following proteins involved in lipid metabolism? ***(A) ApoE4 ***(B) Cholesterol hydroxylase ***(C) Chylomicron transport protein ***(D) High-density lipoprotein receptor ***(E) Low-density lipoprotein recepLOr
13 The answer is E: Low-density lipoprotein (LDL) receptor. Familial hypercholesterolem1a is an autosomal dominant disorder characterized by high levels of LDLs in the blood, accompanied by the deposition of cholesterol in arteries, tendons, and skin. It is one of the most common autosomal dominant disorders, and in its heterozygous form, it affects at least l in 500 adults in the United States. Only l in I million persons is homozygous for the disease. Familial hypercholesterolemia results from abnormalities in the gene that encodes the cell su rface receptor that removes LDLs fmm the b lood. Mutations in the other genes do not cause hypercholesterolemia. ***Diagnosis: Familial hypercholeste rolemia
14 A 10-momh-old boy \~.rho was adop ted from an orphanage in Eastern Europe p resents for a physical examination. His paren i:S believe that he is failing to meet developmental mHestones. The child is fair skinned and has blond hair. On physical examination, the patient is noted to have a "mousy" odor. Laboratory studies demonstrate an inborn error of amino acid metabolism. To prevent mental retardation, this patient should be placed on a special diet that lacks which of the following essential amino acids? ***(A) Isoleucine ***(B) Methionine ***(C) Phenylalanine ***(D) TI1reonine ***(E) Tryptophan
14 The answer is C: Phenylalanine. Phenylketonuria (PKU, hyperphenylalaninemia) is an autosomal recessive disorder characterized by progressive mental deterioration Ln the first few years of life due to high levels of circulating phenylalanine, secondary to a deficiency of phenylalanine hydroxylase. The disorder is based on a genetic defect, but its expression depends on the provision of a dietary consdment. The affected infant appears normal at birth, but mental retardation is evident willun a few months. lnfams with PKU tend to have fair skin, blond hair, and blue eyes because the inability to convert phenylalanine to tyrosine leads to reduced melanin synthesis. These patients exude a umousy" odor due to the formation of phenylacetic acid. The o·eatment of PKU involves the restriction of phenylalanine in the diet. None of the other essemial amino acids accumulates in patiems with PKU. ***Diagnosis: Phenylketonuria
15 Which of the following best describes the pathogenesis of mental retardation in Lhe patient described in Question 14' ***(A) Accumulation of unmetabolized substrate ***(B) Decreased utilization of metabolic end-product ***(C) Formation of an abnormal metabolic end-product ***(D) Increased utilization of metabolic end-product ***(E) Opening of miwchondrial membrane pore
15 The answer is A: Accumulation of unmetabolized substrate. Phenylalanine is an essendal amino acid that is de1ived e..'iclusively from the diet and is oxidized in the liver to tyrosine by phenylalanine hydroxylase (PAH). A deficiency in PAH results in both hyperphenylalaninemia and the fonnadon of phenylkewnes from rhe rransamination of phenylalanine. The excretion in the urine of phenylpymvic acid and its deJivatlves accoums for the original name of phenylkeLOnuria. However, it is now established that phenylalanine itself, rather than its metabolites, is responsible for the neurologic damage cemral to th]s disease. Thus, the term hyperphenylalaninemia is actually a more approptiate designation than PKU. None of the other choices e>q>lains the accumulation of phenylalanine in these patients. ***Diagnosis: Phenylketonuria
16 A 4-year-old boy is found to have extremely pliable skin. His parents note that he bruises easily. His joints can be hyperextended. Biochemical studies demonstrate a deficiency of lysyl hydro:>..)llase. Ultrastructural examination of a skin biopsy of this patiem would most likely reveaJ abnormalities associated with which of the following cell/ tissue components? ***(A) Actin-myosin filaments ***(B) Collagen fibers ***(C) Glycocalyx ***(D) lmermediate filaments ***(E) Mitochondria
16 The answer is B: Collagen fibers. Ehlers-Danlos syndromes (EDS) are a group of rare, autosomal dominam, inherited disorders o[ connective tissue that feature remarkable hyperelasticity and fragility of the skin, joint hypermobility, and often a bleeding diathesis. The common feamre of most types of ED$ is a generalized defect in collagen, including abnormalities in irs molecular stmctttre, synthests, secretion, and degradation. Patients typically can stretch the skin many centimeters, and uivial injuries can Lead lO serious wounds. Because sutures do not hold well, dehiscence of surgical incisions is common. Hypermobility of the joints allows unusual ex1ension and flex]on. Abnormalities would not be e>q)ected in the other cell/tissue components listed. ***Diagnosis: Ehlers-Danlos syndrome
17 A 25-year-olcl woman complains of recunent bone pain and increasing abdominal ginh. Physical examination reveals massive hepatosplenomegaly. Radiologic studies reveal several radiolucent bone defects. A bone marrow biopsy discloses enlarged cells with a fibrillar appearance reminiscent of "wrinkled tissue paper." Microscopic examination of a splenectomy specimen is shown. This paoem most Likely carries mutations in the gene that encodes which or the following rypes of hydro lyric enzymes? ***(A) Galactosidase ***(B) Glucosidase ***(C) Hexokinase ***(D) N-acetylgalactosaminidase ***(E) Neuraminidase
17 The answer is B: Glucosidase. Gaucher disease is characte1ized by the accumulation of glucosylceramidc, primati ly in the 1ysosomes of macrophages. The underlying abnom1ality in Gaucher disease is a deficiency in glucocerebrosidase, a type of lysosomal acid P-gtucosidase. The hallmark of this disorder is the presence of Gaucher cells, which are lipidladen macrophages that are characteristically present in the red pulp of the spleen, liver sinusoids, lymph nodes, lungs, and bone marrow. These cells are derived from the residem macrophages in the respective organs (e.g., KupiTer cells in the liver and aJveolar macrophages in the lung). Galactosidase gene mutations (choice A) are found in patients with Fabry disease. N-acetylgalactosaminidase gene mutations (choice D) are found in patients with Tay-Sachs disease. ***Diagnosis: Gaucher disease
18 Which of the following besL describes the pathogenesis of hepatosplenomegaly and bone pain in the patiem described in Question 1 7? ***(A) Accumulation of unmetabolized substrate ***(B) Decreased utilization of metabolic end-product ***(C) Formation of an abnonnal metabolic end-product ***(D) Increased utilization of metabolic end-product ***(E) Opening of mitochondrial membrane pore
18 The answer is A: Accumulation of unmetaboli:ted substrate. Glucosylceramide is a conunon core structure for membrane glycosphingolipids. The glucosylceramide that accumulates in Gaucher cells in the spleen, liver, bone marrow, and lymph nodes derives principally from the catabolism of senescent leukocytes. The membranes of these cells are rich in the cerebrosides, and when t11eir degradation is blocked by the deficiency of glucocerebrosidase, the intermediate metabolite, glucosylceramide, accumulates. The other biochemical pathways listed do not cause sphingolipid accumulation in patients with lysosomal storage diseases. ***Diagnosis: Gaucher disease
19 A neonate is born with severe motor dysfunction involving the lower extremities. Radiologic sntdies show that vertebral bodies in the lumbar region lack posterior arches. The vertebral defects are covered by a thin membrane. The space underneath the membrane contains a mass of tissue that is composed of meninges and spinal cord. The pa rents ask for info rmation regarding risks for similar birth defects in their future offspring. You mention that dieta ry supplememat:ion of the maternal diet has been shown to reduce the incidence of neural mbe defects. What is this substance? ***(A) Folk acid ***(B) Niacin ***(C) Thiamine ***(D) Vitamin 8 0 ***(E) Vitamin B12
19 The answer is A: Folic acid. Spina bifida is a congertital defect in the closure o[ Lhe spinal canal. Like other dysraphic disorders (anencephaly, meningocele, and menmgomyelocele), spina bifida is of polygenic otigin. Folic acid supplied in Lhe peli conceplional period lowers Lhe incidence of neural rube clefecls. ln 1998, the United States Food and Dmg Administration began requiling manufacturers of emiched flour, bread, and some OLher products LO supplement these foods with folate This mandate has been associated with a significant decrease in the incidence of neural tube defects. Folk acid deficiency can resulL in elevated serum Levels of homocysteine, a maternal risk factor for neural rube defects. Thiamine deficiency (choice C) causes beri-beti (polyneuropathy, edema, and heart failure). Vitamin B12 deficiency (choice E) causes megaloblastic anemia bm not neural tube defects. ***Diagnosis: Spina bifida
20 The paren ts of an infam with cleft lip and palate (iniam shown in rhe image) visit a genetic counselor to discuss the chance t.hat a similar birth defect will occur in their future offspring. ln addition LO teratogen exposure and multifactorial inher itance, whkh of the following is an imponam cause of this error of morphogenesis? ***(A) Down syTtdrome ***(B) First pregnancy before 25 years of age ***(C) Maternal-fetal Rh incompatibility ***(D) Structural chromosomal abnom1ality ***(E) Turner syndrome
20 The answer is D: Structural chromosomal abnormality. Cleft lip and cleft palate exemplify multifactorial inheritance in which multiple genes interact with various environmemaJ factors lO p roduce disease. On the 35th day of gestation, the frontal prominence fuses with the ma;...'illary process to form t.he upper lip. Disturbances in gene expression at this time (hereditary or environmemal) lead to interfe rence wilh p roper fusion and result in cleft lip, with or wi thout cleft palate ln addition to multifactorial inheritance, this developmemal anomaly may be part of a malfom1ation syndrome caused by teratogens (e.g., rubella and anticonvulsants). It is also often encountered in children \:vith chromosomal abnonnalities (conect answer). The incidence of cleft lip, with or without ddt palate, is 1 in 1,000, and the incidence of deft palate alone is 1 in 2,500_ lf one child is bom with a deft lip, the chances are 4% dmt the second child will exhibit the same defect. lf the first two children are affected , the risk of cleft lip increases to 9% for the third child. The more severe the anatomical defect, the greater the probability of tTansnutdng cleft lip will be. Whereas 75% of cases of deft Lip occur m boys, the sons of women with cleft lip have a four times higher 1isk of acquiting ll1e defect than the sons of affected fathe rs. None of the other choices are associated with a significantly increased risk of cleft lip. ***Diagnosis: deft Lip, multifactOrial inheritance
21 A 4-year-old boy is brought to the physician by his parents because he tires easily. Physical examination reveals weakness in the pelvic and shoulder girdles and enlargement of the child's calf muscle. Semm levels of creatine kinase are elevated. A biopsy of calf muscle shows marked variation in size and shape of muscle fib ers. There are foci of muscle fiber necrosis, with myophagocytosis, regenerating fibers, and fibrosis. Molecular diagnostic assays would most likely show alterations in the length of the primary transcript for which of the following muscle-associated proteins? ***(A) Actin ***(B) Desmin ***(C) Dystrophin ***(D) Glycogen phosphorylase ***(E) Myosin
21 The answer is C: Dystrophin. Duchenne muscular dystrophy (DMD) is a severe, X-linked condition characterized by progressive degeneration of muscles, particularly those of the pelvic and shoulder gi rdles. A milder form of the disease is known as Becker muscular dystrophy (BMD). Both DMD and BMD are caused by a deficiency of dystrophin, a member of lhe family of membrane cytoskeletal prmeins, which includes cx-acttnin and specnin. The protein is located on the cytoplasmic face of the plasma membrane of muscle cells and is linked to it by imegral membrane glycoprOleins (dystrophiD-associated glycoprotein complex), wh.ich in turn, are bound to extracellular laminln. lt has been proposed that the absence of dystrophin leads lO a defective membrane that is damaged dming contraction, an effect that predisposes to death of the myocyte. Serum levels of creatine kinase are increased. Glycogen phosphorylase gene mutations (choice D) are found in patients with McArdle disease. ***Diagnosis: Duchenne muscular dystrophy
22 What will be the lil<ely cause of death in the patiem described in Question 21? ***(A) Cardiomyopathy ***(B) Cerebrovascular disease ***(C) End-stage renal disease ***(D) Pulmonary saddle embolism ***(E) Res pi 1-ato ry i nsu £ficiency
22 The answer is A: Cardiomyopathy. The symptoms of Duchenne muscular dystrophy (DMD) progress with age. During the first year of Ilfe, the infanlS appear nonnal, but more than half failw walk by 18 months of age. More than 90% of afflicted boys are wheelchair bound by the age of ll years. In advanced disease. cardiac sympwms are almost universal, and cardiomyopathy is a common cause of dearJ1. The mean age at death m boys with DMD is 17 years. The other choices are unrelated tO DMD. ***Diagnosis: Duchenne muscular dysu·ophy
23 A 2.2-year-old man complains about his inability to conceive a child. On physical examination, the patient is noted to be tall (6 ft, 5 in} and exhibits gynecomastLa and testicular anophy. Laborawry srudies demonstrate increased serum levels of folU cle-stimuladng hormone. Cytogenetic studies reveal a chromosomal abnom1ality. What is the most common cause of this patiem's chromosomal abnormality? ***(A) Expansion of a trinucleotide repeat ***(B) Isochromosome fmmation ***(C) Meiotic nondisjunctLon ***(D) Nonreciprocal translocation ***(E) Rlng chromosome fom1adon
23 The answer is C: Meiotic nondisjunction. Klinefelter syndrome, or testicular dysgenesis, is related to the presence of one or more X chromosomes in excess of the nonnal male XY complement. Most persons with Klinefelter syndrome (80%) have one extra X chromosome (47 ,XXY karyotype). The additional X chromosome(s) arises as a result of n ondisjunction during gametogenesis. ln half o[ cases, nondisjunction occurs during patemal meiosis l, leading to a spenn containing both an X and a Y chromosom.e. FertiJizadon of a nmmal oocyte by such a spenn gives a zygote with a 4 7 ,Xt'\Y complement of chromosomes. Klinefelter syndrome occurs in l per 1,000 male newboms, which is roughly comparable to the incidence of Down syndrome. None of the other choices are associated with trisomy. ***Diagnosis: Klinefelter syndrome
24 A 35-year-old pregnant woman delivers a baby prematurely at 28 weeks of gestation. Shonly after binh, the neonate becomes short of breath , with intercostal retraction and nasal flati ng during respiration. The neonate is placed on a ventilator, but d.ies of respiratory insufficiency and intraventticular hemonhage. Microscopic examination of the lungs at autopsy is shown. The eosinophilic material lining the air spaces represents an accumulation of which of the fo llow1ng proteins? ·- - - ~ -· ***(A) Collagen ***(B) Dystrophin ***(C) Fibnn ***(D) Fibronectin ***(E) Laminin
24 The answer is C: Fibrin. The pathogenesis of respiratory disuess syndrome (RD$) of the newbom is innmately linked LOa deficiency of surfactant. Collapse of the alveoli (atelectasis) secondary to surfactant deficiency results in perfused but not ventilated alveoli, a situmion that leads to hypo>.:ia and acidosis. The leak of fib rin-rich Ouid into the alveoli from th e injured vascular bed cont1ibmes to the typical clinical and pathologic features of RDS. On gross examination, the lungs are dark red and airless. The alveolar ducts are lined by conspicuous, eosinophilic, fi.brin-ti ch, amorphous structures, tenned hyaljne membranes. Although collagen (choice A) , fibronectin (choice D), and laminin (choice E) are found in most tissues, they do not represent the major protein found in hyaline membranes. ***Diagnosis: Respiratory distress syndrome of the neonate
25 If the neonate described in Question 24 had survived, which of the following would be the most l ikely complication related to anO)(ia and acidosis? ***(A) Bilirubin encephalopathy (kernicterus) ***(B) Erythroblastosis fetalis ***(C) Necrotizing enterocolitis ***(D) Pulmonary embolism ***(E) Ventricular sepual defect
25 The answer is C: Necrotizing enterocolitis. The first symptom of RDS (usually appealing within an hour of birth) is increased respirato ry effort, ·with forceful intercostal retraction and the use of accessmy neck muscles. Despite advances in neonatal intensive care, tlle overall mortality of RDS ts about 15%. Necrotizing enterocolitis is the most common acquired gastrointestinal emergency in newborns and is thought to be related to ischemia of the intestinal mucosa. This injury is followed by bacterial colonization, usuaUy with Clo.sl1idium difficile. The lesions vary from those of typical pseudomembranous enterocolitis to gangrene and perforation of the boweL None of the other choices are related to respiratory insufficiency. ***Diagnosis: Respiratory distress syndrome of the neonate, enterocolitis
26 A 16-year-old girl complains that she has not started menstmating like other girls her age. The patient is short (4 ft, 11 in) and has a duck-webbed neck. Physical exammation reveals widely spaced nipples and poor breast developmem. If this patient's genetic disease was caused by nondisjunction during mitosis of a somatic cell in the early stages of embryogenesis, which of the following is the patients most likely kmyotype? ***(A) 45,X ***(B) 45 ,X/-t6 ,X-\: ***(C) 45,X/46,XY ***(D) 47,XX,+21 ***(E) 47)CXY
26 The answer is B: 45,X/46,XX. Mitotic nondisjunction may involve embryonic cells during early stages of development and result in chromosomal aberrations. This condition in which the body contains two or more karyotypically different cell lines is called mosaicism. Mosaicism involving sex chromosomes is found in patients with Turner and Klinefelter syndromes. Turner syndrome re fers to the spectrum o[ abnormalities that resuh from the presence of complete or partial monosomy of the X chromosome in a phenotypic female. rlalf of women vvi1h Turner syndrome lack an entire X chromosome (monosomy X, choice A). The remainder of women with Turner syndrome are mosaics or display s tructural aberrations of th e X chromosome. Mosaics characterized by a 45,X/46,XX karyotype (15%; choice B) tend to have milder phenQ[ypic manifes tations. b1 about 5% of patien ts, the mosaic karyotype is 45,X/46,)CY (choice C). A patient wi th the 47,XX.+21 karyotype (choice D) is a female with Do'\\rn syndrome. The 4 7 .~'\'Y ka1yotype (choice E) is a feature of Klinefelter syndrome. ***Diagnosis: Turner syndrome
27 A34-year-old woman in her second pregnancy delivers a female neonate with severe generalized edema and jaundice A CBC of the neonate shows hemolytic an.emia. Subsequent workup of the mOlher and the newborn reveal an Rh-incompadbility Transplacental passage of which of the following proteins is the ptincipal cause of anasarca and jaundice in this neonate? ***(A) Complemem C3 ***(B) Complement C4 ***(C) IgG ***(D) lgM ***(E) Interferon-a.
27 The answer is C: lgG. E1ythroblastosis fetalis is an antibodymediated hemolyric disease dmt affects the ferus in utero. It is usually caused by transplacenr.al passage of maternal an tibodies w antigens v:pressed on fetal RBCs. The introduction of Rh-posidve fetal eryth rocytes (>1 ml) into the circulation of an Rh-negative mQ[her at the time of delivery sensitizes her to the D antigen. vVhen the antigen-sensitized mother again bears an Rh -posidve fetus, much smaller quanti ties of fetal D antigen elicit an increase in antibody titer. ln contrast to IgM (choice D), IgG antibodies are small enough to cross the placenta and thus produce hemolysis in the fetus. This cycle is exaggerated in multiparous women , and the severity of erythroblastosis tends to increase progressively with each succeeding pregnancy ***Diagnosis: Hemolytic anem1a of the neonate, erythroblastosis fetalis
28 The parents of a 2-year-old boy with hyposadias (urethra opens on the vemral aspect of the penis) visit a genetic counselor to discuss the chances that a similar binh defect \Viii occur in their future offspring. This birth defect shmvs which of the following patterns of inheritance? ***(A) Autosomal recessive ***(B) Autosomal dominant ***(C) Mulrifacroria1 ***(D) X-linked dominant ***(E) X-linked recessive
28 The answer is C: Multifactorial. The inheri tance of a number of binh deJects is multifactorial. Most normal human Lraits are inhetited neither as dominant nor as recessive mendeUan attributes, but rather in a more complex manner. For example, multifactorial inheritance determines imelligence, height, skin color, body habitus, and even emotional disposition. Similarly, most of the common chronic disorders of adults represent multifactorial genetic diseases and are well known to "run in families ." Such maladies include diabetes, atherosclerosis, and many forms of cancer and arthritis, and hypertension. The inheritance of a number of birth defects is also multifactOJial (e.g., cleft Up and palate, pyloric stenosis, hypospadias, and congenital heart disease). The concept of multi factOJia l inheritance is based on the notion that multiple genes interact with various environmental facto rs to produce disease in an individual patient. Such inheritance leads to familial aggregation that does not obey simple mendelian rules (see choices A, B, D, and E). As a consequence, the inheritance of polygenic diseases is smdied by the methods of population genetics, rather than by the analysis of individual family pedigrees. ***Diagnosis: Hypospadias, multifacwrial inheritance
29 A 42-year-old woman gives birth to a neonate with multiple congenital abnonnalities. Physical findings included a flat facial profile, slanted eyes, epicantha1 folds, Brushfield spots, short nose, short neck, dysplastic ears, clinodactyly, a large prottuding tongue, and a pronounced hearr murmur. \oVhat is the most common cause of this developmental birth disease? ***(A) Chromosomal deletion ***(B) Chromosomal translocation ***(C) Expansion of trinucleotide repeat ***(D) Frameshift point mutation ***(E) Nondisjunction
29 The answer is E: Nondisjunction. Nondisjunction during the firsL meiotic division of gametogenesis accoums for most (92% to 95%) paLiems vvith Down syndrome who have trisomy 21. The e..xtra chromosome 21 is of maternal origin in about 95% of Down syndrome children. Translocation of an e"-'tra long am1 of chromosome 21 to another acrocemric chromosome (choice B) causes abom 5% of cases of Down syndrome. The other choices are unrelared to trisomy 21. ***Diagnosis: Down syndrome, trisomy 21
30 As an adult, the brain of the patient described in Question 29 will show histopathologic changes that are seen in patients \Vith which of the following neurologic diseases? ***(A) Alzheimer disease ***(B) Huntington disease ***(C) Krabbe disease ***(D) Multiple sclerosis ***(E) Parkinson disease
30 The answer is A: Alzheimer disease. One of the mosrintriguing neurologic featmes o[ Down syndrome is its association with Alzheimer disease. The morphologic lesions characteristic o[ Alzheimer disease progress in aU patients with DO\vn syndrome and are universally demonstrable by age 35 years. These changes in the brain include (1) granulovacuolar degeneration, (2) neurofibrillary tangles, (3) senile plaques, and (4) loss of neurons. The senile plaques and cerebral blood vessels of both Alzheimer disease and Dovm syndrome always contain an amyloid composed of the same fibrillar protem (I)-amyloid protein). The other chokes are unrelated to Down syndrome. ***Diagnosis: Down syndrome
31 A 50-year-old man wi th a history of type 2 diabetes mellirus asks about the chances that his children v.rill inherit this metabolic disorder. The patient is told that he has a genetic disease that shows which of the following patterns of inheritance? ***(A) Autosomal dominant ***(B) Autosomal recessive ***(C) Multifactorial ***(D) X-linked dominant ***(E) X-linked recessive
31 The answer is C: Multifactorial. Most of the common chronic disorders of adults represent multifactorial genetic diseases that tend to "run in families.·· Such maladies include djabetes, atherosclerosis, and many forms of cancer, anhritis, and hypertension. Fragile X syndrome and Duchenne-Becker muscular dystrophy are examples of X-linked recessive genetic diseases. ***Diagnosis: Diabetes mellims
32 A 2.5-year-old man with a history of autism and mental retardation is seen by a genetic counselor. The man has coarse facial fearures , an increased head circumference, and macro-orchidism. His maternal uncle is similarly affected. After funher evaluation, a diagnosis of fragile X syndrome is rendered. What is the most likely underlying cause of this patients genetic disease? ***(A) Chromosomal nondicsjunction ***(B) Chromosome inversion ***(C) Expansion of trinucleotide repeat ***(D) Frame-shift mutation ***(E) Nonreciprocal translocation
32 The answer is C: Expansion of trinucleotide repeat. FragLle X syndrome, the most common cause of inherited mental retardation, is caused by expansion of a CGG trinucleotide repeat in a noncoding region immediately adjacent to the FMRJ gene on the X chromosome. Ln a poorly understood manner, the e>..rpanded CGG repeat silences the FMRl gene by methylation of its promoter. The abnormal repeat is associated with an inducib le "fragile sire·· on the X chromosome, which appea rs in cytogenetic studies as a nonstaining gap or chromosomal break. The male newborn afflicted with the Cragile X syndrome appears nom1al, but dtning childhood, characteristic features appear, including an increased head circumference, facial coarsening, joint hyperextensibUity, enlarged testes, and abnormalities of the cardiac valves. Mental retardation is profound, '"'ith IQ scores varying From 20 to 60. A significant proportion of autistic male children carry a fragile X chromosome. The other choices do not cause fragile X syndrome. ***Diagnosis: fragile X syndrome
33 A 2.8-year-old man presents to the emergency room l h our after experiencing crushing substernal chest pain. U:lboratory studies and ECG confirm the diagnosis of acute myocardial infarction. The patient dies 24 hours later of cardJac arrhythmia. Thts patient most likely had which of the following genetic diseases? ***(A) Adult-onset (type 2) diabetes ***(B) a 1-Antitrypsin deficiency ***(C) Familial hypercholesterolemia ***(D) Marfan syndrome ***(E) Niemann-Pick disease
33 The answer is C: Familial hypercholesterolemia. Familial hypercholesterolemia is an autosomal dominant disorder caused by mutations of the gene encoding the LDL receptor. lt is one o[ the most common autosomal dominant disorders , affecting 1 in 500 adults in the United States. The gene defect affects the uptake of LDL in the liver, causing hypercholesterolemia. Clinically, the disease presents as severe atherosclerosis, which usually becomes symptomatic at an early age. Diabetes mellitus (choice A) also causes accelerated atherosclerosis bm rarely at this age. Marfan syndrome (choice D) is associated with dissecting aortic aneurysm. Niemann-Pick disease (choice E) is a hereditary lysosomal storage disease. ***Diagnosis: Familial hypercholesterolemia
34 A 5-year-old boy presents with a maculopapular rash. On physical examination, the rash affects the palms and soles. Cracks and fissures are noted around the mouth and anus. There is funduscopic evidence of interstitia] keratitis. Mild hepatosplenomegaly is present. The anterior tibial bones exhibit an outward curvature. What is the mostli.kely etiology of these clinicopathologic findin gs? ***(A) AlDS ***(B) Cytomega1ovims ***(C) Herpes ***(D) Syphilis ***(E) Toxoplasmosis
34 The answer is D: Syphilis. The acronym TORCH refers to a complex of similar signs and symptoms produced by fetal or neonatal infection with a variety of microorganisms, including Toxoplasma (T), Rubella {R), Cytomegalov irus (C), and Herpes {H). The letter "0" represents "mhers" including congenital syphilis The acronym was coined to alert pediatricians w the faCl that infections in the fetus and newborn by TORCH agents are usually indistinguishable from each other and that testing for one of the four major TORCH agents should include testing [or the other three and others as well. The organism that causes syphilis, Treponema pallidum, is transmitted to the fetus by a mother who has acquired syphilis during pregnancy. A maculopapular rash is a common early finding in congenital syphilis. The most common osseous lesion in congenital syphilis is periostitis and outward curving of the anterior tibia (saber shins). Flat raised plaques (condylomata lata) around the anus and female genitalia may develop early or after a few years. The diagnosis of congenital syphilis is suggested by clinical findings and a history of maternal infection. None of the other pathogens cause these clinicopathologic findings. D.iagnosis: TORCH syndrome, syphilis 35 The answer is A: Aplasia . Aplasia is the absence of an organ coupled with perslstence or the organ anlage or a mdiment. Thus, aplasia of the lung refers to a condition in which the main bronchus ends blindly in nondescript tissue composed of rudimentary ducts and connective tissue. Dysraphic anomalies (choice C) are defects caused by the failure of apposed structures to fuse. Hypoplasia (cho ice D) refers to reduced size owing to the incomplete development or all or part of an organ. Examples include microphthalmia and microcephaly Involution failures (chmce E) reOect the persistence of embryonic or fetal strucmres that should involute at cen.ain stages of development A persistent th)'t'oglossal duct is the result of incomplete involution of the tract that connects the base of the tongue witb the thyroid ***Diagnosis: Pulmonary aplasia
35 A 3-year-old boy djes in an automobiJe accident. At autopsy, the right lung is markedly shrunken. Dissection shows that the right main stem bronchus ends blindly in nondescript tissue composed of rudimentary ducts and connective tissue. This finding represents an example of which of the following errors of morphogenesis? ***(A) Aplasia ***(B) Attesia ***(C) Dysraphic anomaly ***(D) Hypoplasia ***(E) Involution failure
36 The answer is 8: Atresia. Atresia refers to defects caused by the incomplete formation of a lumen. Many hollow organs originate as strands and cords of cells whose centers are progranm1ed to die, thus forming a central cavity or lumen. Atresia of the esophagus is characterized by partial ocdusion of the lumen, which was not fully established in embryogenesis. Dysplasia (choice C) is caused by abnormal organization of cells into tissues, which is a situation that results in abnonnal hlc;togenesis. Tuberous sclerosis is a striking example of dysplasia, in which the brain contains aggregates of nonnally developed cells ananged into grossly visible "mbers." Ectopia (choice E) is an anomaly in which an organ is outside its normal anatomic site. ***Diagnosis: Esophageal atresia
36 The mother o[ a newborn boy is alanm:d that her baby regurgicaLes at every feediJl g. An endoscopic examination reveals that the child's esophagus is almost completely occluded. This finding represents an example of which of the following errors of morphogenesis? ***(A) Aplasia ***(B) Atresia ***(C) Dysplasia ***(D) Dysraphic anomaly ***(E) Ectopia
37 The answer is A: Choristoma. Chorlstomas are minute or microscopic aggregates of normal tissue in aberrant locations. Choristomas are represented by rests of pancreatic tissue in the wall of the gastrointestinal tract or of ad renal tissue in the renal conex. By contrast, hamartomas (choice B) represent focal, benign overgrowths of one or more of the mature cellular elements of a normal tissue, often with one element predominating. Hemangiomas (chmce C) are the most frequently encountered tumors in childhood. ***Diagnosis: Choristoma
37 An 87 -year-old woman dies peacefctlly in her sleep. At autopsy, a rest of pancreatic tissue is identified in the wall of the lower esophagus. This finding represents an example of which oflhe following congenitalmmor-llke conditions? ***(A) Choristoma ***(B) Hamartoma ***(C) Hemangioma ***(D) Papilloma ***(E) Teratoma
38 The answer is D: Lecithin. Inmlaturity of the lungs poses one of the most. common and immediate threats to t.he viability of the low birth weight. infam because the lining cells o[ the feral alveoli do noL difTerentiate until ian~ pregnancy. Alveoll are maintained in the e~,.:panded stare, in pan, by d1e presence of pulmonary surfactam. This material, which is produced by type 1 L pneumocytes, is a complex mixture of several phospholipids, 75% phosphatidykholine (lecithin) and 10% phosphatidylglycerol. The concemration of lecithin increases rapidly at the beginning of the third utmester and, thereafter, Jises rapid1y to reach a peak near term. Maturity of the fetal lung can be assessed by measuring pulmonary surfactam released imo the amniotic fluid. A lecithin-to-sphingomyelin ratio above 2:1 implies t.hat the fetus will survive \Vit.hom developing respiratory distress syndrome. Alpha-retoprotein (choice B) is used to monitor for anencephaly. ***Diagnosis: Ery1hroblasLOsis fetalis
38 A 3D-year-old pregnam woman visits her obstetrician for prenatal care and eventual delivery The patiem volunteers that two of her three children had "yellow jatmdice" at birth. Her youngest girl had been severely jaundiced and had been given two blood transfusions. Prenatal laboratory tests indicate that the mother is blood type 0, Rh negative, whereas her husband is blood rype A, Rh positive. The obstetrician samples amniotic fluid aL 36 weeks of gestation to ascertain whether the fetus is rnamre enough for pretem1 delivery Quantitative analysis of which of the fo llowing was most likely used as an indicawr of fetal lung maturity? ***(A) Absorbance aL 450 nm ***(B) AJpha-ferop rmein ***(C) Creatinine ***(D) l ecithin ***(E) Total protein
39 The answer is B: Congestive heart failure. Erythroblastosis fetalis is a hemolytic disease of the newborn caused by maternal antibodies against fetal erythrocytes. Erythroblastosis fetalis does not ordmarily occur dUJing the first pregnanC)', because the quamity of fetal blood necessary to sensitize the mmher is imroduced imo her circulation only at the dme of delivery. too late lO affect the ferus. However, when the sensitized mother again bears an Rh-positive fetus, much smaller quantities of fetal D antigen can elicit an increase in lgG antibody titer. This cycle is exaggerated in multiparous women, and the sevetity of erythroblastosis tends to increase progressively with each succeeding pregnancy. However, even after multiple pregnancies, only 5% of Rh-negative women are ever delivered of infants with ef)'lhroblastosis fetalis. The severily of etythroblastosis fetalis varies from a mild hemolysis to fatal anemia, and the pathological findings are det.ennined by the extem or the hemolytic disease. Hydrops fe taHs refe rs to the most serious form of etythroblastosis fetalis, and is characterized by severe edema secondary to congestive heart failure caused by severe anemia. The od1er choices do not cause of anasarca in erythroblastosis fetal is. ***Diagnosis: Erythroblastosis fetalis, hydrops fetalis
39 The patient described in Question 38 delivers a female at 37 weeks of gestation with evidence of severe generalized edema (neonate shown in the image)_ The baby is given exchange transfusions with Rh-negmive cells but subsequently dies. vVhich of the follov.l}.ng best describes the pathogenesis of anasarca in thi.s baby? ***(A) BiJirubin encephalopathy ***(B) Congestive heart failure ***(C) Nephrotic syndrome ***(D) Respiratmy distress syndrome ***(E) Rupture of the liver
40 The answer is D: Spontaneou.s resolution. Birth injury spans the spectmm from mechanical trauma to anoxic damage. Some birth injuries relate to poor obstetric manipulation, whereas many are unavoidable sequelae of routine delivery. Binh inj uries occur Ln about 5 per 1,000 live births. Factors that precUspose to birth injury include cephalopelvic disproportion, dystocia (dLfficultlabor), prematurity, and breech presentation. Cephalohemmoma is defined as a subperiosteal hemorrhage that is confined to a single cranial bone and becomes apparem \vithin the first few hours after binh. It may or may not be assodated wilh a linear fracture of the underlying bone. Most cephalohemawmas resolve without complicmion and require no Lreatmem. ***Diagnosis: Cephalohemawma
40 An H)-year-old woman delivers a male neonate following a difficult labor and delivery (dystocia). The baby appears vigorous at binh (Apgar score = 9), but a cephalohematoma is apparent 2 hours later. ACT scan of the baby's head shows subperiosteal hemonhage over one of the calvarial bones. What is the most likely outcome of this complication of labor and delivery? ***(A) Facial nerve palsy ***(B) Kernkterius ***(C) Respiratory distress syndrome ***(D) Sponlaneous resolution ***(E) Subaraclmoid hemorrhage
41 The answer is C: Encephalopathy. Kemicte:rus, also termed bilirubin encephalopathy, is defined as a neurological condition associated with severe jaundice and characterized bv bile . ' staining of the brain, particularly o[ the basal ganglia, pontine nuclei, and dentate nuclei in the cerebellum. Kernicterus (German: kem , nucleus) is essentially confined to newborns with severe unconjugated hyperbilirubinemia, usually related to erythroblastosis fetalis. The bilirubin derived from the desuuction of erythrocytes and the cambolism of the released heme is not easily conjugated by the immamre liver, which is deficient in glucuronyl transferase. The developmem of kemictems is directly related w the level of unconjugaLed bilirubin and is rare in tem1 infants when serum bilirubin levels are below 20 mg/ dL Premamre infants are more vulnerable w hyperbilimbinemia and may develop kernictems at levels as low as 12 mgldL Bilimbin is thought w injure the cells of the brain by imerfe1ing with mitochondrial function. Severe kernicterus leads initialJy w loss of the stanle reflex and athetoid movements, which in 75% progresses to letl1argy and death. Most su1viving infants have severe choreoathetosis and mental retardation~ a minority have varying degrees of intellectual and motor retardation. Exchange transfusions may keep the maximum serum bilimbi11 at an acceptable level. However, phototherapy. which converts the to;.dc unconjugated bilirubin into isomers that are nontoxic and ~xcreted in the urine, has greatly reduced the need for exchange LTansfusions. The other chokes are not complications of untreated hyperbilimbinemia in newborns. ***Diagnosis: Kernic terus, physiological jaundice
41 A 42-year-old woman in her third pregnancy delivers a female neonate at 30 weeks of gestation. The baby develops jaundice within 2 days. The unconjugated serum bilirubin is 15 mgldL Which of the following is the most serious complication of untreated hyperbilirubinemia in this neonate? ***(A) Acme pancreatitis ***(B) Bronchopulmonal)' dysp lasia ***(C) Encephalopathy ***(D) Gallstones ***(E) Hemolytic anemia
42 The answer is B: Embryonic lethality. lf a conceptus is exposed to harmful exogenous lnnuences, the noxious agent exerts the same effect on all blastomeres and also causes death . Thus, either a conceptus dies or development proceeds unintenupted, since the interchangeable blastomeres replace lhe loss. As a rule, exogenous toxins acting on preimplantationstage embryos do not produce errors of morphogenesis and do not cause malformations. The most common consequence of toxic exposure at the preimplantation stage is death of lhe embryo, which often passes unnoticed or is perceived as heavy, albeit delayed, mensm1al bleeding. Approximately 30% of fenilized ova are aboned spontaneously, without the woman being aware that pregnancy had occurred. Placenta accreta (choice C) is an abnormal adherence of the placenta to the underlying uterine walL The other choices are errors of morphogenesis that rnanifesr at later stages of development. ***Diagnosis: Spontaneous abortion
42 A 27-year-old woman presents for a pregnancy test. She Tecalls drinking heavily during the week in v,rhich she may have conceived. What is the most likely consequence of toxic exposure to the conceptus during early (preimplantation) development? ***(A) Conjoined £wins ***(B) Embryonic lelhalily ***(C) Placenta accreta ***(D) Neural tube defect ***(E) Vemricular-septal defect
"42 The answer is B: Embryonic lethality. If a conceptus is exposed to harmful exogenous influences, the noxious agent exerts the same effect on all blastomeres and also causes death. Thus, either a conceptus dies or development proceeds uninterrupted, since the interchangeable blastomeres replace the loss. As a rule, exogenous toxins acting on preimplantation- stage embryos do not produce errors of morphogenesis and do not cause malformations. The most common consequence of toxic exposure at the preimplantation stage is death of the embryo, which often passes unnoticed or is perceived as heavy, albeit delayed, menstrual bleeding. Approximately 30% of fertilized ova are aborted spontaneously, without the woman being aware that pregnancy had occurred. Placenta accreta (choice C) is an abnormal adherence of the placenta to the underlying uterine wall. The other choices are errors of morphogenesis that manifest at later stages of development. ***Diagnosis: Spontaneous abortion
1 A 60-year-old man with a histmy o f multiple myocardial L...- infarcts is hospi talized for shortness of breath. Physical examination reveals marked j ugular distension, hepatomegaly, ascites, and pitting edema. A chest X- ray reveals cardiomegaly. The patient subsequently dies of cardiorespiratory failure . Examination of the lungs at autopsy would most likely disclose which of the following pathologic changes? ***(A) Diffuse alveolar damage with hyaline membranes ***(B) Intra-alveolar pumlem exudate ***(C) Lymphocytic interstitial pneumonitis ***(D) Pulmonary aneriopathy with plexifmm lesions ***(E) Vascular congestion and hemosiderin-laden macrophages
1 The answer is E: Vascular con.gestion and hemosiderin-laden ......_ manophages. Left ventricular failure leads Lo chronic passive congestion of the lungs. Blooclleaks from the congested pulmonary capillaries imo the alveoli. Alveolar macrophages degrade RBCs and accumulate hemosiderin. These hemosidetin-laden macrophages are called heart failure cells. Diffuse alveolar damage with hyaline membranes (choice A) is a feature of adult respiratmy distress syndrome. Pumlent exudate (choice B) is observed in bacte1ial pneumonia. Lymphocytic interstitial pneumonitis (choice C) is characteristic of viral pneumonitis. Plexiform lesions (choice D) are typically seen in patien ts with pulmonary hypenension. ***Diagnosis: Congestive hean failure, pulmonary edema
2A 92-year-old woman is brought unconscious to the emergency room from a nursing home. Her blood pressure is 70/30 mm Hg. She is febrile (38°C/l00.5°F) and tachypneic. Laboratory studies demonstrate a WBC count of 22,000/)..tl with 92% neun·ophils. U1inalysis reveals numerous Gram-negative organisms. Which of the folJowing most likely accounts for this patient's signs and symptoms? ***(A) Anaphylactic shock ***(B) Cardiogenic shock ***(C) Hypovolemic shock ***(D) Neurogenic shock ***(E) Septic shock
2 The answer is E: Septic shock. Septic shock results from a systemic inflammatory response syndrome that leads to multiple organ dysfunction and hypotension. Clinical features include two or more signs of systemic inflammation (e.g., fever, tachycardia, tachypnea, leukocytosis, or leukopenia) in the setting of a known cause of inflammation. These processes often p rogress to multiple organ dysfunction syndrome in critically Lll patients. Septicemia with Gram-negati.ve organisms is the most common cause of septic shock Anaphylactic shock (choice A) occurs as a consequence of a systemic type I hypersensitivity reaction. Neurogenic shock (choice D) can follow acme injury to the brain or spinal cord , which impairs Lhe neural control of vasomotor wne, leading to generalized vasodilation. Cardiogenic shock (choice B) is a feamre of advanced hcan fai.lure. Hypovolemic shock (choice C) occurs fo llmving blood loss. ***Diagnosis: Septic shock
3 A 21-year-old pregnam woman expe1iences abruptio placentae at 37 weeks o[ gestation and develops severe vaginal bleeding that is difficult to control. Five momh.o:; later. the patient presents with pro[ound lethargy. pallor, muscle weakness, failure of lactation, and amenonhea. Which of the following best o..'Plains the pathogenesis of pituitary insufficiency in this patient? ***(A) Abscess ***(B) Embolism ***(C) In[arction ***(D) Passive hyperemia ***(E) Thrombosis
3 The answer is C: Infarction. Hypotension caused by postpartum bleeding can , in rare cases, lead to infarction of the #NAME? because its enlargement during pregnancy renders it vulnerable to a reduction in blood !1ow. None of the other choices cause clinical featu res of pan-hypop ituitaJism. ***Diagnosis: Sheeh an syndrome, pituitary infarction
4 A 62-year-old man with a h]story of hypertension is rushed .....__ LO the emergency room with severe "tearing pain" of the anterior chest. His blood pressure is 80/50 mm Hg. Physical examination shows pallor, diaphoresis, and a murmur of aonic regurgitation. Laboratory studies and ECG show no evidence of acute myocardial infarction. Four hours later, the patient goes into cardiac arrest. An ECG reveals dectromechanicaJ dissociation. Which of the foUowing best explains the pathogenesis of cardiac tamponade in this patient? ***(A) Disseminated intravascular coagulation ***(B) Embolism ***(C) Hemorrhage ***(D) Passive hyperemia ***(E) Thrombosis
4 The answer is C: Hemorrhage. Pericardia] J1uid may accu- .__ mulate rapiclly, panicularly •.vith. hemorrhage caused b>• a n.tptured myocardial infarct, dissecting aonic aneurysm (seen in this patient), or trauma. In these circumstances, the pressure in the pericardia] cavity exceeds the filling pressure of the he an, a condition tetmed cardiac tamponade. The term e lectromechanical dissociation refers to a heart rhythm that should produce a pulse, bUL does not. The most common cause of this condiuon is hypovolemia The resulting precipitous decline in cardiac ompur is often fa tal. The pathogenesis of dissecring aortic aneurysm in most instances can be tTaced w a weakening of the aonic media (cystic media] necrosis). Most patients have a history of hypertension. Disseminated intravascular coagulation (choice A) refers to widespread ischemic changes seconda1y to microvascular thrombi. Passive hyperemia (choice D) refers Lo the engorgement of an organ vvith venous blood. ***Diagnosis: Dissecting aonic aneurysm
5 A 58-year-old woman is brought to the emergency department 4 hours after vomiting blood and expe1iencing bloody stools. The pariem was diagnosed with alcoholic cin·hosis 2 years ago. Endoscopy reveals large esophageal varices, one of which is actively bleeding Which of the following best explains the pathogenesis of dilated esophageal veins m this patient? ***(A) Decreased intravascular oncotic pressure ***(B) Increased capilla~y pem1eability ***(C) Increased intravascular hydrostatic pressure ***(D) Vasoconstriction of arterioles ***(E) Vasodilatation of capillaries
5 The answer is C: Increased intravascular hydrostatic pressure. This patient wilh alcoholic cirrhosis has ponal hypenension (increased hydrostatic pressure) and bleeding esophageal valices. Massive hematemesis is a fTequent cause of death in patients with esophageal va1ices. Decreased intravascular oncotic pressure (choice A) contributes to the development of ascites in patients with cirrhosis but not to the development of esophageal valices. ***Diagnosis: Esophageal varices, hematemesis
6 A 69-year-old retired man is brought to the emergency department because of the sudden onset of left-sided chest pain, which is exacerbated upon inspiration. Physical examination reveals dyspnea and hemoptysis. His temperamre is 38°C (101 °f), pulse 110 per minute, respirations 35 per minute, and blood pressure 158/100 mm Hg. A lateral chest wall friction rub is p resent on auscultation. The left leg is markedly edematous with a positive Homans' sign. A chest X-ray reveals a left pleural effusion. What is the most likel)' cause of this patiems pulmonary condition? ***(A) Congesdve heart failure ***(B) Cor pulmonale ***(C) Mitral stenosis ***(D) Subacute endocarditis ***(E) Thromboembolism
6 The answer is E: Thromboembolism. This patiem with mild congestive hean faLlure developed pulmonary embolism . Small pulmonary emboli rarely cause infarctions because of the dual blood supply 1.0 the lungs and because oxygen can diffuse from the alveoli into lung tissue. Symptoms depend upon the extent of blockage of the pulmonaxy anerial tree , whether there is already cardiopulmonary disease, and whether pulmonary infarction occurs. The othe r choices do not induce these pleural signs and sympwms. ***Diagnosis: Pulmonmy thromboembolism
7 A 22-year-old cotL.c;truction worker falls 30 ft and fracmres .__ several bones, including his femoral shafts. Six hours later, the patiem develops shortness of bream and cyanosis. Which of me following hemodynamic disorders best explains the pathogenesis of shock in this patient? ***(A) Acute myocardial infarction ***(B) Deep venous thrombosis ***(C) Fat embolism ***(D) Paradmacal embolism ***(E) Septic shock
7 The an.swer is C: Fat embolism. Fat emboli origmate from adipose tissue in the medulla of fTacrnred long bones. Fat carried by venous blood reaches the lungs, filters through 1.he pulmonary circulation, enters anerial blood, and is disseminated throughout the body. The occlusion of cerebral capillaries is accompanied by petechial hemorrhages in the brain and is the most important complication of fat embolism. Acme myocardial infarction (choice A) would be unlikely in a 22-year-old patient. Deep venous thrombosis (choice B) and septic shock (choice E) would be unlikely within this time frame. Paradoxical embolism (choke D) refers w emboll that mise in the venous circulation and bypass the lungs by rraveling through a11 incompletely closed foramen ovale, suhsequemly entering the arterial ci rculation. ***Diagnosis: Fat embolism
8 A 20-year-old woman presents to the emergency room complaining of having had a severe headache for 4 hours. Physical examination reveals numerous small red spots on Lhe e..'<LremiLies and a stiiT neck. Her temperamre is 38. 7°C (103°F). Lmnbar puncture returns purulent ftuid, with segmented neutmphiJs and Gram-negative organisms resembling meningococci. A few hours later, me patient goes into shock and becomes comatose. Severe endothelial injury in thls patient ts primarily mediated by which or Lhe following proteins? ***(A) 0.-Fetoprotein ***(B) lgG ***(C) Imerferon-y ***(D) Transfonning growth factor-~ ***(E) Tumor necrosis facwr-a
8 The answer is E: Tumor ne<rosis factor-a (TNF·a ). Septicemia with Gram-negative organisms is the most common cause of septic shock. The invading bacteria are responsible for the release of endotoxin, a lipopolysaccharide (l.PS). On entry into the circulation, LPS binds to the surface of monocyt.es/ macrophages. The CD 14 recognition comple.x mediates signaling through activation of nuclear transcliption factorkappa B (NF-KB) and upregulates the expression of TNF-a. In septic shock, this protein is released in great excess, resulting in effects that are often LethaL None of the other mediators cause severe injury to vascular endothelium in patients with septic shock. ***Diagnosis: Meningitis, septic shock
9 A 69-year-old man is brought to the emergency room complaining of visual difficulty and weakness. On physical examination, the patient is aphasic wi th a right-sided henliplegia. Retinal hemonhages are seen bilaterally. You suspect that a thromboembolus coursed to the left middle cerebral artery and smaller emboli traveled w the rednal arteries. Which of the follO\ving anatomic sites is the most likely source [or these emboli in this patient? ***(A) Adrenals ***(B) Deep leg veins ***(C) Heart ***(D) Uver ***(E) Lungs
9 The answer is C: Heart. The hean is the most common source of anetial thromboemboli, which usually arise from mural thrombi or diseased valves. These emboli tend to lodge at points where the vessel lumen narrows abmptly (e.g., at bifurcations or in the area of an aLherosclermic plaque). The viability of the tissue supplied by tl1e vessel depends on the availability of collateral circulation and on the fate of the embolus itself. Paradoxical emboli from the right side of the circuladon are exceedingly rare. ***Diagnosis: Cerebral embolism, stroke
10 The body of a 78-year-old homeless man is broughl to the coroners office. Histologic examination of the lungs under polmized light is shown. Which of l.he following is the most likely cause of the birefringence observed in this pulmonary lesion? ***(A) Alcoholism ***(B) Asptradon of mmeral oil ***(C) Baclcrial pneumonLa ***(D) Cocame abuse ***(E) Imravenous dmg use
10 The answer is E: Intravenous drug use. Embolism is the passage Lhrough the venous or anerlal circulations of any material capable of lodging in a blood vessel and, thereby, obstructing Its lumen. Intravenous drug abusers who use calc as a camer ror illicil drugs may introduce it into lhe lung via the bloodstream (i.e .. venous embolism). None of the other choices exhibit biref1ingence under polalizecllighL ***Diagnosis: Pulmonary embolism, talc embolism
11 A 25-year-old woman dellvers a healthy baby aL 39 weeks of gestation. Six hours later, the mother develops severe shortness of bre<uh and appeal'S cyanotic. Despite resuscitation, she dies 2 hours later. A section of lung at autopsy is shown in the image. These pathologic findings are associated with which of the following mechanisms of disease? ***(A) Amniotic Ouid embolism ***(B) Cardiogenic shock ***(C) Mal.emal-fetal histoincompatibility ***(D) Metastatic squamous cell carcinoma ***(E) Pulmonary thromboembolism
11 The answer is A: Amniotic fluid embolism. Amniotic fluid embolism refers to the entry of amniotic fluid containing fetal cells and debris imo the maternal ci rculaLion through open uterine and cervical veins. lt is a rare maternal complication of childbirth, but when it occurs, it is often catastrophic. This disorder usually occurs at the end of labor when the pulmonary emboli are composed of the epithelial consti.tuents (squamae) contained in the a1m1iotic iluid . None of the mher choices show these pathologic findings. ***Diagnosis: Amniotic fluid embolism
12 Lf the patient dese1ibed in Quesu on 11 had Sltrvived the acme episode of cyanosis and shock, she would have been at risk for developing which of the follov.ring life-threatening complications? ***(A) Bacterial endocarditis ***(B) Disseminated intr avascular coagulation ***(C) Fat embolism ***(D) Neurogenic shock ***(E) Septic shock
12 The answer is 8: Disseminated intravascular coagulation (DIC) . The clinical presentation of amniotic fluid embolism can be dramatic, with the sudden onset of cyanosis and shock, followed by coma and death. lf the mother survives the acute episode, she may cUe of DIC. Should she overcome this compli cation , she is at risk of developing acute respiratory disrress syndrome. DIC is a thrombotic microangiopathy. Fibrin thrombi form in small blood vessels because of uncontrollable coagulopathy, which consumes fibrin and other coagulation facwrs. Once coagulation factors are depleted, uncomrollable hemorrhage ensues. None of the OLher choices are complications of amniotic iluid embolism. ***Diagnosis: Amniotic fluid embolism
13 A 68-year-old man with ischemic heart disease and a history of smoking complains of increasing shormess of breath. On physical examination , the patient has swollen legs. an enlarged liver. and fluid m the pleural spaces (bubbly rales are heard on oscultation). Which of the following hemodynamic disorders explains the pathogenesis of heparomegaly in this patient? ***(A) Arterialthromboembolism ***(B) Chronic passive congestion ***(C) Deep venous thrombosis ***(D) Multiple hepatic infarcts ***(E) Thrombosis of the hepauc vein
13 The answer is B: Chronic passive congestion. A generalized increase in venous pressure, typically fTom chronic hean failure, results in an increase in the volume of blood in many organs (e .g., llver, spleen, and kidneys). The liver i.s panicular1y vulnerable to chrome passive conge.o:;oon because the hepatic veins empty into the vena cava immediately inferior ro the heart. Budd-Chi.ari syndrome (thmmbosis of the hepatic vein; choice E) may cause hepawmegaly. but it is nola complication of congestive heart failure. ***Diagnosis: Congestive hean failure, nuoneg liver
14 The pauem described in Question 13 suffers a massive heart auack and expires. ~1icroscopic examination of the liver at auwpsy would most likely reveal which of Lhe following histopathologic changes? ***(A) Diiiuse hydropic degeneration ***(B) Large iron deposits witl1in hepatocytes ***(C) Massive hepatic necrosis ***(D) Regenerating hepatic nodules surrounded by fibrous bands ***(E) Sinusoids dilated with blood
14 The answer is E: Sinusoids dilated with blood. In patients with chronic passtve congestion of the liver, the cemral veins of tbe hepatic lobule become dilated. The increased venous pressure leads to dilation of the sinusoids and pressure atrophy of the centrilobular hepatocytes. Grossly, the cm surface of the chronically congested liver exJ1ibits dark foci of cennilobular congestion surrounded by paler zones of unaffected peripheral ponions of the lobules. The result resembles a cross section of a nutmeg and is appropriately called nutmeg liver. Longstanding passive congestion leads to bridging fibrosis; however, only in the most extreme cases is the fib rosis sufficiently severe to justify the label cardiac cirrhosis (choice D). None of the other choices are associated v..'ith congestive heart failure. ***Diagnosis: Congestive hean failure, nurmeg liver
15 A 33-year-old woman presents with black stools. Laborawry sLUdies demonstrate a hypochromic, microcyLic anemia. Upper Gl endoscopy reveals a duodenal ulcer. Which of the following best describes the stools in this patient with peptic ulcer disease? ***(A) Hematemesis ***(B) Hematobilia ***(C) Hematochezia ***(D) Melena ***(E) Steatorrhea
15 The answe-r is D: Melena. Melena (black stool) is a sympwm of upper gastroime.slinal bleeding. Blood from ruptured esophageal va1ices or a peptic ulcer is partially digested by hydrochloric acid. Hemoglobin is transfom1ed into a black pigmem (hematin), which imparts a typical "coffee-grounds" color lO the stool. Hemalemesi.s (choice A) is vomiting of blood. Hematobilia (choice B) is bleeding into the billary passages, as a complication of trauma or neoplasia. Hematochezia (choice C) is passage of bloody stools caused by lower gasnointeslinal hemorrhage. Steatorrhea (choice E) is passage of fatty stools caused by pancreatic disease and malabsorption. ***Diagnosis: Peptic ulcer disease
16 A 53-year-old man is hospitalized after injuring his neck in an automobile accident. He is placed in cervical traction . One week later, the patiem develops painful swelUng and erythema of his left calf. Doppler imaging discloses deep venous thrombosis. Wh ich of the following is the most likely cause for the developmem of thrombosis in this patient? ***(A) Age ***(B) Endothelial damage ***(C) Hypercoagulability ***(D) Infection ***(E) Stasis
16 The answer is E: Stasis. Venous thrombosis is caused by the same factors that predispose to arterial tlu·ombosis, namely endothelial injUJy, stasis, and a h)'percoagulable state. Although all of the choices are 1isk factors for deep venous thrombosis, the most likely choice, given the patients' immobilization, is stasis. Most venous thromboses occur in the deep veins of the legs. ***Diagnosis: Deep venous thrombosis
17 A 23-year-old man with hemophilia is recently wheelchair bound. Which of the following best accoums for this development? ***(A) Hemarthrosis ***(B) Hematemesis ***(C) Hematocephalus ***(D) Hematochezia ***(E) Hemoptysis
17 The answer is A: Hemarthrosis. Hemarthrosis refers to bleed· ing into the joint cavity. It is associated with joint swelling and is a crippling complication of hemophilia. Repeated bleeding may cause deformities and may bmit the mobility of the joints. Hematemesis (choice B) is vomiting blood. Hemawcephalus (choice C) is an intTacranial infusion of blood. Hematochezia (choice D) is passage o[blood caused by lower gastrointestinal hemonhage. Hemoptysis (choice E) is coughing up blood. ***Diagnosis: Hemophllia, hemarthrosis
18 A 50-year-old fi re fighter emerges from a burning house with third-degree bums over 70% of his body The patient expires 24 hours later. Whkh of the following was the most likely cause of death? ***(A) Congestive hean failure ***(B) Disseminated inrravascular coagulation ***(C) Hypovolemic shock ***(D) Pulmonary saddle embolism ***(E) Toxic shock syndrome
18 The answer is C: Hypovolemi( shock. Hypovolemic shock m.ay be caused by hemorrhage, fluid loss [rom severe burns, diarrhea, excessive mine formalion, perspiration, or Lrauma. In the case of burns or trauma, direct damage to the microcirculation increases vascular pem1eabilit:y. Persons with third-degree burns weep large amounts of plasma. The oilier choices are unlikely causes o£ death in an acme bum VIC[im. ***Diagnosis: Hyperthermia, hypovolemic shock
19 A 23-year-old woman complains of a recent 011Sel of yellowing of her skin and increasing abdominal girth. Physical examination reveals jaundice and ascites. Ultrasound examination of her abdomen demonstrates thrombosis of the hepatic veins. A liver biopsy discloses severe sinusoidal dilation ·within the cemril obular regions. This patholOgiC finding is caused by which of the following hemodynamic disorders? ***(A) Active hyperemia ***(B) Arterial embolism ***(C) Hematoma ***(D) Hemonhage ***(E) PassiVe hyperemia
19 The answer is E: Passive hyperemia. Passive hyperemia (chronic passive congestion) may be confined to a limb or an organ as a result of locaJ ized obsuuction w venous drainage. E-xamples include deep venous thrombosis of the leg veins, with resulting edema of the lower extremity, and thrombosis of the hepatic veins (Budd-Chiari syndrome, this patient) with secondary chronic passive congestion of the liver. AcLive hyperemia (choice A) is an augmemed supply of blood loan organ, usually as a physiologJc response to an increased functional demand. The most striking active hyperernia occurs in association with inflammation. Arterial embolism (choice B) typically causes infarction. Hematoma (choice C) and hemorrhage (choice D) represent extravascular accumulation of blood. ***Diagnosis: Budd-Chiari syndrome
ZO A 42-year-old woman undergoes a face lift. Two days later, she presents for follow-up care with confluent bluish hemorrhages in the skin around her eyes ("black eyes"). ·which of the following best describes this pattern of superficial skin hemorrhage? ***(A) Ecchymosis ***(B) Hematocephalus ***(C) Maculopapular rash ***(D) Peted:tiae ***(E) Purpura
20 The answer is A: Ecchymosis. Ecchymosis is a larger superficial hemorrhage in the skin. Following hemorrhage, the initially purple discoloralion of the skin tunlS green and then yellow before resolving. This sequence of events reflects Lhe progressive m.::idation of biJimbin released from lhe hemoglobin of degraded erythrocytes. A ''black eye·· is a good example of an ecchymosis. Petechiae (choice D) are pinpoint hemorrhages, usually in tJ1e skin or conjunctiva. Purpura (choice E) is a diffuse superficial hemorrhage in the skin up to 1 em in diameter. ***Diagnosis: Ecch>rmosis
21 A 19-year-old woman complains of swelling of her eyelids, abdomen, and ankles. At bedtime, there are depressions in her legs at the location of the elastic in her socks. A chest X-ray shows bil ateral pleural effusions. U1ine protein electrophoresis demonsu·ates 4+ proteinuria. A percutaneous needle biopsy of the kidney establishes the diagnosis of minimal change nephrotic sy11drome. Soft tissue edema in this patient is most likely caused by which of the following mechanisms of disease? ***(A) Active hyperemia ***(B) Chron ic passive congestion ***(C) Decreased imravascular oncotic pressure ***(D) Hyperalbuminemia ***(E) Increased capillary permeability
21 The answer is C: Decreased intravascular oncotic pressure. The pressure differential bei:Ween the intravascular and the lmersddal compartments is largely determined by the concemration of plasma proteins, especially albumin. Any condition that lowers plasma albumin levels, whether from albuminuria in nephrotk syndrome or reduced albumin synthesis in chronic hver disease, tends to promote generalized edema. ***Diagnosis: Minimal change neplu-otic syndrome
22 A 50-year-old alcoholic is rushed to the hospital wiLh bleeding esophageal varices and expires. At auwpsy, the patients protruding abdomen is [ound w contain a large volume of serous fluid. What is the appropriate term used to desctibe this fluid? ***(A) Ascites ***(B) Exudate ***(C) HemotThage ***(D) Hydrothorax ***(E) lymphedema
22 The answer is A: Ascites. A protruding belly and fluid accumulation in palients with cirrhosis represents ascites (i.e., accumulation of serous O.uid in the abdominal cavity). lt is primarily a consequence of portal hypertension and hypoalbuminernia. None of the other choices describe serous Ouid accumulation in the abdomen. ***Diagnosis: Cirrhosis, portal hypertension
23 A 1-year-old girl is brought to the emergency room by her parenLS who report she has had a fever and diarrhea for 3 days. Her temperature is 38°C (101 "f). The CBC shows a nom1al WBC count and increased hematocrit ( 48 gtdL). Which of the following is Lhe most likely cause of tncreased hematocrit in this patiem7 ***(A) Acute phase response ***(B) Dehydration ***(C) Diabetes insipidus ***(D) Malabsorption ***(E) Septic shock
23 The answer is B: Dehydration. lncreased hematocrit in this patient reflects hemoconcentration caused by dehydration, secondary to diarrhea. This hematologic condiLion, termed relative polycythemia, is charactetized by decreased plasma volume with a nonnal red cell mass. When patiems suffer from bums, vomiting, excessive sweating, or diarrhea, they not only lose Ouid but also suffer electrolyte disturbances. Systemic blood pressure falls with continuous dehydration, and declining perfusion eventually leads to death. Diabetes insipidus (choice C) may cause dehydration bm is an unlikely chmce because the patient has a hiswry of diarrhea. None of th e other choices cause relative polycythemia. ***Diagnosis: Dehydration, relative polyc}rt.hemia
24 A 40-year-old man with a history of bac£erial endocarditis notices numerous pinpoint hemorrhages around the orbit of his eyes (shown in the image; see arrows). 'vVhat is the approprime term used to describe this fo rm of superficial hemorrhage? ***(A) Ecchymosis ***(B) Erythema ***(C) Hyperemia ***(D) Petechia ***(E) Purpura
24 The answer is D: Petechia. Petechiae are pinpoint hemorrhages, usually in the skin or conjunctiva. TWs lesion represems the rupture of a capillary or aneri ole and occurs in conjunction with vasculitis and coagu lopath)~ Petechiae may also be produced by microemboli from infected heart valves (bacte1ial endocarditis). Hyperemia (choice C) refers to increased blood in a tissue or organ. Erythema (choice B) is inAammatory redness of the skin. Ecchymosis (choice A) is a larger superficial hemorrhage in Lhe skin. Purpura (choice E) is a diffuse superficial hemorrhage in the skin up to l em m diameter. ***Diagnosis: Endocarditis, petechia
25 A 67-year-old man presents \IVith sudden left leg pain, absence of pulses, and a cold limb. His past medical history is significant for coronary anery disease and a small aonic aneurysm. Which of the folto\IVing is most likely responsible for development of a cold limb in this patient? ***(A) Acute myocardial infarction ***(B) Aaerialthromboembolism ***(C) Cardiogenic shock ***(D) Deep venous thrombosis ***(E) Ruptured aonic aneurysm
25 The answer is 8: Arterial thromboembolism. Embolism of an artery of the leg leads to sudden pain, absence of pulses, and a cold limb. ln some cases, the limb must be amputated. None of the other choices would cause Lhis clinical presemation. Rupmred aonic aneurysm (choice E) presents with pain, shock, and a pulsatile mass in the abdomen. ***Diagnosis: Arterial thromboembolism
26 A 78-year-old woman dies in her sleep. A Prussian blue stain of the lungs at autopsy is shown in the image. Which of the following is the most likely cause or these histopathologic findings? ***(A) AcULe myocardial infarction ***(B) Congestive hean failure ***(C) Diffuse alveolar damage ***(D) Hereditaty hemochromatosis ***(E) Pulmonary infarction
26 The answer is 8: Congestive heart failure. Chronic failure of the left ventricle constitutes an impediment to the exit or blood from the lungs and leads to chron ic passive congestion of the lungs. The pressure in the alveolar capHlaries increases, and Lhe vessels become engorged with blood . Microhemorrhages release erythrocytes into the alveolar spaces, where they are degraded by alveolar macrophages. The released iron, in the form ofhemosiderin, remains in Lhe macrophages, which are then labeled "heart failure cells." None of the other choices are consistent with chronic microhemorrhages in the lung. ***Diagnosis: Congestive hean failure
27 A 60-year-old man who is recovering from surge1y w conect an abdominal aneurysm suddenly develops acme chest pain and dies. A thromboembolus at the bifurcation of the left and 1ighr pulmonary ane1ies is nmed at autopsy (shown in the image). Which of the following is the most likely cause of this patients pulmonary embolus? ***(A) Bacterial endocarditis ***(B) Complkated atherosclerotic plaque ***(C) Deep venous thrombosis ***(D) Parado:>:ical embolizalion ***(E) Right ventJicular mural thrombus
27 The answer is C: Deep venous thrombosis. One of the most tragic calamities complicating hospilalization is the sudden death or a patienl who appeared to be on the way to recovery. The cause of this catastrophe is often massive pulmomny embolism. A Large pulmonary embolus may lodge at the bifurcation of the main pulmonaty artery (saddle embolus), obstruning blood flow to both lungs. With acute obstruction of more than half or the pulmonary arterial tree, the patient e>..'Periences severe hypotension and may die \vithin minutes. The other choices are causes of anerial embolism. ***Diagnosis: Pulmonary thromboembolism
28 A 20-year-old man is brought to the emergency room after mpturing his spleen in a motorcycle accidem. His blood pressure on admission is 80/60 mm Hg. Analysis of arterial blood gasses demonsrrates metabolic acidosis. This padem is most likely suffering from which of the following conditions? ***(A) Acme pancreatitis ***(B) Cardiogenic shock ***(C) Hypersplemsrn ***(D) Hypovolemic shock ***(E) Sepdc shock
28 The answer is D: Hypovolemic shock. Hypovolemic shock is secondary w a pronounced decrease in blood or plasma volume, caused by the loss of Ouid from the vascular compartment. Hemonhage, fluid loss from severe burns, diarrhea, excessive urine fom1ation, perspiration , and trauma are major mechanisms of fluid loss that can lead to hypovolemic shock. Cardiogenic shock {choice B) is caused by myocardial pump failure. Septic shock (choice E) is improbable in thjs seui.ng. ***Diagnosis: Hypovolemic shock
29 A 72-year-old man is dead on arrival after collapsing at home. Rena] cortical infarcts are noted ar autopsy. A section through the arcuate anery is shown. Which of the following is the most likely source of the atheroembolus occluding this anery? ***(A) Abdominal aorta ***(B) Common carotid anery ***(C) lnferior vena cava ***(D) Left vennicle ***(E) Mesenteric artery
29 The answer is A: Abdominal aorta. ln patients \'lith severe aortic atherosclerosis, embolization of atheromatous debris into the renal aneries and vascular tree may cause acme renal failure. Cholesterol clefts are observed in the photomicrograph shown. None of the other choices are sources of renal atheroemboli. ***Diagnosis: Renal infarct, anerial embolism
30 A 72-year-old woman complains of shormess of breath on exenion . She states that she also becomes shon of breath at night unless she uses three pillows (orthopnea). Phys1cal examination reveals mild obesity. bilateral pitting leg edema, an enlarged liver and spleen, and fine crackling sounds on inspiration (rales). A chest X-ray shows cardiomegaly. What is the most likely cause of orthopnea in this patient? ***(A) Asthma ***(B) Cardiac tamponade ***(C) Emphysema ***(D) Hypovolemic shock ***(E) Pulmonary edema
30 The answer is E: Pulmonary edema. Patients in left~sided congestive heart failure complain of shormess of breath (dyspnea) on exertion and when recumbent (orthopnea). They may be awakened from sleep by sudden eptsodes of shortness of breath (paroxysmal nocturnal dyspnea). Physical examination usually reveals distended jugular veins. Persons with right-sided fallure have pining edema of the lower extremities and an enlarged and tender Liver. Patients in congestive heart failure with pulmona1y edema have crackling breath sounds (rales) caused by the expansion of fluid-filled alveoli. Cardiac tamponade (choke B) occurs when the pressure in the pericardia] cavity rises to exceed the filling pressure of the hean. Orthopnea is nOLa feature of the other choices. ***Diagnosis: Congestive heart failure
31 A 63-year-old man suffers a massive s troke and expires. At autopsy, the pathologist finds a laminated thrombus adherent to the wall of Lhe left ventr icle (shown in the image). Which of the following is the most likely cause of this autopsy finding? ***(A) Atrial fib1illation ***(B) Bacterial endocarditis ***(C) Marantic endocarditis ***(D) Myocardial infarction ***(E) Viral myocarditis
31 The answer is D: Myocardial infarction. Myocardial infarction is the most common cause of mural thrombi in the left ven~ tricle. These mural thrombi are a common source of arterial thromboemboli. Such emboli may occlude cerebral arteries and cause cerebral infarcts, known clinically as strokes. Atrial fibrillation (choice A) predisposes to the formation of mural thrombi in that location. ***Diagnosis: Mural thrombus
32 Histologic examination of the heart in the patient described in Question 31 shows extensive growth of fibroblasts and deposition of collagen in Lhe mural thrombus. Which of the following terms describes this outcome of thrombosis? ***(A) Can alization ***(B) Hyaltnizalion ***(C) Organization ***(D) Propagation ***(E) Regeneration
32 The answer is C: Organization. Once fo rmed, arterial thrombl may undergo (1) lysis, (2) propagation, (3) organization, (4) canalization, or (5) embolization. Organization refers to the invasion of connective tissue elements, which causes a Lhrombus to become firm and appear grayish white. Canalization (choice A) is the process by which new lumina lined by endothelial cells fonn within an orgarLized thrombus. Propa~ galion (choice D) implies an increase in size. ***Diagnosis: Mural thrombus
33 A 50-year-old woman presents with fatigue and shortness of breath. Physical examination shows evidence of pulmonaty edema, enlargement of the left atrium, and calc.ification of the mitral valve. ACT scan demonsmues a large obstructing mass in the left atrium. Before open heart surgery can be perfom1ed to remove the mmor, the patient suffers a stroke and expires. Which of the foiiO\~ng hemodynamic disorders best expla ins the pathogenesis of stroke in this patient? ***(A) Arterial embohsm ***(B) Atherosclerosis ***(C) Card10genic shock ***(D) Hypertensive hemorrhage ***(E) Septic shock
33 The answer is A: Arterial embolism. Cardiac myxoma is the most common p1imary tumor of the heart. One third or patients vlith a left atrial or left ventricular myxoma die from tumor embolization to the brain. Less likely causes of stroke in this patient with a cardiac myxoma include atherosclerosis (choice B) and hypertensive hemonhage (choice D). ***Diagnosis: Cardiac myxoma
34 A 50-year-old woman appears at your office. She was subjected to radical mastectomy and axillary node dissection for breast cancer a year ago. $he now notices that her arm becomes 5\Vollen by the end of the day. What is the appropti· ate name for this rruid accumulation? ***(A) Chylothorax ***(B) Hydrothorax ***(C) Lymphedema ***(D) Pumlem e.>..'1.ldate ***(E) Fibrinous exudate
34 The answer is C: Lymphedema. Obsuuction o£ lymphatic now may occur in a number of clinical settings, but is most common because o£ surgical removal of lymph nodes or rumor obsnuction. For example, the lymphatic system may be obstructed after axillary lymph node dissection for breast cancer. Prolonged lymphatic obstruction in the patient's shoulder causes edema, progressive dilation of lymphatic vessels (lymphangiectasia), <mel overgrowth of fibrous tissue. Lymp· hangiosarcoma has also been described. Chylothorax (choice A) represents an accumulation of lymphedema in the pleura] space. Exudates (choices D and E) are associated witJ1 acute innammation. ***Diagnosis: Lymphedema, breast cancer
35 A 68-year-old man develops sudden, severe substernal chest pain. Laboratory smdies and ECG corlfinn an acute myocardial infarct. Despite vigorous rherapy, the palient cannot maintain his blood pressure and expires 24 hours later. A cross section of £he lefL vemricle is examined at amopsy (shown in the image). The arrows point w a soft, yellow area of necrosis. Which of the following was the most likely cause of death? ***(A) Cardiogenic shock ***(B) Hypovolemlc shock ***(C) Neurogenic shock ***(D) Septic shock ***(E) Pulmonary edema
35 The answer is A: Cardiogenic shock. Cardiogenic shock is caused by myocardial pump failure. This condition usual ly arises as a result of a large myocardial infarction, but myocarditis may also be responsible. Conditions that prevent left or right hean 61Ung reduce cardiac output, resulling in obstructive shock. Such conditions include pulmonary embolism, cardiac tamponade, and (rarely) aLrial myxoma. The other choices do not reflect a loss of cardiac output secondary w the loss of myocardial tissue owing LO ischemia. ***Diagnosis: Acute myocardial infarction
36 An 80-year-old woman with a history of hypertension is rushed to Lhe emergency room complaining of chest pain of 1-hour duration. Physical examination discloses bilateral pirLing leg edema, hepatosplenomegaly, and rates at 1.he bases of bOLh lungs. The pariem is apprehensive and sweating The patient loses consciousness and dies of a cardiac arrhyrlm1ia. Microscopic examination o[ the lungs at auropsy is shown. Which of the following hemodynamic processes best explains this pathologic finding? ***(A) Decreased capillary permeability ***(B) Decreased intravascular oncolic pressure ***(C) Increased intravascular hydrostatic pressure ***(D) Increased inu·avascular oncolic pressure ***(E) Vasoconstriction of precapillary arterioles
36 The answer is C: Increased intravascular hydrostatic pressure. In patients with congestive heart failure , venous engorgement of the lung leads to accumulation of a transudate in the alveoli. Chronic left venuicle failure impedes blood flow out of the lungs and leads to passive pulmonary congestion. As a result, pressure in the alveolar capillaries increase (increased hydrostatic pressure) and these vessels become engorged with blood. Increased pressure forces fluid from d1e blood into the alveolar spaces, resulting in pulmonary edema, which interferes with gas exchange. The photomicrograph shows pink staining fluid in the alveoli. None of the other choices cause pulmonary edema in patients with congestive hean railure. ***Diagnosis: Pulmonary edema, congestive heart failure
37 A 9-momh-old infant is brought to the emergency room with a 3-hour history of intense abdominal pain and bloody diarrhea. Physical examination reveals a tender abdomen without ascites. The child dies 24 hours later, and torsion (volvulus) of the small bowel is discovered at autopsy. The small bowel appears dilated and hemorrhagk (shown in the image). Wl1ich of the following best describes these pathologic findings? ***(A) Ecchymosis ***(B) Infarct ***(C) Petechia ***(D) Purpura ***(E) Ulcer
37 The answer is B: Infarct. Volvulus is an example of intestinal obsu·uction in which a segment of gut twists on its mesentery, thereby kinking the bowel and usually interrupting the blood supply. lschemia leads to infarction and intestinal gangrene (this case). Volvulus is vinuaL\y always a consequence of an underlying congenital abnom1ality. Defective intestinal rotation in fetal life leads w abnom1al positions of the small intestine and colon, anomalous auachments, and bands. The clinical impon ance of such rotational anomalies lies in their propensity to cause catastrophic volvulus of the small and large intestine and incarceration of the bowel in an internal hernia. Malrotation of the bowel permits undue mobility of the bowel loops and predisposes to midgut volvulus. When the cecum or right colon is invested with a mesente1y rather than bei:ng retroperitoneal, the resulL may be cecal volvulus. An unusually long sigmoid colon, which occurs sometimes in patients wi Lh idiopathic constipation, permits the development of sigmoid volvulus. Meconium ileus in babies with cystic fibrosis may be complicated by volvulus and intestinal atresia. Ecchymosis (choice A), petechia (choice C), and purpura (choice D) represent hemorrhages of various sizes in the skin. ***Diagnosis: Volvulus, ischemic colitis
38 An autopsy of a 70-year-old woman reveals a subendocardial, circumferential infarct of the left ventricle. This type of infarct is most commonly associated with which of the following? ***(A) Deep venous thrombosis ***(B) Hypotensive shock ***(C) Pelicardial tamponade ***(D) Tl1rombotic occlusion of the right coronary artery ***(E) Il1rombotic occlusion of the circumflex anery
38 The answer is B: Hypotensive shock. Myocardial infarcts ;ue described as transmural (through the entire wall) or subendocardiaL A transmural infarct results from complete occlusion of a major extramural coronary artery Subendocardial infarction reflects prolonged ischemia caused by partially occluding lesions of the coronary aneries when lhe requirement for oxygen exceeds the supply. Such a siruation prevails in disorders such as shock, anoA'ia, or severe tachycardia. Thrombotic occlusion (choices D and E) is more likely to cause transmural myocardial infarcts. ***Diagnosis: Myocardial infarction
39 A 76-year-old woman is brought to the emergency departmem because of the sudden onset of two episodes of hemoptysis and left-sided chest pain, wh ich is exacerbated upon inspiration. Her temperamre is 38°C (101 °F), pulse 110 per minute, respirations 35 per minme, and blood pressure 158/100 mm Hg. The pariem is admitted, bur suffers a massive stroke and e>rpires 48 hours later. Auwpsy reveals a pulmonmy infarct in upper segments of the lower lobe (shown in the image). Which of the following best explains the color of this patients pulmonary infaTct? ***(A) Accumulation of hemosiderin-laden rnacrophages ***(B) Development of bronchopneumonia ***(C) Hernonhage from bronchtal anenes ***(D) Orgamzauon of a pulmonary thromboernbolus ***(E) Passi,•e congestion of bronchopulmonary segments
39 The answer is C: Hemorrhage from bronchial arteries. The gross and microscopic appearance of an infarct depends on its location and age. Pale infarcts are cypically seen in the hean, kidneys, and spleen. Red infarcts may result from either arterial or venous occlusion. They are distinguished from pale infarcts by bleeding into the necrotic area from adjacent arteries and veins. Red infarcts occur principally in organs with a dual blood supply, such as the lung, or those \Vilh extensive collateral circulation, such as the small intestine and brain. ln the heart, a red in fa rct occurs when the infarcted area is reperfused, as may occur following spontaneous or therapeutically induced lysis of the occluding thrombus . Grossly, red infarcts are sharply circumscribed, finn, and dark red to purple. Over a petiod of several days, acute in fiammatory cells infiltrate the necrotic area from the vi able border. The cellular debris is phagocytosed and digested by polymorphonuclear leukocytes and later by macrophages. Granulation tissue eventually fo rms, to be rep laced ultimately by a scar. None of the other choices would cause hemorrhage into an infarcL ***Diagnosis: Pulmona1y infarction, pulmonary thromboembolism
40 A 22-year-old woman delivers a baby at 29 weeks of gestaLion. Shortly after binh, the neonate becomes short of breath. The neonate is placed on a ventilator, buL dies of respirato ry msufficicncy. The brain at autopsy is shown. Which of the following mechanisms of disease best explams this complication of rcsptratory distress syndrome (RDS) of the neonate? ***(A) Anoxic injury ***(B) Birth trauma ***(C) Chronic passive congesLion ***(D) Hemolytic anemia ***(E) Hypertension
40 The answer is A: Anoxic injury. The pathogenesis of RDS of the newborn is intimately linked to a deficiency of surfaclant. This material lowers the surface tension of the alveoli at low lung volumes and thereby prevents collapse (atelectasis) of the alveoli during expiration. Atelectasis secondary to surfactant deficiency results in perfused but not ventilated alveoli, a situation that leads to hypoxia and acidosis. Intraventricular cerebral hemorrhage is a major complication of RDS. The perivenLricular germinal matrix in the newborn brain is particularly vulnerable to hemorrhage because the dilated, thin-walled veins in this area rupture easil)' (see photograph). The pathogenesis of Lhis complication is believed LO reflect anoxic injury to the perivemricular capillaries, venous sludging and thrombosis, and impaired vascular autoregulation. Despite advances in neonatal intensive care, the overall mortality ofRDS is about 15%, and one third or infams born before 30 weeks of gestational age die of this disorder. Although the other choices are associated with bleeding. they are unlikely causes of perivemricular hemorrhage in a baby with RDS. ***Diagnosis: Respiratory distress syndrome of the neonate
1 A 65-year-old woman \.Vilh a history of smoking presents L...- with a 3-week history of chest pain and bloody sputurn. An X-ray film of the chest reveals a hilar lung mass. The surgical specimen reveals a squamous cell carcinoma growing within the lumen of a bronchus (shov.'tl in the image). Which of the fo llowing chemical agents may be associated with the pathogenesis of the cancer in this patient? ***(A) Allaw:-.ins ***(B) Aromatic hydrocarbons ***(C) Benzene ***(D) Branched chain amino acids ***(E) Carbohydrate polymers
1 The answer is B: Aromatic hydrocarbons. These compounds (e.g., benzopyrene, methyl cholanthrene) are potent expelimental carcinogens. Aflatoxins (choice A) produce experimental liver cancer. The mmor in this patient is a squamous cell carcinoma, which bears a strong resemblance to nom1al squamous cells and synthesizes keratin, as evidenced by epithelial pearls. ***Diagnosis: Squamous cell carcinoma of the lung
2A 75-year-old man who had worked in a shipyard for 25 years dies of a thoracic mmor. Autopsy reveals a pleural rumor !hat encases the lung. lmerstitial pulmonary fibrosis and multiple pleural plaques are noted. Numerous, brown, beaded ferrugtnol. l':; bodies are also found in the lungs (shown in the image). Which of the following agents is most Likely assodated with the pathogenesis of the cancer in this patient? ***(A) AsbesLos ***(B) Beryllium ***(C) Carbon ***(D) Silica ***(E) Tobacco
2 The answer is A: Asbestos. Occupational exposure to asbestos poses a risk for the development of a pleural mmor termed malignam mesothelioma. \ltalignam disease may become evident 20 to 30 years following exposure. The inhalation of asbestos fibers also causes interstitial fibrosis of the lungs and pleural plaques consisting of dense connenive tissue. Asbestos fibers coated with protein and iron are tem1ed asbesws (femlginous) bodies. The mher choices do nor produce ferruginous bodies or cause mesmhdioma. ***Diagnosis: MesOlhelioma, asbestosis
3 A 4 7 -year-old man presems with a 6-week his wry of increasing fatigue and dark-colored swols. Complete blood count shows hemoglobin of 8.6 gldl and microcytic, hypochromic RBCs. Upper gastrointestinal endoscopy reveals a peptic ulcer along the lesser curvature of the stomach. This patient's anemia is most likely caused by deficiency of which of Lhe follovving? ***(A) Folic acid ***(B) lron ***(C) Thiamine ***(D) Vitamin B 12 ***(E) Zinc
3 The answer is B: Iron. Gastrointestinal hemonhage leads to the loss of heme iron at a rate faster than it is replaced from dieta1y sources. The result is microcytic hypochromic anemia. The anemias associated with deficiencies of folic acid (choice A) and vitamin B12 (choice D) are macrocytic. ***Diagnosis: Iron-deficiency anemia
4 A 49-year-old woman presents with a 2-month history of yellow discoloration of her eyes, abdominal pain, weight loss, and low-grade fever (38.4°C, 101 °f). Physical examination shows a distended abdomen, right upper qua dram tenderness, and a palpable liver 2 em below Lhe costal margin. A liver biopsy reveals alcoholic hepatitis. The patiem recovers and is strongly advised to abstain from alcohol. However, she subsequently imbibes some antifreeze containing ethylene glycol and develops acme failure of which organ? ***(A) Brain ***(B) Heart ***(C) Kidney ***(D) Liver ***(E) Pancreas
4 The answer is C: Kidney. The major toxicity of ethylene glycol is acute tubular necrosis of the kidney, which resulLS in renal failure. The compound has li ttle effect on the other organs. ***Diagnosis: Acute renal failure
5 A neonate was noted to have mild growth retardation and facial dysmorphology. The mOLher was a known abuser of several subsmnces. This infants problem most likely resuhed from maternal intake o r which of the following? ***(A) Alcoh ol ***(B) Cocaine ***(C) Ethylene glycol ***(D) Heroin ***(E) lvlarijuana
5 The answer is A: Alcohol. Fetal alcohol syndrome is the most common acquired cause of mental retardation in the United States. The common features of the syndrome include intrauterine growth retardation, facial dysmorphology, neurologic impairment, and other congenital anomalies. ln cases with lesser manifestat]ons, tenned fetal alcohol effect, chJldren later suffer from mental retardation and minor dysmorphic features. Cocaine {choice B) may cause neonatal dirficulties and heroin {choice D) intake may result in neonates that arc addicted to that opiate, but they are not associated with characteristic facial dysmorphology. ***Diagnosis: Fetal alcohol syndrome
6 A 10-ye<n-old boy presents with irritability and ataxia. He is subsequently found to have anemia, basophilic stippling of erythrocytes, and dark-gray pigmentation of the gums. Exposure to which of the following chemical agenlS is most likely associated with this disease? ***(A) Arsenic ***(B) Copper ***(C) Lead ***(D) Mercury ***(E) Nickel
6 The answer is C: Lead. Chronic lead poisoning inhibi!:s deltaarninolevuUnic acid dehydratase and ferrochelatase (enzymes essential for heme synthesis), thereby causing microcytic hypochromic anemia. The inhibition of heme symhesis leads to basophilic stippling of erythrocytes, which is due w residual ribosome cluslers in the cytoplasm. Chronic e)q>osure of children to lead also leads lO cognitive loss. Mercmy (choice D) poisoning has neurologic sequelae, bm nm these hemawlogic characteristics. ***Diagnosis: Lead poisoning
7 A 50-year-old man presems for a routine physical examination, which demonstrates an enlarged liver. During the visit, he describes memos from hjs supervisor at work regarding chronic e:xposure [ 0 vinyl chloride. The patiem has an elevated risk for which of the fo llowing rumors? ***(A) Angiosarcoma of lh e liver ***(B) Carcinoid tumor ***(C) Hepatic adenoma ***(D) Lymphoma ***(E) Metastatic colon cancer
7 The answer is A: Angiosarcoma of the liver. Occupational e".'posme co vinyl chloride (used in the production of plastics) is associated with the developmem of this malignam tumor of endothelial cells in the liver_ Angiosarcoma is also associated with exposure w arsenic (a component of pesticides) and Thorouast (a radioactive contrast medium used by radiologists prior LO 1950). None of the other tumors have been associated \Vilh occupational exposure w vinylchlmide. Hepatic adenoma (choice C) is associated with the use of oral contraceptives. ***Diagnosis: Angiosarcoma of the liver
8 A severely depressed 32-year-old man commits suicide by running his car motor in his closed garage for several hours. The mechanism of death in this case of carbon monoxide (CO) poisoning is through which of the following mechanisms? ***(A) Displacement of oxygen on hemoglobin by CO ***(B) Hepawcellular necrosis ***(C) Inhibition of protein synthesis ***(D) Inhibition of the respiratory chain enzymes ***(E) Myocardial infarction
8 The answer is A: Displacement of oxygen on hemoglobin by CO. CO combines with hemoglobin with an affinity 240 times greater than that of OX')'gen to form carboxyhemoglobin. At concentrations above 50% carboxyhemoglobin, cerebral anoxia, coma, convulsions, and death ensue. The other choices do not reflect the su·ong binding affinity of CO for hemoglobin. ***Diagnosis: Carbon monoxide poisorung
A japanese fisherman who lives in the vicinity of a plastics factory develops severe neurologic symptoms, including constriction of visual fields, paresthesias, ataxia, dysanhria, and hearing loss. Public health authorities find a number of similar cases in the local village. Exposure to which of the follovling chemical agems is most likely associated with the pathogenesis of this man:S neurologic disease? ***(A) Arsenic ***(B) Copper ***(C) Lead ***(D) Mercmy ***(E) Nickel
9 The answer is D: Mercury. Mercmy released imo the environment may be bioconcemrated and enter the food chain, particularly predatory fish. Organic mercurials principally damage the brain, whereas inorganic mercury is wxjc to the kidneys. Large outbreaks attributed to metbyl mercury poisoning have been reponed in Japan (fish) and Iraq (fungicides). Poisonings by the other choices do not elicit these neurologic symptoms ***Diagnosis: Mercmy poisoning
10 A 12-year-old boy is rescued 2 days afrer becoming lost in the Canadian woods in february. Physical examination shows he has gangrene of his fingers and roes. Which of the following mechanisms of cell injury played the most important role in mediating necrosis in the fingers and toes of this patient? ***(A) Activation of proapoptotic proteins ***(B) Generation of activated oxygen species ***(C) Lipid peroxidation ***(D) Membrane disruption by water crystals ***(E) PrO[ein and DNA crossli11king
10 The answer is D: Membrane disruption by water crystals. Exposure of the extremi ties to severe cold resuhs in the CiystallJzation of tissue water, which causes cellular disiUption and vascular changes, resulting in frostbite . Locallzed thrombosis often leads to focal ischemia and gangrene of toes and fingers. Mechanical disruption of cellular membranes by ice crystals occurs during both freezing and thawing. The other choices are associated with cell death, but they are not early events in frostbite-induced necrosis. ***Diagnosis: frostbite
11 A 35-year-old man is hospitalized with third-degree burns after being rescued from a house fire. Initially, he suffers from shock and oligmia, but his renal function retun1S to normal within a few days. Which of the following would be the most likely cause of death i [complications were w arise? ***(A) Ascites ***(B) Curling ulcers with hemonhage ***(C) Cushing ulcers ***(D) Pseudomembranous colitis ***(E) Sepsis
11 The answer is E: Sepsis. The most common cause of death in seriously burned patients is sepsis after infection of the burned skin. Gastric ulcers (stress or Curling ulcers, choice B) are occasionally encountered in burn patients, bmthey do not represem a common cause of death. Cushing ulcers of the stomach (choice C) are associated with trauma LO the central nervous system. ***Diagnosis: Sepsis, thermal injury
12 A healthy adult runs a marathon in the surruner and develops hot d1y skin, cessation of sweating, lactic acidosis, hypocalcemia, and muscle necrosis (rhabdomyolysis). Whlch of the following is the appropriate diagnosis? ***(A) Dysautonomia ***(B) Heat stroke ***(C) Malignant hyperthermia ***(D) Myotonic dystrophy ***(E) Polymyositis
12 The answer is B: Heat stroke. :Exenional heat stroke occurs in healthy men dming unusually vigorous exercise, particularly when the an1bient temperature is high. Lactic acidosis, hypocalcemia, and rhabdomyolysis may be severe problems. Myoglobinuric acute renal failure is not uncommon. Malignam l1ypenhermia (choice C) occurs in surgical parienlS after anesLhesia. ***Diagnosis: Heat stroke
13 A 26-year-old electrician is found unconscious in his backyard beside a metal ladder and an e}.lJOSed elecui cal wire, suffeting from a deep bum on his right hand. ResuscitaUon attempts are unsuccess ful. Whkh of the follm.ving was the most likely cause of death? ***(A) Cardiac arrhythmia ***(B) Disseminated intravascular coagulation ***(C) Myocardial infarction ***(D) Rupture of Lhe ascending aorta ***(E) Uncoupling of oxidative phosphorylation
13 The answer is A: Cardiac arrhythmia. Electrical ene rgy disrupts the electrical system wi thin the bean and frequemly causes death through ventricular fibrillation. The force produced by high-voltage cunents vaporizes tissue water and produces extensive damage. The other choices are nol consequences of powerful electrical currents. Allhough myocardial infarction (choice C) can cause an immediate anhythmia, it reflects obstmction of the coronary circulation. ***Diagnosis: Eleclrical injury, cardiac anhythmia
14 A sailor on a nuclear-powered submarine is seen by a physician after a breach in the reactor comainmem system. Physical examination is unremarkable, but the patiem subsequently develops profound pancytopenia. Hemoglobin is 7.8 gtdl, WBC coum is 900/f.l l , and platelets are 20,000/IJL. How many rads of acute total-body radiation djd this patient most likely receive? ***(A) 1 ***(B) 3 ***(C) 30 ***(D) 300 ***(E) .3,000
14 The answer is D: 300. Acute total-body irradiation o[ abour 300 rads causes depression of the bone marrow, and symptoms related Lo granulocytopenia and thrombocytopenia develop wilhin 2 weeks. Anemia follows more slowl)' because red blood cells have a longer lifespan than leukocytes and pla1.elets. EJ..'Posure w 3,000 rads (choice E) is rapidly fatal owing to ceno·al nervous system damage. ***Diagnosis: Radiation sickness
15 Another sailor on the submarine (see Question 14) dies 10 days after the accident as a result of severe dianhea and dehydration. Which of the fo llowing doses (in rads) of acute tmal-body radiation did this sai lor most like!)' receive? ***(A) 1 ***(B) 3 ***(C) 100 ***(D) 300 ***(E) 1,000
15 The answer is E: 1,000. A radiation dose of 1,000 rads causes destruction of tissues composed of proliferating cells. Damage m the gastrointestinal tract is the most serious consequence and ensues within days of exposure. Death resulLs from massive flu id loss from the denuded intestinal mucosa and superimposed passage of bacteria through the damaged imestine. The lower doses listed in the question do nol destroy 1.he gasLroimestinal epithelium. ***Diagnosis: Radiation sickness
16 A 66-year-old woman presents with a 6-month history of scaling and abnom1al pigmentation of the skin. Her past medical hiswry is significant for the treatment of thyroid cancer I year ago. Biopsy of lesional skin shows atrophy of the epidermis and dense fibrosis of the dennis, which clisplays dl1ated superficial blood vessels. These pathologic findi ngs are most Jjkely caused by previous exposure to which of the following? ***(A) Chemotherapy ***(B) Corticosteroids ***(C) Organic iodine ***(D) Radiation therapy ***(E) Tumor necrosis factor-a.
16 The answer is D: Radiation therapy. Ionizing radiation administered for the treatmem of cancer must firsl traverse the skin, leading to radiation dermatitis. Skin biopsy shows atrophy of 1.h e epidermis and dense fibrosis of the dermis, wl1ich displays dilated superficial blood vessc Is. ln some cases, persistem ulcers require skin grafts. The other choices do not cause these dermal findings. ***Diagnosis: Radiation injury
17 A 28 -year~o ld radar technician aboard an aircraft carrier has been subjected to intense microwave radiation for 7 years. He has an elevated 1isk for developing which of the following? ***(A) Aplastic anemia ***(B) Hodgkln disease ***(C) Lymphocytic leukemia ***(D) Myelogenous leukemia ***(E) None of the above
17 The answer is E: None of the above. The absorption of microwave energy produces only heat and is not associated wi th any knovm health risks. ***Diagnosis: Radiarion injury
18 A 15-year-old African boy has a history of tooth loss, gingiviLis, skin hemorrhages, mnldple infections, and poor wound healing. Physical examination shows Lhat the child is m the 20th percentile for height and lOth percentile for weight. TI1is patient most likely has which of the following underlying conditions? ***(A) Beri-beri ***(B) Irnpetigo ((:) Kwashiorkor ***(D) Pellagra ***(E) Scurvy
18 The answer is E: Scurvy. Vitamin Cis essential for collagen synthesis, and its defidency results in poor wound healing. Pe rifollicular hemorrhages arise from capUlaries £hat have weak walls and are easily damaged by minor trauma. Impai red collagen synthesis leads to gingivitis and alveolar bone resorption. resulting in loss of teed1. Wound healing requires collagen synthesis and is impaired in patients with vitamin C deficiency (scurvy). ***Diagnosis: Scurvy, vitamin C deficiency
19 A 24-year-old woman, who is a food faddist, eats only combased foods. She p resents with dermatitis, diarrhea, and dementia. This patient most like!)' suffers from whkh of the following conditions? ***(A) Be1i-beri ***(B) Impetigo ***(C) KVv·ashiorkor ***(D) Marasmus ***(E) Pellagra
19 The answer is E: Pellagra. Niacin deficiency leads to the 3 Ds: dennatitis, diarrhea, and dementia. A swollen, fissured tongue and chron.i.c watery diarrhea are also characteristic. Dementia reflects degeneration of ganglion cells in the cerebral cortex. ***Diagnosis: Pellagra, niacin deficiency
20 A 45-year-old woman with longstanding Crohn disease and severe Cat malabsorption e.\.'Peliences a fracture of the femoral neck after a minor comusion. This woman most llkely has a dellciency or which of the fo llowing vitamins7 ***(A) Vitamin B1 (thiamlne) ***(B) Vitamin C ***(C) Vitamin D ***(D) Vitamin K ***(E) Niacin
20 The answer is C: Vitamin D. Lipid malabsorption interferes with the absorption of vi tamin D, thereby leading to a deficiency state. ln adults, vitamin D deficiency results in osteomalacia, a disorder characterized by inadequate mineralization of newly formed bone marri.x. The consequent weakness of bone is associated with a vulnerability to spontaneous fractures. Vitanun D deficiency i.n children is termed rickets_ Deficiencies Ln the other choices are not associated with these bone abnormalities. ***Diagnosis: Osteomalacia, vitamin D deficiency
21 A homeless man, who is a chronic aJcohohc, is brought to the hospital in a wasted staLe. He is noted to have a peripheral neuropathy, difficulty balancing, and dememia. He dies suddenly of an arrhythmia, and at autopsy; lesions are found m the mami11ary bodies and the viciniry of the third vetmicle. The vi tamin deficiency associated with these signs and syruproms is which of the following? ***(A) Vitamin B1 (thiamine) ***(B) Vitamin B11 ***(C) Vitamin D ***(D) Niacin ***(E) Pytidoxine
21 The answer is A: Vitamin 81 (thiamine} . This man suffers from b eri-beri (heart) and Wernicke-Korsakoff syndrome (brain). Thiamine deficiency in deteriorated alcoholics results in encephalopathy, peripheral neuropathy. and other disorders. Atrophy of the mammillary bodies, with loss of ganglion cells and mpture of small blood vessels, is characteristic. Deficiencies of the other vitami ns are not related tO these disorders. ***Diagnosis: Beri-ber i, Wernicke encephalopathy
22 A -+0-year-old, malnourished woman presents with a 6-month history of night blindness. Physical examination reveals keratoma! acia and corneal ulceration. \-Vhich of the following viLamin deficiencies would be suspected in this patient? ***(A) Vitamin A ***(B) Vitamin B2 (riboflavin) ***(C) Vitamin C ***(D) Vitamin E ***(E) Folk acid
22 The answer is A: Vitamin A. Vitamin A deficiency causes squamous metaplasia at a number of sites. In the cornea, it leads to xerophthalmia (dry eye), which rnay progress to softening of the tissue (keratomalacia) and corneal ulceration. Deficiencies of the other vitamins are not related to these dimcal and pathologic findings. ***Diagnosis: Vitamin A deficiency, keratomalacia
23 A 26-year-old woman presents to the emergency room with fever and shaking chms. Her temperature is 38PC (l03°F), pulse 120 per minute, and blood pressure 140/80 mm Hg. Ph)'sical examination reveals a harsh systolic murmur. The patient develops a headache. slips into a coma and e:>..'})ircs. The aortic valve is examined at autopsy (shown in the image). Vl/hich of the fo llowing is the most important risk factor for this pathologic finding? ***(A) Alcoholism ***(B) Autoimmune disease ***(C) Cigarette smoking ***(D) lntravenous dmg abuse ***(E) OraJ contraceptive use
23 The answer is D: Intravenous drug abuse. The introduction of bacteria by intravenous drug abuse may lead to septic complications in many organs. Bacterial endocarditis, often involving Staphylococcus aureus , may occur on both sides of the heart. Infected emboli can occlude vessels leading to gangrene. Infected emboli in the b rain can cause cerebral abscess. The photograph shows ad herent vegetations on the aortic valve. These vegetations are comp osed of platelets, fibrin, cell debris, and masses of organisms. In addition to intravenous drug abuse , risk fac tors for bacte1ial endoca rditis include congenital heart disease (children), rheumatic heart disease, prosthetic heart valves, transient bactere!1Ua, and diabetes. Although certain autoimmune diseases (cl1oice B) are associated endocarditis (e g , Libman-Sacks), the verrucous vegetations are sterile (nonbactenal). The other choices are not associated with a significantly increased risk of bacterial endocarditis. ***Diagnosis: Bacterial endocarditis
24 A 50-year-old woman presents with easy fatigability, a smooth sore tongue, numbness and tingling o[ the feet, and weakness of the legs. A complete blood count shows a megaloblastic anemia that is not reversed by folate therapy: Hemoglobin is 5.6 gldL, vVBC count is 5 ,100/!lL, and platelets are 240,000/ JlL This patient most Ukely has a deficiency of which of the following vitamins'? ***(A) Vitamin B1 (thiamine) ***(B) Vitamin B1 (riboaavin) ***(C) Vitamin B 12 ***(D) Vitamin K ***(E) Niacin
24 The answer is C: Vitamin B12 • Except for a few rare situations, vitam]n B11 (cyanocobalamin) deficiency is usually a result of pernicious anemia, an autoimmune disease of the stomach. Vitamin B12 is required for DNA symhesis, and its deficiency results in large (megaloblastic) nuclei. ***Diagnosis: Vitamin B12 deficiency, pernicious anemia
25 A starving, 4-year-old, African boy presents \vim apathy, generalized edema, and an enlarged fatty liver. The physician notes that, despite generalized growth failure, subcutaneous fat is preserved. What is the appropriate cliagnosis? ***(A) Beri·beri ***(B) Kwashiorkor ***(C) Marasmus ***(D) Pellagra ***(E) $curvy
25 The answer is B: Kwashiorkor. Kwashiorkor is a syndrome that results from a deficiency of protein in a diet relatively high in carbohydrates. 1l is one of the most common diseases of infancy and childhood in the nonindustrialized world. It usually occurs after an infant is weaned, when a protein -poor diet, consisting principaUy of staple carbohydrates, replaces mothers milk. Unlike marasmus (choice C), the diso rder fea rures edema, large farcy liver, and depigmemation of the skin. Extreme apathy is notable, dian-hea is common, and anemia is the mle. These changes are reversible if and when sufficient protein is made available. ***Diagnosis: Kwashiorkor
26 Petechial hemorrhages were n oticed on the upper and lower extremities of a 5-day-old infant. Hemorrhagic disease of the neonate was most likely caused by a deficiency of "vhich of the following vitamins? ***(A) Vitamin B2 (riboflavin) ***(B) Vitamin D ***(C) Folic acid ***(D) Vitamin K ***(E) Pyridoxine
26 The answer is D: Vitamin K. Hemon-hagic disease of Lhe newborn may be caused by a deficiency of vitamin K. ViLatnin K Lc; an lmponant coagulation factor, which is necessary for the carboxylation and activation of prothrombin, as well as of clotting facwrs VII, IX, and X. Newborn i.nfams frequent ly exhibit vitamm K deficiency because the vitamin is not transported well across the placenta, and the sterile gm of the newborn does not have bacteria to produce it. The mher viLamin deficiencies do noL impair coagulation. ***Diagnosis: Vitamin K deficiency, hemolytic disease of Lhe newborn
27 A 6-year-old girl is examined at a clinic in central Af1ica. Physical examination reveals wasdJ1g of muscle and fat, and a protuberant abdomen. Her pulse, blood pressure, and temperature are low. The face appears wrinkled. There is no evidence of generalized edema. Wl1at is the appropriate diagnosis? ***(A) Beri-beri ***(B) Kwashiorko r ***(C) Marasmus ***(D) Pellagra ***(E) Scurvy
27 The answer is C: Marasmus. Defictency of all elements of Lhe dieLleads lO marasmus. The condition is common throughout the nonindustrialized world, particularly when breasL feeding is stopped, and a child must subsist on a caloncally inadequate diet. TI1e pathological changes are similar w Lhose in starving adulLs and consist of decreased body weighL, diminished subcutaneous fa t, a protuberant abdomen, muscle wasting, and a \Winkled face. In general , the child appears as a "shrunken old person." Wasting and increased li pofuscin pigmem are seen in most visceral organs. especially the hean and t:he Uver. No edema is present. The pulse, blood pressure, and temperamre are low, and diarrhea is common. Because immune responses are impaired, the child suffers from numerous infections. An important consequence of marasmus ls growth failure. If these children a re not p rovided w1th an adequate diet during ch ildhood, they will not reach their full potential stature as adults. Kwashiorkor (choice B) resulrs from deficiency of protein in the diet. Choices A, D, and E result fTom deficiencies of vitamin B1 , nicacin, and vitamin C. respectively. ***Diagnosis: Marasmus
28 A 40-year-old , chronicalJy Ul man from a Vietnamese village presents with painful sores around his mouth. Physical examination reveals prominem fissures at the angles o£ his mouth. Cheilosis in this patient is most likely caused by a deficiency of which of the following vitamins? ***(A) Vitamin A ***(B) Vitamin B1 {thiamine) ***(C) Vitamin B2 (riboflavin) ***(D) Vitamin B12 ***(E) Vitamin C
28 The answer is C: Vitamin 82 (riboflavin). Cheilosis refers to fissures at the angles of the mouth and is a common finding in patients with vitamin B2 (riboflavin) deficiency. RlboOavin participates in the synthesis of flavin mononucleolides. Seborrheic keratosis and interstitial keratitis of the comea also occur in pmiems wi.th vitamin B1 (riboflavin) deficiency. Except for viLamin A deficiency (which does not cause cheilosis), the other choices (B, D, and E) do nm affect Lhe skin. ***Diagnosis: Cheilosis, vi tamin B2 (ribo flavin) deficiency
29 A 32-year-old woman is a vegan (i.e., a srrict vegetarian who eats no animal products of any kind). She is weak and pale, and laboratory studies show a macrocytic, normochromic anemia (hemoglobin= 6.2 gldl). This patient most likely has whkh ol the following vitamin deficiencies? ***(A) Yilamin A ***(B) Vitamin B 12 ***(C) Vitamin D ***(D) Vitamin E ***(E) Niacin
29 The answer is B: Vitamin 812• Vitamin B1 P which is necessary for DNA synthesis, is contained only m animal products, including eggs. Extreme vegetarians may suffer vitamin B12 deficiency after many years of a restricted dieL. The result is a macrocytlc anemia similar to that seen in pemicious anemia. Macrocytic anemia is not a consequence of the other choices. ***Diagnosis: Vitamin B12 deficiency, megaloblastic anemia
30 A 46-year-old man complains of weakness, dyspnea on exertion, and palpitations. His temperature is 3JOC (98 6°F), pulse 95 per minute, respirations 24 per minme, and blood pressure 120/80 mm Hg. Physical examination reveals pulmonary rales and peripheral edema. Laboratory studies show nom1al serum cholesterol (180 mgldl ) and elevated fasting blood sugar (160 mgldL). An echocardiogram discloses significant cardiomegaly bm no valvular abnormalities or evidence of old myocardial infarcts. An angiogram shows nonnal corona1y arteries. What is the most likely cause o[ his disease? ***(A) Chronic alcoholism ***(B) Cigarette smoking ***(C) Diabetes mellirus ***(D) Diffuse alveolar damage ***(E) Rheumatic heart disease
30 The answer is A: Chronic alcoholism. Alcoholic cardiomyopa· thy correlates \Vith the total lifetime dose of alcohol and leads lO dilation and hypertrophy of the heart. As in this case, the disorder may cause congestive heart failure. Cigarette smoking (choice B) and diabetes mellitus (choice C) are associated with coronary anety disease, and rheumatic heart disease (choice E) features valvular abnormalities. ***Diagnosis: Alcoholic cardiomyopalhy
31 A 28-year-old pregnant woman weighs 86.4 kg (190 lb) and gives birth after 39 ·weeks of gestation to a small baby weighing 2,300 g (5 lb. l oz). The baby has no congenital anomalies. Which of the following maternal factors in this case may be associated with the infants low birth weight? ***(A) Folic acid deficiency ***(B) Obesity ***(C) Premature delivery ***(D) Previous oral contraceptive use ***(E) Smoking
31 The answer is E: Smoking . Fetal tobacco syndrome refers to the deleterious effects of maternal cigarette smoking on the development of the ferus_ Infants born w women who smoke during pregnancy are, on average, 200 g lighter than infants born to women who do nm smoke. These infants are nm bom preterrn, but rather, are small for gestational age. The noxious effect of smoking on t:he ferns is mirrored by its effen on the uteroplacemal unit. The incidences of abruptio placentae, placenta previa. uterine bleeding, and premature mpmre of t:he membranes are all increased in women who smoke. Evidence indicates that the injurious effects of maternal cigarette smoking are nm limited w the fetus but extend to the physical, cognitive, and emotional development o [ children at older ages. Choices A, B, and D are not associated with low birth weight infants. ***Diagnosis: Fetal tobacco syndrome
32 A 52-year-old man pTesents with sudden crushing chest pain and tachycardia He admits Lo cigarette smoking, consumption of alcohol, and abuse of illicit dmgs. An ECG is consistent with ischemic change in the anrerosepLal region of the heart. Laboratory studies show elevated semm levels o£ CK-MB and tmponin I. Semm cholesterol is 240 mgldL Which of the following most llkely contributed to this patients condition'? ***(A) Alcohol consumption ***(B) Cigarette smoking ***(C) Heroin addiction ***(D) Inadequate calcium intake ***(E) Marijuana use
32 The answer is 8: Cigarette smoking. Cigarette smoking is recognized as a major independent risk factor for myocardial infarction and acts synergistically \vith olher risk factors such as high blood pressure and elevated blood cholesterol levels. 1t not only serves to precipitate initial myocardial infarction but also increases the risk for second heart attacks and diminishes survival after a heart attack among those who continue to smoke. Smoking also increases the incidence of sudden cardiac death, possibly by exacerbating regional ischemia. Atherosclerosis of the coronary arteries and the aon a is more severe and extensive among cigarette smokers than among nonsmokers, and the dfect is dose related. Chronic alcohol consumption (choice A) actually protects against coronary aneTy disease, although dilated cardiomyopathy may develop. The other choices are not related to hean disease. ***Diagnosis: Myocardial infarction
33 A 52-year-old man presenrs with a chronic cough and shortness of breath. He admits lO smoking two packs of cigareues a day for 30 years Pulmonary function tests reveal chronic obstructive pulmonary disease. In counseling this patient, you advise him w swp smoking immediately. You also mention Lhat, in addition to emphysema, which of the following organs carries a significantly increased risk of smoking-related cancer7 ***(A) Brain ***(B) Uver ***(C) Pancreas ***(D) Skin ***(E) Small bowel
33 The answer is C: Pancreas. Cancer of the pancreas has shown a steady increase in inctdence, which is panty related LO cigarene smoking. The risk for adenocarcinoma of the pancreas in male smokers is elevated two- lO threefold, and a clear doseresponse relat]onship exists. ln fact, men who smoke more than rwo packs a day have a five Limes greater risk of developing pancreatic cancer than nonsmokers. Smoking does not 1ncrease the risl< of cancer for the other choices. ***Diagnosis: Chronic obstructive pulmonary disease
34 A homeless, poorly nourished man collapses on the street and cannot be revived by the emergency medical technicians. A social service agency notes that he had a long history of abusing many illicit drugs. An autopsy is petfonned and reveals a moderately enlarged heart, with patent coronary arte1ies and no vah'\Ilar abnonnalities. Microscopic examination shows patchy fibrosis of the myocardium. Which of the fol lowing substances most likely caused thjs cardiomyopathy? ***(A) Cocaine ***(B) Heroin ***(C) Lysergic acid diethylamide (LSD) ***(D) Matijuana ***(E) Methamphetamine
34 The answer is A: Cocaine. Cocaine overdose leads to anxiety, delirium, and occasion ally seizures. Cardiac arrhythmias and other effects on the heart may cause sudden death in other\ Vise healthy persons. Chronic abuse of cocaine is associated with the occasional development of a charactensti.c dilated cardiomyopathy, probably because of its effects in small, imramyocarcltat, coronary arteries. The other choices do nol cause cardiomegaly. ***Diagnosis: Cocaine cardiomyopathy
35 A 52-year-old, obese man (BMI = 34 kglm2) presents to his physician with complain£5 of hoarseness for 2 months. He has worked in a chemical factory for 25 years and gives a history of smoking, consumption of one or two beers a day, and occasional use of illicit dmgs. Physical examination reveals enlarged and firm cervical lymph nodes. Direct laryngoscopy reveals a fixed and enlarged left vocal cord, which appears ulcerated. A biopsy of the lesion is interpreted by the pathologist as squamous cell carcinoma. vYhat is the most likely cause of this mans disease? ***(A) Alcohol ***(B) Benzene inhalation ***(C) Cigarette smoking ***(D) Cocaine use ***(E) High-fat diet
35 The answer is C: Cigarette smoking. Cancers of the hp, wngue, and buccal mucosa occur principally (>90%) in wbacco users. Cancer of the la1ynx and esophagus also result fTom smoking (>80% of cases) In some large studies, aU deaths fmm cancer of the la1ynx occurred in smokers. There is no epidemiologic evidence that the other choices are 1isk factors for la ryngeal cancer, although chronic alcoholism may be associated with a slightly increased risk. ***Diagnosis: Laryngeal carcinoma
36 A 48-year~old man complains of weakness and easy fatigability for 6 weeks. He has worked for 20 years in a chemical factory that produces a variety of plastics and other synthetic compOLmds. A complete blood count shows a hemoglobin level of 8.2 g!dL, WBC count of 45,000/~L , and a platelet coum of 40,000/~L Examination of a bone marrow aspirate reveals numerous malignant myeloblasts, and a diagnosis of acme myeloid leukemia is made. Exposure to which of the following agents is the most likely cause of this patient's hematologic disease? ***(A) Benzene ***(B) Benzopyrene ***(C) Carbon tetrachloride ***(D) Glycerin ***(E) Ttichloroethylene
36 The answer i.s A: Benzene. Vinually all cases of acute and chronic benzene toxicity have occurred against the background of industrial exposure. Acute benzene poisoning pnmarily affects the cenu·al nervous system, and death results from respiratOl)' fai lu re. However, the long-term effects of benzene exposure have attracted the most attention. The bone marrow is the principal target in chronic benzene intoxication. Patients who develop hematologic abnonnalities characreristically ex.hibit hypoplasia or aplasia of the bone marrow and pancytopenia. Aplastic anemia usually is seen while the workers are still exposed to high concemmtions of benzene. In a substantial proportion of cases of benzene-induced anemias, acme myeloblastic leukemia develops. Overall, the risk of leukemia is increased 60-fold in workers eA.'Posed to t.he highest atmosphetic concentrations of benzene. The other choices are not linked to t.he development of leukemia. ***Diagnosis: Acute myelogenous leukemia
37 A 48-year-old woman complains she has had weakness, fatigue, and easy bmis.ability for 2 months. She had worked as a technician in a nuclear energy plant for 15 years and was involved in an accident during which she was exposed to considerable radtation. PhysicaJ examination reveals an enlarged liver and spleen. What disease should you suspect as a likely cause of her condition? ***(A) Chronic myelogenous leukemia ***(B) Hairy cell leukemia ***(C) Metastatic carcinoma of the breast ***(D) Metastatic carcinoma of the stomach ***(E) Osteogenic sarcoma
37 The answer is A: Chronic myelogenous leukemia. The evidence that. whole-body radiation can lead to cancer is mcontmverdble and comes from animal experiments and studies of the effects of occuparional eA.1Josurc, radiation therapy for nonneoplastic conditions, the diagnostic use of certain radioisotopes, and the atom bomb explosions. Some survivors of the atom bomb explosions and patien ts subjected to spinal radiation later developed chronic myelogenm..1s leukemia. Although the other choices may lead to hepatosplenomegaly, they are not linked to acute radiation exposure. ***Diagnosis: Chronic myelogenous leukemia
38 A 16-year-old girl has suffered from severe celiac disease for years and reports continued stearorrhea. She suddenly develops abdominaJ pain in the 1ight lower quadrant. A complete blood count shows a hemoglobin level of 14 gldL, WBC of 18,000/!Jl , with 84% neutrophils, and a platelet coum of 280,000/!JL. A diagnosis of appendicitis ts made, and tests before surgery reveal a prolonged prothrombin time of 17 seconds (control = 2). What is the most likely cause of her coagulation problem? ***(A) Hemophilia A ***(B) Hemophilia B ***(C) Hypolipidemia ***(D) Lymphoblastic leukemia ***(E) Vitamin K deficiency
38 The answer is E: Vitamin K deficiency. Vitamin K deficiency is common in severe fat malabsorption, as seen in ce Hac sprue and biliary tract obsuuction. The destruction of intestinal flora by antibiotics may also result in vitamin K deficiency. Vitamin K, whkh conJers calcium-binding properties to certain proteins, is imponant for the activicy of four clmting factors: prothrombin, factor Vll, factor lX, and facto r X. Deficiency of vitamin K can be serious because it can lead to catastrophic bleeding ***Diagnosis: Vitamin K deficiency
39 A 24-year-old beating victim is brought to the emergency room \.Vith a -1--cm linear tear of the skin caused by blum trauma. Which of the following terms best describes this patients skin 1e st.o n_1 ***(A) Abrasion ***(B) Avulston ***(C) Blast injury ***(D) Comusion ***(E) Laceration
39 The answer is E: Laceration. A laceration is a linear tear of the skin produced by a fo rce that causes unidirectional displacement. A surgical incision is a comrolled laceration. Internal organs may also be lacerated by trauma or by the surgeon. An abrasion (choice A) is a skin defect caused by crushes or scrapes. Avulsion (choice B) is a tearing away or forcible separation. A contusion (choice D) is a localized mechanical injury with [ocal hemorrhage. ***Diagnosis: Laceration
· 1 A 2.3-year-old man presenlS \Vilh a 6-day history of fever, sore tJ1roat, swollen lymph nodes, weight loss, and fatigue. Physical examination shows generalized lymphadenopathy, most prominent in the cervical lymph nodes, and mild h epatosple- 11omegaly The pe1ipheral blood smear shows 65% atypical lymphocyJes. A Paul-Bunnell antigen test (heterophile an tibody test) is positive. The semm ALT, AST, and bilintbin are slightly elevated. The atypical lymphocytes in this patients peripheral blood are described as which of the foltmving? ***(A) Activated T cells ***(B) lmmamre B cells ***(C) Mature B ce lls ***(D) Natural killer cells ***(E) Plasma cells
1 The answer is A: Adivated T cells. Infectious mononucleosis is characterized by fever, pharyngitis, lymphadenopathy, and circulming atypical lymphocytes. This systemic viral infection is caused by EpsLein-Barr virus (EBV), a herpesvirus chat is n·m15n)jued through respiratory droplets and saliva and binds w nasopharyngeal cells and B lymphocytes. T cells proliferate in response w activated B lymphocytes and appear in the petipheral blood as atypical lymphocytes. Anemia and thrombocytopenia are conunon. Ln developed countries, mfecrious mononucleosis commonly affects teenagers and young adults and is often referred was the "kissing disease." In underdeveloped commies, EBV infections are typically seen as subclinka] infections in chUdhood and are associated \".:ith an increased risk of Burkitt lymphoma and nasopharyngeal carcinoma. Although EBV infects B ce!Js, the circulating atypical lymphocytes seen in patien ts with infectious mononucleosl'> are not immature B cells (choice B), but rather indirectly activated T cells. ***Diagnosis: Infectious mononucleosis
2Which of the following is the most likely compUcarion for the patient described in Question 1? ***(A) Burkitt lymphoma ***(B) Cirrhosis ***(C) Encephaluis ***(D) Laryngeal stricture ***(E) Rupmre o[ the spleen
2 The answer is E: Rupture of the spleen. Splenomegaly often develops in patients with mfectious mononucleosis due w lymphoid infilo·ation, hyperplasia, and edema. The enlarged spleen may rupture after minor trauma. Burkitt lymphoma (choice A) is associated wiLh Epstein-Barr virus infection in certain parts of the world but is uncommon in Nonh Ametica. ***Diagnosis: Infectious mononucleosis
3 A 19-year-old woman presents with vague lower abdominal pain and a swollen , painful right knee. She denies any tTauma to the knee or history of arthritic disorders. Physical examination reveals an enlarged joint that is red, warm, and painful. Pelvic examination is exquisitely painful and reveals an illdefined thickening in both adnexae. A green-yello'.v purulent vaginal discharge is noted. The patiem is febrile and has an devated 'NBC count of 15,000/).1L Which of the following e£iologic agems is most likely responsible for this patiems condi Lion? ***(A) Escherichia coli ***(B) Neisseria gmwrdweae ***(C) Streptococws pyogenes ***(D) 1h~p011ei1W pallidum ***(E) Yersinia pestis
3 The answer is 8: Neisseria gonorrhoeae. N. gonorrfweae causes an acute suppurative infection of the genital tract. wruch presents as urethrltis in men and endocervicitis in women. lt is 011e of the most commonse>.'Ually transmiued diseases_ Gonorrhea may involve the throat, anus, rectwn, epicUdymis, cervix, fallopian rubes, prostate gland, or joims. Septic arthritis due to N. gonon1weae, a suppurative inflammadon most commonly caused by hemawgenous spread, is usually monoarticular, most commonly affecting hips and knees. The other choices do not typically exhibit arthritis and acute pelvic disease. Primary syphilis (choice D) presents \vith chancre. ***Diagnosis: Gon orrhea
4 A 54-year-old man presents with a worsening skin rash of 10 days in durmion. Physical examination shows an extensive, desquamative maculopapular rash of the palms and soles (shown in Lhe image). The VDRL and FT-ABS tcsLs are both positlve. Which of the following lesions is also expected in this patient at this stage of his disease? ***(A) Chancre ***(B) Dementia ***(C) Endarteritis obliLerans ***(D) Gummas ***(E) Tabes dorsalis
4 The answer is C: Endarteritis obliterans. Seconda1y syphilis represents systemic dissemination and proliferation of the spirochete. Treponema pallidum. This stage is charaCLerized pathologically by lesions in skin, mucous membranes, lymph nodes, meninges, sLOmach, and liver. The lesions show a petivascular lymphocytic infiltration and endaneritis obliterans. In most cases, the rash appears 2 weeks to 3 momhs after 1.he primary lesion (chancre) heals. Other lesions a.ssociated with secondary syphilis include condylomata lata, follicular syphilis, and nummular syphUis. Chancre (choice A) is a characteristic lesion of primary syphilis. Choices B, D, ru1d E are encoumered ln patiems wilh tertimy syphilis. ***Diagnosis: Syphilis
5 A 35-year-old man presems with multiple skin lesions on his left forearm. He had worked in a garment factmy, where he sorted wool imported from Iran. Physical examination shows an elevated cutaneous papule with blood)' pumlem eA'Udate. An adjacem lesion displays a black scab. There is prominent axillary lymphadenopathy This patients necrotizing skin lesions are most likely caused by which of the following pathogens? ***(A) Bacillus anthracis ***(B) Clostridium pe1jringens ***(C) Escherichia coli ***(D) Streptococcus pyogene.s ***(E) Tn~ponema pallidum
5 The answer is A: Bacillus anthracis. Amh rax is a necrmizing disease caused by B. anthracis. The clinical presentation of anthrax depends on the si te of inoculation and includes malignant pusLUle, pulmonary amhrax, septicemic amhrax, and gastroimest.inal amhrax. B. anthacis typically produces extensive tissue necrosis at the site or infection, with a mild neutrophilic infiltration Malignant pusmle is seen in over 95% of all cases of anthrax and represems the cutaneous form of this infectious disease. The infected person presents with an elevated cutaneous papulae that enlarges and erodes into an uker. Local hemorrhagic pustules may progress to carbuncles. Cutaneous lesions comain numerous organisms that rel.ease a potent necrotizing to:>..'in. The other choices do not typically present with these necrotizing cutaneous manifestations. ***Diagnosis: A.nrhrax, malignam pusrule
6 A 24-year-old man, who has just returned from a lrip to India, complains of fatigue and severe, unremitting, watery diauhca. l aboratory studies show severe hypernatremia and hypokalemia. Which of the following metabolic changes in enterocytes is chiefly responsible for lh e pathogenesis of this patients acute gastrointestinal disorder? ***(A) Decreased intracellular calcium ***(B) Decreased plasma membrane biosynthesis ***(C) Decreased ..,.vater secretion ***(D) lncreased inu·acellular cAMP ***(E) Increased intracellular water
6 The answer is D: Increased intratellular tAMP. Cholera is a severe diarrheal illness caused by the enterotoxin or Vibrio cltoleraE an anaerobic Gram-nega tive rod . The organism proliferates in the lumen of the small intestme and causes profuse watery diarrhea and rapid dehydration. Shock and death can ensue within 24 hours from the onset or symptoms. The AB toxin secreted by the organism binds to ganglioside GMl on intestinal epithelial cells and stimulates an increase in innacellular levels of cAMP, thereby leadmg to water secretion. The mucosa does not show significant pathologic changes The other choices directly injure the intestinal mucosa and cause hemorrhage. ***Diagnosis: Cholera, acute gastroenteritis
7_ A 12-year-old girl develops feve r, abdominal pam, and bloody dianhea l to 2 days after eating a hamburger at a fast food restaurant. Physical examill ation reveals an extensive purpuric skin rash. The patient develops oligu ria, and laboratory studles show elevated serum Levels o[ BUN and creati11ine. Which of the foUov.ring is the most likely etiolog~c agem responsible for this patient's condition? ***(A) Campylobacterjejuni ***(B) Escherichia coli 0157-H7 ***(C) Salmonella typhi ***(D) Sl1igella dysenteriae ***(E) Yersinia pestis
1 The answer is 8: Escherichia coli 0151·H1. Emerohemorrhagic E. coli (serotype 0157-H7), which may contaminate meat or mHk, causes bloody diarrhea, which can be fol1owed by the hemolytic-uremic synd rome. The organism adheres to the colonic mucosa and releases an enterowx'in that destroys epithel]al cells. Patients present with abdominal pain, 10\vgrade fever, and bloody diarrhea. Stool examination shows leukocytes and erythrocytes. Hemolytic-uremic syndrome is manifested by microangiopathic hemolytic anemia, thrombocytopenia, and acute renal fai lure. Although they may be associated with bloody diarrhea, the other choices do not present with hemolytic-uremic syndrome. ***Diagnosis: Hemolytic-uremic syndrome, enterohemorrhagic E. coli
8 A 65-year-old man undergoes cardiac bypass surgery and is placed on postoperative, broad-spectmm, an tibiotic prophylaxis. Several days later, he develops feve r, abdominal pain, and bloody diarrhea. Colonoscopic bio psy demonstrates a thick mucopuntlent exudale. Which of the following is the most likely etiology of this patiems gastrointestinal disorder? ***(A) Clostridium botulinum ***(B) Clostriditm1 difficile ***(C) Clost ridium pe•f ringens ***(D) Clost ridiwn tetani ***(E) Escltericl1ia coli 0157-H?
8 The answer is 8: Clostridium ditficile. C. diffic ile is the most common cause of diarrhea in patients on antibiotic therapy (e.g., clindamycin or cephalosporlns) v.tho are has· pilali.zed for more than 3 days. Necrotizing enterocolitis (pseudomembranous colitis) is a disease t11at may affect the colon in segments or in its entirety. The mucosa is covered by yellow-green, necrotic exudates (pseudomembranes). f ood poisoning and necrotizing entercoliris are caused by the emerotoxit15 of C. pe1jringens (choice C). Abom 48 hours after the ingestion of contaminated meal, patiems p resent with abdominal pain and distemion, vomiting, and passage of bloody stools. C. pe1jringens is also the most common cause of gas gan grene foUowing wound infection or septic abortion. C. tetal'li (choice D) produces a potent neurowxin that causes tetany and generalized muscle spasms. Clostridium botulinum (choice A) produces a neurowxin that causes paralysis. ***Diagnosis: Pseudomembranous colitis, Clostridittm difficile
9 A 24-year-old woman develops an expanding erythematous .___ skin lesion after hiking th rough the woods in Connecticut. The rash disappears, buLl year later. the patiem develops anhralgias and right facial nerve palsy Vv'hich of the following is the most likely etiologic agent responsible for this patiem's symptoms? ***(A) Chlamydia ***(B) Mycobacterium ***(C) Prowzoa ***(D) Rkkettsia ***(E) Spirochete
9 The answer is E: Spirochete. Lyme disease is a chronic infection that begins with a characteristic skin lesion and later vatiably manifests cardiac, neurologic, and joinr dismrbances. The causative a gem is Bon-elia br~rgdorferi , a large spirochete rhat is rransmitted from its animal reservoir to huma115 by the bite of the deer tick (Lxodes). 13. burgda~fe ri reproduces althe site of inoculation, spreads to regional lymph nodes, and is eventually dtsseminated throughomthe body. Untreated Lyme disease is chronic, with petiods of remission and exacerbation. Stage 1 is charactetized by erythema chronicum migrans, a skin lesion that appears at the site of the tick bile. Stage 2 features migratory musculoskeletal pain and the development of cardiac or neurologic abnormalities (meningitis and facial nerve palsy). Stage 3 begins months to years after infection and involves joint, skin , and neurologic abnonn alities. Over half of these patients develop a severe arthritis of the hips and knees, which is indistinguishable h·om the symptoms of rheumatoid arthritis. ***Diagnosis: Lyn1e disease
10 A 3-year-old child attending a daycare center develops fever, chills, generalized rash, and a stiff neck. The chUd becomes hypotensive and expires the next day Postmortem examination demonstrates bilateral adrenal hemonhages {shown on light). Which of the following etiologic agents is the most likely cause of this childs disease? ***(A) Haemophiltts injl.ttenzae ***(B) Klebsiella 1meumoniae ***(C) Neisseria meningitides ***(D) Strepto cocctiS pnewnoniae ***(E) Tr·el'onenta pallidum
10 The answer is (: Neisseria meningitides. Acme meningococcal meningitis may develop rapidly and is often fatal. Meningococcal sepsis is marked by profound endotoxemic shock and disseminated imravascular coagulation, knowt1 as Waterhouse-Friderichsen syndrome. Airborne transmission in crowded places (e.g., schools or barracks) can cause "epidemic meningitis.- Fever, malaise, petechial rash, and adrenal hemorrhages are common. Although they may cause meningitis, choices A, B, and D are nor typically associated \vith Waterhouse-Friderichsen syndrome. Diagnods: Meningococcal meningitis, Waterhouse-Friderichsen syndrome
11 A 45-year-old woman from lndia develops paroxysms of feve r, chills, and severe h eadache while vis iting her brmher in the United States. Physical examination demonstrates hepawsplenomegaly and general pallor. Laboratory studies show anemia and hyperbilirubinemia. Her utine appears dark on visual inspection, and a urin e dipstick is positive for hemoglobin. The patient develops tonic-clonic seizures and becomes comatose. Which of the followi ng pathogens is most likely responsible for this patient's symptoms? ***(A) Naegleria Jowleri ***(B) Pla.~modium falciparum ***(C) P/a.-;modium vivax ***(D) Schistosoma 1taematobiLtm ***(E) Trypanosoma bmceii
11 The answer is B: Plasmodium falclparum . Malaria is a mos- L...- quito-borne illness that infects over 200 million persons per year world\vide. There are four species of Plasmodium: P Jalciparum , P. vivax, P. a vale, and P. ma lariae. All of these organisms infect erythrocytes, but P. Jalc i parum causes the most severe disease. ln malignant"" malaria caused by P. falciparum ," ischemic injm)' to the b rain causes a range of symptoms, including somnolence, hallucinations, behavioral changes, seizures, and even coma. The liver, spleen, and lymph nodes are darkened by macrophages chat are filled with hemosiderin and malmia pigmems. Naegleria fowleri (choice A) is associated with a fatal rype of meningitis. Schistosoma l!aematobiwn (choice D) is associated with bladder infections but does not cause the hematologic symptoms seen in this patiem. ***Diagnosis: Malaria
12 A '56-year-old woman undergoes multidrug chemotherapy for breast cancer. After 10 days, she develops cough , feve r, and respirawry distress. An X-ray film of the chest shows multiple areas of consolidation, with a large cavity in the right upper lobe. Despite vigorous therap y. the patiem dies. Examination of the lungs at auwpsy discloses multiple, sharply delineated, gray foci with hemonhagic borders. A section of lung impregnated \vilh silver is shown in the image. Which of the following mechanisms of disease best accounts for these pathologic findings? ***(A) Allergic inOammation ***(B) Necrosis of type 1 pneumocytes and formation of hyaline membranes ***(C) Pe1ivascular plasma cell infiltrate with obliterative endarteritis ***(D) Production of toxins d1at inhibit protein synthesis ***(E) Thrombosis of blood vessels invaded by hyphae
12 The answer is E: Thrombosis of blood vessels invaded by hyphae. The photomicrograph displays mvasive aspergillosis. This is the most serious manifestation of Aspergillus infection, occunmg almost exclusively as an oppommisLic infection in persons wiLh compromised immunity. Aspergillus readily invades blood vessels and causes thrombosis and local infarction. Branching hyphae (visualized by silver stain) are found in the walls and lumens of pulmonary vessels. ***Diagnosis: Invasive aspergillosis
13 A 30-year-old woman presems with persistent dry cough, fatigue, and low-grade fever. Physical examinaLion shows marked pallor, respiratory distress, nasal flaring, and intercostal retractions. A chest X-ray reveals diffuse, bilateral interstitial infiltrates. A photomicrograph of bronchoalveolar lavage is shown. Which of the following etiologic agents is most likely responsible for this patiem's pulmonary disease? ***(A) As11ergillus fumigatlt5 ***(B) Legionella tmeumoniae ***(C) Mycoplasma pneumoniae ***(D) Pneumocystis jiroveci ***(E) Respiratory syncytial virus
13 The answer is D: Pneumocystis jiroveci. P. jiroveci (formerly P. carinii) was identified in malnourished infants at the end of 'vVorld War ll. lt causes progressive, often fatal pneumorua in persons with impaired cell-mediated immunil:)' and is one of the most common opportunistic pathogens in persons with AIDS. The organism is now classified with the fungi. The infection begins with the attachment of trophozoites to Lhe alveolar lining. Trophozoites feed, enlarge, and transform into cysts within the host cells. Eventually, the cysts burst, releasing new trophozoites. Progressive consolidation of the lung ensues. Although the other choices cause pneumonia, they do not exhibit these charactetistic cysts. ***Diagnosis: Pneumocystis pneumonia
14 A 28-year-old man presents with sudden onset of fever, chills, and a productive cough with blood-tinged sputum. His past medical history is significant for a splenectomy following a motor vehicle accident 3 years ago. An X-ray of the chest demonstrates consolidation of the right middle lobe. Sputum culture shows Gram-positive diplococci. ¥Vhich of the following is the most likely cause of thi.s patients respiratory infection? ***(A) Klebsiella 1me~tmo niae ***(B) Legionella pneumophila ***(C) Mycoplasma pneumoniae ***(D) Staphylococcus m~reus ***(E) Streptococws pneumoniae
14 The answer is E: Streptococcus pneumoniae. S. pneumoniae (pneumococcus) causes pyogeruc infections involving the lungs (pneumonia), middle ear (otitis media) , sinuses (sinusitis), and meninges (meningitis) It is one ofthe most common causes of community-acquired pneumonia. Consolidation of lung parenchyma typically produces lobar pneumonia, whid1 passes through four stages: (l) congestion and edema, {2) red hepatization, (3) gray hepatization, and (4) resolution. During the acme phase, the alveoli are packed with neutrophils, fibrin, and debris. Pneumonia is caused by the other organisms with much lower frequency. ***Diagnosis: Pneumococcal pneumonia
15 An 11-year-old boy presents with tea-colored urine and reduced urine output. He was seen for acute pharyngitis 3 weeks previously: Physical examination shows puniness around the eyes and pining edema of the lower extremities. Temperature and blood pressure are normaL Utinalysis reveals 2.+ hematuria and 3+ protein uria. Blood analysis discloses reduced semm levels of C3 and an elevated titer of antistreptolysin 0 andbodies. This patient's renal disease is most likely mediated by which of the following mechanisms? ***(A) Antineurrophil cytoplasmic amoamibodies ***(B) Deposition of circulating immune complexes ***(C) Directly cytotoxic lgG and lgM amibodies ***(D) IgE- mediated mast cell degranulation ***(E) T cell-mediated delayed hypersensitivity reaction
15 The answer is B: Deposition of circulating immune complexes. Infection with Streptococcus pyogenes causes tvvo major nonsuppurative complications, namely rheumatic fever and acute postsu-eptococcal glomeruloneph1itis. Postsu-eptococcaJ glomerulonephritis is a classic immune complex-mediated disease that is associated with nephritic syndrome. Posrsrrepwcoccal illnesses are not mediated by any of the other choices. ***Diagnosis: Poststreptococcal glomerulonephritis
16 A 59-year-old man with colon cancer is treated with chemotherapy. Two months later, he develops increasing cough and respiratory distress. A chest X-ray shows diffuse bilateral interstitial infiltrates. Spun.lm cultures are negative, and the patient does not respond to antibiotic therapy. A lung biopsy reveals acute and chronic interstitial pneumonitis. There are enlarged cells with prominent, dark-blue nuclear inclusions (shown in the image). Which of the following is most likely responsible for this patient's pulmonary condition? ***(A) Cytomegalovirus ***(B) Epstein-Barr virus ***(C) Herpes simplex virus ***(D) Mycoplasma ***(E) Pneumocystis jiroveci
16 The answer is A: Cytomegalovirus (CMV) . CMV infection induces interstitial pneumortia in infants and immunocompromised persons. InJected alveolar ceUs show cytomegaly and display a single, dark basophilic nucleaT inclusion surrounded by a halo. The virus may be Lransmirted from mother to child in mero or acquired during delivery. In adullS, CMV is transmitLed through sexual encoumers, blood transfusions, rransplamarion, and even duough the inhalation of infectious viral panicles. Central nervous symptonls predominare in symptomatic infants and children. In adults, the virus produces mostly respiratory and gastrointestinal symptoms but does not cause encephalitis. Herpes simplex virus {ch oice C) also feamres intranuclear inclusions (also surrounded by a clear halo) but does not cause ch ronic imersddal pneumonia. ***Diagnosis: lnLerstilial pneumoo.iLis, cytomegalovirus pneumonitis
17 A 6-month-old female infam is brought to Lhe physician with a 2.-day history of severe cough, wheezing, and respiratory distress. Physical examination shows rhinitis, mild cyanosis, and fever. Which of the following is the most likely etiology of d1is child's pulmonary infection? ***(A) Adenovirus ***(B) Cywmegalovi rus ***(C) Parainfl uenza virus ***(D) Respirawry syncytial vims ***(E) Rhinovirus
17 The answer is D: Respiratory syncytial virus (RSV). R5V is an RNA vints, which i.s the m~jor cause of bronchiolitis and pneumonia in infams. RSV bronchiolitis or pneumonitis presems with e.:-:p iracory and inspiratory wheezing, cough , and hypere;...'}Jansion of both lung fields. Hyperinflation, imerstidal infiltrates, and segmented atelectasis are expected findings on chest X-ray. The illne.c;s is usually self-limited and typically resolves within 1 to 2 weeks. Monality ls low in healthy babies. The oLher viruses cause pn eumonia much less frequently. ***Diagnosis: Respirawry syncytial virus, bronchiolitis
18 A 75-year-old woman died in February o[ respiratOl)' failure after a febrile disease of 1 week in duration. An au topsy shows necrotizing bronchitis and diffuse , hemonhagic necrotizing pneumonia. Which o[ Lhe following pathogens was most likely responsible for this patients fatal pulmonary infection? ***(A) lnfluenza vims ***(B) Norwalk-like viruses ***(C) Respiratmy syncytial vims ***(D) Rhinovirus ***(E) Rotavirus
18 The answer is A: Influenza virus. In fluenza A and B arc RNA vi1uses. Influenza infections are common in the >vin te rtime, wirh the severity of the illness depending on the immune status of the individual. Patients typically presem with fever, tachypnea, conjunctivitis, and pharyngeal inflammation. In severe cases, they may develop extreme respirat01y distress and prostration Influenza affects all segments of the populati on, bm severe cases are more commonly seen among the very young and the elderly Rhinoviru..c; (choice D) i.s the most f1-equem cause of the .. common cold." Notwatk-like virus (choice B) and rmavin.LS (choice E) cause diarrhea in children. Infection with respiratmy syncytial virus (choice C) is commonly seen. in duldren Lmder 2 years of age. ***Diagnosis: In fluenza vi rus, pneumonitis
19 A 4-year-old girl, whose parents recenlly immigrated from Ecuador, presents with high fever, cough, and skln rash of 3 days in duration . Her parents report that her rash began in the form of pink papules behind the ears and spread around her body Physical examimuion shows an extensive maculopapular rash over the childs face, neck, mmk, and limbs. She displays smal1 white spots on buccal surfaces. Tllis patients skin rash is most likely caused by infection \1./ith which of the following agents7 ***(A) Candida albicans ***(B) Epstein-Barr virus ***(C) Measles virus ***(D) Mumps virus ***(E) Rotavirus
19 The answer is C: Measles virus. Measles virus is an RNA vims Lhat causes an acme, highly contagious, self-limited iUness Lhat is characterized by upper respiraLory tract symptoms, fever, and rash . The measles virus, which is transmitted in respirawry droplets and secretions, is primarily a disease of children, bm its dfects may be particularly severe in adults. The skin rash results from 1.he reaction ofT cells with infected cells of Lhe vascular endothelium. "Koplik spots" appear on the postetior buccal mucosa and consist of minute gray-white dots on an erythematous base. Although measles is usually a selflimited disease, measles pneumonia (particularly in adults) is a serious malady that may be fatal. Epstein-Ban virus infection and mumps (choices Band D) do not present with gen eralized rash. Rotavirus infection (choice E) is the most common cause of severe diarrhea worldwide. The yeast, Candida albicans (choice A), usually causes localized infection. ***Diagnosis: Measles
20 A 50-year-old woman presents whh increasing dry cough and shortness of breath that has lasLed for 3 weeks. She is a bird fancier, and her house is filled Virlth parrots. An X-ray film of the chest shows diffuse lung infiltrates. Spurum cultures are negative, and the patiem does rrot respond to antibiotic therapy. A transbronchial aspirate reveals chronic interstitia] pneumonia. The patient responds well to tetracycline. vVhkh of the following is the most likely etiologic agenl responsible for this patients symptoms? ***(A) Chlamydia ***(B) Fungus ***(C) Gram-positive bactetium ***(D) Mycobacterium ***(E) Rickettsia
20 The answer is A: Chlamydia. Psittacosis is a self-limited pneumonia transmitLed to humans from birds. The etiologic agent, Chlamydia psittaci, is present in blood, feces, and feathers of infected birds. The organism first infects alveolar macrophages, which carry it to the liver and spleen, where it reproduces. The organism is then distributed hematogenously to produce a systemic infection. C. psittaci reproduces in alveolar lining cells, whose destmction elicits an inflammatory response and interstitial pneumonia. Type 11 pneumocy1es are hyperplastic and may show charactetistic chlamydial cytoplasmic inclusions. Clinically, the disease presents with persistent dry cough, fever, headache, malaise, myalgias, and anhralglas. The ocher agents listed do nor cause chronic mterstitial pneu- . moma. ***Diagnosis: PsiLtacosis
21 A 5-year-old boy dies of respiratory insufficiency and complications of pneumonia. Histologic examination of the lungs at autopsy shows giant cells with up to 100 nuclei (shown in the image). Which of the following viruses most likely caused this childs fatal respiratory u act infection? ***(A) Aden ovi rus ***(B) Cytomegal OVl rus ***(C) Measles virus ***(D) Mumps nrus ***(E) Rube1la\1rus
21 The answer is C: Measles virus. Measles virus can cause f·usion of infected cells, producing multinucleated cells termed "Wanhin-Finke1dey giant cells." These multinucleated giam cells are pathognomonic of measles infections. Cywmegalovirus-infected cells (choice B) are very large and contain nuclear and cytoplasmic viral inclusions, but they are not multinucleated. Adeno•"lrus (cholcc A) also feawrcs intranuclear inclusions but not multinucleation. Mumps and rubella viruses (choices D and E) induce a mononuclear infiltrate composed of lymphocytes, macrophages, and plasma cells (no giant cells). ***Diagnosis: Measles
22 A 4'5-year-old man who IS a hem-y smoker complams of sudden onset of h1gh fever and ch1lls. nonproductive cough, and chest pam. An X-ray film of the chest shows bilateral, diffuse, patchy infiltrates. The patient repor ts that a number of similar cases have occurred recently in the building where he works. He responds w antibiotics and recovers. Which of the following is the most likely a reservmr for the microorganism mar is responsible for this patient's respi ratory traer infection? ***(A) Cooling towers ***(B) Elevawr shafts ***(C) Floor cleaners ***(D) Heat pumps ***(E) Industrial solvents
22 The answer is A: Cooling towers. Legionella pneumophila causes a pneumonia that ranges from mild lO a severe lifethreatening, necroti~ing pneumonia refetTed to as "legionnaire disease." The bacterium is found in natural bodies of fresh water and survives chlorination, atl01.ving it to proliferate ln cooling towers, water heaters , humidifie rs, and ventilation systems. l egionella pneumonia begins when microorganisms enter alveoll, where they are phagocytized by resident macrophages. Bacteria multiply within macrophages and are released to infect ne\v macrophages. The disease presents as an acme bronchopneumonia, with a diff·use and pau:hy pattern of infiltration. None o[ the Olher situations creales suitable conditions for bacter ia to multiply. ***Diagnosis: Legionnaire disease
23 A 37-year-old man is admitted to the hospital with a productive cough, fever, and night sweats. An X-ray film of the chest shows an ill-defined area of consolidation at the periphety of the right middle lobe and mediastinal lymphadenopathy. Sputum culture grows acid-fast bacilli. Lymph node biopsy in this patient would most likely show which of the following pathologic findings? ***(A) Caseating granulomas ***(B) Follicular hyperplasia ***(C) Nodular amyloidosis ***(D) Noncaseating granulomas ***(E) Pumlem abscess
23 The answer is A: Caseating granulomas. Tuberculosis is a chronic, communicable disease in which the lungs are the prime Larger. The disease is caused principaUy by Mycobacterium tuberculosis Jwminis (Koch baciJlus), bur infection with other species occu rs, nmably M. tttb ercu losis bovis (bovine tuberculosis) Primary tuberculosis consists of lesions in the lower lobes and subpleural space, rderred to as the Ghon focus. The infection then dnnns to hilar lymph nodes. The combinarion of Chon focus and hilar lymphadenopathy is known as "Ghon complex." The t)1)ica! lesion of tuberculosis is a caseous granuloma, wilh a soft core surrounded by epilh el]oid macrophages, Langhans giant cellc:;, lymphocytes, and pe1ipheral fibrosis. Noncaseating granulomas (choice D) are a feature of sarcoidosis, among other causes. ***Diagnosis: Primary tuberculosis
24 A 24-year-old woman with insulin-dependem (type 1) diabetes mellitus presents to the emergency room with severe respiratory distress and pleuritic chest pain. The pmiem has a history of antibiotic-resistant sinusitis. Physical examhuuion reveals pe1iorbital edema and a mucopmulent postnasal discharge. Despite therapy, the patient dies of acute respiratmy failure. Cross secnon of the lu11g at autopsy shows hemorrhagic infarction. The vessel in the center of the field is occluded by a septic thrombus (sl1own in the image). These clinicopathologic featu res are typical of which of the fo ll O\\rlng pulmonary diseases? ***(A) Haemophi/Lts injl uenzae pneumonia ***(B) Mucormycosis ***(C) Parainfluenza vims pneumonia ***(D) Pneumocystis jiroveci pneumonia ***(E) Psittacosi.s
24 The answer is B: Mucormycosis. Environmental fungi, such as Rhi zopt-ts, Muco r, Rhizomucor, and Absidia species, can produce necrotizing opportunistic infections that begin in the nasaJ sinuses or lungs. Mucor is ubiquitous in the nasal sinuses and invades sun ounding tissues. The hard palate or nasal cavity is typically covered by a black crust, and the underlying tissues become friable and hemorrhagic. The fungal hyphae grow imo arte ries, causing devasLating and rapidly progressive septic embolic infarctions. There are Ll1ree principal fom1s of mucormycosis, namely rhinocerebral, pulmonary, and subcutaneous. Pulmonary mucom1ycosi.s is usually fatal. Microscopic examination shows a p urulem ane1itis \vi.th thrombi composed of hyph ae. Mucormycosis should be suspected in patients who present with a paranasal sinusitis unrespon sive lO antibiotic treatmem, particularly those who also have an underlying chronic disease (e.g., diabetes or leukemja). The other choices do not show rapidly progressive, septic, embohc infarctions of the lungs. Parainfluenza vintS (choice C) does not cause thrombosis or infarction. Pneumocystis j iroveci pneumonja (choice D) is noninvasive and causes interstitial pneumonitis. ***Diagnosis: Pulmona1y rnuconnycosis
25 A 3-year-old boy is brought to the emergency room by his parents with a high fever, sore throat, hoarse voice, and acute respiratory distress. On physical examination, the child is observed to be leaning forward with a hyperextended neck The epiglottis appears swollen and erythematous. Which of 1.he following is the most likely cause of this child's upper respiratory tract infection? ***(A) Bordetella pertussis ***(B) Haemophiltis injlue11zae ***(C) Klebsiella 1mmmoniae ***(D) ParainDuenza vlms ***(E) Resp1rarmy syncytial vims
25 The answer is B: Haemophilus influenzae. H. in.fluenzae is a Gram-negative coccobacillus that is the leading cause of meningitis and epiglotritis in children world\vi.de. Infections may also hwolve the middle ear, sinuses, facial skin, lungs, and joints. H. influenzae spreads from person to person in respiratory droplets and secretions. Inflammation of the epiglottis, aryepiglottis sinus, and pyriform recess produces signHicant ai rway obstruction, which can be fatal. Since the widespread use of the HiB vaccine in the United States, invasive disease due to H. irif7uenzac type B in pediatric patients has been reduced by 80% to 90%. Other agents, such as Streptococcus pyogenes , S. pneumoniac, and Staphy lococcu_s aureus, now represent a larger proponion of pediatric cases of epiglottitis in the United States. ***Diagnosis: Acute epiglottitis, Haemophiltt.~ mfluerzzae
26 A 31-year-old man with AlD$ complams of painful swallowing. Physical examination of his oral cavi ty demonstrates a whitish membrane cove1ing much o[ his tongue and pala1.e. Endoscopic e:xamination reveals the same whitish membrane covering his esophageal mucosa. An endoscopic biopsy is shown in the image. Which of the following is the most Ukely etiologic agem responsible for this patient's symptoms? ***(A) Chlamydia ***(B) Fungus ***(C) Protozoon ***(D) Rickettsia ***(E) Spirochete
26 The answer is B: Fungus. The genus Candida comprises over 20 species of fungi, which i.nclude the most common opportunistic pathogens. Many Car1dida species are endogenous human flora. When the no1mal bacte1ial flora that limit f1.mgal growth a re suppressed, the yeast converts to an invasive form, eliciting an inflammatory reaction. Thrush signilies candida] infection of the tongue and mucous membranes of the mom h. It consists of friable, white , curd-like membranes adherent to the affected area. Removal of this membrane leaves a painful bleeding surface. The other choices do not fonn thrush. ***Diagnosis: Tlu·ush, Candida esophagitis
27 A 35-year-old man with AIDS presents to the emergency room vvith a 2-week history of progressive weight loss and productive cough with bloody sputum. An X-ray film of Lhe chest shows a cavitaty lesion in the right middle lobe. A smear of the spurum reveals many acid-fast rods. Which of the following is Lhe most likely diagnosis? ***(A) Ghon complex ***(B) Miliary tuberculosLs ***(C) Plimary tub erculosis ***(D) Secondaty tuberculosis ***(E) Tuberculous pleuritis
27 The answer is 0: Secondary tuberculosis. Secondary tuberculosis results from the proliferation of M. Luberculosi.s in a person who has been previously infected and has mounted an immunologic response_ The source of infection is usually dormanr bacteria from old granulomas bm may also represem a newly acquired infection. Various conditions may predispose lO the reemergence of endogenous microorganisms, including immunosuppressive states such as cancer, chemmherapy, immunosuppressive therapy, AIDS, and old age. The lungs are the most common sires of reinfection. The bacilli elicit an acute inOammatory response that leads to extensive tissue necrosis and the production of tuberculous cavities. Clinically, patients present with cough, low-grade fever, malaise, fatigue, anorexia, weight loss, and night sweats. The other conditions are not associated with pulmonary cavitation. ***Diagnosis: Secondary mberculosis
28 A 1-year-old girl is brought to the clinic in January by her parents because of a fever, runny nose, congestion, "barking" cough, and difficulty breathing. Physical examination shows a red throaL Parainfluenza virus is Lr;olated . Which of the follmving is chief1y responsible for the barking cough and inspiratory stridor seen in this patient? ***(A) Lmyngeal fibrosing strictures ***(B) La1yngorracheal papillomatosis ***(C) Latyngotracheitis ***(D) Tonsillar hyperplasia ***(E) Vocal cord paralysis
Zl The answer is C: Laryngotracheitis. Parainfluenza vi ruses cause acute upper and lower respiratory tract infections particularly in young children. These RNA viruses are the most conm1on cause of laryngotracheobronchitis, which is referred to as "croup." The infection is charaClerized by a subglouic swelUng and airway obstruction, which lead to acme respiratory distress. The infection spreads fTom person to person thmugh contaminated respiratOJy aerosols and secretions. The parainfluenza virus infects and kllls ciliated respiratmy epithelial cells and elicits an inflaJru11awry response. When lary11gotracheitis occurs, localized edema compresses the upper ai rway enough to obstruct breathing. Symptoms associated with croup include fever, hoarseness. barking cough, and inspiratory snidor. The other conditions are not features of parainfluenza infection. ***Diagnosis: Croup, parainfluenza vims
29 A 7-year-old black girl with sickle cell anemia presents witl1 sudden onset of fauguc and joint pain. Physical examination shows marked pallor of the skm and mucous membranes and anhralgias of the lower limbs. The mmher mdicated that the child had recemly recovered from a minor "flu.·· The CBC shows pancytopema. Wh1ch of the followmg agents JS responsible for thiS paucms symplOITJS? ***(A) Adenovuus ***(B) Norwalk \irus ***(C) Paramfluenza \1TUS ***(D) Parvovmts 819 ***(E) RubellavmJS
29 The answer is D: Parvovirus 819. Human parvovirus Bl9 is a DNA virus that causes systemic infections characte1ized by rash, anhralgias, and a transient defect in e1ytluopoiesis. The virus is spread from person Lo person through respiratmy secretions. Infections are common in children. The virus is cytopathic for erythroid precursor cells in the bone marrow. The nuclei of infected cells are enlarged, and the chromatin is displaced to the periphery. MosL patients suffer a mild exanthem known as erythema infectiosum. However, in patients with chronic l1emolytic anemia (e.g. , sickle cell disease), this transient interruption in erythropoiesis causes a potenLially fatal condition kno\'\'11 as "aplastic c1isis." The other choices are not associated with anemia. ***Diagnosis: Aplastic crisis
30 A 2-day-old prcmalUrc infant develops tonic-donie seizures m the nursery. ACT scan of the head shows microcalcifications. Three days late r, the neonate dies. The brain at auwpsy reveals large areas of subcpcndymal necrosts with calcification (shown in the image, see arrows). Which of the following pathogens is the most li kely cause of death in this neonate? ***(A) Cytomegalovirus ***(B) Herpes simplex type 1 ***(C) Human immunodeficiency virus ***(D) Toxoplasma gondii ***(E) Trel'onema pallidmn
30 The answer is D: Toxopla_~ma gondii. Toxoplasmosis is a worldwide disease caused by the prowzoan T. gondii. Most infections are asympwmatic, but a devastating necrotizing disease may occur when they involve the fetus or an immunocompromised adltlt. Infection of the central nervous system produces a necrotizing meningoencephalitis, which, in the most severe cases, results in destruction of brain parencJl }'l11a, cerebral calcHication, and hyd rocephalus. Ocular infections cause chorioretinitis. None of the other pathogens induce th ese ch aracteristic pathologic findings . ***Diagnosis: Congenital wxoplasmosis, TORCH syndrome
31 A 2-month-old infant presems with fever to 38.6°C (103¢F) and neck tigidity. Cerebrospinal fluid shows numerous neuLrophils, decreased glucose, and increased protein Gramposidve cocci are presem. Which of the following is the most likely cause of meningitis in this neonaLe7 ***(A) Group B streptococcus ***(B) Haemophilus injluenzae ***(C) Neisseria gonorrhoeae ***(D) Neisselia meningitidis ***(E) Staphylococcus aw'eus
31 The answer is A: Group B streptococcus. Several thousand neonatal infections with group B streptococci occur in the Uni ted States every year. About 30% of infected infams die. The mher choices are much less common causes of meningitis in this age group. Meningococci are Gram-negative organisms. ***Diagnosis: Neonatal bacterial meningitis
32 A 40-year-old man, who works in a grain silo in the Amelican Southwest, preseniS with skin rash , headache, and feve r. As pan of his duties, he is requ.ired to uap rodeniS. Physical examination shows bilateral swelling or the parotid glands and a purpuric, maculopapular rash that spares the palms , face , and soles. Biopsy of lesional skin discloses intrace]]ular microorganisms up to l J.tm in length within capillary endothelial cells. Which of the following pathogens is responsible for this patient's d i.sease? ***(A) Bonelia burgdmferi ***(B) C/1/amydia ps itlac i ***(C) Coxiel la hurnetii ***(D) Richettsia tytJiti ***(E) Toxoplasma gondii
32 The answer is D: Rickettsia typhi. Endemic typhus is a severe vasculitis transmitted by R. typhi through the bite of infected lice. The disease begins with localized infection of capillary endothelium. which progresses w systemic vasculitis. Mononuclear cell infi ltrates are found in multiple organs and are typically arranged in typhus nodules. Louse-borne typhus is characterized clinically by fever, severe headache, and myalgias, followed by the appearance of a maculopapular rash on the upper trunk and axillary folds, spreading centrifugally to the extremities. The other choices do not infect endothelial cells or produce a vasculitis. ***Diagnosis: Endemk ryphus
33 A 40-year-old immigram from Brazil presems with a 2-month history of fever, weight loss, and prominent muscle pain. She repons a "suange feeling around her hean." Physical examination shows tachycardia and an irregular heart beat. Laboratory studies reveal marked peripheral eosinophilia. The patient dies 2 days later of an arrhythmia. A photomicrograph of the heart at autopsy is shown. Which of the following pathogens is the most likely cause of ca r· diac arrest? . .- ***(A) Pneumocystis jiroveci ***(B) Schistosoma mansoni ***(C) Treponema pallidttm ***(D) 1hchwis trichiura ***(E) 1iypanosoma cruzi
33 The answer is E: Trypanosoma cruzi. Chagas disease is an lnsect-bome systemic infection in humans caused by t.h e protozoan T auz i. Acute manifestations and long-term sequelae of infection occur p1imarily in the hean and gastrointestinal uact.. The infections are endemic in Central and South America, where they are transmitted by the Reduviid ("kissing") bug, which hides w1tbin the cracks and straw roofs of older homes. The parasite reproduces within the myocardium and causes myocarditis. The other pathogens do not cause myocarditis. ***Diagnosis: Chagas disease
34 A 45-year-old man presents with chesl pain, fever, productive cough, and rust-colored spurum. The patient was diagnosed with mberculosis in his early 20s. A chest X-ray shows multiple, nodular infiltrates and cavitary lesions. A lung biopsy reveals necrotizing inflammation and vascular thrombi with branching fungal hyphae. Which of the following is the most likely diagnosis? ***(A) Actinomycosis ***(B) Aspergillosis ***(C) Candidiasis ***(D) Cryptococcosis ***(E) Histoplasmosis
34 The answer is 8: Aspergillosis. Fungus balls (aspergillomas) consist of rounded, lobulated masses of h yphae and occur in patients with a previous hiswry of cavitating pulmonary disease (e.g., pulmona1y tuberculosis). Aspergi llus is a common environmental fungus that causes opportunistic infections in U1e lungs. Inhaled spores germinate in the warm humid atmosphere provided by cavitary lung lesions, filling them with masses of hyphae. The organisms generally do not. invade the lung parenchyma. There are three different types of pulmonary aspergillosis, namely allergic bronchopulmonary aspergillosis, aspergillomas, and invasive aspergillosis. Candidiasis (choice C) is incorrect because Candida infections an~ not typicaUy angioinvasive. The other choices do not show characteristic branching fungal hyphae. ***Diagnosis: Pulmonary aspergillosis, aspergilloma
35 A 25-year-old man, who recently rerurned from a trip 1.0 Central America, presents with right upper quadrant pain and fever of 3 weeks in duration. A CT scan of the abdomen shows a large cystic cavity of the liver, after which a 12.-cm abscess is excised (shown in the image). Histologic examination of the lesion reveals chocolate-colored, odorless debris, surrounded by a shaggy fibrin lining, scant inOammatory reaction, and organisms auached to adjacem cells. Which of the following pathogens is responsible fo r this patients liver abscess? ***(A) Clonorchis sinensis ***(B) Entamoeba histolytica ***(C) Giardia lmnblia ***(D) Schistosoma mansoni ***(E) Streptococcus pyogene.s
35 The answer is 8: Entamoeba histolytica. E. lti.stolytica resides in the colon of in fected pe rsons and is transmitted by fecaloral cont:acL The trophozoiles invade submucosal veins of the colon, enter the ponal circulation, and gain access to the liver. The amebae kill hepatocytes, producing a slowly expanding, necrotic cavity. The abscess is filled with a dark brown material that resembles anchovy paste. An amebic liver abscess can mpture and extend into the peritoneal cavity Although the oth er choices may involve the liver, they do not cause hepatic abscess. ***Diagnosis: Amebic liver abscess
36 A 24-year-old \Moman presents with severe vomiting, abdominal cramps, and diatThea 2 hours after dining at a local restaurant. Many of 1.he customers that night reponed similar symptoms. Which of the fo llowing mecharusms of disease is chiefly responsible for the development of gastroimesdnal sympLOms in !his patle.nr? ***(A) Activation of membrane-associated tyrosine kinase ***(B) Exposure to preformed enterotoxin ***(C) IgE-mediated mast cell degranulaLion ***(D) Immune-complex deposition and complemem activation ***(E) Receptor-mediated stimulation of intracellular cAMP
36 The answer is B: Exposure to preformed enterotoxin. Stal'ltyJococcus attreus food poisoning is caused by the ingestion of food comaminated with preformed, heaL-stable enterotoxin B. Ombreaks occur when food handlers inoculate foods such as meat or dairy products (salad dressings, cream sauces, and custard-fi1led pasnies) wilh contaminated wounds or infected nasal droplets. Staphylococcal food poisoning typically begins less than 6 hours after a meal. Nausea and vomiting usually resolve with in 12 hours. The other choices do not initiaLe rapid gastrointestinal symptoms. ***Diagnosis: Staphylococcal food poisoning
37 A 48-year-old man with AIDS is admjtted to the hospital with a fever of 38°C (l03°F), night sweats, persistent cough, and prolonged diarrhea. His CD4 celJ count is less than 300/).JL Stool culture reveals the presence of acid-fast bacilli. 'vVhich of the following pathogens is responsible for this patients respiralOry and gastrointestinal disease? ***(A) Campylobacterjejuni ***(B) Cryptosporidit@ ***(C) Clostridium peifringens ***(D) Mycobacterium avium-i ntracellttlare ***(E) Slreptococws pyogenes
37 The answer is D: Mycobacterium avium-intracellulare. M. avit~m and M. intrace llulare are similar mycobacte1ial species that cause idemical cliseases and are, therefore, classified wgether as M. avium-intracellulare complex (MAC). MAC is a rare , granulomatous, pulmonary disease in lmmunocompetem persons, but it is a progressive systemic disorder in patients with AIDS. One third of all AlDS patients develop oven MAC infections. The proliferation of organisms and the recruitment of macrophages produce expanding lesions, ranging fTom epithelioid granulomas comaining few organisms lO loose aggregates with foamy macrophages. Symptoms associated with MAC resemble those of tuberculosis; however, progressive involvemem of the small bowel produces malabsorption and diarrhea. Camplyobacter jejuni (choice A) produces a self-limited bacterial diarrhea. Clyptosporiditm1 (choice B) is a protozoan that causes diarrhea iJ1 immunocompromised patients but is not associated with respiratmy in rections. ***Diagnosis: Atypical mycobacterial infection mAIDS
38 A 14-year-old gul presenlS with yellow and red crusted lesions around her mouth and arms (shown ln the image). She bas a recent hiswry of imem1iltent low-grade fever. This patients skin lesions are most likely caused by which of the fo ll ov.l]ng microorganisms? ***(A) Staphylococcus epidennidis ***(B) Streptococcus pneumoniae ***(C) Streptococc t~s pyogenes ***(D) Streptococcus viridam ***(E) Treponema pallidum
38 The answer is C: Streptococcus pyogenes. lmpetigo in this patienL represents a localized, Lmraepidermal infection wuh 5. pyogenes. lt spreads by dose comaCL and most commonly affects children. Minor trauma allows inoculation of the bacteria, forming an imraepithelial pusmle that eventually ruptures and leaks a purulent e".'udare. 5. pneumoniae (choice B) is a major cause of lobar pneumonia, otitis media, sinusitis and meningitis 5. viridans (choiCe D) is a major cause of bacterial endocarditis. Treponema pallidtvn (choice E) produces a maculopapular rash of the palms and soles in secondary syphilis. ***Diagnosis: Impetigo
39 A 28-year-old dntg abuser presents with high fever, chUls, productive cough, hemoptysis, and right-sided chest pain. Physical examination shows the stigmata of intravenous drug abuse, as well as splenomegaly and pleuritis. Auscultation reveals a sysLolic ejection murmur over the tricuspid area. Which of the following is the most likely etiology of this parems valvular heart disease' ***(A) Candida albicans ***(B) Legionella pneumophiLa ***(C) Mycobacterium tuberculosis ***(D) Mycoplasma pnewnoniae ***(E) Stapllyl ococcus aureus
39 The answer is E: Staphylococcus aureus. 5. aureus is a Gram-positive coccus that is tile most common cause of suppurative infections involving the skin, joints, and bones. lL is also one of the most common causes of acute bacterial endocardi tis. This infection features colonization of heart valves or mural endocardium, leading to the formation of friable vegetations composed of thrombotic debris and microorganisms. Bacterial growth is often associated with dest.ruction of the underlying valve tissue. Tricuspid insufficiency sec· ondary to bacterial endocarditis is one of tbe most common complications of IV drug abuse. The most common source of bacteria in these patients is the skin_ The tricuspid valve is infected m half of the cases. The other choices do not cause endocarditis_ ***Diagnosis: Bacte rial endocarditis
40 A 50-year-old woman presents ·...vi th sudden-onset, crampy abdominal pain and watery diarrhea. She also complains of low-grade fever with chtlls, nausea, and vomiting. She ate partially cooked eggs 24 hours prior to the onset of these symptoms. Which of the following pathogens is most likely responsible for this patient's gastrointestinal disorder? ***(A) Clostridium pe1jringens ***(B) Escherichia coli ***(C) Hepatitis A virus ***(D) Salmonella sp. ***(E) Stapllylococcus aureus
40 The an.swer is 0: Salmonella. Nomyphoidal species of Salmonella contaminate a va1iety of foods, including poultry. eggs, meat, and daily products. Salmonella infections are characterized clinically by djanhea, which begins 12 £O 24 hours after ingestion of the contaminated food. Salmonella rood poisoning is self-llnuted, lasting from l to 3 days. The bacte1ia pro· liferate in the small intestine and invade enterocytes, where they produce several toxins that contTibute to the dysfunction of the intestinal epithelium. The mucosal surface of the ileum and colon become acutely inOamed and occasionally ulcerated. Pathogenic Esd1erichia coli (choice B) does not typicaJly inJect eggs. Stap!tylococcus autetts (choice E) characteristically causes diarrhea 1 to 6 hours after ingestion. ***Diagnosis: Salmonella enterocolitis
41 A 30-year-old man presents with inguinal swelling and pain· less penile and perianal ulcers. He admits to having unpro· tected sexual intercourse with multiple partners. A silver slain of a biopsy from a skin lesion (sho\'.rn in the image) reveals organisms clustered in large macrophages. Which of the following is the most likely diagnosis? ***(A) Chancroid ***(B) Gemtal herpes ***(C) Gonorrhea ***(D) Granuloma ingumale ***(E) Syphtlis
41 The answer is D: Granuloma inguinale. Granuloma inguinale is a sexually transmitted, chronic, superficial ulceration of the genital, inguinal, and perianal region. It is caused by Calymmatobacte rium granulomatis , a small Gram-negative bacilltts. The characteristic lesion is a beefy-red superficial ulcer. Microscopically, the dermis and subcutis are infiltrated by macrophages and plasma cells. Skin lesions show microorganisms, termed "Donovan bodies," clustered \\'ithin enlarged macrophages. The other choices do not display visible lnnacellular microorganisms. ***Diagnosis: Granuloma inguinale
42 A 42-year-old man presents with a 1-week history of myalgia, low-grade fever, and swelling o( the left calL The patiem repons recently attencUng a fireman:S pig-roasL Laboratmy data show elevated serum levels of creatine kinase. Examination of a muscle biopsy in this patiem would most likely reveal an infiltrate of which of the following cell types? ***(A) Eosinophils ***(B) Mast cells ***(C) Plasma cells ***(D) Segmented neutrophils ***(E) Smooth muscle cells
42 The answer is A: Eosinophils. Trichinosis is p roduced by the roundworm Tr ichinella spiralis. After mating, the females liberate larvae imo the circulation. The larvae can invade almost any tissue but survive only in skeletal muscle i.n an encapsulated form. Elevated serum levels of creatine kinase indicate muscle cell necrosis. Early muscle involvement elicits an intense inflammatory infiltrate rich in eosinophils. The other cells do not typically respond to acute parasitic infestations. ***Diagnosis: Trichinosis
43 A 32-year-old immigram from Viernam presents with a 4-year hi.swry of progressive loss of eyebrows; patchy hypopigmentation of skin; coarse facial features; and multiple, firm subcutaneous nodules. Acid-fast organisms are seen on skin biopsy. Which of the following is the most likely etiology of this patients condition? ***(A) BacillLLS anthracis ***(B) Mycobacterium lcprae ***(C) Mycobacterium tuberculosis ***(D) 1i·e1)011ema pallidwn ***(E) 1iypanosoma cntzi
43 The answer is B: Mycobacterium leprae. Leprosy (Hansen disease) is caused by M. leprae and appears in two fom1s, namely tuberculoid and lepromawus. The tuberculoid type occurs in patiems who moum an immunologic response, whereas those with the lepromatous form are anergic. Leprornawus leprosy is a chronic, slowly progressive, destructive p rocess involving peripheral nerves, skin, and mucous membranes. Patiems exhibit multiple nodular lesions of the skin, eyes, testes, nerves, lymph nodes, and spleen. The skin infiinates o -.. pand slowly lO distort and disfigure the face, ears, and upper airvvays. There is also Lnvolvemenl of the eyes, eyebrows, eyelashes, nerves, and testes. M_ tuberculosis (choice C) is acid-fast, but does not cause Lhe symptoms listed for this case. Amhrax (choice A), syphilis (choice D), and u·ypanosomLasis (choke E) manifest diffe rentlYDiagnosis: Leprosy
44 A 16-year-old girl presen i:S >vith fever and swollen lymph nodes. Physical examination reveals painful lymphadenopathy in her left axill a. The gl rl remembers that she was scratched by her cat 3 weeks ago. A sllver stain of a lymph node biopsy is shown in the image. Which of the following is the most likely cause oflymphadenopathy in this patient? --- - ***(A) Bartonella henselae ***(B) Eih ene/Ja cormdens ***(C) Pastwrella multocida ***(D) Staphylococcus attretLS ***(E) Streptococcus l'Yogenes
44 The answer is A: Bartonella henselae. Cat-scratch disease is a self-Limited infection caused by B. 1-Jenselae or (more rarely) B. qt~i~ltarw. These bacteria are small, Gram-negative rods that are difficult to culture bm easlly seen in a lymph node biopsy when s tained with silver. B. llenselae multiplies in the walls of small vessels and extracellular collagen fibers at the site of inoculation. Tl1e organisms are carried w rJ1e lymph nodes, where they produce suppurative lymphadenitis. The lymph nodes enlarge and drain through the skin. About half of infected patients present with systemic symptoms such as fever, malaise, rash, and erythema nodosum. Pasteurella multocida {choice C) is associated with wound infection after animal bites. Eihenella conodens (choice B) produces wound infections after human bites. ***Diagnosis: Cat-scratch disease
45 A 12-year-old farm boy from Mexico complains of generalized weakness and abdominal pain. Physical examination shows pallor and pitting edema of the lower extremities. A CBC reveals severe microcytic, hypochromjc anemia (hemoglobin = 8.2 gldL). Whkh of the following pathogens is the most likely cause of anemia in this patient? ***(A) Ascaris lumbricoides ***(B) Giardia Iamblia ***(C) Hookw01m (Ancy lostoma duode r1ale) ***(D) Pinworm (Enterobius vennicrt laris) ***(E) Whipworm (1i-ich11ris trichiura)
45 The answer is C: Hookworm (Ancylostoma duodena/e) . Hookworms are intestinal nematodes that inJect the small boweL A duodenale molts within the duodenum and attaches to the mucosa. With extensive Infections, particularly with A duodenale, considerable blood loss results in iron-deficiency anemia. The other choices do not cause intestinal bleeding and iron-deficiency anemia. ***Diagnosis: Hookwonn, ancylostomiasis
46 A 65-year-old man with a hisLOry of Hodgkin lymphoma develops a painful erythematous rash with a band-like distribmion over 1.he left side of his chest, which becomes vesicular over the next several days. Biopsy of lesional skin is shown in the image. Which of the following is the most likely etiology of this patient's rash7 • ***(A) Cytomegalovirus ***(B) Epstein-Barr virus ***(C) Human herpesvirus-6 ***(D) Human herpesvims-8 ***(E) Varicella-zoster virus
46 The answer is E: Varicella-zoster virus. Varicella-zoster virus initially infects cells of the respirat01y tract or conjunctival epithelium. It then reproduces and spreads via the bloodstream and lymphatic system. First exposure ro the virus produces chickenpox, an acute systemic illness whose dominant feature is a generalized vesicular skin eruption. Reactivation of la tent virus in adults causes herpes zoster. Microscopically, intraepithelial vesicles contain multinucleated giam cells and nuclear in clusions. Human herpesvirus-8 (choice D) is associated with Kaposi sarcoma in pmiems with AlDS. The other choices do not produce vesicular eruptions. ***Diagnosis: Herpes zoster
47 A 23-year-old woman presems with low-grade fever and multiple. painful, vesicular lesions on the vulva. A Pap smear shows multinucleated giam cells with intranuclear inclusions. Which of the following pathogens is the mosL likely cause of this pmient's genital lesions? ***(A) Calymmatobacterium grant4lomalis ***(B) Epstein-Barr virus ***(C) Herpes simplex virus cype 2 ***(D) Human papillomavirus ***(E) 1i·eponema paUidum
47 The answer is C: Herpes simplex virus type 2. Herpes simplex vi ruses are common human pathogens, which most frequent!)' produce recurrent painful vesicular eruptions of the skin and mucous membranes. The other choices do not show grouped vesicles. CaTymmatobacte1·ium granttlomatis (choice A) is associated with a painful genital ulcer (chancroid). Human papillomavi rus (choice D) relates to genital warts. Trepor1ema pallidum (choice E) causes syphilis. ***Diagnosis: Genital herpes
48 TI1e patiem described in Question 47 asks you about the possibi. lity of future outbreaks and the risks of se.lfual transmission. ln addressing her concerns, you might consider that the vi rus is harbored in a latent fonn in which of the fol lowing anatomic locations? ***(A) Germ cells of the ovary ***(B) Glandular epithelium of the endocervix ***(C) Mucosa of the external genitalia ***(D) Sensory neurons of sacral ganglia ***(E) Squamous mucosa of the exocervix
48 TheanswerisD: Sensory neurons of sacral ganglia. He1pesvirus ascends from genital lesions along sensory neurons and survives in a latent fom1 in the sacral ganglia. Nonspecific stimuli (including sexual intercourse and menses) can reactivate the virus, which then descends along axons to the genital mucosa, causing recurrent b listers on the external and internal genitalia. ***Diagnosis: Gen ital herpes
49 A 6-year-old girl presents \"'ith intense perianal itching, especially at nighL Physical examination reveals perianal excor iad. on. An adhesive tape test is positive for worms. vVhich of the following is the most likely parasite m this patient? ***(A) Ancy lostoma duodenale ***(B) Asca1is lwnbricoides ***(C) Enterobi tiS vennicularis ***(D) Necator wnericam1s ***(E) Toxocara cani..~
49 The answer is C: Enterobius vermicularis. E. vennicularis ("pinworm") is an imestinal nematOde Lhat is encountered worldwide btu is more common in temperate zones. Individuals can be infected at any age, but parasilism is more common in children. Most people complain of pruritus caused by migrating worms. Ancylostoma duodenale and Necator americanus (choices A and D) are hookworms associated with intestinal bleeding and iron-deficiency anemia. ***Diagnosis: Enterobiasis
50 An Egyptian fishetman develops lower abdominal pain and pain on urination, and repons seeing blood in rus mine. Which of the following parasites is the most lU<ely cause of urinary symptoms in this patient? ***(A) Clonorchis sinensis ***(B) Dipltyllobothrium latum ***(C) Fasciola hepatica ***(D) Schistosoma haematobiLtm ***(E) Schistosoma mansoni
50 The answer is D: Schistosoma baematobium. Schiswsomiasis is the most imponam helmmthic disease of humans. It is characterized by intense inflammatory and immunologic respon ses that damage the liver, intestine, and urinmy bladder. In this case, the patient presents \\lith urogeniwl schiswsomiasis. 5. haematobiwn causes urogenital infections and increases the risk for developing squamous cell carcinoma of the bladder. 5. mansoni (choice E) affects the liver. ***Diagnosis: Schistosomiasis
51 A 34~year-old man with history of diabetes mellitus presents 'Nith a fever of 3 days duration and a painful , swnllen finger (shown in the image). Physical examination reveals erythema and edema affecting the fourth digiL Which of the following is the most likely etiology of this lesion? ***(A) Pseudomonas aeruginosa ***(B) Sporotllrix sciJenchii ***(C) Staphylococcus attreLL~ ***(D) Staphylococcus epidermidis ***(E) StTeptococcus l'yogenes
51 The answer is E: Streptococcus pyogenes. Erysipelas is an erythematous swelling of the skin caused chiefly by 5. pyogenes infection. 5. pyogenes, also known as group A streptococcus, is one of the most frequent bacterial pathogens of humans, producing various diseases ranging from acute self-limited pharyngitis to rheumatic fever. The rash usually begins on the face but can affect any part of the body. Cutaneous microabscesses and foci of necrosis are common. The other choices are noL typically associated with erysipelas. ***Diagnosis: Erysipelas
52 A 22-year-old student living in a college dormitmy presents with a 4-week history of a nonproduc tive cough and previous low-grade fever. An X- ray film of Lhe chest shows patchy consolidation of the right lower lobe, with evidence of interstitia] involvement. Which of the following is the most likely etiology of this paliems pulmonary infection? ***(A) Klebsiella 1meumoniae ***(B) Mycobacterium t11bercu/osis ***(C) Mycoplasma pneumoniae ***(D) Staphylococcus mmms ***(E) Streptococcus pneumoniae
52 The answer is C: Mycoplasma pneumoniae. M. pneumoniae produces an acme self-hmiLed lower respirmory tracl infecdon, primarily in children and young adults. Most infections occur in groups of persons living in close contact. M. pneumoniae tends to be milder Lhan oLher baclerial pneumonias and has , therefore, eamed the appellmion "walking pneumonia." Fever usually persisrs for no more Lhan 2 weeks, although a cough may linger for 6 weeks or more. Chest X-ray commonly shows patchy consolidation of a single segment of a lower lung lobe. M. pneumoniae is responsible for abom 20% of all pneumonias in developed commies. The other chokes do not cause imerstitial pneumonia. ***Diagnosis: Mycoplasma pneumoniac
53 A 47-year-old woman receiving chemotherapy for leukemia complains of headache, cough , and dyspnea. An X-ray film of the chest shows nodular pulmonary infiltrates and thin -walled cavities. A mucicarn~ne stain of a lung biopsy discloses budding yeast surrounded by a mucin-1ich capsule. vVhich of the following is the most likely pathogen? ***(A) Aspergillus Jlavus ***(B) Candida albicam ***(C) Coccidioides immitis ***(D) C1yptococcus neojom1ans ***(E) Histoplasma capsulatum
53 The answer is D: (ryptococcus neoformans. Clyptococcosis is a mycosis thaL primarily affects the meninges and lungs. C. neofomwns is unique among pathogenic fungi because il has a proteoglycan capsule, which is essential for pathogenicity The main reservoir for this fungus is pigeon droppings. The organisms appear as faintly stained, basophilic yeast with a clear, 3- LO 5-f..Lm thick mucinous capsule. Cryptococcus almosl exclusively a(fects persons with impaired cell-mediated immunity. The other choices do not stain with mucicannine. ***Diagnosis: Cryptococcosis
54 A JO~year~old man presents with abrupl onsel of (ever, chUls, myalgia, nausea, and vomiting. He had just returned from a month-long nip Lo the jungles of South America. lf this patient has yellow feve r, pathologic changes would most Ukely be observed in which of the following organs? ***(A) Brain ***(B) Kidney ***(C) Liver ***(D) Lung ***(E) Stomach
54 The answer is C: Uver. Yellow fever is an acute hemorrhagic fever, which is associated ·wiLh hepatic necrosis and jaundice. The illness is caused by a mosquito-borne Oavivirus. Extensive injury to vascular endothelial cells may cause hemorrhage and shock. This virus l1as a tropism for liver cells, where it causes extensive hepatocellular injury CounciJm.an bodies (apoptotic bodies) and miCrovesicular fa tty change are evident. In severe cases, the entire liver lobule may become necrotic. The other choices are incorrect, because the yellow fever vims is hepatorropic. ***Diagnosis: Yellow fever
55 A SO~ye~n·~old man, who recently returned from a trip to Africa, complains of darkening o[ his skin, weight loss, a11d an increased tendency to bleed. His Lemperature is 38°C (lOl °F), pulse raLe 22 per minute, and blood pressure 90/80 mm Hg Physical examination reveals a pale cachectic man wi.lh massive splenomegaly. CBC shows anemia, thrombocytopenia, and leukopenia. A bone marrow biopsy (silver srain) displays macrophages filled with proliferating organisms. Which of the following infectious diseases is mosLiikdy responsible for this patient's condition? ***(A) BiJharzi asis ***(B) Chagas disease ***(C) Cysticercosis ***(D) fascioliasis ***(E) Leishmaniasis
55 The answer is E: Leishmaniasis. Leishmaniae are protozoans that are uan..smiued w humans through insect bites. They cause a spectrum of clinical syndromes, ranging from indolent self-resolving cutaneous ulcers w fatal disseminated disease. Leishman iasis is transmiued by the bite of phlebowmus sandflies, which acquire infections from feeding on infected animals. The infestation is prb11arily a disease of less developed commies, where over 20 million people are believed lO be infected. Three distinct clinical entities are recognized: (l) localized cutaneous leislunaniasis, (2) mucocutaneous leishmaniasis, and (3) visceral leishmaniasis. Patients with visceral leishmaniasis suffer persistent fever, progressive weight loss , hepatosplenomegaly, anemia, thrombocytopenia, and leukopenia. Light-skinned persons develop darkening of the skin. lf untreated, the disease is fatal. Aside from clinical differences, the other choices do not represent infection of macrophages and do not lead to massive splenomegaly. ***Diagnosis: Leishmaniasis
56 A 6-year-old boy presents with a 10-month history of diarrhea, crampy abdominal pain, and progressive weight loss. He recently imnugrated to the United $rates from Mexico. Physical examination shows mild abdominal distension and evidence of malabsorption. The stool guaiac test is negative. Which of !he following is the most likely etiology of this child's gastrointestinal disease? ***(A) Campylobacterjejuni ***(B) Escherichia coli ***(C) Giardia Iamblia ***(D) Shigella dy senteriae ***(E) Staphylococcus attrnts
56 The answer is C: Giardia Iambiia. Gia rdiasis is an infestation of the smaU intestine by the flagellated protozoan G. lmnblia . The organisms can be acquired from contaminated water or food, and the in fection is characterized by abdominal cramping and nonbloody diarrhea. The gastrointestinal symptoms usually resolve in 1 w 4 weeks, but chronic giardiasis may lead to malabsorption, weight loss, and growth re tardation. The organisms are recovered from stool specimens, duodenal aspirates, or imestinal biopsies. The O£her choices do noL lead lO chronic infection. ***Diagnosis: Giardiasis, Gtardta Iamblia
57 A 52.-year-old man presents with a 3-day history of sore throat, cough, fever, and runny nose. The symptoms of this patients common u pper respiratory tract infection are primarily caused by which of the following infectious agents? ***(A) Aden ovirus ***(B) Epstein-Barr virus ***(C) Herpes simplex virus ***(D) Lentivirus ***(E) Rhinovirus
57 The answer is E: Rhinovirus. The common cold is an acute , self-limited disorder of the upper respiratory tract caused by infection with a variety o[ RNA viruses, inclucllng over 100 distinct rhinoviruses and several coronavimses. These viruses infect nasal respiratory epithelial cells, causing edema and increased mucus production. Clinically, Lhe common cold is characterized by rhinorrhea, pharyngitis, cough, and lowgrade feve r. Sympwms last about a week. HlV ts a lemivhus (choice D) ***Diagnosis: Common cold
58 A 29-year-old woman (gravida Ill, para 11) delivers a premature infant at 28 weeks of gestation. At birth, the neonate shows signs of profound anemia and generalized edema (hydrops fetalis). The disease is most likely caused by an intrauterine infection with which of the following TORCH agents? ***(A) Cytomegalovirus ***(B) Herpes simplex virus ***(C) Parvovirus Bl9 ***(D) Rubellavims ***(E) Toxoplasma gondii
58 The answer is C: Parvovirus 819. Human parvovirus Bl9 is a DNA vims that causes systemk infections characterized b}' rash, anhralgias, and transient interruption i11 etythropoiesis. The vims produces characterLstic cytopathic effects in erythroid precursor cells. The nucleus of an affected cell is enlarged, and the chromatin is displaced peripherally by central, glassy, eosinophilic ma terial. When the fetus is infected with parvovirus Bl9, a transient cessation of erythrocyte production leacls to severe anemia, hydrops feLalis, and often death in utero. The other choices do not interfere with erythropoiesis ***Diagnosis: Hydrops fetalis, TORCH syndrome
59 A 36-year-old woman presents witl1 a 2-day history of feve r, myalgias, headache, and watery diarrhea (5 to 10 per day) tl1at contains some blood. A stool smear shows leukocytes and Gram-negative curved bacilli. The patiem reports drink· ing raw milk 7 days ago. Which of the follO\ving microorganisms is the most likely cause of this womans gastrointestinal disorder? ***(A) CatntJylobacterjejuni ***(B) Escltericltia coli ***(C) Salmonella typlJi ***(D) Shigella dysent.eriae ***(E) Staphylococcus attreus
59 The answer is A: Campylobacter jejuni. C. jejuni causes an acute, self-limited, inGammatory diarrheal illness. The organism is distributed worldwide and is acquired through contaminaLed food or waLer. It is a major cause of childhood mortality in developing counnies and is responsible for many cases of travelers' diarrhea. C. jejuni causes a superficial enterocolitis primarily involving the terminal ileum and colon. Focal necrosis of d1e intestinal epithelium is accompanied by an acute inflammatory infiltrate. ln severe cases, focal disease progresses w small ulcers and patchy inflammatory exudates (pseudomembranes). The symptoms typically Tesolve in 5 Lo 7 days. A few patients develop a severe, protracted ilJness resembling acute ulcerative colitis. The mher choices are characterized by a more rapid onset of symptoms after infection. ***Diagnosis: Campylobacrer emeritis
60 A 45-year-old construction worl<er sLtrfers a penetrating wound of the left leg, which is cleaned and sutured. Three days later, the patient presents \vith sudden onset of severe pain at the site of injury. Physical examination shows darkening of the surrounding skin , hemorrhage, and cutaneous necrosis. The wound shows a thick serosanguinous discharge with gas bubbles and a fragrant odor. Which of the following is the most likely etiology of this patient's wound infeclion? ***(A) Clostridiwn botulimw1 ***(B) Clostridium pelfringens ***(C) Staphylococcus aureus ***(D) StC1phy lococcus epidermidis ***(E) St reptococcus pyogenes
60 The answer is B: Clostridium perfringens. Gas gangrene (clostridial myonecrosis) is a necrotizing, gas-forming infection that begins in contaminated wounds and spreads rapidly to adjacent tissues. The disease can be fatal within hours of onset. Gas gangTene follows the deposition of C. pe1jringens into tissues under anaerobic conditions. Such conditions occur in areas of extensive necrosis (e.g., severe trauma, wartime injuries , and septic abortions). Clostridial myonecrosis is rare when the wound is subjected to prompt and thorough debridement of dead tissue. Damage to previously heal thy muscle is mediated by a myotoxin. C. botulinum (choice A) secretes a preformed neurotoxin. ***Diagnosis: Gas gangrene, closnidial myonecrosis
61 A 32-ye.ar-old man presents with the sudden onset of tonicdonie seizures and dies the next day The brain at autopsy is shmvn in the image. This patient most likely contracted which of the following infectious diseases? ***(A) Aspergillosis ***(B) Clonorchiasis ***(C) Cysticercosis ***(D) Fascioliasis ***(E) Paragonimiasis
61 The answer is C: Cysticercosis. Pigs acquire cysticerci by ingesting eggs of Taenia solium shed in human feces. However, when humans accidentally ingest the eggs from human reces and become infeCLed with cysticerci, the consequences may be catastrophic. The eggs release oncospheres, which penetrate the wall of the gut, enter the bloodstream, lodge in tissues, and differentiate to cysticerci. The cysticercus is a sphe1ical milky white cyst of about 1 em in diameter that contains fiuid and an invaginated scolex (head of the worm) Viable cysts can be shelled out from the infected tissue. Multiple cysticerci in the brain may impan a "Swiss cheese" appearance and manifest cJinically as headaches and seizures. The other worms (choices B, D, and E) do not infect the brain. Aspergillosis of the brain {choice A) is distinctly uncommon. ***Diagnosis: Cysticercosis
62 A 5-year-old boy is brought to the emergency room with a fever of 103"F (38 7°C), chest pain, and productive cough The patient has a history of recurrent pulmonary disease and respiratory distress. What microorganism recovered from the lungs of this child is virtually diagnostic of cystic fibrosis? ***(A) Klebsiella sp. ***(B) Legiorwe/la sp. ***(C) Pneumocystis sp. ***(D) Pseudomonas sp ***(E) Streptococcus sp.
62 The answer is D: Pseudomonas sp. Cystic fibrosis is the most conunon lethal amosomal recessive disorder m the white population. The disease is characterized by (1) chronic pulmonary disease, (2) deficient exocrine pancreatic function, and (3) other complications of inspissated mucus in a number of organs, including the small intestine, the Liver, and the reproductive tract. It results from abnonual electrolyte transport caused by impaired function of the chloride channel of epithelial cells. The pulmonary symptoms of Cf begin with cough, which eventually becomes productive of large amoums of tenacious and purulem spu rum. Episodes of infectious bronchitis and bronchopneumonia become progressively more frequent, and eventually shormess of breath develops. Respiratory failu re and the cardiac complications of pulmonary hypeltension (cor pulmonale) are late sequelae. The most common organisms that infect the respiratory tract in CF are Staphy lococcus and Pseudomonas species. As the disease advances, Pseudomonas may be the only organism culntred from the lung. ln fact, the recovery of Pseudomonas sp., particularly the mucoid variety. from the lungs of a child with chronic pulmonary disease is vinually diagnostic of CF ***Diagnosis: Cystic fibrosis
1 An 80-year-old man with long-standing diabetes and systemic hypertension dies of congestive heart failure. The luminal sur· face of the abdominal aona is shown in the image. Which of the following pathologic changes would you expect to see on microscopic examination! ***(A) Acute inflammation of Lhe vessel wall ***(B) BaClerial colonies in the vessel wall ***(C) Cystic medial necrosis ***(D) Lipid deposition and smooth muscle cell hyperplasia ***(E) Obliterative endarteritis of the vasa vasomm
1 The answer is D: Lipid deposition and smooth muscle cell .____ hyperplasia . Atherosclerosis is a disease of large- and medium- sized elastic and muscular aneries that results in the progressive accumulation wi Lhin the intima of inflammatory cells, hyperplastic smooth muscle cells, lipids, and connective tissue. The resulting characLeJisdc lesion, the lipid plaque (atheroma), contains pools of extracellular lipid and numerous lipid-laden macrophages (foam cells). lt is not an acULe inflammatory or infectious process (choices A and B). Obliterative endanelitis (choice E) is a syphilitic lesion. ***Diagnosis: Atherosclerosis
2 A 60-year-old mildly obese woman is admiued w the hospital '-Vith a chief complaim or recurrem chest pain on exenion. The patient repons several episodes of chest pain over the past several years and painful leg cramps when walking. Fasting blood glucose (160 m&'dl) and total serum cholesterol (3 70 mgl dl) are high. The ECG is normal and blood tests for ca rdiac· specifi c proteins are negative. Chest pain in this patient is most likely due to which of the following u nderlying conditions? ***(A) Atherosclerosis of coronary artery ***(B) Congenital anomalous origin of coronary artery ***(C) Coronary arteritis ***(D) Imramural course of the LAD coronary artery ***(E) Thrombosis of coronary artery
2 The answer is A: Atherosclerosis of coronary artery. Patients \viLh severe atherosclerosis present with organ-specific vascular disorders, including intermittent claudicanon, abdominal aortic aneurysms, coronary anery disease (chest pain), cerebrovascular disease, and pe1ipheral vascular disease. Angina pecwris is the pain of myocardial ischemia. It typically occurs in the substernal portion of Lhe chesL and may radiate 1.0 the left arm, jaw, and epigasuium. lL is lhe most common symptom of ischemic heart disease. Laboratory findings in Lhis patiem show evidence of diabetes mellitus and hyperlipidemia, which are 1isk factors for atherosclerosis. Thrombosis of a ruptured atheromatous plaque (choice E) usually p recipitates acute myocardial infarction. The other conditions (choices C. D, and E) may limit coronary blood flow and present with chest pain on exertion, but they are less conunon than coronmy atherosclerosis. ***Diagnosis: Ischemic hean disease, atherosclerosis
3 A 69-year-old woman presems with crushing substernal chest pain and nausea. Laboratory studies show elevated serum levels of cardiac proteins (CK-MB = 8.5 nglml; noponin-1 = 3.2 nglml). A diagnosis or myocardial infarction is confirmed by ECG. Despite treatment, the patient becomes hypotensive, and resuscitation attempts are unsuccessful. A cross section of lhe patient's 1ight coronary anery at autopsy is shown in the image. Which of the following pathologic changes are evident in this aulOpsy specimen? ***(A) Arteritis and atherosclerosis ***(B) Atherosclerosis and thrombosis ***(C) Microaneurysm and canalization ***(D) Thrombosis and calcification ***(E) Vasodilation and aneJitis
3 The answer is B: Atherosclerosis and thrombosis. The photomicrograph shows severe atherosclerosis and a recem thrombus in the narrowed tumen. The mamre atheroma is highly Lhrombogemc, and Lhrombosis of an atherosclerotic p laque can abmpLly occlude Lhe lumen of med1um-sized muscular arteries causing ischemic necrosis of dependent tissues. Thrombotic occlusion may manifest as myocardial infarction, stroke, or gangrene of intestinal loops or lower exLremities. Aneritis (choice A) would exhibit i11 Oammatory cells. Calcification (choice D) would appear as inegular blue material with H&E stain. ***Diagnosis: Myocardial infarcLion
4 Upids sequestered by foam cells within the coronary arteries of the patient described in Question 3 were derived primari ly from which of the following sources? ***(A) Apoptotic bodies of smooth muscle cells ***(B) Upids found in platelet granules ***(C) Membranes of dead cells at the sile of vascular injllly ***(D) Secretory product of activated macrophages ***(E) Serum lipoproteins
4 The answer is E: Serum lipoproteins. Hyperlipidemia is cor- #NAME? disease. Cholesterol carried by serum lipoproLeins is deposited in the aLheroma, where it is endocytosed by macrophages (lipid-laden foam cells}. ***Diagnosis: Myocardial infarcLion, atherosclerosis
5 A 55-year-old man with a history of hypertension and type 2 diabetes is rushed to Lhe emergency room after collapsing He describes "tearing chest pain" radiating to the back. His blood pressure is 90/50 mm Hg, and pulse is diminished. Cardiac auscul tation reveals a dlastohc murmur, consistem with aortic regurgitation. The ECG is nonnal, and blood tests for cardiacspecific proteins and enzymes are negadve. TI1e patient dies within 24 hours of admission. The thoracic aona at amopsy is shown in the image The pathogenesis of this lesion is most closely related to which of the following underlymg conditions? ***(A) Connective tissue weakness of the aortic wall ***(B) fibrinoid necrosis and smooth muscle hyperplasia ***(C) Inunune complex-mecUated vasculitis ***(D) Neovascularization of a complicated atheromatous plaque ***(E) Subintimal lipid deposition and smooth muscle necrosis
5 The answer is A: (onnedive tissue weakness of the aortic wall. Dissecting aneurysm refers to the emry or blood into the anerial wall and its extension along the length of the vessel, which is associated with a degeneration and weakening of the aortic media. Some cases are seen in patients with Marfan syndrome, a systemic connective tissue disorder associated with mutations in the gene encoding the extraceullar matrix glycoprotein, 6brillin. Although the other choices may lead to saccular or fusifonn aneurysms in other locations, they do not cause an aortic dissection. ***Diagnosis: Dissecting aortic aneurysm
6 A previously healthy 67-year-old man presents to the emergency room with numbness of his left leg. Temperature and blood pressure are nmma1. Physical examination shows pallor and a cool left leg with absence of distal pulse. An ECG reveals no abnormalities. An arteriogram demonstrates a markedly dilated abdominal aorta and occlusion of the left popliteal anety The blockage is removed surgically, and the patient recm-ers. Which of the following is the most likely source of the anerialthromboembolus in this patient? ***(A) Deep venous thrombosis ***(B) Left vennicular mural1.hrombus ***(C) Nonbacterial endocarditis ***(D) Paradmdcal emboli ***(E) Thrombus from an aLheromatous aorta
6 The answer is E: Thrombus from an atheromatous aorta. This patient has an abdominal aortic aneurysm. A thrombus that forms over the aneurysm may embolize and lodge in a distal vesseL ln this case, a thmmboembolus occluded the popliteal artery, causing ischemia of the patient's left leg. Although a mural thrombus of the left ventricle (choice B) can embolize, there is no evidence of underlying myocardial infarction in this case. Choices B, C, and D are much rarer causes of thromboembolism. ***Diagnosis: Anerial thromboembolism
7 A 28-year-old man develops the "worst headache of his life" and then becomes comatose. A CT scan of the head reveals subarachnoid hemorrhage. The patient eventually dies, and autopsy reveals an aneurysm al the base of the brain (shown in the image; see arrow). The pathogenesis of this abnormalily is most closely linked to which of the £ollm:ving condi tions? ***(A) Arterial waU defect due to diabetes ***(B) Atherosclerotic plaque deposition ***(C) Muscle weakness of the arterial wall ***(D) Cystic medial necrosis ***(E) EndarteriTis of the vasa vasorum
7 The answer is C: Musde weakness of the arterial wall. The most conunon tJl)e of cerebral aneurysm is a saccular aneuI)' Sm, also referred to as a berry aneurysm. The lesion results from a congenital defect in smooth muscle distribmion at a branch poim of the arterial wall. The most common site of berry aneurysm formation is between the anterior communicaling and the anterior cerebral aneries in the circle of ·willis. In this case, the berry aneurysm arose from the posterior cerebral an ery (see photograph). Diabetes (choice A) and atherosclerosis (choice B) do not cause berry aneur)•sms. Cystic medial necrosis (choice D) is associated \vith dissecting aortic aneurysm. Endaneritis of the vasa vasorum (choice E) is associated with syphilitic aneurysm of the ascending aona. ***Diagnosis: Berry aneurysm, subarachnoid hemon-hage
8 A 45-year-old man presents with pain in the legs upon exerc1se and destmnion of the tips of his fingers. He has an SO-packyear history of smoking. Laboratory values include hemoglobin of 16gldl, WBC of 8,500/jll, serum cholesterol of 22 0 mgldL, fasting blood sugar of 90 mgldl, and negative tests for antinuclear antibodies. Biopsy of the affected area (shown in the image) reveals intraluminal thrombi in medium-sized anenes and inflammation extending from arteries to neighboring veins and nerves. What is the appropriate diagnosis? . ..,.. ***(A) Buerger disease ***(B) Churg-Strauss disease ***(C) Kawasaki disease ***(D) Polyaneritis nodosa ***(E) Takayasu arteritis
8 The answer is A: Buerger disease. Buerger disease (thromboangiitis obliterans) is an occlusive inflammatory disease of medium and small aneries of the distal arms and legs. The etiologic role of smoking has been emphasized by the observation that cessation of smoking can be followed by remission. Microscopic examination of affected vessels shows polymorphonuclear infiltrates extending ro neighboring veins and nerves. Inflammation of the endOlhelium is associated with thrombosis and obliteration of the affected vessels. The other choices are not associated with smoking and do not exhibit these characteristic histologic findings. ***Diagnosis: Buerger disease
9 A 45-year-old man is brought to the emergency room with rapid pulse and cold and clammy skin. Blood pressure is 90/50 mm Hg. An X-ray film of the chest demonstrares dilation of the ascending aona. Cardiac auscultation reveals a diastolic murmur in the aortic region. Laboratory studies show that serum cholesterol is 160 mgldl , hematocrit is 35%, and hemoglobin is 13.6 gldL The fluorescent Treponema amibody rest is positive The patiem suddenly becomes hypotensive and dies. The luminal surface of rhe ascend ing aona at amopsy is shown in the image. Which of the following was most likely involved in the pathogenesis of this aon ic lesion? ***(A) Arterial wail defect due to diabetes ***(B) Atherosclerosis ***(C) Congenital defect of the arterial wall ***(D) C)'Stk medial necrosis ***(E) Endaneritis of the vasa vasorum
9 The answer is E: Endart.eritis of the vasa vasorum. The results of laboratory tests indicate that the patient has syphilis. Syphilitic anemysm typicalJy a!Tects the ascending aona. Microscopk examination shows obliterative endarteritis and periarteritis of the vasa vasomm. The vasa vasomm ramify in 1.he adventitia and penen·are the outer and middle third of the aorta. ln syphilitic disease, they become encircled by lymphocytes, plasma cells, and macrophages Obliteration oflhe vasa vasomm causes focal necrosis and scaning of the media, with dismption and disorganization of the elastic lamellae. The inner surface of the affected aona shows a typical utree bark" appearance. The other choices are not involved in the pathogenesis of syphilitic anemysm. ***Diagnosis: Syphilitic aneurysm
10 A 38-ye.a r-old woman with a history of ischemic heart disease presents with disfiguring skin lesions. Physical examination shows xanthomas on the dorsal surface of both hands (shown in the image) and xanrhelasmas of the eyelids. Laboratory studies reveal serum cholesterol of 820 mgldL and significantly elevated serum ttiglycerides and LDL Which of the following histopathologic findings would be expected in a biopsy of this patients skin lesions7 ***(A) Detmal calcium deposits ***(B) Foamy macrophages ***(C) Hypertrophic scar tissue ***(D) Multinuclealed giant cells ***(E) Smooth muscle hyperplasia
10 The answer is 8: Foamy macrophages. This patiem has familial hypercholesterolemia and has inherited mutations in the LDL receptOr gene. The LDL receptor is a cell surface glycoprotein that regulates plasma cholesterol by mediating endocyLOsis and recycling of apolipoprotein (apo)E. Lacking LDL receptor function , high levels of LDL circulate, are taken up by dssue macrophages, and accumulate to form occlusive arterial plaques (atheromas) and papules or nodules of Lipid-laden, foamy macrophages (xanthomas). The other choices are not expected findings in xamhomas. ***Diagnosis: Familial hypercholesterolemia, xanthoma
11 A 55-year-old man suffers from an acme myocardial infarction after occlusion of the Left anterior descending cororuuy artery. The patient undergoes coronary bypass surgery 3 days later. Wl1ich of the follm:ving is the most frequent cause of saphenous vein graft failure seveml years fo llowing coronary bypass surgery? ***(A) Acme inOammalion ***(B) Atherosclerosis ***(C) Graft-versus-host disease ***(D) MetasLatic calcification ***(E) Microaneurysm
11 The answer is 8: Atherosclerosis. Saphenous veins are used as aurografts in coronary anery bypass surgety These grafts usually undergo a se ries of adapdve and reparative changes. Venous grafts in place for 5 to lO years typically show athcrosderotic plaques th at arc indistinguishable from those found in native coronal)' arteries, a process referred LO as atherosclerotic restenosis. ***Diagnosis: Atherosclerosis
12 A 45-year-old black man undergoes renal biopsy for evaluation of chronic renal faUure. The patient has a 60-pack-year hiswry of smoking. Physical examination reveals a blood pressure of 190/120mm Hg. A renal biopsy shows thickening of small arteries and anerioles, as well as edemawus intimal expansion and fibrinoid necrosis. The Congo red stain is negative. Labomtory studies show hemoglobin is 10.2 gldl and serum cholesterol is 250mgldl. BUN and serum crearinine are 42 and 5.5 mgldl, respectively. Which or the fo llowing is the most likely cause of renal failure in this padem7 ***(A) Amytoid nephropathy ***(B) Chronic pyelonephritis ***(C) Malignam hype rtension ***(D) Polya1teritis nodosa ***(E) Proliferative glomerulonephr itis
12 The answer is C: Malignant hypertension. Malignant hyperLension refers to an elevated blood pressure that may result in rapidly progressive vascular disease affecting the brain, heart, and kidney. The disease injures endothelial cells, causing increased vasculax permeability, which leads to the insudation of plasma proteins imo the vessel wall and morphologk eVIdence of fibrinoid necrosis. Polyanerilis nodosa (choice D) is an inDammaLOry process. 1n this patiem, acme vascular injury is followed by smooth muscle proliferation with an increase in concentric layers of smooth muscle cells that yield the socalled uonion skin" appearance. Poor perf·usion of the kidneys stimulates the release of renin, which serves to elevate systemic blood pressure even further (renovascular hypertension). ***Diagnosis: Hypertensive nephTosclerosis
13 A 32-year-old woman from Africa presents with a 4-momh hiswry of swelling of her tighr leg (patient shown in the image). Laboratory studies demonstrate a parasitic infestation. Soft nssue swelling of this lJatiems leg was most likely caused by which of the following conditions' ***(A) Congesuve hean failure ***(B) Deep venous thrombosis ***(C) Kidney disease ***(D) Uver disease ***(E) Lymphauc obstrucuon
13 The answer is E: Lymphatic obstruction. The patient has lilar ial worm infestation of an inguinal lymph node which has caused elephantiasis. Vnder normal circumstances, more nuid is fillered into the intersti tial spaces than is reabsorbed in to the vascular bed . This excess interstitial nuid is removed by the lymphatics. Thus, obstruction to the lymphatic now leads to localized edema fonnalion. Lymphattc channels can be obstructed by (l ) malignant neoplasms, (2) fibrosis resulting from innammation or in-adiation, and (3) surgical ablation_ The m nammatory response to filarial worms (Bancroftian and Malayan filariasis) can result in lymphatic obstruction that produces massive lymphedema of the scrotum or extremity (elephantiasis). Lymphatic edema diffe rs from other forms of edema in its high protem content, since lymph is the vehicle by which proteins and interstitial cells are returned w the circulation. This increased protein concentration may be a fibrogenic stimulus in the formation of dermal fibrosis in chronic edema (indurated edema). Congestive hean failure (choice A), kidney d isease (choice C), and ]jver disease (choice D) can cause generalized noninnammatory edema, but the swelling is bilateral. Although deep venous thrombosis can cause tenderness and swelling, it does not cause elephantiasis. ***Diagnosis: Elephamiasis, lymphedema
14 A 60-year-old man presems with dizziness , nausea, and severe shonness of breath of several months' duration. Physical examination shows hepatomegaly, ascixes, and anasarca. His blood pressure is 2001115 mm Hg An X-ray film of the chest demonstrates cardiomegaly and mild pulmonary edema. Although different mechanisms may have comTibmed to the pathogenesis ofhypenension in this patient, the common end result for all of them is which of the following? ***(A) Arterial cystic medial necrosis ***(B) Decreased plasma oncotic pressure ***(C) Gene ralized vasodilation ***(D) lncreased peripheral vascular resistance ***(E) Increased vascular pem1eabihry
14 The answer is D: Increased peripheral vascular resistance. In patients with systemic hypertension, the end result of autoregulation is al1..vays iJlcreased peripheral resistance. Most cases of hypertension represent an imbalance between renal function and sodium homeostasis. ln this context, the renjn-angiotensin system increases blood pressure, whereas this axis is antagolljzed by atrial nauiuretic factor. Cystic medial necrosis (choice A) is a factor in aortic dissecting aneurysm. Vasodilation (choice C) decreases blood pressure. Choices Band E may be a consequence of uncontrolled hypertension, but do not cause it. ***Diagnosis: Hypenensive heart disease
15 A 40-ye.ar-old woman presents with an 8-momh history of severe headaches, weakness, and dizziness. Blood pressure is 180/110 mm Hg. Physical e.:mmin.ation shows diminished tendon reflexes. An abdominal CT scan reveals a 4-cm mass in the light adrenal gland. The results of laboratory studies include serum potassium of 2. 3 mEq/L, semm sodium of 155 mEq/L, plasma cortisol of 25Jlgfdl (8AM) and 20J.1gfdl (4PM), and low plasma renin. These clinical and laboratory findings are consistent with an adrenal mmor that secretes which of the following honnones? ***(A) Aldosterone ***(B) Conisol ***(C) Epinephrine ***(D) Renin ***(E) Testosterone
15 The answer is A: Aldosterone. Th is patient has Conn syndrome, an endocrine disorder most commonly caused by an adrenal cortical adenoma. Aldosterone secreted by Lhe adrenal conical tumor causes hypertension, hypematremia, and hypokalemia. Epinephrine, renin, and testosterone (choices C, D, and E) are nm produced by adrenal conical rumors. Cordsol (choice B) may be produced by adrenal con ical mmors but does not significantly alter eleCLmlyte homeostasis. ***Diagnosis: Conn syndrome, hyperaldosteronism
16 A 25-year-old woman with a recent histOt)' of acute hepatitis B infection presems with reddish-blue lesions on her ]ower extremities, fever, muscle pain, and mild weight loss. Physical e.xamination reveals numerous regions of red-purple discoloration a([ecting the skin of both legs. Laboratory tests demonstrate positive P-ANG'\ and an elevated erythrocyte sedimemation rate. Urinalysis shows 2+ proteinuria. Biopsy of lesional s!Cn is shown in me image. Which of the following is the most Hkely diagnosis? ***(A) Benign arteriosclerosis ***(B) Ftbromuscular dysplasta ***(C) Henoch-Schonlcm purpura ***(D) Monckcberg mcdtal sclerosis ***(E) Polyaneritis nodosa
16 The answer is E: Polyarteritis nodosa (PAN) . PAN is an acute necrotizing vasculitis that affects medium-sized and smaller muscular arter ies. On occasion, PAN extcnds into larger arteries , such as the renal , splenic, or coronary artelies. The most common morphologic feature of affected arteries is fibrinoid necrosis, in which the medial muscle and adjacent tissue are fused imo an eosinophilic mass that stains for fibrin. PAN affecting small vessels is frequently associated with the presence of P-ANCA. Approximately 15% of patients vvith PAN demonstrate either HBsAg or anti-HCV antibodies. Choices A, B, and D are not characterized b)' necrosis of the arterial wall. The immune complexes seen in Henoch-$chonlein purpura (choice C) are unrelated to hepatitis B. ***Diagnosis: Polyarteritis nodosa
17 A 33-year-old man with AIDS presems with multiple, purplecolored skin nodules on his hands and feet. The lesions vary in size from 1 mm to 1 em in diameter. Biopsy of lesional skin is shown in lhe image. Which of d1e following vimses is implicated in the pathogenesis of this patients skin neoplasm? ***(A) Cywmegalovirus ***(B) Human herpesvirus-6 ***(C) Human herpesvirus-8 ***(D) Human immunodeficiency vims ***(E) Human papillomavirus
17 The answer is C: Human herpesvirus-8 (HHV-8). Patiems with AIDS may develop Kaposi sarcoma. Microscopic examination of a skin lesion shows numerous poorly dilferemiated spindleshaped neoplastic cells and vascular lesions filled with red blood cells, characteristic of Kaposi sarcoma. HHV-8 is implicated in the paLhogenesis of this tumor in HIV-infected patients. HIV (choice D) by itself is not a cause of Kaposi sarcoma. ***Diagnosis: Kaposi sarcoma, AtD$
18 A 30-yeaT-old woman presents with a widespread skin rash that she has had for 5 days. She is taking sulfa medication fo r recurrent cystitis. A skin biopsy shows leukocyLOclastic vasculitis involving dermal venules. What is the appropriate diagnosis? ***(A) Buerger disease ***(B) Giam cell granulomatous arteritis ***(C) Henoch-Schonlein purpura ***(D) Hypersensitivity angiitis ***(E) Polyarter itis nodosa
18 The answer is 0: Hypersensitivity angiitis. Hypersensitivity angiitis refers to a broacl spectnnn of inflammatory lesions that represem a reaction to foreign matetials (e.g., bacterial products or more commonly drugs). When the vascular lesions are confined to Lhe skin, the Lerms leukocytoclaslic vasculitis, cutaneous vasculltis, or cutaneous necrotizing venulilis are applied. The Q[her choices are not caused by sulfa drugs. ***Diagnosis: Hypersensitivity angiitis
19 A 70-year-old woman complains of a throbbing unilateral headache and vision problems. She repons weight loss and mandibular pain while eating. The patient also has a history of recurrem bouts of fever accompanied by malaise and muscle aches. Physical examination reveals nodular enlargement o[ the temporal artery with pain on palpation. A biopsy is obtained (shown in the image). What is the approptiate diagnosis? ***(A) Giam cell arteritis ***(B) Hypersensitivily angiitis ***(C) Kawasaki disease ***(D) Polyarteritis nodosa ***(E) Wegener granulomatosis
19 The answer is A: Giant cell arteritis. Giam cell (temporal) anerius is the most common vasculitis. The disease is a local, chronic granulomawus inflammation of the temporal aneties. The average age at onset is 70 years. Hiswlogic examination shows gjam cell granulomawus inflammation. whtch destroys the media of the temporal anety and predisposes to thrombosis (see photomicrograph). Headaches, cypically in the form or throbbing temporal pain and visual problems, may appear. A palpable, wnuous, and s'vollen temporal anery may be the only finding on physical examlnation. \¥egener granulomatosis (choice E) causes granulomatous inflammation, but not in the temporal artery. The other choices are not associated with multinucleated giant cells. ***Diagnosis: Giam cell granulomatous aneritis, temporal aneritis
ZO A 48-year-old man presents with an exquisitely painful. raised, red lesion on the dorsal surface o f his left hand. Hiswlogic examination of a skin biopsy reveals nests of round regular cells within cormective tissue associated 'Nilh branching vascular spaces. \Vhich of the following is the most likely diagnosis? ***(A) Angiosarcoma ***(B) Dermatofibroma ***(C) Glomus LUmor ***(D) Hemangioma ***(E) Upoma
20 The answer is C: Glomus tumor. A glomus tumor is a benign tumor of the glomus body, which is often extremely painful. Glomus bodies are normal neu romyoarteJial recepto rs that are sensitive to temperature and regulate arte1iolar blood flow. The lesions are small, usually smaller than l em in diameter. The two main histologic components are branching vascular chann els in connective tissue stroma and aggregates of specialized glomus cells. The mher choices tend nm LO be painful. ***Diagnosis: Glomus tumor
Z1 A 60-ye~u·~old man has his left forearm amputated because he has invasive rhabdomyosarcoma. The pathologist notes calcification in the wall of the radial anery, which otherwise appears w1remarkable. Which of the following is the appropriate diagnosis? ***(A) Churg~Strauss disease ***(B) Complicated atherosclerotic plaque ***(C) Fibromuscular dysplasia ***(D) Monckeberg medial sclerosis ***(E) Polyarteritis nodosa
21 The answer is D: Monckeberg medial sclerosis. Moncl<eberg medial sclerosis is charactelized by calcification or the media of large· and medium·sized arteries of older persons who are not othenvise affected by atherosclerosis (choice B). On gross examination, the involved aneries are hard .and dilated. These arterial changes are usually asymptomatic. None of the other choices display calciGcation. ***Diagnosis: Monckeberg medial sclerosis
22 A 20-year-old woman complains of double vision, fainting spells, tingling of the fingers of her left hand, and numbness of the fingers of her tight hand. Physical examination reveals absence of pulse in her right arm. LaboraLory tests sho\v elevated erythrocyte sedimentation rate and thrombocytosis. An aonogram demonstrates narrowing and occlusion of branching aneries, including the right subclavian artery. The patient subsequently develops hean failure and dies of massive pulmonary edema. At amopsy, the aorta has a thickened wall and shows vasculitis and fragmentation of elastic fibers. Which of the follo\ving is the most likely diagnosis? ***(A) Buerger disease ***(B) Churg-Strauss disease ***(C) Kawasaki disease ***(D) Polyartelitis nodosa ***(E) Takayasu arteritis
22 The answer is E: Takayasu arteritis. Takayasu aneritis is an inflammawry disorder of large arteries, classicaUy !he aortic arch and its major branches. On gross examination. the aona is thickened, and the intima exhibits focal, raised plaques. The branches of 1.he aorta often display localized sl.enosis or occlusion, which interferes with blood flow and accounts for the symptoms of ''pulse less" disease. Ischemic cerebrovascular episodes in a young woman and a differential between the blood pressure in the left and the right ann suggesL the diagnosis of Takayasu disease. More than 90% of patients aTe women under 30 )'ears of age. The o!her chokes are diseases of arteries that have di£ferem clinical and pathologic manifestations. ***Diagnosis: Takayasu arteritis
23 A 19-year-old man with a history of recent-onset asthma presents with chest pain, intermittent claudication, and respiratory distress that is unresponsive to bronchodilators and anlibiolics. Physical examination reveals mild hypertension (blood pressure = 150/100 mm Hg), bilateral wheezing, and numerous purpuric skin lesions on the feeL Laboratory studies demonstrate that leukocytes are increased to 14,000/ J..LL with increased eosinophils and platelets are increased to 450 ,000/J..LL. BUN is elevated to 30 mgldL, and semm creatinine is elevated to 3.5mgldL The serum antineutrophil cytoplasmic antibody test is positive Urinalysis discloses 3+ proteinmia and RBCs. A renal biopsy demonstrates vasculitis of medium-sized aneries, accompanied by eosinophilia. Which of the follo\ving is the most likely diagnosis? ***(A) Churg-So·auss disease ***(B) Henoch-Schonlein purpura ***(C) Loeffler syndrome ***(D) Polyarter itis nodosa ***(E) Wegener granulomatosis
23 The answer is A: Churg-Strauss disease. Churg·Strauss disease is an idiopathic, sys1.emic, granulomatous disease of small- and medium-size arteries characterized by vasculitis of many organs, fluctuating eosinophilia, and late-onset asthma. The majority o[ patiems display anlineurrophil cytoplasnuc ami bodies. The disease is also known as allergic granulomatosis and angiitis. Transbronchiallung biopsy shows granulomarous lesions in vascular and extravascular sites, accompanied by intense eosinophilia. The vasculitis hiswlogicaUy resembles the lesions of polyaneritis nodosa (choice D) and Wegener granulomawsis (choice E), bm these diseases do not cypically present wil.h an asthmatic syndrome. ***Diagnosis: Churg-$rrauss disease
24 A 40-yea l~old man presenlS with a :Z-week history of recurren t oral ulcers, gen]tal ulcers, inteJmittent arthritic pain of the knees, and abdominal pain. Physical examination reveals shallow ulcerations of the mucosa of the glans penis, as well as oral aphthous ulcers and conjunclivids. Which of the following is the most Likely diagnosis? ***(A) Behc;et disease ***(B) Genital he11)es ***(C) Gonon hea ***(D) Polyane1itis nodosa ***(E) Syphilis
24 The answer is A: Beh~et disease. Behc;et disease is a systemic vasculitis characterized by oral aphthous ulcers, genital ulceration, and ocular inflammation, with occasional involvement of the nervous, gastrointestinal, and cardiovascular systems. The mucocutaneous lesions show a nonspecific vasculitis of arterioles, capillaries, and venules. The cause of the necrotizing inflammaLion of small blood vessels is not known, but an association with specific HLA subtypes suggests an immune basis. Herpes (choice B) does not presem with arthritis. ***Diagnosis: Be h c;;et disease
25 A neonate has a well-demarcated lesion in the upper eyelid and forehead resembling a tumor (shown in the image) A biopsy shows large vascular channels imerspersed with small, capillary type vessels. What is the appropriate diagnosis? ***(A) Angiosarcoma ***(B) Capillary lymphangioma (C} Cavernous hemangioma ***(D) Hemangioendothelioma ***(E) Hemangiopericytoma
25 The answer is C: Cavernous hemangioma. Congenital cavernous hemangioma is a benign lesion consisting of large vascular channels, frequently interspersed wtth small capillary- type vessels. These lesions occur ptimarily in the skm, where they are te rmed pon-wine stains. Cavernous hemangiomas may also be fow1d in the brain. where, after a long quiescem period, they may slowly enlaTge and cause neurologic symptorns. Cavemous hemangiomas can w1dergo a vatiety of changes, in cluding thrombosis and fibrosis, cystic cavitations, and intracystic h emorrhage. Capillary lymphangioma (ch oice B) is a rumor of the lymphatic system and is not discolored. The other ch oices are not characteristically encoumered as congenital lesions of the face. ***Diagnosis: Congenital cavernous hemangioma
26 A 48-year-old man wilh diabetes presents with a history of progressive pain in both legs for several years. The pain is severe after walking two blocks or climbing one flight of stairs. Blood pressure is 145/90 mm Hg. Laborawry studies show a serum cholesterol of 320 mgldL. He neither smokes nor drinks. Bruits are evident upon auscultation of both femoral aneries. The pathogenesis of intermittent claudication ln this patient is most closely associated with which of the following risk factors? ***(A) Hyperglycemia ***(B) Hyperlipidemia ***(C) Obesity ***(D) Sedentary lifestyle ***(E) Systemic' hype rtension
Z6 The answer is 8: Hyperlipidemia. The pathogenesis of atherosclerosis is believed to involve in,imy to the imb11a, insudation of cholesterol, activation or platel ets, and growth factor-mediated recruitment of fibroblasts, macrophages, and smooth muscle ce lls. The principal risk factors for atherosclerosis are age, male sex, heredity, lipid metabolism (hypercholesterolemia), obesity, h )'Penension, diabetes mellitus, smok· ing, and sedentary lifestyle. Although several of these lisl< factors apply to this patient (choices A, 0, and E), h yperli pidemia is considered to be the major fac tor in the pathogenesis of atherosclerosis. Chronic ischemia of the lower limbs due to atherosclerosis causes hypoperfusion of the leg muscles. When the blood supply becomes inadequate, usually upon exertion , the muscles develop cramps (imermittent cJaudication). ***Diagnosis: Atherosclerosis
27 A 70-year-old, previously healthy man presents \Vi.th right upper quadrant pain. Physical examination demonstrates hepatomegaly. A llver biopsy reveals a vascular lesion composed of pleomorphic endothelial cells ''lith hyperchromatic nuclei and numerous mitoses. Laboratory tests ror HIV infection are negative. Vv'hich of the following is the most likely diagnosis? ***(A) Angiosarcoma ***(B) Dem1atofibroma ***(C) Glomus tumor ***(D) Hemangioma ***(E) Kaposi sa rcoma
Z7 The answer is A; Angiosarcoma. Angiosarcoma is a highly malignant tumor composed of masses of malignant endothelial cells, The most common locations are the skin, breast, bone, liver, and spleen. Angiosarcoma exhibits varying degrees of di[[erentiarion, ranging from mmors composed of distinct vascular elements to undifferentiated wmors with nuclear pleomorphism, and frequent mitosis. Kaposi sarcoma (choice E) is most commonly associated with AIDS pariems who are inJected by HHV-8_ ***Diagnosis: Angiosarcoma
28 A 60-year-old man with a history of recurrent headaches and blurred vision presents to the emergency room with excruciating pain radiating to his back, beginning 2 hours prior tO admission. Blood pressure on admission is 110/70 mm Hg. He appears pale and sweaty. Echocardiogram shows an enlargemem of the left vermicular wall. ECG is normal. and blood tesrs for cardiac proteins are negative. One hour after admission, the patient experiences pain radiating to his left flank and right side of his neck. The patient suddenly becomes hypotensive and expires. Microscopic examination of the thoracic aona at autopsy is shown in the Image (aldehyde fuchsin stain). Which of the following best characterizes these pathologic findings? ***(A) Acute inflammation ***(B) Atherosclerosis ***(C) Chronic inflammation ***(D) Cystic medial necrosis ***(E) Fibrinoid n ecrosis
28 The answer is D: Cystic medial ne<rosis. Dissecting aneurysms typically affect the thoracic aorta, but may extend to other vessels. The morphologic findings are characteri sLic of a degenerative process known as cystic medial necrosis (of Erdheim). Focal loss of etastic and muscle fibers in the aortic media leads to "cystic" spaces filled with pools of metachromatic myxoid material (see photomicrograph). Neither inflammation (choices A and C), atherosclerosis (choice B), nor fib rinoid necrosis (choice E) are presenL ***Diagnosis: Dissecting aortic aneurysnc, cystic medial necrosis
29 A 65-year-old man presen lS with a 2-week history of abdomi· nal discomfort. Physical examination reveals a pulsatile, abdominal mass in the periumbilical region. ACT scan shows a segment of abdominal aorta proximal to the bifu rcation that is dilated (5 em) and calcified. The patient is scheduled for corrective surgery but suffers a massive stroke and e.:-.:pires. The abdominal aorta is examined at autopsy (shown in the image). Which or the follovving is the most likely underlying cause of this patient's abdominal mass? ***(A) Angiosarcoma ***(B) Atherosclerosis ***(C) Cystic medial necrosis ***(D) Hypercalcemia ***(E) Thromboembolism
29 The answer is B: Atherosclerosis. Aneurysms are localized dilations of blood vessels caused by either congenital or acquired weakness. An aneurysm is defined as an increase in the vessels diameter by at least 50%. Fonns of aneur}'Sm include saccular, fusiform , and dissecting (tear in the media) The large majority of aneurysms of the abdommal aona in elderly patients are related to atherosclerosis. The aneurysm in this patient was opened longimdinally to reveal a large mural thrombus within the lumen (see photograph). The tisk of rupture of an abdominal aortic aneurysm is a funcdon of its size. Aneurysms under 4cm in diameter rarely rupmre, whereas up to 40% of (hose larger than S em mpture within 5 years. Cystic medial necrosis (choice C) is associated with dissecting aortic aneurysm. The other choices do nm cause abdominal aneurysm. ***Diagnosis: Aneurysm, atherosclerotic
30 A 48-year-old woman with familial hypercholesterolemia complains of severe, crushin g, substernal chest pain. The pain is relieved by administration of sublingual nitroglycerin or bed resL The patient subsequendy goes into cardiorespiratory arrest and expires. The left coronary anery at autopsy is shown in the image. The material that has acutely occluded the lumen of this blood vessel is largely composed of which of Lhe following cellular components? ***(A) Activated endothelial cells ***(B) Lipid-laden (foamy) macrophages ***(C) Multinucleated giant cells ***(D) Platelets and red blood cells ***(E) Segmemed neutrophils
30 The answer is D: Platelets and red blood cells. Patiems '.vilh familial hyperlipidemia develop complications of aLherosderosis at an early age, including coronary artery thrombosis. A thrombus is a collection of fibrin and retained blood elements (e.g., platelets and RBCs) that adhere to the vascular \valL Thrombi may result in complete ane1ial occlusion, followed by ischemia and infarction. The mher choices do not represem cellular components of a thrombus. ***Diagnosis: Myocardial infarction
31 A 30-year-old woman with Sjogren syndrome presents with a 24-hour history of a purpuric skin rash_ Which of the following is the most likely diagnosis? ***(A) Buerger disease ***(B) Giant cell granulomatous aneritis ***(C) Hypersensitivity vasculitis ***(D) Thrombotic thrombocytopenic purpura ***(E) Wegener granulomatosis
31 The answer is C: Hypersensitivity vasculitis. Purpura or skll1 rash in a patient with a known amoimmune disease, such as Sjogren syndrome or systemic lupus erythematosus, is usually amibuted to hypersensitivity vasculitis. This vasculitis is caused by the deposition of immune complexes in dem1al venulcs. The other choices are not p1imarily immune complex diseases. ***Diagnosis: Hypersensitivity vasculitis
32 A 62-year-old man is discovered to have hyperlipidemia on screening tests after a routine physical examination_ Laboratory studies show total serum cholesterol of 285 mgldL, LDL of 215mgldl, HDL of 38mg!dL, and triglycerides of 300mgl dL This patient is most at risk of developing an aneurysm in which o[ the followmg anammic locations? ***(A) Abdominal aorta ***(B) Ascending aorta ***(C) Circle of Willis ***(D) Coronmy artery ***(E) Renal ane1y
32 The answer is A: Abdominal aorta. Abdominal aortic aneurysms, which are defined as an increase in aortic ruamerer of 50% or more, are rhe most frequent aneurysms, usually developing after the age of 50 years. The most common cause of these abdominal aonic aneurysms is atherosclerosis. Aneul)' Sms secondary to atherosclerosts are tess common in the other anatomic locations listed. ***Diagnosis: Atherosclerosi.s
33 A 3-year-old gtrl presents wnh htgh fever, extensive skin rash, and conjunctival congestion. Physical examination reveals cervical lymphadenopathy, erythematous palms and soles, and a dry and red oral mucosa. A throat culture is negative. Rouune CBC ts normal, and the monospot test for infectious mononucleosts IS ncgauvc. Addmona] laboratory tests rule om C)'l.OmegaloVlrus mfccuon and toxoplasmosis. Two months later, the chtld develops stgns and symproms of hean fatlure and C\'Cntually goes mto cardtac arrest. The hean at autopsy is shown m the tmage. What 1s the appropnate dtagnosis? ***(A) Buerger disease ***(B) Churg-Strauss disease ***(C) G1am cell granulomawus ancnus ***(D) Kawasaki dtsease ***(E) Takayasu anentis
33 The answer is D: Kawasaki disease. Kawasaki disease, or mucocutaneous lymph node syndrome, is a vasculitis of unknown etiology that presen ts with fever, skin rash, mucosal innammation, and lymph node enlargemem. The disease usuaUy has a self. [imited course but may involve the coronary arteries and lead to aneurysm formation. Heart failure occu rs in l% to 2% of cases. The other choices are not associated with aneurysms of coronary arteries. ***Diagnosis: Kawasaki disease
34 A 45-year-old woman presents with a 4-momh history of severe headaches and pain, and blanching of the hands upon c>rposu re to cold. She is a nonsmoker. Over the past6 months, she has noticed progressive difficulty in swallowing solid food. Phys•cal examination reveals smooth and uglu ski11 over the face and fingers. The serolog.c test for ami-Scl-70 (amitopo•somcrasc) IS positive. Painful hands m this patient arc best descnbed usmg \Vh1ch of the followmg terms? ***(A) Chilblains ***(B) Homans' sign ***(C) lmermittem claudication ***(D) Raynaud phenomenon ***(E) Trousseau phenomenon
34 The answer is 0: Raynaud phenomenon. This patient suffers from scleroderma complicated by Raynaud phenomenon. The latter refers to intermittent, bilaLeral attacks or vasospasm of the fingers or wes (sometimes affecting the ears or nose), resulting in pallor, pa resthesias, and pain from ischemia. The symptoms are precipitated by cold or emotional stimuli and relieved by heat. Raynaud phenomenon may occur as an isolated disorder or as a feature of a number of systemic diseases, including systemic lupus e1ythematosus and sclerodenml. Intermittent claud[cation (choice C) is associated with peripheral vascular disease (atherosclerosis). Chilblains (choice A) represem iLchy or tender red or purple bumps that occur as a reaction to cold. ***Diagnosis: Scleroderma
35 A 6-year-old gi rl presems with a 2-week history of a skin rash over her buttocks and legs and joint pain. The parents report seeing blood in the mine. Physical examination reveals palpable purpuric skin lesions and markedly swollen knees. The resltlts of laboratory studies reveal abnormally high erythrocyte sedimentation rate (30mmlh), BUN of 25 mgtdl , and serum creatinine of 3 mgldL Urinalysis demonstrates RBCs and RBC casts. The stool guaiac test is positive. Biopsy of lesional skin reveals deposits of lgA in the walls of small blood vessels. Which of the following is the most likely diagnosis? ***(A) Henoch-SchonJein purpura ***(B) Hypersensitivity vasculitis ***(C) Kawasaki disease ***(D) Polyaneritis nodosa ***(E) Poststreptococcal glomerulonephritis
35 The answer is A: Henoch-Schonlein purpura. HenochSchtmlein purpura is the most common type of childhood vasculitis and is caused by vascular localization of immune complexes. comaining predominantly lgA. Purpuric skin lesions and glomerulonephritis in a child suggest the diagnosis of Henoch-Schonlein purpura. The other choices do nor include deposits of IgA in tl1e blood vessels. ***Diagnosis: Henoch-SchonJein purpura
36 A 504 year-old woman with type 2 diabetes and hypenension undergoes renal biopsy for evaluation of chronic renal disease. The biopsy is shown in the image. What is the appropriate diagnosis? ***(A) H)'aline aneriolosclerosis ***(B) Kimmelstiel-Wilson disease ***(C) Monckeberg medial sclerosis ***(D) Papillary necrosiS ***(E) Polyarteritis nodosa
36 The answer is A: Hyaline arteriolosclerosis. Hyaline aneriolosclerosis is demonstrated by vessels (smallest aneries and arte1ioles) that are markedly thickened by the deposition of basement membrane material and accumulation of plasma proteins. HyaliJ1e arteriolosclerosis is often seen in the elderly. but more advanced lesions are observed in persons 'Nith diabetes or long-standing hypertension. The other choices are incorrect because they are not diseases of artelioles. Kimmelstiel- Wilson disease (choice B) affects glomeruli in patients with diabetes mellin1s. ***Diagnosis: Hyaline arteriolosclerosis
37 A 79-year-old man presents with a history or extensive ulce rs on both legs for 4 years. A photograph or the patients legs is show·n in the image. What is the appropriate diagnosis? ***(A) Deep venous thrombosis ***(B) Lymphangitis ***(C) Mil roy disease ***(D) Severe aneriolosclerosis ***(E) Varicose veins
37 The answer is E: Varicose veins. The principle diseases affecting the veins are tl1rombophlebitis (tbrombosis-induced local inflammation) and vaticosities. A varicose vein is an enlarged and tortuous blood vessel. Superficial vaticosities of the leg veins, usually in the saphenous system, are among the most common ailments of humans due w our upright posture. Blood How becomes mrbulent and slow. and leaks from the engorged capillaries imo the surrounding tissue cause stasis dermatitis (as shown here). Varicose veins predispose w deep venous thrombosis (choice A). bmthere is no specific evidence [or this condition ln the patients clinical histoty Lymphangitis and Milroy disease (choices Band C) represent diseases of lhe lymphatic syslem. ***Diagnosis: Varicose veins
38 A 76-year-old woman presents whh a 1-hour histmy of substernal chest pain. Shortly after admission the patient expires. At autopsy. extensive calcium deposilS are noted in the cora· nary and otl1er arteries aiTected by severe atherosclerosis. Which of the following tenns best describes these autopsy findings? ***(A) Dystrophic calcification ***(B) Hyperplastic calcification ***(C) Hypemophic caldficalion ***(D) Metastatic calcification ***(E) Physiologic calcification
38 The answer is A: Dystrophic calcification. Dystrophic calcification is a response to cell injury. Serum levels of calcium are normal, and the calcium deposits are located in previously damaged tissue. This patient suffered from coronaty ane1y atherosclerosis. The pathogenesis of the atherosclerotic plaque is a dynamic process that usually occurs over decades, leading to erosion, ulceration or fissuring of the surface of the plaque; plaque hemorrhage; mural thrombosis; and calciFication. Calcification is thought to depend on a balance of mineral deposition and resorption in areas of vascular necrosis. These apposing metabolic processes are regulated by osteoblast-like and osteoclast-like cells in the vessel wall. Metastatic calcification (choice D) is associated with hypercalcemia. Physiologic cakificadon (choice E) occurs in normal bone. ***Diagnosis: Atherosclerosis, dystrophic calcification
39 A 48-year-old man with a longscanding hiswry of chronic constipation complains of anal itching and cliscomfon toward the end of the day. He desCJibes a perianal pain when sitting and finds himself sitting sideways to avoid discomfort. Physical examination reveals painful varicose dilations in the anal region, associated with edema. \IVhich of the following is the most lil<ely diagnosis? ***(A) Anal cancer ***(B) Anal fissure ***(C) Hemorrhoids ***(D) Ischiorectal abscess ***(E) Rectal cancer
39 The answer is C: Hemorrhoids. Hemorrhoids are dilations of veins of the rectum and anal canal, which may occur inside or outside the anal sphincter. This condition may be aggravated by constipation and pregnancy and can also result from venous obstruction by rectal mmors. Thrombosis of hemorrhoids is exquisitely painful. or interest, internal hemorrhoids typically bleed without pain, whereas external hemorrhoids typically hurt but do not bleed. The other choices do not feature dilaled veins. ***Diagnosis: Hemorrhoids, varicose veins
40 A 45-year-old woman complains that her fingers [eel stiff. On physical examination the skin of the patiem's face appears tense and radial funows are evident around the mouth. Laboratmy stucUes establish a diagnosis of scleroderma. Several years later, the patiem subsequently develops renal insufficiency. A renal biopsy is shown in the image. Which of the following best describes the pathogenesis of renal vascular involvement in this patient wi!.h progressive systemic sclerosis? ***(A) Accelerated atherosc1erosis ***(B) Cystic medical necrosis ***(C) Organization of microthrombl ***(D) Perivascular arteritis ***(E) Subintimal fibromuscular thickening
40 The answer is E: Subintimal fibromuscular thickening. Scleroderma is charaClerized by vasculopalhy and excessive collagen deposition in the skin and a variety of imemal organs. The disease occurs four times as often m women as in men, mosdy in persons between 25 and 50 years of age. Lesions in the arteries, arterioles, and capilJaries are typical, and m some cases may be the first demonstrable pathological finding in the disease. The kidneys are involved in more than half of patients \'lilh scleroderma. They show marked vascular changes, often with focal hemorrhage and cortical infarcts. Among the most severely affected vessels are the interlobular aneries and afferent arterioles. Early fibromuscular thickening of the subimLma causes luminal narrowing, followed by fibrosis. The other pathologic lesions may occur, but they are not the principal cause of vasculopathy in patiems with scleroderma. ***Diagnosis: Scleroderma
41 An 80-year-old woman with a history of smoking experiences acme chest pain and is rushed to the emergency room. Laboratory studies and ECG confirm a diagnosis of myocardial in[arction; however, coronary artery angiography 2 hours later does not demonstrate evidence of vessel occlusion. 'vVhich of d1e following proteins mediated fib rinolysis at d1e site of temporary blockage in this patiem:S coronary anery? ***(A) Bradykinin ***(B) Collagenase ***(C) Kallikrein ***(D) Plasmin ***(E) Thrombin
41 The answer is D: Plasmin. A thrombus may undergo several fates including (l) lysis, (2) growth and propagation, (3) detachment and embolization, or (4) organization and canalization . The combination of aggregated platelets and clotted blood is made unstable by the activation or plasmin. Plasmin is a fibrinolytic enzyme that is fom1ed from plasminogen by tissue plasminogen activator (tPA). tPA is a serine protease that catalyzes the conversion of plasminogen to plasmin. Once formed, plasmin lyses fibrin and dissolves the thrombus. Occlusive thrombi can also be dissolved by enzymes, such as streptokinase, that activate plasma fibrinolytic activity The conversion of plasminogen to plasmin, and the activity of plasmin, are regulated by specific inhibitors. None of the other choices are involved in fibrinolysis. ***Diagnosis: Myocardial infarction, thrombolysis
1 A heart murmur is noted during the preschool physical examination of a 4-year-old girl. An echocardiogram reveals a defect between the right and left atrium involving the limbus of the toramen ovale. What is the most likely diagnosis? ***(A) Aerial septal detect (ASD), ostium primum ***(B) ASD, ostium secundum ***(C) Tetralogy ofFallOl ***(D) Tmncus arteriosus ***(E) Ventricular septal defect
1 The answer is B: ASD, ostium secundum. Ostium secundumtype ASD accoums for 90% of all ASDs. It reflects a true deficiency of the atrial sepLum and should nm be confused with patem foramen ovale. OsLium secundum defects commonly occur in lhe middle ponion of the septum and vary in size from a trivial opening to a large defect of the entire fossa ovalis. A small ASD is unlikely Lobe functional, but LefL-to-rtght shunting may occur in Large defects, causing dilation and hypertrophy of lhe right atrium and ventricle. The ostium pnmum anomaly occurs adjacent LO the AV valves and is usually associated with a cleft ame1ior mitral leaflet. Ventricular septal defect (choice E) occurs in the imervemncular septum The diagnoses in choices C and D are characterized by multiple congenital abnormalities and early cyanosis. ***Diagnosis: Ao·ial septal defect
2A 15-year-old girl is brought to the emergency room with heart palpitations and dyspnea. Her past medical history is significant for an unrepaired atrial septal defect (.ASD). Physical examination reveals cyanosis, cUstended JUgular veins, hepatosplenomegaly, and a sysmlic ejection murmur. This patient has most likely developed which of the following complications of congenital heart disease? ***(A) Aortic anelll)'Sm ***(B) Myocardial infarction ***(C) Parado:Aical embolism ***(D) Pneumonia ***(E) Pulmonary hypertension
2 The answer is E: Pulmonary hypertension. Patieni.S with uncorrected ASDs may develop narrowing of the pulmonary vasculature, in which case the (low of blood through the defect may be reversed, thereby creating a Ti gh t-to- left shunt. The increase in pulmonary capillary pressure is transmitted to the pulmonary arte1ies, a condition termed pulmonary hypertension, ultimately resulting in right-sided hean failure. Complications of ASDs include cyanosis, atrial arrhythmias, right ventricular hypertrophy, tight heart failure, bacterial endocarditis, and paradoxical emboli. None of the other cholces are associated wi rh a right-to-left shunt. ***Diagnosis: Eisenmenger syndrome, atrial septal defect
3 A 5-year-old boy is found to have a harsh holosystolic murmur heard at the left 4th intercostal space. The child has a history of recurrent pneumonias and respiratoty tract infections. An echocardiogram reveals a hean defect and biventricular cardiac hypertrophy. Cardiac catheterization discloses pulmona.y hypertension. This patient likely has which of the fol lowing congenital heart diseases? ***(A) Coarctation of aorta ***(B) Hypoplastic left ventricle ***(C) Patent ducLus aneriosus ***(D) Pulmonic stenosis ***(E) Ventricular septal defect
3 The answer is E: Ventricular septal defect {VSD) . VSD is the ..__ most common congenital heart lesion. Such anomalies occur as isolated lesions or in combination with other malformations. VSDs vary in size, occuni.ng as a small hole in the membranous septum, a large defect involving more than the membranous portion, defects in the muscular portion, or a complete absence of septum. Small VSDs may have little functional signlficance and often close spontaneously. Large VSDs are initially characterized by a left-to-tight shunt, with left ventricular dilation and congestive hean failure. If the patient survives long enough, pulmonary hypenension may cause reversal of the direction of the shunt and Lead to cyanosis. Coarctation of aorta (choice A) presents only with left ventricular hypertrophy and does not typically cause pulmonary hypertension. Pulmonic stenosis (choice D) presems \vith right ,·emricular hypertrophy. ***Diagnosis: Vennicular septal defect
4 An 8-month-old girl with Turner syndrome is brought to the emergency room by her parents, who complain that their daughter is breathing rapidly and not eating. Physical examill ation reveals tachypnea, pallor, absent femoral pulses, and a munmn heard at the left axilla. There is hypertension in the upper extremities and low blood pressure in both legs. A chest X-ray shows notchi.ng or scalloping of the ribs. WhaL is the appropriate diagnosis? ***(A) Aortic valve stenosis ***(B) Atrial septal defect ***(C) Coarctation of aorta ***(D) Patent ductus aneriosus ***(E) Tetralogy ofFalloL
4 The answer is C: Coardation of aorta. Aortic coarctation is common, accounting for 6% of all cases of congenital heart disease. Coarctation usuaUy occurs in the thoracic portion of the descending aorta, distal to the takeoff of the left subclavian artety at the site of the ductus arteriosus. The incidence of a bicuspid aonic valve in association with coarctation is 80% to -85%. The cardinal physical finding is dimmution or absence of femoral pulses and hypenension in the upper extremities. Notching or scalloping of the tibs caused by marked enlargement of the intercostal collaterals can be seen on X-ray. None of the other choices exhibit these charactetistics. Aortic valve stenosis {choice A) does not present with hypertension in the upper extremities. ***Diagnosis: Coarctation of aorta
5 A 2.-week-old boy is i tTitable and feeding poorly On physical ....__ examination, the infant is irritable, diaphoretic, tachypneic, and lachycardic. There is circumoral cyanosis, which is not alleviated by nasal oxygen. A systolic thrill and holosysLolic murmur are heard along the left sternal border. An echocardiogram reveals a hean defect in which the aorta and pulmonaty artery form a single vessel that overrides a ventticular septal defect. What is the approptiate diagnosis? ***(A) Atrial septal de feet ***(B) Coarctation of aorta , preductal ***(C) Patem ductus aneriosus ***(D) Tetralogy ofFallot ***(E) Truncus aneriosus
5 The answer is E: Truncus arteriosus. Truncus arteriosus refe rs to a common trunk for the origin of the aorta, pulmonary aneries, and coronary aneries.lt results from absenL or incomplete partitioning of the truncus arteriosus by the spiral septum during development. Most infants with persistent u1.mcus anen osus have wrrenual pulmonary blood flow, whtch leads w heart failure. None of the other choices are chsLi.nguished by a single vessel that carries blood from the heart. ***Diagnosis: Truncus arteriosus
6 A 2-week-old girl is found to have a harsh munnur along the left sternal border. The parents report that the baby gets "bluish" when she cries or drinks fTom her bottle. Echocardiogram reveals a congenital hean defect associated v.rith pulmonaty stenosis, ventricular septal defect, dextropositjon of the aona, and ligh t ven tricular hypertrophy. What is the appropriate diagnosis? ***(A) Ao·ial septal de feeL ***(B) Coarctation of aorta, postductal ***(C) Coarctation of aorta, preductal ***(D) Tetralogy of Fal1ot ***(E) Tmncus aneriosus
6 The answer is D: Tetralogy of Fallot. Tetralogy of Fallot is defined by four anatomic changes: pulmonary stenosis, ventTicular septal defect, dextroposilion of the aorta, and right ventricular hypertrophy. lt is the most cotrunon cyanotic congenital heart disease, accounting for 10% of all con genital bean defects. Cyanosis appears shortly after birth or in early infancy due wright- to-left shunting of venous blood from the 1ight venui cle into Lhe de.-..moposed aorta. The aorta overrides the ventricular septal defect and receives blood from both ventricles. Narrowing of the pulmonary an ery impedes the entry of blood lnro the lung, thereby increasing the pressure in the 1ight ventricle. None of the other choices exhibit this particular constellation of congenital h eart defects. ***Diagnosis: Tetralogy of Fallot
._7_ A 2-momh-old boy is admitted to the pediatric ward in acme respiratory distress. Physical ex:ammation shows pallor, peripheral cyanosis, tachypnea, intercostal retractions, and nasal flaring with gruming. Cardiac auscultation reveals third heart sounds. An X-ray film of the chest shows severe cardiomegaly and bilateral pleural effusion. The patient expires 2 days after admission. The heart at autopsy is shO\vn in the linage. Which of the following is the most likely diagnosis? ***(A) Cardiac amyloidosis ***(B) Endocardial fibroelastosis ***(C) Hypoplastic left hcan syndrome ***(D) Patent ductus anenosus ***(E) Tetralogy of Fallot
7 The answer is B: Endocardial fibroelastosis (EF) . EF is characterized by fibroelasmric thickening of the endocardium of the left ventricle, which may also affect the valves. The endocardium and valves of the left ventricle show im~gular gray-white patd1es of fibroelastotic thickening usually accompanied by degeneration of sub endocardial myocytes. EF occurs in association witl1 underlying cardiovascular anomalies that lead to left vennicular hypenrophy in rhe face of an inability to meet the increased oxygen demands of the myocardium (e.g., aortic stenosis and coarctation of the aona). Endocardial thickening observed m the photograph is nola feature of the other choices. ***Diagnosis: Endocardial fibroelastosis
8 A 29-year-olcl woman complains of a 3-momh history of nervousness and weakness. $he feels hot and sweaty and has experienced a 9-kg (20-lb) weight los_<; over the past 2 months, despile increased caloric intake. She frequently finds her heart racing and can feel it pounding in her chesl. Physical examination reveals an enlarged thyroid , warm hands , and bulging eyes. This patient is at risk of developing which of the following cardiovascular complications? ***(A) Cardiac tamponade ***(B) Cor pulmonale ***(C) High-output heart failure ***(D) Pericardia! effusion ***(E) Ventricular aneurysm
8 The answer is ( : High-output heart failure. The patient suffers from hypenhyroidism (Graves disease). This autoimmune condjtion causes decreased periph eral resistance, which in time requires increased cardiac omput. Hypen byroidism may lead to high-outpm heart failure None of the other choices are characterized by increased cardiac output. ***Diagnosis: Graves disease , hyperthyroidism
9 A 56-year-old man presems to the emergency room Virith 1 hour of chest pain. Laboratory studies show an increased leukocyte count and increased serum levels of cardiac enzymes. ECG confirms a massive transmural myocardial infarction of the left ventride. The patient dies 2 days later. Histologic examination of the left main corona1y artery at autopsy is shO\.vn in the image. Examination of injured heart muscle would be expected to show which of the following pathologic changes by Hght microscopy? ***(A) CoUagen-rich scar tissue ***(B) Extensive infiltration of myocardium with mononuclear cells ***(C) Necrosis of cardiac myocytes and infiltrates of neutrophils ***(D) No obvious changes evident by light microscopy ***(E) Proliferation of fibroblasts and capillary endothelial cells
9 The answer is (: Necrosis of cardiac myocytes and infiltrates of neutrophils. Two days after myocardial infarction, the affected hean muscle will show myofiber necrosis, edema, and focal hemorrhage Polymorphonuclear leukocytes accumulate at the border of the zone of infarction and infiltrate the necrotic tissue. Lack of changes evident by light microscopy (choice D) is expected during the first 24 hours but not at 2 days. The other changes (choices A, B, and E) occur after longer periods of Lime. ***Diagnosis: Acute myocardial infarction
10 A ++-year-old man presents to the emergency room "vith acute chest pain_ The ECG is normaL Analysis of which pair of serum markers given below would be most helpful in excluding a diagnosis of acme myocardial infarction in this patiem? ***(A) Cardiac troponin-1 and myoglobin ***(B) CK-BB and myoglobin ***(C) CK-MB and cardiac troponin-l ***(D) CK-MM and lactate dehydrogenase (LDH)-1 ***(E) Myoglobin and CK-BB
10 The answer is C: CK-MB and cardiac troponin·l. Laboratory evaluation of myocardial infarction is based on measuring blood levels of intracellular macromolecules that leak out of necrotic myocardial cells. These proteins include myoglobin, cardiac troponin-T and -1, creatine kinase (CK), and LDH. The preferred biomarkers for myocardial damage are cardiac-specific proteins, panicularly troponin-1 and troponin- T. The MB isofom1 of creatine kinase (CK-MB; ch oice C) is also cardiac specific. CK-BB (choice B) is elevated ptimarlly in patients with infarction of brain tissue. CK-MM (choice D) and myoglobin are usually elevated following necrosis of skeletal muscle. ***Diagnosis: Ischemic hean disease
11 A 35-year-old man presents with acute chest pain and nausea_ ECG and laboratory studies confirm the diagnosi5 of myocardial infarction_ The patient expires .2 days later_ A phowmicrograph of hi5 left main coronary anery is shown in Lhe image. This patient most likely suffered from which of the following hereditary diseases? ***(A) Ehlers-Danlos syndrome ***(B) Familial hypercholesterolemia ***(C) Kawasaki disease ***(D) Marfan syndrome ***(E) Systemic hypertension
11 The answer is 8: Familial hypercholesterolemia. Patients with familial hypercholesterolemia have a defective LDL receptor, whkh inhibits cholesterol uptake by the liver. They develop complications of atheroscl erosis ar an early age. Cholesterol and serum Lipoproteins are deposited in the atheroma, where they are continuously endocytosed by macrophages (lipidladen foam cells) The mamre atheroma is highly thrombogenic Although systemic hypertension (choice E) may accelerate atherosclerosis, it is nol a common cause of early myocardial infarction. The other choices (choices A, C, and D) do not accelerate the development of atherosclerosis. ***Diagnosis: Myocardial infarction, familial h)rpercholesterolemia
12 A 60-year-old man with a history of emphysema remrns home from the hospital after su ffering a myocardial infarction involving the apex of the left vetmicle. $b{ months later, an echocardiogram reveals the development of a ventticular bulge that does not contract during syswle. The padem subsequently suffers a massive stroke and suddenly e>.'Pires. Which of the following is an expected pathologic finding ar autopsy"? ***(A) Calcific aortic stenosis ***(B) Dilated cardiomyopathy ***(C) Milral valve prolapse ***(D) Mural thrombus ***(E) Ventricular rupture
12The answer is D: Mural thrombus. Mural thrombi Conn on the endocardium, over Lhe affected myocardium, early after infarction and are found in half of all patients who die after myocardial infarction. Mural thrombi also form over ventricular an eurysms, as in this case, which are found at the site of a h ealed, transmural myocardial infarct. Such thrombi are a source of systemic emboli. Mural thrombi may fonn in cases of dilated cardiomyopathy (ch oice B), but there is no clinical evidence for Lhat disease in th is vignette. ***Diagnosis: Mural lhrombus, venmcular aneurysm
13 A 50-year-old man with familial hyperlipidemia undergoes resection of an abdominal aneurysm. Signs of congcslive hean failu~ develop shortly after surgery. Despite treatment, the patient becomes hypotensive and expires 2 days Later. Autopsy reveals marked narrowing of coronary ane1ies, v.Tithomthrombosis. Multiple foci of necrosis a rc found circumferemially around the inner walls of borh ventricles. Which of the following is the most likely cause of congestive heart failure in this patient? ***(A) Calcific aortic stenosis ***(B) Dilated cardiomyopathy ***(C) Rupmre of papiUary muscle ***(D) Subendocardial myocardial infarction ***(E) Transmural myocardial infarction
13 The answer is D: Subendocardial myocardial infarction. Subendocardial circumferential infarcts generally occur as a consequence of hypoperfusion of the heart secondaty to poor coronaty blood flow, often in the setting of hypotension. Coronary artery narrowing is common, but total occlusion is usualiy not seen. $ubendocarcUal myocardial infarcts aiTectthe inner one third ro one half of the ventricle. They may arise within the terrirory of one of the major coronary arteries or may involve the subendocardial distribution of all coronary aneties. Transmural myocardial infarction (choice E) generally follows occlusion of a major coronary anery. ***Diagnosis: Subendocardial myocardial infarction
14 A 68-year-old obese woman (BMI = 32 kglm1 ) presents witl1 substem al chest pain and a histmy of recurrent angina pecwris and imermittent claudication. The following day, she develops a fever of 38°C (101 °f). Results of laboratmy studies include an elevated WBC count (13,000/J.t L), CK-MB of 6 .6nglmL, and troponin-1 of 2.5ngtml. ECG confirms a myocardial infarction of the left ventricular walL \ .Vhich of the following mechanisms is most likely responsible for the m>'ocardial infarction in this palicnt? ***(A) Coromuy artery tllrombosis ***(B) Corona1y artery vasospasm ***(C) Decreased collateral blood fiow ***(D) Deep venous thrombosis ***(E) Paradox'ical embolism
14 The answer is A; Coronary artery thrombosis. Coronary artery thrombosis is the most common cause of acute myocardial infarction and is often secondary to rupture of an atherosclerotic plaque. Coronary artery vasospasm (choice B) would be unlikely in this clinical setting. ***Diagnosis: Myocardial infarction
15 Two months later, the patient described in Question 14 expeliences several days of severe, sharp, re trosternal chesL pain radiating to Lhe neck and shoulders. The pain is worse when she is in the supine position and less imense when she is sitdng upright and leaning fon.vard. Which of the following is the most likely cause of the patients symptoms? ***(A) Congestive hean failure ***(B) Dressler syndrome ***(C) Left vennicular aneurysm ***(D) Mural thrombi ***(E) Ruptured papillary muscle
15 The answer is 8: Dressler syndrome. Postmyocardial infarction syndrome (Dressler syndrome) refers to a delayed form of pericarditis that develops 2 to 10 weeks after myocardial infarction. The pain associated with pericarditis may be confused with that resulting from postinfarction angina or recurrem infarction. The other choices would nm present whh a prolonged history of chest pain or the clinical fearures seen in this case. ***Diagnosis: Dressler syndrome
16 A 65-year-old man with a 2-year history of angina pecwris is admiued w the h ospital with excmciating substernal chest pain that is not reli.eved by rest or medication. Physical examination shows diaphoresis and dyspnea. Results of laborawry studjes include Vv'BC of 13,000/jlL, CK-MB of 6.8nglmL, and troponin-l of 3.0 nglmL. An ECG shows ST segment elevation. The patient expires 1 hour after admjssion. At auwpsy, the hean is found to be enlarged bm otherwise anatomically normal. Which of the following is the most likely cause of death? ***(A) Cardiac tamponade ***(B) Postmyocardial infarction syndrome ***(C) Ruptured myocardial infarct ***(D) Septal perforation ***(E) Ventricular fib1illation
16 The answer is E: Ventricular fibrillation. Arrhythmias account for half of all deaths within the first 2.4 hours after an acute myocardial infarction. Acute infarction is often associated \vith pre rna Lure ventricular beats, vemricular tachycardia, com plele heart block, and venLricular fib tillation. The cause of arrhyth· mias is multifacrorial but acute ischemia may promote conduction dismrbances and myocardial i rritabilil y. The other choices occur after longer time pe1iods. ***Diagnosis: Myocardial infarction
17 After suffering an acute myocardial infarction , a 54-year-old man presents to the emergency room 3 ·weeks later, complaining of sharp pain on the left side of his chest. On physical examination, the patient is apprehensive and sweating His blood pressure is 80/40mmHg and the pulse rate is 100 per minme. The patient dies within minutes. The ventricular wall at autopsy is shovvn in Lhe image. What is Lhe most Hkely cause of death? ***(A) Cardiac tamponade ***(B) Dissecting aortic aneurysm (C} Pulmonary edema ***(D) Pulmonary thromboembolism ***(E) Septic shock
17 The answer is A: Cardiac tamponade. The gross specimen shows a ruprured myocardial wa1l, which causes cardiac tamponade. This catastrophe reflects the accumulation of pericardia] nuid (blood in this case), which restricts the mmion of the hean. Pulsus paradoxus (>lOmmHg fall in anerial blood pressure with inspiration) is commonly observed in such cases. Although aortic dissection (choice B) can break through to the pericardium. it does not cause rupture of the myocardium. The other choices do not cause cardiac tamponade. ***Diagnosis: Hemopericardium
18 A 48-year-old man complains of chest pain upon exertion. He had been well umil 4 months previously. when he first developed a chest discomfort while jogging. His symptoms have progressed to the point that he now develops chest pain after climbing a single flight of stairs. He has a history of diabetes controlled by diet and of 25 pack-years of cigarette smokin g. His father and maternal grandfather both died of hea1t disease before the age of 60. On the 5th hospital day, the patient develops chest pain dming periods of mild activity, which is minimally responsive to sublingual nitroglycerin. Results of laboratory studies include WBC of 8,100/J.ll, CK-MB of 4.5 nglmL, and troponin-1 of 0.5nglmL Which of the following is the most likely diagnosis? ***(A) Acute myocardial infarction ***(B) Cardiac arrhythmia (C} Dressler syndrome ***(D) Pulmonary thromboembolism ***(E) Unstable angina
18 The answer is E: Unstable angina. Unstable angina refers to a pattern of pain that occurs progressively and wid1 increasing frequency. It often occurs at rest and tends to be of more prolonged duration. Unstable angina is often a prodrome of myocardial infarction. Acme myocardial infarction (choice A) is ruled out 111 this case because blood ana1vsis does not show ' evidence of infarction. ***Diagnosis: Ischemic heart disease
19 On the morning of the sixth day. the patient described in Quesdon 18 experiences acute , substernal chest pain and nausea. He is rreated with plasminogen activator and O),.'ygen but expires several hours later. A cross section of the heart at autopsy is shown in the image. This infarct was most likely caused by thrombotic occlusion of which of the follo\ving coronary arteries? ***(A) Left anterior descending ***(B) Left meum ncx ***(C) Main right ***(D) Postenor descendmg ***(E) Sinoamal nodal
19 The answer is A: Left anterior descending. The left amenor descending coronary ane1y supplies the anterior and part of the lateral portions of the left ventricle. Acme blockage of this ane1y produces an infarct of the apical. ame1ior, and ameroseptal walls of the left ventricle. Sinoatrial (SA) nodal anery (choice E) primarily supplies the right atrium and the SA node. The distribudon of choices C and D largely encompass the poste1ior wall of the heart. ***Diagnosis: Myocardial infarction
ZO A 69-year-old woman presents w1th cruslung substernal chest pain and nausea. She ts treated w1th plasminogen acth•awr, o.>.:ygen, and morphme sulfate. Laborarmy studies show a WBC count of 13 ,000/J.l l , CK-MB of 6.6nglml, and troponin- 1 of 2.5 n?fmL. An ECG shows ST segment elevation. Cardiac catheterizauon reveals diffuse atherosclerosis of all major coronary anencs. The panent subsequently becomes acutely hypotensive and undergoes cardiac arrest. The surface of the hean aL autopsy ts shown m the image. At what point in dme following acuLc myocardial infarction did this pathologic condition most likely occur? ***(A) 0 lO 6 hours ***(B) 6 w 12 hours ***(C) l2 to 24 hours ***(D) 1 to 4 days ***(E) 6 momhs to 1 year
20 The answer is D: 1 to 4 days. The autopsy reveals hemopelicarclium {see photograph). Myocardial rupture and hemorrhage imo the pericardia! sac may occur at almost any time during the first 3 weeks following infarction, but is most commonly seen between the 1st and 4th days. During this critical interval, the infarcted wall is weak, being composed of soft necrotic tissue. The extracellular matrLx within the infarct is degraded by prOleases released by inflammatory cells. Choices A, B, and C are incorrect because the strength of the ventricular wall is maintained during the first 24 hours. Choice E is incorrect because the scar tissue that has formed by this nme provides mechanical stability w the site of injury. ***Diagnosis: Hemopericardium, cardiac tamponade
21 A 66-year-old woman collapses while shopping and expires suddenly of cardiac arrest. Her past medical history is signifi· cant for long-standing type 2 diabetes mellitus. Her relatives note that she had complained of chest heaviness and shor tness of breath for the past 2 weeks. Sterile fibrinous peri· carditis and pericardia! effusion are observed at autopsy. ·wh at additional finding would be expected during autops)' of this patient? ***(A) Endocardial fibroelastosis ***(B) Marantic endocarcUtis ***(C) Mitral valve prolapse ***(D) Myocardial infarct ***(E) Right ventlicular hypertrophy
21 The answer is D: Myocardial infarct. Fibrinous pericarditis may develop 2 to 10 weeks after a nansmural myocardial infarction. Patients with long-standing diabetes mellitus are particularly susceptible to coronary atherosclerosis and myocardial infarction. One fourd1 w one half of all nonfatal myocardial infarctions arc asympLOmatic. ln such patients, sudden dead1 usually reGects a cardiac arrhydunia. Right ventricular hypertrophy (choice E) may be encountered but is not related w the development of pericarditis. ***Diagnosis: Myocardial infarction
22 A 68-year-old woman with a history of diabetes mellitus expires suddenly of cardiac arrest The padent suffered a massive anterior myocardial infarction l year earlier. The heart at autopsy is shown in the image. 'vVhich of the following is the most likely complication of this condition7 ***(A) Aonic stenosis ***(B) Coronary artery aneurysm ***(C) Hypertrophic cardiomyopathy ***(D) Pulmonary embolism ***(E) Stroke
22 The answer is E: Stroke. As myocardial infarcts heal, newly deposited collagenous matrix is susceptible to stretching and may become dilated. Mural thrombosis in the aneurysm is common and may lead LO the release of emboli to the brain (stroke). Pulmonary embolism (choice D) involves the venous system and the right-sided circulation of the hean. ***Diagnosis: Ventlicular aneurysm, mural thrombus
23 A 60-year-old woman with a 30-pack-year history of smokJng and a 10-year history of emphysema expires of congestive heart failure. There is no evidence of coronary artery disease or valvular heart disease. The heart at autopsy is shown in the image. Which of the following is the most likely cause of right vcnuicular hypertrophy? ***(A) Endocardial fibroelastosis ***(B) Essential hypertension ***(C) Pulmonary hyperten.':;ion ***(D) Pulmonary sLenosis ***(E) Systemic hypertension
23 The answer is C: Pulmonary hypertension. Cor pulmonale is defined as right ventricular hypemophy and dilation secondary to pulmonary hypenension. The most common cause of cor pulmonale is chronic obstructive pulmonary disease usually as a result of smoking. Pulmonary stenosis (choice D) is a rare cause of cor puJmonale. ***Diagnosis: Cor pulmonale, pulmonary hypertension
24 A 72-year-old man presenl5 with difficulty breathing He says that he becomes shan of breath at night unless he uses three pillows to prop himself up. Measuremems of VItal signs reveal normal temperature, mild tachypnea, and a blood pressure of 180/100 mm Hg Physical e..xamination discloses obesity, bilateral 2+ pitting leg edema, hepatosplenomegaly, and rales at the bases of both lungs. An X-ray film of the chest shows mild enlargement of the hean and a mild pleural effusion. Echocardiography reveals left ventricular hypenrophy withom valvular hean defects. Which of the following is the most likely diagnosis? ***(A) Acute cor pulmonale ***(B) Consnktive pericarditis ***(C) Dilated cardiomyopathy ***(D) Hypertensive heart disease ***(E) R£nal failure
24 The answer is D: Hypertensive heart disease. Systemic hypertension is one of the most prevalen t and serious causes of corona1y artery and myocardial disease in the United States. lt is defined as a persistem increase in systemic blood pressure Lo levels above 140mm Hg systolic or 90 mm Hg diastolic, or both. Chronic hypertension causes compensatory hypertrophy of the left venttide as a result of the increased workload imposed on the heart muscle. The left ventricular wall and interventricular septum become uniformly thickened. Congestive heart failure is the most common cause of death in untreated hypertensive patients. lnn·acerebral hemorrhage is also a frequent fatal complication. ln addition, death may result from coronary atherosclerosis and myocardial infarction, dissecdng aneurysm of the aorta, or ruptured beny anemysm of the cerebral circulation. Renal failure may supervene \Vhen nephrosclerosis induced by hypertension becomes severe. Although dilated cardiomyopathy (choice C) may present with similar clinical findings, it does not feamre hypertension. The other choices do not exhibit left ventricular hypertrophy ***Diagnosis: Hypertensive hean disease
25 A 62-year-old man with a lustory or hypertension is brought to the emergency room with severe left chest and back pain. His blood pressure is 80/50 nm1 Hg. Physical examination shows pallo r, diaphoresis, and a murmur of aortic regurgitation. An ECG does not show myocardial infarction. An X-ray fih11 of the chest reveals mediastinal widening. Which of the following is the most likely diagnosis' ***(A) Bacterial endocarditis ***(B) Dissecting aneurysm ***(C) PeJicarditis ***(D) Pulmonary thromboembolism ***(E) Ruptured myocardial wall
25 The answer is 8: Dissecting aneurysm. Aortic dissection re n.ects hemorrhage between and along the laminar planes of the media, \vith the fonnalion of a blood-filled channel withln the aortic wall. lf it begins in the ascending aorta, it may extend backward toward the aortic valve or distally to involve the thoracic and abdominal aona. More than 90% of dissections occur in men between the ages of 40 and ()0 with amecedent hypertension. The second major group of patients, usually younger, has a systemic or localized abnonnality o[ connective tissue that affects the aon:a (e.g. , Marfan syndrome). Although bacterial endocarditis (choice A) can result in aortic regurgitation, it does not lead to acme chest pain. The other choices are not associated \Vith aortic regurgitation ***Diagnosis: Aortic dissection
26 A 30-year-old woman presents with a heart murmur. There is a history of recunent episodes of arthtitis, skin rash, and glomerulonephtili. s. Blood cultures are negative. Laboratory tesl5 for antinuclear antibodies (ANA) and anti-double-stranded DNA are positive. Which of the following is the most likely cause of heart munnur jn this patiem? ***(A) Libman·Sacks endocarditis ***(B) Mitral valve prolapse ***(C) Myocardial infarct ***(D) Mitral valve prolapse ***(E) RJ1eumatic fever
26 The answer is A: Libman-Sacks endocarditis. In patients with systemic lupus erythematosLLS , endocarditis is the most suiking cardtac lesion, termed Libman-Sacks endocardins. Nonbacterial vegetations are seen on the undersurface of the mitral valve close to the origin of the leaGets from the valve ring (Ubman-Sacks endocarditis). There is fib1inoid necrosis of small vessels with focal degeneration of imerstiLial tissue. Rheumatic (ever (choice E) is not commonly associated with AN As seen in this case. ***Diagnosis: Systemic lupus erythematosis
27 A 50-year-old man wi.th adenocarcinoma of the pancreas is brought to the emergency room in a comatose state. A CT scan of the brain is consistent wi.th a recent infarct in the left temporal lobe. Blood cultures are negative. The patiem n ever regains consciousness and expires 2 days later. The heart at autopsy is shown in the image. Which of the follmving is the most likely underlying cause of s troke in this patient? ***(A) Calcine aonic stenosis ***(B) Carcinoid heart disease ***(C) Cardiac metastases ***(D) Nonbacterial thrombotic endocarditis ***(E) Subacute bacterial endocarditis
27 The answer is D: Nonbacterial thrombotic endocarditis. Nonbacterial thrombotic endocarditis, also known as maramic endocarditis, refers to the presence or sterile vegetations 011 apparently normal cardiac valves, almost always in association with cancer or some other wasting disease. The caLLSe of marantic endocarditis is poorly understood, but it has been attributed Lo increased blood coagulabili ty and immunecomplex deposition. The principal danger is embolization to distant organs. Subacme bacterial endocarditis (choice E) features positive blood cultures and is not panicularly associated with cancer. ***Diagnosis: Marantic endocarditis
28 A 78-year-old man with a histoty of recurrem syncope undergoes surgery for aortic valve disease. A hard. markedly defmmed valve is obse1ved . but the patiem ex'Plres dming surgery. The aonic valve at autopsy is shown in the image. What is the appropriate diagnosis? ***(A) Bacterial endocarditis ***(B) Bicuspid aortic valve ***(C) Calcific aortic stenosis ***(D) Hypenropl1ic subaonic stenosis ***(E) Maranuc endocardins
28 The answer is C: Calcific aortic stenosis. The aortic valve shows calcific aonic s tenosis in a three-cLLSpid valve in an elderly person. There is no commissural fusion. Calcific aonic stenosis refe rs to a narrowing of the aonlc valve or ifice as a result of the deposition of calcium in the valve cusps and ring. There are three main causes of calcific aortic stenosis: rheumatic disease, senile calcific stenosis, and congenital bicuspid aortic stenosis. Calcific aortic stenosis is related Lo the cumu· lative effect of years of trauma due to turbulent blood now around the valve. Bicuspid aortic valve (choice B) is incorrect because three valve cusps are shown. Vegetations of marantic endocardi tis (choice E) are absent. Patients with hypertrophic cardiomyopathy may develop subvalvular obstruction or the aonic oULflow tract (choice D), but the autopsy specimen does not show this pathology. ***Diagnosis: Calcific aortic stenosis
Zl A 16-year-old girl, who arrived in the United States from Afri ca, comes to the hospital with ch est pain and respiratory distress. On physical examination, the patient is short of breath, wheezing, and gasping for ai r. A prominent pansystolic heart murmur and a prominem third heart sound are heard on cardiac auscultation. An X-ray study of the chest shows marked enlargement of the heart The patient expires despite intense supportive measures. At autopsy, microscopic examination 0 r the m yocardi urn discloses aggregates 0 [mononuclear cells an·anged around centrally located deposiLS of eosinophilic collagen. What is the appropriate diagnosis? ***(A) Acule bacterial endocarditis ***(B) Rheumatic heart disease ***(C) Subacute bacterial endocarditis ***(D) Systemic lupus erythematosus ***(E) Viral myocarditis
29 The answer is B: Rheumatic heart disease. Focal inflammato ry lesions are found in various tissues in patients with acme rheumatic fever. These inflammatory lesions are most cUstinctive wirJ1in the heart, where they are termed "Aschoff bodies." They consisL of foci of eosinophilic material surrounded by I lymphocytes, occasional plasma cells, and plump tnacrophages called h Anitschkow cells ." These distinctive cells have a bundam cytoplasm and central round-to-ovoid nuclei, in which the chromatin is disposed in a centra 1, slender, wavy ribbon (caterpillar cells). These lesions are pathognomonic for rheumatic fever and are not encountered in the other choices. ***Diagnosis: Rheumatic hean disease
30 A lO·year-old boy with a 2-week hist01y of an upper respiratory infection was admiued to the hospital with malaise, fever, joint swelling, and diffuse rash. The patient is treated and discharged However, the patienl suffers from recurrent pharyngitis and, a few years later, develops a heart murmur. This patients heart murmur is most likely caused by eArposu re to Which of the following pathogens? ***(A) Beta-hemolytic sneptococcus ***(B) Candida albicans ***(C) Epstein-Ban virus ***(D) Staphylococn4s attrws ***(E) Streptococcus vi ridans
30 The answer is A: Beta-hemolytic streptococcus. Rheumatic fever develops after antibodies to surface antigens of group A (beta-hemolyLic) sneptococd cross react with similar antigens found in the l1ean. join ts, and connective tissue of lhe skin. Cardiac lesions caused by acute rheumatic fever include endocarditis, myocarditis, and pericarditis, or all three combi ned. Chronic rheumatic endocarditis causes fib rous scaning and defmmity of cardiac valves, leading to heart murmurs and functional defects. None of the other pathogens cause rheuJnadc heart disease. ***Diagnosis: Rheumatic hean disease
3t For the patienL described in Question 30, which of the following is the most common life-threatening complication of his valvular hearr disease? ***(A) Congestive heart failure ***(B) Dissecting aneurysm ***(C) Hemolytic anemia ***(D) Myocardial in farction ***(E) Pulmonary thromboembolism
31 The answer is A: Congestive heart failure. Chrome rheumatic disease was a frequent cause of heart failure, secondary to valvular stenosis or insufficiency. Although the disease is uncommon in the industrialized world today, it remains a problem ln underdeveloped countries. None of the other choices are related to left-sided rheumatic valvulitis. ***Diagnosis: Rheumatic hean disease
32 A 34-year-old intravenous drug abuser presents to the emergency room with a 24-hour history of fever and shaking chills. His temperature i.s 38.JOC (l03°F), pulse rate 110 per minute, and blood pressure 140/80 mm Hg. Physical examination reveals a harsh systolic murmur. The. patient rapidly develops a headache and righ t-am1 paralysis. A CT scan of the brain demonstrates an infarct of the right frontal lobe. Which of the following is the most likely cause of stroke. in this patient? ***(A) Atrial myxoma ***(B) Bacterial endocarditis ***(C) Dissecting aortic aneurysm ***(D) Myocardial infarction ***(E) Paradoxical embolus
32 The an.swer is B: Bacterial endocarditis. Septicemia and cardiac murmurs in an intravenous drug abuser suggest bacterial endocarditis. Infected thromboemboli from the heart valves cause infarcts and abscesses ln various organ..c;, including the brain, kidneys, spleen, intestines, and upper and lower extremities. One third of the vlctlms of bacterial endocarditis manifest some evidence of neurological dysfunction, O\ving to the frequency of embolization to the brain. Mycotic aneurysms of cerebral vessels, brain abscesses, and intracerebral bleeding are observed. The prognosis depends, lO some extem, on the offending organism and the stage at which the infection is treated. The most common source of bacteria in intravenous dmgs users is the skin (Staphylococcus au reus). Although auial myxom.a (choice A) is theoretically correct, the condition is rare and not associated with either increased temperature or tachycardia. The other choices are not associated with septicemia. ***Diagnosis: Bacte1ial endocarditis
33 A 68-year-old woman with metastatk breast cancer develops multiple organ dysf·unction and expires. The heart at autopsy weighs 380 g (nonnal = 230 to 280 gin women). The patients myocardium (right) is compared to normal myocardium (left). These pathologic findings are mostly likely due to which of the following con ditions? ***(A) Amyloidosis ***(B) Chemotherapy ***(C) Hypertension ***(D) Inflammation ***(E) Ischemia
33 The answer is C: Hypertension. Chronic hypertension causes compensatory left vennicular hype rtrophy. The overall weight of the heart increases, exceeding 375 g in men and 350 g in women. Microscopically. h ypertroph ic m)rocardial cells exhibit an increased diameter \:vith enlarged, hyperchromatic, rectangular ("boxcar') nuclei (see photomicrograph). Myocardial hypertrophy adds m the ability of the hean to handle an increased workload. However, there is a limit beyond which additional hypenrophy is no longer compensatory. This upper limit to useful hypertrophy may reflect the increasing diffusion distance between the interstitium and the cemer of each myofiber; if the distance becomes too great, the su pply of oxygen w the myofiber will be deficient. lnterstitial fibrosis cypica11y develops as pan of the hypertrophic response, causing left vcnnicu]ar stiffness. Hypertension also increases the severity of atherosclerosis. The combination of increased cardiac workload (systolic dysfunction) , diastolic dysfunction, and narrowed coromuy arteries leads to a greater 1isk of myocardial ischemia, infarction, and heart failure. None of the other choices feature boxcar nuclei. ***Diagnosis: Hypertensive heart disease, boxcar nuclei
34 A 45-year-old woman presents with sudden attacks of wh eezing, shonness of breath , episodic hot flashes, abdominal cramps, and diarrhea. Physical examination shows facial redness, as wel1 as hepatomegaly and pitting edema of the lower legs. A 24-hour urine specimen reveals elevated levels of 5-hydroxyindoleacetic acid (5-li lAA). ACT scan of the abdomen demonstrates multiple 2- to 3-cm nodules throughoUL the liver and a 2-cm nodule in the jejunum. An echocardiogram would be mos t expected to demonstrate which of the following? ***(A) Aortic ste11osis ***(B) Bacterial endocarditis (C} Mitral valve prolapse ***(D) Nonbacterialthrombodc endocarditis ***(E) Pulmonic stenosis
34 The answer is E: Pulmonic stenosis. Carcinoid syndrome reOects the release of active tumor products and features diarrhea, flush ing, bronchospasm, and skin lesions. Carcinoid heart disease typical1y affects the right heart, causing changes in the pulmonary and uicuspid valves. It is associaled with intestinal carcinoids that have metastasized to the liver. The cardiac lesions consist of plaque-like deposits of dense fibrous tissue on the uicuspid and pulmonary valves, producing tricuspid insufficiency and pulmonic stenos1s. Elevated levels of HlAAA. a metabolite of serotonin, are diagnostic of carcinmd syndrome. The mher d 1olces are not complications of carcinoid syndrome. ***Diagnosis: Carcinoid heart disease
35 A 40-year-old woman with a history of rheumatic fever presents with shortness of breath, weight loss, fatigue, and abdominal distension. Physical examinati.on shows rales in the lungs, hepatosplenomegaly, and 2+ p1uing edema of the legs A chest X-ray reveals only left atrial enlargement and pulmonary edema. \¥hat is the most likely cause of pulmonary edema in this patient? ***(A) Aonic insufficiency ***(B) Aonic stenosis ***(C) Mitral stenosis ***(D) Pulmonic stenosis ***(E) Tricuspid insuffidency
35 The answer is C: Mitral stenosis. The mmal valve is the most commonly and severely affected valve in chronic rheumatic disease. The mitral valve snaps shut under systolic pressure and thus bears the greatest mechanical burden of all cardiac valves. Chronic rheumatic valvulitis is characterized by irregular thickening and calcification of the leaflets, with fusion of the commissures and chordae tendineae. Evemually, the valve mifice becomes reduced to a "fish mouth" appearance with a narrow orifice. The pressure in the left auium rises and is transmiu.ed via the pulmonary veins to the pulmonary vasculature. In cases of aortic insufficiency (choice A) or stenosis (choice B), the left atrium is in itially protected by closure of the mitral valve. The other choices are not associated with auial enJargemem or pubnonary edema. ***Diagnosis: Mitral stenosis, rheumatic heart disease
36 A 55-year-old man with idiopathic dilated cardiomyopathy receives a heart uansplam. Which of the [allowing is the most likely cause o[ death in this patient 2 years after transplantation? ***(A) Acute cellular graft rejection ***(B) Aortic valve stenosis ***(C) Chronic vascular rejection ***(D) Hyperacute graft rejection ***(E) Pulmonary fibrosis
36 The answer is C: Chronic vascular rejection. Chronic vascular rejection, also referred to as accelerated coronary anery disease, is the most common cause of death in heart transplant patients after tJ1e first year of transplantation. h usually affects the proximal and distal coronary arwies and their penetrating branches. Microscopically, the disorder is characterized by concentric intimal proliferation, leading to occlusion and myocardial infarction. This complication is painless because the tran.splamed heart is denervated. Acute cellular graft rejection (choice A) occurs dunng the first few months afte r transplantation, and hyperacute graft rejection (choice D) occurs quickly after n·ansplamation. ***Diagnosis: Coronary anery disease
37 A 19-year-old college basketball player suddenly collapses on the court. A cardiac monitor shows ventricular tachycardia and then vemricular fibrillation. He is successfully resuscitated and hospitallzed. The patient's history indicates that his father died suddenly at 38 years of age. The boy's echocard!Ogram reveals a thickened left ventricular wall, with a small slit-like chamber. The interventricular septum is also markedly thickened . Five yea rs later the patiem expires suddenly. Hiswlogic examination of the hean muscle at auwpsy is shown in the image. 'vVhkh of the following is the most likely diagnosis? ***(A) Amyloidosis ***(B) Cardiac myxoma ***(C) Dilated cardiomyopathy ***(D) Hypertrophic cardiomyopathy ***(E) Toxic cardiomyopathy
37 The answer is D: Hypertrophic cardiomyopathy. Hypenrophic cardiomyopathy (HCM) refers to a condition in which cardiac hypertrophy is om of proponion to the hemodynamic load placed on the hean. This disorder is an auLOsomal dominant rrait in about half of the patienLS, usually involving mutauons of contractile proteins. The walJ of the left vemricle is thtckened (hypertrophic), and its cavity is small. The most notable histologic charaneristic of this disorder is myofiber disanay, which is most extensive in the imervenuicular septum. Despite an absence of symptoms, persons with HCM are at risk of sudden death, particularly dming vigorous exercise. The other ch oices do not demonstrate this distinctive histopathology. ***Diagnosis: Cardiomyopathy, hypertrophic
38 A 58-year-old man presems with dyspnea on exertion and fatigue. Physical examination shows evidence of congestive hean failure and echocardiography discloses a dilated left ventricle and a left ventricular ejection fraction of 20%. The liver appears fatty by MRL Cardiac catheterization demonstrates minimal coronary artery atherosclerosis. Which of the following is the most likely cause of these signs and sympwms7 ***(A) Alcoholic cardiomyopathy ***(B) Aonic stenosis ***(C) Amytoidosis of the hean ***(D) Hypertrophic cardiomyopathy ***(E) Viral myocarditis
38 The answer is A: Alcoholic cardiomyopathy. Alcoholic cardiomyopathy is the most common identifiable cause of dllated cardiomyopathy in the United States and Europe. The mechanism by which alcohol injures the heart remains unclear, but the degree of myocardial damage has been con-elated with the total lifetime dose of ethanol. Abstinence ameliorates or even reverses the early stages of alcoholic cardiomyopathy, but this is nOL true in late-stage disease. Viral myocarditis (choice E) might also presem in this manner, but it is much less common than alcoholic cardiomyopath y. ***Diagnosis: Cardiomyopathy, alcoholic
39 A 60-year-old man presents with increasing girth and fatigue. Physical examination reveals peripheral edema, ascites , and hepatomegaly. Uver function tests are normaL An echocardiogram shows a remarkably enlarged right hean and no signs of valvular hean disease. Endomyocardial biopsy chsdoses interstitial, pink amorphous deposits belween cardiac myocytes. Which of the following is the appropriate diagrtosis? ***(A) Carcinoid heart disease ***(B) Cardiac amyloidosis ***(C) Dilated ca rdiomyopathy ***(D) Hypertrophic cardiomyopathy ***(E) Rheumatic hean disease
39 The an;swer is 8: Cardiac amyloidosis. Cardiac amyloidosis causes restrictive cardiomyopathy, which is characterized by reduced diastolic fiJJing and righL-sided hean failure Infiltration of the conduction system can result in arrhythmias or sudden cardiac death_ Amyloid appears amorphous, glass~~ and eosinophilic_ It is readily documemed using a Congo red stain, which demonstrates red-green birefringence when viewed under polarized light. None of the other choices exhibit such am011')hous, eosinophilic exuacellular deposits_ ***Diagnosis: Cardiac amyloidosis
40 A 50-year-old man underwent hean transplantation for low· output hean failure that was unresponsive to medical treatment. The affected heart at autopsy is shown in the image. It weighs 950 g (normal up to 350 g) and shows no evidence of coronary artery atherosclerosis. Histologically, the myocardium demonstrates hypertrophic myocytes and foci of myocardial fibrosis but no evidence of inflammation or myofiber disarray. Which of the following is the most likely diagnosis? ***(A) Cardiac amyloidosis ***(B) Dilated ca rdiomyopathy ***(C) Hypemophic cardiomyopathy ***(D) Restrictive cardiomyopathy ***(E) Ventricular aneurysm
40 The answer is 8: Dilated cardiomyopathy. Dilated cardiomyopathy (DCM) is the most common type of cardiomyopathy and is characterized by bivemricular dilation, impaired contractillty, and evenrually congestive heart failure In the majority of cases, the cause is unknown (idiopathic DCM). although genetic [actors often play a role . The disorder can also develop in response LO a large number of insults that may directly affect the myocardium (secondary DCM), including alcohol , cytotoxic drugs. and viral infections. The heart is typically enlarged, often to remarkable propottions ("cor bovinum"). Hypemophic cardiomyopathy (choice C) displa}'S myofiber disanay. The other choices do not lead lO bivenuicular dila· don and hypertrophy. ***Diagnosis: Cardiomyopathy, dilated
41 A 40-year-old woman p resents with dyspnea, heart palpitations, and pitting edema. She was seen for flu-1ike symptoms and pron)jnem muscle pain 3 weeks ago. Physical examination shows tachycaTdia and irregular he an beats. A chest X-ray reveals cardiomegaly and pulmonary edema. The patient subsequently dies of cardiorespiratory failure. Histopathology of the heart muscle at autopsy is shown Ln the image. What is the appropriate diagnosis? ***(A) Acme bacte1ial endocardjtis ***(B) Acute myocardial infarction ***(C) Endocardial fibroelastosis ***(D) Rheumatic heart disease ***(E) Viral myocarditis
41 The answer is E: Viral myocarditis. The pathogenesis of viral myocardjtis is believed w involve direct viral cytoto>dcity, as well as cell-mediated immune reactions directed against virally infected myocytes. The hearts of patients with myocarditis who develop congestive heart failure typically show biventricular dilation and hypokinesis. Histologic changes in myocarditis are nonspecific, but most cases show a patchy or diffuse interstitial mon.onuclear inOammatory infiltrate, composed primarily of T lymphocytes and macrophages (as seen in this case) Coxsackievirus type B is one of me most common pathogens associated with myocarditis_ ***Diagnosis: Viral myocarditis
42 A 50-year-old woman presents with fatigue and shortness of breath. Physical examination shows clinical evidence of pulmonary edema, enlargement of the left atrium, and calcification of the mitral valve. A CT scan demonstrates a large mass in the left atrium. Before open hean surgery can be performed, the patient expires of an ischemic stroke. The hearL at autopsy is shown in Lhe image. Which of the following is the most likely diagnosis? ***(A) Calcine aortic stenosis ***(B) Cardiac myxoma ***(C) Fibroelastoma ***(D) Metastanc melanoma ***(E) Mural thrombus
42 The answer is B: Cardiac myxoma. Cardiac myxoma is the most common primary tumor of the heart. Most myxomas arise in the left atrium, but they can occur in any chamber or valve. Unlike a mural thrombus (choice E), these tumors appear as a glistening, gelatinous polypoid mass, usuaUy 5 to 6cm in diameter. Microscopically, cardiac myxoma has a loose myxoid stroma, containing abundant prmeoglycans. More than half the patien ts have clinical evidence of mitral valve dysfunction. Although the tumor does not metastasize, it may embolize. ***Diagnosis: Cardiac myxoma
43 A 45-year-old male immigrant from Haiti presenrs \l,.•ith a 3-week histmy of difficulty breathing, chest pain, and abdominal discomfon. Physical examination shows hepatomegaly, ascites, and 2+ pitting edema of the legs. The patient was diagnosed with active pulmonary tuberculosis 2 years before. Cardiac auscultation reveals a pe1icard ial friction mb and respiraw ry crackles in at the bases of both lungs. An X-ray film of the chest shows an enlarged cardiac silhouene withom visible borders. The patient eventually dies and the hean at autopsy is shown in the image. What is the most likely underlying cause of death in this patient? ***(A) Consrrictive peticarditis ***(B) Infective endocarditis ***(C) Resnictive cardiomyopathy ***(D) RheumatLc hean disease ***(E) Viral myocarditis
43 The answer is A: Constrictive pericarditis. Constrictive peticarditis is a chronic fibrosing disease of the pericardium that compresses the heart and restricts inflow. It results from an e>mberam healing response following acute pericardia} injury in which the pericardial space becomes obliterated and the visceral and parietal layers are fused in a dense mass of fibrous tissue. Pericarditis may be caused by bacter ia, viruses, or fungi. Active ruberculosis (as in this case) is a major cause of this condition in underdeveloped cou ntries. Previous radiation therapy to the mediastinum and cardiac surgery account for one third of the cases, whereas, in the other cases, consu·ictive pericarditis evolves from chronic mfection. The other choices are not peti cardial diseases, as illustrated in the phomgraph shmvn. ***Diagnosis: Constrictive pericarditis
44 A 43-year-old woman with Marfan syndrome suffers a dissecting aneurysm of the aortic arch and expires_ An abnormal n'Uu-al valve is noted at autopsy (shO\vn in the image)_ Patients with this valve disorder are at greater risk for developing which of the following conditions? ***(A) Cardiac Lamponade ***(B) Cerebral embohsm ***(C) Dressler syndrome ***(D) Libman-Sacks en docardiLis ***(E) Ventricular aneurysm
44 The correct answer is 8: Cerebral embolism. The hean displays mitral valve prolapse (MVP), a condition in which the mirral valve leaflets become enlarged and redundant. The chordae tendin eae are thi nned and elongated, and the billowed leaners prolapse imo the left auium during systole. Valves from patients with MVP exhibit a striking accumulation of myxomatous connective tissue. Many cases are transmitted as an autosomal dominant trait, related to an uru dentified defect in extracellular matrix metabolism. Most (>90%) of patienLs with Marfan syndrome have clinical evidence of MVP Although most patients with MVP are asymptomatic, they are at higher risk for developing complications, including bacterial endocarditis, a condition in which vegetations on the valve can break off and embollze to tl1e brain. Ubman-Sacks endocarditts (choice D) occurs in patients with systelnic lupus erythematosus. ***Diagnosis: Mitral valve prolapse
45 A 40-year-old man with a histmy of intravenous drug abuse develops rapidly progressive tight-sided heart failure. These symptoms are most likely due to which of the following condi!:ions7 ***(A) Aortic insufficiency ***(B) Mitral regurgitation ***(C) Ruptured chordae tendineae ***(D) T1icuspid insufficiency ***(E) Tlicuspid stenosis
45 The answer is D: Tricuspid insufficiency. Tricuspid ins uffi ciency secondary to bacterial endocard itis is one of the most common complications of intravenoLLS drug abuse. Intravenous drug abusers inject pathogenic organisms along with their illicit dtugs. h1 such patients, 80% have no underlying cardiac lesion, and the tricuspid valve is infected in half of cases. Septic pulmonary emboh characterize uicuspid valve endocarditis in drug addicts. Despite amiblonc therapy, a third of cases of endocarditis caused by 5. at1reus are fatal Other risk factors for bacter ial endocarditis include aging, diabetes, pregnancy, tTansiem bacreremta, and prosthetic valves. The most common predisposing condition for bacterial endocarditis in children is congenital heart disease. Choices A, B, and C primatily affect the lefL ventricle. Tricuspid sten osis (choice E) is distinctly uncommon and does not occur rapidly. ***Diagnosis: Bacterial endocarditis
46 A 53-year-old woman presents with a 6-week history of fever, fatigue, and weight loss. Her temperature is 38.7"C (l03"F), pulse rate 110 per minme, and blood pressure 140/SOmm Hg. Physical examination reveals pe techiae and clubbing of the fingers. The patient develops mental status changes, suffers a massive stroke, and expires. The mitral valve is examined at autopsy (show11 in the image). Which of the following is the appropriate pathologic diagnosis? ***(A) Bactetial endocarditis ***(B) Carcinoid heart disease ***(C) Ubman-$acks endoca rditis ***(D) Marantic endocarditis ***(E) Mitral valve prolapse
46 The answer is A: Bacterial endocarditis. The mitral valve in this case shows destructive vegetations that have eroded through the fTee margms of the valve leaflets. Vegetations in bacterial endocardi tis fom1 on the atrial side of the atrioventricular valves and the vennkular side of the semilunar valves, often at points of closure of the leaflets or cusps. They are composed of platelets, fib tin, cell debris, and masses of organisms. The underlying valve tissue may become so damaged that the leaflet perforates, causing regurgitation. The disease begins with nonspecific symptoms of low-grade fever, fatigue, anorexia, and weight loss. Heart murmurs develop almost invariably. ln cases of more than 6 weeks duration, splenomegaly, petechiae, and clubbing of the finge rs are frequem. ln a third of patients, systemic emboli (e .g., petechiae) are recognized at some time during the illness. More than half of adults \'lith bacterial endocarditis have no predisposing cardiac lesion. Mitral valve prolapse and congenital heart disease are today the most frequent bases for bacterial endocarditis in adults. Carcinoid heart disease affects the right side of the hean and produces tricuspid regurgitation and pulmonary stenosis. The other choices do not cause desmtctive aortic valve lesions. ***Diagnosis: BaCLerial endocarditis
47 A 72-year-old woman rues o [ congestive heart [ailure. The aortic valve is examined at autopsy (shown in the image). Which of the following is the most likely cause of aortic stenosis in this patient? ***(A) Acme bacterial endocardilis ***(B) Aging-related degene raLion ***(C) Chro11ic rheumalic valvulitis ***(D) Congenical bicuspld valve ***(E) Hypertrophic cardiomyopathy
47 The answer is C: Chronic rheumatic valvulitis. Recurrem episodes of rheurmatic fever result in progressive damage to the mitral and aonic valves. In this case, the aonic valve shows severe aonic stenosis. Three sinuses of Valsalva are recognizable, but the valve cusps are rigidly fibrotic and calcified, and there is eX£ensive fusion of the commissures. Chronic rheumatic valvulitis has narrowed the orifice into a fixed sliLlike configuration that does not change dming the cardiac cycle. None of the other choices displa)' such fusion of the conunissures. ***Diagnosis: Rheumatic heart disease, aortic stenosis
1 A 63 -year~old man witl1 smaiJ cell carcinoma of tl1e left mainstem bronchus begins chemotherapy. During the treatmenL pe1iod, he becomes febrile and develops a produClive cough. The temperature is 38.rc (103°F), respirations are 32 per minute, and blood pressure is 125/85 mm Hg. A CBC shows leukocytosis (WBC"' 18,500/~ L). The patient's cough worsens, and he begins expectorating large amounts of foulsmelling sputum. A chesL X-ray shows a distinct cavity with an air/fluid level distal to the mmor area. Which of the follo\ving is the most likely diagnosis? ***(A) Ate ]ectasis ***(B) Bronchiectasis ***(C) Ghon complex ***(D) Lobar pneumonia ***(E) Pulmonary abscess
1 The answer is E: Pulmonary abscess. Lung abscess is a localized accumulation of pus accompanied by the destruction of pulmonary parenchyma, including alveoli, airways, and blood vessels. The most common cause of pulmonary abscess is aspiration, often in the seuing of depressed consciousness. Pulmonary abscess is also a common complication of lung cancer. A cystic abscess contains p urulem exudates and is contained by a fibrous walL Abscess cavities are often partially filled with pus and air, which accow1ts for the X-ray finding of an "air/ fluid leveL" Jnfiammation is usually present in the surrounding pulmonary parenchyma. Foul-smelling sputum may be expecLOrated Han abscess is connected to a bronchus. None of Lhe other choices presem as cavitary lesions. ***Diagnosis: Pulmonary abscess, small cell care:i noma of lung
2 A 64-year-old man presems with fever, chills, and increasing shortness of breaLh. The patient appears in acute respiratory distress and complains of pleuritic chest pain. Physical examination shows crackles and decreased breath sounds over both lung fields. The patient exhibits tachypnea, with Garing of the nares. The sputum is rusty-yellow and displays numerous netmophils and erythrocytes. Which of Lhe following padlogens is the mosL common cause of this patients pulmonaty infection? ***(A) Legionellcr pneumophlla ***(B) Mycoplasma pneumoniae ***(C) Pseudomonas aeruginosa ***(D) Staphylococcus au reus ***(E) Screptococws pneumoniae
2The answer is E: Streptococcus pneumoniae. All of the choices cause pneumonia. However, despite the impact of an tibiotic therapy, pneumonia caused by StreptOcocms pneumoniae (pneumococcus) remains the most significant problem. The onset of pneumococcal pneumonia is acute with fever and chms. ***Diagnosis: Pneumonia, Streptococcus pnntmoniae
3 1f the patient described in Question 2 is approp1i ately treal.ed with andbiotics, which of the fo11owing ls the most likely outcome? ***(A) Abscess formation ***(B) Bronchopleural fistula ***(C) Bullous emphysema ***(D) Resolution ***(E) Scar fonnation
3 The answer is D: Resolution. Although abscesses and fistulas may occur (choices A and B), the most common omcome of acu te bacterial pneumonia is resolution, particularly \\litl1 appropriate antibiotic treatment. ***Diagnosis: Pneumonia, Streptococcus pneurnoniae
4 A 40-year-old alcoholic man is admined to the hospiLal in severe respiramry distress. TI1e temperature is 38.7°C (103°F), respirations are 32 per mrnme, and blood pressure is 130/90mmHg. He coughs constamly and expecwrates currant-jelly spmum. A chest X-ray reveals bilateral diffuse pulmonary consolidation. Physical examination shows bilateral crackles, dullness to pe rcussion over both pulmona1y fields, and use of accessory muscles. The patient subsequently dies from complications of bacte1ial sepsis. The left lung ar auwpsy (shown in the image) shows a red, engorged lower lobe. What is the appropriate diagnosis? ***(A) Atypical pneumonia ***(B) Bronchopneumonia ***(C) ImersLitial pneumonia ***(D) lobar pneumonia ***(E) Pulmonary abscess
4 The answer is D: lobar pneumonia. The term lobar pneumonia refers to consohdation of an entire lobe; bronchopneumonia (choice B) signlfies scattered solid foci in the same or several lobes. Lobar pneumonia presents 'Ati th a diffuse consolidaEion of one or more pulmonary lobes. In contrast to lobar pneumonia, interstitial pneumonia (choice C) primanly involves the alveolar septa. Atypical pneumoma (choice A) is most often encounte red in mycoplasma pneumonia. PulmonaJy abscess (choice E) may be a complication of lobar pneumoma or bronchopneumonia. ***Diagnosis: Lobar pneumonia
5 A 28-year-old woman with cysLic fibrosis presents with increasing shortness of breath and production of abundant foul-smelling sputum. The sputum in this patient is most likely associated with which of the following pulmonary conditions? ***(A) Atelecrasis ***(B) Bronchiectasis ***(C) Empyema ***(D) Pneumothorax ***(E) Pyothorax
5 The answer is 8: Bronchiectasis. Bronchiectasis refers to ,___ the irreversible dilation of bronchi, which is caused by the destruction of the muscular and elastic elements of bronchial walls. Bronchiectasis is often localized to a segment of the lung distal to mechanical obstmction of a bronchus by a vmiety of lesions, including tumors, inhaled foreign bodies, mucous plugs (e.g., cystic fib rosis and asthma) , and compressive lymphadenopathy. Nonobstructive bronchiectasis is usually a complication of chronic pulmonary infections. Panems with bronchiectasis present with chronic productive cough, often with copious mucopumlent spmum. Pymhorax (choice E) is infecnon of pleural effusion. ***Diagnosis: Bronchiectasis, cystic fibrosis
6 A 60-year-old alcoholic woman presents to the emergency room with fever, chills, and shortness of breath. The spmum is rusty-yellow and comains numerous neutrophils, red blood cells, and Gram-positive cocci. A chest X-ray shows diffuse haziness over both lungs. One week following admission, the patient develops empyema. TI1is pulmonary condition is associated with the spread of bacteriaJ infection to whkh of the folLO\oving anatomic locations? ***(A) Blood ***(B) Bronchi ***(C) Interstitial space ***(D) Pericardium ***(E) PleuraJ space
6 The answer is E: Pleural space. Complications of bacterial pneumonia include pleuritis (extension of inn.ammation to the pleural surface), pleural effusion, pyothorax (infection of pleural effusion), pulmonary abscess , and pulmonary fibrosis. Empyema is a loculated collection of pus with fibrous walls that follows the spread of bacteri al infection to the pleural space. All of the other choices are possible routes of spread, but do not describe empyema. ***Diagnosis: Bacterial pneumonia, empyema
' 7 A 10-year-old boy suffers head trauma and lies unconscious for 2 weeks. He is now intubated. His temperature rises to 38. 7°C (l0}°F), and m.')'genation becomes more difficult. A chest X-ray reveals a pleu ral effusion and multiple abscesses in the lung parenchyma. Which of the following microorganisms is the most likely cause of this pulmonary infection? ***(A) Legionclla pneumophila ***(B) Mycoplasma pneumoniae ***(C) Pncurnocyscis calinii ***(D) Scaphylococcus aureus ***(E) Streptococcus pneumoniae
,__7__ The answer is D: Staphylococcus aureus. Staphylococcal pneumonia is an uncommon community-acquired disease, accounting for only 1% of bacwial pneumonias. However, pulmonary infection with Scaphy lococws aureus is common as a superinfection after inl1uenza and other viral respiratory rract infections. Nosocomial (hospital acquired) staphylococcal pneumonia typically occurs in chronically ill patients who are prone to aspiration or who are intubated. Although lung abscess can conceivably follow ru1y Tespiratory infection, the other choices do not usually do so. ***Diagnosis: Bacte1ial pneumonia
8 A 22-year-old man \vith AIDS complains of persistent cough, night sweats, Low-grade fever, and general malaise. A chest X-ray reveals an area of consolidation in the periphery of the left upper lob e, as well as hilar lymphadenopathy. Sputum cultures show acid-fast bacilli. Whlch of the follmving is the most likely diagnosis? ***(A) Bronchopneumonia ***(B) Pulmonary abscess ***(C) Sarcoidosis ***(D) Tuberculosis ***(E) vVegener granulomatosis
8 The answer is D: Tuberculosis. Tuberculosis represents infection with Mycob£lctenum tuberculosis, although atypical mycobacterial infections may mimic iL. The Ghon complex includes pru·enchymal consolidation and enlargement of ipsilateral hilar lymph nodes and is often accompanied by a pleural effusion. The spumm contains M. tttberculoSIS, which is acid- fast in smears stained by the Ziehl-Neelsen technique. After resolution of primary mberculosis, reemergence may occur (secondary mberculosis). None of the ot11er choices feamre acid-fast . orgamsms. ***Diagnosis: Tuberculosis, Mycobacterium tubermlosis
9 A 53-year-old man develops weakness, malaise, cough \vith bloody sputum, and night sweats. A chest X-ray reveals numerous apical densities bilaterally, some of which are cavitaty E'1)osure to Mycobacte1ium tuberculosis was documemed 20 years ago, and M. wberwlo.m is idemified in his spmum. Which of the folimving describes the expected lung pathology in this pariem? ***(A) Dense fibrosis ***(B) Eosinophilic in filrrar ion ***(C) Granulomas ***(D) lmersritial pneumonia ***(E) Plasma cell infilLration
9 The answer is C: Granulomas. Secondary (reactivation) tuber- L-- culosis is characte1ized by the formation of granulomas and extensive tissue destruction (caseous necrosis). Mycobactelia typically spread lO the apices of the lungs and produce large cavities, which are associated with hemoptysis. Miliary tuberculosis refe rs to widespread seeding of bactetia in the lungs and distant organs. Granulomatous in flammation may induce fib rosis (choice A) as a secondary feature. ***Diagnosis: Tuberculosis, Mycobacterium tLrberculom
10 A 3-day-old girl shows signs of cyanosis and respirawry distress. Her temperature is 38. 7°C {103°F), pulse rate is 140 per minute, respirations are 60 per minute, and blood pressure is 90/58 mm Hg. Laboratory studies indicate that the baby lS positive for HIV The infant does nm respond to conventional antibiotic therapy and expires. Histologic examination of the lungs at autopsy is shown in tJ1e image. The alveolar cells are very large and display single basophilic nuclear inclusions, with a peripheral halo and multiple cytoplasmic basophilic inclusions. Which of the following is the most likely etiologic agem in this child's pulmonary infection? ***(A) Adenovirus ***(B) Cytomegalovi rus (OvlV) ***(C) Herpesvi rus ***(D) Pneumocysns catinu ***(E) Rubcllavmts
10 The answer is B: Cytomegalovirus (CMY) . CMV produces a characteristic imerstitial pneumonia. Initially described in infants, it is now well recognized in immunocompromised persons. The virus may be transmitted from mother to child transplacentally (a feamre of TORCH syndrome). Although infected children are usually asympwrnatk, in symptOmatic infants and chHdren, cenmll nervous symptoms predominate. As implied by its name, CMV causes marked enlargement of infected cells, which contain typical in tranuclear and often cymplasmic inclusions. The other TORCH agents may cause pneumonia, but only CMV exhibits Lhis cellular morphology ***Diagnosis: Cytomegalovirus, viral pneumonia
11 A 56-year-old woman with disseminated breast cancer undergoes mullidrug chemotherapy. Ten days later, she deve lops cough and a feve r of 38.7°C (103°F). A chest X-ray shows multiple areas of consolidauon and a large cavity in the nght upper lobe. Multiple pulmonary miarcts are also 1denufied. The patient subsequeml)• d1es of mulnsystem organ fat lure. HtsLOlogic exammauon of the lungs at amopsy IS shown 111 the Image. Whtch of the folJowmg is me most hkely d~agnos1s? . -~ - ........ _- ***(A) Actinomycosis ***(B) Blastomycosis ***(C) Coccidioidomycosis ***(D) Cryptococcosis ***(E) Invasive aspergillosis
11 The answer is E: Invasive aspergillosis. Invasive aspergillosis is d1e most serious manifestation of Aspergillus infection, occun ing almost exclusively as an opponunistic infection in lmmunocompromised persons (e.g. , undergoing cytotoxic therapy or diagnosed with AIDS). Invasion of blood vessels and tissue infarcts are common. Aspe1gillus species may also grow in preexisting cavities caused by tuberculosis or bronchiectasis. They proliferate to form fungus balls, which are also rdened to as asperglllomas or mycetomas. Vascular invasion is not a feaLUre of the other choices. ***Diagnosis: Pulmonary aspergillosts
12 A 40-year-old woman with leukemia is treated with chemotherapy. During treaunent she develops increasing cough and shormess of breath. A chest X-ray shows djffuse lung infiltrates. Sputum cultures are negative, and the patiem does not respond to routine antibiotic therapy. An open lung biopsy is diagnosed by the pathologist as viral pneumonia. Which of the following histopathologic findings would be expected in the lungs of this padem? ***(A) Clusters of epithelioid macrophages ***(B) ConOuent areas of caseous necrosis ***(C) Fibrous scaning of lung par-enchyma ***(D) Hyaline membranes and interstitial inflammation ***(E) Sheets of bacilli-filled macrophages
12 The answer is D: HyaUne membranes and interstitial inflammation. Viral infections of the pulmonaty parenchyma produce diffuse alveolar damage (DAD) and imerstidal pneumonia. Necrosis of type I pneumocytes and the forma tion of hyaline membranes result in an appearance that is indistinguishable from DAD in other settings. Choices A, B. and Dare not characteristic of interstitial pneumonia. Choice C (fibrous scarring) may be a late complication of some fom1s of this disorder. ***Diagnosis: Di iTuse alveolar damage, viral pneumonia
13 A 38-year-old woman, who is being treated with corticosteroids for systemic lupus erythemawsus, presents \vith chronic nonproduCLive cough. She breeds pigeons for aviru1 hobbyists. A chest X- ray reveals a 2-cm nodule in the upper lobe of the Iight lung. The lung nodule is resected. Hiswlogic examination reveals granulomas and budding yeast fonns, which stain positively for polysaccharides (mucicarmine smin, shown in the image). What is the appropriate diagnosis? ***(A) Actinomycosis ***(B) Coccidioidomycosis ***(C) Crypwcoccosis ***(D) Histoplasmosis ***(E) Mycoplasma pneumonia
13 The answer is C: Cryptococcosis. CrypLOcoccosis results from U"le inhalation of spores of Cryptococcus neoformans, an organism frequently encountered in pigeon droppings. Most serious cases occur in immunocompromised persons. Other examples of fungal infecdons of the lungs are histoplasmosis (ch oice D), coccidioidomycosis (choice B), and aspergillosis. However, crypwcoccus s tains positively wid"l a mucicarmine stain for capsular polysacchatides. These diseases are also acquired by inhaling spores. ***Diagnosis: Cryptococcal pneumonia, pigeon breeder lung disease
14 A 36-year-old man -,vith AIDS presents with fever, dry cough, and dyspnea. A chesL X-ray shows bilateral and diffuse infiltrates. Laboratory studies reveal a CD4+ cell coum of less than 50/).LL A lung biopsy discloses a chronic interstitial pneumoniLis and an inrra-alveolar foamy e>..'Udate. A silver stain of a bronchoalveolar lavage is shown in the image. Which of the following organisms is the most likely pathogen responsible for these pulmonary findings? ***(A) Cryptococcus 11eojorma11S ***(B) Cytomegalovi rus ***(C) Histoplasma capsulatum ***(D) Mycop lasma pneumoniae ***(E) Pneumocystis jirovw
14 The answer is E: Pneumocystis jirovecl. P. jrroved (fo rmerly P. ca1inii) is the most frequent cause of infectious pneumonia in patients with AIDS. Once considered a protozoan, the organism has been reclassified as a fungus. The classic lesion of Pneumocystts pneumonia comprises an interstitial infiltrate of plasma cells and lymphocytes, diffuse alveolar damage, and hyperplasia of type II pneumocytes. The alveoli are filled \vith a characteristic foamy exudate. The organisms appear as small bubbles in a background of proteinaceous exudates. ln this case, a centrifuged bronchoalveolar lavage specimen impregnated with silver shows a duster of cysts. The cysts appear as round or indented ("crescent moon") bodies, which are approximately 5 J.lm in diameter. Clyptococcus neoformans (choice A) and Histoplasma capsulatum (choice C) do not typically cause interstitial pneumonia. ***Diagnosis: Pneumocystis pneumonia, AIDS
15 A 48-year-old man with AIDS is admixted to the hospital with a fever of 38.7"C (l03°f). The pariem has a 2-week hiswry of persistent cough and diarrhea. Laboratory studies show that the CD4• cell count is less than 500/~ll. A sputum culture reveals acid-fast organisms, which are funher identified as MycobacteriLtm avrum-mtracelltdare . This patients pneumonia is characterized by extensive pulmonary infiltrates of \Vi1ich of the rollowing cell types? ***(A) CD 4~ helper T cells ***(B) Eosinophils ***(C) Macrophages ***(D) Mast cells ***(E) Neutrophils
15 The answer is C: Macrophages. Mycobacce1ium aviumintracdlulare (MAl) complex is a progressive systemic disorder, often occurring in patients who have AIDS. One third of all pa tients v.rith AIDS develop oven MAl infections because depletion of CD4+ helper T cells cripples the immune response. This pneumonia is characterized by an extensive infiltrate of macrophages. The proliferation of MAl and the recruitment of macrophages produce e>.rpanding lesions, ranging from epithelioid granulomas containing few organisms w loose aggregates of foamy macrophages. Symptoms associated with MAl resemble those of tuberculosis. The other infiammawry cells listed are not characteristic of infection with MAL ***Diagnosis: Mycobacwium avtLtm-imracellulare, AIDS
16 A 65-year-old man who is a heavy smoker complains of sudden onset of malaise, fever, productive cough, abdominal paiJ1, and muscle aches. A cl1est X-ray shows bilateral, diffuse , patchy. alveolar infi ltrates. Laboratory studies reveal that the patient is infected with Lcgwnella pneumophlla. This patients pneumonia is characterized by extensive pulmonary infihrates of which of the following cell types? ***(A) CD4- helper T cells ***(B) CD8• 1<iller T cells ***(C) Eosinophils ***(D) Macrophages ***(E) Mast cells
16 The answer is D: Macrophages. Legwnella pneumonia begins when microorganisms enter the alveoli, where Lhey are phagocytozed by macrophages. Bacteria multiply within macrophages and are released to infect new macrophages. Smoking, alcoholism, and chronic pulmonary diseases interfere with nom1al host defenses thereby increasing the risk of developing Legionella pneumonia. Patients typically present with an acute bronchopneumonia. One third of cases of Legionella pneumonia are complicated by subsequem emphysema. l11.e other inflammatory cells listed are scarce or absent in the alveolar exudate. ***Diagnosis: Legionnaire disease
17 A 16-year-old boy is rushed tO the emergency room after sustaining a stab wound to the chest during a figh t. Physical examination reveals a 1-cm emry wound at the right 5th intercostal space in the midclavi.cular tine. His temperature is 37"C (98.6°F), respirations are 35 per minute, and blood pressure is 90/50 mm Hg A chest X-ray shows air in the right pleural sp ace. vVJ1ich of the following pulmonary conditions is the expected complication of pneumothorax arising in this patient? ***(A) Atelectasis ***(B) Chylothora-x ***(C) Diffuse alveolar damage ***(D) Empyema ***(E) Pyothorax
17 The answer is A: Atelectasis. Pneumothorax, wluch is defined as the presence of air in the pleural cavity, may be due tO traumatic perforation of the pleura or may he spontaneous. Traumatic causes include penetrating wounds of the chest wall (e.g., stab wound or a 1ib fracnue). Traumatic pneumothora.'< is most commonly iatrogenic and is seen after aspiration of Ouid from the pleura (thoracemesis). pleural or hmg biopsies, transbronchial biopsies, and positive pressure-assisted vemllation. Pneumothorax causes collapse of a previously expanded lung, a condition that is termed aleleclasis. Additional causes of atelectasis include deficiency of surfactam, compression of the lungs , and bronchial obstruction. Chylothorax (choice B) is the accumulation oflymphatic fluid within the pleural space and is a rare complication of trauma. ***Diagnosis: Atelectasis
18 A 62-year-old woman is rushed lO the emergency room following an automobile accident. She has suffered internal lnjuries and massive bleeding and appears to be in a state of profound shock. Her temperature is 3JCC (98.6°f), respirations are 42 per minule, and blood pressure is 80/40mm Hg. Physical examination shows cyanosis and the use of accessory respiratory muscles. A CT scan of the chesL is nonnal on arrival. Her condition is complicated by fever, leukocytosis, and a positive blood culture for staphylococci (sepsis). T'.:vo days later, the patiem develops rapidly progressive respiratory distress, and a panern of "interstitial pneumonia~ can be seen on a chest X-ray. Which of the foUowing is the most likely diagnosis? ***(A) Aclue b ronchiolitis ***(B) Alveolar proteinosis ***(C) Atelectasis ***(D) Desquamative imerstilial pneumonitis ***(E) Diffuse alveolar damage
18 The answer is E: Diffuse alve~lar damage {DAD). DAD refers LO a nonspecific pattem of reaction to injury of alveolar epithelial and endothelial cells fTDm a variety of acute insulLS. TI1e clinical counterpart of severe DAD is acme respiratOry distress syndrome. ln chis disorder. a patient with apparemly normal lungs sustains pulmonary damage and then develops rapid progressive respirawry failure. DAD is a final common pathway of pathologic changes caused by a variety of insults, including respiratmy infections, sepsis, shock, aspiration of gastric contents, inhalation of tmdc gases, near-drowning, radiation pneumonitis, and a large assortment of dmgs and other chemicals. Desquamative imerstirial pneumonia (choice D) is a chronic, fibrosing, interstitial pneumonitis of unknown etiology ***Diagnosis: Diffuse alveolar damage
19 A 64-year-old woman who has suffered shock and sepsis experiences declirung respiratory func tion and is placed on a ventilator. Despite intensive therapy, the patiem dies 3 weeks later in respiratory failure. Histologic examination of the lungs at autopsy is shown in the image. Which of the following best describes the pathologic findings in rhis amopsy specimen' ***(A) Alelectasis ***(B) Bronchiectasis ***(C) Bronchopneumonia ***(D) Lobar pneumonia ***(E) Pulmonary fibrosis
19 The answer is E: Pulmonary fibrosis. The photomicrograph shows hyaline membranes, thickening of the alveolar walls, and loose connective dssue. This organizing phase of diffuse alveolar damage begins aboUL 1 week after the initial injury. This phase is marked by the proliferation of fibroblasts within the alveolar walls. Alveolar macrophages digest the remnanLS of hyaline membranes and other cellular deb tis. Loose fibrosis then thickens the alveolar septa. This fibrosis resolves in mild cases, but in severe cases, it may progress to restrucmring of Lhe pulmonary parenchyma and cyst formation. The photograph does not display features of the Olher choices. ***Diagnosis: Diffuse alveolar damage
20 A 22-year-old man who is being trea£ed for leukemia complains of shortness of breath on exertion, plemitic chest pain, and a low-grade fever. Physical examination reveals crackles m both lung bases and clubbing of the fingers. Bronchoalveolar lavage demonstrates PAS-positive material and elevated levels of surfactant proteins. An open-lung biopsy tS shown m the image. Whtch of the following is the most likcl}· dtagnosts? ***(A) Alveolar proteinosis ***(B) EosinophHic pneumonia ***(C) Goodpasture syndrome ***(D) Hyaline membrane disease ***(E) Radiation pneumonitis
20 The answer is A: Alveolar proteinosis. Alveolar proteinosis (also tenned lipoproteinosis) is a rare condition in \Vhich the alveoli are filled with a granular, proteinaceous, eosinophilic mate1ial, which is PAS-positive, diastase resistam, and rich in lipids. The disease was initially desnibed as idiopathic, but recent studies have associated alveolar proteinosis with compromised immunity, leukemia and lymphoma, respiratoty infections, and e;.,•posuTe to environmental inorganlc dusts. Repeated bronchoalveolar lavage 1s used to remove the alveolar material, and repeated lavage may halt progression of the disease. None of the mher choices exhibit an acellular eosinophiLic material that fills the alveolt. ***Diagnosis: Alveolar proteinosis
21 A 50-year-old woman presents with a 4-week hisrory of fever, shormess of breath, and dry cough. She reports that her chest feels "ught." The patiem is a pigeon fancier. Blood tests show leukocyrosis and neutrophilia, an elevated erythrocyte sedimentation rate, and increased levels of immunoglobulins and C-reactive protein. A lung biopsy reveals poorly fom1ed granulomas composed of epithelioid macrophages and mllltinucleated g~am cells. Which of the following IS the appropriate diagnosis? ***(A) Actinomycosis ***(B) Goodpasture syndrome ***(C) Hypersensitivity pneumonitis ***(D) Nocardiosis ***(E) Wegener granulomatosis
21 The answer is C: Hypersensitivity pneumonitis. Hypersensitivity pneumonitis (extrinsic allergic alveolitis) is a response w inhaled amigens. The inhalation of a vmiety of antigens leads to acute or chronic interstitial inflammation m the ltmg. Hypersensitivicy pneumonitis may develop in response to repeated exposure w a vatiety of organic materials (e.g., bird droppings, feathers, mushrooms, and tree bark). Histologically, the Lung comains poorly fo rmed granulomas. whid 1 differ from the compact (solid) noncaseating granulomas of sarcoidosis and the caseating granulomas of tuberculosis or histoplasmosis. Actinomycosis and nocardiosis (choices A and D) do not induce granulomas. 'vVegener granulomatosis (choice E) is not known to be associated \.vith environmental e}..1)osu re. ***Diagnosis: Hypersensitivity pneumonitis, pigeon breeder lung disease
22 A 55-year-old man is admitted to the hospital with increasing shortness of breath and dry cough for the past few years. He smokes L'5 packs of cigarenes and drinks abom four bottles of beer a day. He is cm15tamly "gasping for air" and now walks with difficulty because he becomes breathless after only a few steps. Prolonged expiration with wheezing is noted. Physical examinadon shows a barrel chest, hyperresonance on percussion, and clubbing of the diglts. The patient's face is puffy and red, and he has pining edema of the legs. A chest X-ray discloses hypennnation, flattening of the diaphragm, and increased retrostemal air space. vVh ich of the following is the appropriate diagnosis? ***(A) Asthma ***(B) Chronic bronchitis ***(C) Emphysema ***(D) Hypersensitivity pneumonjtis ***(E) UsuaJ imersddal pneumonia
22 The answer is C: Emphysema. Chronic obstructive pulmonmy disease is a nonspecific term that describes patients with chronic bronchitis or emphysema who evidence a decrease in forced expiratory volume. Emphysema is characterized principally by hyperinflated lungs. Chronic bronchitis (choice B) occurs after recunent infections and, like asthma (choice A), is not generally associated with l1yperinOated lungs. The major cause of emphysema is cigarette smoking, and moderate-tosevere emphysema is rare in nonsmokers. ***Diagnosis: Emphysema
23 vVhich o[ the following best describes the expected histopathology of the lungs 1n the patient described in Question 221 ***(A) Destruction of the walls of airspaces without 6brosis ***(B) Interstitial fibrosis of the lung parenchyma ***(C) Lymphocytes restricted to the interstitium ***(D) Prominent bronchial smoorh muscle cell hyperplasia ***(E) n1ickening of the epithelial basement membrane
23 The answer is A: Destruction of the walls of airspaces without fibrosis. Emphysema is a chronk lung disease characterized by enlargement of the airspaces distal to the terminal bronchioles, \Vith destruction of their walls, but wilhout fibrosis or inflammation. ***Diagnosis: Emphysema
24 A 35-year-old woman with a long history of dyspn ea, chron ic cough, sputum production, and wheezing dies of respiratory failure following a bout of lobar pneumonia. She was a nonsmoker and did not drink alcoholic beverages. The lung at autopsy is shown in the image. Which of the following underlying conditions was most likely associated with the pathologic changes shown here? ***(A) a 1-Antitrypsin deficiency ***(B) Cystic fibrosis ***(C) Goodpasmre syndrome ***(D) Hypersen sitivity pneumonitis ***(E) Kanagener syndrome
24 The answer is A: a,·Antitrypsin deficiency. Hereditary deficiency of a 1-amitrypsin accounts for about 1% of all patients with a clinical diagnosis of chronic obstructive pulmonary disease and is considerably more common in young persons with severe emphysema. Emphysema in patients with this genetic disease is diffuse and is classified as panacinar. ln the lung, the most importam action of <X1-amitrypsin is its inhibition of neutrophil elastase, an enzyme that digests elastin and other structural components of the alveolar septa. Most patients with clinically-diagnosed emphysema who are youn ger than 40 years of age have a 1-amin)<psin (PiZZ phenotype) deficiency. Emphysema is nOla feature of the other choices. ***Diagnosis: a 1-Antitrypsin deficiency, emphysema
25 A 55-year-old man was admitted to the hospital with a chief complaint of tncreasing shortness of breath over Lhe past several years. The patient was a heavy smoker over the past 40 years. Physical examination reveals cyanosis, elevated jugular venous pressure, and pe1ipheral edema. A high-resolution CT scan shows bullae over both lungs. Chronic intra-alveolar exposure to which of the follmvi.ng proteins is most likely associated with the pathogenesis of chronic obstmctive pulmonaty disease in dlis patient? ***(A) Alkaline phosphatase ***(B) o.1-Antitrypsin ***(C) CoUagenase ***(D) Elastase ***(E) a 2-Macroglobulin
25 The answer is D: Elastase. The dominant hypothesis concerning the pathogenesis or emphysema is the proteolysis-andproteolysis theory. In most patients with emphysema, it is thought that tobacco smoke induces an inflammatory reaction. Serine elastase in neutrophils is a particu] arly potent elastolytic agem, which injures the elastic tissue of the lung. Over time, an imbalance in elastin generation and catabolism in the lung leads to emphysema (Le., emphysema results when elastolytic activity is increased or amielaswlytlc activity is reduced) a1-Aminypsin (choice B), a circulating glycoprotein produced in the liver, is a major inhibitor of a variety of proteases, including elastase. and accow1ts for 90% of amiprotelnase activity in the blood. ***Diagnosis: Emphysema
26 A 48-year-old man with a llistory of heavy smoking presents with a 3-year history of persistent cough and frequent upper respiratory infections, associated with spULum production. Physical examination reveal-; prominent ~-xp i ra tory wheezes and peripheral edema. Analysis o[ arterial blood gases reveals hypoxia and C02 retention. Which of the follov.ling is the appropriate diagnosis? ***(A) Are lectasis ***(B) Chronic obstructive pulmonary disease ***(C) Goodpasture syndrome ***(D) Hypersensitivity pneumonitis ***(E) Vsual interstitial pneumonia
26 The answer is 8: Chronic obstructive pulmonary disease. Chronic obstructive pulmonary disease is a nonspecific term th at describes patients with chronic bronchitis, emphysema, or bot11 who evidence obstruction to air flow in the lungs. lL is often difficult to separate the relative contribution of each disease to the clinical presentation. Chronic bronchitis is defined clinically as the presence of a chronic productive cough without a discernible cause for more than half the time over a period of 2 years. It is primarily a disease of cigareue smoking, with 90% of all cases occurring in smokers. TI1e frequency and severity of acute respiratmy traCL infections is increased in parients with chronic bronchitis. Exenional dyspnea and cyanosis supervene, and cor pulmonale may ensue. The combination of cyanosis and edema secondary to cor pulmonale has led to the label "blue bloater" for such patients. In contrast to patients with predominantly chronic bronchins, th ose with emphysema are at lower 1isk of recunent pulmonaly infections and are not so prone to the development of cor pulmonale. The clinical cou rse of emphysema is marked by an inexorable decline in respiratory function and pmgressive dyspnea, for which no rreatment is adequate. ***Diagnosis: Chronic obstructive pulmonary disease, chronic bronchitis. Emphysema
27 An 8-year-old girl is brought imo the physicians office in mild respiratory distress. She has a history of allergies to cats and wool, and her parems state that she has recurrent episodes o[ upper respiratory tract infections. Physical examjnation shows expLratory wheezes, use of accessmy respiratoty muscles, and a hyperresonam chest to percussion. Analysis of arterial blood gases discloses respiratory alkalosis, and the petipheral eosinophil coum is increased. What is the appropriate diagnosis? ***(A) Acute bronchiolitis ***(B) Asthma ***(C) Cystic fi brosis ***(D) Kanagener syndrome ***(E) Usual interstitial pneumonia
27 The answer is 8: Asthma. Asthma is a chronic hmg disease caused by increased responsiveness of the airways to a va1iety of s timuli. Patiems typically have parm,'Ysn1S of wheezin g, dyspnea, and cough. Acme episodes of asthma may altemate with asymp· tomatic periods or they may be superimposed on a background of chronic airway obstruction. The consensus hypothesis attributes bronchial hyperresponsiveness in asdmu to an inflammatory reaction to diverse sdmuli, either extlinsic (e.g., pollen) or inninsic (e.g., exercise). Exni T1Sicasthma is typically a ch1ldhood disease , whereas intrinsic asthma usually begins in adults. The other choices do not lead lo wheezing and eosinophHia. ***Diagnosis: Asthma
28 A 10-year-old boy dies followi ng a severe episode of srams asthmaticus. Hiswlogic examination of the h.mg at autopsy is shown in the image. Which of the follmving best describes the path ologic featu res evidem in this autopsy specimen? ***(A) Destruction of the walls of airspaces withom fibrosLs ***(B) Hyaline membranes and intersthial edema (C} Interstitial fibrosis of the lung parenchyma ***(D) Intra-alveolar hemorrhage and exudates comain]ng neutrophils ***(E) Smooth muscle hyperplasia and basement membrane thickening
28 The answer is E: Smooth muscle hyperplasia and basement mem· brane thickening. When severe acme asthma is unresponsive to therapy, it is referred to as stams asrhmaricus. Hisrological examination of lung from a patient who died in stams asthmaticus often shows a bronchus containing a luminal mucous plug, submucosal gland hyperplasia, smooth musde hyperplasia, basement membrane thickening, and increased numbers of eosinophils. All of the other choices concern alveolar damage, whereas the photograph demonstrates a section of bronchus. ***Diagnosis: Asthma
29 A 60-year-old mason complains of shortness of breath, which has become progressively worse during the past year. A chest X-ray shows small nodular shadows ln both lungs. Pulmonary function studies reveal a pattem consistenL with restrictive lung disease. The patient subsequently develops congestive hean failure and expires. Autopsy discloses numerous small, fibrotic nodules in both lungs. Histologic examination of these nodules is shown in the image. ·which of the following is the most likely diagnosis? ***(A) Anthracosis ***(B) Asbestosis ***(C) Sarcoidosis ***(D) Silicosis - . --.""" ~-~---.-~~ --- - · ***(E) Wegener gramtlomatosis
29 The answer is D: Silicosis. Silicosis is caused by inhalation of small crystals of quartz (silicon dioxide), which are generated by stone cutting, sandblasting, and mining. The condition is marked by the insidious development of fibrotic pulmo· nary nodules containing quartz crystals. The disease may be asymptomatic for prolonged periods of time or may cause only mild to moderate dyspnea. Continued exposure may lead to progn~ssive fibrosis and severe respiratory embarrassment. Anthracosis (choice A) by itself does not cause resuictive lung disease, whereas asbestosis (choice B) is characterized by interstitial fibrosis. The nodules of sarcoidosis (choice C) and Wegener granulomatosis (c hoice E) are not fibrotic. ***Diagnosis: Silicosis
30 A 65-year-old coal miner is admitted for evaluation of chronic lung disease. The patient admil5 to smoking one pack of cigareues a day for 40 years. On physical examination , he is noticed to have a bane} chest and use accessory muscles for inspiration. His face is puffy and red. He has 2+ pitting edema of the lower extremities. A chest X-ray is compatible with diffuse fibrosis, ~rith some nodularity in central areas. Which of the following is the most likely diagnosis? ***(A) Anthracosilicosis ***(B) Asbestosis ***(C) Diffuse alveolar damage ***(D) PsiLtacosis ***(E) Sarcoidosis
30 The answer is A: Anthracosilicosis. Coal dust is composed of amorphous carbon and other constituents of lhe earth's sur· face , including variable amounts of silica. Amorphous carbon by itself is not fibrogenic owing to Jts inability to kill alveolar macrophages. lt is simply a nuisance dust that causes an innocuous anthracosis. By conu·ast, silica is highly fibrogenic, and the inhalation of rock particles may therefore lead to the lesions of anthracosilicosis. Coal workers· pneumoconiosis is also known as "black lung disease'' clue to massive deposits of carbon panicles. The characteristic pulmonary lesions of complicated coal workers' pneumoconiosis include palpable coaldust nodules scattered throughoUL the lung as 1- to 4 -mm black foci. Nodules consist of dust-laden macrophages associated with a fibrotic stroma. Coal miners are not predisposed to the other choices. ***Diagnosis: Coal workers' pneumoconiosis, anthracosilicosis
31 A 75-year-old man who had worked in a shipyard dies of a chronic lung disease. Auropsy reveals extensive pulmonary fibrosis, and iron stains of lung tissue show numerous fermginous bodies. The dome of the diaphragm is shown in rhe image. What is the appropriate diagnosis? ***(A) Anthracosis ***(B) Asbestosis ***(C) Bel)'lliosis ***(D) Sarcoidosis ***(E) Silicosis
31 The answer is B: Asbes1osis. Asbestosis refers to the diffuse interstitial fibrosis that results from the inhalation of asbestos fibers. The disease occurs as a result of the processing and handling of asbesros, rather than mining, whjch is a surface operation. Asbestosis is characterized by bilateral, diff1.1se imerstitial fibrosis and asbestos bodies in the Lung. These fermginous bodies are golden brown and beaded, whh a cenrral colorless core fi ber. Asbestos bodies are encrusted with protein and iron. In this patient, the dome of the diaphragm is covered by a smooth, pearly white, nodular fibrotic lesion (pleural plaque), a common feature of asbesws exposure. A clear-cut relationshlp between occupational asbestos ex-posure and malignant mesothelioma is establish ed. None of the other choices display pleural plaques or fenuginous bodies. ***Diagnosis: AsbestOsis
32 A 69-year-old retired man is brought to the emergency department because he experienced sudden onset of left-sided chest pain, which is exacerbated upon msptration. He is raking no medications and has been in good health. Physical examination reveals dyspnea and hemoptysis. Temperature is 38°C ( 101 °F). pulse rate is 98 per mjnme, respirations are 3 5 per minme, and blood p ressure is 158/100 mm Hg. A pleural friction rub is presem on ausculmrion. The left leg is markedly edematous. with a positive Homans' sign. An ECG shows a nonnal sinus rhythm A chest X-ray reveals a left pleural effusion. Whm is the most likely cause of this patients pulmonary condition? ***(A) Congestive hean [allure ***(B) Cor pulmonale ***(C) Mitral stenosis ***(D) Subacute endocarditis ***(E) Th romboembolism
32 The answer is E: Thromboembolism. Pulmonary arterial embolism is potentially fatal Mosl pulmonary emboli arise from the deep veins of t.he lower extremities. This patiem had signs of deep venous thrombosis of the leg. However, only half of patients 'Nith p ulmonary thromboembolism have deep vein Lhrombosis. ln this patient, pulmonary embolism was associated with pulmonary infarction, pleuritic ch est pain, hemoptysis, and pleural effusion. None of the other choices feature pleuritic signs and symptoms. ***Diagnosis: Pulmonary thromboembolism
33 A 25-year-old black woman p resents with a 3-month history of cough and shormess of breath on exertion. A chest X-ray reveals enlargemem of hilar and mediastinal lymph nodes. Laboratory studies show elevated serum levels of angiotensinconverting enzyme and an increase in 24-hour urine calcium excretion. An open-lung biopsy is shown in the image. Stains for microorganisms in the tissue are negative. Which of the following is the most likely diagnosis? ***(A) Goodpasmre syndrome ***(B) Sarcotdosts ***(C) Sthcosts ***(D) TuberculosiS ***(E) Wegener granulomatosis
33 The answer is B: Sarcoidods. Sarcoidosis is a granulomatous disease of unknown etiology. ln sarcoidosis, the lung is the most frequently involved organ, but the lymph nodes, skin, and eyes are also common targets. Angiotensin-convening enzyme (ACE) is produced by epitheLioid macrophages and is elevated in the blood. Spontaneous regression of lesions is common, but in some cases, Lhe disease causes pulmonary fibrosis and respiraLory failure. Symptoms of pulmonary disease include dyspnea, cough. and wheezing. None of the oilier choices are associated with increased serum levels of ACE. ***Diagnosis: Sarcoidosis
34 A 4 3-year-old woman \.vith SJogren syndrome and a '5-year history of cough and shortness of breath develops end-stage lung chscase and dies of respirawry fat lure. Histologic examJnauon of the lung al aULopsy ts shown in the image. Which of the followmg is the most Likely diagnosLS? ***(A) Alveolar proteinosis ***(B) Churg-Strauss syndrome ***(C) Langerhans cell histiocywsis ***(D) Lymphangioleiomyomatosis ***(E) Lymphocytic interstitial pneumonia
34 The answe-r is E: Lymphocytic interstitial pneumonia. Lymphocytic interstitial pneumonia {UP) is a rare pneumonitis in which lymphoid in filtrates are disuibmed diffusely ln the interstitial spaces of the lung. ln this case, the walls of the alveolar septa are diffusely infiltrated with chronic inflam- 111atmy cells. UP often occurs in a var iely of clinical seuings, including Sjogren syndrome and HN infection. The course of the disease vmies from an indolent condition to one tl1at progresses to end-stage lung cUsease and resplratmy failure. Langerhans cell histiocytosis (choice C) features nodular infiltrates. lmerstitiallymphocytic infilu·ates are not characteristic of the other choices. ***Diagnosis: Lymphocyuc Interstitial pneumonia
35 A 23-year-old man complains o r nasal obstruction, sero· sanguinous discharge, cough, and bloody spulUm. A chest X-ray shows cavitated lesions and multiple nodules over both lung fields. ACT scan discloses obliteration of several maxillary sinuses. U1inalysis reveals hematuria and RBC casts. Laboratory studies demonstrate anemia and elevated serum levels of C-ANCA. An open-lung biopsy is show11 in the image. Which of the following is the most likely diagnosis7 ***(A) Adenocarcinoma oflung ***(B) Churg-Strauss syndrome ***(C) Necrotizing sarcoid granulomatosis ***(D) Tuberculosts ***(E) Wegener granulomatosis
35 The answer is E: Wegener granulomatosis (WG) . WG is a djsease of unknown cause that is characterized by aseptic, necrotizing, granulomatous inflammation and vasculitis. This disease affects the upper and lovver respiratory tract and kidneys. Pulmonmy features of WG include necrotizing granulomatous inflammation, parenchymal necrosis, and vasculitis. ln most cases, multiple nodules averaging 2 to 3 em in diameter are seen in the lungs. \VG most commonly affects the head and neck, followed by the lung, kidney, and eye. Respiratory manifestations include sinusitis, cough. hemoptysis, and pleuritis. Sinus involvement is not common in the incorrect choices. Churg-Strauss syndrome (choice B) shares some features with WG, but is characterized by asthma, peripheral eosinophilia and P-ANCA. ***Diagnosis: Wegener granulomatosis
36 A 31-year-old woman smoker complains of nonproducdve cough, chest pain, shortness of breath on exenion, and fatigue. A CBC is normaL A chest X-ray shows ill-defined nodules, reticulonodular infilnates, a small cavitary lesion in the tigh t middle lobe, and mediastinal adenopathy. A transbronchial biopsy is shown in the image. Which of the following is the most likely diagnosis? ***(A) Goodpasture syndrome ***(B) Langerhans cell histiocytosis ***(C) LymphangioleiomyomaLOsis ***(D) Pulmonary imerstillal fibrosis ***(E) Wegener granulomatosis
36 The answer is B: langerhans cell histiocytosis. Different pre· sentations of Langerhans cell h istiocytosis have been called eosinophilic granuloma, Hand-Schuller-Ch ristian disease, and Letterer-Siwe disease. ln adults, the disorder occurs most often as an isolated form known as pulmonary eosinophilic granuloma. Vinually aU of these patients are cigarette smokers. The pulmonary lesions consist of varying proportions of Langerhans cells adm.Lxed with lymphocytes, eosinophils, and macrophages. Eosinophils are not typical of the other choices. ***Diagnosis: Langerhans histiocytosis
37 A 30-year-old woman presents wi th shonness of breath and bloody sputum. Physical examination reveals pulmonary crackles and abdomlnal ascites. A chesL X-ray shows bilateral pleural eCfusions and marked hyperinOation of the lungs. A CT scan of the chesl discloses thin-walled, air-containlng cysts il1 a diffuse symmetric pattern. A lung biopsy is shown in the image. The patient responds favorably to amiestrogen and amiprogesterone therapy. Which of the following is the most likely diagnosis? ***(A) Bronchiectasis ***(B) Histiocytosis X ***(C) Lymphangioleiomyomatosis ***(D) Tuberculosis ***(E) Wegener granulomatosis
37 The answer is C: Lymphangioleiomyomatosis. Lymphangioleiomyomarosis is a rare interstitial lung disease that occurs in women of childbearing age. It is characterized by the widespread abnormal proliferation of smooth muscle in the lung (see photomicrograph), mediastinal and retroperitoneal Lymph nodes, and the major lymphatic ducts. On gross examination, the lungs show bilaleral. diffuse enlargement, with extenstve cystic changes resembling those of emphysema. Hormonal ablation through oophorectomy and antiestrogen and progesterone therapy has s11own some promise. None of the mher choices exhibit this morphologic pattern. ***Diagnosis: Lymphangioleiomyomawsis
38 A 22 -year-old man presents with a 6-momh history of increasing shonness of breath and persistent cough with rusty spurum. A chest X-ray shows diffuse bilateral alveolar infiltrates. Urine dipstick analysis reveals 2+ hematuria. A nansbronchial lung biopsy is shown in the image. Linear deposits of lgG and complement C3 are detected in the alveolar basement membrane by immunofluorescence. Which of the following is the most likely diagnosis? ***(A) Churg-Strauss syndrome ***(B) Goodpasrure syndrome ***(C) Hypersensitivity pneumonitis ***(D) Loeffler syndrome ***(E) vVegener granulomatosis
38 The answer is B: Goodpasture syndrome. Goodpasture syndrome is an amoimmune disease in which auwantibodies bind lO the noncollagenous domain of type TV collagen. This cmmective tissue protein is a major structural component of bmh pulmonary and glomerular basement membranes. Local complement activation results in the recruim1ent of neutrophils, tissue injury, pulmonary hemorrhage. and glomemlonephritis. Anti-~pe lV collagen amibodles are noc encountered in the other choices. ***Diagnosis: Goodpasmre syndrome
39 A 28-year-old man presents •vith 6 days offever and shorllless of breath. His temperature is 38. JOC (l03°F), respirations are 30 per minute, and blood pressure is 120/80mmHg. A chest X-ray reveals diffuse interstitial and alveolar mfiltrates. Sputum cultures are negative, and the patient does nDL respond w standard amibiotic therapy A transbronchiallung biopsy is shown in the image. \Vhich of the foUowing is rhe approptiate diagnosis~ ***(A) Eosinophilic pneumonia ***(B) Lipid pneumonia ***(C) Pneumococcal pneumonia ***(D) Pneumocystis pneumonia ***(E) Usual interstitial pneumonia
39 The answer is A: Eosinophilic pneumonia. Eosinophilic pneumonia is principally an allergic disorder. It refers to Lhe accumulation of eosinophHs in alveolar spaces and is classified as either idiopathic or secondary to an underlying illness. ln acme eosinophilic pneumonia, the alveolar spaces are filled with an inflammatory e>:udate composed of eosinophils and macrophages. The alveolar sepla are thickened by the presence of numerous eosmophils and hyaline membranes are presem. Patiems respond dramatically to corticosteroids, and, in conm1st m chronic eosinophilic pneumonia, acut.e eosinophilic pneumonia does not recur. Excess eosinophils are not encountered in the other choices. ***Diagnosis: Eosinophilic pneumonia
40 A 53-year-old man presents with increasing shonness of breat11 on exenion and dry cough that has developed over a period of a few years. Physical examination shows clubbing of the fingers. A chest X-ray discloses diffuse bilateral infillrates, predominamly in the lower lobes, in a reticular pattern. Two years later, the patlem suffers a massive stroke and ex'Pires. Histologic examination of the lung at autopsy is shown in the image. Patchy scan ing with extensive areas of honeycomb cystic change predominantly affects the lower lobes. Which of the following is the most likely diagnosis? ***(A) Churg-Strauss syndrome ***(B) Desquamative imerstiLlal pneumonia ***(C) Goodpasture syndrome ***(D) Usual imerstitial pneumonia ***(E) Wegener granulomatosis
40 The answer is 0: Usual interstitial pneumonia (UIP). UIP is one of the most common types of imersdlial pneumonia and demonsu·ates a histologic pattern that occurs in a variety of clinical settings, including collagen vascular disease, chronic hypersensitivity pneumortilis, and drug LOxicity. Most commonly, it has no known cause, although viral, genetic, and immunologic factors may be implicated. A microscopic view of the lung in this case shows patchy, subpleural fib rosis with rrticroscopic "honeycomb" cystic change. Diffuse fibrosis is not characteristic of the other choices. ***Diagnosis: Usual interstitial pneumonia
41 A 55-year-old woman complains of sudden onset of feve r, dry cough, and shortness of breath. She was seen for flu -like symptoms 6 weeks ago A chesL X-ray shows bilateral patchy alveolar consolidations. An open-lung biopsy reveals narrow inflamed airways containing plugs of fibrous tissue (shown in the lmage). Which of the following is Lhe most likely diagnosis? ***(A) AIDS-related pneumonia ***(B) Alveolar proteinosis ***(C) Cryptogenic organizing pneumonia ***(D) Diffuse alveolar damage ***(E) Wegener granulomatosis
41 The answer is C: Cryptogenic organizing. pneumonia. Organizing pneumonia was previously referred to as bron chiolitis obliterans-organizing pneumonia. The hiswlogic pattern is nor specific for any particular etiologic agent and may be observed in many se ttings, including respiratory tract infections, inhalation of toxic matelials, and collagen vascular diseases. ln the absence of a speci£c etiology, the tenn cryptogenic organizing pneumonia is applied. loose fibrous tissue in the alveoli and bronchioles is a typical finding in patients with C1)1Jtogen ic organizing pneumonia. Di ff·use alveolar damage (choice D) features intra-alveolar fibrin (hyaline membranes). ***Diagnosis: Cryptogenic organizing pneumonia
42 A 45-year-old woman with severe kyphoscoliosis presents ' IVith fatigue, shortness of breath on exenion, (aiming spells, and bloody spumm. A CBC is nom1aL Physical examination shows splitting of the second hean sound with accentuation or the pulmonic component, distended neck veins with a prominent V wave, a right ventricular third hean sound, and peripheral edema. A chest X-ray film shows enlargemem of the central pulmonary arteries. A rransbronchial lung biopsy is shown in the image. Which of the foUowing is the most likely djagnosis? ***(A) Churg-Strauss syndrome ***(B) Diffuse alveolar damage ***(C) Eosinophilic granuloma ***(D) Pulmonary hypertension (£) 'vVegener granulomatosis
42 The answer is D: Pulmonary hypertension. Pulmonary hypertension is characterized by Lhickening of the media of pulmO• nary muscular atteries_ As pulmonary hypertension becomes more severe, there is extensive intimal fibrosis and muscle thickening \vithin aneries and atterioles, which may be occlusive_ Churg-$trauss syndrome (chmce A) exhibits vascuHtis and eosinophilia but is excluded in this case on the basis of a normal CBC The other choices do not principally affect anenes_ ***Diagnosis: Pulmonary hypertension
43 A 56-year-old man with a history of cigarette smoking presems with difficulty swallowing and a mufned voice. Laryngoscopy reveals a 2-cm laryngeal mass. If thic; mass is a malignant neoplasm, which of the following is the most likely histologic diagnosis? ***(A) Adenocarcinoma ***(B) Leiomyosarcoma ***(C) Small cell carcinoma ***(D) Squamous cell carcinoma (£) Transitional cell carcinoma
43 The answer is D: Squamous cell carcinoma. The vast majorit)' of laryngeal cancers are squamous cell carcinomas and occur principally in smokers. Adenocarcinoma (choice A), leiomyosarcoma (choice B), and small cell carcinoma (choice C) arc rarely encountered Ln lhe larynx. ***Diagnosis: La ryngeal cancer
44 A 56-year-old man undergoes a routine chest radiograph as pan of a comprehensive physical examination. The X-ray film of the chest shows a solitary, centrally located coin lesion, with a "popcorn" pattern of calcification. A lung biopsy is performed and reveals nodules of benign mature car tilage and respirawry epithelium (shovm in the image). What 1s the most likely diagnosis? ***(A) Carcinoid ntmor ***(B) Extra lobar sequestration ***(C) Leiomyoma ***(D) Pulmonary fibroma ***(E) Pulmonary hamartoma
44 The answer is E: Pulmonary hamartoma. AlLhough the term hamartoma implies a malformation, hamartomas are uue mmors_ They are composed of cartilage, fibromyxoid cOJmeCLive tissue, fat, bone, and occasional smooth muscle. They typically occur in adults, with a peak in the sixth decade of life_ Hamartomas are the cause of approximately 10% of "coin" lesions discovered incidemally on chest radiographs. A characteristic "popcorn" pattern of calcification is often seen by X-ray. Canilage is not encountered in the other choices. ***Diagnosis: Pulmonaty hamartoma
45 A 68-year-old man complains of shormess of breath, hoarseness, productive cough, and bloody sputum of 2 weeks in duration. He admits to smoking two packs a day for 45 years and drinks occasionally. Recemly, he has experienced a signH1canr loss of appetite and weight loss. Physical examination shows pallor, cachexia, clubbing of the finge rs, and banel-shaped chest. A chest X-ray reveals a mass at the right lung apex. Hiswlogic examination of a transbronchiaJ biopsy is shown in the image. What is the appropriate histologic diagnosis? ***(A) Adenocarcinoma ***(B) Mesothelioma ***(C) Mctastanc adenocarcmoma ***(D) Small cell carcinoma ***(E) Squamous cell carcinoma
45 The answer is E: Squamous cell carcinoma. Squamous cdJ carcinoma accounts for 30% of aU invasive lung cancers in the United States. Well-differentiated squamous cell carcinoma displays keratin "pearls,'' which appear as a small round nest of brightly eosinophilic aggregates of keratin surrounded by concenuic ("onion skin") layers of squamous cells. Gland for· mation is exhibited in adenocarcinoma (choices A and C). ***Diagnosis: Squamous cell carcinoma of lung
A 53-year-old woman with a hislOry of cigarette smoking presems wah a 3-momh h]stOJy of chest pain, cough, and mild fever. A chest X-ray reveals a penpheral mass in the left upper lobe. The surgical specimen is shown in the image. What IS the most hkely diagnosis? ***(A) Adenocarcinoma ***(B) Large cell carcinoma ***(C) MesOlhelioma ***(D) Smal1 cell carcinoma ***(E) Squamous cell carcinoma
46 The answer is A: Adenocarcinoma. Adenocarcinoma usually presents as a pe1i pheral subpleural mass composed of neoplastic gland-like structures. Cemral (hilar) cancers of the lung can be of any of the histologic types (e.g. , choices B, D, and E), whereas peripheral lung cancers are most commonly diagnosed as adenocarcinomas. They are often associated with pleural fib rosis and subpleural scars. At initial presentation, adenocarcinomas usually appear as irregular masses, although they may be so large that they completely replace the entire lobe of the lung. Mesothelioma (choice C) is pleural based. ***Diagnosis: Adenocarcinoma of lung
47 A 67-year-old woman wiLh a history of smoking presents \>lith a 3-week history o[ chest pain and bloody sputum. A chest X-ray reveals a bulky mass within the pulmonary parenchyma. An open-lung biopsy is shown in the image. Immunohistochemical stains for keratin and chromogranin are negative. WhaL is the appropriate diagnosis? • ***(A) Adenocarcinoma ***(B) Carcinoid lUmor ***(C) Large cell carcinoma ***(D) Metastatic adenocarcinoma ***(E) Small cell carcinoma
47 The answer is C: Large cell carcinoma. Large cell undifferentiated carcinoma is composed of atypical neoplastic cells that do not resemble any no rmal cells in the lung. These cells do not form glands (like adenocarcinoma) and do not ex-press cywkeralin (choices A and D). Chromogranin is expressed in carcinoid tumors (choice B) and often in small cell carcinomas (choice E). ***Diagnosis: Large cell carcinoma of lung
48 A 58-year-old man presents with a long history o[ persistent cough, chest pain, and recunent pneumonia He denies smoldng or consuming alcohol. The patient subsequently dies of sepsis. Autopsy reveals malignant cells that diffusely infiltrate the lung parenchyma Histopathologic examination of the lung shows well-differentiated, mucus-producing, colunmar neoplastic cells lining the alveolar spaces (shown in the image) Neoplastic cells are nm found in any other organ. What is the most likely diagnosis? _ =-::. .... =•-- __.__, _ __ ........ _ ***(A) Bronchioloalveolar carcinoma ***(B) Carcinmd rumor ***(C) Large cell carcmoma ***(D) McsOLhdioma ***(E) Small cell carcinoma
48 The answer is A: Bronchioloalveolar carcinoma. Bronchioloalveolar carcinoma is a primary pulmonary adenocarcinoma originating from stem ceUs in the terminal bronchioles. The cells may be columnar and mucus producing or cuboidal and similar to type li pnewnocytes. They tend to grow along the alveolar septa, as depicted in the photomicrograph. A similar growth pauern may be seen in metastatic adenocarcmomas. None of the other tumors produce alveolar mucus or d1splay alveolar spaces lined by a colunmar epithelium. ***Diagnosis: Bronchioloalveolar carcinoma
49 A 52-year-old woman presems with a 1-year history of upper truncal obesiry and moderate depression. Physical examination shows hirsutism and moon facies. Endoctinologic sLUdies reveal hypokalemia, high plasma conicotropin levels, and increased concemrations of semm and urine cortisol. CT scan of the thorax. demonstrates a hjlar mass. A transbronchial lung biopsy is shown in the image. Electron mkroscopy mscloses neuroendocrine granules within the cytoplasm of some tumor cells. What is the approptime diagnosis? ***(A) Adenocarcinoma ***(B) Bronchioloalveolar carcinoma ***(C) Carcinoid mmor ***(D) Metastatic carcinoma ***(E) Small cell carcinoma
49 The answer is E: Small cell carcinoma. Small cell carcinoma (previously referred to as "oat-cell" carcinoma) is a rugh ly malignam epithelial tumor of the lung that exhibits neuroendocrine features. lt accoums for 20% of all1ung cancers and is strongly associated with cigarette smoking. Metastases occur early and are widespread. Carcinoid rumors (choice C) also contain neuroendocrine granules, but the tumor cells are arranged in a distinctive pauern. Moreover, Cushing syndrome is often encountered in patients with small cell carcinoma, but not carcinoid mmor (choice C). ***Diagnosis: Small cell carcinoma of lung
50 A 55-year-old man presents with increasing chest pain, bloody sputum, and weight loss over the past 3 months A highresolution CT scan reveals a mass circumscribing the right main bronchus, ex1:endlng into its Lumen_ Histologic examination of an open-lung biopsy ts shown in the image. Electron microscopy shows numerous neuroendocrine granules within tumor cells_ What is the appropriate diagnosis? ***(A) Adenocarcinoma ***(B) Bronchioloalvcolar carcmoma ***(C) Carcmoid tumor ***(D) Large cell carcinoma ***(E) Squamous cell carcinoma
50 The answer is C: Carcinoid tumor. Carcinoid tumors account for 2.% of all primary lung can cers. They comprise a group of neuroendocrine neoplasms derived from the pluripotential basal layer of the respiratory epithelium. Carcinoid tumors occur most often in the wall of the major b ronchus and may pronude into its lumen. The tumors are characterized by an organoid growth pattern and uniform cytologic features Carcinoid tumors exhibit a neuroendocrine differentiation similar to that of resident Kulchitsky cells. The indolent nature of carcinoid tumors is reOected in the finding that half of the patiems are asymptomatic at the time of presentation, bm regional lymph node metastases occur in 2.0% of patients. Atypical carcinoids exhibit a more aggressive behavior. Neuroendocrine features are absem in the other tumors. ***Diagnosis: Carcinoid tumor of lung
51 A 64-year-old man who has worked in a manufacmring plant aU his life complains of an 8-month history of chest discomfort, malaise, fever, night sweats, and weight loss. A chest X-ray reveals a pleural eiT1.Lsion and pleural mass encasing the lung. The patiem subsequently dies of cardiorespiratory failure. Histologic examination of the pleural mass at autopsy shows a biphasic pattern of epithelial and sarcomawus elements. What is the most likely diagnosis? ***(A) Carcinoid tumor ***(B) Large cell carcinoma ***(C) Malignam melanoma ***(D) Malignam mesothelioma ***(E) Metastatic carcinoma
51 The answer is D: Malignant mesothelioma. Mesothelioma is a malignam neoplasm of mesothelial cells that is most common in the pleura, but also occurs in the peritoneum, pericardium, and the runica vaginalis of the testis. The tumor is strongly linked lO occupational inhalation of asbestos. Patiems are often first seen \vith a pleural effusion or a pleural mass, chest pain, and nonspecific symptoms, such as weight loss and malaise. Pleural mesotheliomas tend to spread locally and extensively \-\lithin the chest ca\lity, but do not typically invade the pulmonary parenchyma. Widespread metastases can occur. Mesothelioma is typically composed of both epithellal and sa rcomatous elements (i.e., biphasic pattern). The other choices do not ordinarily encase the lung. ***Diagnosis: Malignant mesothelioma
52 A 48-year-old woman with a long-standing history of ulcerative coUtis presents ~'ith anemia and shormess of breath. Laboratory studies show increased serum levels of carcinoembryonic antigen. A chest X-ray reveals multlple, round masses in both lungs. Histologic examination of an open-lung biopsy discloses nodules that are composed o( gland-like structures. What is the most likely diagnosis? ***(A) Adenocarcinoma ***(B) Bronchioloalveolar carcinoma ***(C) Eosinophilic granuloma ***(D) Large cell undifferentiated carcinoma ***(E) Metastatic carcinoma
52 The answer is E: Metastatic carcinoma. Pulmonary metastases represent the most common neoplasm of the lung. In one third of all fatal cancers, pulmonary rnetastases are evident at autopsy Metastatic carcinomas typically present as multiple, round masses scattered at random throughout the parenchyma of lungs and Liver. Although pulmonary adenocarcinoma (choice A) and bronchoalveolar carcinoma (choice B) cannot be excluded on histologic grm.mds, this patient with ulcerative colitis is predisposed ro develop adenocarcinoma of the colon, which most likely accounts for the anemia and lung metastases. ***Diagnosis: Metastatic carcinoma of lw1g
53 A 72-year-old woman complains of shortness of breath upon exertion. She states that she also becomes shon of breath at night unless she uses three pillows. Physical examination reveals mild obesity, bilateral pitting leg edema, an enlarged liver and spleen, and fine crackling sounds on inspiration. A chest X-ray is consistent with pulmomuy edema and cardiomegaly. Which of the following is the most likely pulmonary compli.cauon in this patiem' ***(A) Chylothorax ***(B) Hemothorax ***(C) Hydrothorax ***(D) PneumoLhorax ***(E) Pyothorax
53 The answer is C: Hydrothorax. The elevation of hydrostatic pressure in patients 'vith congestive hean failure causes transudation of edema Ouid into tl1e pleural cavity (i.e., hydrothorax). Chylothorax (choice A) and hemothorax (choice B) refer to lymph and blood in the pleural space, respectively. Pneumothorax (choice D) and pyothorax (choice E) refer to air and acute inflammatory cells in the pleural space, respectively. ***Diagnosis: Congesuve hean failure , hydrothorax
54 A 27-year-old man with history of alcoholism and repeated bouts of aspiration pneumonia comes to the emergency room with a high fever and plemitic chest pain. Physical examination reveals dullness on percussion and absence of breath sounds in rhe tight lower lung field A chest X-ray demonstrates pleural fluid on the right side. Thoracemesis returns a th ick, foul-smelling fluid. Which of the following is the most likely djagnosis? ***(A) Chylothorax ***(B) Empyema ***(C) Hemothorax ***(D) Hydrothorax ***(E) Pneumothorax
54 The answer is B: Empyema. Pleuritis (inflammation of the pleura) may result from the extension of any pulmonary infecdon to the visceral pleura. Causes of pleuri tis include bacterial infections, viral infections, and pulmonary infarction involving the surface of the lung. Pyothorax refers to a turbid dfusion containing many neutrophils. Empyema is a var iant of pyothoTax in which thick pus accumulates within the pleural cavity, often with loculation and fibrosis. Hydrothorax (choice D) refers to transudation of edema fluid into Lhe pleural cavi ty. ***Diagnosis: Empyema
55 A 22-year-old man has been treated for lymphoma and comes Lo the emergency room complaining of severe shottness of breath. Physical exarrunation reveals decreased breath souncts and shifting dullness. Ce rvicaJ adenopathy is prominem Thoracocentesis yields a milky-white fluid with a high lipid comem. Which of the follmving is the most Hkely diagnosis? ***(A) Chylothorax ***(B) Empyema ***(C) Hemothorax ***(D) Hydrothorax ***(E) Pyothorax
55 The answer is A: Chylothorax. Chylothorax is defined as the accumulation in the pleural cavity of a milky, Upid-rich nuid as a result of lymphatic obstruction It has an ominous portent because ob stmction of the lymphatics suggests disease of the lymph nodes in the posterior mediastinum. Chylothora.-x is thus found as a rare complication of malignant tumors in the mediastinum, such as lymphoma. Empyema (choice B) is a loculated collection of pus with fibrous walls LhaL follows the spread of bacterial infection to the pleural space. ***Diagnosis: Chylothorax
1 A .3'5-year-old man complains of difficulty swallowing and a tendency to regurgitate his food . Endoscopy does not reveal any esophageal or gastric abnor malities. Manometric srudles of the esophagus show a complete absence of peristalsis, failure of the lower esophageal sphincter to relax upon swalJov. ling, and increased intraesophageal pressure. vVhich of the following is the most Hkely diagnosis? ***(A) Achalasia ***(B) Barrett esophagus ***(C) Esophageal stricrure ***(D) Mallory-Weiss syndrome ***(E) Schatzki ring
,_1 The answer is A: Achalasia. Achalasia is characterized by failure of the lower esophageal sphincter to relax in response to swallowing and the absence of peristalsis in the body of the esophagus. As a result of these defects in both the outflow tract and the pumping mechanisms o[ the esophagus, food is retained wilhin the esophagus, and the organ hypertrophies and dilates. Achalasia is associated with a depletion or absence of ganglion cells in the myenteric plexuses, which regulate contraction of the esophagus. In Latin America, achalasia can be a manifestation of d1agas disease, in which the ganglion cells are destroyed by Iiypanosoma cruzi. The other choices are usually associated with visible mucosal abnormalities and do not primarily affect peristalsis. ***Diagnosis: Achalas1a 2 The answer is D: Esophageal web. Esophageal rings and webs cause dysphagia. ·webs are chin mucosal membranes that project imo rhe lumen of the esophagus. Rings are thicker than webs and comai11 smooth muscle. The clinical symptoms of esophageal webs and rings include dysphagia, esophageal substernal pain, and aspiration or regurgitation of foods and Hquids. Plummer-Vinson syndrome is characterized by a cervical esophageal web, mucosal lesions of the mouth and pharynx, and iron-deficiency anemia. Carcinoma of the oropharyrL'< and upper esophagus are complications of Plununer-Vlnson syndrome. The other choices are not associated \Vith an emia. Schatzki ring (choice E) occurs near the gastroesophageal junction. ***Diagnosis: Esophageal web
2 A 20-year-olcl woman presents Virith a 2-year history of difficulty swaltowing and increasing fatigue. A CBC shows iron-deficiency anemia. Upper endoscopy reveals an annular narrowing tn the upper third of the esophagus. A mucosal biopsy shows no evidence of inflammation or neoplasia" Which of the following is the most likely diagnosis? ***(A) Achalasia ***(B) Barrett esophagus ***(C) Diverticulum ***(D) Esophageal web ***(E) Schatzki ring
3 The answer is B: Dysphagia. This patient exhibits signs of scleroderma (progressive systemic sclerosLc;), which is characterized by vasculopathy and excessive collage deposition in the skin and internal organs. Patients often suffer from intermittent episodes of ischemia of the fingers , marked by pallor, paresthesias, and pain (Raynaud phenomenon). Ami-Sd-70 amibocUes to nuclear ropoisomerase are vinually specific for this autoimmune disease. Scleroderma can involve any portion of the gastrointestinal tract, although esophageal dysfunction is the most common and troublesome complication. The disease affects principally the lower esophageal sphincter, which may become so impaired that the lower esophagus and upper stomach are no longer distinct functional entities. ln some affected patients, Lhere may be a lack of peristalsis in the entire esophagus. The other choices are not associated with sc l erodenna. ***Diagnosis: Sclerodem1a
3 A 45-year-olcl woman presents w1th general discornfon and increasing tightness m the skin of her face. She repons inrermiuem pain in the tips of her fingers when exposed to the cold. Physical examination shows ''stone facies" and edema of the fingers and hands. Serologic tests for antinuclear and ami-Scl-70 antibodies are both positive Which of rhe following gastroimestinal manifestations is expected in this pari em? ***(A) Adenocarcinoma of the esophagus ***(B) Dysphagia ***(C) Esophageal mpture ***(D) Esophageal V¥ices ***(E) Squamous cell carcinoma of the esophagus
4 The answer is D: Glandular metaplasia The biopsy shows .______ Barrett esophagus, which is defined as a replacement of the esophageal squamous epithelium by columnar epithelium as a result of chronic gastroesophageal reflux. This disorder occurs typically in the lower third o[ the esophagus. The lesion is characterlzed histologically by distinctive intestine~Hke epitheUum composed of goblet cells and surface cells similar to those of incompletely intestinalized gastric mucosa. Com~ plete intestinal metaplasia, with Paneth cells and absorptive cells, may occur. Barrett esophagus is more resistant to peptic juices than normal squamous epithelium and appears to be an adaptive mechanism that serves to limit the harmful effects of gastroesophageal reflux. None of the other choices lead to metaplastic changes, ***Diagnosis: Barrett esophagus, reflux esophagitis
4 A 54-year-old man with a long hislmy of ind1gestion after meals and "heartburn" presems with upper abdominal pain. He was tremcd with proton-pump inlubitors for gastroesophageal ref1ux 3 years previously. An endoscopic biopsy of the lower esophagus is shown in the image. \Much of the following best describes these pathologic findings? ***(A) Candida esophagitis ***(B) Esophageal diverticulum ***(C) Esopbageal varices ***(D) Glandular metaplasia ***(E) Schatzki ring
5 The answer is 8: Adenoe:arcinoma. Barrett esophagus carries a serious risk of malignant transfonnation to adenocarcinoma, and the risk correlates with the length of the involved esophagus and the degree of dysplasia Virtually all esophageal adenocarcinomas arise in the background of the metaplastic epithelium o[ Barrett esophagus. The symptoms and clinical course of adenocarcinoma or the esophagus are similar to those of squamous cell carcinoma and include dysphagia, pain, and, occasionally, bleeding. None of the other choices reflect a complicacion of Baneu esophagus. ***Diagnosis: Barrett esophagus
5 The patient dese1ibed in Question 4 is at increased risk of developing whkh of the following diseases of the esophagus? ***(A) Achalasia ***(B) Adenocarcinoma ***(C) Candidiasis ***(D) Plummer-Vinson syndrome ***(E) Vatices
6 The answer is E: Zenker diverticuJum. Zenker diverticulum is an uncommon lesion that apr>ears high in the esophagus and affects men more than women_ 1L was once believed LO resuh fTom luminal pressure exerted in a struc turally weak area and was, therefore, classed as a pulsion diveniculum_ The cause is probably more complicated, but disordered function of the cricopharyngeal musculature is still thoughL m be involYed m the pathogenesis of this false diveniculum_ Most affected persons who come to medical auemion are older than 60 years, an observation that suppons the belief that Zenker diverticulum Lc; an acquired lesion. Epiphrenic diverticuli (choice A) are located immediately above the diaphragm. lnrramural pseudodivertlculum (choice B) is characterized by numerous small diverticula in the wall of the esophagus. Traction diverticuli (choice D) are outpouchings that occur principally in the midportion or the esophagus. ***Diagnosis: Zenker diverticulum
fi A 65-year-old woman complains of a 4-momh history of ,____ bad breath, regurgitation of undigested food, occasional aspiration of food, and change in the sound of her voice. A barium swallow examination shows a posterior, midline pouch greater than 2 em in diameter arising just above the cricopharyngeal muscle. Which of the following is the most likely diagnosis? ***(A) Epiphrenk diveniculum ***(B) Intramural pseudodiverticulum ***(C) Meckel diverticulum ***(D) Traction diverticulum ***(E) Zenker divemculum
....._7_ The answer is D: Hiatal hernia. Hiatal hernia is a protrusion of the stomach through an enlarged esophageal hiatus in the diaphragm. Two basic LYTJeS of hiatal hernia are observed. In sliding hiatal hernias, an enlargement of the hiatus and laxity of the circumferential connective tissue allows a cap of gastric mucosa to move upward above the diaphragm. Paraesophageal hiatal hernias are characte iized by hemiaLion of a porrion of the gastric fundus alongside the esophagus through a defect in the diaphragmmic com1ective tissue membrane Ll1aL defines the esophageal hiatus_ Symptoms of hiatal hernia, particularly heartburn and regurgiLation, are attributed LO the reflux of gasuic contents, which is primarily related to incompetence of the lower esophageal sphincter. Classically, the symptoms are exacerbated >vhen Lhe affected person is recumbent. Large herniations cany a risk of gastric volvulus or intrathoracic gasnic dilation. Boerhaave syndrome (choice A) represems ru pture of the esophagus as a result of vomiting. 1vlallory-Weiss syndrome (choice E) refers to mucosal lace ration of the upper stomach and lower esophagus m the setting of severe retching. ***Diagnosis: Paraesophagic hiatal hemi.a
7 A 45-year-old man presents with long-standing heart bum and dyspepsia. An X-ray film of the chest shows a retrocardiac, gas-filled strucm re. TI1is pariem most likely has which of the following conditions? ***(A) Boerhaave syndrome ***(B) Esophageal varices ***(C) Esophageal webs ***(D) Hiatal hernia ( E) Mallory-Weiss syndrome 8 A 3-year-old boy is mshed to the emergency room in acme disness. The child has vague chest pain and difficulty swallowing. He refuses to drink water. Physical examination shows drooling and salivation. Viml signs are normal. The mother states that she saw the boy ingesting a liquid used to clear drains. I[ this chemical was a strong acid, which of the following histopathologic findings would be e.xpected in the esophagus of this child? ***(A) Apoptosis ***(B) Coagulative necrosis ***(C) Fat necrosis ***(D) Hyaline sclerosis ***(E) Uquefacove necrosis
8 The answer is 8: Coagulative necrosi,s. Chemkal injury to th e esophagus usualJy reflects accidental poisoning il1 children, attempted suicide in adults, or contact with medication. lnge..<;tion of strong acids produces an immediate coagulative necrosis in the esophagus, which results in a proteCLive eschar that limits injury and furthe r chemical penetration. By conLra. o;t, ingestion of strong alkaline solutions Lo; accompanied by liquefactive necrosis (choice E), with inflanm1ation and saponification of membrane lipids. Alkaline solutions are particularly insidious because they are generally odorless and tasteless and, therefore, easily swallowed before protective reHexes come into play. ***Diagnosis: Chemical esophagitis
9 A 70-year-old woman presents with difficulty swallowing .....__ and a 9-kg (20-lb) weight loss over the past several momhs. Endoscopy reveals irregular narrowing of the lower third of the esophagus. A biopsy shows markedly atypical cuboidal cells lining irregular gland-like structures. Which of the fol lowing is the most lil<ely diagnosis? ***(A) Adenocarcinoma ***(B) Esophageal stricture ***(C) Leiomyosarcoma ***(D) Scleroderma ***(E) Squamous cell carcinoma
9 The answer is A: Adenocarcinoma. Adenocarcinoma of the esophagus is now more common (60%) in the United States than squamous carcinoma. Adenocarcinoma originates in the glandular metaplasia of Barren esophagus Endoscopic surveillance for adenocarcinoma is now commonly done in patients with Barrett esophagus, panicularly those with dysplasia. Tumors tend to grow into the lumen of the esophagus. The affected region of the esophagus is rypically indurated and ulcerated, causing pain and bleeding. The other choices do not exhibit the histologic features described. ***Diagnosis: Adenocarcinoma of the esophagus
10 A 60-year-old man presents \\rith a '5-week history of difficulty swallowing. Physical examination is unremarkable. Upper endoscopy shows a large mass in the upper third of the esophagus. A biopsy is shown in the image. What is the appropriate histologic diagnosis for this esophageal mass? ***(A) Adenocarcinoma ***(B) Glandular metaplasia ***(C) Malignam melanoma ***(D) Squamous cell carcinoma ***(E) Transitional cell carcinoma
10 The answer is D: Squamous cell carcin-oma. Most cancers of the esophagus world·wide are squamous cell carcLnomas, although adenocarcinoma is now more common in the United States. Squamous cell rumors range from well-differentiated cancers with "epithelial pearls" to poorly differentiated neoplasms that lack evidence of squamous difl'erentiation. The most common presenting complain1 of patients \<Vith esophageal cancer is dysphagia, bm by 1.his time, most tumors are tmresectable. Adenocarcinoma (choice A) and Barrett esophagus (choice B) are incorrect because the biopsy does not show glandular features. P1imary malignam melanoma (choice C) of the esophagus is extremely rare, although melanoma metastases to the esophagus may occur. ***Diagnosis: Squamous cell carcinoma of the esophagus
11 Which of the following is the most imponam risk factor for development of the esophageal mass identified in the patient described in Question 10? ***(A) Cigarette smoking ***(B) Exposure w afiawx:in ***(C) Herpetic esophagitis ***(D) Hot and spicy food ***(E) Reflux esophagitis
11 The answer is A: (jgarette smoking. Risk factors for squamous cell carcinoma of the esophagus include chronic alcoholism, wbacco use, diets lacking in fresh fruits, exposure 1.0 aniline dyes, chronic esophagitis, and congenillll disorders of the esophagus (e.g., Plummer-Vinson syndrome) Cigareue smolGng is associated with a 5- to 10-fold increased risk of esophageal cancer, and the number of cigareues smoked correlates with the presence of dysplasia in the esophageal epilhelium. Epidem10logic data suggest that there are additional, as yet unidentified Tisk factors prevalent in cenain geographical regions of the world (Chlna. lran, and South Afnca). Reflux esophagitis (choice E) leads LO Banett esophagus and adenocarcinoma. Afiaw},.'in (choice B) is a well-knmvn hepatotoxin linked to the development of hepatocellular carcinoma. Herpetic esophagi lis (choice C) frequently occurs in immunocompromised persons bm is not associated with the development of carcinoma. ***Diagnosis: Esophageal cancer
12 A 50-year-old obese man (BNri = 32 kglm1) comes to the physician complaining of indigestion after meals, bloating, and heattbum. Vital signs are normal. A CT scan of the abdomen reveals a hiatal hernia of the esophagus. Endoscopic biopsy shows thickening of the basal layer of the squamous epithelium, upward extension of the papillae of the lamina propria, and an increased number of neutrophils and lymphocytes. Which of lhe following is the most likely diagnosis? ***(A) Esophageal varices ***(B) Mallory-Weiss syndrome ***(C) Reflu).; esophagitis ***(D) Schatzki mucosa] ring ***(E) Squamous cell carcinoma
12 The answer is C: Reflux esophagitis. Esophagitis may be caused by infections, reflux of gasntc j uice, or e;xogenous irritants. Of these, lhe most common type is reflux esophagitis, which is often found in conjunction with a sllding hiatal hernia but may also a1ise through an incompetent lower esophageal sphincter withou t any demonstrable anatomical lesion. Chronic exposure lO stomach juice causes reactive thickening of the squamous epithelium (leukop lakia) and the underly· ing stroma. Areas affected by gastric rdlux are susceptible to mucosal erosions and ulcers which appear as linear vertical streaks. Neutrophils and lymphocytes accumulate in the mucosa. The other choices are not lypical complications of hiatal hernia. ***Diagnosis: Reflux esophagitis, hiatal hernia
13 A 30-year-old man wilh AIDS complains of severe pain on swallowing. Upper Gl endoscopy shows elevated, white plaques on a hyperemic and edematous esophageal mucosa. Which of the following is the most likely diagnosis? ***(A) Barrett esophagus ***(B) Ca11dida esophagitis ***(C) Herpetic esophagitis ***(D) Reflux esophagiris ***(E) Squamous cell carcinoma in situ
13 The answer is B: Candida esophagitis. Candtda esophagitis has become commonplace because of an increasing number of immunocompromised persons. Esophageal candidiasis also occurs in patients with diabe tes and in those receiving antibiotic therapy. The pseudomembranes are composed of hmgal mycelia, fibrin, and necrotic debris. Involvement of deeper layers of the esophageal wall can lead to disseminated candidiasis, as w·ell as fibrosis, which is sometimes severe enough to create esophageal snicmre. Symptoms include dysphagia and odynophagia (pain on swallowing). Herpetic esophagitis (choice C) features mucosal vesicles. The orher choices are not charactenzed by the formation or elevated white plaques on the esophageal mucosa. ***Diagnosis: Infective esophagitis
14 A SB~year~old woman is brought to the emergency departmem 4 hours after vomiLing blood and experiencing bloody stools. The patiem was diagnosed with alcoholic cirrhosis 2 years ago. TI1e patient subsequently goes into shock and expires The hiswlogic appearance of the esophagus at amopsy is shown in the image. Which of the following is the most likely underly~ ing cause of hematemesis and hemawchezia in this patient? ***(A) Alcoholic hepatitis ***(B) Ischemia of the gasuic mucosa ***(C) Mallmy-Weiss syndrome ***(D) Peptic ulcer djsease ***(E) Portal hypertension
14 The answer is E: Portal hypertension. Esophageal vatices are dilated (varicose) veins immediately beneath the mucosa, whkh are prone to rupture and hemorrhage. They arise in the lower third o[ the esophagus, most often in the setting o[ portal hypertension, secondary to cirrhosis. The Lower esophageal veins are linked lO the ponal system through gastroesophageal anastomoses. If the portal blood pressure exceeds a Clitical level, these anastomoses become prom.inem. When varices become greater than 5mm in diameter, they are likely to rupture, in which case life-threatening hemorrhage may ensue. The m.her choices are not associated with bleeding esophageal valiccs. Alcoholic hepatitis (choice A) by itself does not cause varices, bm Long-term alcohol abuse often leads to cirrhosiS and esophageal varices. ***Diagnosis: Esophageal varices, cirrhosis
15 A 3-week-old boy is brought to the physician by his parents, who report that he vomits forcefully immediately after nursing. Physical examination reveals an "olive-like" palpable mass and visible peristaltic movements within the infant's abdomen. What is the most likely cause of projectile vomiting in this infant? ***(A) Appendicitis ***(B) Congenital pyloric stenosis ***(C) Hirschsprung disease ***(D) Meconium ileus ***(E) Tracheoesophageal fistula
15 The answer is B: Congenital pyloric stenosis. Congenital pylonc stenosis is a concentric enlargement of the pyloric canal that obstructs the outlet of the stomach. The disorder is the most common indication for abdominal surgery in the first 6 months ofhfe. Congenital pyloric; stenosis has a familial tendency, and tl1e condition is more common in identical twins than in fraternal ones. The only consistent microscopic abnormality is hypertrophy of the circular muscle coaL. Projectile vomiting is not characteristic of the other choices, particularly in neonates. ***Diagnosis: Congenital pyloric stenosis
16 A 50-year-old woman with long-standing rheumatoid anhriti.c; complains of weakness and fatigue . She states that her srools have recently become black after taking a new nonsteroidal anLi-inOammatmy dmg (NSAlD). Gastroscopy shows numerous superficial, bleeding mucosal defects. Which of the following is tl1e most likely diagnosis? ***(A) Acme erosive gasrriLis ***(B) Earl)' gastric cancer ***(C) Helicobaaer pylori gastritis ***(D) Meneuier disease ***(E) Peptic ulcer disease
16 The answer is A: Acute erosive gastritis. Acu te hemorrhagic gasnilis is charac terized by necrosis of the mucosa and is commonly associaled wilh the intake of aspirin, other NSAIDs, alcohol, or ischemic injury. The facwr common to all forms of acme hemonhagic gastrilis is thougl1l w be the breakdown of the mucosal b anier, which permits acid-induced injury. Mucosal injury causes bleeding from super ficial erosions. Defects in lhe mucosa may extend into the deeper tissues w fom1 an ulcer. The necrosis is accompanied by an acme lnDammatory response and hemorrhage, which may be severe enough to result in exsanguination and hypovolemic shock. The O£her choices are not associated with the use of NSA1Ds. ***Diagnosis: Acme erosive gasLritis
17 A 34-ycar-old man prcsems with a 5-momh histmy of wealmess and fatigue. There is no history of drug or alcohol abuse. A CBC shows megaloblastic anemia and a normal reticu]ocytc coum. Funher laboratmy studies reveal viLamin B11 deficiency. Anemia in this patiem is most likely caused by which of the following? ***(A) Acute erosive gastritis ***(B) Autoimmune gastritis ***(C) Heltcobaaer pylori gasnitis ***(D) Menetrier disease ***(E) Peptic ulcer disease
17 The answer is B: Autoimmune gastritis. Autoimmune gastritis refers to chronic, diffuse inOammatory disease of Lhe stomach that is restricled lo Lhe body and fundus and is associated with other amoimmune phenomena This disorder typically featu res diffuse atrophic gastritis, amibodies to parietal cells and the intrinsic factor, and increased serum gastrin due Lo G-ceU hyperplasia. Immunologic desmtction of parielal cells and antibody targeting of intrinsic factor interfere with imestinal absorption of vitamin B12 . As a result, all lineages of bone manow precursors show asynchronous maturation between the nucleus and cytoplasm (megaloblastic cells), and the peripheral blood displays megaloblastic anemia. Megaloblastic anemia that is caused by malabsorprion of vitamin B11 , occasioned by a deficiency of the inntnsic factor, is referred to as pernicious anemia. The other choices are nor causes of pernicious anem.ia. ***Diagnosis: Autoimmune atroph.ic gastritis, pernicious anemia
18 A 40-year-old woman presents with a 2.-momh history of burning epigastric pain that usually occurs between meals. The pain can be relieved with antacids or food. The patient also reports a recent history of La rry stools. She denies taking aspirin or NSAIDs. Laboratory studies show a microcytic, hypochromic anemia (serum hemoglobin = 8.5 gldl) Gastroscopy reveals a bleeding mucosal defect in the antrum measming 1.5 em in diameter. An endoscopic biopsy shows that the lesion lacks mucosal lining cells and is composed of amorphous, cellular debris and numerous netmophils. Which of the following is the most important factor in the pathogenesis of this patients disease? ***(A) Achlorohydria ***(B) Acute ischemia ***(C) Autoimmunity ***(D) Gastrinoma ***(E) Helicobaaer pylo1i infection
18 The answer is E: Helicobacter pylori infection. Peptic ulcer disease refers to breaks in the mucosa of the stomach and small intestine, principally the proximal duodenum, which are produced by the action of gastric secretions. The pathogenesis of peptic ulcer d.isease is believed Lo involve an underlying chronic gastritis cau..<;ed by H. pylori. This pathogen has been isolated from the gastric antrum or vinually all patients with duodenal ulcers and from about 75% or those with gastric ulcers. H. pylori gastritis is the mosL common type of gasrritis in the United States and is characterized by prominem chronic innammation of the antrum and body of the stomach. In addition to peptic ulcer disease, H. pylon gasnitis is a risk factor for development of gasnic adenocarcinoma and lymphoma. Eradication of H. pylori infection is curative of peptic ulcer disease in most patients. Gastrinoma (choice D) is a rare cause of peptic ulcers. Achlorhydria (choice A) is incorrect because the fom1ation of peptic ulcers requires at least some gasnic acid secretion. ***Diagnosis: Chronic infectious gastritis, peptic ulcer disease, gastric ulcer
19 A 58-year-old woman suffers a massive snake and expires. The sLomach at autopsy is shmvn in the image. Prior Lo her death, this patiem would most likely have exhibited which of the fo llowing? ***(A) Dysphagia ***(B) Hypersecretion of gastric acid ***(C) Melena ***(D) Steatonhea ***(E) Va1iceal bleeding
19 The answer is C: Melena. Peptic ulcers of Lhe stomach and duodenum are estimated to afOict 10% of the population of ·western industrialized countries at some time duting their lives. Peptic ulcers appear as punched ouL, rounded Ltlcers. Erosion through aneries causes bleeding and iron-deficiency anemia. Melena refers to black, tarry stools composed largely of blood from Lhe upper digestive tract that has been processed by the action of gastric juices. Melena is commonly seen in patients who sulfer from chronic peptic ulcer disease. Unlike duodenal ulcers, most patients wilh gastric ulcers secrete normal or decreased amounts of gastric acid {see choice B). ***Diagnosis: Peptic ulcer disease, gastric ulcer
20 A 45-year-old woman presents with a 6-momh history of fatigue and swelling in her neck Physical examinmion shows a goiter. A CBC discloses megaloblastic anemia and a normal reticulocyte count. Additionally, there is an elevated serum level of TSH and amithyroid amibodtes. Needle aspiration of the left lobe of the Lhyroid reveals benign follicular cells and numerous lymphocytes Anemia in this patiem is most likely caused by antibodies directed to which of the following targets? ***(A) Chief cells ***(B) Intrinsic factor ***(C) Paneth cells ***(D) TSH receptor ***(E) Vitamin D
ZO The answer is B: Intrinsic factor. This patient has chronic lymphocytic thyroiditis (Hashimow thyroiditis) and pernicious anemia. Pemidous anemia is a megaloblastic anemia mat is caused by malabsorption of vitamin B12 due to a deficiency of the intrinsic factor. ln many cases, pernicious anemia is associated with other auwimmune diseases (e.g., Hashimoto thyroiditis, Graves disease, Addison disease, or diabetes mellitus type 1). Circulating antibodies lO parietal cells, some of which are cytotoxic in the presence of complement, occur in 90% of patients with pernicious anemia. Two thirds of patients display an antibody to the intrinsic factor that prevents its combination with vitamin B12 , thereby preventing formation of the complex that is later absorbed in the ileum. Half of all patients with pernicious anemia have ci rculati ng antibodies to myroid tissue. ***Diagnosis: Pemicious anemia, Hashimoto thyroiditis
21 A 45-year-old man describes burning epigastric pain 2 to 3 hours after eating. Foods, antacids, and over-Lhe-counter medications provide no relief, and p rescribed inhibitors of acid secretion are only moderaLely effective. Recemly, the patient noticed Lhat his stools were black. Physical examination reveals abdominal tendemess. The blood pressure is 120/SOmmHg in the supine position and 90/50 mm Hg sitting up. The patient complains of lightheadedness upon remrning w a standing position. CBC shows a hemoglobin of 6.3 gldL Endoscopy reveals multiple gastric and duodenal ulce rs. Epigastric pain and anemia are most likely related to a neoplasm arising in which of the following anatomic locations? ***(A) Adrenal medulla ***(B) Ampulla of Vater ***(C) Duodenum ***(D) Esophagus ***(E) Pancreas
21 The answer is E: Pancreas. Zollinger-Ellison syndrome is characterized by unrelenting peptic ulceration in the smmach or duodenum (or even proximal jejunum) by the action of tumor-derived gasuin. Gastrin-producing neuroendocrine rumors (gasuinomas) usually mise in d1e pancreatic islets. Among islet cell rumors, pancreatic gastrinomas are second in frequency only to insulinomas. Duodenum (choice C) Ls incorrecL because only 15% of cases of Zollinger-Ellison syndrome are due to gastrinomas outside the pancreas (e.g., duodenum). Most gastrinomas are malignant. ***Diagnosis: ZoUinger-Ellison s>rndrome, peptic ulcer disease, gasnic ulcer
22 A 60-year-old man presents with an 8-week hiswry of progressive weighr loss, nausea, and upper abdominal pain thaL does noL respond LO amacids or H2-receptor amagonls[S. Laboratory smdies show iron-deficiency anemia. GasLroscopy reveals a crater-like, ulcerated lesion in the antnun, with raised, irregular, and indurated margins. The patient undergoes partial gastrectomy and the surgical spedmen is sho\VJl in the image. Which of the Following is the most likely diagnosis? ***(A) Acme erosive gastritis ***(B) Adenocarcinoma ***(C) Curling ulcer ***(D) Linitis plastica ***(E) Peptic ulcer disease
22 The answer is B: Adenocarcinoma. Adenocarcinoma of the stomach accounts for more than 95% of all malignant gastric twnors. Most patients have metastases by the time they are seen for examination. The most frequent inuial symptom of gastric cancer is weight loss, usually associated \vtth anorexia and nausea. Most patiems complain of epigasnic pain-a symptom LhaL mimics benign gastric ulcer disease, and i..s often relieved by antacids or H2-receptor antagonists. On gross inspection, gasuic cancer appears as a polypoid. fungating, or ulcerated mass, or diffuse infiltration of the stomach wall. This patient has an ulcerating carcinoma. Acute erosive gastritis {choice A) and peptic ulcer disease (choice E) do nm lYl)ically p resent wim rapid weight loss, and these benign ulcers usually do not have heaped-up (raised), ragged margins. Curling ulcers (choice C) occur in severely burned patiems. ***Diagnosis: Gastric adenocarcinoma
23 A 58-year-old woman presents with a 2-month hiswry of abdominal discomfon and dark swols. Physical examination shows pallor bm no evidence of jaundice. Laborarory smdies disclose a microcytic, hypochromic anemia, with a hemoglobin leve 1 of 6. 7 gldl. A barium swallow radiograph reveals a leather boule appearance of the swmach. Microscopic examination shows diffusely infiltrating maUgnam cells, many of which are "signet ring" cells, in the stomach wall. Which of the following is the most Ukely diagnosis? ***(A) Fungating adenocarcinoma ***(B) Gastric leiomyosarcoma ***(C) Gastric lymphoma ***(D) Linitis plastica ***(E) Menenier disease
23 The answer is D: Unitis plastica. Diffuse adenocarcinoma constituteslO% of all swmach cancers. No true tumor mass is seen macroscopically. lnstead, the waH of the stomach is conspicuously thickened and finn, accounting for the radiologic leather boule appearance. ·when the emire stOmach is involved, the term linitis plastica is applied. The invading L1.tmor cells induce extensive fibrosis in the submucosa and muscularis of the stomach wall. Gasu·ic carcinomas typically metastasize w regtonallymph nodes and the live r. Signet ting cells are so named because intracellular mucin displaces the nuclei w the periphery of the tumor cells. Gastric carcinomas and linins plastica, in panicular, have a poor prognosis. The other choices do nm show £he characteristic morphologic appearance of linitis plastica and generally do nor exhibit signel ring cells. ***Diagnosis: Gastric adenocarcinoma
24 A 42.-year-old man presents with long-standing abdomina] pain after meals, whkh is relieved by over-the-counter amacids. The patient has lost 9kg (20lb) in the past year. Physical examination reveals peripheral edema and ascites. La.boratmy studies show decreased serum albumin bm normal serum levels of transaminases and gastrin. Gross and microscopic examination of this patients stomach would most likely show which of the following pathologic changes? ***(A) Atrophic gastritis ***(B) Enla rged rugal folds ***(C) Intestinal metaplasia ***(D) Multiple hemorrhagic ulcers ***(E) Proliferation of neuroendoc1ine cells
24 The answer is B: Enlarged rugal folds. Menerrier disease (hyperplastic hypersecretmy gastropathy) is an uncmm11on disorder of the stomach characterized by enlarged rugae. It is often accompanied by a severe loss of plasma proteins (including albumin) from the alte red gastric mucosa. The disease occurs in two forms: a childhood form due to cytomegalovims infection and an adult form amibuted to overexpression of TGF-a. The folds of the greater curvature in the fundus and body of the stomach (occasionally in the antrum) are increased in height and tl1idmess, forming a convoluted brain-like surface. The other choices do not feature protein-losing enteropathy ***Diagnosis: Menenier disease
25 A 55-year-old woman complains or upper gastrointestinal pain and tarry stools. Upper endoscopy shows a firm, smooth, yellowish submucosal ulcerated mass in the stomach. Gastroscopic biopsy reveals spindle cells wiLh vacuolated cytoplasm. The mass is removed, and me surgical spenmen is shown in the image. Which of the following is the most likely diagnosis' ***(A) Gastric adenocarcinoma ***(B) Gastnc lrmphoma ***(C) Gastrointestinal stromal rumor ***(D) Peptic ulcer ***(E) Tubular adenoma
ZS The answer is C: Gastrointestinal stromal tumor {GIST) . GlSTs are derived from the pacemaker cells of Cajal. They include the vast majority of mesenchyme-derived stromal tumors of the entire gastrointestinal tract- Gastric GISTs are usually submucosal and covered by intact mucosa. Microscopically, the mmors show spindle ceUs with vacuolated cytoplasms. GISTs are considered to be of low malignant pO£ential and are removed surgically. Gastric adenocarcinoma (choice A) does not often dedifferentiate to a spindle cell morphology ***Diagnosis: Gastrointestinal stromal rumor
26 A 56-year-old woman comes r.o the physician after noticing mulliple lumps in her neck. Physical examination shows enlarged and nomender supraclavicular lymph nodes. Upper endoscopy discloses thickening of r.he gasrric mucosa, without an obvious mmor. The resu!LS of gastric biopsy are shown in the image. Which of the following is the most likely diagnosis' ***(A) Adenocarcinoma ***(B) Gasrric lymphoma ***(C) Leiomyosarcoma ***(D) Linitis plastica ***(E) Menetlier disease
26 The answer is B: Gastric lymphoma. Gastric lymphoma is the most common form of extranodallymphoma, accounllng for 20% of all such rumors. Gasuic lymphoma has a considerably better prognosis than gastric carcinoma { 45% 5-year survival). The clinkal symptoms of gastric lymphoma are nonspeci~c and indistinguishable from those of gastric carcinoma (e.g. , weight loss, dyspepsia, and abdominal pain). Most gastric lymphomas are low-grade B-cell neoplasms of the MALToma (mucosa-associated lymphoid tissue) type, which atise in the seLting of chronic Helicobacter pylon gastritis with l>rrnphoid hyperplasia. Some of these lymphomas regress after eradication of the infection. The other choices do not demonstraLe the lymphoid lesion depicted here. ***Diagnosis: Gastric lymphoma, MALToma
27 A 23-year-old woman with a history of an eating disorder complains of vonUtlng, nausea, and severe abdominal pain. Physical examination shows abdominal distension and constipation. An X-ray film of the abdomen reveals air-nuid levels and a hyperlucem shadow at the epigastric area. The material obstructing the gastrointestinal tract is removed surgically and shown. Which of the following is the most likely diagnosis? ***(A) Diverticulum of the stomach ***(B) Gastric lymphoma ***(C) Phytobezoar ***(D) Trichobezoar ***(E) Volvulus of the stomach
27 The answer is D: Trichobezoar. &zoars are foreign bodies in the stomach that are composed of food or hair that have been altered by the digestive process. The mass removed from the stomach in this patiem is a hairball (uichobezoar) within a gelatinous matrL"X. Ttichobezoar is usually seen in long-haired girls or young women who eat their own hair as a nervous habit (uichotiJ1omania; also called URapunzel" syndrome). Such a uichobezoar may grow by accretion to fonn a complete cast of the stomach. Strands of hair may e;o,.1:end imo the bowel as far as the transverse colon (Rapunzel syndrome). Phytobezoars (choice C) are concretions of plant matetial, which usually occur in persons with conditions that interfere with gasoic empLying. ***Diagnosis: Tnchobezoar, bezoar, Rapunzel syndrome
28 A 60-year-old man presems with epigastric pain alter meals, with some nausea and vomiting. A burning sensation in the midepigastrium is relieved by antacids and H2 antagonists. Upper endoscopy demonstrates paired ulcers on both walls of the proximal duodenum. Which of the following represenr.s rh.e most common complication of this patiems duodenal disease? ***(A) Bleeding ***(B) Malignam transformation ***(C) Obstmction ***(D) Perforation ***(E) Peti wnit..is
28 The answer is A: Bleeding. Bleed ing is the most common complication of peptic ulcer disease, occurring in abouL 20% of patients. Chronic blood loss due to occult bleed ing is often a feature of peptic ulcers, whereas massive b leeding occurs less often. Perfo ration (choice D) is a serious complication L.hat occurs in 5% of patients. Perforating ulcers are commonly encountered in Lhe duodenum. Duodenal peptic ulce rs do not undergo malignam transformation (choice B). The other choices are uncommon. Diseases associated with peptic ulcers include cirrhosis, chronic renal failure, hereditary endocrine syndromes (MEN-1), n 1- amitrypsin deficiency, and chronic pulmonary disease. ***Diagnosis: Duodenal ulcer, peptic ulcer disease
29 A 45-year-old woman presents with sudden attacks of wheezing, shortness of breath, and episodic hot flashes. She also reports abdominal cramps and diarrhea. Physical examination shows facial redness, pitting edema of the lower legs, and a mummr of tricuspid regurgitation. A 24-hour urine specimen contains elevated levels of 5-hydroxyindoleacetic acid (5-HIAA) ACT scan of the abdomen demonstrates muldple 1- £O 2-cm nodules in distal ileum. A small bowel resection is perfonned (show'll in the image). Tl1e arrows point to submucosal tumors. Microscopic examination shows nests of cells with round and uniform nuclei. Which of the following is the most Hkely diagnosis? ***(A) Carcinoid rumor ***(B) Mediterranean intestinal lymphoma ***(C) Mucosa-associated lymphoid tissue (MALT) lymphoma ***(D) Peutz-jeghers syndrome ***(E) Whipple disease
29 The answer i.s A: Carcinoid tumor. Carcmoid rumors are low-grade malignam neoplasms composed of neuroendocrine cells, which usually show considerable nuclear unifonnity. They are most commonly located in the submucosa of the intestines (appendix, terminal ileum, and recmm). Carcinoids are distinguished fTom lmestinal carcinomas based on their location, histologic featu res, malignant potential, endocrine activity, and clinical features. Carcinoid syndrome is a systemic paraneoplastic disease caused by the release of hormones from carcinoid tumors into ve11ous blood. Clinical features of carcinoid rumors (e.g., flushing, bronchial wheezing, watery diarrhea, and abdominal colic) are presumably caused by the release of serotonin, bradykinin, and histamLne. Release of n1mor secretions from hepatic metastases leads Lo the formadon of fibrous plaques in the tricuspid and pulmonic valves and may result in tricuspid insufficiency or pulmonic stenosis. The other choices are not associated with secretion of 5-HIAA acid or mher neuroendocrine peptides. ***Diagnosis: Carcinoid syndrome
30 A 5-year-old girl is brought to the physician after her parents noticed red blood in her stool. Physical examination reveals mucocutaneous pigmentation. Small bowel rad iography discloses multiple, small- LO medium-sized polyps tha t are mag• nosed pathologically as hamartomas. Which of the following is the most likely diagnosis? ***(A) Congenital teratoma ***(B) Hyperplastic polyp ***(C) Peutz-j egh ers polyp ***(D) Tubular adenoma ***(E) Vi1lous adenoma
30 The answer is C: Peutz.Jeghers polyp. Peutz-Jeghers syndrome is an autosomal dominant, hereditary disorder characterized by intestinal hamartomatous polyps and mucocutaneous melanin pigmentation, which is particularly evident on Lhe face, buccal mucosa, hands, feeL, and perianal and genital regions. The polyps seen in Peutz-Jeghers syndrome are hamartomatous, characterized by a branching network of smooth muscle fibe rs continuous with the muscularis mucosa that suppon the glandular epithelium of the polyp. Congenital teramma (choice A) does not involve the imestine. The oth er choices are principally colome polyps that derive from the luminal epithelium. ***Diagnosis: Gastroimestinal polyp, Peutz-Jeghers polyp
31 A 55-year-old man undergoes routine colonoscopy. A small, raised, mucosal nodule measuring 0.4cm in diameter is idend fied in the rectum and resected_ The surgical specimen is shown in the image. Microscopic examination reveals goblet cells and absorptive cells wi.rh exaggerated crypt architecture, bm no signs of nuclear atypia Which of the following is the most likely diagnosis? . - ***(A) Adenocarcinoma ***(B) Hyperplastic polyp ***(C) InOammatory polyp ***(D) Peutz-jeghers polyp ***(E) VLllous adenoma
31 The answer is B: Hyperplastic polyp. Hyperplastic polyps are small, sessile mucosal excrescences that display exaggerated crY1X architecture. They are the most common polypoid lesions of the colon and are particularly fTequent in the recrum. The crypts of hyperplastic polyps are elongated and may exhibit cystic dilations. The epithelium is composed of goblet cells and absorptive cells, \vithout any dysplasia. There are no dysplastic features indicative of adenocarcinoma (ch oice A). Villous adenomas (choice E) are considerably larger and exhibit prominent Lhin , t.'lll, finger like processes. PeuLZJeghers polyps (choice D) are hamartomatous. ***Diagnosis: Gastrointestinal polyp, hyperplastic polyp
32 A 65-year-old woman undergoes routine colonoscopy. Duling the procedure, a 2-cm mass is identified in the rectosigmoid region and resected. The surgical specimen is shovm in the image. Microscopic examination shows inegular crypts bned by pseudostratified epithelium with hyperchromatic nuc1ei, without dysplastic features. Which of the following is the most likely diagnosis for this patient's colonic lesion7 ***(A) Adenocarcinoma ***(B) Carcinoid rumor ***(C) Hyperplastic polyp ***(D) Tubular adenoma ***(E) Villous adenoma
32 The answer is D: Tubular adenoma. Tubular adenomas constitute two thirds of benign colonic adenomas. They are cypically smomh-surface lesions, usually less than 2 em in diameter, and often have a stalk. Microscopically, mbular adenomas exhibit closely packed epithelial tubules , which may be uniform or irregular v.rith excessive branching. Dysplasia and carcinoma often develop in mbular adenomas. As long as dysplastic foci remain confined to the polyp mucosa, the lesion is almost always cured by resection. Adenocarcinoma (choice A) is incorrect because the lesion does not have dysplastic feamres. Pseudostratified epithelium is not a feature of carcinoid tumor (choice B) or hyperplastic polyp (choice C). The incorrect choices do not typically exhibit a stalk. ***Diagnosis: Gastrointestinal polyp, tubular adenoma
33 A 63-year-old woman complains of rectal bleeding of 1 week in duration. Laboratory studies show hypochromic, microcytic anemia (hemoglobin = 7 6gldL and MCV = 70J.tm 3). Colonoscopy reveals a large polypoid mass, which is removed (surgical specimen shown in the image). The arrow points w a malignant mmor. The patient asks about the relative 1isk of cancer arising in various cypes of gasLromtestinal polyps. Which or the follO\Ving types of colonic polyps is most likely w undergo malignant trans fonnarion? ***(A) Hyperplastic polyp ***(B) Lymphoid polyp ***(C) Peutz-Jeghers polyp ***(D) Tubular adenoma ***(E) Villous adenoma
33 The answer is E: Villous adenoma. These polyps comprise one third of colonic adenomas and are found p redominantly in the rectosigmoid region. They are typically large, broadbased, elevated lesions that display a shaggy, cauliflower-like surface. Microscopically, villous adenomas are composed of thin, tall, fi.nge rlike processes, which superfici.ally resemble the villi of small intestine. Compared to mbular adenomas (choice D), villous adenomas more frequently comain foci of carcinoma. Hyperplastic polyps (choice A) have a much lower 1isk for malignant transformation. ***Diagnosis: Gastrointestinal polyp, villous adenoma
34 A 59-ye<ar-old man complains of progressive weakness. His fli ends have noticed that he has become pale, and he reports that his stools are tinged with blood. On abdominal palpation, there is fullness in the right lower quadrant. U!boratory studies show iron-deficiency anemia, with a hemoglobin level of 7.4gldL. Stool specimens are positive for occult blood. Colonoscopy n:veals an elevated and centrally ulcerated les ion of the sigmoid colon. The biopsy is shown ln the image. Which of the following is the most likely diagnosis? ***(A) Adenocarcinoma ***(B) Carcinoid tumor ***(C) Gastrointestinal stromal rumor ***(D) Lymphoma ***(E) Mucinous cystadenoma
34 The answer is A: Adenocarcinoma. Adenocarcinoma of the recrum or sigmoid colon often presents as a circumferential mass nanowing the imestinal lumen. The gross appearance of the colorectal cancer is sin1.Uar to that seen elsewhere in the gastmimestinal u·act. The most important risk facwrs associated with the developmem of colonic adenocarcinoma are age, pnor colorectal cancer, ulcerative colitis, genetic factors, and perhaps diet. Colorectal cancer invades lymphatic cham1els and initially involves the lymph nodes immediately underlying the mmor. As the tctmor gmws, the most common sign is occult blood in feces. Btight red blood more often occurs in distal lesions. In either case, bleeding typically causes iron-deficiency anemia. Choices B, C, and D are p1indpally lesions of the intestinal wall. Choice E {mucinous cystadenoma) is an ovarian rumor. ***Diagnosis: Colorectal cancer, adenocarcinoma of colon
35 A portion of the large bowel was removed from a 34-year-old man with a familial disease that affects his gasrroimeslinal tract. The surgical specimen is shown in the image. This patient most likely carries a gennline mutation in which of the following genes? ***(A) APC ***(B) C-myc ***(C) DCC ***(D) p53 ***(E) Ras
35 The answer is A: APC. The photogTaph shows numerous adenomas of the colon, consistent with familial adenomaLOus polyposis (FAP), also termed adenomatous polyposis coU (APC). This autosomal dominam inherited disease accounts for about 1% of colorec tal cancers. It is characterized by the progressive development of innumerable adenoma taus polyps of the colorectum, particularly in the rectosigmoid region. Gem1line mutations in the APC gene, a putative lUmor suppressor gene, are responsible for FAP. Carcinoma of the colon and rectum is inevitable in these patients, and the mean age of onset is 40 years. The DCC gene ("deleted in colon cancer"cholce C) is a putative tumor suppressor gene tl1at is often missing in colorectal cancers. Activating mutations of the ras protooncogene (choice E) occur early in mbular adenomas of the colon. ***Diagnosis: Adenomatous polyposis coli
36 A 65-year-old woman presents with a 3-month history of diarrhea and abdominal pain. She has lost 9 kg (20 lb) in the past 6 months. The patient had two benign colonic polyps removed 3 years ago. Laboratory studies reveal mild iron-deficiency anemia, and swol specimens are positive for occult blood. Sigmoidoscopy demonstrates an ulcerated mass, and a biopsy shows malignant glands. A segment of the colon is resected, and the surgical specimen is shown in the image. Based on current models of colonic carcinogenesis, which of the following genes was most likely mutated in the transi tion from benign adenoma to carcinoma Ln thLs patient? ***(A) BRCAl ***(B) C-myc ***(C) p53 ***(D) Ras ***(E) VHL
36 The answer is C: p53. In most cases of colorectal carcinoma, lt has been estimated that a minimum of eight LO ten mutational events must accumulate before the development of invasive cancer. This process is initiated in morphologically normal mucosa, proceeds through an adenomatous precursor, and terminates as invasive adenocarcinoma. The APC gene is conside red to play an important role in the early development of most colorectal neoplasms, whereas mutations in the p53 tumor suppressor gene are thought to participate in the late transition from adenoma to carcinoma. BRCAl (choice A) has been implicated in the pathogenesis of breast and ovarian cancers. VHL (choice E) has been incriminated in the pathogenesis of clear cell renal cell carcinoma. ***Diagnosis: Adenocarcinoma of colon
37 A 60-year-old woman complains of increasing abdominal ginh of 4 weeks in duration. Physical examination discloses ascites, and cytologic examination of the Ouid reveals malignant cells. Exploramry laparotomy shows multiple tumor nodules on the serosal surface of the intestines. Which of the following is the most likely diagnosis? ***(A) Carcinoid tumor ***(B) Gastrointestinal stromal rumor ***(C) Liposarcoma ***(D) MALToma ***(E) Metastatic car-cinoma
37 The answer is E: Metastatic carcinoma. Metastatic carcinoma is by far the most common malignam disorder affecting the peritoneum. Ovarian, gastric, and pancreatic carcinomas are particularly likely to seed the peritoneum, but any intra-abdominal carcinoma can spread to the peritoneum. Metastatic carcinoma to Lhe abdomen presents in the fom1 of multiple serosal nodules and ascites fluid that contains malignant cells. ***Diagnosis: Metastatic carcinoma
38 A 34-year-old man -with AID$ presents with a 3-month history of constipation and lower abdominal pain. The patiem has a history of chronic diarrhea and persistent cough Recenuy, he noticed blood in his stooL Laboratory studies reveal mild irondeficiency anemia. $tool specimens are positive for occult blood. A CBC shows a CD4 count of less than 50/)lL Sigmoidoscopy discloses a mass in the rectosigmoid region. ln addition to B-cell lymphoma, this patient is at increased risk of developing which of l11e following mmors of the gasu·ointestinal (GL) n·act7 ***(A) Carcinoid rumor ***(B) Colonic adenocarcinoma ***(C) Kaposi sarcoma ***(D) Leiomyosarcoma ***(E) Melanoma
38 The answer is C: Kaposi sarcoma. Kaposi sarcoma of Lhe Gl tract is found almost exclusively in patients with AIDS. One third to one half of AIDS patients, with cutaneous Kaposi sarcoma, exhibit involvemem of the GI tract. In mosL patients, intestinal Kaposi sarcoma does not lead to sympwms, although Gl bleeding, obstmction, and malabsorption have been reported. The other choices are not specifically associated with AIDS. ***Diagnosis: AIDS, Kaposi sarcoma
39 A 2 7 -year-old woman presents \Vith a 9-month history of bloody diarrhea and crampy abdominal pain. Three weeks ago, she noticed that her left knee was swollen, red, and painful Her temperature is 38"C (lOl " f ), respirations are 32 per min me, and blood pressure is 130/90 mmHg. Abdominal palpation reveals tenderness over the left lower quadrant. Laboratory studies show moderate anemia, \vith a hemoglobin level of 9.3 glclL Microscopic examination of me swol reveals numerous red and wrute blood cells. A dtffusely red , bleeding, friable colonic mucosa is visualized by colonoscopy. The colon is subsequently removed and the surgical specimen is shown in the image. Which of the following is the most likely diagnosis? ***(A) Adenocarcinoma ***(B) Carcinoid tumor ***(C) Crohn disease ***(D) Pseudomembranous colitis ***(E) Ulceralive colitis
39 The answer is E: Ulcerative colitis. Ulcerative colitis is an inflammatory disease of the large intestine characterized by chronic diarrhea and rectal bleeding. 1t is associated wi.th a pattern of remission and exacerbations and the possibility of serious local and systemic complications. The disorder occurs principally, bm not exclusively, in young adults. Ulc erativ-e colitis is essentially a disease of the mucosa. The process extends from the rectum for a variable distance proximaUy and is limited to the colon and rectum. Pseudomembranous colitis (choice D) is usually a complication of antibiotic therapy, and the mucosal surface of the colon is covered by raised, irregular plaques composed of necrotic debris and an acme inflammatory exudate. Crohn disease (choice C) typically affects the colon i.n a patchy distri bution wi!.h transmural inflammation. ***Diagnosis: Ulcerative colitis
40 The patient described in Question 39 is at increased risk of developing which of the following comp1icaLions? ***(A) Adenocarcinoma ***(B) Fismla ***(C) Granulomatous lymphadenitis ***(D) Transmural inflammation ***(E) Volvulus
40 The answer is A: Adenocarcinoma. Patients with longstanding ulcerative colitis have a higher risk of developing colorectal cancer (adenocarcinoma) than does the general population. This risk is related to the extent of colorectal involvemem and the duration of the inOammawry process. Thus, people with involvement of the entire colon are at the greatest risk of developing colorectal cancer. Intestinal fistula (choice B) is a complication of Crohn disease. Ulcerations in ulcerative colitis are largely confined to the mucosa (not transmural, choke D). ***Diagnosis: Ulcerative colitis
41 A 44-year-old woman complains of having yeUow eyes, dark urine, and recurrent fever for about 3 months. She has a long hiswry of chrome diarrhea. On physical examination, the pauent lS d1i.n and jaundiced. The liver edge descends lcm below the right costal margin and is nontender. Laboratory smdies show elevated serum bilimbin of 3.8 mgldl, normal levels of AST and ALI, and an elevated level of alkaline phosphatase ( 440 Uldl). Endoscopic retrograde cholangiopancreatography demonstrates a beaded appearance of the extrahepatic biliary nee. Which of the following is the most likely underlying cause of diarrhea in this patient? ***(A) Amebiasis ***(B) Amyloidosis ***(C) Carcinoma of the ampulla of Vater ***(D) Celiac spme ***(E) Ulcerative coULis
41 The correct answer is E: Ulcerative colitis. Ptimaty sclerosing cholangitis (PSC) is characterized by innammmion and obliterative fibrosis or inn·ahepatic and exuahepalic bile ducts. Approximately 70% o[ patients with PSC have longstanding ulcerative colitis. ahhough the prevalence of PSC in such patients is only 4%. The clinicopathologic findings are complemented by a characteristic radiographic appearance of a beaded biliary tree, representing sporadic strictures. The other ch oices are not associated with PSC. ***Diagnosis: Prima1y sclerosing cholangitis, ulcerative colitis
42 A 25-year-old woman is brought to the emergency room with symptoms of acute intestinal obstruction. The patient has an 8-month his LOry o fblood-tinged diarrhea and cramping abdominal pain. Her temperature is 38°C (101 °F), and respirations are 3 7 per minute. There is abdominal tendemess to palpation. Laboratory studies show moderate anemia, with serum hemoglobi11 of 9.3 gldL Microscopic e-xamination of the stoo] reveals numerous RBCs and WBCs. A CT scan of the abdomen shows massive distention of the transverse colon. Which of the fo1lo\ving is lhe most likely underlying cause of this patients colonic disorder? ***(A) Adenocarcinoma ***(B) Carcinoid tumor ***(C) Crohn disease ***(D) Pseudomembranous colitis ***(E) Ulcerative colitis
42 The answer is E: Ulcerative colitis. Local complications of ulcerative colitis include toxic megacolon, perforation, inflammatory pseudopolyps, hemorrhage, and adenocarcinoma. The other choices are noL associated with the development of Loxic megacolon. ***Diagnosis: To;-..ic megacolon, ulcerative colitis
43 A 21-year-old man is brought to the emergency room with symptoms of acme imesr:inal obstruction. His temperamre ls 38°C (101 O"f), respirations are 25 per minme, and blood pressure is 120/80 mm Hg. Physical ~\':ami nation reveals a mass in the tight lower abdominal quadrant. The patiem subsequently undergoes surgery, and a segmemal lesion involving the terminal tleum is resected (shO\vn in the image). Which of the following is the most likely diagnosis? ***(A) Adenocarcinoma ***(B) Carcinoid tumor ***(C) Crohn disease ***(D) Pseudomembranous colitis ***(E) Ulcerative colilis
43 The answer is C: (rohn disease. Crohn disease is a transmural, chronic innammatory clisease that may affect any part of the digestive tract buL occurs principally in the distal small intestine and occasionally the 1igh t colon. lt has variously been referred to as tem1inal ileitis and regional ileitis when it involves the ileum and granulomatous colitis when it principally affects the colon. Skip lesions are common. The affected mucosa has a characteristic "cobblestone" appearance (show11 in the image) due to the presence of linear ulcerations and edema, and inflammation of the imervening tissue. The other choices do not show the characteristic cobblestone morphology that is seen in this case. ***Diagnosis: Crohn disease
44 A 24-year-old man is brought w the emergency room with symptoms of acute intestinal obstruction. His temperature is 38"C {101 "F), respirations are 25 per minme, and blood pressure is 120/80 nm1 Hg. Physical examination reveals a mass in the right lower abdominal quadrant. At laparoscopy, there are numerous small bowel strictures and a fistula extending illto a loop o[ small bowel. Which of the foLlowing is the most likely dI. agnosi.S ?. ***(A) Adenocarcinoma ***(B) Carcinoid tumor ***(C) Crohn disease ***(D) Pseudomembranous colitis ***(E) Ulce rative colitis
44 The answer is C: Crohn disease. Crohn disease is a transmural, chronic inflammatory disease that may affect any pan of the digestive tTact. Intestinal obstruction and fistu las are the most common intestinal complications of Crohn disease. Occasionally, free perforation of the bowel occurs. The risk of small bowel cancer is increased at least threefold in patients with Crolm disease. Pseudomembranous colitis (choice D) and ulcerative colitis (choice E) are not associated with fistula formation. Adenocarcinoma (choice A) rarely, if ever, arises in the temtinal ileum. ***Diagnosis: Crohn disease
45 A 30-year-old woman presems wilh 2 days of abdom]nal cramping and diarrhea. Her temperature is 38"C (lOl"F), respirations are 32 per minute, and blood pressure is 100/65 mm Hg. Stool culture shows a toxigenic Eschelichfa col1 infection. 'vVhich of the following best explains the pathogenicity of this organism in this patient? ***(A) Destruction of Peyer patches ***(B) Invasion of the mucosa of the colon ***(C) Invasion of the mucosa of the ileum ***(D) Stinmlation of acute inflammation in the superficial bowel mucosa ***(E) Stimulation of fluid transport into the lumen of the inlestine
45 The answer is E: Stimulation of fluid transport into the lumen of the intestine. The most significant factor in infecnous diarrhea is increased intestinal secretion, stimulated by bacterial wxins and enteric hormones. Organisms that produce diarrhea by secreting specific toxins include Vibno c/1Qiera and to:\igenic strains of E. coli . There is minimal or absent damage to lhe intestinal mucosa (choices A lO D) in cases of toxigenic diarrhea. The organisms remain attached to the imesti11al mucosa and elaborate toxins, which stimulate the transmucosal transport of fluicllnto the lumen, resulting in dianhea. Patients may become severely dehydrated, particularly m the case of cholera. ***Diagnosis: Bacterial dianhea
46 A 1-year-old girl is brought to the emergency room by her parents who report that s he had a fever and diarrhea for 3 days. The childs temperature is 38°C (101 °F). The CBC shows a normal WBC count and increased hematocrit. 'vVhich of the following microorganisms is the most likely cause of diarrhea in this young child? ***(A) Cywmegalovirus ***(B) Rotavirus ***(C) Salmonella typhi ***(D) Slt1gella dysenteriae ***(E) Yersiniajcjuni
46 The answer is B: Rotavirus. Ror.avirus is the most common cause of infantile diatThea and can be demonstrated in duodenal biopsy specimens in half the cases of acme diarrhea in hospitaUzed children under the age of 2 years. Choices C, D, and E can cause diarrhea, but are llncommon in developed countries. ***Diagnosis: Viral diarrhea
47 A 70-year-old man is rushed w Lhe emergency room complaining of severe abdominal pain and rectal bleeding of 2 hours in duration. He has a history or coronary artery disease. Bowel sounds are absent on physical examination. A CT scan of the abdomen shows distention of the stomach and air-iluid levels in the small bowel. Abdominal pain and bleeding in this patient most Hkely involved acute occlusion of which of the following arte ries? ***(A) Celiac nun!< ***(B) Gast roduodenal artery ***(C) lnferior mesenteric arte1y ***(D) lnfetior rectal anery ***(E) Superior mesenteric artery
47 The answer is E: Superior mesenteric artery. Sudden occlusion of a large artery by thrombosis or embolization leads to small bowel infarction before collateral circulation comes into play. Depending on the size o[ the artery, infarction may be segmental or may lead to gangrene of virtually the entire small bowel. The small intestine, which is supplied by the superior mesenteric artery, is more likely to suffer transmural hemonhagic infarction than the large intestine. The inferior mesenteric artery (choice C), which supplies blood to the colon, is a less common site for atherosclerotic ernbolization than the superior mesenteric artery because of the smaller size o[ the Iauer and its oblique origin from the aorta. Pathologically, ischemic bowel disease is classified as occlusive or nonocclusive. Occlusive disease is caused by thrombi or emboli, whereas nonocclusi\'e disease is secondary LO arterial nanowing by atherosclerosis. The other choices are not speclfically associated with small bowel ischemia. ***Diagnosis: Ischemic colitis
48 A 16-year-old girl complains of chronic abdominal distention, flarulence, and diarrhea after drinking milk. Elimination of milk and other dairy products h-om the patients cUet reHeves th ese sympwms. This example of malabsorption is caused by a functional defidency of which of the following enzymes assodated with the intestinal brush border membrane? ***(A) Disaccharidase ***(B) Glycogen phosphorylase ***(C) Hyaluronidase ***(D) Mannosidase ***(E) Sphingomyelinase
48 The answer is A: Disaccharidase. Acquired lactase deficiency is a widespread disorder of carbohydrate absorption. The symptoms of this disease typically begin in adolescence, when patients complain of flamlence and diarrhea after the ingestion of dairy products. Lactose is one of the most common disaccharides in dairy products. The imestinal brush border contains disaccharidases Lhat are imponam for cleavage of lactose to free glucose and galactose for absorption. Congenital lactase deficiency is rare but may be lethal if not recognlzed. The other choices do not hydrolyze lactose. ***Diagnosis: Lactose mtolerance
49 A 2-year-old girl with a history o[ chronic constipation since binh is broughL to the emergency room because of nausea and vomiting. Physical examination shows marked abdominal distension. Abdominal mdiography reveals distended bowel loops with a paucity of air in the recmm. A rectal biopsy shows an absen ce of ganglion cells. Which of the following is the most likely diagnosis? ***(A) Acquired megacolon ***(B) Anorectal stenosis ***(C) Hirschsprung disease ***(D) 1m perforate anus ***(E) Rectal atresia
49 The an.swer is C: Hirschsprung disease. Hirschsprung disease, also referred to as congenital megacolon, results from a congenital defect in the innervation of the large inLestine, usually in the rectum. Severe chronic constipation is typical. Marked dilation of the colon occurs proximal to the stenoLic recmm, with clinical signs of intestinal obsu·uction. The other choices are not associated with loss of ganglion cells. ***Diagnosis: Hirschspn.mg disease
50 A 25-year-old woman presems \Vith persistent bloody diarrhea of 4 weeks' duration. She has experienced severe abdominal cramping for the past} days. Hertemperatu re is 38°C (101 "F), respirations are 22 per minute, and blood pressure is 120/70mmHg. Physical examination reveals abdominal tenderness and mild abdominal distension. Bowel sounds ru·e diminished. LaboratOiy smdies show mild hypoclu·omic, nonnocydc anemia. Stool cultures are negative for pathogens, and no ova or parasites are detected. A blood test for Closcridium d!fficiie toxin is negative. Rectosigmoidoscopy shows hemorrhagic mucosal lesions in the distal colorectal region. A biopsy of the colon reveals c typt abscesses, basal lymphoplasmacywsis and ctypt distortion. Which of the following represents the most common exu aimesLinal manifestation of the colonic disorder in this patient? ***(A) Arthritis ***(B) Cysdtis ***(C) Gasniris ***(D) Pancreatitis ***(E) Sepsis
50 The answer is A: Arthritis. The case history is indicative of ulcerative colitis. Arthritis is seen in 25% of patients with ulcerative colitis. Uveitis and skin lesions develop in apprm.imately 10% of patiems. The most common cutaneous lesions are erythema nodosum and pyodenna gangrenosum. Liver disease occurs in about 4% of patien lS, the most common pathologic findings being pericholangitis and fatty liver. The other choices do not represent extraimestinal manifestations of ulcerative coliti'i. ***Diagnosis: U1cerative colitis, arthlitis
51 A 10-year-old boy is brought to the emergency room after 48 hours of £ever and severe abdomb1al pain. t-Ie had developed edema o£ the legs several weeks previously. The temperature on admis..c;;ion i.s 38. 7°( (103°F). Physical examination shows rebound tenderness, guarding, and ascites. An abdominal tap returns numerous segmemcd neun·ophils. This childs spomaneous bacte1ial pe1itonitis is most often associated with whkh of the following underlyi ng conditions? ***(A) Celiac spme ***(B) Diverticulosis ***(C) Hirschsprung disease ***(D) Meconium ileus ***(E) Nephrotic syndrome
51 The answer is E: Nephrotic syndrome. Most cases of peritonitis are caused by bacteria that emer the abdominal cavi ty from a perforated viscus or through an abdominal wound. However, spomaneous bacterial pe1itonitis occurs in children withom an obvious perforation. Most of these patients have a nephrotic syndrome and a systemic infection that seeds the ascitic nuid \.vilh bacteria. ln aduiLS, spomaneous bacterial peritonitis is a feared complication of cirrhosis_ The other choices are not associated with the development of spontaneous bacte1ial periwnitis. ***Diagnosis: Spontaneous bacterial peritonitis, nephrotic syndrome
52 A 74-year-old woman presents with 3 weeks of left lower quadrant abdominal pain, changes in bowel habits, and mtcrnlittcnt fever. Her temperature is 38°C (101 °F), respirations arc 19 per minute, and blood pressure is 130/80 mmHg. Physical examinadon shows left lower quadram tenderness. A CBC reveals neutrophilia. An abdominal-pelvic uhrasound examination is no1maL Which of the [allowing is the most likely diagnosis? ***(A) Appendicitis ***(B) Divenicullds ***(C) Ovarian card noma ***(D) Renal colic ***(E) Uterine leiomyoma
52 The answer is 8: Diverticulitis . Diverticular disease refers to two entities: a condition tenned diverticulosis and an inflammatory complication cailed cliveniculitis. Diverticulosis ls generally asymptomatic. Diveniculltis resulLS from the itTitadon caused by retained fecal material that obstructs the lumen of a diverticulum. CHnically, the most common symptoms of diverticulitls usuall}' follow microscopic or gross perforation of Lhe dive11iculum. Diverticula are most conm1on in the sigmmd colon, which is affected in 95% of cases. PerilOJ1ills and sepsis are serious complications. Appendicitls (choice A) usually presems with right lower quadrant pain. None of the other choices presents with gasrroimestinal symptoms and fever. ***Diagnosis: Diveniculitis
53 A 9-year-old boy undergoes emergency surge1y l'or presumptive acute appendicitis, During the operation, the surgeon notices that the boy's ileocecal lymph nodes are enlarged and matted together. One of the nodes is sent for a frozen section. The pathologist finds granulomatous inflammation with central necrosis. The specimen is cultured. Which of the following pathogens is the most likely cause of lymphadenopathy in this patient? ***(A) Campylobacterjt:jUnl ***(B) Shtgella dysenLeriae ***(C) Toxigenic E. coli ***(D) Vibrio cholerae ***(E) Yersinia enterocolitfca
53 The answer is E: Yersinla enterocolitica. Yerslnia can cause mesenteric adenitis and pain in the right lower quadrant (pseudoappendicitis). infected children not infrequently have undergone laparotomy because of a mistaken diagnosis of appendicitis. The lymph nodes show a gran ulomarous inflammation. Other symptoms are diarrhea, reactive anhritis. erythema nodosum, and septicemia. The other choices are not associated with the development of mesenteric adenitis and do not present with symptoms that mimic acute appendicitis. ***Diagnosis: Yersinia lymphadenitis
54 A 36-year-old man presents \¥ith fever and painful joints for 2 weeks. Physical examination shows skin pigmentation, glossitis, angular cheilitis, and generalized lymphadenopathy. The patient has lost 9 kg (20 lb) over the past 6 momhs. He reports that his swols are pale and [oul smelling. Blood cultures are negative. The patient is started on antibiotic therapy and exhibits remarkable clinical improvement. Biopsy of the small intestine shows marked distortion of the intesti11al villi, and a peliodJc acid-Schiff stain reveals large, foamy macrophages filled with glycoprotein-1ich gran· ules (shown in the image). Which of the following is the most likely diagnosis? ***(A) Angiod}•splasia o[ ileum ***(B) Croh:n disease ***(C) Menenier disease ***(D) Peutz-Jeghers syndrome ***(E) Wh1pple disease
54 The answer is E: Whipple disease. Whipple d isease is a rare in fectious disorder of the small intestine in which malabsorption is the most prominent feature. The disorder typically features infiltration of the small bowel mucosa by macrophages that are packed \vith small, rod-shaped bacilli (Tropheryma whippelii). l nfi.ltrates o[ macrophages containing bacilli ma>' be found in other organs, including regional lymph nodes and the heart. The other chokes do not feature these distinctive aggregates of foamy macrophages. ***Diagnosis: Whipple disease
55 Which of me following cellular/biochemkal mechanisms best explains the pathogenesis of malabsoqJtion in the patient described in Question 5-!-? ***(A) Bile salt inactivation ***(B) Blind loop syndrome ***(C) Extrahepatic cholestasis ***(D) Impaired mucosal function ***(E) Obstruction of the common bile duct
55 The answer is D: Impaired mucosal function. Normal imestinaJ absorpuon is characterized by a luminal phase and an intestinal phase. The intestinal phase includes those processes that occur in epithelial cells and o·ansport channels. lnjUl)' to the mucosa in pauents \'lith Whipple disease (secondary to inflammation) results in impaired transport of nurrien ts thxough the imestinaJ wall. Histologic examination of the small intestine reveals Oat, thickened villi and extensive infiltration of the lamina propria with foamy macrophages. The other choices pertain to luminal phase processes that are unaffected in patiems with Whipple disease. ***Diagnosis: Whipple disease
56 A 4-year-old girl is brought to the physician because her parents noticed that she has been having pale, fauy. foul~smelling s tools. The patient is at the 50th percentile for height and lOth percemiJe for weight. Her sympwms respond dramatically to a gluten-free diet. Which of the following is the most likely diagnosis? ***(A) Celiac sprue ***(B) Cystic fibrosis of the pancreas ***(C) Menenier disease ***(D) Tropical sprue ***(E) Whipple disease
56 The answer is A: Celiac sprue. Celiac sprue, which is also referred to as gluten-sensitive enteropathy, is characterized by (l) generalized malabsorption, (2) small intestinal mucosal lesions, and (3) prmnpt clinical and lustopathologic response to the \vi thdrawal of gluten-containing food. C1iticaJ factors in the development of celiac sprue include genetic predisposition and gliadin exposure. The hallmark of celiac disease is a llat mucosa, with blunting of villi, damaged epithelial cells, imraepithelial I cells, and increased plasma cells in the lamina propria The other choices do not respond to a gluten-free cliet. ***Diagnosis: Celiac spme
57 A 53-year-old woman complains of acute diarrhea and severe abdominal pain. She was recently treated with broad-spectrum antibiotics for communny-acquired pneumonia. A CBC shows a WBC count of 24,000/~LL. The patiem subsequemJy develops septic shock and dies. A portion of her colon is shown at autopsy. These findings are typical of which of Lhe following gastrointestinal diseases? ***(A) Crohn cUsease ***(B) Diverticulitis ***(C) Ischemic colitis ***(D) Pseudomembranous colitis ***(E) Ulcerative colids
57 The answer is D: Pseudomembranous colitis. Pseudomembranous colitis is a generic term for an inflammatmy disease of the colon that is charam:1ized by exudative plaques on the mucosa. Amibiotic therapy eliminates the normal mL"Xed flora of the colon and facilitates the overgrowth of Clostridium difficile, leading to an acute infection of the colon. The exotoxins produced by C. diffictle cause intestinal necrosis, with superficial ulcers covered by a thick fibropumlent exudate. The other choices are not related to antibiotic therapy and are not associated with the development of these exudative plaques. ***Diagnosis: Pseudomembranous colitis
58 Physical examination of a newborn female after delivery reveals an imperforate anus. Meconhm1 is visible behind the thin, cutaneous membrane. The classification of this anorectal malformation is based on the relationship of Lhe terminal bowel to which of the following anatomic structures? ***(A) Ganglia in the wall of the rectum ***(B) Inferior mesenteric anery ***(C) Levawr ani muscle ***(D) Muscularis mucosae of rectum ***(E) Urachus
58 The answer is C: Levator ani muscle. Anorectal malformations are among the most common anomalies and va1y from minor narrowing to serious and complex effects The classification of these anomalies is based on the relation of the terminal bowel w the levator ani muscle. The other choices are not associated with anorectal malfonnations. ***Diagnosis: Anorectal malfonnation
59 ln addition to anorectal malformation, the infant described in Question 58 is most likely to have which o[ the fo llowing birth defects-:> ***(A) Cle rr lip/cleft palate ***(B) Congenital pyloric stenosis ***(C) Esophageal atresia ***(D) Gastroimestinal fisrula ***(E) Persistent urachus
59 The answer is D: Gastrointestinal fistula. Anorectal malformations resu1L from arrested development of the caudal region of the gm in the fi rst 6 momh.s of fetal life . The cause is unknown. Fistulas between rhe malfom1ation and the bladder, ureLhra, vagina, or skin may occur in all types of anorectal anomalies. The other choices are nm associated \l.fith anorectal malformations. ***Diagnosis: Anorectal malfoLmation
60 A 2-year-old boy is brought to the emergency room with a 48-hour history of nausea, vomiting, and abdominal discomfon. Physical examination reveals right lower quad ram guarding. Ultrasound examination of Lhe abdomen reveals a 2-cm mass in the right iliac fossa. A segmem of the small intestine is removed (shown in the image)_ Which of the following best describes this pathologic finding? ***(A) Intestinal in Carel ***(B) Intussusception ***(C) Meckel diverticulum ***(D) Peutz-Jeghers polyps ***(E) Volvulus
60 The answer is B: lntuuesception . Mechanical obstruction to the passage of intestinal comems can be caused by (1) a luminal mass, (2) an intrinsic lesion of the bowel wall, or (3) exuinsic compression. Obstruction m this case was caused by imussusception, in which a segmem of bowel (imussusceptum) pron·uded distally into a sun ounding outer pardon (intussusCipiens). This condition is usually a disorder of infams or young children, in whom it occurs without a known cause. In adults, the leading point of an imussusception is usually a lesion in the bowel wall, such as Meckel diverticulum or a tumor. Once the leading point is en trapped in the intussuscipiens, pe1istalsis drives the intussusceptum forward. ln addition to acute intestinal obstruction, intus· susceptlon compresses the blood supply to the imussuscep· tum, >vhich ma>' become infarcted. l f the obstruction is not relieved spontaneously, treatment requires surge1y None of the other choices display "telescoping" of the small lntesdne. Meckle diveniculum (choice C) is an outpouching of the gut caused by perisitence of the embryonic vitelline dueL It is the most common congen ital anomaly of the small intestine and is usually asymptomatic Peutz-Jeghers polyps (choice D) al'e hamartomas of the small intestine. Volvulus (ch01ce E) is an example of intestinal obstruction and acute abdomen, in whkh a segment of the gm twists on its mesentery, kinking the bowel and usually Lme.n upd.ng its blood supply. ***Diagnosis: Intussusception
61 An 85-year-old man complains of abdominal pain and bright red blood in his stooL An X-ray film of the abdomen shows fecal impaction in the rectosigmoid region. Which of lhe follov. l]ng pathologic Lesions is most likely to be found in this patient? ***(A) Curling ulcer ***(B) Cushing ulcer ***(C) Melanosis coli ***(D) Peptic ulcer ***(E) $tercoraJ ulcer
61 The answer is E: Stercoral ulcer. lncomptete evacuation of 1:he feces, usually in assoCiation with debilimLing disease or old age, may lead to the formation of a large mass of srool that cannot be passed, tenned fecal impaction. Stercoral ulce rs result from pressure necrosis of the mucosa caused by the fecal mass. Complications include rectal bleeding and perforation. The other ulcers do not occur in the recmm. ***Diagnosis: Stercoral uker
62 A 45-year-old woman complains or chronic, right lmver quadrant pain. An abdominal CT scan reveals a globular, smomh-wa11ed mass protruding imo the cecum. The pauent subsequently has the mass removed and the surgical specimen is shown in the image. Which of the foUowing is the most likely diagnosis? ***(A) Acute appendicitis ***(B) Adenocarcinoma ***(C) Carcinoid tumor ***(D) Foreign body ***(E) Mucocele of the appendix
62 The answer is E: Mucocele of the appendix. Mucocele refers to a dilated mucous- filled appendix. The pathogenesis may be neoplastic or nonneoplastic. ln the n01meoplastic vatiety, chwnic obstruction leads to the retention of mucus in the appendiceal lumen. ln the presence of a mucinous cystadenoma (in this case) or mucinous cystadenocarci11oma, the dilated appendLx is lined by a vil1ous adenomatous mucosa. A mucocele may become secondarily infected and ntpture Rupture of a neoplastic mucocele may seed the peritoneal cavity wiLh mucus-secreting mmor cells, a condition referred to as "pseudomyxoma petitonei. ~ ***Diagnosis: Mucocele
63 A 70-year-old woman with a history of ovarian cancer presents with dianhea. She completed radiation therapy for her cancer 3 months ago. Physkal examination shows cachexia, hyperactive bo·wel sounds, and generalized pallor. The stools are found to contam blood. A CBC shows dec reased hemoglobin (7.8gldL) and decreased mean corpuscular volume (70 Jlm3). Which of the following is the most likely cause of GI bleeding in this patient? ***(A) Angiodysplasia ***(B) Hemonhoids ***(C) lschemic colitis ***(D) Radiation enterocolitis ***(E) Solitary rectal ulcer
63 The answer is D: Radiation enterocolitis. Radiation therapy for malignant disease of the pelvis or abdomen may be complicated by injury to the small intestine and colon. Clmically signilicant radiation enterocolitis is most common in the recrum. The Lesions produced by radiation therapy range from a reversible mjury of the intestinal mucosa to chronic inflammation, ulceration, and fibrosis of the imestine. ***Diagnosis: Radm£ion enrerocolir:i.s, ovarian cancer
1 A 62-year-old man is bro1.1ght to the emergency room in a dis01iemecl state. Physical examination reveals signs of poor hygiene and an odor of alcohol, as well as jaundice, splenomegaly, and ascites. The patient has a coarse napping tremor of the hands, palmar erythema, and diffuse spider angiomata. The abdomen displays dilated paraumbilical veins. Serum levels of ALT, AST, alkaline phosphatase, and bilirubin are all mildly elevated. Soon after admission, the patient vomits a large amount of blood. Which of the following is the most likely underlying cause of hematemesis in this patient? ***(A) Acme alcoholic hepatitis ***(B) Acme gasuitis ***(C) Cirrhosis ***(D) Hepatic steatosis ***(E) Mallory-Weiss tear
1 The answer is(: (irrhosis. Cirrhosis represents the end stage of chronic liver disease and is characterized by extensive fibrosis and the formation of regenerative nodules. Patients with cin h osis often present with complications of portal hypen ension, including ascites, splenomegaly, and bleeding esophageal varices. Esophageal varices atise from the opening of pOital-systemic venous collaterals. Engorged collaterals in the submucosa of the lower esophagus and upper stomach, \Vhich dilate and prmmde into the lumen, are susceplible to bleeding. The prognosis of patients wilh bleeding esophageaJ vatices is poor, with a 40% mortality rate. Mallory-Weiss tear (choice E) is a possible cause ofhematemesis, but is only seen in patients with protracted vomiting. The mher choices do nm present with portal hypertension or systemic signs of endstage liver disease. ***Diagnosis: Alcoholic cinhosis, bleeding esophageal vances
2 For the patient described in Question l , which of the following pathophysiologic mechanisms is most directly associated with the development of ascites7 ***(A) Decreased aldosterone secretion ***(B) Decreased intravascular volume ***(C) Hyperalbuminemia ***(D) Increased intravascular oncotic pressure ***(E) Increased portal hydrostatic pressure
2 The answer is E: Increased portal hydrostatic pressure. Ascites refers [Q the accumulation or fluid in the pelitoneal cavity, oflen caused by portal hypenension. In the sening of cirrhosis, decreased intravascular oncotlc pressure due to hypoalbuminemia is also an important factor in the pathogenesis or ascites (see choice D). Aldosterone secretion (choice A) is increased in cinhotic patients. Overall, imbalances in Starling forces lead lO transudation of fluid in the abdomlnal cavity. ***Diagnosis: AJcoholic cinhosis, ponal hypenension
3 An 18-year-old man presents with a 2-week histOl)' o[ yellow skin and sclerae but is otherwise asymptomatic. He recalls a similar episode 2 years previously His brother also has recurrent jaundice. The serum bilirubin is 5.2 mgldl, mostly in the unconjug<ued form. Sen1m AST and ALT levels are normal, as is the minalysis. Two weeks later, the jaundice resolves spontaneously. What is the most likely diagnosis? ***(A) 0. 1-Antiuypsin deficiency ***(B) Dubin-johnson syndrome ***(C) Gil ben syndrome ***(D) Hereditary hemochromawsis ***(E) Wllson disease
3 The answer is C: Gilbert syndrome. Gilbert syndrome is an inherited, mild , recurrent, unconjugated hyperbilirubi nemia (<omgldl) that is caused by impaired clearance of bilirubin in the absence of any detectable liver disease. The syndrome tends to mn in families, and both autosomal dominant and recessive patterns have been observed . Aside from jaundice, patients a re asymptomatic. factors that increase semm bilirubin concentration in normal persons, such as fasting or illness, produce an exaggerated increase in semm bilirubin levels in persons with Gilbert syndrome. Although Dubin-]ohnson syndrome (choice B) is familial, iL presents \Nilh conjugated hyperbilimhinemia. The other choices do nol present \ViLh episodic jaundice. ***Diagnosis: Gil bert syndrome
4 A 48-year-old woman has a 3-week history of fatigue as well as yellow skin and sclerae. Physical examination is unremar kable except for mild jaundice. The serum bilirubin level is 3. 7 mgldl, mostly in the unconjugated form. Liver function tests including serum AST, ALT, and alkaline phosphatase are nonnal. The hemoglobin level is 6.0 gldl. After corticosteroids are administered, the .iaundice resolves. Which of the following diseases is the most likely cause of hyperbilirubinemia in this pauent? ***(A) Acme hepatitis B infection ***(B) Autoimmune hemolytic anemia ***(C) GallstOne in the common bile duct ***(D) Primary biliary cirrhosis ***(E) Primary sclerosing cholangitis
4 The answer is B: Autoimmune hemolytic anemia. Autoima...__ mune hemolytic anenua is characterized by antibody-mediated erythrocyte destruction and may lead to severe anemia, as in this case. Intravascular hemolysis produces increased serum levels of unconjugated bilirubin, which exceed the capacity of the hepatocyte to conjugate bilirubin. ln most cases, the disease is ameliorated by treatrnem with corticosteroids. The other choices are parenchymal liver diseases that manifest p timalily as conjugated hyperbilirubinemia and are unresponsive to steroids. ***Diagnosis: Autoimmune hemolytic anemia
5 A 20-year-old woman p resents with a 2-week hiswry of feve r, malaise, and brown-colored mine. She recemlyVIsited Mexico. Physical examination reveals jaundice, mild hepatomegaly, and tenderness in the right upper quadrant. The serum bilirubin is 7.8mgldl, with 60% in the conjugated form. Serum levels of AST and ALT are markedly eleva ted (400 and 392 U!L, respectively) . Serum albumin and immunoglobulin levels are normal. Serum IgM anti- hepatitis A virus (anti-HAY) is positive. IgG ami-hepatitis B surface antigen (anti-HBsAg) antibodies are positive. Anti-hepatitis C virus antibodies are negative. \Nhat is the most likely diagnosis? ***(A) Acute viral h epatitis A ***(B) Acute vi ra l hepatitis B ***(C) Acute viral hepatitis C ***(D) Autoinmmne hepatitis ***(E) Chronic viral h epatitis B
5 The answer is A: Acute viral hepatitis A. HAY is an RNA virus that is transmitted b}' the fecal-oral rome and may be contracted by comamination of water and food. Hepatitis A, the disease p resented here, never pursues a chronic course, does not have a cani er state, and provides life-long immunity. lgM ami-HAY is idemified in acute infections. The presence of serum 1gG ami-HBsAg indicates prior exposure to hepatitis B virus but does not re flect active disease (choice B). Individuals '<vith chronic hepatitis B (choice E) do not have detectable an dHBsAg in their blood. Acute and chronic hepatitis C (choice C) are ruled out by the negative serology Serum immunoglobulins are typically increased in patients with autoimmune hepatitis (choice D). ***Diagnosis: Hepatitis A, acute
6 A 3-day-old neonate born after a 32-week gestation develops yellow skin. Physical examination of Lhe inCant is unremarkable. Which of the following is most likely w be increased in this neon ate's serum? ***(A) Alanine aminon·ansferase ***(B) Carotene ***(C) Conjugated bilimbin ***(D) Ga lactosyltransferase ***(E) Unconjugated biUrubin
6 The answer is E: Unconjugated bilirubin. Approximately 70% of normal newborns exhibit transient unconjugated hyperbilirubinemia. Immatmity of the liver leads to inadequate conjugation of b1lirubin. This physiologic jaundice is more pronounced in premature infams due LO inadequate hepatic clearance of bilimbin and increased erythrocyte mrnover. Fetal bilirubin levels in utero remain lmi'.' because bilirubin crosses the placenta, where it is conjugated and excreted by the mother's liver. The mher enzymes are unrelated to neonatal jaundice. Eleva£ed serum levels of carotene (choice B) reflects hypervitaminosis A. ***Diagnosis: Neonatal (physiologic) jaundice
7 A previously healthy 38-year-old man complains of yellow discoloration of his eyes, abdominal pain, and low-grade [ever of 1-momh duration. Physical examination demonstrates a distended abdomen, right upper quadram tenderness, and a palpable liver edge 2. em below the right costal margin. Total serum bilintbin is 7. 4 mgldL. Serum levels of AST and ALT are elevated (22.9 and 495 UIL, respectively). The prothrombin time is prolonged (18 seconds). A liver biopsy is shown in the image. The anows point to Councilman bodies. The pathologic findings are indicative of which of the follmving liver diseases? ***(A) Acute vi ral hepatitis ***(B) Alcoholic cinhosis ***(C) Cardiac cirrhosis ***(D) Hemochromatosis ***(E) Primary bHiary clnhosis
7 The an.swer is A: Acute viral hepatitis. Pathologic changes of acme viral hepatitis include disarray of liver cell plates, ballooning degeneration of h epatocytes, intracellular and extracellular bile stasis, apoptoLic (CounciJman) bodies, and mononuclear inflammatory cell infiltrates. Histologic manifestations are simllar in acute hepatitis A, B, and C. Liver damage in acute viral hepatitis is reflected in elevations of serum u·ansaminases and hyperbiJimbinemia. Severe liver damage leads to impaired production of serum proteins, including prothrombm and other coagulation factors The other choices are examples of chronic liver disease. ***Diagnosis: Viral hepatitis, acute
8 A 30-year-old man presents with a 9-momh hismry of fatigue and recurrent fever. He also complains of yellow skin and sclerae, abdominal tenderness, and dark urine. Physical examination reveals jaundice and mild hepawmegaly. Laboratory studies demonstrate elevated serum bilirubin (3.1 mgl dl), decreased serum albumin (2.5 gldl), and prolonged prothrombin time (17 seconds). Serologic tesL<> reveal antibodies to hepatitis B core amigen (lgG ami-HBcAg). The serum is positive for HBsAg and HbeAg. A liver biopsy is shown in the image. What is the most likely diagnosis? ***(A) Acme hepatitis B ***(B) Alcoholic hepatllis ***(C) Chronic hepatitis B ***(D) Delta virus infection ***(E) Subacute hepatic n ecrosis secondary to hepatitis B infection
8 The answer is C: Chronic hepatitis B. Chronic hepatitis B refe rs LO infection with hepatitis B vims (HBV) that is associated '"litll necrosis and inOammadon in the liver for more than 6 months. HBV is a DNA virus that is transmiLted through blood transfusion , sexual contact, or shared needles. Most patients recover completely from acute infection, but some 10% develop chronic infection. Of the latter, 10% to 30% develop chronic hepatitis and cirrhosis. The biopsy in this case shows hepatocellular nodules and chronically-inflamed fibrous septa (see photomicrograph). Surface antigen (HBsAg) is present in the serum of patients with chronic hepatitis B, and tl1e presence of HbeAg is often associated with progression of the disease. Choices A, B, and E do not demonsuate cirrhosis as depicted and do not show the serologic characte!istics of HBV infection. ***Diagnosis: Hepatitis B, chronic
9 The patiem described in Question 8 is most likely lO develop L-- which of the following vascular b1flammatory diseases? ***(A) Allergic angiitis ***(B) Buerger disease ***(C) Giant cell arteri tis ***(D) Polyarteritis nodosa ***(E) Wegener granulomatosis
9 The an:swer is D: Polyarteritis nocfosa . Some HBV caniers manifest circulating immune complexes, which cause a variety of extrahepatic ailments, including a serum sickness-like syndrome, glomerulonephritis, cryoglobulinemia, and polyarteritis nodosa. Polyane1itis nodosa is a necrotizing arteritis of medium-sized vessels that can lead to pseudoaneurysm fonnation, renal thrombosis, inflammation, and hemorrhage. The other choices are not associated with chronic hepatitis. ***Diagnosis: Hepatitis B, chronic~ polyarteritis nodosa
10 A 40-year-old woman presents with a long history of vague upper abdominal pain and frequent indigestion Physical examination reveals an obese woman with jaundice and abdominal tendemess. Serum bilimbin is elevated (4.2 mgldL). There is a mild increase in serum AST and ALT (62 and 57UIL, respectively) and a moderate increase in alkaline phosphatase (325 U/L). Markers for viral hepatitis are negative. Abdominal ultrasound examination shows echogenic stone-like mateJial within the gallbladder and thickening of the gallbladder wall. An intrahepatic mass is also visualized adjacent to the gallbladder. A cholecystectomy is performed. Histologic examination shows dense fibrosis and glandular strucrures in the wall of the gallbladder. Wl1at is the most likely diagnosis? ***(A) Carcinoma of the gallbladder ***(B) Hemangiosarcoma ***(C) Hepatic adenoma ***(D) Hepawcellular carcinoma ***(E) Metastatic carcinoma of the stomach
10 TheanswerisA:Cardnomaof thegallbladder. Adenocarcinoma of the gallbladder is an incidental finding in 2% of patients who undergo gallbladder surgery due to chronic cholelithiasis. The tumor arises from the mucosal surface epithelium and may cause obstructive jaundice (as in llus case) by involvemem of the eA'trahepatic billar}' tree. The other choices are not associated with a histmy of chronic cholecystitis and cholelithiasis and infrequently cause obstructive jaundice. ***Diagnosis: Carcinoma of the gallbladder
11 A 60-year-old man is found in a state of disoriemation and is brought ro the emergency room in a comatose state. He lived alone, aLe poorly, and drank large amounts of hard Liquor. Physical examination reveals an emaciated man witl1 a dis· tended abdomen, jaundice, ascites, and a slightly enlarged liver and spleen. A liver biopsy is shown in the image. What blood tesl would confirm a dtagnosis of hepatic coma? ***(A) Alanine aminommsferase ***(B) Alkaline phosphatase ***(C) Ammonia ***(D) Bilimbin ***(E) Urea nitrogen
11 The answer is C: Ammonia. The photomicrograph shows cirrhosis, with regenerative nodules of liver cells sun-ounded by fibrous septa. Hepatic encephalopathy, a syndrome frequently observed in patients with cirrhosis or the liver, is characterized by personality changes, intellectual impaimlent, and a depressed level of consciousness. The developmem of hepatic encephalopaLhy is caused by increased serum concemrations of neurotoxic substances, among which is ammonia. Choices A, B, and D are elevated in a variety of liver diseases bm are unrelated w hepatic encephalopathy Blood urea ninogen (choice E) is used w assess kidney function. ***Diagnosis: Hepatic encephalopathy, alcoholic cirrhosis
12 A 20-year-old woman presents wilh a 4-week history of dry momh, fatigue, feve r, and yellow sclerae. Physical examinatjon shows mild jaundice and hepatomegaly. Serum total bilirubin is 3.3 mgldL Serologic markers for viral hepatitis are negative. The anti- mitochondrial antibody test is negative. A liver biopsy rusdoses parenchymal and periportal inOammatory cell infiltrates composed primarily of lymphocytes and plasma cells. The patients signs and symptoms abate following 2 months of treatment with steroids. Which of the following is the most likely diagnosis' ***(A) Autoimmune hepatitis ***(B) EJ.'trahepatic jaundice ***(C) P1imary biliary cirrhosis ***(D) P1imary sclerosing cholangitis ***(E) Wilson disease
12 The answer is A: Autoimmune hepatitis. Autoimmune hepatitis is a type of chronic hepatitis, which is associated with circulating autoantibodies (e.g., antinuclear antibodies) and high levels of serum immunoglobulins. The disease typically affects young women but occasionally afflicts older women and men. 1L is often accompanied by other autoimmune diseases (e.g., Sjogren syndrome, systemic lupus erythematosus). None of the other choices respond to steroids. Primary biliary cin-hosis (choice C) features ami-mitochondrial an tibodies. Primary biliary ci rrhosis (choice C) and primary sclerosing cholangitis (choice D) do not manifest the described histologic findings. ***Diagnosis: Autoimmune hepatitis
13 A 52-year-old recent immigram from Vietnam complains of abdominal swe!Jing, weight loss, and upper abdomjnal pain of 3 weeks in duration. His past medical history includes malaria and infection with the liver fluke ClonotchLs si11enm. The liver is hard m palpation. An abdominal CT scan shows a hypoattenuated mass with lobulated margins in the liver. A biopsy discloses well-differentiated neoplastic glands embedded in a dense fibrous s troma. Which of the following is the most likely diagnosis? ***(A) Carcinoma of the gallbladder ***(B) Cholangiocarcinoma ***(C) Hemangiosarcoma ***(D) Hepatocellular carcinoma ***(E) Metastatic colon adenocarcinoma
13 The answer is B: Cholangiocarcinoma. Carcinoma miginates anywhere in the biliary tree, h·om the large intrahepatic ducts at the porta hepatis to the smallest bile ductules at the periphery of the hepatic lobules. Cholangiocarcinoma arising within the Hver i.s associated with substantial fibrosis and can be confused with metastatic carcinoma and reactive fibrosis. These rumors occur at an increased frequency in persons iJl fected \'lith the liver fluke C. sinensis, which takes up residence in the biliary tree. Primary sclerosing cholangitis is another 1isk factor for this cancer. Patient.'> with cholangiocarcinoma have a poor prognosis. The other choices are not associated with a history of C. sinensis infestation. ***Diagnosis: Cholangiocarcinoma
14 A 58-year-old man with longstanding alcoholic cirrhosis presents with abdominal pain, fever, and an episode of hematemesis. Physical examination reveals jaundice and a markedly distended abdomen. The patient is disoriented and has a coarse napping m~mor of the hands. Laborarory studies reveal modeslly elevared serum Levels of AST and ALT (96 and 92 U/L, re spectively) and a high serum level of alkaline phosphatase (320 UIL). PrOlhrombin rime is prolonged (20 seconds). The WBC count is 18,000/~-LL Shonly after admission, the patient develops coma, adult respiratmy distress syndrome, and renal failure (oliguria and elevated serum levels of BUN and creatinine), leading lO death within 3 days. Histologic examination of the patient's kidney at autopsy would most likely show which of the following? ***(A) lmerstitial nephritis ***(B) Membranous nephropathy ***(C) No histologic changes ***(D) Proliferative glomerulonephritis ***(E) Pyelonephritis
14 The answer is C: No histologic changes. Heparorenal syndrome usually occu rs in the setting of cirrhosis and heralds a poor prognosis The diso rder is characterized by fean. tres of renal hypoperfusion, including oliguria, azotem1a, and increased levels of serum creadnine. Microscopically, the kidney appears normaL Renal fallure is caused by vasoconstriction and hypoperfusion of the kidneys, a combination mediated by various hom10nes and vasoactive substances, some of which may nm be cleared by the cLrrhotic liver. Similarly, a kidney from a patient in hepawrenal failure may be successfully transplanted into another person and assume nonnal functioning. The other choices are assoclated with direct injury Lo the renal parenchyma and exhibit characteristic histologic findings. ***Diagnosis: Hepatorenal syndrome
15 A liver biopsy in the patient described in Question l4 would definitely show which of the following pathologic chan ges? ***(A) Dilated bile ducts and portal inflammation ***(B) Fatty liver ***(C) Nodular regeneration and scarring ***(D) Petiponal necrosis and pe1ipheral cholestasis ***(E) Scattered single cell necrosis and acidophilic bodies
15 The answer is C: Nodular regeneration and scarring. In about 15% or alcoholics, hepatocellular necrosis, fibrosis, and regeneration eventually lead to the formation of fibrous septa surrounding hepatocellular nodules, which are features that define cirrhosis. Morphologic changes described in the other choices may be presem in cases of alcoholic liver dlsease but are not directly associated with portal hypertension. Fatty liver (choice B) and Malloty hyaline are associated vvilh alcoholism, but they are nOL specific indtcators of drrhosis. ***Diagnosis: Alcoholic cirrhosi..c;
16 A 40-year-old black woman has frequem indigestion after meals and abdominal pain. Physical examination demonstTates a moderately obese woman in no acute distress. An ultrasound examination demonsu·ates numerous echo genic objects within the gallbladder. A cholecystecwmy is perfonned, and the surgical spedmen is shown in the image. The gallstones seen in this patient are typically associated with which of the following diseases? ***(A) Chronic pancreatitis ***(B) Diabetes mellitus ***(C) familial hypercholesterolemia ***(D) HyperparathyrOidism ***(E) Sickle cell disease
16 The answer is E: Sickle cell disease. Black (pigmented) gallstones are irregular and measure less than 1 em across. On cross section, the surface appears glassy Black s tones contain calcium bili.rubinaLe, bilirubin polymers, calcium salts, and mucin. Hemolysis in patients with sickle cell disease or other chronic hemolytic anemias generates excess bilirubin, which predisposes to pigment stone fonnation. Cirrhosis, either through liver cell damage or hemolysis, predlsposes to black stones. Gallstones can cause pancreatic duct obsLruclion, increasing the risk for development of acute and chronic pancreatitis (choice A). Hypercholesterolemia is a risk factor for development of cholesterol gallstones. ***Diagnosis: Cholelithiasis, sickle cell disease
17 A 25-)'ear-old herom add1ct presents m a d1sonemed state wuh a 5-day htstory of faugue, mala1se, and dark-colored urine. Physical exammauon reveals 1aund1ce and multiple petechial hemorrhages on the upper extremities. Laboratory studies show serum bthrubin of 15.6 mgldl, mostly in the conjugated form, 10-fold elevations of serum AST and ALT, high levels of blood ammonia, and mcreased prothrombin time (15 seconds). The pauems condlllon detcnorates and he develops stage 4 hcpatk encephalopathy. A liver biopsy is shown in the image. Which of the following viruses is most likely responsible for the cli nical and patho logic findings in this patient? ***(A) C)•LOmegalovirus ***(B) HepatlllS A virus ***(C) Hepatitis B virus ***(D) Hepauus C VlTUS ***(E) Hepatitis E \'irus
17 The answer is C: Hepatitis B virus. Massive hepatic necrosis often leads to fulminant hepatic failure. A common cause of massive hepatic necrosis is hepatitis B virus. The liver appears shrunken, the capsule is wrinkled, and the parenchymal tissue is soft and flabby. On microscoplc examination , the n ecrotic liver lobules are hemorrhagic, and lhe reticulin framework has collapsed. Hepatitis A virus, C vi.rus, and E virus (choices B, D, and E) rarely present with massive hepatic necrosis. ***Diagnosis: Fulminant hepatitis B infection, h epatic failure
18 A 25-ycar-old woman complains of sudden onset of acUic abdomit1al pain. Physical cxammation shows abdominal distention. Her temperature is 37°C (98.6°F), respirations 22 per mmute, hean rate 110 per mmutc, and blood pressure 70/50 mm Hg. A tap of the abdomen returns blood. ACT scan reveals a solitary 20-cm mass of the hver. A surgically resected pomon of the hver is shown tn the tmage. Thts patients tumor was most hkely associated with chrome exposure to which of the following? ***(A) Carbon tetrachloride ***(B) Halothane ***(C) L-thyroxine ***(D) Oral comraceplives ***(E) Vinyl chloride
18 The answer is D: Oral contraceptive-s. Hepatic adenoma usually occurs as a soliLa1y, sharply demarcated mass up to 40cm in diameter and 3 kg in weighL On gross examination, the rumor is encapsulated and paler than the surrounding liver parenchyma. Hepatic adenoma is a complication of oral connaceptive use in women. In abom 30% of patiems with hepatic adenomas, the rumor tends lO bleed into lhe pe1itoneal caviiy, inducing hypovolemic shock that requires emergency treatment. The other choices do nol induce hepatic tumors. ***Diagnosis: Hepatic adenoma
19 A 15-year-old boy complains of a 2-momh history of fatigue, abdominal pain, and yellow eyes and skin. Physical examination shows tremor of his hands, lacl< of coordination, and mild jaundice. The results of an oph thalmic examination are shown in the image. This patient most likely has an inborn en-or of metabolism associated with tissue overload of which of the following elements? ***(A) Copper ***(B) Iron ***(C) Lead ***(D) Magnesium ***(E) Mercury
19 The answer is A: Copper. Wilson disease is an autosomal recessive condition in which excess copper can be deposited in the hver and brain. Chronic hepatitis leads to cinhosis in young people. Ocular lesions, so-called Kayser-Fleischer rings, represent deposition of copper in Descemet membrane in the iris (note peripheral bro\\m color). Extrapyramidal neurologic symptoms (e.g., lack of coordination and tremor) are related to degenerative changes in the corpus striamm. Toxicity of the OLher elemems are associated \vi.th omer manifestations. ***Diagnosis: Wilson disease
20 A 49~year-old woman presents wilh a 1-momh history of yellow discoloration of her eyes, abdominal pain, malaise, weight loss, and low-grade fever (38.4°C, 101 °F). Physical examination shows a distended abdomen with right upper quadrant tenderness and a palpable liver 2 em below the tight costal margin. Laboratory studies reveal decreased semm albumin (2.6gldL), elevated serum AST (225U!L) and ALT (l50UIL), and increased alkaline phosphatase (2lOUIL). The prolhrombin time is prolonged (15 seconds). A moderaLe leukocytosis (13,500/!lL, 80% neurrophils) is observed. A liver biopsy is shown in the image. These pathologic findings are most commonly associated with which of the following liver diseases' ***(A) Alcoholic hepatitis ***(B) Chronic hepatitis B ***(C) Chronic hepatitis C ***(D) Hemochromatosis ***(E) Primary blliary cirrhosis
20 The answer is ~: Alcoholic hepatitis. Acute alcoholic hepatitis is characterized by hepatic steatosis and hydropic swellLng of hepawcytes, focal hepawcellular necrosis, neutrophilic infillration, and cytoplasmic hyaline inclusions within the h.eparocytes (Mallory bodies), which represent precipirated imermediate filament proteins. ClinicaUy, alcoholic hepatiris presems with malaise and anorexia, fever, tight upper quadram pain. and jaundice. Mallory hyaline is seen in patients wilh primary biliary cirrhosis, bm the other hiswlogic findings in this patiem's liver biopsy (e.g., hepatic s teatosis, hydropic swelling of hepawcytes, focal hepawcellular necrosis, and neutrophilic infi ln·ation) are noL feamres of primary biliary cirrhosis (choice E). Mallory bodies are rare in the other choices. ***Diagnosis: Alcoholic hepatitis
21 A 36-year-old woman presents with a 6-momh history of progressive generalized itching, weight loss, fatigue , and yellow sclerae. She denies use of oral conn·aceptives or any other medkation. Physical examination reveaJs mild jaundice and steatorrhea. Blood studies show a high cholesterol level of .350 mgl dL, elevated sennn alkaline phosphatase (240UIL), and nonnal levels of AST and ALT. An intravenous cholangiogram shows no evidence of obstruction. An amimitochondrial antibody test is positive; antinuclear antibodies are not present. 'Which of the following skin manifestations is expected in this patient? ***(A) Acanthosis nigricans ***(B) Hyperpigmentation ***(C) Keratoacanthoma ***(D) Seborrheic keraLOsis ***(E) Xanthoma
21 The answer is E: Xanthoma . Primary biliary cirrhosis (PBC) is a chronic progressive liver disease that is associated with many immunologic abnormalities and is, therefore, widely held to be an autoimmune disease. The hallmark of this condition is the presence in serum of amimitochondrial andbodies. These autoantibodies recognize epitopes associated with the miwchondrial pyruvate dehydrogenase complex. Despite the specificity of the antimitoch ondrial antibodies, they have no inhibitory effect on mitochondrial function and play no known role in the pathogenesis of the disease. The comple· ment system is chronically activated. PBC occurs more often in women than in men (10:1 female predominance)_ It presents with fatigue, anore::..'ia, jaundice, xanthomas of me skin, and pruritus. The other choices are not associated with PBC or hypercholesterolemia_ ***Diagnosis: Primary biliary cirrhosis
22 For the patient described in Question 2l, a liver biopsy would most likely show which of the following pathologic findings? ***(A) Central hyaline sclerosis ***(B) Cholangiocarcinoma ***(C) Hemosiderosis ***(D) Intrahepatic bile duct damage ***(E) Macrovesicul.ar steatosis
22 The answer is D: Intrahepatic bile duct damage. Primary biliary drrhosis (nonsuppuralive destructive cholangitis) is caused by chronk destruction of intrahepatic bile ducts in the portal tracts. Primal)' biliary cin·hosis evolves through ductal lesions, scarring, and eventually drrhosis. Early PBC feamres chronic destructive cholangitis aCfecting intrahepatic small and medium-sized bile ducts. The bile ducts are surrounded p1imarily by lymphocytes (CD8~ T cells), but plasma cells and macrophages are also seen. ln some portal tracts, lymphoid follicles are conspicuous Discrete epithelioid granulomas often occur in the portal tracts and may impinge on the bile ducts. As a result of the destmctive chronic inflammatory process, small bile ducts virtually disappear, and scarri ng of medium-sized bile ducts is conm1on. Proliferation of bile ducrules within ponal tracts is common and may be ilorid. CoUagenous septae e~'tend from the portal n·acts imo the lobular parenchyma and encircle some lobules. Cholestasis, when presem, may be seve re and is located at the periphery of the ponal tracts. The end-stage of PBC is cirrhosis, characte1ized by a dark green bile-stained liver that exhibits fine nodularity. The other choices do not feature destruction of intrahepatic bile ducts. ***Diagnosis: Primary biliary ctrrhosis
23 A 60-year-old man has a 6-month history of abdon1lnal swelling. On a daily basis, he smokes two packs of cigarettes, dtinks five cups of coffee, and repons t.hat he consumes 2 si.x:packs of beer. Physical examination shows a distended abdomen with a palpable Uver 2 em below the costal margin. A liver biopsy is shown in rhe image. If this patiem becomes abstinem, his liver will mosL likely do which of t.he following? - ***(A) Develop hepatocellular carcinoma ***(B) Progress to cirrh osis ***(C) Progress co inflammatory hepatitis ***(D) Remain unchanged ***(E) Reven w normal
23 The answer is E: Revert to normal. Excessive alcohol consumption induces fat accumulation within hepatocytes, enlarging the liver to as much as three times the nonnal weight. The amoum of fat deposited varies with the amount of alcohol consumed, as well as the patient's hormonal starus, diet, and mher facwrs. T1iglyceride accumulanon by itself is not ordina1ily damaging, and the condition is fully reversible upon discontinuation of alcohol abuse (abstinence). ***Diagnosis: AkohoHc fatty liver
24 A 54-year-old man presents with a 9-momh history of progressive skin pigmentation. He passes Large amounts of urine and is always thirsty. His father died of liver cancer. Physical examination reveals a dark skin color and an enlarged liver. Laboratory studies show nonnal serum levels of corticotropin. A glucose tolerance test indicates chemical diabetes. A liver biopsy stained wilh Pntssian blue is shown in the image. If untreated, which of the following conditions is most likely to develop in this patient? ***(A) Acme hepatitis ***(B) Addison disease ***(C) Cholangiocarclnoma ***(D) Cholelithiasis ***(E) Hepatocellular carcinoma
24 The answer is E: Development of hepatocellular carcinoma. 1-lereditary hemochromatosis (HH) is a common, auwsomal recessive, genetic disorder thal lS characterized by excessive iron absorption and the toxic accumulation of iron in parenchymal cells, particularly of the liver, hean, and pancreas. In the liver, HH leads tO cirrhosis and a high mcidence of primary hepawceUular carcinoma. The clinical hallmark of advanced HH is the presence of other diseases such as diabetes, skin pigmemation, and cardiac failure. The Pmssian blue stain binds iron and provides hiswlogic evidence for iron overl oad. Addison disease (choice B) presents with skin pigmentation but rellects autoimmune destruction of the adrenal glands. ***Diagnosis: Heredttary hemochromatosis
25 A 28-year-old woman presents with a 4-day history of abdominal pain and increasing abdominal girth. She does not drink a\coho]jc beverages, but smokes a pack of cigareLtes a day. Except for oral contraceptives, she Lakes no medications. Physical examination shows hepatomegaly, ascites, and mild jaundice. A l]ver biopsy is obtained (shown in the image). Which of lhe following is the most likely diagnosis? ***(A) Budd-Chiari syndrome ***(B) Chronic hepatitis B ***(C) Extrahepatic cholesmsis ***(D) Primary biliary cirrhosis ***(E) Secondary biliary cirrhosis
25 The answer is A: Budd·Chiari syndrome. Budd-Chiari syndrome is a congestiYe disease of the liver caused by occlusion of the hepatic veins and their tributaries. The principal cause of Budd-Chiari syndrome is thrombosis of the hepatic veins. Intrahepatic venous thrombosis may be associated \Vith increased blood viscosity (as in polycythemia vera or other myeloproliferative disorders) and hypercoagulable states associated with hemaLologic cancers, certain solid tumors , pregnancy, and paroxysmal nocLurnal hemoglobinuria. However, in more than half the cases, the cause of Budd-Chiari syndrome is not apparent. Complete thrombosis of the hepatic veins presents as an acute illness characterized by abdominal pain, enlargement of the liver, ascites, and mild jaundice Acute hepatic failure and death often occur rapidly. The needle biopsy m this case shows severe cemrilobular necrosis and hemorrhage (see photomicrograph) The sinusoids of the central zone are dilated and packed wjth erythrocytes. The other choices do not present with cenuilobular hemorrhage and are not associated wilh oral contraceptives. ***Diagnosis: Budd-Chiari syndrome
26 A 4 7 -year-old woman p resems with a 3-momh history of vague upper abdominal pain afLer fatty meals, some abdominal distension, and rrequent indigestion. Physical examination shows an obese woman (BMI = 30 kglm2) with right upper quadram tenderness. An ulcrasound examination discloses multiple echogenic objects in the gallbladder. The opened gallbladder is shown in the image. Vv'hich of the following metabolic changes is most likely associated with Lhe fonnmion of gallstones in this patient? ***(A) Decreased hepatic bilirubin conjugmion ***(B) Decreased serum albumm ***(C) Increased bilirubin uptake by the hvcr ***(D) Increased heparic calcium secreuon ***(E) Increased hepatic cholesterol secretion
Z6 The answer is E: lncrused hepatic cholesterol secre· tion. Cholesterol stones are round or faceted, yellow to tan , and may be single or multiple. Risk factors for cholesu:: rol stones include female sex, diabetes, pregnancy, and estrogen therapy. Solimry, yellow, hard gallstones are associated with bile that is supersaturated with cholesterol. During their reproductive years, women are up to three times more likely to develop cholesterol gallstones than men. If the b ile contains excess cholesterol or ls deficient in bile acids, it becomes supersaturated with cholesterol and precipitates to form stones (lithogenic bile). ln obese women, cholesterol secretion by the liver is increased. Impaired gallbladder motor function is another 1isk fac tor that leads to gallstone formation. ln this case, stasis permits the formation of biliary sludge, whicfl then progresses to macroscopic stones. Choices A and B are not associated with gallstones. Choices C and D are not physiologic. ***Diagnosis: Cholelithiasis, cholesterol gallstones
27 For the patient described in Question 26, which of the following is a common compHcation? ***(A) BiJe p eritonitis ***(B) Chronic passive congestion of the liver ***(C) Connuem hepatic necrosis ***(D) EJ..'trahcpaLic biliary obstruction ***(E) P1imary hepatocellular carcinoma
Z7 The answer is D: Extrahepatic biliary obstruction. Most complications associated with cholelithiasis are related to obstruction of the biliary tree. Passage o [ stones into the cystic duct often causes severe bi liary colic. Lodgement of stones in the common bile duct leads to obstructive jaundice, cholangitis, and acute pancreatitis. Patient.c; with cholelithiasis have a 25-fold increased risk of acute pancreati tis compared with the general population. AddiLional complications are rare and include empyema of the gallbladder, perforation, fismla formation, bile peritonjtis, and gallstone lleus. In most cases, gallstones are associated with chronic cholecystitis. Choices B, C, and E are not associated with gallstones. ***Diagnosis: Cholelithiasis, obstructive jaundice
28 A 68-year-old man complains of vague abdominal pain, imermitLent rever, and a 20-lb (9-kg) weight loss over Lhe pasL 6 months. For the past 12 years, he has suffered fTom chronic hepatitis B. On physical examination, the patient shows diffuse abdominal tendemess, hepatomegaly, and mild j aundice. A CT scan of the abdomen reveals a diffusely nodular liver, with a dominan l mass measuring 3 em in diameter. A needle biopsy is shown in the image. Which of the following serum markers is useful for monitoring the progression of disease in this patient? • • ***(A) Alkaline phosphatase ***(B) Alpha-fewprO[ein ***(C) Ami-H& antibody ***(D) Carcinoembryonic antigen ***(E) Human chorionic gonadotropin
28 The answer is 8: Alpha-fetoprotein (AFP) . AFP is a glycoprotein that is normally synthesized in the fetus by the yolk sac, liver, and gastroimestinal tract. In adults, an elevated serum level of AFP is a useful i11dicaror of hepatocellular carcinoma and germ cell tumors of the testis. AFP levels decline rapidly after surgical resection of liver cell cancer or treatmem of pauems with metastatic genu cell mmors. Alkaline phosphatase (choice A) is a common indicator of hepatobilia~y disease. Carcinoembryonic antigen (choice D) is principally used to monitor gastroimeslinal cancers . ***Diagnosis: Hepawcellular carcinoma
29 A 30-year-old man presents with a 3-week history of fatigue , occasional fever, yellow skin and sclerae, tenderness below the right costal margin, and dark urine. Physical examination reveals jaundice and mild hepatomegaly. laboratory sLUdies show elevated serum levels of bilirubin, decreased albumin, and prolonged prothrombin time. Serologic tests disclose antibodies to hepatitis C virus. Which of the following tests is the most accurate method for assessing the extem of liver disease in this patient? ***(A) Liver biopsy ***(B) Serum alkaline phosphatase ***(C) Serum ammonia ***(D) Serum immunoglobulins ***(E) Serum transaminases
29 The answer is A: Liver biopsy. Microscoplc examination of a liver biopsy is the best method currently avrulable for assessing the extent of liver disease in a patienL with viral hepatitis. The major histologic features of acute viral hepatitis are liver cell injury and inOammation. Microscopic examination shows ballooning degeneration of liver cells, intracellular and extracellular bile stasis, acidophilic bodies, and a mononuclear cell infiltrate. Serum alkaline phosphatase and transaminases (choices B and E) are also useful indicators of the severity of liver disease, but do not allow for an assessment of the chronicity or stage of the disease. Serum ammonia (choice C) is used to monitor patients at risk for hepatic encephalopathy and ordinarily refiects end-s tage liver disease. ***Diagnosis: Hepatitis C, acute
30 A 38-year-old man is brought to the emergency room with clouded sensorium and lethargy He had been degreasing the engine pans of an old car the previous day, using industrial solvents. Later that evening he felt ill, and by moming, he was difficult to rouse. Serum AlT is exmmely high (2,400Uil)_ He dies 2 days later in hepatic coma. Which of the follmving liver injUJies is the most likely diagnosis? ***(A) Alcoholic hepatitis ***(B) Allergic reaction ***(C) Budd-Chiari syndrome ***(D) Idiosyncratic reaction ***(E) Predictable tOxic liver injury
30 The answer is E: Predictable toxic liver injury. Acute, chemically induced hepatic injury spans the entire spectrum of Uver disease, from transient cholestasis to massive hepatic necrosis. Drug-induced liver injury can be either direct or indirecL. !ndiTect injmy is caused by metabolites and free radicals that are produced as byproducts of xenobi.otic metabolism. lmmune reactions against a chemical or its metabolites are also causes of indirect liver damage. Chemically-mduced hepatic injury is classified as "predictable" when toxicity is inm1ediare and dosedependent and as "unpredictable" or uidiosyncraric" when roxicity occurs withom explanation {choice D). In this case, exposure to industrial solvenlS, such as carbon tetrachloride, caused predictable toxic liver injury, charactetized by centrilobular necrosis and elevated serum levels of transaminases. ***Diagnosis: Toxic liver injury
31 A 66-year-old man presen ts \\rith a 2-week history of abdominal bloating, weight loss, and pain in the right upper quadrant. The patient had a serious motor vehicle accident 16 years ago, in which he required transfusion of lOU of whole blood. On phystcal examination, he exhibits massive distension of the abdomen. The liver is hard on palpation and occupies the enti re right side of the abdomen. Laboratory smdies show a low serum albumin (2 2 gldL) and a markedly elevated serum alpha-fewprotein An abdominal ultrasound examination reveals ascites. The patient e"'Pires 6 months later. The liver at autopsy is shown in the image. Which of t.he following is the most common cause of this disease world\\ride? ***(A) Alcoholic hepatitis ***(B) Autoimmune hepatitis ***(C) Chroni.c hepatitis B ***(D) Chronic hepatitis C ***(E) Hepatitis E
31 The answer is C: Chronic hepatitis B. Patients with persistent hepatitis B virus (HBV) infection have a 200-fold increased risk of developing primary hepatocellular carcinoma (HCC), the diagnosis in this case. More than 85% of cases of HCC occur in counnies \'<ith a high prevalence of chronic infection with HBV. One fourth of patients with chronic hepatiLis B ultimately develop HCC. Chronic h epatitis C (ch oice D) is associated with most cases of HCC in Europe and North America, but chron ic hepatitis B remains the major global cause of HCC. ***Diagnosis: Hepatitis B. chronic: hepatocellular carcinoma
32 A 60-year-old woman presents with several years of abdominal pain radiating to her back and a 5-day history of yellow skin and sclerae. She has lost lSib dming the past several months, and her stools have become lighter in color. On physical examination, the patiem is cachectic and jaundiced. The liver edge descends 1 em below the right costal margin and is nomender. Her right calf is tender and erythematous Serum AST and ALT are at the upper limits of nmmal, but alkaline phospha1.ase is increased w 4 30 U/L. A CT scan shows a mass in the head of the pancreas. What is the most likely cause of jaundice in this patient? ***(A) Acute viral hepatitis ***(B) Alcoh olic hepatitis ***(C) a 1-Antinypsin deficiency ***(D) Drug-induced hepatitis ***(E) Extrahepatic biliary obstruction
32 The answer is E: Extrahepatic biliary obstruction. This patiem present:5 with signs and sympLOms of biliary obstruction d ue to obstruction of the biliary uee by adenocarcinoma in the head of the pancreas. Symptoms of pancreatic cancer in thLs patient include pain radiating to the back and weight loss. Diarrhea and steatorrhea result from fat malabsorption, which 1s seconda ry to extrahepatic obstruction to bile flow by encroachment of the mmor and metastatic lymph nodes on the common bile duct. Choices A, B, and D are incon·ect because the ALT and AST were nonnal, which is unlikely in the seuing of hepatitis. High serum alkaline p hosphatase signals obsnucLive jaunclice. ***Diagnosis: Extrahepatic biliary obstruction, pancreatic carcmoma
33 A 40-year-old >voman complains of having seve re back pain for about 3 months and recu rrent fever Her past medical history is significant for ulcerative colitis. On physical examination, the patiem is thin and jaundiced The liver edge descends l em below the right cos~al margin and is nomender. Laboratory studies show normal semm levels o[ AST and ALT but elevated serum levels of alkaline phosphatase ( 420 U/L). Endoscopic retrograde cholangiopancreatography demonstrates a beaded appearance of the extrahepatic biliary u·ee. Which of the following diseases is a late complication of this patients condition? ***(A) Adenocarcinoma of the gallbladder ***(B) Cholangiocarcinoma ***(C) Hepatic adenoma ***(D) Hepatic angiosarcoma ***(E) Hepamcellular carcinoma
33 The answer is B: Cholangiocarcinoma. Prima1y sclerosing cholangitis (PSC) is characterized by inflammation and oblirerarive fibrosis of intrahepatic and exrrahepadc b ite ducts, with dilation of preserved segments. Approximately 70% of patients with PSC have longstanding ulceralive colitis, although the prevalence of PSC in such patients is only 4%. PSC tends to occur in the third through fifth decades of li fe, with a signifi· cam male predom.lnance (2:1). The cllnicopalhologic findings are complemented by a characte1istlc radiographic appearance of a beaded bitiary tree, representing sporadic s trictures. Cholangiocarclnoma is a late complication of PSC. The other choices are not complications of PSC. ***Diagnosis: Primary sclerosing ch olangitis
34 A 32.-year-old man presen ts with a 6-month history of yellow skin and sclerae. Physical examination shows mild jaundice, pitting edema, and ascites. Laboratory studies reveal decreased serum albumin (2.6gldL) and increased serum AST and ALT (12.0 and l40UIL, respectively). A llver biopsy s tained with period acid-Schiff (PAS) reagent and diastase digestion is shown in the image. This patiem has which of the following genetic diseases? ***(A) a 1-Antiuypsin deficiency ***(B) Glycogen swrage disease ***(C) Herednary h emochromawsis ***(D) Hurler syndrome ***(E) Pompe syndrome
34 The answer is A: <X1·Antitrypsin deficiency. Ci 1-Antitrypsin deficiency is the most common genetic cause of liver disease ln infants and children and the most frequent genetic disease for which liver transplantation is indicated. The liver may be involved \vtth or without pulmonary disease in the fom1 of emphysema a 1-Antitrypsin deficiency is characte1ized by the presence o£ round-to-oval cytoplasmic globular inclusions of n:usfolded a. 1-antitrypsin p rotein.<; in hepatocyres. These globules stain red with PAS after removing glycogen \Vith diastase. These inclusions are not featured in the other choices. ***Diagnosis: <X 1-Amluypsin deficiency
35 A 42-year-old man is brought to the emergency room with 1ight upper quadnmt pain, shaking chills , and a fever of .38.rC (l030f). His past medical history is significant for an appendectomy 2 weeks previously. Physical examination reveals hepatomegaly and tenderness in the right upper quadrant. Laboratmy studies show nom1allevels of semm albumin, ALT, and bilirubin, as well as increased alkaline phosphatase of 240U!L. The \NBC coum is 28,000/~LL. vVhich of the following is the most likely diagnosis? ***(A) AcUle cholecystitis ***(B) AcUle hepatitis ***(C) Diffuse peritonitis ***(D) Extrahepatic biliary obstruction ***(E) Pyogenic liver abscess
35 The answer is E: Pyogenic liver abscess. Pyogenic liver abscesses are caused by sraphylococci, streptococci, and Gram-negative enterobacteria (i.e., anaerobic inhabitanlS of the gasu·oimestinal nact). The bacteria gain access to the liver by direct extension from contiguous organs or through the ponal vein or hepatic anery. Extrahepatic biliary obstruction (choice D), which leads to ascending cholangiUs, is the most common cause of pyogenic abscess and is usually associated \vith s tones in the common bile dun (choledocholithiasis), benign and malignant tumors , or postsurgical strictures of the bile duelS. As in this instance of appendicitis, the infectious organisms can also originate within the abdomen and reach th e liver by embolization. Other abdmninal causes of pyogenic abscess in the liver include diverticulitis and inOammawry bowel diseases. Diffuse pe ritonitis (choice C) is a possible complication of perforated appendicitis but is not suggested by the clinical presentation described in this vignette. Acute cholecystitis (choice A) is a very unlikely complication or appendiclds. ***Diagnosis: Pyogenic liver abscess
36 A previously healthy, 2'1--year-old woman presents with a 1-week history of internuttent fever, lethargy, and yellow skin and sclerae. Physical examination shows jaundice Laboratmy studies reveal decreased serum albumin (2.2 gldL), extremely high levels of AST and ALT (1 ,200 and l,800UIL, respectively) , and elevated alkaline phosphatase (300UIL) Her ceruloplasmin level is normal. She is admiued to the hospitaL Her condition progressively deteriorates, and she develops hepat]c encephalopathy and hepatorenal syndrome. Which of the fo llmving is the most likely diagnosis' ***(A) Extrahepatic bihary obstruction ***(B) Hereditary hemochromatOsis ***(C) Massive hepatic necrosis ***(D) Primary bllimy cirrhosis ***(E) Sclerosing cholangitis
36 The answer is C: Massive hepatic necrosis. Massive hepatic necrosis is the most feared variant of acute hepatitis. The laboratory fmdings seen in this patient (markedly elevated ALT and AST) are characteristic for Lhis condition. Grossly, the liver loses about one third of its normal weight, and Glisson capsule is wrinkled and mottled. Microscopic examination reveals massive death o£ the hepatocyu~s. leaving a collapsed collagenous framework. Ahhough the other choices can lead to hepaLic failure, patients are typical1y symptomatic prior to hepadc decompensation. ***Diagnosis: Massive hepatic necrosis
37 A 36-year-old , alcohol1c woman p resents with a 1-week history of yellow skln and sclerae. She has suffered persistem headaches. Her vital signs are normaL Physical examination reveals jaundice. Laboratory smdies disclose markedly elevated levels of AST and ALT (956 and l,400UIL, respectively). A few days later, she develops hepatic encephalopathy and renal failure. A Uver biopsy shows prominent cent1ilobular necrosis. Which of the following is the most like ly diagnosis? ***(A) Acetaminophen toxicity ***(B) Fauy liver of pregnancy ***(C) Metastatic carcinoma ***(D) Reye syndrome ***(E) 'Nilson disease
37 The answer is A: Acetaminophen toxicity. Drug LO::\.'i.city should be suspected in all cases of acme hepatitis. ln this case, centTilobular necrosis suggests acetaminophen toxicity. The mxic dose of acetaminophen after a single acute ingestion is in the range of 150 mglkg in children and 7 g in adults. Acetaminophen is rapidly absorbed fTom the stomach and small intestine and conjugated in the liver tO nontoxic agents, which then are eliminmed in the urine. In cases of acute overdose, nmmal pathways of acetaminophen metabolism become samrated. Excess acetaminophen is then metabolized in 1.he liver via the mL-xed function oxidase P450 system, yielding o::\.idative metabolites that cause predictable, hepatocellular necrosis. The centrilobular zones are particularly affected (centrilobular necrosis). Centrilobular necrosis is not seen in the other choices. Reye syndrome (choice D) occurs in children. Fatty liver of pregnancy (choice B) features microvesicular steatosis. ***Diagnosis: Acetaminophen wxicit)~ hepawrenal syndrome
38 A 30-year-old man from Me;..'ico presems with a 3-week history of constam pain in his upper 1ight quad ram and epigastnum and persistent cough. The patient also reports a recem histmy of nausea, vomiti ng, and bloody diarrhea. Physical examination shows hepatomegaly and tenderness over the right upper quadrant A liver bi.opsy displays fibroblastic proliferation and trophozoites (shown In the image). Which of the follo v.ring is the most likely diagnosis? ***(A) Ameb1c liver abscess ***(B) Cysuc hydatid chsease ***(C) HepatiC malaria ***(D) Pyogemc hver abscess ***(E) Well disease
38 The answer is A: Amebic liver abscess. Amebic liver abscess is Lhe most common form of extraintestinal amebiasis, although fewer than 30% of patients have a history or antecedent intesti nal amebiasis. The male-to-female ratio is 10:1 , and the dis· ease is rare in children. Amebic liver abscess appears \•.rith an abmpt onset of fever and dull aching abdominal pain in the right upper guadrant or epigastrium, usually lasting less than lO days. jaundice is unusuaL The diagnosis is usually made by radiologic or ultrasound demonstration of the Uver abscess, in conjunction with serologic testing for antibod1es to Entamoeba hisLolytica. Cystic hydatid disease (choice B) is caused by infection with Ed-J inococcus granulosus and is characteJized by cyst [ormation in the liver over several years. Hepatic malaria (choice C) causes hepatomegaly secondary to hypertrophy and hyperplasia of Kupffer cells. Weil disease (choice E) is caused by infections wirl1 Lepwspira spirochetes. ***Diagnosis: Amebic liver abscess
39 A 69-year-old woman arrives in the emergency room com plammg of weakness, abdommal pam, and a 9 kg (20 !b) we1ght loss during the past momh. Physical exammanon reveals jaundtce, consp1cuous hepatomegal}', and asc1tes. The panem exptres, and a section of hver is examined at amops}' (shown m the tmage). Which of the follov.'lng Js the most hkely diagnosis? ***(A) Hemangiosarcoma of the liver ***(B) Metastatic carcinoma of the liver ***(C) Miliary tuberculosis ***(D) Primary hepatocellular carcinoma ***(E) Sarcoidosis
39 The answer is 8: Metastatic carcinoma of the liver. Liver metastasis is the most common cause of massive hepatomegaly and the most common mmor of the lhrer. The liver is involved in one third of all metastatic cancers, including half of those of the gastrointestinal tract, b reast, and lungs. Other tumors that characteristically metastasize to Lhe liver are pancreatic carcinoma and malignant melanoma, although any cancer may find its way to the liver. Al though hemangiosarcomas of the liver (choice A) are frequemly multi[ocal, the mmors are hemorrhagic. Primary hepatocellular earnnoma (choice D) is incorrect because it usually shows a solitary, poorly circumscribed mass, generally in the background of cirrhosis. Miliary tuberculosis (choice C) and sarcoidosis (choice E) feaLUre mm-sized inflammawry nodules (minute granulomas). ***Diagnosis: Metastatic carcinoma of the liver
40 A 22-year-old man presents with a 3-week history of yellow skin and sclerae but is otherwise asymptomatic. He recalls a similar episode 2 years p reviously. Occasion aU)~ he has noticed dark-colored urine. The serum bilirubin is 4.-+ mgldl, mostly in the conjugated fonn. Serum AST and ALT levels a re normaL A liver biopsy is shown in the image. Whkh of the following is the most likely diagnosis? • ._.. .... ou._,-, • ***(A) a ,-Amin-ypsin deficiency ***(B) Crigler-Najjar syndrome ***(C) Dubln-johnson syndrome ***(D) Gilben syndrome ***(E) Wilson disease
40 The answer is C: Dubin-Johnson syndrome. Dubin-Johnson syndrome is a benign autosomal recessive disease that is characterized by chronic conjugated hyperbilimbinemia and conspicuous melanin-like pigment deposition in the liver. The disease is linked w mmarions that result in Lhe complete absen ce of multidrug resistance protein 2 in hepatocytes. The microscopic appearance of the liver in patients witb Dubinjohnson syndrome is normal. except for the accumulation of coarse, iron-free, dark brown granules in hepawcytes and Kupffer cells. Although all of the other choices are genetic disorders that affect the liver, none present with the clinicopathologic findings seen in this case. ***Diagnosis: Dubin-johnson syndrome
41 A 22-year-old woman from lnctia presents with a l -week history of fever, malaise, and nausea. The patient is 6 months pregnant. Physical examination reveals jaundice and right upper quadrant pain. Results of laboratory s tudies include serum bilirubin of 52 mgldL (60% conjugated), AST of 400UIL, ALT of 392 U/L, alkaline phosphatase of 70UIL, ami-HAV antibodies negative, HBsAg negative, and lgM antihepatitis E virus {anti-HEY) antibodies positive. The patient is at high risk for which of the following? ***(A) Diabetes mellitus ***(B) Fulminam liver failure ***(C) Pulmonary thromboembolism ***(D) Renal failure ***(E) Sclerosing cholangitis
41 The answer is B: Fulminant liver failure. The patiem suffers from hepatitis E infection based upon antibody liters against the virus. HEY is an eme1ic Rl'JA virus transmitted by the fecal-oral route. lt accounts for more than half of cases of acute viral hepatitis in young to middle-aged persons in poor regions of the world. HEV is endemic in pans of Asia and South America, where there is poor sanitation. LU<e hepatitis A, hepatitis E usually pre..<>ents as an acute self-limited iJlne.ss. The average incubation period is 35 to 40 days. The symptoms (jaundice, fever, and anhralgia) usually resolve within 6 weeks. Although ove rall mortality rates range from l% to 12%, the disease is especially dangerous in pregnant women due to fulminant hepat1c failure, with mortality rates as high as 20% to 40%. None of the other choices are directly associated wiLh HEV infection. ***Diagnosis: Hepatitis E, acu te
42 A 55-year-old, obese man (BMl = 34kglm2) comes to the physician for a routine physical examination. His past medical history is significant for type 2 diabetes meUitus that is controlled by medication and djet. The patient neither drinks nor smokes. Physical examination shows mild hepatomegaly. Laboratory studies reveal normal serum levels of albumin and bilirubin and mildly elevated serum levels of AST and ALT (80 and lOOU/L, respectively). The serum level of alkahne phosphatase is normal (70 UIL), and total serum cholesterol is elevated to 290 mgldL The CBC is normal. Abdominal ultrasound reveals diiTuse fatty infiltration of the liver. Which of the following is the most likely diagnosis? ***(A) Autoimmune hepatitis ***(B) Cirrhosis of the liver ***(C) Diabetic ketoacidosis ***(D) Glycogen swrage disease ***(E) Nonalcoholic fatry llver disease
- 42 The answer is E: Nonalcoholic fatty liver disease. Nonalcoholic fatty liver is so named because it closely resembles alcoholic fatty live r. This condition represents a specuum of liver injuries that initially display steatosis, with or without hepatitis. Non alcoholic fatty liver not infrequently progresses to bridging fibrosis and cirrhosis of the liver. Risk factors for nonalcoholic fatty liver disease include obesity, type 2 diabetes mellitus, and hyperlipidemia. ChoJCes A, B, and D are not fatty liver diseases. Diabetic ketoacidosis (choice C) may be associated with increased fat in the liver, but the patient clearly does not have this disorder. ***Diagnosis: Nonalcoholic fa tty liver rusease.
43 A 6-monlh-old girl is brought to the physician by her parents, who noticed gradual enlargemen t of the child's abdomen. Physical examination reveals massive hepatomegaly. Laboratory sllldies show norn1aJ serum levels of albumin, bilirubin, and hepatic enzymes. Liver biopsy discloses enlarged hepatocytes with PAS-positive inclusions. Laboratory studies reveal the presence of amylopectin. DepositS of this abnormal glycogen are also found in a biopsy of skeletal muscle. Which of the following is the appropriate diagnosis? ***(A) Andersen disease ***(B) Fab1y disease ***(C) Gaucher disease. ***(D) Krabbe disease ***(E) Metachromatic leukodystrophy
43 The answer is A: Andersen disease. Andersen disease, which is also known as glycogen swrage disease type lY, is an autosomal recessive genetic disease caused by deficiency of a glycogen- branching enzyme. This enzyme deficiency resullS in 1.he accumulation of abnormal glycogen (amylopectin) in the liver, muscle, and other tissues. In most affected persons, the condition becomes evidem in the first months of U fe. Clinical fealUres of Andersen disease include fai lure to thrive and hepatosplenomegaly. The disease course is typically characterized by progressive hepatic fibrosis leading to cirrh osis and liver failure. None of the other choices are glycogen storage diseases. ***Diagnosis: Andersen disease
44 A 20-year·old woman complains of imermitH:nt, colicky abdominal pain, fine tremors of her hands, excess S\'Veatin g, and a general fee ling of restlessness. Laboratory studies reveal an inhe rited defect in the biosynthesis of heme. This patients genetic disease. is most likely caused by deficiency of which of the follo•ving liver enzymes? ***(A) Alanine aminotransferase ***(B) Alkaline phosphatase. (C} PorphobiliJ1ogen deaminase ***(D) Uridine ci.J.phosphate glucuronyl tTansferase ***(E) Uropo rphyTinogen decarbo.li.'Ylase
44 The answer is C: P~rphobilinogen deaminase. Acute intermittent porphyria is the most common genetic porphyria. This autosomal dominam genetic disease is caused by a deficiency of porphobilinogen deaminase activity in Lhe liver. Clinical symptoms include colicky abdominal pain and neuropsychiatiic symptoms. Choices A, B, and D are not involved in heme biosynthesis. Deficiency of uroporphyrinogen decarboxylase (choice E) causes a chronic hepatic porphyria that typically presents \vith cmaneous photosensitivity and iron overload in the middle-aged or elderly. ***Diagnosis: Porphyria, acute intermittent
45 A 4-week-old infam has been evaluated for jaundice and hepatomegaly since binh. Laboratory studies reveal markedly elevated serum levels of bilirubin and alkaline phosphatase and high serum levels of AST and ALT. A liver biopsy is shmvn in the image. Which of the following is the most likely diagnosis? ***(A) Aucoimmune heparlus ***(B) Dubin-jolmson syndrome ***(C) Neonatal hepatitis ***(D) Reye syndrome ***(E) Sclerosing cholangitis
45 The answer is C: Neonatal hepatitis. Histologic features of neonatal hepatitis include prolonged cholestasis, inflammation, and cell injury. Giant cell transformation of heparocytes is common . These cells conlain as lllilny as 40 nuclei and may appear detached from other cells in the liver plate. Most infants with uncomplicated neonatall1epatitis eventually recover. The other choices do not afflict newborns and do not feature multinucleation. ***Diagnosis: Neonatal hepatitis
46 Which of the following is a possible cause of liver disease in the patient described in Question 45? ***(A) Acme hepatitis A ***(B) Annular pancreas ***(C) Biliary atresia ***(D) Cholelith r.
46 The answer is C: Biliary atresia. ln abom hal[ of all cases of neonatal hepatitis, the cause is discemible , and about 30% of cases are assigned to a 1-annrrypsin deficiency alone. Most of the other cases with known causes can be attributed to chromosomal abnonnalities or intrauterine infections. Anmher cause of neonatal hepatitis is biliaty atresia, most often presenting with the extrahepatic form. Unu·eated biliary obsrrucd. on causes progressive fibrosis and may result in secondary biliary cirrhosis. The other choices are not associated with neonatal hepatitis. ***Diagnosis: Biliary atresia
47 Which of the following is the most likely diagnosis? ***(A) Acute cholecystitis ***(B) Acute pancreatitis ***(C) Adenocarcinoma of the gallbladder ***(D) Adenocarcinoma o[ the pancreas ***(E) Primary biliary cirrhosis
47 The answer is A: Acute cholecystitis. Acute cholecystitis refers to diffuse inflammation of the gallbladder, usually seconruny to obsuuction of the gallbladder omlet. Approximately 90% of cases of acme cholecystitis are secondary w gallstones (cholelithiasis). In patients with acute cholecystitis, the external surface of the gallbladder is intensely congested and often layered with a fibrinous exudate. Acute pancreatitis (choice B) is incorrect because the serum amylase level in this patient is normaL Unsuspected gallbladder cancer (choice C) may be discovered in cholecystectomy specimens, bm such an occurrence is uncommon. Pancreatic carcinoma (choice D) often presems as painless jaundice. ***Diagnosis: Acme cholecystilis
48 A 59-year-old man complains of vague abdominal pain, interntiuem fever, and a 9-kg (20-lb) weight loss over the past 8 months. His past medical history is sigrtificant for drug abuse , although he claims lObe drug free for the past 10 years. On physical examination, the patiem shows diffuse abdominal tenderness, hepatomegaly, and mild Jaundlce. Serologic tests for antibodies to Lhe hepatitis B core andgen (lgG ami-HBcAg) and surface antigen (HBsAg) were negative elsewhere. A CT scan of the abdomen reveals a diffusely nodular liver, \¥ith a dominant mass measuring Scm in diameter. A liver biopsy shows neoplastic hepatocytes. Whkh of the follm~o,'ing is the most likely underlying cause of this patients neoplasm? ***(A) a 1-Antirrypsin deficiency ***(B) Autoimmune hepatitis ***(C) Hemochromatosis ***(D) Hepatitis C vims ***(E) Primary blliary cin·hosis
48 The answer is D: Hepatitis C virus (HCV}. Although, hepatitis c has a lower global prevalence Lhan hepatitis B, the ronner is assoclated with most cases of hepatocellular carcinoma in the United States. Most patients with HCV who develop hepatocellular carcinoma have evidence o[ chronic liver disease and cinhosis. a.L-Amitrypsin (choice A) deficiency and hemochromatosis (choice C) are both associated with an elevated risk of hepatocellular carcinoma, but they are far less common than hepatitis C. Primary biliary cirrhosis (choice E) does not lead w hepatocellular carcinoma. ***Diagnosis: Hepatocellular cardnoma
49 A 59-year-old man complains of weakness and 5 kg (l llb) weight loss during the past momh. An abdominal (T scan suggests metastatic cancer involving the liver and the rerroperilOnewn. A CT-guided biopsy displays a poorly-diiierentiated neoplasm. Electron microscopy of the biopsy is shown. Which of the follm.ving organs is the most likely primary site for this patients malignant neoplasm? ***(A) Adrenals ***(B) Kidneys ***(C) Prostate ***(D) Testes ***(E) Thyroid
49 The answer is A: Adrenals. The electron micrograph or this pariem's merastadc cancer shows cells with granules with dense cores. Membrane-bound dense core granules are characteristic of endocrine and neuroendocrine cells such as catecholamineproducing cells and rumors of rhe adrenal medulla (e.g .. pheochromocytoma). None of the other organs is composed of endocrine or neuroendocrine cells. Electron microscopy can be used as an adjunct for pathologic diagnosis when other markers of cellular diffe rentiation are lacking. Carcinomas often exhibit desmosomes and specialized junctional complexes. The presence of melanosomes signifies a melanoma. ***Diagnosis: Pheoch romocywma
50 A 65-year-old man is brought to the emergency room in a dis01iented state. The patient has an odor of alcohol on his breath. Physical examination reveals palmar erythema, diff~use spider angiomata on the upper trunk and face , and gynecomastia. A liver biopsy shows micronodular cinhosis, massive steatosis, and MaUory hyaline. Serum levels of ammonia are elevated. Which o[ the fol1owing is the most like!)' underlying cause of gynecomastia in this patient? ***(A) Hyperbilirubinemia ***(B) Hyperestrogenism ***(C) Hypersensitivity vasculitis ***(D) Hypoalbuminemia ***(E) Ketoacidosis
50 The answer is B: Hyperestrogenism. This patient is in hepatic coma and likely suffers [TOrn cirrhosis. Chronic liver failure in men leads [0 reminization, characterized by gynecomastia, female body habitus, and a change in pubic hair distribution. Other features of chronic liver disease include spider angiomata and palmar erythema. These findings are attributed to reduced hepatic catabolism of estrogens (i.e., hyperestrogenism). The other chokes are unrelated to feminization. ***Diagnosis: Alcoholic liver disease
1 A 1-month-old infant is brought to the ph yskian by her parents. She has had repeated bouts of bilious vomiting over the past month and cannot b e fed adequately. She is in the lOth percentile for weigh t and the 50th percentile for length. An upper Gl series discloses marked narrowing of the midportlon of the duodenum. What is the most likely cause of this infants Gl obstmction? ***(A) Annular pancreas ***(B) Duodenal polyp ***(C) Islet cell adenoma ***(D) Pancreatic pseudocyst ***(E) Pyloric stenosis
1 The answer is A: Annular pancreas. Annular pancreas is a L--- congenital condition in which the head of the pancreas surrounds the second portion of the duodenum. The anomaly may be associated with duodenal atresia. lnfams present with feeding disorders and growth retardation. Pyloric stenosis (choice E) involves the gastric oudet. Duodenal polyp (choice B) does not occur in infants. ***Diagnosis: Annular pancreas
2 A 42-year-old obese woman (BMI = 32 kglm2) presems with severe abdominal pain that radiates to the back. There is no hiswry of alcohol or dmg abuse. The blood pressure is 90/45 mm Hg, respirations are 32 per minute, and pulse is 100 per minute. Physical examination shows abdominal tenderness, guarding, and rigidity. An X-ray film of the chest shows a left pleural effuslon. Laboratory studies reveaJ elevated serum amylase (850UIL) and lipase (675UIL), and hypocalcemia (7 .8 mgldl). Which of the follo'Ning is the most likely diagnosis? ***(A) Acme cholecystitis ***(B) Acme pancreatitis ***(C) Alcoholic hepatitis ***(D) Chronic calcifying pancreatitis ***(E) Dissecting aortic aneurysm
2 The answer is B: Acute pancreatitis. Acute pancreatitis is defined as an inflammatory condition of the exoctine pancreas that results from injury to acinar cells. The disease presenLc; with a spectrum of signs and symptoms. Severe forms are characterized by the sudden onset of abdominal pain, often accompanied by signs of shock (hypotension, tachypnea, and tachycardia). The release of amylase and lipase from the injured pancreas into the serum provides a sensitive marker for monitoring injury to acinar cells. Left pleural effusion is a common finding in patients with acme pancreatitis due to local irritation below the diaphragm. The other choices do not feature increases m serum amylase and Upase. ***Diagnosis: Pancreatitis, acute
3 Which of the follO\.ving is most likely associated with the pathogenesis of the condition of the patient described in Question 2? ***(A) Carcinoid syndrome ***(B) Cholelithiasis ***(C) lnsulinoma ***(D) Pancreatic adenocarcinoma ***(E) Portal hypertension
3 The answer is B: Cholelithiasis. Some 45% of all patients with acute pancreatitis also have cholelithiasis, and the risk of developing acute pancreatitis in patients with gallstOnes is 25 times higher than that in the general population Chronic alcoholism accoums for approximately one third of the cases of acute pancreatitis. Other causes include obsnuction of the pancreatic duct by gallstones, intake of d rugs, and viral infections. The other choices do not cause acme pancreatitis. ***Diagnosis: Pancreatitis, acute; cholelitluasis
4 A 60-year-old alcoholic man presems wilh a 6-month history of recurrem epigastric pain, progressive \Veight loss, and foulsmelling diarrhea. The abdominal pain is now almost constam and imractable. An X-ray film of the abdomen reveals multiple areas of calcification in the mid-abdomen. Which of the following is the most likely diagnosis? ***(A) Carcinoid syndrome ***(B) Chronic pancreatitis ***(C) Crohn disease ***(D) lnsuHnoma ***(E) Miliary tuberculosis
4 The answer is B: Chronic pancreatitis. Chronic pancreatitis is .....__ characterized by the progressive destruction of the pancreas, with accompanying in·egular fibrosis and chronic inflammation. Calcification and intraductal calculi often develop. Pancreatic insufficiency results in malabsorption syndrome. Chronic pancreatitis is most commonly seen in paLiems with a histmy of alcohol abuse (70% of cases). The orher choices are not associated with pancreatic calcifications. Although islets may be affected by chronic pancreatitis, hypoglycemia is an uncommon and late feamre of the disease. ***Diagnosis: Pancreatitis, chronic
5 Which of the fo llowing findings is most Hkely to he encoun- L-- Lered in rhe pariem described in Question 4? ***(A) Achlorhydria ***(B) Hypoglycemia ***(C) Melena ***(D) Pernicious anemia ***(E) Steatorrhea
5 The answer is E: Steatorrhea. Fat malabsorption in the setting of chronic pancreatitis is most often assoctated with steatorrhea. In patients with steato rrhea, the fecal matter is foul smelling and floats because of a high fat content. Longstanding malabsorptive disease is accomparued by nutritional deficiency, including weight loss, anemia, osteomalacia, and a tendency w bleed. Hypoglycemia (choice B) is incorrect because loss of pancreatic islet cells would be associated \vith hyperglycemia. ***Diagnosis: Pancreatitis, chronic; steaton hea
6 A 50-year-old woman complains of persistem abdominal pain, anorexia, and abdominal distention . . Her past medical histmy is significant for a previous hospitalization for acute pancreatitis. Physical examination shows jaundice and a nonpulsati le abdominal mass. Laboratory studies reveal nom1al serum levels of bilirubin, AST, and ALT. A CT scan of the abdomen shows a fluid -filled cavity in the head of the pancreas. What is the most likely diagnosis? ***(A) Acute hemonhagic pancreatitis ***(B) lnsulinoma ***(C) Pancreatic cystadenoma ***(D) Pancreatic islet cell tumor ***(E) Pancreatic pseudocyst
6 The answer is E: Pancreatic pseudocyst. Pancreatic pseudOC)'St ls a ]ate complication or acme pancreatitis, in \>Vh ich necrotic pancreatic tissue is liqUefied through the action of panc reatic enzymes (e.g., peptidases, Upases, and amylase). The necrotic tissue becomes encapsulated by granulation tissue, which then develops into a fibrous capsule. Pseudocysts may enlarge lO compress and even obstntct the duodenum. They may become secondarily infected and form an abscess. Choices B, C, and D are not consequences of acute pancreatitis. ***Diagnosis: Pancreatic pseudocyst
7 The surgical specimen is shown in Lhe image for Lhe paLiem L-- described in Ques[ion 6. In addilion lO blood and necrmic debris, which of Lhe following best desctibes che comems of Lhis cystic lesion? ***(A) Bile ***(B) Chylous fluid ***(C) Lymph ***(D) Mucopolysaccharides ***(E) PancreaLic enzymes
7 The answer is E: Pancreatic enzymes. Pancreatic pseudocysts are lined by con nective tissue and contain blood, necrotic debris, and secreted pancreatic enzymes. Renux of bile (choice A) is not characteristic of a pancreatic pseudocyst. The other choices (B, C, and D) may be present in small quantities. ***Diagnosis: Pancreatic pseudocyst
8 A 60-year-old man presents with a 3-week histmy of weight loss, vague abdominal pain, and progressive yellowing of his skin and scleTae. He also reports Lhe recem onset of intermitLent pain in the upper and lower eA.1:remities. LaboratOry studies show a serum bilimbin level of 15 mgldL, mostly in the conjugated fonn. ACT scan of the abdomen reveals a mass in the head of the pancreas. The patient develops sudden shortness of breath and is diagnosed with puhnonary thromboembollsm. Which of the following is the most likely cause of thromboembolism in this patient? ***(A) Aden ocarcinoma o[ the ampulla of Vater ***(B) Gastrinoma of the pancreas ***(C) lnsulinoma o[ the pancreas ***(D) Pancreatic adenocarcinoma ***(E) Pancreatic pseudocyst
8 The answer is D: Pancreatic adenocarcinoma. Adenocarcinoma is the most common malignant tumor of the pancreas. Although it accounts for only 3% of all cancers in the United Stales, it is the fou rth leading cause of cancer death in men and the fifth leading cause of cancer death in women. Migratory thrombophlebitis, whi.ch is also refened to as Trousseau syndrome, may accompany adenocarcinoma of the pancreas as well as other malignancies. The cause or migrat01y thrombophlebitis is not enti rely understood, bUL it is Lhoughr that Lhe LUmo r releases thrombogenic substances into the circulation (e.g , serine proteases) that initiate the coagulation cascade. The CT scan excludes adenocarcinoma of the ampulla of Vater (choice A). Endocrine tumors of Lhe pancreas (choices Band C) are no t expected to induce Trousseau syndrome. ***Diagnosis: Pancreatic adenocarcinmna
9 Despite best efforLS at treatmem, the patient described in QuesLion 8 subsequently dies. The gross appearance of the pancreas and liver aL amopsy is shown in the image. This patients mmor most likely arose fmm which of the following types o[ cells? ***(A) AciJ1ar cells ***(B) Alpha cells ***(C) Beta cells ***(D) Delta cells ***(E) Ductal cells
9 The answer is E: Ductal cells. The majority of pancreatic carcinomas arise from pancreatic duel epithelium. Acinar cell carcinoma (choice A) is much less common. The other chokes represent uncommon islet cell tumors. ***Diagnosis: Pancreatic adenocarcinoma
A 65-year-old woman presents with a '5-week hismry of yellow skin and sclera, anorexia, and epigastric pain. Her past Tnedical hiswry is significam for insulin-dependem diabetes melHrus. She smoked one pack of cigarettes a day for the past 20 years. Physical examination reveals jaundice and a palpable gallbladder. Laboratmy studies show a serum bilirubin level of lOmgldL, mostly in the conjugated fom1, and an elevated alkaline phosphatase (260U/L). A CT scan of the abdomen discloses a mass in the head of the pancreas and multiple nodules in the liver measuring up to 3 em. Which of the fol lowmg is the most likely cause of jaundice in this patient? ***(A) Cholelithiasis ***(B) Cirrhosis ***(C) Extrahepatic biliary obstruction ***(D) Hemolysis ***(E) lmrahepatic cholestasis
10 The answer C: Extrahepatic biliary obstruction. Pancreatic adenocarclnomas often cause obsnuction of the common bile duct due to the proximity o[ the duct w the head of the pancreas. Painless jaundice is a frequent initial symptom of pancreatic cancer. Dilation of the gallbladder in this setting is tenned Courvoisier sign . Cirrhosis (choice B) is a late complication of extrahepatic biliary obstruction. ***Diagnosis: Pancreatic adenocarcinoma
11 vVhich of the following is the most 1mponam risk factor for the neoplasm atising in the patient described in Question 10? ***(A) Alcohol abuse ***(B) Cholelithiasis ***(C) Cigarette smoking ***(D) Diabetes mellims cype l ***(E) High-fat diet
11 The answer is C: Cigarette smoking. Cigareue smoking is associated with a fivefold increased 1isk for adenocarcinoma of the pancreas. Cholelithiasis (choice B) and alcohol abuse (choice A) are associated witl1 pancreatitis, not pancreatic adenocarcinoma. ***Diagnosis: Pancreatic adenocarcinoma
12 A 47 -year-old man suffers from long-standing peptic ulcer disease. which is largely unresponsive w pharmacologic therapy. Endoscopic examination reveals multiple, nonhealed ulcerations of the duodenum and jejunum. Whtch of the following is the most likely diagnosis? ***(A) Carcinoid syndrome ***(B) lnsulinoma ***(C) Pancreatic adenocarcinoma ***(D) Verner-Morrison syndrome ***(E) Zollinger-Ellison syndrome
12 The answer is E: Zollinger-EIIison syndrome. ZollingerEllison syndrome is charaC£crized by int:ractable gastric hypersecretion, severe peptic ulceration of the duodenum and sometimes the jejunum, and elevated levels of gastrin in blood. The LUmor responsible for Zollinger-Ellison syndrome is pancreatic gasninoma composed of G cells. Gastrinomas arc most often located in the pancreas, but they may arise in orher pans of the gastrointestinal tract, notably the duodenum. Most gastrinomas are malignant. Carcinoid syndrome (choice A) is a systemk paraneoplastic disease caused by the release of hom10nes from carcinoid rumors into venous blood. Clinkal features of carcmoid tumors (e.g., flushing, bronchial wheezing, watery diarrhea, and abdominal colic) are presumably caused by the release of serotonin, bradykinin , and h1stamine. ***Diagnosis: Gastrinoma, Zollinger- Ellison syndrome
13 A 35-year-old woman presents \.vith 6-month history of skin rash and fatigue. Physical examination shows pallor and a necrotizing erythematous skin rash of her lower body Laboratory studies reveal mild anemia and fas ting blood glucose of 160 mgldL. A CT scan of the abdomen demonstrates a 2-cm mass in the pancreas. Which of the following is the most likely diagnosis? ***(A) Carcinoid tumor ***(B) Gasninoma ***(C) Glucagonoma ***(D) lnsulinoma ***(E) Pancreattc polypeptide-secreting mmor
13 The answer is C: Glucagonoma. Necrotizing migratmy erythema develops in assoc1ation with the hypersecretion of glucagon by alpha cell-containing tumors {glucagonomas). These patients also have mild hyperglycemia and anemia. The other choices do not presen t with these clinical signs and symptoms. ***Diagnosis: Glucagonoma
A 40-year-olcl woman comes w the physician with a 6-week histOJ)' of episodic hunger and fainting spells. She is currently seeing a psychiatrist because she is ir ritable and quaneling with her family. Laboratory studies show a serum glucose concentration of 35 mgldl. A CT scan of the abdomen demonstrau::s a 1.5-cm mass in the pancreas. TI1e gross appearance of the bisected tumor is shown ln the image. What is the most likely diagnosis? ***(A) Adenoc-.ncinoma ***(B) Gastrinoma ***(C) Glucagonoma ***(D) Insulinoma ***(E) Somawstatinoma
14 The answer is D: lnsulinoma. lnsulinoma is t.he most common islet cell rumor. These tumors of the endocrine pancreas are low-grade malignant neoplasms. lnsulinomas secrete insulln and cause hypoglycemia. Symptoms of hypoglycemia include hunger, sweatlng, irritability, epileptic seizures, and coma. Infusion of glucose alleviates these symptoms. The other tumors do nol cause marked hypoglycemia. Patients with a glucagonoma (choice C) typically presem with necrotizing migratory erythema, mUd hyperglycemia, and anemia. Patients with a somatostatlnoma (choice E) l)'pically presem with mild diabetes mellitus, gallslones, steatorrhea, and hypochlorhydria. ***Diagnosis: lnsulinoma
15 A 36-year-ol.d \VOman complains of a 4-week history o r unremitting watery diarrhe."l. She repons that she is always Lh1rsty and dtinks continuously. Laboratory studies show achlorhydria, hypokalemia, and mild acidosis. A CT scan or the abdomen demonstrates a 1.5-cm pancreatic mass. vVhicl1 of the [ollo·wing is the most likely diagnosis? ***(A) Carcinoid tumor ***(B) Gastrinoma ***(C) Pancreatic polypeptide-secreting tumor ***(D) Somawstatinoma ***(E) VlPoma
15 The answer is E: VlPoma. intractable diarrhea, hypokalemia, and low levels of chloride in gastric jllice constitULe the S}rndrome of "pancreatic cholera." This disorder is secondary to the secretion of vasoactive intestinal polypeptide (VlP) by an islet celJ tumor. VlP stimulates adenylyl cyclase activity, wluch in tum leads to the production of large amounts of cAMP. The lauer causes increased secretion of potassium and water into the intestinal lumen. The ensuing diarrhea results in toss of wmer, amounting to as much as 5 L per day The other choices do not present with these signs and symptoms. ***Diagnosis: VlPoma
16 A 45-year-old woman complains of right upper quadrant abdominal pain, weight loss. dry momh, lncreased urine p roduction, and foul-smelling fatty stools. She has a recem hiswry of mild diabetes mellirus. Abdominal ultrasound examination reveals gallstones and a solitmy 1.5-cm mass in the pancreas. Which of the following hormones would most likely be elevated in the blood of this patient? ***(A) Calcitonin ***(B) Gastlin ***(C) Insulin ***(D) Somatostatin ***(E) Vasoactive imestinal polypeptide
16 The answer is D: Somatostatin. Delta cell rumors (somatostatinomas) produce a syndrome consisting of m1ld diabetes mellin1s, gallstones, steatorrhea, and hypochlorhydria. These effects result from the inhibitory action of somawstatin on the secretion of h ormones by cells of the endocrine pancreas, acinar cells of the pancreas, and certain hormone-secreting cells in the gastrointestinal tract. Somatostatin also inhibits the pituitary release of growth hormone. None of the other choices are associated wi th mild diabetes or cholelithiasis. ***Diagnosis: SomatostaLinoma, cholelithiasis
17 A 35-year-old man complains of severe acute periumbilical pain that radiates to his back and nausea. The patient recently had a heart transplant for idiopathic cardiomyopathy and is taking azathioprirte for immunosuppression. Physical examination reveals bruising of both flanks. Blood pressure is 120/70mmHg, pulse rate 100 per minute, and temperature 37 .8°C ( l00°F). Laboratory studies show elevated serum levels of amylase (950U!L) and lipase (780U!L), nonnallevels of serum calcium, and a nonnal serum lipid profile. The patient expires, and the pancreas is examined at autopsy (shown in the image). Which of the following is the most li kely underlying cause of these pathologic findings? ***(A) Acute ischemia ***(B) Drug-induced pancreatins (C} Graft-versus-host reaction ***(D) Hypercalcemia ***(E) Hyperlipidemia
17 The answer is B: Drug-induced pancreatitis. Acute pancreatitis may be encountered in patients taking immunosuppressive drugs, antineoplastic agents, suUonamides, and dim·edcs. Severe cases of acute pancreatitis cause retroperitoneal hemorrhage, which can track to the nank and petiumbilkal region (see photograph). The other choices may induce pancreatitis but are exceedingly unlikely in this clinical setting. ***Diagnosis: Pancreatitis, acute
18 A 63-year-old woman presents with a 6-month history of recurrent epigastric pain and nausea. Abdominal ultrasound reveals a 13-mm hypoechoic lesion in Lhe rail of the pancreas. Physical examination shows nushing of the face , periorbital edema. and hypotension (blood pressure = 90/50nunHg). LaboraLOry studies disclose nonnal serum levels of gastrin, amylase, in.suUn, and vasoactive intestinal polypeptide. Urinalysis demonstrates elevated levels o[ metanephrines (lOmg per 24 hours) Which or the following is the most likely diagnosis? ***(A) Adenocarcinoma o[ pancreas ***(B) Glucagonoma ***(C) lnsu]jnoma ***(D) Pancreatic carcinoid ***(E) Somatostatinoma
18 The answer is D: Pancreatic carcinoid. Carcinoid tumors of the pancreas are rare malignant neoplasms that closely resemble intestinal carcinoids. 'vVhen confined to the pancreas, Lhey may induce the so-called atypical carcinoid syndrome, wh ich is associated with severe facial flushing, hypotension, peliorbital edema, and tearing. Hepatic metastases cause the fu!J blown carcinoid syndrome. Adenocarcinoma of the pancreas (choice A) does not produce hormones. The other chmces lead to other endocrine syndromes. ***Diagnosis: Carcinoid tumor
19 A 65-year-old man \J..'i.th a history of acromegaly complains of recurrem epigastric pain and dark-colored tarry stools. Laboratory studies reveal moderate hypercalcemia, hyperlipidemia, and elevated serum levels of PTI-1 and gastrin. Serum glucose is within nom1al limits. Abdominal ultrasound shows a mass in the tail of the pancreas. Which of the following is the most likely diagnosis? ***(A) Glucagonoma ***(B) Insu}jnoma ***(C) Multiple endocrine neoplasia type l ***(D) Multiple endocriJ1e n eoplasia type 2 ***(E) Pancreatic cardnoid
19 The answer is ( : Multiple endocrine neoplasia type 1. The patient exhibits signs and symptoms of MEN-1 (Wermer syndrome), including adenoma of the pituitary (acromegaly), hyperplasia or adenoma of the parathyroids (hypercalcemia), and adenoma of the endocrine pancreas (gastrinoma). Gastrin-producing tumors of the pancreas may produce Zollinger-Ellison syndrome, chara cterized by intractable peptic ulcers. MEN-1 is caused by mutations in the MENI tumor suppressor gene. MEN-2A (Sipple syndrome, choice D) fe.atures medullary thyroid carcinoma and pheochromocytoma, and is associated with RET protooncogene mutations. ***Diagnosis: Zollinger-ElJ]son syndrome, multiple endoc1ine neoplasia
1 The mother or a 2-momh-old child palpates a mass on the left side of the childs abdomen. Vital signs are normal. A CT-gulded renal biopsy sl1ows undifferentiated tubules surrounded by undifferentiated mesenchyme, smooth muscle, and islands of cartilage. The mass is removed (shown in the image) and displays variably sized cysts Which of Lhe following is the most likely diagnosis for this chUd's Oank mass7 ***(A) Infamile polycystic disease ***(B) Medullary sponge kjdney ***(C) Neuroblastoma ***(D) Renal dysplasia ***(E) Wilms tumor
1 The answer is D: Renal dysplasia. Renal dysplasia is characterized by undifferentiated tubular structures surrounded by primitive mesenchyme, sometimes with heterotopic Lissue such as smomh muscle and cartilage. Cysts often fonn from the abnormal mbules. Renal dysplasia results £rom an abnormality in meLanephric differentiation. Variants of renal dysplasia include aplastic, multicysric (seen in this case), diffuse cystic, and obstructive forms. ln most patients with multicystic renal dysplasia, a palpable flanl< mass is discovered shortly after birth. Unilateral multicystic renal dysplasia is the most common cause of an abdominal mass in newborns. Infantile polycystic disease (choice A) is invanably bilateral, and the kidneys are usually very large. Medullary sponge kidney (choice B) is charactetized by multiple small cysts in the renal papil1ae. Wllms mmor (choice E) may comain heterologous elements but does not fom1 large cysts. ***Diagnosis: Multicystic renal dysplasia
2A 38-year-old man presems with vague flank pain and describes the passage of blood clots in his mine. Phystcal examination reveals bilateral fian.k and abdominal masses. Laboratory smdies show elevated blood urea ninogen and creatinine. Urinalysis reveals hematuria, proteinuria, and oliguria A CT scan discloses bilaterally, massively enlarged kidneys. The patient subsequently develops end-stage kidney disease and receives a renal transplant. The patiems kidneys are removed during surgery (shown in the image). What is the most likely diagnosis? ***(A) Autosomal dominant polycystic kidney disease ***(B) Autosomal recessive polycystic kidney disease ***(C) Hydronephrosis ***(D) Medulla1y sponge kidney ***(E) Multicystic renal dysplasia
2 The anlwer is A: Autosomal dominant polycystic kidney disease. Autosomal dominam polycystic kidney disease , wruch is charanerized by enlarged multicystic kidneys, is the most common of a group of congenital diseases that are characteJized by numerous cysts witrun the renal parenchyma. Most cases are caused by mu tations in the polycystic kidney disease 1 gene, which encodes polycystin (function unknown). Half of all patients evemually develop end-stage renal failure. Most patients develop clinical manifestadons in the founh decade of life, which i.s why this condition was also called adul t polycystic kidney disease. Symptoms include a sense of heaviness in the loins, bilateral flank and abdominal masses, and passage of blood clots in the urine. Azotemia is common and, in l1alf of patients, progresses to uremia (clinical renal failure) over a period of several years. Autosomal recessive polycystic kidney disease (choice B) occurs in infants. Hydronephrosis (choice C) docs not feature multiple cysts. Medullary sponge kidney (choice D) consists of multlple small cysts. Multicystic renal dysplasia (choice E) is usually unilateral. ***Diagnosis: Autosomal dominant polycystic kidney disease
3 The patient described in Question 2 carries an increased 1isk for which of the follovving abnormalities? ***(A) Hepatic cysts ***(B) Horseshoe kidney ***(C) Pulmonary cysts ***(D) Renal cell carcinoma ***(E) Transitional cell carcinoma of the bladder
3 The answer is A: Hepatic cysts. One thi.rd of patients with autosomal dom.inant polycystic kidne}' disease (ADPKD) also have hepatic cysts, whose lining resembles bile duct epithelium. The other ch oices do not arise in ADPKD. ***Diagnosis: Autosomal dominant polycystic l<idney disease
4 A 46-year-old woman p resentS with a 6-month history of .._ vague upper abdominal pain after fatty meals, some abdominal distension, and frequent indigestion. Physical examination shows an obese woman (BMI = 32 kglm2) \.Vith right upper quadrant tenderness. A (T scan discloses gallstones and an ectopic kidney. Which o[ the following is the expected location of the ecLOpic kidney? ***(A) Adjacem to gallbladder ***(B) Atmched to the left adrenal gland ***(C) f used laterally with the contralateral kidney ***(D) Pelvis ***(E) Poste tior epigasnium
4 The answer is D: Pelvis. Most ectopic kidneys are located along the pathway of renal migration during fetal development and are caudal to their nomtallumhar position Duting fetal life, the kidneys are i.nitially located in the lower abdomen. As development progresses, they nonnally move upward toward their permanent position. Kidneys that do not reach the lumbar area but remain in the pelvis or presacral area are considered ectopic Fusion of both kidneys results in so-called h orseshoe kidney. ***Diagnosis: Ectopic kidney
5 A 12-year-old girl complains of swelllng of her eyelids. abdo- .._ men, and ankles. She had been in good health until several months ago, when she gained some weight and noted swelUng of her lower legs. An X- ray film of the chest shows bilateral pleural effusions, without evidence of lung disease. Urinalysis reveals heavy proteinuria (8g per 24 hours) without hematuria. A percutaneous needle biopsy of the kidney discloses no morphologic abnormalities by light microscopy. Which of the following best describes this patient's medical condition? ***(A) Amyloid nephropathy ***(B) focal segmental glomemlosclerosis ***(C) Hereditary neph ritis ***(D) Membranous glomerulopm.hy ***(E) Nephrotic syndrome
5 The answer is E: Nephrotic syndrome. This patient has minimal change glomemlopat.hy with nephmtic syndrome. The nephrotic syndrome is charactetized by heavy proteinmia (>3 Sg protein per 24 hours), hypoalbuminemia, hyperlipidemia, and edema. In minimal change glomerulopathy, there is effacemem of visceral epithelial cell (podocyte) fom processes, which allows protein to be lost from the plasma into the urine (proteinuria). The mher choices are characterized by morphologic changes in glomeruli. ***Diagnosis: Nephrotic syndrome, minimal change disease
6 An 8-year-old boy presems with headaches, dizziness, and .._ malaise. He was seen for a severe sore throat 2 weeks ago. Physical examination reveals facial edema. The blood pressure is 180/llOmm Hg A 24-hour urine collec[ion demonstrates oliguria, and minalysiS shows hematuria. Which of the folIo ·wing best desc1ibes this patient's medical condition? ***(A) HerediLary nephritis ***(B) Membranous glomerulonephritis ***(C) Minimal change nephritic syndrome ***(D) Postinfectious glomerulonephritis ***(E) Thin glomerular basemem membrane nephropathy
6 The answer is D: Podinfectious glomerulonephritis. This case is illustrative of nephritic syndrome in the setting of poststrepwcoccal glomemlonephritis. Nephritic syndrome is charactelized by hemawria (either microscopic or visible grossly), variable degrees of proteinuria, and decreased glomerular filrratio. lt results in elevations of sentm blood urea nitrogen and creacinine, as well as oliguria, salt and waLer retention, edema, and hypertension. Glomemlar diseases associated with the nephritlc syndrome are caused by inflammatory changes in glomeruli, such as infiltration by leukocytes, h)1)erplasia of glomerular cells, and, in severe lesions, necrosis. The other choices are not related to streptococcal pharyngitis. Choices B, C, and E do not present with hematuria. ***Diagnosis: Postinfectious glomerulonephritis, nephritic syndrome
7 'vVhat finding on microscopic u tinalysis indicates th at hematuria in the patient described in Question 6 is caused by a renal process, rather than bleeding from another site in the urinary tract? ***(A) Blood clolS ***(B) Hemoglobin crystals ***(C) Phagocytosed hemoglobin ***(D) Red blood cell caslS ***(E) White blood cell casts
The answer is D: Red blood cell casts. Injury to the glomerular capiUaries resu Its in spillage of protein and blood cells into the urine. Hematuria is also seen in pad ems 'vVith bleeding from the lower urinary tract. However, RBC casts in the urine sediment miginate from erythrocytes compacted during passage lhrough the renal tubules and denote a renal origin of hemamria. ***Diagnosis: Posdnfecl:ious glomerulonephritis, nephritic syndrome
8 A 60-year-old man complains of chronic back pain and fatigue, excessive urination, and increased thirst. X-ray exammarion reveals numerous lytic lesions in the lumbar venebral bodies . Laboratory srudies show hypoalbuminemia, mild anemia, and thrombocywp enia. U1inalysis displays 4+ proteinuria. A monoclonal immunoglobulin light-chain peak is demonstrated by serum electrophoresis. A bone marrow biopsy discloses foci o[ plasma cells, which accoum for 20% of an hemawpoietic cells. A kidney biopsy is obtained (shown in the image). Which of the following is the most likely cause of n ephrotic syndrome in this patient? ***(A) Amyloid nephropathy ***(B) Crescentic glomemlonephritis ***(C) lgA nephropathy (Berger disease) ***(D) Membranous glomerulonephritis ***(E) Nodular glomerulosclerosis (Klmmelstiel-'vVilson disease)
8 The answer is A: Amyloid nephropathy. The clinicopathologic findings establish a diagnosis of multiple myeloma. The neoplastic plasma cells typically secrete a homogeneous immunoglobulin cham, which can be detected in semm or urine by electrophoresis. Amyloid nephropathy is caused by the deposition of secreted Lambda or kappa light chains in the glomerular basemem membranes and mesangial matrix. Amorphous acellular material expands the mesangium and obstructs the glomerular capillaries. Deposits of AL amyloid may also appear in tJ1e tubular basement membranes and in the walls of renal vessels . Renal amyloidosis usually presents with neph· rotic syndrome. The deposits of amyloid may take on a nodular appearance, reminiscent of the Kimmelstiel-Wilson lesion of diabetic glomerulosclerosis (choice £). However, amyloid deposits are not PAS positive and are identifiable by Congo red staining \Vith characteristic apple-green birefringence. IgA nephropathy (choice C) and membranous glomerulonephritis (choice D) are unrelated to light-chain disease. ***Diagnosis: Amyloid nephropathy, multiple myeloma
9 A 49-year-old man with a history of heavy smoking presenrs .....__ with a 5-year hisrory of shortness of breath and cough and production of abundam foul-smelling spmum. A pulmonary work-up demonstrates chronic bronchiectasis. Laboratory srud:ies reveal hypoalbuminemia and hyperlipidemia. Utinalysis shows heavy proteinuria (>4 g per day). Which of the following is the appropriate diagnosis' ***(A) Amyloid nephropathy ***(B) Berger disease (lgA nephropathy) ***(C) Focal segmental glomerulosclerosis ***(D) Minimal change glomerulopathy ***(E) Wegener granulomatosis
9 The answer is A: Amyloid nephropathy. Amyloidosis is a wellknown complication of chronic inflammatory disorders, such as chronic suppurative bronchiectasis, rheumatoid arthritis, or osteomyelitis. These conditions s timulate the production of amyloid from the senun amyloid A (SAA) protein , an acutephase reactant secreted by the liver. The kidneys, liver, s pleen, and adrenals are the most common organs involved. Renal amyloidosis leads w nephrotic syndrome (as in this case) and renal failure. Nephrotic syndrome caused by deposition of SAA amyloid is clinically indistinguishable from that related w Al amyloid. The mher choices have not been linked w chronic in£lammawry conditlons. Wegener granulomatosis (choice E) affects the lungs and kidneys. bm bronchiectasis is not a f earure of th Lc;; disease. ***Diagnosis: Amyloid nephropathy, bronchiectasis
tO A 12-year-old boy complains of swelling of his feet for the past J weeks. He is otherwise healthy, with no known previous illness. Vilal signs are normaL Physical examination reveals pitting edema of the lower legs and a swol1en abdomen. Urinalysis shows 4+ protein but no RBCs or WBCs. Which of d1e following are the most likely diagnoses to consider in your evaluation of this patient7 ***(A) Henoch-Schonlein purpura, lupus nephritis ***(B) Mallgnam hypertension, renal vein Lhrombosis ***(C) Minilnal change disease, focal segmental glomerulosclerosis ***(D) Pyelonephritis, acute tubular necrosis ***(E) 'vVilms tumor, renal cell carcinoma
10 The answer is C: Minimal change disease, focal segmental glomerulosclerosis. Minimal change glomerulopathy causes 90% of the nephrotic syndrome in young children and 15% in adults. Proteinmia is generally more selective (albumin > globulins) than in the nephrotic syndrome caused by other diseases. but there is too much overlap for this selectivity to be used as a diagnostic criterion. This disease is characteri zed pathologically by fusi.on (effacement) of visceral epithelial foot processes; however, this can be vist1al izecl only by electron m.icroscopy. Minimal change glomerulopathy is successfully treated \•lith corticosteroids and does not progress to renal failure. Focal segmental glomerulosclerosis (FSGS) is the cause of nephrotic syndrome in 10% of children and 30% of adults. FSGS is the term applied to a heterogeneous group of glomerular diseases that have different causes, tncluding mutations, vilu ses , drugs, and semm factors. Henoch-Schonlein purpura and lupus neph1itis (choice A) generally presem with nephritic syndrome and rash, among other signs and symptoms. vVilms rumor (choice E) is not a cause of nephrotic syndrome. ***Diagnosis: Minimal change nephrotic syndrome
11 A 4-year-old girl presents with S\.velling of the legs and ankles. Physical examination reveals pitting edema of the lower extremities. Urinalysis show 2+ proteinuria. The urinary sediment contains no inflammatory cells or red blood cells. Serum levels of BUN and creatinine are normaL The patient recovers compleLely after a course of corticosteroids. Which or the following pathologic findings might be expected in the urine prior to treatment with corticosteroids? ***(A) Amyloid casts ***(B) EosinophUs ***(C) Lipid droplets ***(D) Red blood cell casts ***(E) White blood cells casts
1 t The answer is ( : Lipid droplets. The loss o[ protein in the urine in pallems with nephrotic syndrome leads to hypoalbuminemia. A compensatory increase in Lipoprotein secretion by the liver results in hyperlipidemia, which is reflected in the presence of lipid droplets in the urine. For this reason, minimal change disease is also referred to as lipoid nephrosis. EosinophUs (choice B) may be observed in allergic nephr itis. White blood cell casts (choice E) are features of pyelonephritis ***Diagnosis: Minimal change nephrotic syndrome
12 For the patiem described in Question 11, elecnon microscopy of a renal biopsy specimen prior to treatment would mosL likely demonsnaLe '<Vhich of the following abnormalities? ***(A) Duplkation of capillary basement membranes ***(B) Electron-dense immune deposits in the capillary basement membranes ***(C) Electron-dense immune deposits in the mesangium ***(D) Fusion of podocyte foot processes ***(E) Loss of microviUi by the mbular lining cells
12 The answer is 0: Fusion of podocyte foot processes. The light microscopic appearance of glomemli in minimal change glomerulopathy is essentially nonnaL However, eleCLron microscopic e,'Caminadon of glomemli reveals roral effacement of visceral epithelial cell foot processes. This retracrion presumably results from extensive cell S\Velllng and occurs in virtually all cases of proteinuria in the nephrotic range. It is not a specific marker, but is characteristic of minimal change glomemlop<uhy. Minimal change disease is not cha:racte1ized by deposits of immune complexes (choices Band C) and does not reflect changes in the basemem membrane (choice A). Choice E is incoiTect because minimal change disease involves changes in glomemli, not renal tubules. ***Diagnosis: Minimal change nephrotic syndrome
13 A 44-year-old man complains of swelling of his legs and puffiness around his eyes. His abdomen has become promberant and he feels short of breath. Physical examination reveals generalized edema and ascites. Total serum protein is 5.2 gldL (reference = 5.5 to 8 0 gldL), and albumin is 1.9 gldL (reference = 3 .5 to 5.5 g!dL). Serum cholesterol is elevated at 530 mgldL There are 5 g of protein in a 24-hour urine collection. The urinary sedimem contain..<; many hyalin casts but no RBCs or inflammatory cells. A renal biopsy stained by direct immunofluorescence for IgG is shown in the image. Which of the follmving is the most likely diagnosis' ***(A) Amyloid nephropathy ***(B) Focal segmental glomerulosclerosis ***(C) Membranoproliferative glomemlonephritis type l ***(D) Membranous glomerulopathy ***(E) Minimal change disease
13 The answer is 0: Membranous glomerulopathy. Membranous glomemlopathy is a frequem cause of the nephrotic syndrome in adults and is caused by the accumulation of immune complexes in the subepithelial zone of glomerular capiltaties. Immunofluorescence mkroscopy shows granular deposits of IgG outlining the glomerular capillary loops. The course of membranous glomemlopathy is highly va1iable, \'\rith a range of possible outcomes, including spomaneous remission (25%). persistent proteinmia and srable or only partial loss of renal funnion (50%), and renal faiJure (25%). Membranoproliferallve glomerulon epluitis type 1 (choice C) is a chmnic immune complex disease that fearures granular deposition of immunoglobuli11 and complement in glomerular capillary loops and mesangium. ***Diagnosis: Membranous glomerulopathy
14 The pathogenesis of nephrotic syndrome in the patient described in Question 13 is best characteiized by which of the follmving mechanisms of disease? ***(A) Deposition of anti glomerular basement membrane antibody ***(B) Deposition of lgA in the mesangium ***(C) Expansion of the glomerular basemem membrane with PAS-positive glycoprOLeins ***(D) Subendothelial deposits of immune complexes ***(E) Subepithelial deposits of immune complexes
14 The answer is E: Subepithelial deposits of immune complexes. Membranous glomerulopathy exhibits localization of immune complexes in the subepithelial zone (between the visceral epithelial cell and the glomerular basement membrane) as a result of immune complex formation in situ or the deposition of circulating immune complexes. Granular deposits of IgG ou tlining the glomerular capillary loops are identified by immunofluorescence microscopy. Deposition of antiglomerular basement membrane antibody (choice A) is a feature of GoodpasLure syndrome. Deposition o[ IgA in the mesangium (choice B) occurs in Berger disease. Subendothelial deposits of immune complexes (choice D) are encountered in lupus nephritis and membranoproliferative glomerulonephritis. ***Diagnosis: Membranous nephropathy
15 The glomerular changes in the patient described in Question l3 are frequemly seen in patients with which of the following systemic diseases? ***(A) Amyloid nephropathy ***(B) Goodpasture syndrome ***(C) Sclerode1ma ***(D) Systemic lupus erythemawsus ***(E) Wegener granulomaLosis
15 The answer is D: Systemic lupus erythematosus. Immune complexes formed against DNA, RNA, and autologous proteins in patients with systemk lupus erythematosus may be deposited along the basement membrane of Lhe glomeruH to form a pattern that may be indistinguishable from that of icbopathic membranous nephropathy. However, membranous nephropathy of lupus also features mesangial and subendoLhelial deposits of immunoglobulins. Immune complex deposition does not occur in the other choices. ***Diagnosis: Systemic lupus erythematosus
16 A 14-year-old girl presems with a 5-day his LOry of hypertension, oliguria, and hemaruria. She was seen 2 weeks earlier for a severe throat infection with group A <P-hemolyuc) streptococci. A kidney biopsy displays glomerulonephritis. Immunofluorescence staining for which of the fol lowing prmeins would provide rhe strongest evidence that tl115 patients glomerulonephtitis is mediated by inunune complexes? ***(A) Complement ***(B) Fibrinogen ***(C) Hageman factOr (cloumg factor Xlt) ***(D) Plasminogen ***(E) Thrombin
16 The answer is A: Complement. In acute postinJecrious glomerulonephtilis, immune complexes localize in glomeruli by deposition (rom the ci rculation or by [ormation in siru as bacterial anngens bind circulating antibodies. The renal biopsy shows complemem fixation. Complemem acti\•ation is so extensive that over 90% of patienlS with postinfecrious glomemlonephritis develop hypocomplementemia. Complemem and other inflammatory mediawrs amact and activate neutrophils and monocytes, which stimulate the proliferation of mesangial and endothelial cells, resulting in diffuse proliferative glomerulonephritis. Typically, the level of serum C3 is depressed dming the acute synd rome but retums to nom1al within 1 to 2 weeks. The other choices involve the coagulation system and are not components of immune complexes. ***Diagnosis: Postinfecdous glomerulonephritis, nephrilic syndrome
17 A 28-year-old man complains of nasal obstruction, bloody nose, cough, and bloody sputum. A chest X-ray displays cavitated lesions and muhiple nodules within both lung fie lds. Utinalysis reveals 3+ hematuria and red blood ceU casts. Labonnory studies show anemia and elevated serum levels of C-ANCA (amineutrophil cytoplasmic amibody). Peripheral eosinophlls are not increased. A renal biopsy exhibits focal glomemlar necrosis with crescems and vasculitis a[ecting arte1ioles and venules. What is the appropriate diagnosis? ***(A) Churg-Strauss syndrome ***(B) Goodpasture syndrome ***(C) Hypersensitivity vasculitis ***(D) Polyarteritis nodosa ***(E) Wegener granulomatosis
17 The answer is E: Wegener granulomatosis. Wegener granulomatosis is a systemic necrotizing vasculi tis of unknown etiology that is characterized by granulomatous lesions of the nose. sinuses, and lungs and is associated with rena] glomerular disease. Lesions associated with this condition featu re parenchymal necrosis, vasculitis, and a granuloma· wus inflammation composed of neutrophils, plasma cells, and macrophages. More Lhan 90% of patients with Wegener granulomawsis exhibit Ai"JCA, of whom 75% have C-ANCA. ANCA glomemlonephritis is an agg;ressive, ncmrophilmediated disease that is characterlzed by glomemlar necrosis and crescents. Goodpasnue syndrome (ch oice B) is characterized by both kidney and pulmonat)l involvemem but does not display ANCA. Churg-Suauss syndrome (choice A) features eosinophlUa and asthma. ***Diagnosis: Wegener granulomawsis
18 Which of the follmving best dese1ibes the renal disease of the patient described in Question I 7? ***(A) Chronic nephritic syndrome ***(B) Nephrotic syndrome ***(C) Rapidly progressive glomerulonephritis ***(D) Type I membranoproliferative glomemlonephritis ***(E) Type 11 membranoproli ferative glomerulonephritis
18 The anlwer is C: Rapidly progressive glomerulonephritis (RPGN) . Focal n ecrotizing glomemlonephritis is one of the early fearures of Wegener granulomatosis. The p athogenesis of this renal disease is not known, bu t it is th ought to be immune mediated because most patients have an tibodies to neutrophils (ANCA). These autoantibodies activate neutrophils and cause them to adhere to endothelial cells, release toxic oxygen metabolites, degranulate, and kill the endothelial cells. Exudation of inflammatory cells through the diswpted, segmentally necrotic basement membrane leads to the fonnation of crescents. Clinically, the disease p resents as RPGN, a clinical term that ls used to denote the rapid onset of renal failure caused by severe glomerular injury. Wegener granulomatosis does not cause membranoproliferative glomerulonephritis (choices D and E). ***Diagnosis: Wegener granulomatosis, crescentic glomerulonephritis
19 A .30-year-old man with a history of dntg addiction presents ·with a 6-month history of progressive swelling in his ankles and abdomen. Urinalysis shows heavy proteinuria (>4 g per H hours) but no evidence o[ inflammatory cells or RBCs. LaboratOty studies reveal hyperlipidemia and hypoalbuminemia. Semm creatinine level is nom1aL The blood test for ANCA is negative. The patient responds well to treatment \\lith corticosteroids, bm edema ru1d proteinuria recur the following year. The steroid treatment is repeated with the same resulLs. Upon the third recurrence of edema a11d prmeinmia, the patient becomes steroid resistant. A renal biopsy is shown in the image. Which of the following is the most likely diagnosis for this patients glomerulopathy? - -- ***(A) Acme glomemlonephr itis ***(B) Amyloidosis ***(C) Crescentic glomemlonephritis ***(D) Diffuse proliferative glomerutonephritis ***(E) Focal segmental glomemlosclerosis
19 The answer is E: Focal segmental glomerulosclerosis . Focal segmental glomerulosclerosis (FSGS) is characterized by glomerular scarrn1g (sclerosis) that affects some (focal), but not all, glomeruli and initially involves only pan of an affected glomerular mft (segmental). By light microscopy, varying numbers of glomeruli show segmental obliteration of capillary loops by increased collagen and the accumuJa tion of lipid or proteinaceous materiaL FSGS is the cause of the nephrotic syndrome m 30% of aduhs and 10% of children. It is also the most common renal complication o[ mtravenous drug abuse. Clinically, it presems with proteinurla, which occasional ly may be so massive as to produce nephrotic syndrome. Nephropathy associated with HIV infection is a severe and rapidly progressive collapsing form of FSGS. Patiems typically progress wend-stage renal disease in less than a year. The other choices involve glomeruli diffusely. Crescen ts (choice C) are not observed in the photomicrograph shown. ***Diagnosis: Focal segmental glomemlosclerosis
20 A 20-year-old woman ts involved in an automobile accident and loses a large amount of blood. In response to hypoxia, imerstitial peritubular cells of the kidney would be expected w release which o( the following honnones7 ***(A) Aldosterone ***(B) Angiotensin ***(C) Angiotensinogen ***(D) Erythropoietin ***(E) Renin
20 The answer is D: Erythropoietin. Erythropoietin is released by the imersddal peritubular cells of the kidney in response to hypoxia and activates specific receptors on the cell membrane of erythroid progenitor cells in the bone marrow. This effect rescues progenitor cells from programmed cell death, promotes colony growth, and restores n ormal red blood cell mass. Renjn (choice E) is released by the juxtaglomerular apparatus. ***Diagnosis: Anemia, hypoxia
21 A 32-year-old man complru ns of recun ent hematuria since his youth The hematuria typically occu rs following upper respira tory tract infections. Vital signs are normaL Urinalysis shows proteinuria, hematuria, and a few red blood cell casts. Laboratory studies disclose normal levels of BUN and creatinine. The ANA and ANCA tests are negative. vVhich of the following is the most likely diagnosis? ***(A) Amyloid nephropathy ***(B) Berger disease (lgA nephropathy) ***(C) Hereditary n ephtitis (Alpon syndrome) ***(D) Membranous glomerulopatl1y ***(E) Wegener granulomatosis
21 The answer is B: Berger disease (lgA nephropathy) . Berger disease is the most common fonn or glomeru lonephritis in adults. Deposition of lgA-dominam immune complexes is the cause of the n eph ropathy, but the constituent amigens and the mechanism of accumulation have not been determined. Exacerbations of lgA nephropathy are often initiated by infections of the respiratory or gastrointestinal tracts. The dlagnosric finding on renal b1opsy is imense mesangial s taining for lgA, which is almost always accompanied by staining for C3. lgA nepluuparhy manifests a continuum o[ glomerulopathies, ranging from no discernible light microscopic changes to chronic sclerosing glomerulonephritis. Patien ts frequently present with hematmia and proteinuria, and 20% of patients develop renal fai lure after 10 years. Neither amyloid nephropathy (choice A) nor membranous nephropathy (choice D) features RBC casts. Hereditary neph ritis (Alport syndrome; choice C) reOects abnormal type IV collagen in the glomemlar basement membrane. Hematuria is present early in bfe; proteinuria, progressive renal failure, and hypertension develop later in the course of the disease. Wegener granulomatosis (choice E) is usually positive for ANCA. ***Diagnosis: Berger disease, lgA nephropathy
22 For the patient desctibed ln Question 2l, which of the following patterns of lgA immunofluorescence would be expected in the renal biopsy? ***(A) Granular capillaty membrane deposition ***(B) Unear basement membrane staining ***(C) Mesangial deposition ***(D) Petivascular location ***(E) Subepithelial deposiL<;
22 The answer is C: Mesangial deposition. IgA nephropathy (Be rger disease) is caused by immune complexes of lgA, which are located within the mesangium, where they most bkely activate complement through the alternative pathway. The diagnostic finding is mesangial staining that is more intense for lgA than for IgG or lgM. ***Diagnosis: Berger disease, lgA nephropathy
' 23 A 25-year-old man complains of intenniltem hematuria over the past 8 years. Urinalysis shows microscopic hematuria. U1ine cultures are negative. A renal biopsy (shown in the image) displays mesangial proliferation with1n some glomeruli, whereas others appear normal. Immunofluorescence staining discloses mesanglal deposition of lgA. Which of the following is the appropriate pathologic djagnosis? ***(A) Amyloid nephropathy ***(B) Crescentic glomerulonephritis ***(C) Focal proliferative glomerulonephritis ***(D) Membranous nephropathy ***(E) Nodular diabetic glomerulosclerosis
23 The answer is C: Focal proliferative glomerulonephritis. Focal protiferatwe glomerulonephritis typically presents \vith pathologic changes in some glomeruli , whereas others remain normaL This group of diseases includes lupus nephritis, nephritis that accompanies several vasculitides, Henoch-Schon1ein purpura, and seve ral other disorders. It also includes lgA nephropathy (Berger disease) , which , as in this case, presents with mesangial deposits of lgA and mesangial ceU proliferation. The clinical presentation is variable, \Vhich reflects the varied pathologic severity of disease. ***Diagnosis: Berger disease, IgA nephropathy
24 An 8-year-old boy presents \Vith headaches, dizziness, and malaise approximately 2 weeks after a severe sore rhroaL His mother desc1ibes puffiness of his face and darkening of his urine. She also notes that her son is passing less urine and that he is becoming increasingly short of breath. On physical examinarion, there is anasarca, hypenension (190/130 mm Hg), and tachycardia. The u rine is seamy and brownish red. Urinalysis shows 3+ proteinuria. Microscopic examination of the urine discloses numerous RBCs, as well as occasional granular and red cell casts. A renal biopsy is stained by direct immunofluorescence microscopy for complemem (3, and the results are shown. Which of the following best describes the pattern of immunoflu orescence observed in this renal b iopsy? ***(A) Dense deposits in glomerular crescents between epithelial cells ***(B) Deposits limited lO the mesanglum ***(C) Granular deposits along the perimesanglal reflec tions ***(D) linear staining along the glomerular basement membranes ***(E) Subepithelial and subendothelial deposits
24 The answer is E; Subepithelial and subendothelial deposits. The most distinctive ultrastrucmral feamres of acme postinJecdous glomerulonephntis are subepithelial dense deposits that are shaped like ·'humps " These deposits are Lnvanably accompanied by mesangial and subendothelial d eposits, which may be more dirficulr to find bm are probably more important in pathogenesis because of their proximity w inOammaLO J)' mediator systems in the blood. Choices A, B, and C describe limited deposition of immune complexes, whereas choice D is a feamre of amiglomerular basemem membrane disease (Goodpasture syndrome). ***Diagnosis: Posdnfectious glomerulonephritis, nephritic syndrome
25 Which of the following is the mosL likely cause of acme postil1 [ectious glomemlonephritis in the patient described in Question 24? ***(A) Eschclicflla coli ***(B) Epstein-Barr virus ***(C) Group A (~-hemolytic) streptococci ***(D) Klebsiella sp. ***(E) Scaplty lococcu.s sp.
25 The answer is C: Group A (~·hemolytic) streptococci. Acme postinfecrious glomerulonephrltis is an immune complex disease of childhood, which occurs after an infection with group A (1}-hemolylic) streptococci and is caused by the deposition of immune complexes in glomeruli. Occasional examples are caused by staphylococcal infection (e.g., acute staphylococcal endocarditis, staphylococcal abscess), and rare cases result from viral (e.g., hepatltis B) or parasitic (e.g., malaria) infec· tions. The ptimary infection involves the pharynx or, in hot and humid environments, the skin. ln recem years, the prop onion of cases of acute pos tinfectious glomemlonephritis caused by staphylococcal infection (choice E) has been increasing ***Diagnosis: Nephritic syndrome, acute postinfectious glomemlonephruis
26 Which of the fol lowing is the most likely outcome of glomeru· lonephritis in the patient described in Question 24? ***(A) Bilateral corneal necrosis ***(B) Development of nephrotic syndrome ***(C) Membranoproliferative glomerulonephritis ***(D) Recovery withom serious consequences ***(E) Transition into crescentic glomeruloneph1i tis
26 The answer is D: Recovery without serious consequences. Overt neph1itis after postinfecrious glomerulonephr itis usually resolves after several weeks, although hematuria and especially protelnuna may persist for several months. ***Diagnosis: Nephtitic syndrome, acute postinfectious glomemlonephritis
27 A 30-year-old woman \Mith systemic lupus erythematosus presems with ohguria. LaboraLory srudies show elevated serum levels of creatinine and BUN. Ulinalysis reveals 4+ proteinuria and hematLnia. The renal biopsy (shovm in the image) exhibits segmental endocapillary hypercellularity and thickening of capilla1y walls, and 90% of the glomemli appear hypercellular. vVhich of L11e following is the approp1iate pathologic diagnosis? ***(A) Crescemic glomerulonephritis ***(B) Focal segmental glomerulosclerosis ***(C) Membranopro1ifermive glomerulonephritis, type II ***(D) Membranous nephropathy ***(E) Proliferative glomerulonephritis
27 The answer is E: Proliferative glomerulonephritis. Systemic lupus erythematosus (SLE) is an auwimmune disease characterized by a generalized dysregulation and hyperactivity of 13 cells, with production of amoantibodies w a vanety of nuclear and nonnuclear antigens. Nephritis is one of the most common complications of SLE. lnunune complexes may localize in glomeruli by deposition from the circulation, fom1atlon in situ, or both. Diffuse proliferative glomerulonephritis is a severe form of lupus nephritis, characte1ized by widespread involvemem of glomeruli and diffuse proliferation of mesangial and endothelial cells and even of epithelial cells. Deposits of immune complexes, visible by electron microscopy or immunofluorescence mkroscopy. are present on both sides of the basement membrane, in the mesangial areas, and even tnside the capillary loops. The thickened basement membranes of the glomeruli are colloquially knovm as "wire loop ~ lesions. Membranous nephropathy (choice D) may occur in SLE, but the current biopsy displays hypercellularity. SLE is not a cause of membrnnoproliferative glomerulonephritis, cype 11 (choice C). ***Diagnosis: Systemic lupus erythematosus
28 A 35-year-old man with a history of smoking presents with hematuria and bloody spmum. Over the next 2 days, he develops oliguria and renal fallure, after which he is placed on dialysis. A renal biopsy is stained with fluorescein-conjugated goat antihuman lgG, and the results are shown. Which of the following best describes Lhe pattern of direct immunofluorescence observed on this photomicrograph? ***(A) Discontinuous and peripheral ***(B) Finely granular along the petimesangial reflections ***(C) Linear along the glomerular basement membrane ***(D) Mesangial with a stalk predominance ***(E) Peripheral granular humps
28 The answer is c~ Linear along the glomerular basement mem· brane ('BM). Anti-GBM antibody glomerulonephtitis is an uncommon but aggressive form of glomerulonephritis that occurs as a renal limited disease or is combined with pulmonary hemorrhage (Goodpasture syndrome). The disease is mediated by an autoimmune response against a component of the GBM that is located within the noncollagenous domain of type lV collagen. A charaCLe ristic feature of ami-GBM glomerulonephtitis is the presence of diffuse linear staining of GBMs for IgG. which indicates auwamibodies bound to the basemem membrane. By light microscopy, over 90% of patients with anti-GBM glomerulonephritis have glomerular crescents (crescentic glomerulonephritis). Linear inununofluorescence for IgG is seen along the GBM. Anti-GBM glomerulonephritis typically presents with rapidly progressive renal failure and nephritic signs and symptoms. ***Diagnosis: Goodpasture syndrome
29 A 54~year~old woman with squamous cell carcinoma of the lung develops bilateral pining edema of the lower exuem1~ Lies. Laboratory smdies show hyperlipidemia, hypoal~ buminemia, and 4+ proteinuria. Urinal>·sis reveals no inOammawry cells or RBCs. Renal biopsy in this patient would most likely show which of the following patterns of glomerulo pathy? ***(A) Berger disease (lgA nephropathy) ***(B) Goodpasture syndrome ***(C) Membranous glomeruloparhy ***(D) Minimal change glomerulopathy ***(E) Nodular glomerulosclerosis
29 The answer is C: Membranous glomerulopathy. Many malignant neoplasms may be accompanied by a variety of paraneeplastic syndromes, among which is membranous nephropathy. Other causes of secondary membranous nephropathy include autoimmune diseases (e.g., systemic lupus erythematosus), infectious dise.ases (e.g., hepatitis B), and therapeutic agen ts (e.g., penicillamine). Immune complex deposition is found in all of these conditions. lgA nephropathy and Goodpasture syndrome (choices A and B) are not paraneoplastic disorders. Minimal change glomerulopathy (choice D) is usually found in children and is not a paraneoplastic disorder. Nodular glomemlosclerosis (choice E) reflects diabetic lesions. ***Diagnosis: Membranous nephropathy, paraneoplastic syndrome
30 A 30~year~old man with a history of smoking suddenly develops oliguria, hematuria, and hemoptysis. Serologic studies reveal anlibodies to the glomerular basement mem~ brane (GBM) A renal biopsy is shown. Which of the following pathologic changes is visible by light microscopy in this biopsy specimen? ***(A) Crescents in the urinary space ***(B) Leukocytic infi1Lrates in the glomeruli ***(C) Mesangial cetl proliferation ***(D) Thickening of the GBM ***(E) Thrombi in glomerular capillaries
30 The answer is A: Crescents in the urinary space. Crescentic glomerulonephritis is the morphologic equivalent of acute renal failure, which may develop in rapidly progressive glomerulon ephritis of Goodpasture syndrome. This disease is mediated by antibodies lO collagen type TV, whid 1 attack the GBM. The same antiborues attack the lung and cause hemoptysis. Ruprure of the GBM and extravasation of blood and infiammato1y cells into the minary space (i.e., the space between Bowman capsule and the glomerular capillary rufts) leads to the appearance of hypercellutar, crescent-like tissue. These structures are composed of proliferating par ie tal epithelial cells, as well as visceral epithelial cells and macrophages. Crescentic glomerulonephritis can be caused by other diseases, such as Wegener granulomatosis or polyaneritis nodosa, which are also diseases that damage the capillary loops of the glomeruh and allow an inflammatory exudate to accumulate in the urinary space. The other choices are not representative of epithelial crescents. ***Diagnosis: Goodpasture syndrome
31 A 52-year-old woman who suffers from diabetes mellitus and frequent urinary tract infections presents with a 3-day history of nank pain, undulating fever, and general malaise. A CBC shows neutrophilic leukocytosis (16 ,000/~L). Urine cultures reveal more tJ1an 100,000 bacte1ial colonies, composed predominantly of Gram-negative microorganisms. Blood pressure is 170/100 mm Hg, BUN is 30 mgldL, and creatinine is 2.0 mgldL Fasting serum glucose is 190 mgldL Urinalysis shows 2+ sugar and 1+ protein. Microscopic examination of the urine sedimem reveals neutrophils and occasional leukocyte casts. vVhich of the following is the most likely diagnosis7 ***(A) Acute pyelonephritis ***(B) Acme rubular necrosis ***(C) Diabetic nephropathy ***(D) Postinfectious glomerulonepiu·ilis ***(E) Nephrolithiasis
31 The answer is A: Acute pyelonephritis. Pyelonephritis refers to bacterial infection of the kidney parenchyma. Gram-negative bacteria, most commonly Escherichia coli, cause 80% of acute pyelonepluitis. The infection reaches the kidney by ascending through the urinary tract, a process that depends on the following several factors: (1) bacte1ial infection of the urine, (2) reflux of the infected urine up the ureters imo the renal pelves and calyces, and (3) emry of the bacteria through the papillae into the renal parenchyma. Bacteriuria is a typical finding in patients with acute pyelonepluitis. Diabetic patients with glucosuria are at increased risk for developing acme pyelonephritis. Acute bacterial infection is not a typical feature of the other choices. ***Diagnosis: Acute pyelonephritis
32 A 22-year-old woman in the second trimester of pregnancy presents with Oank pain, fever of 38.7°C (l03°F), and chills. Hemoglobin is 13.4 gldL, VvBCs are elevated (13,500/~LL with 78% neutrophils), and there are 265,000 platelets/j.IL Physical examination reveals costovertebral angle tenderness. The urine shows numerous WBCs and WBC casLS. Wh1ch of the following is the most likely diagnosis7 ***(A) Cystitis ***(B) Endometritis ***(C) Glomemlonephritis ***(D) Pyelonephritis ***(E) Urethritis
32 The answer is D: Pyelonephritis. Symptoms of pyelonephritis include fever, chills, malaise, and llank pain. There is an increased mcidence of pyelonephritis in pregnancy. On gross examination, the kidneys of acme pyeloneplu·itis may have small white abscesses on the subcapsular surface. The urothelium of the pelvices and calyces may be hyperemic and covered by a purulem exudate. Acute pyelonephritis is often a focal disease, and much or the kidney often appears normal. Renal biopsy shows an extensive infiltrate of neuuophils in the collecting tubules and interstitial tissue. Cystitis (choice A) and ureliu-itis (choice E) are incorrect because the finding of leukocyte casts in urine supports Lhe diagnosis of an upper urinary tract infection. ***Diagnosis: Acme pyeloneplu-itis
33 Which o [ the following is the most likely cause of the renal disease in the patient described in Question 32? ***(A) Gram-negative bacteria ***(B) Gram-positive bacteria ***(C) Human papillomavirus ***(D) Inunune complex deposil:ion ***(E) Mycobacteria
33 The answer is A: Gram-negative bacteria. Acute pyelonephritis and chronic pyeloneph1itis are bacterial diseases that usually develop from ascending infections related to the rellu. -.x of infected urine from the lower urinary tract. Gram-negative bacteria from the feces, most commonly E. coli, cause 80% of cases of acute pyelonephritis. Infection of the bladder often pre· cedes acute pyelonephritis. Bladder infection is more common in females because of a shan urethra, Jack of antibacterial prostatic secretions, and facilitation of bacterial migration by sen1al intercourse. Hemawgenous dissemination of organisms may lead to urosepsis. infection with Gram-positive bacteria (choice B) can occur but is not conu11on . Viruses (choice C) and mycobacteria {choice E) do not ordinarily cause renal disease. Choice D (immune complex deposition) is associated with glomerular disease. ***Diagnosis: Acute pyelonephritis
34 A 50-year-old woman complains of severe headaches and dizziness. The patient has a hiswry of repeated urinmy tract infections. The blood pressure is 180/llOmm Hg. Laboratory studies show elevated levels of BUN (38 mgldl) and creatinine (2.8mgldl). ACT scan of the lower abdomen reveals small, irregularly shaped kidneys \vith deep coarse scars. A percuLaneous renal biopsy is shown. Which of the foUowing is the appropriate diagnosis? ***(A) Acute pyelonephritis ***(B) Acme rubular necrosis ***(C) Chronic pyelonephritis ***(D) Nephrosclerosis ***(E) Tubulointerstitial nephritis
34 The an.swer i.s C: Chronic pyelonephriti.s. Patients with chronic pyelonephlitis suffer episodic manifestations of uriJ1ary tract infection or acute pyelonephritis, such as recurrent fever and flank pain. Urinalysis demonstrates leukocytes, and imaging studks reveal cortical scarring. The microscopic appearance of chronic pyelonephritis is nonspecific. ln this case, the biopsy shows tubular dilation and atrophy. Many tubules contain eosinophilic hyaline casts resembling the colloid of thyroid follicles (so-called thyroidization). The interstitium is scarred and contains a chronic inflammatory cell infiltrate (see photmnlcrograph). With the exception o[ acute pyelonephtitis, Lhe other choices are not related to bacterial infections. Acme pyeloneph1itis (choice A) is not characterized by scarred and shrunken kidneys. ***Diagnosis: Chronic pyelonephritis
35 The pathogenesis of the renal disease in the patient desCJibed in Question 34 is related to which of the following conditions? ***(A) Amyloidosis ***(B) Antiglomemlar basement membrane disease ***(C) Chronic hepatitis B infection ***(D) Hypenension ***(E) Repeated bouts of acute pyelonephlitis
35 The answer is E: Repeated bouts of acute pyelonephritis. Chronic pyelonephritis is caused by recurrenL and persistent bacterial infection secondary to urinary tract obstruction, ~1rine re nux, or both Choices A, B, and C cause glomerular disease, and choice D (hypertension) is a vascular disorder that ls nm associated with deep conical scarring. ***Diagnosis: Chronic pyelonephritis
36 A 52-year-old woman presenLS with swelling of her ankles of 6 weeks ln duration. Physical examination reveals an obese woman (BMI = 32 kg/m2) with pitting edema of the lower extremities and pe1iorbital edema. Laboratory studies show hyperlipidemia and hY1)oalbuminemla. Utinalysis discloses 3+ proteinmia and 3+ glucosuria but no evidence of inflammatmy cells or RBCs. A ldclney biopsy stained with PAS (shown in the image) displays a prominem increase in the mesangial marrL'<, fo rming nodular lesions, and thickening of capillary basement membranes. Whlch of the following is the most likely pathologic diagnosis? ***(A) Acme glomerulonephritis ***(B) Amyloid nephropathy ***(C) Diabetic glomerulosclerosis ***(D) MaJignam nephrosclerosis ***(E) Membranoproliferative glomerulonephritis
36 The answer is ( : Diabetic glomerulosclerosis. Diabetes mellitus, a complex metabolic disease associated with glucosuria and polyuria, is the leading cause of end-stage renal disease in the United States, accounting for a third of all patiems with chronic renal failure. Diabetic glomerulosclerosis is a component of the vascular sclerosis that involves many small vessels throughout the body. In this condition, the glomeruli show diffuse mesangial matrix expansion with focal. segmemal, nodular, and sclerotic lesions. Nodular widening of the mesangiaJ areas is associaLed with hyalinization of arterioles and focal hyaline changes of Bmvman capsule. Diabetic glomemlosclerosis eventually resul ts in progressive renal failure. The other choices are not associated with diabetes and glucosuria. ***Diagnosis: Diabetic nephropathy, diabetes mellitus
37 Which of the follov.ring serum abnonnalilies is expected in the patient described in Question 36? ***(A) Hyperbilirubinemia ***(B) Hypergammaglobulin emia ***(C) Hyperglycemia ***(D) Hyperuricel1Ua ***(E) HypobilirubinemJa
37 The answer is ( : Hyperglycemia. The cardinal sign of diabetes meiUtus is increased levels of blood glucose (hyperglycemia). Abnom1al nonenzymatic glycosylation of serum and matrix proteins, including those of the glomerular basemem membrane and mesangial manix, may induce binding of plasma proteins, such as immunoglobulins and, thereby, stimulate excessive matrix production. As a result, the GBMs are thickened and hyperpermeable w albumin, which leads to proteinuria. Oven proteinuria occurs 10 to 15 years after the onset of diabetes and often becomes severe enough to cause nephrotic syndrome. The other choices are nor chamcteristic of djabetes. ***Diagnosis: Diabetes melutus, diabetic glomerulosclerosis
38 A 70-year-old diabetic woman presenlS with sudden onset of excruciating groin and flank pain. Physical examination shows pitting edema of the lower extrenuties. Laboramry smdies reveal decreased serum albumin and increased serum lipids. Urine cultures reveal more than 100,000 bacterial colonies composed predominantly of Gram-negative microorgamsms. Which of the following is the most likely diagnosis? ***(A) Acute tubular necrosis ***(B) Crescentic glomeruloneph1itis ***(C) Diabetic glomerulosclerosis ***(D) Renal papillary necrosis ***(E) Renal vein tl1rombosis
38 The answer is D: Renal papillary necrosis. Glucosuria of djabetes predisposes to acute pyelonephritis by providing a rich meruum for bacte1ial growth Necrosis of the papiJiary tips may occur in severe cases. Symptoms indude fever, urinaty colic, and severe groin and Oank pain. The other choices are not complications of pyelonephriuts. ***Diagnosis: Papillary necrosis, diabetes mellitus
39 A 40-year-old man with Alport syndrome p resents \.vilh a 3-month history of headaches. His blood pressure is 165/100 mm Hg. A urinalysis shows 3+ proteinuria and 2+ hematuria. Laboratory studies disclose elevated levels of BUN (48mgldL) and creatinine (3.6mgldL). This patients renal disease is caused by mutacion in a gene that encodes which of the following extracellular matrt-x proteins? ***(A) Collagen ***(B) Emactin ***(C) Fibrillin ***(D) Fibronectin ***(E) Laminin
39 The answer is A: Collagen. Herem my nephritis (Alpon syndrome) reflects abnom1al type lV collagen in the glomerular basemem membrane. The syndrome is a proliferative and sclerosing glomerular disease, often accompanied by defects of the ear or the eyes. which is caused by a genetic abnonnality in type lV collagen. Hematuria is present early in life in males with X-linked disease and in both sexes with autosomal recessive disease. ProteinUJia, progressive renal failure, and hypertension develop later in the course of the disease. Virtually all men with the X-linked syndrome and both sexes with autosomal recessive di.sease develop end-stage renal disease by ages 40 to 50 years. Patients with Marfan syndrome have mutations in the fibrillin gene (choice C). ***Diagnosis: Hereditary nephrins, Alpon syndrome
40 A 35-year-old man pres ems wuh fever and rash after beginning treatment with penicillin 2 weeks earlier for a sinus infection. Urinalysis shows 3+ hematmia, as well as mononuclear cells, neun·ophils, and eosinophits. A percmaneous renal biopsy is shown. Which of Lhe fo llowing is the most likely diagnosis? ***(A) Acme rubuloimersdtial nephritis ***(B) Chronic pyelonephritis ***(C) Crescemic glomemlonephritis ***(D) Focal necrotizing glomerulonephritis ***(E) Focal segmental glomerulosclerosis
40 The answer is A: Acute tubulointerstitial nephritis. Druginduced (hypersensitivity) acute rubuloimerstitial nephritis reflects a celt-mediated immune response. It 1s characterized llliwlogically by infiltrates of activated lyn1phocytes (T lyn1- phocytes) and admL><ed eosinophils, a pattern that indicates a type lV cell-mediated immune reaction. Allhough eosinophils may be presem, they are nm essemial for !he diagnosis of druginduced nephropathy. Acute rubuloimerstitial nephritis typically presents as rapidly progressive renal failure, beginning approximately 2 weeks after drug administration is start-ed. Most patients recover fully if the drug is discontinued. The other choices are not assoclated with an eosinophilic response and are not related to drug hypersensitivity. ***Diagnosis: Acme rubutoimerstitial nephritis
41 A 58-year-old man with a history of coronary anery disease, peripheral vascular disease, and a recent hean attack suddenly develops painless hematuria. He subsequemly suffers a massive stroke and e.\.rpires. The patient's kidney at autopsy is shown. Which of Lhe following is the most likely diagnosis? ***(A) Bemgn nephroscleroSIS ***(B) Chrome pyt:lonephnus ***(C) Corocal abscess ***(D) Corucal in farcl ***(E) Mahgnam nephrosclerosis
41 The answer is D: Cortical infarct. Renal cortical infarcts are, for the most part, caused by arterial obstruction, and most represent embolization to the Lnterlobar or larger branches of the renal anery Common sources of emboli include mural thrombi, infected valves, and complicated atherosclerotic plaques. A cross section of the kidney shows a periphe ral infarct, characterized by marked palior extending to the subcapsular surface. Choices A and E {berugn and malignant nephrosclerosis) are vasntlar djsorders that are general rather than localized. Choices B and C do not cause ischemic lesions. ***Diagnosis: Renal cortical infarct
42 A 36-year-old woman in the third nimester of pregnancy (gravida II, para I) presents to the emergency room with the sudden onset of severe vaginal bleeding. Ulrrasound examination of the abdomen discloses abrupdo placentae. A healthy neonate is delivered; however, the morhers blood loss is uncontrollable. She becomes hypmensive and obmnded and subsequently dies of hypovolemic shock. The kidneys at auwpsy are shown. Which of the follovving is the most likely diagnosis? ***(A) Acute mbuloimerst.itial nephritis ***(B) Bilateral renal con ical necrosis ***(C) Crescentic glomerulonephritis ***(D) Necrotizing glomerulonephritis ***(E) Renal papillary necrosis
42 The answer is B: Bilateral renal cortical necrosis. Bilateral renal conical necrosis is a syndrome characte1ized by massive tubular necrosis involving large portions of the cortex of both kidneys. Massive bilateral renal cortical necrosis typically occurs in the setting of hypovolemia and endotoxic shock The tem1 infarct is used when there is one area (or a few areas) of necrosis caused by occlusion of aneries, whereas cortical necrosis implies more widespread ischemic necrosis. The other choices are not associated with grossly visible cortical necrosis. ***Diagnosis: Renal cortical necrosis
43 A 33-year-old woman in her thi rd uimester of pregnancy (gravida I, para 0) is rushed LO the emergency room after suffering a seizure. The patient is hypenensive and laboratmy studies show that the patient manifests nephritic syndrome. Whar is the appropriate diagnosis? ***(A) Acute mbular necrosis ***(B) Crescentic glomemlonephritis ***(C) Eclampsia ***(D) Malignant nephrosclerosis ***(E) Preeclampsia
43 The answer is C: Eclampsia. Preeclampsia, which is characterized by the triad of hypenension, proteinuria, and edema, complicates the third ttimester o [ pregnancy (choice E). When these fea tures are complicated by convulsions, the Lenn eclampsia is applied. On histologic examination, the glomemli are uniformly enlarged and the endothelial cells are swollen, an appearance that results in an apparemly bloodless glomemlar tuft. The other choices are not ordinarily seen as complications of pregnancy. ***Diagnosis: Eclampsia
44 A 60-year-old man undergoes resection of an abdominal aneurysm, which is complicated by massive hemorrhage Two days after surgery, the patiem develops acme renal insufficiency. He is placed on dialysis but suffers a massive heart anack and dies. Microscopk examination of the kidneys at amopsy reveals necrotic epithelial cells within the lumina of some tubules (shown in the image). The arrows idemify enlarged, regenerative epithelial cells. What is the. appropriate diagnosis? ***(A) Acme imerstitial nephritis ***(B) Acute tubular necrosis ***(C) Eosinophilic interstitial nephritis ***(D) Fanconi syndrome ***(E) Polyaneritis nodosa
44 The answer is B: Acute tubular necrosis (ATN) . ATN is a severe, but potemially reversible, impai.Jmem of tubular epithelial function caused by ischemia or toxic injury, which results in acute renal failu re. lschemic ATN results from reduced renal perfusion, usually associated ~lith hypotension. Tubular epithelial cells, with their high rate of energy· consuming metabolic activity and numerous organelles, are particularly sensitive to hypoxia and anoxia. lschemic ATN is characterized by swollen kidneys that have a pale conex and a congested medulla. No pathologic changes are seen i11 the glomeruli or blood vessels. Necrosis of indivjdual tubular epithelial cells is evident both from focal denudation o[ the tubular basement membrane and from the individual necrotic epithelial cells presem in some mbular lumina. Acute mterstiLial nephritis (choice A) and eosinophilic interstitial nephritis (choice C) feature intersLitial ii1Ilammation, which is not seen in this case. ***Diagnosis: Acute mbular necrosis
45 A 70-year-old obese woman (BML = 34 kglm2 ) presents with a 3-momh history of progressive renal insufficiency. She has a longstanding history of hypertension. An intravenous pyelogram shows that both kidneys are small, and the pelves and calyces appear dilated. The patient subsequently suffers a massive stroke and expires. Examination of the kidneys at auwpsy reveals symmetrically shrunken small kidneys, with a uniformly finely granular surface (shown in the image). Which of the following is the appropriate diagnosis? ***(A) Amyloidosis ***(B) Hydronephrosis ***(C) lschemic acute tubular necrosis ***(D) Nephrosclerosis ***(E) Tubulointerstitial nephritis
45 The answer is D: Nephrosclerosis. Hypertensive nephrosclerosis (so-called benign nephrosclerosis) leads to obliteration of glomeruli and may lead to end-stage kidney disease. Hypertensive nephrosclerosis 1s identified in approximately 15% o[ patients v.rith benign hypertension. Even mildto- moderate hypertension causes hypertensive nephrosclerosis. On histologic examination, most glomeruli are hyalinized, and the tubules arc either atrophlc or replaced by fibrous tissue. Arterioles exhibit concentric hyali_ne thickening o[ the ,~o,,all , often with the loss of smooth muscle cells or thei_r displacement to the periphery. This arteriolar change is tem1ed hyaline arteriolosclerosis. The other choices are not related to hypertension. ***Diagnosis: Nephrosclerosis, benign; systemic hypertension
46 A 60~year~old man presents wilh acute renal insufficiency. He treated his garden last week with a number of herbicides and insecticides, some of which may have contained heavy metals. Labora tory s tudies confirm oliguria and increased levels of BUN (54mgldl) and creatini ne (3.7mgldl). A renal biopsy is shown . What is the most likely diagnosis? ***(A) Acme tubular necrosis (ATN) ***(B) Bilateral cortical necrosis ***(C) Papillary necrosis ***(D) Rapidly progressive glomerulonephritis ***(E) Tubulointerstitial nephritis
46 The answer is A: Acute tubular necrosis (ATN) . Nephrotoxic ATNis caused by chemically induced injury to epithelial cells. Because they absorb and concentrate the chemicals, lllbular epithelial cells are preferred targets for certain toxins, including some antibiotics, radiographic contrast agents , heavy metals (e.g., mercury), and organic solvems. The photomicrograph shows widespread necrosis of proximal mbular epithelial celts \Vith sparing of distal and collecting rubules. Tubuloimerstitial nephritis (choice E) may be a response to certain d rugs but realUres interstitial in nammation. ***Diagnosis: Acute rubular necrosis
47 A 45-year-old man undergoes renal biopS}' for evaluation of chronic renal failure. The paliem is obese (BMl = 37 kglm2) and admits LO smoking two packs per day for 30 years. Physical examinaLion reveals a blood pressure of 190/110 mm Hg. An echocardiogram shows conspicuous left ventricular hypertrophy. A renal biopsy discloses pathologic changes in small renal arteries, including "onion-skinning" and fibrinoid necrosis. The Congo red stain is negative. Laborawry smdies show hemawcrir of 40%, hemoglobin of 18.7 gldl, serum cholesterol of 250 mgldl, BUN of 45 mgldL, and serum creatinine of 5.5 mgldl. Which of the following is the most likely underlying cause of chronic renal failure in this patiem? ***(A) Amyloid nephropathy ***(B) Chronic pyelonephritis ***(C) Congestive hean failure ***(D) Cushing syndrome ***(E) Malignam hypertension
47 The answer is E: Malignant hypertension. The te rm malignant hypertension refers to a severely elevated blood pressure that results in rapid ly progressive vascular disease, affecting the brain, heart, and kidney Malignant hypertension injures endothelial ceUs and causes increased vascular permeability, which leads lO the insudation of plasma proteins imo the vessel wall and morphologic evidence of fib rinoid necrosis. Acme injury is rapidly followed by smooth muscle proliferation and a concentric increase in the number of layers of smooth muscle cells, yielding the so-called "onion skin" appearance. This form of smooth muscle cell hyperplasia may be a response to the release of growth facwrs derived from platelets and other inflammatory cells at the site of vascular injury. Amyloid nephropathy (choice A) is ruled out by the absence of Congo red staining. The other choices do not cause malignant neph rosclerosis. ***Diagnosis: Malignant hypertension
48 A 58-year-old man wiLh a history of l1yperlipidemia and rugh blood pressure presenL'> w the emergency room for evaluation of headaches and blurred vision. His blood pressure is 200/115mm Hg, and pulse is 95 per minute. Funduscopic examination reveals several small retinal microaneurysms and cotton-like zones of retinal edema and necrosis. Intravenous pyelography discloses small kidneys bilaterally. Renal arteriography shows stenoses of both renal ane1ies. Hypertension in this patient is caused by the renal release of which of the fol lowing hormones? ***(A) Aldosterone ***(B) Angiotensin ***(C) Erythropoietin ***(D) Plasminogen ***(E) Renin
48 The answer is E: Renin. Renal artety stenosis causes cells of the juxtaglomerular apparatus to release renin, which induces aldosterone-mediated retention of sodium and water by the kidney (renovascular hypertension). In cases o[ unilateral renal artery stenosis, the level of renin in the renal vein of the ischemk kidney is elevated, whereas it is normal in the contralateral kidney. Renal artery stenosis is caused by atherosclerosis in adults, but in children it renects fibromuscular dysplasia of the renal artery. Aldosterone (ch oice A), angiotensin (choice B), and plasminogen (choice D) are not synthesized in the kidney Erythropoietin (choice C) influences the production of red blood cells. ***Diagnosis: Renovascular hypertension
49 A 6-year-old child develops fever, abdominal pain, and bloody diarrhea. Several other children in the neighborhood had similar symploms. The common femure was rraced to eadng hamburgers at a fast food restauram. The clinlcal course is complicated by the development of acute renal failure. Wl1ich of Lhe following is the most likely diagnosis? ***(A) Acute postinfectious glomemlonephritis ***(B) Churg-Strauss syndrome ***(C) Hemolydc uremic syndrome ***(D) Malignam hypertension ***(E) Polyaneriris nodosa
49 The answer is C: Hemolytic uremic syndrome (HUS). HUS features microangiopathic hemolytic anemia and acute renal failure, with little or no evidence for significant vascular disease outside the kidneys. It is the most conunon cause of acute renal failure in children. Major causes for HUS are Shiga ro:<in- producing strains of Eschend1ia coli, which are ingested in contaminated food. The toxin injures endothelial cells, thereby setting in motion the sequence of evems that produces thrombotic microangiopathy. Panems present with hemorrhagic diarrhea and rapidly progressive renal failure. Postinfectious glomerulonephr itis (choice A) follows strepLOcoccal infections and is not charactetized by acute renaL failure. ***Diagnosis: Hemolytic uremic syndrome
50 A 5-year-old girl presents with the sudden onset of diffuse arthralgias and skin rash. Physical examination shows a violaceous maculopapular rash on the lower torso. Urinalysis discloses oliguria and 2+ hematUlia. Urine cultures are negative. This child's clinical presentation is commonly associated with which of the following diseases? ***(A) Berger disease ***(B) Goodpasture syndrome ***(C) Hemolytic uremic syndrome ***(D) Henoch-Schonlein purpura ***(E) Polyarteritis nodosa
50 The answer is 0: Henoch-Schonlein purpura. HenochSchonlein purpura is Lhe most common type of childhood vasculitis and is caused by vascular localization of Immune complexes containing predominantly lgA. The glomerular lesion is identical with that of lgA nephropathy. Hemolytic uremic syndrome (choice C) is caused by exposure to Shiga toxin-producing strains of E.scl1enchLa coli and is not associated With angiopathy OULSide o[ the kidney. The other choices are not typically associated witb rash. ***Diagnosis: Henoch-Schonlein purpura
51 A 50-year-old man is found to have blood in his urine during a rouune checkup. He is otherwise in excellent health, except for a mild microcyuc, hypochrorruc anemia. An enlarged right kidney IS found on X-ra}' exammanon, and CT scan reveals a renal mass of mcgular shape, measuring 6cm in diameter. Which of the followmg lS the most hkel)' chagnoSlS? ***(A) Ang10myohpoma ***(B) MetastatiC carcmoma ***(C) Nephroblastoma ***(D) Renal cell carcmoma ***(E) Wilms tumor
51 The answer is 0: Renal cetl carcinoma (RCC) . RCC is the most common cancer of the kidney, accounting for 90% of kidney cancers. Most cases ofRCC are sporadk, but about 5% are inherited. The 5-year survival is 90% if the RCC has not extended beyond Lhe renal capsule; survival drops to 30% if there are distant metastases. The rumor spreads most frequently to the lung and the bones. Oncocytoma and anglomyolipoma (choice A) are often difficult to differentiate from RCC by imaging techniques. Tumors such as nephroblastoma (choice C) and Wilms rumor (choice E) occur in the pediaui.c age group. ***Diagnosis: Renal cell carcinoma
52 For the patient dcscnbed in Quesuon 51, a fine-needle aspmldon of the rena l mass shows glycogen-rich rumor cells. Molecular studies would most ltkely 1dennfy mmactons in which of the following growth regulatory genes? ***(A) ADPKD ***(B) JGF-2 ***(C) PAX6 ***(D) VHL ***(E) \VTl
52 The answer is D: VHL Clear cell carclnoma is the mosl common cype of RCC. The cytoplasm appears clear because it is 1ich in glycogen and fat, which are washed om during histologic processing of the ti.ssue. loss of one allele of the VHL gene occurs in vinually all (98%) of sporadic clear cell RCC, and mutations in the gene are found in more than half or these rumors. Thus, the evidence strongly suggests that loss of the n1mor suppressor function of the VHL gene product is an lmponam evem in the genesis of clear cell RCC. WTl is also a rumor suppressor gene, bur it is tmplicated in the development of Wilms tumor (choice E). ***Diagnosis: Renal cell carcinoma
53 The mother of a 12-month-old boy palpates a mass on the 1ighr side of the infams abdomen. The surgical specimen is shown. tvhcroscopicallr. the tumor lS composed of multiple elements, mcludmg blastema!, stromal, and epithelial tissues. Which of the followmg lS the most hkely diagnosis? ***(A) Ganglioneuroma ***(B) Neuroblastoma ***(C) Renal cell carcinoma ***(D) Terarocarcinoma ***(E) Wilms tumor
53 The answer is E: Wilms tumor. This malignant neoplasm of emb1yonal nephrogenic elements is composed of element.c; that resemble nom1al fetal tissue, including (l) metanephric blastema, (2) immatme stroma (mesenchymal tissue), and (3) immarure epithelial elements. lt is the most frequent abdominal solid tumor in children, with a prevalence of 1 in 10,000. Wilms rumor usually presems between 1 and 3 years of age, and 98% occur before 10 years of age. ln most instances of Wilms rumor, the neoplasm is sporadic and unilateral. However, in 5% of cases, i t arises in the conte.:<t of several congenital syndromes. Choices A, B, and Dare not renallumors. Renal cell carcinoma (choice C) is a tumor of adults. ***Diagnosis: Wllms tumor
54 The parems or a 6~momh-old girl palpate a mass on lhe left side of the child's abdomen. Urinalysis shows high levels of vanillyl~ mandelic acid. A cr scan reveals an abdominal tumor and bony metastases. Which of the following is the most likely diagnosis? ***(A) Dysgerminoma ***(B) Ganglioneuroma ***(C) lmmarure teratoma ***(D) Nemoblasroma ***(E) Wilms tumor
54 The answer is D: Neuroblastoma. Abdominal masses in children include \Vilms rumor (choice E), neuroblastoma, and multicystic renal dysplasia. Of these, only neuroblastoma secretes catecholamines and causes elevaLion of vanillylmandelic acid in the urine. ***Diagnosis: Neuroblastoma
55 A 50-year-o ld man is found to have blood in his urine duringaroutinecheckup.ACBCshO\vsmicrocytic,hypochromic anemia. An enlarged right kidney is found on X-ray examination. ACT scan reveals a renal mass of irregular shape measuring 5cm in diameter. The nephrecwmy specimen is shown. Th1s maUgnam neoplasm most likely originates from which of the following tissues in the kidney? ***(A) Glomeruli ***(B) Juxtaglomerular cells ***(C) Lymphatics ***(D) Renal papillae ***(E) Renal tubules
"55 The answer is E: Renal Tubules. Renal cell carcinoma origi- nates from renal tubules or ductal epithelial cells. The tumor is composed of cuboidal cells that form either tubules or solid nests. It accounts for 90% of all renal cancers and more than 11,000 cases a year in the United States. Most of these tumors are of the clear cell type and almost all show loss of one allele of the von Hipple-Lindau (VHL) gene. *** Diagnosis: Renal cell carcinoma
56 A 56-year-old woman presents with acute renal failure. A frozen section of a renal biopsy demonstrates birefringent, intratubular deposits of uric acid crystals (shown in the image). This finding suggests that the patient has been treated recently for which of the following underlying conditions? ***(A) Chronic hepatitis B ***(B) l eukemia ***(C) Porphyria ***(D) Rheumawid arthri£is ***(E) Ulcerative coliLis
56 The answer is B: Leukemia. Any condition associated with elevated levels of uric acid in the blood may cause mbular deposits of uric acid crystals (see phOLomicrograph). The classic chronic disease in this category is primary gout. Chronic urate nephropathy caused by gout is characte1ized by tubular and interstitial deposition of crystalline monosodium urate. Acute urate nephropathy can also be caused by increased cell mmover. For example, leukemic pacients who undergo chemotherapy develop hypemricemia due to the increased fom1ation of uric acid from nucleic acids released from destroyed leukemic cells. This oversupply or urares may cause renal changes similar to those of gout or orher forms of hype rmicemia. The other choices are not associated with hypemricemia. ***Diagnosis: Acute renal failure, urate nephropathy
57 A 46-year-old man wiLh no past medical h istory presents with excruciating episodic (colicky) light flank pain. A renal stone is passed. In the United St ates, this stone is most likely composed of wWch of the following? ***(A) Calcium oxalate ***(B) Calcium phospha£e ***(C) Cystine ***(D) Magnesium ammonium phosphate ***(E) Uric acid
57 The answer is A: Calcium oxalate . Nephrolithiasis and urolit11iasis are stones within the col1ecting system of the kidney (nephrolithiasis) or elsewhere in the collecting system of the urinary tract (urolithiasis) The pelves and calyces of the kidney are common sites for the forma tion and accumulation of calculi. Calcium oxalate stones are the most common (80%) form of kidney stones in the United States, whereas calcium phosphate stones (choice B) are more common in England. Both are usually related to idiopathic catciuria and increased absorption of calcium in the imestine. Magnesium ammonium phosphate stones (choice D) are typically fom1ed in urine made alkaline by urea-splitting bacteria. Uric acid stones, found in 25% of patients with gom, are smooth, yellow, hard, and radiolucent. Cystine stones (choice C) occur in children with hereditary cystinuria, an inborn error of amino acid metabolism marked by an excess of C}'Stine in the urine. ***Diagnosis: Nephrolithiasis
58 A 75-year-old homeless man is brought w the emergency room in a coma. Upon admission to rhe hospital, the BUN is 74 mgl dL, and the creatinine is 6.5 mgtdL He dies thereafter, and an amopsy reveals abnom1al kidneys (shown in the image). The pathogenesis of this disease is most likely related to which of the fo llowing? ***(A) Acute pyelonephrids ***(B) Amyloidosis ***(C) Polycyslic kidney disease ***(D) Systemic hypertension ***(E) Urinary tract obstruction
58 The answer is E: Urinary tract obstruction. Obstructive uropathy is caused by structural or functional abnom1alities in the urinary tract that impede urine 11ow, ·which may cause renal dysfunction (obstructive nephropathy) and dilation of the collecting system (hydronephrosis). In this neglected patlem, severe prostatic hyperplasia caused minary tract obsuuction. In early hydronephrosis, lhe most prominent microscopic finding is dilation of the collecting ducts, followed by dllation of the proximal and distal convoluted tubules. Grossly, progressive dilation of the renal pelves and calyces occurs, and atrophy of the renal parenchyma ensues. The other choices do not cause bilateral hydronephrosis. ***Diagnosis: Hydronephrosis
59 A 55-year-old man dies of chronic renal failure. Examination of his kidneys at autopsy reveals a "s taghom" calculus. Which of me following best describes the pathogenesis of this renal stone? ***(A) GoUl ***(B) Hereditary cystinuria ***(C) Hypercalcemja ***(D) Hyperparathyroidism ***(E) Infection
59 The answer is E: Infection. ln most cases, the presence of a urinaJy stone is associated with an increased blood level and urinaJy excretion of its p1incipal component. Most kidney stones contain calcium complexed with oxalate or phosphate, or a mixture of these anions. However, some 15% of stones result from infection. In the presence of mea-splitting bactetia, the resulting alkaline urine favors the precipitation of magnesium ammonium phosphate (struvile) and calcium phosphate (apatite). They fonn the so~called staghorn calculi that fill the entire pelvis and calices. Whereas the other choices may by associated 'vith nephrolithiasis; they do not appear as staghorn calculi. ***Diagnosis: N eph rolit hi a sis
60 A 36-year·old woman presents with advanced cervical car· cinoma, and a CT scan shows widespread pelvic spread . If this condition is not surgically corrected, the patients kidneys will most likely develop which of the following conditions? ***(A) Acme vasculitis ***(B) Hydronephrosis ***(C) Polycystic kidney disease ***(D) Staghom calculi ***(E) Tubulointerstitial nephritis
60 The answer is B: Hydronephrosis. Obstructive uropathy is caused by s tructural or functional abnormalities in the urinary tract that impede urine flow, which may cause renal dysfunction (obstructive nephropathy) and dilation of the collecting system (hydronephrosis). Metastatic cervical cancer is a frequem cause of bilateral ureteral obstruction and resulting hydronephrosis. Polycystic kidney disease (choice C) is a congenital disease. The other choices are nor related ro obstruction of the lower urinary tract. ***Diagnosis: H yd roneph rosi.s
61 A 34-year-old man undergoing c:isplatin-based chemotherapy complains of a 1-week history of increasing faLigue and headaches. He also reports seeing blood in his mine. Blood pressure is 150/lOOmmHg. Physical examination reveals diffuse purpma over his upper trunk and anns. Laboratory sn.tdies show elevated levels of BUN and creatinine, and 24-hour urinalysis reveals hematuria and oliguria. Urine cultures are negative. A CBC demonstrates severe anemia 01ematocrit 28%) and thrombocytopenia (50,000/J..!.L). The direct Coombs test is negative. A peripheral blood smear reve.als schistocytes. Which of the following is the most likely cause of acute renal failure in this patient? ***(A) ANCA glomerulonephritis ***(B) Henoch-Schonlein purpura ***(C) Nephrotoxic acme tubular necrosis ***(D) Polyarteritis nodosa ***(E) Thrombotic microangiopathy
61 The answer is E: Thrombotic microangiopathy. Thrombotic microangiopathy has a variety of causes, all of which cause endothelial damage that initiates a final common pathway of vascular changes. Injured endothelial surfaces promote thrombosis, which may cause focal ischemic necrosis. Pathologic changes in the kidney are comparable to mose seen in malignam hypenensive nephropamy. TI1ese lesions include aneriolax fibrinoid necrosis, anerial edematous intimal expansion, glomerular congestion, and vascular thrombosis. Patients typically present with thrombocytopenia, hypertension, and renal frulu re. The causes of thrombotic microangiopamy include infections, drugs (e.g., cisplatin chemotherapy), autoimmune diseases, malignant hypenension, and pregnancy. Alterations in blood Oow lead to mechanical fragmentation of erythrocytes (schistocyres). Henoch-Schonlein purpura (choice B) does not have microangiopathic features that lead to anemia or mrombocytopenia. ***Diagnosis: Thrombmic microangiopathy
62 A 16-year-old black girl \vitl1 sickle cell anemia presents to the emergency room because she is experiencing severe bone pain (avascular necrosis) An abdominal CT scan shows evidence of splenic infarcts Which of the following renal dtseases is a djrect complication of this patients vasoocclusive disease? ***(A) Acute pyelonephritis ***(B) Papillary necrosis ***(C) Polycystic kidney disease ***(D) Urate neplu·opathy ***(E) Urolithiasis
62 The answer is B: Papillary necrosis. Patients with sickle cell disease develop painful, episodic crises. The rigidity of sickled erythrocytes results in obstruction of the microcirculation, with subsequent hypoxia and ischemic injmy in many organs. Patients expetience severe pain, especially in the chest, abdomen, and bones. Sickle eel] nephropathy is the most common organ manifestation of sickle cell disease. The interstiual tissue in which the vasa recta course has a low oxygen tension. As a resuiL , in patients with sickle cell disease, erythrocytes in the vasa recta tend to sickle and occlude the lumina. lnfarcts in the medulla and papillae ensue, someLimes severe enough LO cause renal papillary necrosis. The glomemli are conspicuously congested with sickle cells. None of the other choices are direcL complications of sickle cell anemia. Choices C, D, and E do not cause papillary necrosis, and acute pydonephntis (choice A) does so only rarely. ***Diagnosis: Sickle cell disease, papillary necrosis
63 A 35-year-old woman with end-stage renal disease of unknown etiology is transplanted with a cadaver kidney. The patient develops oliguia shonly after transplantation and a renal biopsy shows immediate (hyperacute) rejection. Immunosuppression improves renal function. Which of the follo·wing represents the principle target for immune attack directed against this patients allogra ft? ***(A) ABO antigens ***(B) Bacterial amigens ***(C) Glomerular basement membrane antigens ***(D) P2-Microglobulin ***(E) Urothelium
63 The answer is A: ABO antigens. Incompatible ABO histo-blood group antigens, which are expressed on endothelial cells and erythrocytes, are absolute barriers to a successful transplant. ABO-incompatible grafts encounter prdormed circulating antibodies, which bind to endothelial cells and cause immediate (hyperacute) rejection. By conn·ast, the most common patterns of acute and chronic rejection are caused primaJily by donor-recipient differences in HLA molecules encoded by the major h.istocompatibili[y complex. These molecules are expressed on mosL cell surface membranes. Other causes of transplant rejection tend to be chronic, because they do nm involve prefonned antibodies. None of the other choices mediates hyperacute graft rejection. ***Diagnosis: Graft-versus-host disease
64 A 12-year-old girl complains of headaches and blurred vision. She has a history of high blood pressure, but is not currently taking medication. Her blood pressure is 160/95mm Hg and pulse is 95 per mJnute. Funduscopic examination reveals small retinal microaneurysms and couon-like zones of red nal edema and necrosis. She is hospitalized for funher evaluation. Renal aneriography shows segmental stenoses forming multiple ridges that project into the lumen. What is the most likely cause of secondary hypenension in Lhis young patient? ***(A) Buerger disease ***(B) Fibromuscular dysplasia ***(C) Giam cell arteritis ***(D) Kawasaki disease ***(E) Takayasu arteritis
64 The answer is B: Fibromuscular dysplasia. The most frequenr cause of renovascular hypertension in chiJdren is fibromuscular dysplasia. This disease is characterized by fibrous and muscular stenosis of the renal artery. Areas of medial thicken] ng alternate with areas of atrophy producing a "suing of beads" pattern in angiograms. Stenosis or total occlusion of a main renal artery produces hypertension that is potentially curable by reconstitution of the arterial lumen. Buerger disease (choice A) and Kawasaki disease (choice D) do noL ~:ypically affect the renal arteries. Giant ce11 arte ritis (choice C) and Takayasu arteritis (choice E) may cause secondary hypertension by producing sclerotic thickening o[ the renal arteries; however, these vascular diseases are distinctly uncommon in children. ***Diagnosis: Fibromuscular dysplasia
A baby girl has an open ddect of the lower abdominal wall (patient shown in the image). Which of the following best describes the pathogenesis of this congenjta] birth defect? ***(A) Cystic renal dysplasia ***(B) Developmem of bifid ureter ***(C) Exstrophy of urinary bladder ***(D) Failure of the urachus to involute ***(E) Formation of Meckel diveniculum
1 The answer is C: Exstrophy of urinary bladder. Exstrophy .....__ of the bladder is a serious developmental abnormality characterized by the absence of the anteJior abdominal waU. In some male infams, it is associated with epispadias (incomplete fo rmation of the penile urethra). Exstrophy of the bladder develops from incomplete resorption of the anterior cloacal membrane. ln normal embryogenesis, thjs thin membrane is Teplaced by smooth muscle. Developmenml anomalies of the renal pelvis and ure ters are found in 2% m 3% of all persons. Bifid ureters (choice B) are of no clinical significance. Failure of the urachus to involme (choice D) may resull in a vesicleumbilical fisrula or urachal diverticulum. ***Diagnosis: Exstrophy of the urinary bladder
2 The abdominal wall defect in the patiem desc1ibed in Question 1 is repaired surgically. Despite this corrective surgery, the child is at increased risk for developing which of the following neoplasms? ***(A) Bladder carcinoma ***(B) Endometrial carcinoma ***(C) Cervical carcinoma ***(D) Renal cell carcinoma ***(E) Ureteral carcinoma
2 The answer is A: Bladder carcinoma. b1 patienLS with exsn·ophy of rhe bladder, the posterior wall of the bladder is continuously C.'Cposed w mechanical injury and undergoes squamous or glandular metaplasia, thereby rendering it prone to frequent infections. Despite surgical repaiT, the metaplastic bladder mucosa remains at an increased 1isk for malignant rransfom1ation. A greater t.han expected incidence of bladder cancer occurs 50 to 60 years after su rgical repair of exstrophy. Due to squamous metaplasia of the eArposed bladder mucosa, squamous carcinoma develops rather than the usual urothelial cell cardnoma. The other choices do not involve tissue that has been exposed to the external environment and do not reOect metaplastic change. ***Diagnosis: Exstrophy of che urinary bladder
3 A 64-year-old man presents with a 4-day history of dysmia and hematuria. He has a h istory of repeated boms of acute cystiLis. Urine culmres are positive for E. coli. Ultrasound examinad. on reveats an echogenic object in a bladder diverticulum. Which of the follmving conditions most likely connibuted to the formation of a bladder diverticulum m this patient? ***(A) Diabetes mellitus ***(B) Malakoplakia ***(C) Nephrolithiasis ***(D) Nodular prostatic hyperplasia ***(E) Urothelial cell carcinoma
3 The answer is D: Nodular prostatic hyperplasia. Nodular hyperplasia of the prostate is a common disorder characterized clinically by enlargement of the gland and obsnl.tction to the Oow of urine through the bladder outlet and pathologically by the proliferation of glands and stroma. Nodular prostatic hyperplasia results in the retention of urine in the bladder and urtnary tract infections. Srructural changes develop in the bladder \\rtdl , including bladder diverticula , which are solitary or multiple saclike outpouchlngs of the bladder wall . Vrine retained inside a diverticulum is often infected, a complication mat may lead to the formation of bladder stones. The other choices do not cause structural changes in the bladder wall. ***Diagnosis: Bladder diverticulum, nodular prosmtic hy-perplasia
4 A 20-year-old pregnant woman (gravida 11 , para I) complains of lower pelvic discomfort, fever, and pain during mination for the past 2 days. She also reports seeing blood in her mine. Which o [ the following is the most Ukely cause of hematuria in this patient? ***(A) Acute cystitis ***(B) Acme pye.lonephritis ***(C) Bladder calculi ***(D) Postinfectious glomerulonephr itis ***(E) Urothelial ceU carcinoma of the bladder
4 The answer is A: Ac:ute cystitis. Acme cystitis is an inOammation of the urinary bladder that is particularly common in women and is a frequem nosocomial infection in hospitalized patients. ln most cases, cystitis reOects ascending infenion of the lower urinary traCL. Factors related to bladder infection include bladder calculi, bladder ourlet obstruction, diabetes mellitus, immunodeficiency, prior instrumentation or catheterization, radiation therapy, and chemotherapy The tisk of cystitis in females is increased because of a short urethra, especially dming pregnancy. Examination of the urine usually reveals inflammatory cells, and the causative agent can be identified by u1ine culrure. Acute pyelonephlitis (choice B) is more likely to present with flank pain, without gross hemamlia. Bladder calculi (choice C) and urothelial cell carcinoma of the bladder (choice E) would be unlikely causes of gross hematuria in this patient's age group. ***Diagnosis: Cystitis, acute
5 ln the patient described in Question 4, which of the following "vould be the most likely etiologic agem? ***(A) Enterobacter sp. ***(B) Escherichia coli ***(C) Proteus vulgaris ***(D) Pset.tdomonas aeruginosa ***(E) Streptococws pyogenes
5 The answer is B: Escherichia coli. Gram-negative bacteria from Lhe feces, most commonly E. coli, cause 80% of upper and lower urinary tract infections in females. Asymptomatic bacteriuria occurs in 10% of pregnant women, one founh of whom go on w develop acme pyelonephritis. Less common causes of acute cystitis include Proteus vulgari.s (choice C), Pswdomanas aeruginosa (choice D), and Enteroba cteJ' sp. (choice A). Most cases of cystitis respond well to treatment "'-'ith antimicrobial agents. Stromal edema, hemorrhage, and a neutrophilic in filu-ate of variable intensity are typical of acme cystitis. ***Diagnosis: Cystitis, acu te
6 A 65-year-old man presents with a recent episode of painless hematuria. Vital signs axe nonnaL All blood tests and urinalysis are normal , excepc for the presence of blood in the mine. The patient smokes cigaretles bm does not drink alcoholic beverages. Which of the following is che mosl likely cause of hematuria in this pallem7 ***(A) Acme cystitis ***(B) Acute pyelonephritis ***(C) Bladder calculi ***(D) Carcinoma of the bladder ***(E) Prostatlc carcinoma
6 The answer is 0: Carcinoma of the bladder. Bladder cancer accounts for 3% to 5% of all cancer-related deaths. Urotheli.al cell carcinoma of the bladder tYl)ically manifests as sudden hematuria and, less frequently, manifests as dysuria. Bladder cancer may he encountered at any age, but most patients (80%) are 50 to 80 years old. tvten are affected three times more often than women. Smoking is a known risk factor for bladder cancer. Choices A, B, and C are most oflen symptomatic. Prostatic carcinoma (choice E) is an u ncommon cause of hematuria. ***Diagnosis: Urothelial cell carcinoma of the bladder
7 A 67-year-old man complains of frequency of urination, pain on utination, and pelvic discomfort. The patient had a transurethral resection of the prostate 3 monLl15 ago, whicl1 required an indwelling catheter (both before and after surgery). Unne cultures are negative. Cytoscopy reveals multiple areas of hemorrhage on the bladder walL Biopsy' shows fibrosis of the lamina propria and a predominance of lymphocytes (shown in the image). Which of the following is the most likely cause of urinary sympton15 in this patient? ***(A) Acute cystitis ***(B) Bladder diverticulum ***(C) Chronic cystiLis ***(D) Malakoplakla ***(E) Malignam lymphoma
7 The answer is C: Chronic cystitis. Vlnually all patiems with acute or chronic cystitis complain of excessive frequency of mination, pain on urination (dysuria), and lower abdominal or pelvic discomfort . lmroduction of pathogens into the bladder may occur during instrumentation (e.g., cystoscopy) and is particularly common in patiems in whom mdwelllng catheters remain for prolonged peti ods. l ack of resolution of lhe inOammatory reaction is associated with the hallmarks of chron ic cystitis, including a predominance of lymphocytes and fibrosis of the lamina proptia. Acute cystitis (choice A) is characterized by acute inflammation and hemorrhage. Bladder diverticuli (choice B), malakoplakia (choice D) , and malignant lymphoma of the bladder (choice E) are uncommon. ***Diagnosis: Cystitis, chronic
8 A 72-year-old man presents with a sudden episode of painless h.eman.1ria. Cystoscopy reveals a solitaty, 2-cm papillary rumor in the posterior bladder wall. The biopsy is shown in the image. Which of the following is the most likely diagnosis? • ***(A) Adenocarcinoma ***(B) Exophytic papUioma ***(C) Papilla1y urothelial cell carcinoma, high-grade ***(D) Tubular adenoma ***(E) Villous adenoma
8 The answer i,s B: Exophytic papilloma. Epithelial tumors, most of which are urothelial cell carcinomas, constitute more rhan 98% of all prima1y rumors of the bladder. These les ions comprise a spectrum that includes, at one end, benign papillomas and low-grade exophytic papillary carcinomas and, at the other end, invasive urothelial cell carcinomas and oll1er highly malignant rumors. UrO[helial cell papilloma of the urinary bladder is often encountered incidentally or after painless hematuria. Benign exophytic papillomas make up 2% to 3% of bladder epithelial tumors and occur most frequently in men above the age of 50 years. These tumors feature papmary fronds rhat are lined by urothelial epithelium, which are virtually indistinguishable from normal urothelium. On cystoscopy, most patients show single lesions, 2 to 5 em in diameter, although multiple papillomas are not unusual. Papillary urothelial carcinoma, high-grade (choice C) is incorrect because the lesion shown has minimal cytologic atypia. ***Diagnosis: Papillary urothelial cell carcinoma of the bladder
9 A 62-year-old man presents w1lh a 1-momh history of intermittent painless hematuria_ Cystoscopy reveals multiple, red, vdvery flat patches in the bladder mucosa_ A biopsy is shown in the image. Which of the foUowing is the appropriate diagnosis? ***(A) Chronic interstitial cystids ***(B) invasive urmheHal cell carcinoma ***(C) Malakoplakia ***(D) Urothelial cell carcinoma in situ ***(E) Urothelial cell papilloma
9 The answer is D: Urothelial cell carcinoma in situ. The term .....__ carcinoma in situ is reserved for malignant changes confined w nat urothelium in nonpapillary bladder mucosa. The lesion is characterized by a urothelium that exhibits cellular atypia of th e enrire mucosa, from the basal layer to the surface. Invasive urmhelial cell carcinoma (choice B) is incorrect because there is no invasion of th e lamina propria. ln one third of cases, bladder carcinoma in situ is associated with subsequent invasive carcinoma. Most invasive urothelial cell carcinomas arise from carcinoma in sim rather than from paplllaty urothelial cell cancers. Confined to the mucosal surface , the in situ lesion is most frequently observed endoscopically as multiple, red, velvety, Oat patches, which are close to exophytic papillary urothelial cell carcinoma. Urothelial cell papilloma (choice E) does not exhibit atypia as shown in the photomicrograph. ***Diagnosis: Urmhelial cell carcin oma in situ
10 A 68-year-old man presents with a +-week history of painless hematuria. Cytoscopy reveals a large exophytic tumor near the neck of the bladder. The cystectomy specimen is shown in the image. ln addition Lo cigarette smoking, which of the fo llmving is the most signilicant risk factor for the development of this patients malignant neoplasm? -------~B O!OU0 .. 40H·~•·nOH000 .. 00T00 ~------------" ***(A) Bladder calculi ***(B) Chronic human papillomavirus infection ***(C) Diabetes mellitus ***(D) Exposure to aromatic amines and azo dyes ***(E) Previous catheterization
10 The answer is D: Exposure to aromatic amines and azo dyes. The most important risk factors for bladder cancer are cigarette smoking (fourfold increased risk) , industrial eA.-posure lo azo dyes, in fection with Schistosoma haematobium , drugs such as cyclophosphamide and analgesics, and radiation therapy (cervical, prostate, or rectal cancer). The other chokes do not increase the risk for bladder carcinoma. ***Diagnosis: Urothelial cell carcinoma of the bladder
11 A 9-month-old boy is brought to the physician by his mother, who noticed thaL her son had developed scrotal swelling. Physical examination reveals a scrotal mass. The lesion can be transilluminated and is composed of clear serous fluid. 'vVJ1at is the appropriate diagnosis? ***(A) Epididymitis ***(B) Hemacocele ***(C) Hydrocele ***(D) Spennatocelc ***(E) Varicocele
11 The answer is C: Hydrocele. The term hydrocele refers to a collection of serous Ouid in the scrotal sac between the two layers of the tunica vaginalis. The cavity is lined by mesothelium. Congenital hydrocele reOects a patent processus vaginalis tesds or its incomplete obli te ration. lt is the most common cause of scrotal swelling in infants and is often associated \vith inguinal hernia. Acquired hydrocele in adults is secondary to some mher disease affecting the scrotum, such as infection, tumor, or trauma. The diagnosis may be made by ultrasound examination or by transillu minati:ng the fluid in the cavity: Hematocele (ch oice B) is caused by an accumulation of blood between the layers of tunica vaginalis_ lt may develop after nauma or hemorrhage imo hydrocele. Testicular mmors and infections may also lead to a hemawcele. Spermatocele (choice D) contains milky fluid and does nm occur in this age group. ***Diagnosis: Congeni tal hydrocele
12 A 50-year-old man presents with painless hematuria. A CT scan of the abdomen reveals a mass in the left ureter, which almost completely obliterates the lumen and has resulted in mild hydronephrosis. The surglcal specimen is shown in the image. Whkh of the following is the mosl likely histologic diagnosis for this malignam neoplasm? ***(A) Adenocarcinoma ***(B) Neuroblastoma ***(C) Pheochromocytoma ***(D) Renal cell carcinoma ***(E) Urothelial cell carcinoma
12 The answer is E: Urothelial cell carcinoma. Tumors of the renal pelvis and ureter resemble !hose of the urinary bladder bm are much less common. The etiologic factors associated \Vith epithelial mmors of the renal pelvis and ureter are similar to those observed in bladder cancer, suggesting a field effect. Patients most frequently presem \vith hematuria and flank pain. Excision of the entire ureter is necessary because of the high frequency of concurrent and subsequent carcinomas. The other choices do not derive from urothelial cells. ***Diagnosis: Urothelial ce ll carcb1oma of the ureter
13 A 4-year-olcl girl is brought to the physician by her parenlS who rep on seeing blood in the childs urine. ACT scan reveals a neoplasm of the urinary bladder. Which of the follm,ving ls the most likely diagnosis? ***(A) Angiosarcoma ***(B) Embryonal rhabdomyosarcoma ***(C) Leiomyosarcoma ***(D) Liposarcoma ***(E) Malignant fibrous histiocytoma
13 The answer is B: Embryonal rhabdomyosarcoma. Rhabdomyosarcoma, typically of the embryonal type, manifests most commonly in children as sarcoma borryoides. These edemamus, mucosal, polypoid masses have been likened to a ucluster of grapes. Combined neanuent with radiation therapy" and chemotherapy has greatly increased survival rates for children with this neoplasm. The other choices are rarely, l f ever, encountered in children. ***Diagnosis: Rhabdomyosarcoma, sarcoma botryoides
14 A 55-year-old woman presents ·with a 2-week hiswry of pain and bleeding near the urethral meaws. Urine cultures are negative. Biopsy of the affected tissue shows chronic inflammation, granulation tissue, and epithel ial hyper plasia. Which of the Collowing is the most likely dia~osis? ***(A) Adenomatous polyp ***(B) Carcinoma of the urethra ***(C) Urothelial cell carcinoma ***(D) Urothelial cell papilloma ***(E) Urethral camncle
14 The answer is E: Urethral caruncle. Urethral caruncles are polypoid inllammatory lesions near the urethral meams that produce pain and bleed1ng. They occur exclusively in women, most frequently after menopause. Uretlu-al caruncle presents as an exophytic, often ulcerated, polypoid mass of 1 to 2 em in diameter at or near the urethral meams. Mkroscopically, the lesion exhibits acutely and chronically inflamed granulation tissue, ulceration, and hyperplasia of urothelial or squamous epithelium. Carcinoma (choices B and C) is unlikely given the lack of cellular atypia. ***Diagnosis: Urethral caruncle
15 A 40-year-old Egyptian fisherman presents wi th painless hematLUia. The panems past medical history is significant for chronic schistosomiasis, which is endemic in his country of or igin. Urinalysis shows malignant cells and cyswscopy reveals a mass in the wall of the urinary bladder. Which of the follov.ring is the most likely diagnosis? ***(A) Adenocarcinoma ***(B) Leiomyosarcoma ***(C) Papillary urothelial cell carcinoma ***(D) Squamous cell carcinoma ***(E) Urothelial cell carcinoma in situ
15 The answer is D: Squamous cell carcinoma. A high incidence of bladder cancer in Egypt, Sudan, and other African countries is attributed lO endemic schistosomiasis. Parasitic infestation of the bladder causes squamous metaplasia of the bladder epithelium. Squamous cell carcinoma of the bladder then develops in foci of squamous metaplasia. Virtually all patients with this tumor demonstrate invasion of the bladder wall at the time of initial presentation and have a poor prognosis. The other choices do not reflect squamous metaplasia. ***Diagnosis: Squamous cell carcinoma of the bladder
16 A 28-year-old man presentS with multiple, raised lesions on the shaft of his penis. Physical examination shows condyloma acuminata_ Biopsy of the lesion is shown in the image. This benign lesion is caused by infection with which of the following pathogens? ***(A) Clllm"ffydia traclwmatis ***(B) Haemophiltts ducrey i ***(C) Herpes simplex virus type 2 ***(D) Human papillomavims ***(E) 1i·eponcma palfidtmt
16 The answer is D: Human papillomavirus (HPV) . HPV is a DNA virus that infects a vmiety of skin and mucosal surfaces to produce wart-like lesions, referred to as verrucae and condylomata. Microscopically, the lesion shows epidermal hyperkeratosis, parakeratosis, acanthosis, and papillomatosis. Carcinoma of the penis is also a complication of infection with HPV Although the other choices represent sexually-transmitted diseases, they are not involved in neoplastic Lransformation. ***Diagnosis: Condyloma acuminatum of the penis
17 A 20-year-old man presents with dysuria, urgency, and urethral discharge. Physical examination shows suppuralive urethritis, with redness and swelling at the urethral meatus. Which of the following is the most likely etiology of urethritis in Lhis patient? ***(A) Borrelia recurnmtis ***(B) Chlamydia tracJwmatis ***(C) Haemophiltts ducre:yi ***(D) Neisseria gonorrhoeae ***(E) Treponema pall id11m
17 The answer is D: Neisseria gonorrhoeae. Urethritis is the most common manifestation of se,.'Ually transmitled diseases in men, Ln whom it typically presents with urethral discharge. Both gonococcal and nongonococcal urethritis have an acme onset and are related to recent sexual intercourse. The infection manifests with urethral discharge, typically pmulem and greenish yellow Symptoms include pain or tingling at the meatus of the urethra and pain on micturition (dysuria). Redness and swelling of the urethral meams are usually seen in both sexes. In gonococcal urethritis, the urethral discharge contains N. gononhoeae, which can be identified microscopically in smears of the urethral exudates. The other choices do not present with urethral suppurative discharge. ***Diagnosis: Gononhea, urethritis
18 A 27 -year-old man presents with acme and chronic inflammation o£ his glans penis. Which of the following is Lhe most likely compHcation of chronic balanins in this patient? ***(A) Carcinoma ***(B) Epididymitis ***(C) Epispadias ***(D) Hypospadias ***(E) Phimosis
18 The answer is E: Phimosis. Balanitis usually extends from the glans of the penis to the foreskin and i.s called balanoposthitis. Most often, il is caused by bacterial infection, bUL in immunosuppressed persons and in diabetics, it can also be caused by fungi. Typically, balanitis is a consequence of poor hygiene ln uncircumcised men. Significant complications of chronic balanoposthitis are stricture of the meatus, phjmosis, and paraphimosis. The orifice of the prepuce may be too narrow to allow retraction over the glans penis, in which case the condition is referred to as phimosis. lf the narrow prepuce is forcefully retracted, it may strangulate the glans and impede the outflow or venous blood, a disorder known as paraphimosis. Epispadias (choice C) and hypospadias (choke D) are congenital anomalies. Epididymitis (choice B) does not reside in the penis. Carcinoma (choice A) is possible bm not likely. ***Diagnosis: Phimosis, balan1tis
19 A 65-year-old man presents with multiple lesions on his penis that he has had for 2 months. Physical examination reveals shiny, so ft, e1yLhematous plaques on Lhe glans and foreskin. Biopsy of lesional skin shows neoplastic epithelial cells, connected by intercellular bridges, with invasion into the dermis. Which of the following is the appropriate histologic diagnosis for this patient's penile neoplasm? ***(A) Adenocarcinoma ***(B) Lichen planus ***(C) Melanoma ***(D) Squamous cell carcinoma ***(E) Urothelial cell carcinoma
19 The answer is D: Squamous cell carcinoma. Penile carcinoma occurs as a preinvasive fonn (carcinoma in situ or erythroplasia of Queyrat) or invasive squamous cell carcinoma. Erythroplasia of Queyrat manifests as solitary or multiple, shin)~ soft, erythematous plaques on the glans and fo reskin. The other choices feature neiLher intracellular bridges nor these characteJistic physical signs. ***Diagnosis: Squamm.IS cell carcinoma of the penis
20 A 67 -year-old man complains of increased urgency lo void. He could not completely empty his bladder and felt "dLc:;tended" and "irritated" all the time. Rectal digital examination reveals an enlarged nodular prosLate. A biopsy discloses hyperplastic prostatic glands (shown in the image). If this patients prostate continues to enlarge, which of the following is a possible complication' ***(A) Adenocarcinoma of prostate ***(B) Hydroureter and hydronephrosis ***(C) Imratubular germ cell neoplasia ***(D) Malakoplakia of the bladder wall ***(E) UrOlhelial cell carcinoma of bladder
20 The answer is B: Hydroureter and hydronephrosis. The biopsy shows hyperplastic prostatic glands embedded in enlarged, fibrovascular stroma. Immunoperoxidase staining of the hyperplastic epithelium is consistently positive for prostate-specifLc antigen and prostatic acid phosphatase. Approximately 75% of men 80 years of age or older have some degree of benign prostatic hyperplasia (BPH). The pathogenesis ofBPH appea rs to be related Lo age-related changes in circulating levels of testosterone and dihydrotestosterone. The clinical symptoms of this dlsorder result from compression of tlH~ prostatic urethra and consequent obstruction to the bladder omlet. A history of decreased vigor of the urinary stream and increasing urin~ uy frequency is typicaL lf severe obsn·uction is untreated, back pressure results in hydroureter, hydronephrosis, and ultimately renal failure and death. The mher choices are not complications of BPH. ***Diagnosis: Hydroureter, nodular prostatic hyperplasia
21 A 60-year-old man \mth a history of nodular prostatic hyperplasia and recurrent cystitis presents with pain in the scromm. His temperature is 38"C (lOl"F). Physical examination reveals a small, tender nodule attached to the tesLis. Which o[ the fo llowing is the most likely diagnosis? ***(A) Epididymitis ***(B) Orchitis ***(C) Spem1atoce le ***(D) Urethr itis ***(E) Varicocele
21 The answer is A: Epididymitis. Epididymitis is an innammation of the epididymis, usually caused by bacteria, which may be acute or chronic. Bacterial epididymitis in young men most often occurs in an acllle form as a complication of gonorrhea or as a sexually acquired infection with Chlan1ydia. It is characterized by suppurative inflammation. In older men, E. coli from associated urinary tract infections is the most common causaLive agem. PatienLS present with intrascrotal pain and tenderness, with or without associated fever. Varicocele (choice E) is inconect because it does not typically presem with pain and fever. Neither orchitis (choice B) nor urethritis (choice D) would presem with a nodular scrotal mass. ***Diagnosis: Epididymitis
22 A 20-year-old woman presents for questions regarding fertility. Laboratory studies previously identified a 21-hydroxylase deficiency and adrenogenital syndrome. Physical examination reveals virilization of the vulva and an enlarged clitor is. 'What is the most likely karyOlype of this patient? ***(A) 45,XO ***(B) 46,XX ***(C) 46,XY ***(D) 47,XXY ***(E) -H,XXX
22 The answer is B: 46,XX. Female pseudohermaphroditism is associated with virilization of the external genital organs and may occur in genetic females ( 46 ,XX) who have normal ovaries and internal female genital organs. Virilization of the vulva, which may show fusion into scrotal fo lds and is associated with ditoromegaly, is most often found in the adrenogenital syndrome caused by 21-hydroxylase deficiency. Lack of this enzyme leads to overproduction of androgens in the adrenal gland dming fetal Hfe. Adrenal hyperplasia and ambiguous genitalia are seen at birth. The other choices are not associated \vith virilization of the external genital organs. ***Diagnosis: Female pseudohermaphroditism, adrenogenital syndrome
23 A 20-year-old intersex woman presems with questions regarding her sexual diffe rentiation. Physical examination reveals ambiguous [emale external genital organs with signs of virilization. Cytogenetic studies show a normal 46,)CY karyotype. Which of the following is the most likely diagnosis for this patients medical condition? ***(A) Congenital adrenal hyperplasia ***(B) Crypwrchidism ***(C) Klinefelter syndrome ***(D) Testicular feminizauon syndrome ***(E) Turner syndrome
23 The answer is D: Testicular feminintion syndrome. Male pseudohem1aphrod1tism represems a spectrum of congenital disorders that aiTects generic males who have a normal 46,/CY katyotype. The gonads are crypwrchid testes, but the external genital organs appear feminine or ambiguously female, with signs of vililizarion. Male pseudohermaphroditism is most often encoumered in androgen-insensitivity syndromes due to a congenital deficiency of rbe androgen receptor, also known as testicular feminizatim1 syndrome. Patients with con genital adrenal hyperplasia (choice A) have a normal 46,XX karyotype. ***Diagnosis: Male pseudohem1aphrodirism, androgen insensitivity syndrome, testicular reminization syndrome
24 An 8-year-old boy is brought to the physician because his paren ts noticed a mass on his left testicle. Physical exami· nation reveals a solid mass that cannot be transilluminated, and biopsy shows a haphazard arrangement of benign differentiated tissues, including squamous epithelium, glandular epithelium, cartilage, and neural tissue. The left testicle was removed surgtcaJly, and the patient is symptom free 5 yea rs later. Which of the following is the mosL likely diagnosis? ***(A) Embryonal carcinoma ***(B) Mature teraLoma ***(C) Mixed ge1m cell tumor ***(D) Seminoma ***(E) Teratocarcinoma
24 The answer is B: Mature teratoma. Teratomas are the most common testicular tumor in the age group between 4 and 12 years. They are believed to be derived from p1imordial germ cells. Benign te ratomas in the p repubertal testes are composed of mature somatic tissues representing the three embtyonlc germ layers (ectodem1, mesoderm, and endodem1). The other choices represent malignant tumors rbat would be uncommon in thts patients age group. ***Diagnosis: Mature teratoma
25 A 2-year-old boy is brought w the physician because his parems noticed a mass on hls right tesdcle. Physical examination confinns the paren ts' observation. An orchiectomy is perfonned . Microscopic examination of the surgical specimen shows n eoplastic celts fom1ing glomeruloid Schmer-Duval bodies. \Vhich of the following serum markers is most useful for monitoring the recurrence of mmor in this patient? ***(A) CA-125 ***(B) Carcinoembryon.ic antigen ***(C) Estrogen ***(D) u -Fetoprotein ***(E) Human chorionlc gonadolTopin
25 The answer is D: a -Fetoprotein. Most testicular neoplasms in the first 4 years of life are classified as yolk sac mmors. Microscopic examination of a yolk sac tumor shows mterlacing strands of epithelial cells surrounded by loose cmmective stroma. The lobular arrangement of cells, surrounded by empty spaces, leads to the rorrnalion of glomeruloid srrucmres referred to as Schiller-Duval bodies. Although yolk sac tumors are malignam, rimely orchiectomy results in a 95% cure rate. These rumors produce a -fetoprotein, which can be used for monitoring the recurrence of disease following surgery Human chorionic gonadotropin (choice E) is secreted by choriocarcinoma. ***Diagnosis: Yolk sac tumor
2& A 32-year-old man prcsems With a testicular mass rhat he first nouccd 2 weeks ago. The mass cannot be transiUununated and appears solid and homogeneous on ultrasound examinauon. No tumor markers are detected on scrolog~c tesnng. An orchtccwmy is performed, and the surg~cal spec1men is shown in rhc image. Which of the followmg IS the most hkely diagnosiS? ***(A) Choriocarcinoma ***(B) Embryonal carcinoma ***(C) Lymphoma ***(D) Seminoma ***(E) Yolk sac carcinoma
26 The answer is D: Seminoma. Malignam genn cells that retain lhe phenotypic features of sperrnaLogonia give rise Lo seminomas, Lhe mosL common testicular cancer, which accounts for 40% of all gem1 cells tumors in thaL organ. The peak incidence occurs in men between 30 and 40 years of age. The only consistent cytogeneLic abnormality in testicular germ cell Lumors is an additional fragment of chromosome 12 (isochromosome pl2). On gross examination, seminomas appear as solid, rubbery-firm masses (see photograph). Neoplastic cells are arranged as nests or sheeLS lhat are separated by fibrous seplae and infiltrated with chronic inOammatory cells (lymphocytes, plasma cells, and macrophages). Seminomas are exquisitely sensilive to radiation and me cure rate is over 90%. The other choices are much less common than seminoma. Moreover, choriocardnoma (choice A) and yolk sac carcinoma (choke E) release tumor markers that can be identified in blood. ***Diagnosis: Seminoma
27 A 38-year-old man presents with a 10-month history of a painless testicular mass. Physical examination reveals a smaU nodule of the left testis. The mass cannot be rransilluminared and appears w be solid on ultrasound examinalion. A testicular biopsy is shown in the image. The multinucleated giant cells in this neoplasm are derived from which of the rollowiJlg cell types? ***(A) Chondrocytes ***(B) Leydig cells ***(C) Oligodendrocytes ***(D) Smooth muscle cells ***(E) Trophoblastic cells
27 The answer is E: Trophoblastic cells. Nonseminomawus germ cell mmors are derived fTOm embryonal cells that can give rise tO clones of malignam C)'lO tl'Ophoblastic and S)'11C)'tiotrophoblastic cells, as well as other differemiated elements. SyncytiOLrophoblastic cells arc multinucleated. Tumors composed exclusively of malignant chorionic epithelium are tcnned chmiocardnomas. TI1e phowmicrograph shows syncytiotrophoblastic giant cells and mononuclear cytotrophoblastk ce1ls. The invasive growth of trophoblastic cells in these tumors is associated with hemorrhage. The other choices do not give 1ise to multinucleated giam cells in testicular cancers. ***Diagnosis: Choriocarcinoma
28 A left orchiecwmy is performed in the patiem desctibed in Question 27. Which of the following serum markers would be most useful for moniwring tumor recurrence of Llus neoplasm follov,ring surgery? ***(A) CA-125 ***(B) CarcinoembryOJuc antigen ***(C) a -Fetoproteln ***(D) Human chorionic gonadotropin ***(E) Placental alkaline phosphatase
28 The answer is D: Human chorionic gonadotropin (hCG) . Syncyrioo ·ophoblast cells in chotiocarcinomas release hCG, a hormone of pregnancy that is not ordinarily found in males. This marker is useful in the postoperative follow-up of patients \Vho have been treated for nonseminomatous gem1 cell tumors (NSGCTs). a-Fetoprotein (choice C) is secreted by yolk sac rumors (a common componem ofNSGCTs). Placental alkaline phosphatase (choice E) is a membrane-associated histochemical marker for seminoma and testicular carcmoma in situ (i.e., intrawbular germ cell neoplasia) ***Diagnosis: Choriocarcmorna
29 A 3-month-old boy is brought to the physician because his parents cannot find one of his testicles. Physical examination confinns the parems' observation. 'vVhich of the following is the most likely diagnosis? ***(A) Anorchia ***(B) Cryptorchidism ***(C) Klinefelter syndrome ***(D) Macroorchidism ***(E) Male pseudohe rmaphroditism
29 The answer is B: Cryptorchidism. Ctyptorchidism, clinically known as undescended testis, is a congenital abnormality in which one or both testes are not found in their nom1al position in the scrotum. It is the most common urologic condition requiring surgical treatment in infants . ln 5% of male infants born at tem1 and 30% of those bam prematurely, the testes are not located in the scrotum or are easily retracted. In the large majority of these infants, the testis will descend into the s<:rotum clurlng the first year of life. The descent of the testis may he arrested at any point from Lhe abdominal cavlLy to Lhe upper scrotum. According ro their location, the cryptorchid testes can be classified as abdominal, inguinal, or upper scrOLaL Anorchia (choice A) refers ro congenital absence of testes. Macroorchidism (choice D) is a pathologic finding in adult patients with fragile X syndrome. ***Diagnosis: Cryptorchidism
30 The patient de.scribed in Question 29 develops a urogenital tumor 30 years later. An abdominal-pelvic CT scan reveals metastases to lumbar periaortic l)rrnph nodes. Which of the following is the most Hkely pathologic diagnosis? ***(A) Leydig cell rumor ***(B) Malignant lymphoma ***(C) Renal cell carcinoma ***(D) Seminoma ***(E) Urothelial cell carcinoma of the bladder
30 The answer is 0: Seminoma. The clinical significance of undescended testes is not related to the abnom1al position of the gonad (patients are asymptomatic) but wan increased incidence of infertility and gem1 cell neoplasia. All men with untreated bilateral cryptorchid testes have azoospennia and are infertile. Unilateral crypwrchidism is associated in 40% of cases wilh oligospe1mia Cryptorchidism is associated with a 20- to 40-fold greater than nmmal risk for testicular cancer. Conversely, 10% of patients with germ cell neoplasia have cryptorchid testes. Seminoma is the most common germ cell malignancy. The other choices are not complications of cryptorchidism. ***Diagnosis: Seminoma
31 A 65-year-old man presents >vith a 4-momh hiswry of a scrotal mass. Which of the follmving is the most likely diagnosis? ***(A) Choriocarcinoma ***(B) Embryonal carcinoma ***(C) Leydig cell rumor ***(D) Malignant l>'lnphoma ***(E) Seminoma
31 The answer is D: Malignant lymphoma . Malignant lymphoma is the most frequent ly encoumered neoplasm iJ1 the testes of men older than 60 years. It usually occurs in the context of systemic disease, but a few cases of primary lymphoma of the testis have been reponed. Most patients with lymphomatous involvement of the testis have a poor prognosis. TI1e other testicular tumors tend to occur in younger men. ***Diagnosis: Malignant lymphoma
32 An 8-year-old boy is brought to the physician by his parents who have begun ro notice the onset of puberty in their child. Physical examination reveals enlargemem of the external male genualia and facial hair. Which of the following neoplasms is the most likely cause of precocious puberty in this patiem? ***(A) Craniopbmyngloma ***(B) Leydig cell rumor ***(C) Pheochromocytoma ***(D) Seminoma ***(E) Yolk sac carcinoma
32 The answer is 8: Leydig cell tumor. Sex cord mmors comprise approximately 5% of testicular rumors. Leydig cell mmors are rare gonadal stromaVsex cord tumors composed of cells resembling interstitial (Leydig) cells of the testis. They can be hormonally aClive and secrete androgens, estrogens, or both. Leydig cell rumors can occur at any age , wi th two distinct peaks, one in childhood and one in adults from the third w the sixth decade. The androgenic effeclS of testicular Leydig cell tumors in prepubertal boys lead to precocious physical and se:\'Ual developmem. By contrast, feminization and gynecomastia are observed in some adults with this rumor. The mher choices do not induce precocious pubeny. ***Diagnosis: Leydig cell mmor
33 A 16-year-old boy from Africa presents v:ith a 5-day history of fever and testicular pain. Physical examination shows swolJen, tender parotid glands and testes. Which of the following is the most likely responsible pathogen? ***(A) Haemophilus ducreyi ***(B) Human immunodeficiency virus ***(C) Human papillomavirus ***(D) Mumps virus ***(E) StreptococctH r:ryogenes
33 The answer i-s D: Mumps virus. Orchitis occurs in 20% of adult males 'With mumps, but widespread immun]zation against mumps has reduced the incidence of the disorder in the United States. Viral infection is characterized by testicular pain and gonadal swelling, most commonly unilateral. The other choices do not involve either tl1e parotid gland or the testis. ***Diagnosis: Mumps orchi tis
34 A tall and slender 16-year-old boy presems with breast enlargement. Cytogenetic s tudies reveal a 4 7 ,XXY karymype. Which of the following urogenital disorders is anticipated in this patient? ***(A) Anorchidism ***(B) Cryptorchidism ***(C) Hype rand rogenism ***(D) Polyorchidism ***(E) Testicular atrophy
34 The an-swer is E: Testicular atrophy. Testicular atrophy is typically found in patients with Klinefelter syndrome ( 4 7 .XXY). The most common cause of Klinefelter syndrome is meiotic nondisjunction dming oogenesis. However, a 46,XX karyotype is found in l or 25 patients with classical signs of Klinefelter syndrome. Patients wllh Klinefelter syndrome are infertile, and the testes reveal atrophy and loss of meiotic and postmeiotlc germ cells. Cryptorch idism (choice B) occurs in nom1al males and persons with Klinefelter syndrome display hyvoandrogenism (opposite of choice C). Although the paired testicles are atrophic, they are nevertheless present (opposite of choices A and D). ***Diagnosis: Testicular atrophy, Klinefelter syndrome
35 A 78-ycar-old man was admitted to the hospital due to actue urinary tract obstruction. For the past few years, he has had recunem bouts o[ cystitis. Two days before being admitted to the hospital, he could not urinate at all. What is the probable cause of bladder omlet obsnuction in this patient? ***(A) Nephrogenic metaplasia ***(B) Nodular prostatic hyperplasia ***(C) Prostatic adenocarcinoma ***(D) Urothelial cell carcinoma of the bladder ***(E) Urethral stricture
35 The answer is B: Nodular prostatic hyperplasia. The clinical symptoms of nodular hyperplasia result from compression of the prostatic urethra and the consequent obstTuction to the bladder omlet. A history of decreased vigor of the urinary stream and increased urinary frequency is typical. Rectal examination reveals a fim1, enlarged, nodular prostate. Early nodular hyperplasia o[ the prostate begins in the region of the proximal urethra (the urothelial zone). The developlug prostatic nodules compress the centrally located urethral lumen and the more peripherally located nom1al prostate. ln well-developed cases, the nonnal prostate gland is limited to an attenuated rim of tissue beneath the capsule. The other choices are much less frequem causes of bladder obstruction. ***Diagnosis: Nodular prostatic hyperplasia
36 A 68-year-old man is found to have an elevated sennn PSA level (9.5 ng/mL, normal= 0 to 4 nglmL). Biopsy of the prostate gland reveals a poorly differentiated adenocarcinoma. Which of the following best describes the putative precursor of this malignam neoplasm? ***(A) Basal cell hyperplasia ***(B) Chronic epididymitis ***(C) Chronic prostatilis ***(D) Nodular hyperplasia of the prostate ***(E) Prostatic intraepithetial neoplasia
36 The answer is E: Prostatic intraepithelial neoplasia (PIN). In 1990, prostatic adenocarcinoma became the cancer most fTequemly diagnosed in American men, surpassing the incidence of lung cancer [or the first time. PIN refers to prostatic ducts lined by atypical (dysplastic) epithelial cells and a diminution in the number of basal cells (see choice A). It is generally accepted that PIN lesions progress to invasive prostatic adenocarcinoma. There is no evidence that prostatic adenocarcinoma originates from hyperplastic nodules seen in patients with nodular prostatic hyperplasia (choice D). The other choices are unrelated w the parnogenesis of invasive adenocarcinoma of the prostate. ***Diagnosis: Prostate i.ntraepithehal neoplasia
37 A 55-year-old man presents with urinaty symptoms of urgency and frequenc)' Rectal examination reveals an enlarged prostate. Laboratory smdies show an elevated serum PSA level of 4. 9 nglmL. The patient subsequently undergoes a prostate needle biopsy series, which demonstrates two cancer-positive needle cores: Gle.ason grades 2+2(4) and 3+'2(5). Which of the following is the appropriate cliagnosis7 ***(A) Adenocarcinoma ***(B) Nodular prostatic hyperplasia ***(C) Prostate imTaepithelial neoplasia ***(D) Squamous cell carcinoma ***(E) Vrothelial cell carcinoma
37 The an$wer is A: Adenocarcinoma. Prostatic adenocarcinomas, whlch account for 98% of all prostatic tumors, are commonly multicentJic and located in lhe peripheral zones. The cut surface of the prostate shows inegular, yellow-white, indurated subcapsular nodules. One tenth of all cases of prostate cancer are initially discovered in the fragments of Lis..c;ue obtained aL the time of transurethral resection for prostatic hyperplasia. The aggressiveness of prostatic carcinoma con-elates ·with the Gleason grade. Squamous (choice D) and urothelial cell carcinomas (choice E) involving the prostate are rare. ***Diagnosis: Prostate adenocarcinoma
38 A 70-year-old mru1 presents wilh pain in his back. Relevant clinical findings include a rock-hard, enlarged prostate palpated on rectal examination. Radiologic studies show mulncenn ·k, osteoblastic lesions of the lumbar vertebral boclies. TI1e patient is treated with leuprolide acetate (lupron), an inhibitor of gonadotropin release by the pituitaty Which of the follov.ring statements best summarizes the rationale for this m~carment? ***(A) Leydig cells release tumor chemotactic fac tors. ***(B) Prostate carcinomas frequently metastasize £O the gonads. ***(C) Senoli cells release mmor chemotactic factors. ***(D) The tumor Lc; well knovvn LO invade the testes. ***(E) Tumor cells exhibit androgen-dependem growth.
38 The answer is E: Tumor cells exhibit androgen-dependent growth. The androgenic control of nom1al prostatic growth and the responsiveness of prostate cancer to castration and exogenous estrogens suppon a role for male hormones. Chemical castration by the administration of androge11ic amagonists (e.g., leuprolide) is used in the treatment of prostate cancer. Chemotactic factors (choices A and C) are not involved in bone metastases of prostatic cancer, and invasion of the gonads (choices B and D) has no influence on the growth of these metastases. ***Diagnosis: Prostate adenocarcinoma
39 A 52-year-old woman complains of dysuria, frequency, and urgency. She bas a long history of UJinary lracl infections. Urine cultures are positive for E. coJi. Cystoscopy reveals soft yellow plaques on the mucosal surface. Histologic examina· Lion shows mucosal chronic infl ammatory cells with numerous macrophages (shown in the image). The arrow (see inset) identifies smaJl intracytoplasmic, calcium-rich spherical structu res (Michaelis-Gutmann bodies) Which of the following is the approptiate diagnosis? ***(A) Cystitis cystica ***(B) Eosinophillc cystitis ***(C) Malakoplakia ***(D) Polypoid cystitis ***(E) Urothelial cell carcinoma
39 The answer is C: Malakoplakia. Malakoplakia is an uncommon inflammatory disorder of unknown etiology characterized by the accumulation o[ macrophages. lt is often associated with an infection of the urinary tract by E. coli, although a direct causal relationship is dubious. A clinical background of immunosuppression, chronic infections, or cancer is common. Malakoplakia is characterized by soft, yellow plaques on the mucosal surface of the bladder. Histologically, the most suiking fealUre is a chronic i.nOammatory cell infiltrate composed predominantly of large macrophages with abundant eosinophilk cytoplasm containing PAS-positive granules. Some of these macrophages exhibit laminated, basophil ic calcospherites tenned Michaelis-Gutmann bodies. None of the other choices exhibit the described histopathologic feaLUres. ***Diagnosis: Malakoplakia
40 A 60-year-old woman with a history or chronic cystitis is referred to a urologist because of hematuria. Cystoscopy reveals a mass in the dome of the bladder. Biopsy shows tumor cells arranged as gland-like stmcrures. Special stains demonstrate mucin in the cytoplasm of the tumor cells. \¥hat is the appropriate diagnosis? ***(A) Adenocarcinoma ***(B) Inverted papilloma ***(C) Squamous cell carcinoma ***(D) Urothelial cell carcinoma ***(E) Urothelial cell carcinoma in silt!
40 The answer is A: Adenocarcinoma. Adenocarcinoma of the bladder accounts [or only 1% of all malignant rumors of the bladder. It originates from foci of cystitis glandulatis or imeslinal metaplasia or from remnants of urachal epithelium in Lhe bladder dome. Most bladder adenocarcinomas are deeply invasive at the time o[ initial presentation and are not curable. The other choices do not featu re mucin production . Squamous cell carcinoma of the bladder (choice C) develops in foci of squamous metaplasia, usually due to schistosomiasis. ***Diagnosis: Adenocarcinoma of bladder
41 A 65-year-old woman suffers a massive stroke and expires. At autopsy, the bladder appears edematous with a prominent cenu-al ulcer (shown in the image). Histologic examination reveals lymphocytes and mast cells, as well as extensive fibro sis within the bladder mucosa and muscularis. 'vVh.ich of the following is the most likely diagnosis? ***(A) Chronic interstitial cystitis ***(B) Invasive urotheUal cell carcinoma ***(C) Malakoplakia ***(D) Polypoid cystitis ***(E) UrOLhelial cell carcinoma in situ
41 The answer is A: Chronic interstitial cystitis. Chronic interstitial cystitis typically affects middle-aged women and featun~ ·s transmural inflammation or the bladder wall, which is occasionally associated with mucosal ulceration (Hunner ulcer). The cause is unknown. Chronic inflammation, including an increased number of mast cells and fibrosis, is commonly obse rved within the mucosa and the muscularis. A Hunner ulcer displays an imense acute inftanm1awry reaction. The most common symptoms of chronic interstitial cystitis are long-standing suprapubic pain, frequency, and urgenC)~ with or withom hematmia. The other choices do not demonstrate Lhese histologic findings. ***Diagnosis: Cystitis, chronic imerstitial; Hunner ulcer
42 Dming the physical examination of a newbom boy, the pedjatrician notices that the urethral meatus is positioned on the lower side or the penlle shaft. What is the appropti ate diagnosis for this congenital birth defect? ***(A) Epispadias ***(B) Hydroureter ***(C) Hypospadias ***(D) Peyronie disease ***(E) Phimosis
42 The answer is C: Hypospadias. Hypospadias refers to a congenital anomaly in which the urethra opens on the underside (ventral surface) of the penis so that the meatus is proximal to its nonnal glandular location. The condition results from incomplete closure of the ureltual folds of the urogenital sinus. Hypospadias has a frequency of 1 in 350 male neonates. Most cases are sporadic, but a familial occun-ence has been noted. Surgical repair is usually uncomplicated. Epispadias (choice A) refers to a congenital anomaly in which the urethra opens on the upper side (dorsal surface) of the penis. In phimosis (choice E), the orifice of rhe prepuce may be roo narrow to aUow retraction over the glans penis. ***Diagnosis: Hypospadias
43 An 18-year-old man presents with a 3-week history of scrot. al swe Uing. The a[ected area can be transUluminated and is found lO contain a clear. milky fluid. Microscopic examination of the aspirated fluid reveals degenerating spennatozoa. What is the appropriate diagnosis? ***(A) Hematocele ***(B) Hydrocele ***(C) Scrotal edema ***(D) Spennatoccle ***(E) Varicocele
43 The answer is D: Spermatocele. ${:ro tal masses and conditions that lead to swel]jng or enlargement of the scrotum often reflect abnonnalities of testicular, epididymal, or scrotal development. Clinical problems related to these pathologic conditions are most often encountered in children bm may be found in adults. A spennawcele is a cyst formed from the protrusions of widened efferem ducts of the rete testis or epididymis. It manifests as a hilar paratesticular nodule or as a fluctuating mass filled with milky fluid containing spermatozoa in various stages of degeneration. Hydrocele (choice B) is incorrect because it does not comain spermatozoa. ***Diagnosis: Spennatocele
44 A 48-year-old man presents for a routine physical examination. The patient hac; a history of hyperlipidemia and nodular prostatic hyperplasia. Physical examination reveals a large mass on Lhe lateral side of the scrotum, which can be tmnsilluminaLed. lt is found to be composed of dilated blood vessels draining the left testicle. What is the appropriate diagnosis? ***(A) Epididymitis ***(B) Hematocele ***(C) Hydrocele ***(D) Spe1matocele ***(E) Varicocele
44 The answer is E: Varicocele. Varicocele represents a local dilation of testicular veins and presents as nodularity on the lateral side of the scrotum. Most varicoceles are asymptomatic and are discovered during routine physical examination. Massive vaticoceles occur and have been likened to a "bag of worms." Hematocele, hydrocele, and spermarocele (choices B, C, and D) involve the scrotum but do noL exhibit dilaLed blood vessels. Epididymitis (choice A) is an inflammawry condition located outside the scrotum. ***Diagnosis: Varicocele
45 The patient described in Question 44 is at increased risk for whlch of the following complications? ***(A) Adenocarcinoma ***(B) Embryonal carcinoma ***(C) Infenility ***(D) Nodular prostatic hyperplasia ***(E) Seminoma
45 The answer is C: Infertility. Varicocele is considered a common cause of male infertility and oligospennia, although it is not. dear why the dilation of veins should have such consequences. Testicular atrophy is found only mrely and onl}' in long-standing disease. Surgical resection of varicocele by ligation of the mternal spemtatic vein often improves reproductive function. Varicocele is not associated with neoplasia (choices A, B, and E) or prostatic hyperplasia (choice D). ***Diagnosis: Varicocele
46 A 25-year-old man presents with a 4-week history of a painless mass in the scrotum. Physical examination reveals a testicular mass that cannot be transillumjnated. Semm levels of AFP and hCG are nom1aL A hemiorchiectomy is perfonned. On gross examination, the testicular tumor shows fod of hemorrhage and necrosis. Microscopic examination of the mmor is shown in the image. The pa[iem was cured by orchiectomy followed by chemotherapy. Which of the follO\ving is the most likely diagnosis? ***(A) Choriocarcinoma ***(B) Embryonal carcinoma ***(C) Lymphoma ***(D) Mature Leratoma ***(E) Yolk sac carcinoma
46 The uswer is B: Embryonal carcinoma. NonseminomaLOus genn cell tumors (NSGCTs) constitute 55% of all testicular germ cell t.umors. Terawcarcinoma accoums for two thirds of all NSGCTs, foUowed by mLxed germ cell mmors (which contain seminoma) and pure embryonal carcinomas. In pure embryonal carcinoma, the tumor is composed exclusively of undifferentiated embryonal carcinoma cells that. are similar w cells from early emb1yos. Embryonal carcinoma invades the testis, epididymis, and blood vessels and metastasizes w abdominal lymph nodes, lungs, and other organs. These malignant cells are highly sensitive to chemOlherapy, and the cure rates are now over 90%. The pathologic findings in this case show undifferentiated neoplastic cells, forming sheets and chords, surrounded by dilated vascular channels filled with red blood cells. Choriocarcinoma (choice A) secretes hCG. Lymphoma (choice C) is more common in older men and does not have the morpholog)' shown. Mature teratoma (choice D) features heterologous elements. Yolk sac carcinoma (choice E) secretes AFP. ***Diagnosis: Embryonal carcinoma
1 A 36-year-old woman presents with infertility. She complains of having had dull pelvic pain for 9 months, which is accentuated during menstruation. Physical examination and endocrinologic studies are normal. Lapamscopy reveals multiple, small he.monhagic lesions over the surface of both ovaries and fallopian tubes and abundant pelvic scarring. Which of the following is the most likely ruagnosis7 ***(A) Borderline serous rumor ***(B) Ectopic pregnancy ***(C) Endometriosis ***(D) Metastalic cervical carcinoma ***(E) Pelvic inflammatory disease
..__1_ The answer is C: Endometriosis. Endometriosis refers to the presence of benign endomenial glands and stroma outside the uterus. lL afflicts 5% to 10% of women of reproductive age and regresses following menopause. The sites most frequently involved are the ovaries (>60%); other uterine adnexae; and the pelvic peritoneum covering the uterus, fallopian tubes, rectosigmoid colon, and bladder. With repeated cycles, hemon hage, and the onset of fibrosis, the affected surface may take on a grossly brown discoloration ("powder bums") and ronn cysts up to 15 em in diameter, which contain chocolatecolored material ("chocolate cysts"). The other chokes do not present as small hemorrhagic lesions in these anatomic sites. ***Diagnosis: Endometriosis
2A 58-year-old woman complains of recent swelling in her vagina. There is a past medical history of prenatal exposure to diethylstllbestrol. Physical examination reveals a 3-cm fi rm mass in the anterior wall of the upper vagina. Biopsy of the vaginal mass will most likely show which of the (ollowing pathologic findings? ***(A) ctear cell adenocardnoma ***(B) Endodennal sinus tumor ***(C) Granular cell tumor ***(D) Mucinous adenocarcinoma ***(E) Squamous cell carcinoma
2 The answer is A: Clear cell adenocarcinoma. Of women exposed in mero to diethylstilbestrol, 0.1% develop clear cen adenocarcinoma. The rumor is most common between ages I 7 and 22 years and is most frequent on the anterior wan of the upper third of the vagina. Almost all clear cell adenocarcinomas are associated wilh vaginal adenosis, but very few women \\lith adenosis develop this cancer. The abundant clear cytoplasm, reflecting the presence of glycogen, accoun ts for the name "clear cell." The other cholces are not associated with prenatal exposure to diethylstilbesn·ol. ***Diagnosis: Clear cell adenocarcinoma of vagina
3 A 60-year-o\d woman presems with a 3-week histmy of a painful genital lesion and bleeding. Physical examination reveals an exophytic, ulcerated 1-cm polypoid mass near tJ1e external end of the uretlu·a. What is the most likely diagnosis? ***(A) Bartholin gland cyst ***(B) Caruncle ***(C) Condylmna acuminarum ***(D) Lichen sclerosis ***(E) Lymphogranuloma venereum
3 The an-swer is B: Caruncle. Tllis polypoid infl anunatory lesion near the female urethral meatus eUcits pain and bleeding. It occurs exclusively in women, most frequemly after menopause. Urethral canmcle presents as an exophytk, often ulcerated, polypoid mass of lw 2 em in diameter. Microscopically, the lesion exhibits acutely and chronically in flamed granulation tissue and ulceration and hyperplasia of transitional-cell or squamous epnhelium. The other choices do not typically involve the u rethral meatus. ***Diagnosis: Catunde
4 A 30-year-old woman p resents wtlh a 5-month history of increasing abdominal ginh and pelvic discomfort. Imaging studies reveal a mass replacing the lefL ovary. A multilocular rumor filled with thick, viscous fluid is removed (shown in Lhe image) Tumor spaces are lined by mucinom, columnar epithelial cells, showing no evidence of atypia. There are no papillary structures and no evidence of stromal invasion. Which of the following is lhe appropriate pathologic diagnosis! ***(A) Endometrioid adenoma of ovary ***(B) Granulosa cell mmor ***(C) Mucinous cystadenocarcinoma ***(D) Mucinous cystadenoma ***(E) Serous cystadenocarcinoma
4 The answer is D: Mucinous cystadenoma. Benign common .._____ epithelial tumors of the ovary are almost always serous or mucinous adenomas and generally arise in women benveen the ages of20 and 60 years. The neoplasms are frequemly large and often 15 w 30 em in diameter. Some of these tumors, panicularly the mucinous variety. reach Lruly massive proportion, exceeding 50 em in diameter. As opposed to their malignant counterparts, benign ovarian epithelial tumo rs tend to have thin walls and lack solid areas. lack of stromal invasion and atypia in this case exclude mucinous cystadenocarcinoma (choice C). ***Diagnosis: Mucinous cystadenoma of the ovary
5 The ovarian mmor desctibed in Question 4 most closely resembles which of the following panems of mullerlan-type di fferemiation? ***(A) Endometrial glands in pregnancy ***(B) Epithelium of the fallopian tube ***(C) Glandular epithelium of the endomeLtium ***(D) Mucosa of the bladder ***(E) Mucosa of the endocervix
5 The answer is E: Mucosa of the endocervix. During embryonic life, the celomic cavity is lined by a mesothelium. This mesotheHal lining gives rise to mullerian duCls, from which the faUopian tubes, uterus, and vagina arise. Common epi Lhelial rumors of the ovary, in order o[ decreasing frequency, include: serous rumors that resemble the epithelium of the fallopian rube (choice B): mucinous tumors that mimic the mucosa of rhe endocervi..x (choice f); endometrioid tumors that are similar to glands of the en dometrium (choice C); clear cell tumors rhat display glycogen-rich cells that resemble endometrial glands in pregnancy (choice A); and transitional cell tumo rs that resemble the mucosa of the bladder (choice D). These rumors are broadly classified as benign, borderline (atypical proliferative), and malignant. ***Diagnosis: tvlucinous cystadenoma of the ovary
6 A 19-year-old student presents to the uruversity health service with lower abdominal pain and a painful swollen right knee. She denies any trauma to the knee. Pelvic examination is exquisitely painful and reveals an ill-defined thickening in the right and left adnexae. A vaginal discharge is nored. The patient is febrile (38. 7"01 03"F). Exam:inarion of her tight knee reveals an enlarged, tender, and warm joint The WBC count is 18,500/lll (normal= 4,000 to 11 ,000/~tl). If untreated, which of the following would be the most likely complication in th:is patient? ***(A) Bronchopneumonia ***(B) Lung abscess ***(C) Meningitis ***(D) Tuba-ovarian abscess ***(E) Vaginal ulceration
6 The answer is D: Tub-o~ova.rian abscess. Gonorrhea is caused by NeiSsena gonorrhoeae, a Gram-neganve diplococcus. The infection is a frequem cause of acute salpingitis and pelvic inOammatory disease. The organisms ascend through the cervix and the endomeni al c avit)~ where they cause an acute endometritis. The bacteria then attach to mucosal cells in the fallopian tube and elicit an acute inflammatory reaction, whlch is confined to the mucosal surface {acute salpingitis). From the tubal lumen, the infection spreads lO involve the ovary, sometimes resulting in a tubo-ovarian abscess. Systemic complications of gonorrhea include septicemia and septic arthritis. The healing process distorts and destroys the plicae of the [allopian tube, often leading lO sterility Infections by N. gononhoeae at other sites (choices A, B, C, and f ) are rare. ***Diagnosis: Gonorrhea, pelvic inDammawry disease
1 A 59-year-old woman presents with a 2-year history of vulvar itching and burning. Physical examination reveals a red, moist lesion of the labium major. Biopsy reveals clusters of pale vacuolated cetls within the epidermis that stain positively for petiodic acid-Schiff (PAS) and carcinoembryonic antigen (CEA). Which of the following is the most likely diagnosis? ***(A) Extramammary Paget disease ***(B) HPV-induced papilloma ***(C) Verrucous carcinoma ***(D) Vulvar imraepithelial neoplasia ***(E) Vulvar melanoma
7 The answer is A: Extramammary Pa.get disease. Paget disease of the vulva is named after simllar-appearing tumors in rhe nipple and extramammary sites, such as the axilla and perianal region. The typkal Paget cell has a pale, vacuolated cytoplasm that contains glycosaminoglycans. lt stains with PAS and mucicarmine and expresses CEA. The disorder usually occurs on the labia majora in older women. \1\/omen with Paget disease of the vulva complain o[ pmritus or a burning sensation for many years. The other choices do not feature these specific histologic findings. ***Diagnosis: Extramammary Paget disease
8 A 52-year-old woman -..vith hypothyroidism p resents with a 2-year history of vulvar itching and painful intercourse. Physical examination reveals vulval white plaques, atrophic skin, and a parchmem-like appearance. Biopsy o[ the lesion (shown in the image)demonstrates hyperkeratosis, loss of rete tidges, and a homogeneous, acellular zone in the upper dermis. This patients vulvar dem1atitis is most commonly associated with which of Lhe [allowing underlying conditions' ***(A) Amyloidosis ***(B) Autoimmune disease ***(C) Diabetes mellirus ***(D) Hyperlipidemia ***(E) Prenatal exposure to diethylstilbestrol
8 The answer is B: Autoimmune disease. Lichen sclerosis is an inflammatOJy disease of the vulva, which is often associated with autoimmune disorders such as vi tiligo, pernicious anemia, and thyroiditis (e.g., Hashimoto thyroiditis). The condition is represen ted by white plaques, atrophic skin, a parchment-like or crinkled appearance, and, occasionally, marked contracture of the vulvar tissues. Histologically, there is hyperkeratosis, loss of rete ridges, and a homogeneous, acellular zone in the upper dermis. A band of chronic inflammatory cells typically lies beneath this layer. ltching is the most common symptom, and dyspareunia is frequent. Women with symptomatic lichen sclerosis have a 15% chance o[ developing squamous cell carcinoma. The other choices are not associated with lichen sclerosis. ***Diagnosis: Lichen sderosus
9 A 29-year-old woman is evaluated for an abnormal cervi.cal Pap smear. Colposcopy reveals condyloma acuminatum of the exocervix. A biopsy of the cervix i.s shown in the image. PCR ampllfication of this biopsy specimen will mosL li kely demonstrate evidence of which of the following infectious agenrs? ***(A) Cytomegalovirus ***(B) Herpes simple.'< virus ***(C) Human papillomavirus ***(D) Molluscum contagio.sum ***(E) Treponenw pallidum
9 The answer is C: Human papillomavirus (HPY) . Condyloma acuminatum is a benign, exophytic, papillomatous lesion on the skin or mucous membranes oft he lower female genital tract. HPV is a DNA virus that infects a vatiety of skin and mucosal surfaces w produce condylomaLa, which are also referred w as venucae. The median time from infection LO first detection of HPV is 3 months. HPV types 6 and 11 are detected in over 80% of macroscopicaUy visible condylomata. Several suains of HPV are now considered the major etiologic facwr in U1e develop· mem of squamous cell cancer in the female lower genital tract. Types 16, 18, 31, and 45 are the most representative high-risk types linked to innaepithellal neoplasia and invasive cancer. The vacuolated cells in the cervical biopsy (see photomicro· graph) are typical of HPV infection and are tenned koilocytes. The other pathogens do not infect the cervix and do not produce this histopathologic appearance ***Diagnosis: Condyloma acuminatum
10 A 31-year-old Haitian woman is evaluated for infertility. Pelvic examination shows a markedly enlarged vulva, inguinal lymph node enlargement, and rectal stricture. Biopsy of an inguinal lymph node reveals necrotizing gTanulomas, neutrophilic infiltrates, and inclusion bodies \5/ithin macrophages. Which of the following is the mosLlikely etiology of infertility in !his patient? ***(A) Chlmnydia traclwmatis ***(B) Gardnaella vaginalis ***(C) Molluscum con tagiosum ***(D) Mycobacterium tuberculosis ***(E) Ji·e1Jonema pallidwn
10 The answer is A: Chlamydia trachomatis. Lymphogranuloma venereum is a sexually transmiued infection that is endemic in tropical countries but rare in developed ones. The disease is caused by C. traclwmatis. which is a Gram-negati\'e obligate, intracellular tickeusia. This organism has been found in the genital tract of abom 8% of asympwmatic women and in 20% of women presenting \vith symptoms of a lower genital tract infection. After a few days to a month, a small painless vesicle forms at the site of inoculation. lt heals rapidly, and in many instances, the vesicle is not even noticed. The second smge presems with bilaterally enlarged mgulnal lymph nodes that may rupture and form suppurative fistulas. ln some untreated patients, a third stage appears, which causes lymphatic obstruction and resulting gen ital elephantiasis and rectal Slrictures. Mycobacterium tLrbe~u!osis (choke D) induces granulomatous inflanunation but does not feature inclusion bodies. Gardnerella vaginahs (choice B) causes nonspecific vaginitis. Moi!Lrscw11 conLagiosum (choice C) does not involve the lymph nodes. Treponema pa1lidum (choice E) does not cause granulomas. ***Diagnosis: lymphogranuloma venaereum
11 A .35-year-old woman in Africa presents with fever, chills, and malaise. She further complains of a painful genital sore. She had sexual intercourse 5 days previously. Physical examination reveals vesiculopustular lesions on the labium major and cervix. There is bilateral inguinal lymphadenopathy. A lymph node biopsy reveals granulomatous inOammation. Which of !he foUo\ving is the most likely etiology of this constellation of signs and symptoms? ***(A) Cytomegalovirus ***(B) Gardnerella vagiMlis ***(C) Haemophilus ducrcyi ***(D) Mycobacterium tuberculos is ***(E) Neisseric1 gonon1weae
11 The answer is C: Haemophilus ducreyi. Chancroid, also called soft chancre, is caused by H. dLtcreyi, a Gram-negative bacillus. This disease is rare in the United States but i.s common in underdeveloped counnies. Usually 3 to 5 days after sexual congress with an infected panner, single or sometimes multiple small, vesiculopusmlar lesions appear on the cervL"{, vagina, vulva, or pe1ianal region. Hiswlogic examination reveals a granulomalOus inflammatory reaction. The lesion often ruptures to form a purulent ulcer that is painful and bleeds easily. There may be associated inguinal lymphadenopathy. fever, chills, and malaise. A major complication is scar formation during the healing phase, which is an outcome that sometimes causes urethral stenosis. Mycobactc1ium tuberculosl~ (choice D) causes granulomatous salpingitis but is not transmitted acutely, as in this case. The other choices do not elicit granulomatous inflammation. ***Diagnosis: Chancroid
12 A roULine cervical Pap smear taken during a gynecologlc examination of a 31-year-old woman shows numerous, loosely arranged cells with high nuclear-to-cytoplasmic ratio. Colposcopy shows white epithelium, punctation, and a mosaic pattern in !he transfonnation zone (shovm in !he image). vVhich of the following is the most likely diagnosis? ***(A) Adenocarcinoma o[ endoceTvix ***(B) Chronic cervicitis ***(C) Clear cell adenocarclnoma ***(D) Dysplasia o[ the cervix ***(E) Herpes simplex virus infection
12 The answer is D: Dysplasia of the cervix. Cervical imraepithelial neoplasia is defined as a spectrum of inrraepithelial changes that begins with minimal atypia and progresses through stages of more marked intraepithelial abnormalities to invasive squamous cell carcinoma. Dysplasia and carcinoma in sim can often be detected on colposcopic examination by signs associated with their altered epithelial and vascular changes: epithelial mosaicism (irregular surface resembling inlaid woodwork) and vascular dms differentiated from the sun·otmding £issue surface by color and texture. The OLher choices do not demonstrate these gross morphologic features, although they may share dysplastic morphology: ***Diagnosis: Cervical inrraepirhelial neoplasia
13 A 36-year-old woman is evaluated for an abnormal Pap smear. A cervical biopsy shows atypical squamous cells tluoughout the entire thickness of the epithelium, ~ith no evidence of epithelial maturation (shown in the image). The basal membrane appears intact What is the appropriate diagnosis? ***(A) Clear cell adenocarcinoma ***(B) Invasive squamous cell carcinoma ***(C) Mild dysplasia (cervical intraepithelial neoplasia IClN]-1) ***(D) Severe dysplasia (ClN-3) ***(E) Squamous metaplasia of the transfom1ation zone
13 The answer is D: Severe dysplasia (CIN-3) . The nonual process by which the cervi.cal squamous epithelium matures is disturbed in CIN, as evidenced morphologically by changes in cellularity, differentiation, polarity, nuclear features, and rnimtic activity. ln CJN-1 (mild dysplasia), the most pronounced changes are seen in the basal third of the epithelium. However, in this case, abnormal cells are present throughout the emire thickness of Lhe epithelium. ln ClN-2 (moderate dysplasia, choice C), most of the cellular abnormallties are ln the lower and middle thi rds of the epithelium. ClN-3 is synonymous with severe dysplasia and carcinoma in silu and shows abnormal cells occupying the full thickness of the epithelium, with no evidence of epithelial maturation. Invasive carcinoma (choice B) features extension of neoplastic cells through the basal membrane. Dysplasia is not synonymous with squamous metaplasia (choice E). ***Diagnosis: Cervical inrraepithelial neoplasia
14 A 35-year-old woman presems with a 6-week history of vaginal discharge, which is occasionally blood tinged. Pelvic examination reveals a 2-cm pedunculated, lobulated, and smooth cervical grmvth; it is excised. Htstologic examination of the specimen would most likely reveal which of the fol lo-wing? ***(A) Condyloma acuminamm ***(B) Embryonal rhabdomyosarcoma ***(C) Endocervical polyp ***(D) Leiomyosarcoma ***(E) Microglandular hyperplasia
14 The answer is C: Endocervical polyp. Endocervical polyp, the most common cenrical growth, appears as a single smooth or lobulated mass, l)1)ically smaller than 3 em in greatesL dimension. It LypicalJy manifests as vaginal bleeding or discharge. The lining epithelium is mucinous, with \'arying degrees of squamous metaplasia, buL may [ealure erosions and granulation tissue in women wi th symptoms. Simple excision or curettage is curative. Cancer rarely arises in an endocervical polyp (0.2% of cases). The other chotces are rare causes of an endocenrical polyp. ***Diagnosis: Endocervical polyp
15 A 28-year-old woman, who is 28 weeks pregnant, presems with vaginal bleeding She does not have a history of uterine contractions. Pelvic examination reveals b1ighr red blood in the endocervical canal. An ulcerated exophytic mass is identified on the left side of Lhe cervix .. There is no evidence of direct rumor extension into the parametrium. The pelvic lymph nodes are slightly enlarged, raising the possibility of nodal involvemem by the rumor. A Caesarian section is performed, followed by a radical hysterecwmy. The cervix is shown in the image. Which of the following is the besL prognostic indicawr of survival in this patient? ***(A) BRCA gene mmation ***(B) Degree of keratinization ***(C) Nodal involvement ***(D) Presence of carcinoembryonic antigen (CEA) in serum ***(E) Small cell rather than large cell carcinoma
15 The answer is C: Nodal involvement. Squamous cell carcinoma is by far the most common type of cervical cancer. In the earliest stages of ce rvical cancer, patientS complain most freq uently of vaginal bleeding after intercourse or douching. With more .advanced tumors, the S)IJ11ptoms are referable to the route and degree of spread. The clinical stage of cervical cancer is the best prognostic index of survival. Radical hystereClomy is favored for localized tumor, especially in younger women; radiation therapy or combinations of lhe two are used for more advanced tumors. Histologic or cytologic findings (choices Band E) are of seconda1y imponance. CEA (choice D) is nor typically e:-..'Pressed by squamous carcinoma cells. ***Diagnosis: Cervical cancer
16 Imaging studies establish a diagnosis of stage IV cervical cancer_ If untreated, which of the following ·will be Lhe most Ukely cause of death in the patiem desctibed in Question 15' ***(A) Adrenal conical failure ***(B) Brain metastases ***(C) Lung metastases ***(D) Renal failu re ***(E) Yenebral fractures
16 The answer is D: Renal failure. Cervical cancer spreads by direct extension and through lymphatic vessels and only rarely by the hematogenous route, which would result in distant metastases (choices A, B, C, and E). Local extension into surrounding tissues (parametrium) results in ureteral compression. The corresponding clinical complications of local extension are hydroureter, hydronephrosis, and renal failure, the last being the most common cause of death (50% of padems). Bladder and rectal involvement may lead to fistula formation. Metastases to regional lymph nodes iJwolve the paracervical, h)1Jogaslric, and extemal iliac nodes. ***Diagnosis: Cervical cancer
17 A 50-year-old nulliparous woman with a history of diabetes complains that her menstrual blood flow is more abundant than usual_ During the last two menstrual cycles, she noticed spotting Lhroughout the entire cycle. The patient is obese (BMI = 32 kglm2 ) , and her blood p ressure is 160/100 mm Hg. An utn·asound examination reveals a thickened endometrial snipe wilh a polypoid mass in the uterine fundus. The patient undergoes a hysterectomy. The uterus is opened to reveal a partially necrotic mass (shown in the image). A biopsy of the mass shows moderately differentiated adenocarcinoma. Which of the following represents the most likely precursor of this patient~ malignant disease? ***(A) Adenomyosis ***(B) Atyptcal hyperplasta ***(C) Chronic endometnns ***(D) Complex hyperplasia ***(E) Glandular metaplasta
17 The answer is B: Atypical hyperplasia. Endometrial hyperplasia refers to a spectrum that ranges from simple glandular crowding to conspicuous proliferation of atypical glands, which are difficult Lo distinguish from early carcinoma. The risk of developing endometrial cancer increases with progressively higher degrees of endometrial hyperplasia. The progression f-rom hyperplasia free of atypia (complex type, choice D) to invasive cancer requires some 10 years, bm the corresponding time for hyperplasia with acypw is only 4 years. Atypical hyperplasia is charactetized by cywlogic atypia and marked glandular crowdi ng, frequemly as back-to-hack glands. The epithelial cells are enlarged and hyperchromatic and have p rominent nucleoli and an increased nuclear-to-cytoplasmic ratio. One fourth of these cases progress to adenocarcinoma. Adenomyosis (choice A) and chronic endometritis (choice C) are not premalignant conditions . ***Diagnosis: Endometrial adenocarcinoma
18 Neoplastic cells obtained from the patient described in Question 17 would most llkely show loss of function or which or the following cell cycle cont rol proteins? ***(A) p'53 ***(B) PTEN ***(C) Rb ***(D) RET ***(E) WT-1
18 The answer is 8: PTEN. The PTEN mmor suppressor gene, \vhich is hormonally regulated in normal endometrium, is an in formative biomarker for endometrial carcinogenesis. Loss of this gene function occurs in two thirds of endometrial carcinomas. PTEN knockout mice uniformly develop "endometrial hypeq)lasia" that evolves lO carcb1.oma in one fifth of the animals. Loss of Rb function (choke C) has been implicated in HPV-induced cervical carcinoma. Mutations in p53 (choice A) are found in many tumors, bm loss of p53 function is not associaled with endometrial carcinoma. Loss of WT-1 tumor suppressor protein (choice E) is related to Wilms rumor. ***Diagnosis: Endometrial adenocarcinoma
19 A -+5-ycar-old obese woman (BMl = 32 kglm2) with a htsLory of diabetes and poorly comrolled hypertension complains of increased mensuual blood flow of 3 months in duration. An endometrial biopsy ts shown in the 1mage. Which of the following most likely accoums for the pathogenesis of endomemal hyperplasia in this paucnt? -----·--- -- --- ***(A) Excess estrogen stimulation ***(B) Exposure lO exogenous progestational agenlS ***(C) History of chronic endometritis ***(D) HIStory of oral comracepuvc use ***(E) Prenatal exposure to dJeth)•lsulbcstrol
19 The answer is A: Excess estrogen stimulation. Endometria] hyperplasia and adenocarcinoma are frequently associated with exogenous or endogenous estrogen excess. For example, endometr ial hyperplasia may result from anovulawry cycles, polycystic ova1y syndrome, an estrogen-producing tumor, or obesity. ln such cases, therapy aimed at the primary disease may alleviate the estrogenic stimulation. Estrogenic stimulati. on o[ the endometrium beyond the 2-week interval of a normal proliferative menstrual cycle causes progressive changes that have been associated with a 2- to lO-fold increased 1isk of endomerrial cancer. In contrast to benign hyperplasia, endomerrial i.ntraepithelial neoplasia (EIN) is recognized as monodonal neoplastic growth of genetically altered cells. The other choices do not predispose to endometrial hyperplasia, EIN, or carcinoma. ***Diagnosis: Endomeuial hyperplasia
20 A 33-year-old woman with a history of menorrhagia presents with a 6-momh history of increasing fatigue. A CBC reveals a hypochromic, microcytic anemia (hemoglobin = 8 gldl ). Bimanual pelvic examination reveals an enJarged uterus with multiple, irregular masses. A hysterectomy is performed, and a sharply circumscribed fleshy tumor is found wilhin the utetine wall (shown in the image). Which of Lhe following is Lhe most likely cause of vaginal bleeding and anemia in this patiem7 ***(A) Adenomyosis ***(B) Cervical cancer ***(C) Endometrial carcinoma ***(D) Endometriosis ***(E) Uterine leiomyoma
20 The answer is E: Uterine leiomyoma. Leiomyoma is a benign nnuor of smooth muscle origin that is colloquially known as a fibroid. These tumors are rare before age 20 years, and most regress after the menopause. Estrogen promotes the growth of leiomyomas, although it does nm initiate them. Grossly, leiomyomas are fim1, pale gray, whorled, and without encapsulation. Most leiomyomas are intramural, but some are submucosal, subserosal, or pedunculated. Submucosal !ciamyomas may cause bleeding, which is an effect due to ulceration of the thinned, overlying endometrium. Adenomyosis (choice A)cloes not present as a discrete mass. Endomeuial carcinoma (choice C) is much less common than leiomyoma. ***Diagnosis: Leiomyoma of the u tems
21 A 52-year-old woman presents with chronic pelVIc discomfon. A CT scan of the pelyjs shows a 10-cm, well-circumscribed uterine mass. A hys terec tomy is performed. On gross examination. the mass is soft with areas of necrosis and irregular borders extending into the myometrium. Histologic examination demonstrates large zones of necrosis surrounded by a rim of disorganized spindle cells that display numerous rniwses. lmmunohiswchemkal s taining for smooth muscle actin is positive. Which of the following is the most likely diagnosis? ***(A) Adenomyosis ***(B) Carcinosarcoma ***(C) Endometrial stromal sarcoma ***(D) Leiomyoma ***(E) Leiomyosarcoma
21 The answer is E: Leiomyosarcoma. Leiomyosarcoma is a malignant tumor of smooLh muscle cell otigin. 1t should be sus· pected i.[ an apparem leiomyoma is soft, shows areas of necrosis on gross examination, has irregular borders, or does not bulge above the surface when cut. The following features are considered evidence for the diagnosis of leiomyosarcoma: (1) ten or more mitoses per high-powered field (HPF); (2) five or more mitoses per 10 HPFs, ·with nuclear atypia and necrosis; and (3) myxoid and epithelioid smooth muscle mmors with five or more mitoses per 10 HPFs. Adenomyosis (choice A) refers w the presence of benign endomenial glands and stroma in the myometrium. Carcinosarcoma (choice B) is a mixed rumor vvith malignant epithelial and srromal components. Endometrial stromal sarcomas (choice C) show a vascular supporting framework with neoplastic cells concentrically arranged around blood vessel; they are much rarer than leiomyosarcoma. ***Diagnosis: Leiomyosarcoma of the uterus
22 A 50-year-old woman complains of having intennenstmal bleeding for 4 months. A Pap smear is normal. An ultrasound examination shows a mass in the endometrial cavity. The patient elects w undergo a hysterectomy. A large polyp is found upon opening the endomeotal cayjty (shown in the image). Histologic examination of this polyp will most likely show which of the following pathologic finru ngs7 ***(A) A£ypical endometrial hyperplasia ***(B) Chronic endometritis ***(C) Complex endometria] hyperplasia ***(D) Endometrial glands and fibrous stroma ***(E) Multiple foci of squamous metaplasia
22 The answer is D: Endometrial glands and fibrous stroma. cndomeuiaJ polyps occur most commonly in the perlmenopausal pe1iod and are vinually unknown before menarche. They are thought to arise from endometrial foci that are hypersensitive to estrogenic stimulation or umesponsive to progesterone. In either case such foci do not slough during menstruation and cominue to grow. Microscopically, the core of a polyp is composed of (1) endometrial glands , which often are cystically dilated and hyperplastic; (2) a fibrous endometrial stroma; and (3) thick-walled, coiled, dilated blood vessels. The other choices may be observed occasionally in an endometrial polyp. ***Diagnosis: Endometrial polyp
23 A 40-year-old woman presents with a 5-year 1-llsrory of dysmenorrhea. Physical examination and endocrine smdies are normaL A hysterectomy is performed. Hiswlogic examination of the uterine wall reveals areas of extensive adenomyosis. Which of the following best describes this patients uterine pathology? ***(A) Benign neoplasm of glandular epithdial cells ***(B) Displacemem of endometrial glands and stroma ***(C) Endometrial ilmaepithelial neoplasia ***(D) Hyperplasia of trophoblast as a sequel of incomplete abortion ***(E) Premalignant utertne lesion composed of smooth muscle
23 The answer is 8: Displacement of endometrial glands and stroma. Adenomyosis refe rs to the presence of endometrial glands and stroma within the myomeLrium. One fifth of aiJ uteri removed at surgery show some adenomyosis. Microscopic examination of these lesions reveals glands lined by mildly proliferative to inactive endomeuium and surrounded by endometrial stroma with varying degrees of fibrosis. \'lany patients \vith adenomyosis are asymptomatic; however, it is not uncommon for patients to exhibit varying degrees of pelvic pain, dysfunctional ute rine bleeding, dysmenorrhea, and dyspareunia. Adenomyosis does not represem a neoplastic process (choices A, C, and E). ***Diagnosis: Adenomyosis
24 A 60-year-old women presents wiLh a 2-week hisLOry of uterine bleeding. Gynecologic examination reveals an enlarged uterus. The hysterectomy specimen shows a large polypoid mass involving the endometrium and myometrium. Hiswlogic e.'<amination reveals mahgnam glands and malignam stromal elements, including striated muscle and cartilage. What is the approptiate diagnosis? ***(A) Carcinosarcoma ***(B) Endometrioid adenocarcinoma ***(C) Leiomyosarcoma ***(D) Pleomorphic adenoma ***(E) Rhabdomyosarcoma
24 The answer is A: Carcinosarcoma. Carcinosarcoma is an aggressive, mixed mesodennal tumor, in which the epithe1ial and stromal components are both highly malignant. These neoplasms are derived from multipotemlal stromal cells. The overall 5-year rate survival is 25%. Pleomorphic adenoma (choice D) is a mixed tumor of salivary gland. The other choices do n ot fearu re biphasic components. ***Diagnosis: Carcinosarcoma
25 A 2.5-year-old woman is referred to the gynecologist for treatment of infertility The patiem is obese (BMl = 32. kg/m2) and has pronounced facial hair. She states that she has always had irregular menstTual periods. On gynecologic examination, both ovaries are found to be symmetrically enlarged. This patients ovaries would likely show which of the following pathologic findings? ***(A) Bilateral endometriomas ***(B) Cystic teratoma ***(C) Mucinous cys~.adenoma ***(D) Serous cystadenoma ***(E) Subcapsular cysts
25 The answer is E: Subcapsular cysts. Polycysuc ovary syndrome, also known as Stein-Levemhal syndrome, describes (1) clinical manifestations related to the secretion of excess androgenic hormones, (2) persistem anovulation, and (3) ovalies containing many sma11 subcapsular cysts. lt was described inilially as a syndrome of secondary amenonhea, hirsmism, and obesicy: The clinical presemation is now recognized to be far more variable and includes amenorrheic women 'Nho appear otherwise nonnal and, even mrely, have ovaries lacking polycystic features. Up to 7% of women experience the polycystic ova1y syndrome, making this condition a common cause of infertility. Unopposed acyclic estrogen secretion in women with polycystic ovary syndrome results in an increased incidence of endometrial hyperplasia and adenocarcinoma. On gross examination, both ovaries are enlarged. On cut section, the cortex is thickened and discloses numerous cysts (typical! y 2 to 8 mm in diameter) arranged peripherally around a dense core of stroma. The other ch oices are nol typically associated with Stein-Leven thal synd rome. ***Diagnosis: Polycystic oval)' syndrome
26 Endoc rine studies of the woman described in Question 2.5 would most likely show which of the following results in the serum? ***(A) High levels of conicosteroids ***(B) High levels of follicle-stimulating hormone ***(C) High levels of luteinizing hormone ***(D) Low levels of esn·ogens ***(E) Lm>.' levels of corticosteroids
Z6 The answer is C: High levels of luteinizing hormone. Polycystic ova~y syndrome represems a state of functional ovarian hyperandrogenism associated ·with increased levels of luteinizing hormone (LH), although the increase in LH is probably a result raLher than a cause of the ovarian dysfunction. The central abnormality is thought to be increased ovarian p roduction of androgens, but adrenal hypersecretion of androgens may also contribute to the clinical mani£estations. ***Diagnosis: Polycystic ovary syndrome
27 A 50-year-old woman who has a family history of breast cancer presen ts with a 6-month history of increasing abdominal girth. On close questioning, she volunteers a histmy of vague abdominal pain dating back 1 year. She has no children and has never been pregnant. Bimanual pelvic examination reveals a 10-cm 1i ght adnexal mass. Percussion of the abdomen indicates ascites_ Aspiration cytology of the ascites nuid reveals malignant papillary su-uctures with psammoma bodies_ A mutation in which of the following genes is most likely associated with this patient's malignant disease? ***(A) BRCAl ***(B) p53 ***(C) Rb ***(D) VHL ***(E) WT-1
Z7 The answer is A: BRCA 1. Malignant papillary structures and psammoma bodies (laminated calcified concretions) in a patient with asciLes is most compatible with the diagnosis of papillary serous cystadenocarcinoma of the ovary: The same gene impUcated in hereditaty breast cancers, namely BRCAl , has been incriminated in the pathogenesis of famiUal ovariat1 cancer. Women who bear BRCAl gene mutations Lend to develop ovarian cancer considerably earlier than ·women who have sporadic ovarian cancer, bm their prognosis is considerably better. Mmations in the WT-1 tumor suppressor gene (choice E) are related to Wilms tumor. ***Diagnosis: Ovatian cancer, papillary serous cystadenocarcinoma
28 The patient described in Question 27 undergoes surgery to have the mass removed_ Histologic examination of the surgical specimen is shoY.rn in the image. The arrow points to a calcified focus (psammoma body)_ This neoplasm most likely originated from which of the follmving ova1ian cells/tissues? ***(A) Germ cells ***(B) Granulosa cel1s ***(C) Senoli-Leydig cells ***(D) Surface epithelium ***(E) Theca cells
28 The answer is D: Surface epithelium. The tumor depicted is a papilla ry serous cystadenocarcinoma. The most frequently encountered ovarian ntmors (e.g., benign and malignant serous and mucinous neoplasms) arise fTom the surface epithellum and are tenned common epithelial tumors. Epidemiologic studies suggest that common epithelial neoplasms are related to repeated disntption and repair of the epithelia] surface during normal cyclic ovulation. Thus, these tumors most commonly afflict women who are nuUiparous and, conversely, occur least often in women in whom ovulation has been suppressed (e.g., by pregnancy or oral comracepnves). Gem1 cells (choice A) give tise w benign teratomas and a vatiety of malignam tumo rs. The other cells give rise w sex cord!so·omal tumors. ***Diagnosis: Ovarian cancer, papillary serous cystadenocarcinoma
29 Which of the following statemenLS best characterizes the endocrine staLUs of the malignant cells m the patient described in Questions 2 7 and 28? ***(A) They are hormonally inactive. ***(B) They cause arterial hypenension. ***(C) They cause polymia and polydipsia. ***(D) They secrete polypeptide hormones. ***(E) They secrete steroid hom1ones.
29 The answer is A: They are hormonally inactive. Ovariru1 mmors that arise from the surface (germinal or celomic) epithelium are honuonally inactive and do nor produce endocrine syndromes. Ovarian masses rarely cause symptoms umiJ they are large. When they distend the abdomen, they cause pain, pelvic p ressure, or compression of regional organs. By the time ovarian cancers are diagnose~ many have metastasized to the surfaces of the pelvis, abdominal organs, or bladder_ Overall 5-year survival is only 35%. ***Diagnosis: Ovarian cancer
30 A 50-year-old woman presents with a 1-month hislOry of intermittent vaginal bleeding. A Pap smear is nonnal. Pelvic examination reveals a left adnexal mass. A uterine curettage shows complex endometrial hyperplasia without atypia. ACT scan of the abdomen reveals a 5-cm mass replacing the left ovary. The patient undergoes hysterectomy and bilateral salpingo-oophorectomy. Histologic examination of the ovarian mass is shown in the image. Which of the following is the appropriate pathologic diagnosis? ***(A) Dysgerminoma ***(B) Endometrioid carcinoma ***(C) Granulosa cell Lumor ***(D) Mucmous cystadenocarcmoma ***(E) Sertoli-Leyd•g cclllUmor
30 The answer is C: Granulosa cell tumor. Granulosa cell rumor is the prototypical functional neoplasm of the ovary associated with esLrogen secretion. The tumor is derived from sex cord stromal cells. Most granulosa cell tumo rs occur after the menopause. A juvenile form occurs in children and young women and has distinct clinical and pathologic features (hyperestrogenism and precocious puberty). Microscopically, granulosa cell tumors display haphazard orientation of the nuclei about a central degenerative space (Call-Exner bod- 3es), which results in a characteristic follicular histologic pat· tern. Three fourtl1s of granulosa cell rumors secrete estrogens. Consequently, endometrial hyperplasia is a common presenting sign. Hyperplasia may progress to endometrial adenocarcinoma if the functioning granulosa cell tumor remains undetected. Sertoli-Leydig cell tumors (choice E) typically secrete weak androgens. The oLher choices do not secrete hom10nes. ***Diagnosis: Granulosa cell tumor of the ovary
31 A 40·)'Car-old woman presenLS Wl[h 6 mom.hs of mcrcasmg abdommal ginh. Gynecologic examination reveals large bilateral ovariar1 masses. The pauem undergoes bilateral oophorectomy. l11e pathology repon reads ~ Krukenbcrg tumor:· and the histopatholOgiC findmgs arc shown m the unage. Which of the following tesLS would hkcly provtde the htghcst diagnosoc yield? ***(A) Serum AFP level ***(B) Biopsy of the cervix and endometrial curettage ***(C) Laparoscopy ***(D) Serum hCG level ***(E) Ga.snic endoscopy
31 The answer is E: Gastric endoscopy. Krukenberg mmors are ovatian metastases in which che tumor appears as nests of mucin-filled usignet ring" cells within a cellular sn·oma detived from the ovmy The stomach is the primaty si.te in 75% of cases, and most of the other Krukenberg mmors are fTom the colon. Bilateral ovarian Lnvolvement and multinodularity are lmponam clues to the djagnosis of metastatic carcinoma. ***Diagnosis: Krukenberg tumor of lhe ovary, gastric adenocarcmoma
32 A 15-year-old girl presems with left lower abdominal pain $he has noted recem enlargement of her breasts. Her last menstrual period was 10 weeks ago. She denies having had sexual intercourse. Serum levels of hCG are markedly elevated. Which of the following is the most likely diagnosis? ***(A) Choriocarcinoma ***(B) Hydatidifonn mole ***(C) Mature cystic teratoma ***(D) Serous cystadenocarcinoma ***(E) Yolk sac carcinoma
32 The answer is A: Choriocarcinoma. Choriocardnoma of the ovary is a rare tumor that mimics the epithebal cove1ing of placental vUii (cytou·ophoblast and syncytiotrophoblast). Choriocarcinoma of gem1 cell origin manifests in young girls as precocious sexual development, menstrual irregulatities, and rapid breast enlargement. In women of reproductive age, ovarian chmiocarcinoma may represent metastasis from an imraurerine gestational mmor. Microscopically, it displays an admixture of malignant cytotrophoblast and syncytiotrophoblast The syncytial cells of choriocarcinoma secrete hCG, which accounts for the frequem finding of a posirive pregnancy. test result. The tumor is highly. aggressive bur responds w chemotherapy. Hydatidiform mole secretes hCG bm is a gestational trophoblastic disease. Tbe other choices do not secrete hCG. ***Diagnosis: Choriocarcinoma of the ovary
33 A 20-year-old woman presents ·with increasing abdominal glnh of 3 months in duration. Physical examination reveals ascites. A pelvic examination discloses a right ovarian mass. A 7 -em ovarian mass is removed at surgery. The hisLOlogic appearance o[ this ovarian neoplasm (shown in the image) most closely resembles which of the following malignant neoplasms seen in males7 _.,_..,. ***(A) Choriocarcinoma ***(B) Embryonal carcinoma ***(C) Immature teratoma ***(D) Seminoma ***(E) Sertoli cell tumor
33 The answer is D: Seminoma. Dy.sgenninoma is the ovarian counterpart of testicular seminoma and is composed of activated germ ceUs. The neoplasm demonstrates large nests of monotonously uniform cells, which have a clear glycogen- filled cytoplasm and in·egularly nattened central nuclei. Fibrous septa containing lymphocytes traverse the tumor. The ot.her choices are also found in both se.,xes but do not show this histologic appearance. ***Diagnosis: Dysgerminoma
34 A 60-year-old woman presents with a 1-year history of vulvar itching, bleeding, and inflammation. Physical examination reveals a 1-cm exophytic mass on the labium major. Biopsy of the mass is shown in the image. These neoplastic cells would most likely express which of the following tumor markers? ***(A) Alpha-fetoprotein ***(B) Carcinoembryonic antigen ***(C) Cytokeratil15 ***(D) Estrogen/progesterone receptors ***(E) Her2/neu polypeptides
34 The answer is C: Cytokeratins. The rumor depicLed is a welldiJierentiated squamous cell carcinoma with keralin pearls. Squamous cell carcinoma is the mosL common ptimary malignant neoplasm of the vulva, and these tumors commonly express cyrokeratins. Squamous cell carcinoma of the vulva is the end result of a multistep process that has its migin in vulvar innaepitheltal neoplasia. Two thirds of larger tumors are exophytic: the others are ulcerative and endophytic. The tumors grow slowly and then extend to the contiguous skin, vagina. and rectum. They metastastze to the superficial inguinal and then the deep inguinal, femoral, and pelvic lymph nodes. The OLher rumor markers are not expressed by vulvar squamous cell carcinoma. ***Diagnosis: Squamous cell carcinoma
35 A 22~ ye.ar-otd woman presems to the emergency room with a 2-hour history of acme abdominal pain and vaginal bleeding Her vital signs are normal. Physical examination reveals blood oozing from the vaginal opening. Laparotomy shows an enlarged right fallopian tube with hemorrhage and ruprure. What is the most hkely cause of hemorrhage in this patient? ***(A) Choriocarcinoma ***(B) Ectopic pregnancy ***(C) Infarcted rubal polyp ***(D) Intramural leiomyoma ***(E) Tubal adenocarcinoma
35 The answer is 8: Ectopic pregnancy. Over 95% of ectopic pregnancies occur in the fallopian tube. Ectopic pregnancy results when the passage of the conceptus along the fallopian mbe i.s impeded, for example, by mucosal adhesions or abnormal tubal motilicy secondary to inflammatory disease or endometriosis. The trophoblast readily penetrates the mucosa and mba! wall. The thin tubal wall usually ruptures by the 12tll week of gestation. Tubal rupture i.s life threatening because it can result in rapid exsanguination. The other choices are rare. ***Diagnosis: Ectopic pregnancy
36 A 25-year-old woman in the last trimester of her first pregnancy presents for a routine obstet,.k evaluation. Her blood pressure is 160/100 mm Hg, and her pulse is 75 per minute. Physical examination shows pitting edema of the extremities. Urinalysis demonstrates 3+ proteinuria vVhich of the following is the most dangerous com plica lion of preeclampsia in this patient? ***(A) Amniotlc Ouid embolism ***(B) Chorioanmi.onitis ***(C) Choriocarcinoma ***(D) Disseminated imTavascular coagulation ***(E) Rupture of the fallopian tube
36 The answer is D: Disseminated intravascular coagulation. Preeclampsia usually begins insidiously after the 20th week of pregnancy \vith (1) excessive weight gain occasioned by iluid retention, (2) increased maternal blood pressure, and (3) the appearance of proteinuria. As the disease progresses from mild to severe preeclampsia, the diastolic pressure persisteml>• exceeds 110 mm Hg. Proteinuria is greater than 3 g per day, and renal funcnon declines. Disseminated mtravascular coagulation (DlC) often supervenes. DlC is a prominem feature of preeclampsia, manifested as fibrin thrombi in the liver, brain, and kidneys. The definitive therapy i.s rJ1e removal of the placenta, hopefully by normal delivery. The other choices are not complications of preeclampsia. ***Diagnosis: Preeclampsia
37 A 17 -year-old woman presents to her gynecologist v.rith a S~day history of vaginal bleeding. A home pregnancy test had been positive 1 week previously. This morning, the patient passed tlssue with the appearance of small grapes. An ultrasound shows a dilated endometrial cavity but no evidence or a fetus. Endometrial evacuation of the uterus by suction curettage reveals grapel.ike clusters, v.ith individual units measuring up to 5 mm in diameter (shown in the iJnage). Cytogenetic examination of this tissue \Vi.ll most likely demonstrate which of the following genetic pamms7 ***(A) Aneuplo1dy ***(B) Diploidy ***(C) Haploidy ***(D) Polypl01dy ***(E) Tnploidy
37 The answer is B: Diploidy. The term gestational trophoblastic disease embraces the spectrum of trophoblastic disorders that exhibit abnonnal proliferation and mamradon of trophoblast, as well as neoplasms derived from the trophoblast. Complete hydatidiform mole is a placenta that has grossly swollen chorionic villi, resembling bunches of grapes, in which there are varying degrees of trophoblastic proliferation. Complete mole results from the fertilization of an empty ovum that lacks funnional DNA. The haploid (23,X) set of paternal chromosomes duplicates to 46,XX. Hence, most complete moles are homozygous 46,XX, bm all of the chromosomes are of paternal origin. Since the embryo dies at a ve ry early stage, fetal pans are absent. Malignant transformation (choriocarcinoma) develops in about 2% of cases. Triploidy (choice E) is encountered in partial hydatidiform mole, but this diagnosis is ruled out by the absence of fetal tissue. ***Diagnosis: Complete hydatidiform mole
38 A 41-year-old immigrant woman from Asia presenLS for prenatal care. Her uterus is significantly larger than expected, and her serum hCG level is much higher than expected for her due date. No fetus is found on uln-asound examination. The abnormal placenta is removed. One month later, this patient presents w the emergency room with abdominal pain. ExploratOJy laparotomy reveals rupmre of the posterior uterine fundus with grape-like tissue exnuding from the defect. Two liters of blood are present in the abdominal cavity. Histologic examination of the ucerine mass is shown in the image. The arrows point to syncytial cells. Which of the following is the most likely diagnosis? ***(A) Carcinosarcoma ***(B) Choriocarcinoma ***(C) Embryonal carcinoma ***(D) Endomeuial adenocarcinoma ***(E) Yolk sac carcinoma
38 The answer is B: Choriocarcinoma. Cho1iocarcinoma occurs in 1 in 30,000 pregnancies in the United States. ln Asia, the frequency is far greater. Choriocarcinoma develops in about 2% of patients after a complete hydatidiform mole has been evacuated. Abnom1al uterine bleeding is the most frequent initial indication tha t heralds choriocarcinoma. Occasionally, the first sign relates to metastases m the lungs or brain ln some cases, chmiocarcinoma only becomes evident 10 or more years after the last pregnancy. The other choices are not sequelae of gestational trophobtastic disease. ***Diagnosis: Choriocarcinoma
39 A 34-year-old woman in the third trimester of her second pregnancy presents with a l-week history of vagin al bleedi11g. The patient subsequently gives birth to a healthy female at 35 weeks of gestation. lmmecliately after delive ry, the patient begins to hemorrhage transvaginally The bleeding cannot be comroUed, and the patiem undergoes emergency hysterectomy: Examination of the hysterectOmy specimen reveals penetration of chmionic villi deep into the myometrium, causing failure of the placemaltissue to fully separate from the uterine walL Which of the following best describes the uteroplacemal abnormality seen in tlcis patient? ***(A) Gestational choriocarcin oma ***(B) Abruptio placentae ***(C) Placenta increta ***(D) Placenta previa ***(E) Preeclampsia
39 The answer is C: Placenta increta. Abnonnal adherence of the placenta m the underlying U[erine wall is subclassified according to the depth of villous invasion into the myomeuium. Placenta accreta refers co the attachment of villi to the myomeu ·ium withoUL further invasion. Placenta increta (correct answer) defines villi invading the underlying myometrium. Placenta percreca describes villi penetrating the full thickness of the uterine wall. Most patients with placenta acreta have a normal pregnancy and delivery However, bleeding in the third trimester is the most common presenting sign he fore delivery. ln patients with placenta increta and percreta, substantial fragments of placenta may remain adherent to the uterine wall after delivery and are a source of postpartum hemorrlUlge. Abruptio placentae (choice B) refers to retroplacental hemorrhage in the absence of clinical hemorrhage. A deficiency of decidua at the implantation s ite may result from implantation of the placenta close to or over the ce rvLx (placenta previa, choice D). ***Diagnosis: Placenta increta
40 A 30-year-old pregnant woman asks for infonnation regarding mechanisms of sex detennination dming development. You explain that theY chromosome detennines male phenotype and that specific genital organs are inhibited from developing by hormones secreted by the developing testes. For e..-xample, muUetian-inhibiting substance released by SertoH cells causes the involution 0 r which of the following urogenital organs 7 ***(A) Breast ***(B) CliLOrls ***(C) Ovary ***(D) Ulerus ***(E) Vulva
40 The answer is D: Uterus. A central tene t of genital tract development in both sexes holds that the mullerian tubes will develop along female lines unless specifically impeded by embryonic testicular factors. ln males, $en oli cells in the developing testis produce mullerian-mhibiting substance, a protein that causes the mullerian ducts to regress. These ducts are the precursors of the fallopian ducts, merus, and upper third of the vagina. Fonnation of the ovary (choice C) and vulva (choice E) is not affected by this honnone. ***Diagnosis: Sex detemlination, mullerian-inhibiting substance
41 A 33-year-old woman presems after 3 weeks of a painful genital lesion. Physical examination reveals a Lender. eryLhematous, submucosal lesion of the labium minor (show11 in the image). Which of the following is the most likely diagnosis? ***(A) Bartholin gland cyst ***(B) Caruncle ***(C) Condyloma acuminatum ***(D) Extramammary Paget disease ***(E) Lichen sclerosis
41 The answer is A: Bartholin gland cyst. The Bartholin glands produce a clear mucoid secretion that cominuously lubticates Lhe vestibular surface. The ducrs are prone LO obstruction and cyst f01mation. Infection of the cyst leads to abscess fomlation. Bartholin gland abscess was formerly associated with gonorrhea, but staphylococci, chlamydia, and anaerobes are now more frequemly the cause. The other choices do not presem as discrete submucosal nodules. ***Diagnosis: Bartholin gland abscess
42 A 22-year-old woman presents to the emergency room with an 8-hour history of high fever, vomiting, diarrhea , and night sweats. Her temperature on admission is 38.7"C (l03°F), blood pressure 100/60 mm Hg, and respirations 24 per minute. She has a diffuse desquamative erythematous rash. Upon pelvic examination, the patient is found to be menstruating, and a tampon is in place. A purulem exudate is found within the vagina, which is cultured and grows Staphylococcus aureus. The hemoglobin is 12 g!dL, and the platelet count is 40,000/Jll. 'vVl"lich of the following represents the most common li[e-lluearening compUcation of this patients systemic disorder? ***(A) Acute tubular n ecrosis ***(B) Anemia ***(C) Cardiac arrhythmia ***(D) Disseminated inn·avascular coagulation ***(E) Pulmonary thromboembolism
42 The answer is D: Disseminated intravascular coagulation. Toxic shock syndrome is an acute, sometimes fa tal disorder characterized by fever, shock, and a desquamative erythenlawus rash. In addition, vomiting, diarrhea, myalgias, neurologic signs, and thrombocytopenia are common. Certain strains of Staphylococcus aureus release an exoro>..in called toxic shock syndrome toxin-1. ln addition lO the pathologic alterations characteristic of shock, the lesions of disseminated intravascular coagulation (DlC) are usually prominent. The disease was fi rst recognized when long-acting tampons were first introduced, prov1ding sufficient time for the staphylococcal . 1 . - I organisms to proliferate. The other chokes are less common and may be secondary w DIC. ***Diagnosis: Toxic shock syndrome
43 A 35-year-old woman complains of vaginal discomfort for 2 weeks. Physical examination reveals a scanty vaginal discharge. The nuid develops a "fishy" odor after treatment with 10% potassium hydroxide. A Pap smear taken during the pelvic examination shows squamous cells covered by coccobacilJj ("clue" cells). Which of the following is the most likely etiology of vaginal discomfort in l11is patient? ***(A) Chlamydia trachomatis ***(B) Gardnerella vagina/is ***(C) Herpes simplex virus ***(D) Human papUlomavirus ***(E) Tt·ichomonas vagina/is
43 The answer is B: Gardnerella vaginalis. Se.'Cual transmission of G. vagtnalrs, a Gram-negative coccobacillus, causes a substamial proportion of cases classified as nonspecific vaginitis. The diagnosis of Gardnerella infection is best established by identifying the organisms either in a wet mount specimen of a vaginal discharge or in a Papanicolaou-stained smear. The "clue cell" is pathognomonic and shows squamous cells covered by coccobacilli. Other aids w the diagnosis are a thin, homogeneous, milk-like vaginal discharge , a vaginal pH above 4.5, and the presence of a "fishy" odor from the discharge once alkalinized with 10% potassium hydroxide. Vi ruses (choices C and D) do not produce vaginal discharge. Choices A and E are not associated with "clue" cells. ***Diagnosis: Vaginitis, cervicitis
' 44 A 56-year-old woman presems 'vith a 3-momh history of vaginal bleeding. A cervical Pap smear reveals malignant, glandular epithelial cells. This patient most likely has a neoplasm miginating in which of the followmg anatomic locations? ***(A) Cervi'< ***(B) Endometrh.tm ***(C) Ovary ***(D) Vagina ***(E) Vulva
44 The answer is A: Cervix. Adenocarcinoma of the endocervi.x accounts for 20% of malignant cervical tumors. An increased incidence of cervical adenocarcinoma has been reported recently, with a mean age at p resentation of 56 years. Most of the tumors are of the endocervical cell (mucinous) type, bu t the various subtypes have little importance for overall survivaL Adenocarcinoma shares epidemiologic factors with squamous cell carcinoma of the cervix and spreads similarly The tumors are often associated with adenocarcinoma in situ and are frequently infected with HPV types 16 and 18. Malignam cells de rived from endometrial carcinoma (choice B) may be identi.fied occasionally by cervical Pap smear. ***Diagnosis: Adenocarcinoma of the exocervix
45 A 20-year-old woman presents for a complete physical examination. During the pelvic examination, a 5-cm cystic mass is found in the region of the right ovary. Radiographs show focal calcifications in the mass. The tumor is removed, and the surgical specimen is shown in the image. Which of the follmving is 1.he most likely diagnosis? ***(A) Dysgerminoma ***(B) Mature teratoma ***(C) Mucinous cystadenoma ***(D) Serous cystadenocarcinoma ***(E) Teratocarcinoma
45 The answer ls B: Mature teratoma. Mature teratoma is a tumor of gem1 cell origin that differentiates toward somatic snuctures. More than 90% contain skin, sebaceous glands, and hair follicles. Half of the tun.1ors exhibit smooth muscle , sweat glands, cartilage, bone, teeth, and respiratory tract epithelium. Ti.ssues such as gm, thyroid, and brain are encountered less frequently. Haploid (postmeioric) gem1 cells are believed to amo-fertilize, yielding diploid tumor cells that are genetically female (46,Xt'<). Teratocarcinoma (choice E) feamres inm1amre embryontc tissues and malignant stem cells. ***Diagnosis: Mature cystic teratoma of the ovary
A 4 3-year-old woman presents with a 6-momh hisrory of increasing abdominal glnh. On physical examinaLion, Lhere is pronounced ascites. Pelvic examinaLion reveals a lefL adnexal mass. A 6-cm ovarian rumor L<> removed. The mmor is solid and while. Histologically, it is composed of cells resembling normal ovarian srroma sunounded by collagen fibers. Which of the following is the appropriate diagnosis7 ***(A) Fibroma ***(B) Granulosa cell tumor ***(C) Leiomyosarcoma ***(D) Papillary cystadenoma ***(E) $ertoli-Leydig cell tumor
46 The diagnosis is A: Fibroma. Fibromas are the most common ovarian stromal rumors, accounting for 75% of all stromal n1mors and 7% of all ovarian rumors. They occur at all ages, with a peak in the perimenopausal petiod, and are vinually always benig11. The tumors are solid, finn, and white. Microscopically, the cells resemble the stroma of the normal ova1ian conex, being composed of well-differentiated fibroblasts and variable ammmts of collagen. Half of the larger mmors are associated with ascites and, rarely, ,,vith ascites and pleural effusions. Ascites is not a typical clinical feature of the other choices. ***Diagnosis: Fibroma o[ the ovary
47 A 25-year-old woman presents with a 6-month history of increasing facial hair, deepened voice, and amenonhea. Physical exanlinarion confirms virilization. A CT scan reveals a left ovarian mass. The tumor is surgically removed_ It measures 10 em m diameter and has a yellowish-tan appearance on cross section. The mmor i.s malignant and consists or two distinct cell populations. Some cells form solid nests, whereas Olhers are ananged in trabecular and gland-like structures. Which of the fo llo'wing is the appropriate diagnosis? ***(A) Brenner rumor ***(B) Dysgerminoma ***(C) Granulosa cell rumor ***(D) Mature cystic teratoma ***(E) Sertoli-Leydig cell rumor
47 The answer is E: Sertoli-leydig cell tumor. Senoli-Leydig cell mmor is a rare mesenchymal neoplasm of the ovary of low malignant potential that resembles the embryonic testis. It is the prowtypical functional tumor assodated with androgen secretion. The neoplastic ce lls typically secrete weak androgens (dehydroepiandrosterone), which account.<; fo r the large mmor size required w achieve masculmizing signs. SenoliLeydig cell tumor occu rs at all ages bm is most common in young >vomen of childbearing age. Nearly half of all pauenrs with Senoli-Leydig cell tumors exh ibit androgenic effects (i.e., signs of virilization, evidenced by hirsutism, male escutcheon, enlarged clitoris, and deepened voice). The initial sign is often defeminization, which is manifested as breast atTOphy, amenorrhea, and loss of hip fat. Once the rumor is removed, the signs disappear or are at least. ameliorated. The other choices are not associated with virilization. ***Diagnosis: Sertoli-Leydig cell mmor
48 A 25-year-old woman presents 1Nith a 6-month hisLOry of breasL enlargement and menstrual irregularities_ An endomeu ·ial biopsy 3 months previously showed complex hyperplasia withom atypia_ A CT scan of the pelvis reveals a left ovarian mass, which is subsequemly removed. The surgical specimen is solid and yellow, and measures 8 em in diameter. Histologkally; il is composed of lipid-laden theca cells. Following removal of dus neoplasm, a marked decrease in serum levels of which of the foU ovving hormones would be eAl)ected in this patient.? ***(A) Chorionic gonadotropin ***(B) Estrogen ***(C) Progesterone ***(D) Prolactin ***(E) Testosterone
48 The answer is B: Estrogen. Thecomas are functional ovarian tumors that arise in postmenopausal women. In most cases, they produce signs of estrogen production. Thecomas are solid tumors of 5 to 10 em in diameter. The cut section is yellow, ovving to the p resence of many lipid-laden theca cells. Microscopically, the cells are large and oblong to round, with a vacuolaLed cytoplasm thaL comains lipid. Bands of byalinized collagen separate nests of theca cells. Thecomas are almost always benign. Because of estrogen outpUL by the tumor, thecomas in premenopausal women commonly cause irregularity in menstrual cycles and breast enlargement. Endometrial hyperplasia and cancer are well-recognized complications. The oLher choices do noL produce endomemat hyperplasia. ***Diagnosis: Thecoma
49 A 34-year-old woman presents with increasing abdommal girth of 3 months in duration. Physical examination reveals a ldt ovatian mass and mild ascites_ The ovarian mass is removed, and the pathology report states "yolk sac carcinoma_" Which of the foUO\ving proVldes the best serologic marker to monitor the course of disease in tl1is patiem after surgery? ***(A) Alkaline phosphatase ***(B) Alpha-fewprotei n ***(C) Carcinoembryonic antigen ***(D) Human chorionic gonadotropin ***(E) Sex hormones (esu·ogen/progesterone)
49 The answer is B: Alpha-fetoprotein. Yolk sac tumor is a highly malignant tumor of women under the age of 30 years that histologically resembles mesenchyme of the primitive yolk sac. The mmor secretes alpha-feroprorein (AFP), which can be demonstrated histOchemically wirhln eosinophHic droplets. Detection of AFP in the blood is useful both for diagnosis and for monitoring the effectiveness of therapy. The hormone human chorionic gonadon·opin (choice D) IS secreted by choriocarcinoma. Estrogen (choice E) is secreted by sex cord rumors. ***Diagnosis: Yolk sac ca rcinoma
50 A 20-year-old woman presents to her gynecologist with a 3-day history of vaginal bleeding. An ultrasound shows a dilated endometrial cavity. Evacuation of the uterus by suction curettage reveals grape like clusters and fetal pans. Cytogenetic examination of this tissue will most likely demonstrate which of the following genetic patterns? ***(A) Aneuploidy ***(B) Diploidy ***(C) Euploidy ***(D) Haploidy ***(E) Triploidy
50 The answer is E: Triploidy. Cytogenetic examination of a partial hydatidifonn mole \\rill reveal niploidy. This abnom1al chromosomal complement results from the fenilization of a normal ovum (23,X} by two normal spermatozoa, each carrying 23 chromosomes, or a single spennatozoon that has not undergone meiotic reduction and bears 46 chromosomes. The fetus associated wnh a partial mole usually dies after 10 weeks of gestation, and the mole is aborted shortly thereafter. ln contrast lo a complete mole, which exhibits diploidy (choice B), fetal parts are commonly present in a panial hydatidifonn mole. ***Diagnosis: Partial hydaticUform mole
51 A 55-year-old nulliparous woman presents for a physical examination. The patient is obese (BMl c 33 kglm2) and has mild , adult-onset diabetes. Compared wiLh multiparous women, this patient is at increased risk of developing a neoplasm in which of the following anatomic locations? ***(A) CervLx ***(B) Endomemum ***(C) Endosalpinx ***(D) Vagina ***(E) Vuh·a
51 The answer is B: Endometrium. Tile major fo rm of endometrial cancer, endometrioid adenocarcinoma, is linked to prolonged estrogenic stimulation of the endometrium. ln addition to treatment with exogenous esrrogens, the most common risk factors are obesity, diabetes, n ulliparity, early menarche, and late menopause. Each risk factor points to relative hyperestrinism. A higl1 frequency of endomettial cancer is also found in women. with estrogen-secreting granulosa cell tumors. In the case of obesily, the Lncidence correlates with body weight, with the tisk being increased 10-fold for women who are more than 23 kg (50 lb) overweight. This effecL of obesity is related to Lhe enhanced aromatizallon of androstenedione LO estmne in adipocytes. Cancers of the other organs are nor related to estrogenic stimulation. ***Diagnosis: Endometrial adenocarcinoma
1 A 30~year-old woman suffers traumatic injury to her breast while playing soccer. Physical examination reveals a 3-cm area of ecchymosis on the left breast. Two weeks later, Lhe patient palpates a fim1 lump beneath the area where the bruise had been located. Which of the following is the most likely pathologic diagnosis? ***(A) Duct ectasia ***(B) Fat necrosis ***(C) Fibrocystic change ***(D) Granulomatous mastitis ***(E) Imraductal papillomatosis
1 The answer is 8: Fat necrosis. A ltist01y of trauma can usually be elicited in cases o[ fat necrosis occuning in the breasr. Initially, the lesion consists of necrosis of adipocytes and hemorrhage, after which phagocytic cells remove the lipid debris. Fibroblastic proliferation during healing leads w fingers of fibrous scar tissue that extend into the adjacem breast tis· sue. As a result, an irregular, fixed, hard mass may ensue and clinically resemble breast cancer. Dystrophic calcification, a common feanne of breast cancer, may also be detecred radio· graphically in areas of fat necrosis. Thus, the lesions often require biopsy to establish their benign character. The other choices are not associated with trauma. ***Diagnosis: Fat necrosis of lhe breast
2 A 50-year-old woman presents with a mass in her left breast mat she first detected 6 months earlier. A firm 4-cm mass is palpated on breast examination. Excisional biopsy reveals malignam cuboidal cells that fonn gland-like suuctures and solid nests, surrounded by dense collagenous sLroma. \Vhich of the following tem1s best describes the adaptive response of this patients normal breast tissue to the tumor? ***(A) Anaplasia ***(B) Desmoplasia ***(C) Fibnnolysis ***(D) Lipohyalinosis ***(E) Metaplasia
2 The answer is 8: Desmoplasia. Breast cancer is the most common malignancy of women in the United States, and the monallty from this disease among women is second only to that of lung cancer. Invasive, or infiltrating, ductal carcinoma is the most common fonn of breast cancer. In this cancer stromal invasion by malignam cells usually incites a pronounced fibroblastic proliferation. This "desmoplasia" creates a palpable mass, which is the most conunon initial sign of ductal carcinoma. Invasive ductal carcinoma usually manifesl5 as a nard, fixed mass, which is often referred w as scirrhous carcinoma. On gross examination, the rumor is typically fixm and shows irregular margins. The cut surface is pale gray and gritt)' and flecked with yellow, chalky streaks. Microscopically, invasive ductal carcinoma grows as irregular nests and cords of epithelial cells, usually within a dense fibrous stroma. Metaplasia (choice E) is the conversion of one differentiated cell type LO another. Lipohyalinosis (choice D) is a pan lcular form of fibrosis associated with fat deposition. Fibrinolysis (choice C) is related to dOL ctissolution. ***Diagnosis: Invasive ductal carcinoma of the breast
3 A 54-year-old woman complains of bloody discharge from her left nipple. Physical examination reveals a 0.5-cm nodule in the subareolar breasL tissue, which is surgically excised. Histologic examination (shown in the image) reveals cuboidal and myoepithelial cell-lined vascular connective tissue cores, which project imo the lllmen of a major lactiferous duct. Which of the following is the appropnate diagnosis? ***(A) Ductal carcinoma in siru ***(B) Intraductal papilloma ***(C) Lobular carcinoma in siru ***(D) Medullary carcinoma ***(E) Paget disease
3 The answer is B: Intraductal papilloma. lntraduCEal papilloma is a benign breast tumor that usually causes nipple discharge (serous or hemorrhagic) and occurs in the lactiferous ducts of middle-aged and older women. Because intraductal papilloma is situated in the large , subareolar ducts, the lesion may be associated vvith a serous oT bloody nipple discharge. This lesion must be distinguished from papillomatosis, which occurs in the pelipheral ducts as a component of proliferative fibrocystic change. Intraductal papil1omas are a ttached to the wall of the duct by a nbrovascular stalk. The papillomatous portion consists o[ a double Layer of epithelial celts, an outer layer of cuboidal or columnar cells, and an inner layer of more rounded myoepilheltal cells. Solitary tntraductal papilloma is not a premalignant Lesion or a marker for increased tisk of cancer in the breast. Ductal carcinoma tn situ (choice A) and lobular carcinoma in situ (choice C) feamre neoplastic celts confined to ducts and lobules, respectivel)~ and typically lack myoepithelial cells. Paget disease (choice E) is a form of carcinoma that involves the epideJmis or the nipple and areola. ***Diagnosis: Intraductal papilloma
4 A 53-year-old woman discovers a lump in he r breast and physical examination confirms a mass in the lower, outer quadrant o[ the left breast. Mammography demonstrates an ill-defined, stellate density measuring 1 em_ Needle aspiration reveals malignam ductal epithelial cells. A modified radical mastectomy is performed. The surgical specimen reveals a finn irregular mass (anows}. Which of the following celluJar markers would be the most usefulLO evaluate before considering therapeutic options for this patient? ***(A) CoUagenase ***(B) Estrogen receptors ***(C) Galactosylrransferase ***(D) Lysosomal acid hydrolases ***(E) Myeloperoxldase
4 The answer is B: Estrogen receptors. Over half of breast cancers exhibit nuclear esLrogen receptor protein. A slightly smaller proportion also has progesterone receptors. 'Nomen whose cancers possess hormone receptors have a longer disease-free survival and overall survival chan those with earlysLage cancers \"lho are negalive for these receptors. The beneficial effects of oophorectomy on survival in patients wiLh breast cancer led to the use o[ esnogen antagonists in the tream1ent of breast cancer_ ln general, antiesu·ogen therapy seems w prolong disease-free survival, particularly in postmenopausal and node-positive women. It also lowers the risk of cancer in the contralateral breast. The latter discovery has led to the use of amiestrogens as chemoprevention in women at high risk for developing breast cancer. None of the other choices are related prognostically to breast carcinoma. ***Diagnosis: Invasive ductal carcinoma of the breast
5_ A 35-year-old nulliparous woman complains rhaL her breasts are swollen and nodular upon palpalion. A mammogram discloses foci of calcification in both breasts. A breast biopsy reveals cystic duct dilation and ductal epithelial hyperplasta vvithom atypia (shmvn in the tmage) What is the appropriate dtagnosis? ***(A) Ductal carcinoma in situ ***(B) Fibroadenoma ***(C) Fibrocystic change ***(D) Granulomatous mastitis ***(E) lmraductal papilloma
5 The uswer is ( : Fibrocystic change. fibrocystic change of the breast refers to a constellation of morphologic fea tures characterized by (1) cystic dilation of terminal ducts, (2) relative increase in fibrous stroma, and {3) variable proliferation of tenninal duct epithelial elemen ts. Some of the fio rid manifestations appear to be indicators for an increased risk for breasL cancer. Such lesions are designated proliferative fibrocystic change. Forms of fibrocystic change that do nOl carry an in creased 1isk for the developmem of cancer, termed nonproliferative fibrocys tic change, are far more prevalent. Ductal carcinoma in situ (choice A) features apparently malignant epilhelial cells that have not penetrated the basemem membrane. 1mraductal papilloma (choice E) occurs in the subareolar lactiferous ducts. None of the remaining inconect chokes reature cystic duct dilation. ***Diagnosis: Fibrocystic change, proliferative
6 A 2.4-year-old woman delivers a 3.5-kg baby and begins breastfeeding her infant. The patient presents 2 weeks later with a fever of 38°C (101 °F). Physical examination shows no abnormal vaginal discharge or evidence of pelvic pain but does reveal redness on the lower side of the left breast. The patient stops nursing the infant temporarily, bm the symptoms persist, and the entire breast becomes swollen and painful. vVh.at is the most likely diagnosis? ***(A) Acute mastitis ***(B) Chronic mastitis ***(C) Duct ectasia ***(D) Granulomatous mastitis ***(E) Lactating adenoma
6 The answer i.s A: Acute mastitis. Acute mastitis is a bacterial infection of the breast. lt may be seen at an)' age. but by far the most frequent setting is in the postpartum lactating or involming breast. This disorder is usually secondary to obstmction of the duct system by inspissated secretions. The other choices are not typically associated \\ri.th fever. ***Diagnosis: Acute mastitis
7 A 35-year-old woman consults her family ph ysician because of painful swelling of her breasts, particularly as she approaches the end of her menstrual cycle. On self-examiJ1ation she recently felt a tender nodule in the right b reast. Physical examination reveals an inegular nodularity of both breasts wilh diffuse tendemess. Examination of the a..."Xilla is negative. A mammogram demonstrates irregular areas of density in the lower, outer quadrants of both breasts. Which of the following histopathologic features is considered to be a risk factor for the development of carcinoma in this patient? ***(A) Apocrine metaplasia ***(B) Cystic change ***(C) Duct ectasia ***(D) Papillomatosis ***(E) Stromal fibrosis
The answer is D: Papillomatosis. Proliferative fibrocystic change increases the risk of cancer. The most common proliferative change is an increase in the number or cells lining the dilated ten11inaJ ducts , desCiibed as ductal epithelial hyperplasia. Proliferative fi brocystic change can , at times, become exuberam and f01m papillary stmcturcs within the lumen of the distended ducmle (papillomatosis). The morphologic spectrum of ductal hyperplasia in patients with p roliferative fibrocystic change includes (1) minor degrees of h yperplasia; (2) Oorid, but cytologically benign hyperplasia; (3) hyper plasia with cytologic atypia not sufficient to warrant a diagnosis of malignancy; and ( 1-) ductal carcinoma in situ. The other choices do not increase the 1isk of breast cancer. ***Diagnosis: Fibrocystic change, proliferative
8 A 60-year-old man presents with painless, bilateral cnlargemem of both b reasts. The patient has a hisLDry of nodular prostatic hyperplasia and is taking medication for hypercholesterolemia. Physical examination reveals no discrete breast masses or axillary lymph node enlargement. Which of the following is the most likely Lmderlying cause of breast enlargement in dlis patient' ***(A) Chronic glOTnemlonephlitis ***(B) Cirrhosis ***(C) Nonseminomarous germ cell neoplasm ***(D) Parathyroid adenoma ***(E) Progressive systemic sclerosis
8 The answer is B: Cirrhosis. Gynecomastia refers to an enlargement of the adult male b reast and is morphologically similar to juvenile hypertrophy of the female breast. ln the adult man, gynecomastia is caused by an absolute increase in circulating esu-ogens or by a relative increase in the estrogen/ androgen ratio. Gynecomastia associated with excess estrogens occurs with (1) the imake of exogenous estrogens, (2) the presence of hormone-secreting adrenal or testicular rumors, (3) the paraneoplastic production of gonadotropins by cancers, and (4) metaboUc disorders, such as liver disease and hyperthyroidism, which are dcaracterized by increased conversion of androstenedione into estrogens Gynecomastia is often idiopathic, in which case it is commonly unilateraL The other choices are not associated with gynecomastia. ***Diagnosis: Gynecomastia
9 A 30-year-old woman presents with nipple discharge of 3 weeks in duration. Physical examination reveals a white djscharge from both nipples. The patient has nOL menstruated £or th e past 4 months, and she is not pregnant. The breasts are fim1 and nontender. A cytologic smear of the discharge shows no evidence of acute or chronic infiammawry cells. Which of the following is the most likely cause or galactorrhea in this patient? ***(A) Adrenal conical adenoma ***(B) Fibroadenoma of the breast ***(C) Fibrocystic change of the breast ***(D) Pituitary adenoma ***(E) Sheehan syndrome
9 The answer is D: Pituitary adenoma. Pintitary adenomas are benign neoplasms of the anterior lobe of the piruitary, which are often associated with excess secretion of pituitary hormones and evidence of corresponding endocrine hyperfuncdon. They occur in both sexes at almost any age bur are more cotru11on in men between the ages of 20 to 50 years. Small, nonfunctioning pituhary adenomas are found incidentally in as many as 25% of adult autopsies. Hyperprolactinemia is the most common endoetinopathy associated with pintitary adenomas. Prolactin secreted by pituitary lactotrophic adenomas may cause galactorrhea, most often in young women . Galactorrhea is not associated with the other choices. ***Diagnosis: Prolactinoma
10 A woman consults her physician because of painful swelling of her breasts. Physical examinalion reveals nodularity of both breasts. Mammography shows irregular areas of increased density in the lower, outer quadrants of both breasts. A breast biopsy reveals increased fibrous stoma, cystic dilation of the tenninal ducts, and varying degrees of apocrine metaplasia. This patients condition is most commonly seen in which of the fo llowing groups? ***(A) Patients with testicular feminization syndrome ***(B) Posonenopausal women ***(C) Pubenal girls ***(D) Women of reproductive age ***(E) 'Nomen treated with oral contraceptives
10 The answer is D: Women of reproductive age. Fibrocystic change is most often diagnosed in women from their late 20s to the time of menopause. and some fibrocystic change occurs in 75% of adult women in the United States. The mmvhologlc hallmarks of nonprohferative nbrocystic change seen in this patiem are an increase in fibrous stroma and cystic dilation of the terminal ducts. fibrocystic change occurs in multiple areas of both breasts. A dominant cyst or aggregate of fibrous connective tissue containing smaller cysts may manifest as a dLc;crete mass, prompting biopsy to exclude the possibility of cancer. The large cysts often contain dark Ouid that imparts a blue color-the so-called "blue-domed cysts of Bloodgood." Aspiration of a large cyst will usually cause it to collapse and the mass to disappear. A frequent concomitant of nonproliferative fibrocystic change is an alteration of the epithelial lining, termed apocrine metaplasia. The metaplastic cells are larger and more eosinophilic than the cells that usually line the ducts and resemble apocri11e sweat gland epithelium. The frequency of fibrocystic change decreases progressively after menopause (choice B). Fibrocystic change is not encountered during puberty (choice C). Oral contraceptives (choice E) do not increase the frequency of fibrocystic change. ***Diagnosis: fibrocystic change, nonproliferative
11 A 22-year-old woman presents with a painless nodule in the lower outer aspect of her right breast that she has had for 2 months. The nodule appears to be freely movable, sharply demarcated from the sunounding parenchyma, and finn. A mammogram demonstrates a ciTcumscribed, homogeneous density. A biopsy of the b reast mass is sho·wn in the image. 'vVhich of the follO\,ving best estimates the risk of subsequent invasive breast cancer developing in this patient? ***(A) Greater than 90% lifetime risk ***(B) Greater tJ1an 50% lifetime risk ***(C) Risk is doubJed ***(D) Risk is halved ***(E) No risk at all
11 The answer is C: Risk is doubled. Fibroadenoma is the most common benign neoplasm of the breao;L and is composed of epithelial and stromal elements that originate from the terminal duct lobular uniL Fibroadenomas are usually found in women between the ages of 20 and 35, although they may occur in adolescent girls. The tumor is round and rubbery, is sharply demarcated from the surrounding breast, and thus, is freely movable. The cut surface appears glistening gray-white On microscopic examination, fibroadenomas are composed of a mixture of fibrous connective tissue and elongated epithelial ducrs (see photomicrogravh). This connective tissue , which forms most of the mmor, often compresses the proliferated ducts, reducing them to curvilinear slits. The risk of subsequem invasive cancer in a breast from which a fibroadenoma has been removed is doubled. Surgkal removal is curative. Choices A and B are principally associated with BRCA mutations. ***Diagnosis: Fibroadenoma
12 A 20-year-old woman asks for your advice regarding her risk of developing breast cancer. Her mmher, maternal aum, and maternal grandmother all developed breast cancer. She would like to know if she has a genetic prerusposition. Laboratory tests for mutations in which of the fo!Jowing genes would be most likely to answer your patiems question? ***(A) BRCAl ***(B) C-myc ***(C) Estrogen receptor ***(D) HER21neu (£) Rb-1
12 The answer is A: BRCA 1. BRCAl is a tumor suppressor gene that has been implicated in the pathogenesis of hereditai)' breast and ovarian cancers. Mutations in this tumor suppressor gene are thought carried by 1 in 700 Lo 400 people in the United States. Ge.rmline poim mutations and deletions in BRCAl place a woman at a remarkable 60% to 85% lifedme risk for breast cancer. Moreover, breast cancer develops in more than half of these women before the age of 50 years. Jt is curremly suspected that mutated BRCAl is responsible for 20% of aU cases of inherited breast cancer (about 3% of all breast cancers). Somatic mutations in BRCAl are uncommon in sporadic (nonfamilial) breast cancers. Women with BRCAl mutations are also at greater lifetime 1.isk of ovarian cancer. Estrogen receptor expression (choice C) ls often increased in breasL cancer cells, bur the gene for the esn-ogen receptor is nor mutated. Neither estrogen receptor status nor HER2/rteu expression (choice D) predict genetic predisposition. ***Diagnosis: Breast cancer
13 A 26-year-old woman presems with a breast mass that was detected on self-examination l week earlier. Mammography reveals a round, sharply demarcated 1-cm nodule in Lhe right breast (shown in the image). Biopsy of the breast mass shows neoplastic epithelial ductal stmcrures sit~uated within a fibromyxoid suoma. The padem refuses further treatment and informs you that she wishes lO become pregnanL \Nhich of the following is the most likely effect of pregnancy on this breast lesion? ***(A) Developmem of invasive ductal carcinoma within the lesion ***(B) Fibrocyslic change with sclerosing adenosis ***(C) Fonnadon of imraductal papilloma ***(D) Metastasis to regional lymph nodes ***(E) Rapid growth
13 The answer is E: Rapid growth. Fibroadenomas commonly enlarge more rapidly during pregnancy and cease w grow after the menopause. Although they are hormonally responsive, a causal relationship between hormones and me pathogenesis of fibroadenoma has not been established. Development of invasive ductal carcinoma (choice A) in a fibroadenoma is rare. ***Diagnosis: Fibroadenoma
14 Upon self-examination, a 53-year-old woman discovers a lump in her left breast. Physical examination reveals a palpable lump about 1 em in diameter in the outer quadrant of the left breast. No palpable lymph nodes are found in the axilla. Mammography reveals an ill-defined, stellate density measuring 1 em in the left breast. Fine-needle aspiration of t11e mass discloses malignant epithelial cells. A panial mastectomy is perfonned and shows invasive ductal adenocarcinoma. 'vVhich of the following is the most imponant prognostic factor for this patient? ***(A) Estrogen receptor status of the tumor tissue ***(B) Histologic grade of the tumor ***(C) Inherited BRCAl gene mutation ***(D) Somatic mmation of t11e p5J tumor suppressor gene ***(E) Status of the axilla1y lymph nodes
14 The an.swer is E: Status of the axillary lymph nodes. Although all of the choices are prognostic indicators for breast cancer, the most imponam prognostic factor at the time of diagnosis ls stage. A seminel node assessment often is performed intraoperatively 1:0 assess the status of the ipsilateral lymph nodes. The sentinel lymph node is the most proximate lymph node and is assumed to be the inilial site of nodal metastasis. It is identified with a dye or radioactive male rial. An axillary lymph node dissection is performed if metastatic tumor is identified ln the sentinel lymph node. The p resence or invasion indicates that tumor cells l1ave access to lymphatic and blood vascular channels in the stroma, increasi ng the possibility of metastases to regional lymph nodes and distant sites. The prognosis for women with distant metastases (stage IV) i.s poor in terms of survival, but paUiative treatment may significantly prolong life. With the expanding use of screening mammography, more than half of the breast cancers cun-ently diagnosed in the UnLted States manifesc as stage l disease, and almost all of these women will be cured by surgery. ***Diagnosis: Invasive ductal carcinoma of the breast
15 A mammogram of a 52-year-old woman demonstrates calcifications in her left breasL No axillary lymph node enlargemem is detected on physical examination. An excisional biopsy is shown in the image. H this patient foregoes further tream1ent, which of the following best estimates her risk of developing invasive carcinoma in this breast over the next 20 years? .,., - --~-- --- ***(A) 1% ***(B) 5% ***(C) 30% ***(D) 90% ***(E) 100%
15 The answer is C: 300Jo. The biopsy reveals inlraductal carcinoma in siw, which arises in the tem1inal duct lobular unit, greatly distoning the ducts by its growth lntraduclal carcinoma in silu has two main histologic types, namely comedocarcinoma and noncomedocarcinoma. Noncomedocarcinomas exhibit a specrmm of cytologic atypia. The pauems are classified as micropapillary, cribn£onn (shown in the image), and solid. The tumor cells and nuclei are smaller and more regular than those of !.he comedo type. Noncomedo intraductal carcinoma in sim is less likely than the comedo type to incite a desmoplastic response in the surrounding tissue. Ductal carcinoma in situ, treated only by biopsy, car ries a 20% to 30% risk of developing invasive ca rcinoma in the same breast over the ensuing 20 years. The risk of cancer in the contralateral breast is also increased. Choices A and B are inconect because they suggest that the risk of invasive carcinoma is vety small, whereas choices D and E are far too great. ***Diagnosis: Ductal carcinoma in situ
16 A 54-year-old woman presems with a mass in her righL breast Lhat she fiTst palpated 5 days before. A breast biopsy reveals malignant cells, and a mastecwmy is perfonned. lmmunohLswchemical staining is perfmmed for HERl/neu (shown in Lhe image). Which or the following genetic mechanisms best accoums for the intensity or staining in thls specimen? ***(A) Gene amplification ***(B) Insertional mutagenesis ***(C) Chromosomal nonhomologous crossing over ***(D) Polyploidy ***(E) Single nucleotide polymorphism
t& The answer is A: Gene amplification. Overexpression of HER21 neu is identified in 10% to 35% of primary breast rumors and is mostly auributable lo gene amplification. Amplification or overexpression of HER2/neu has also been desctibed in cancers of the lung, ovary, and stomach. Overexpression can be determ. ined by immunohiswlogic detection of the c-erbB2 protein on the cell membrane or by analysis of the HER2/neu gene using nuorescent in situ hybridization. Patiems whose tumors demonstrate HER2 gene amplification benefit from therapy wilh a monoclonal antibody (Hercepti.n) thaL selectively binds to the extracellular domain of the protein. Although the other genetic processes occur in some cancers. they are unrelated to HER2/neu expression. ***Diagnosis: Breast cancer
17 A 45-year-old woman discovers a solitary, fTeely movable mass in her right breast on sel£-examination, which is confim1ed on physical examination. Mammography demonstrates focal calcification , with a linear configuration in the region of the breast mass. A breast biopsy (shown in the image) reveals large, pleomorphic epithelial cells con fined to d1lated ducts, ·with central zones of necrosis. What is the appropriate pathologic diagnosis? ***(A) Colloid carcinoma ***(B) Ductal carcinoma in situ, comedocarcinoma type ***(C) Medullary carcinoma ***(D) Phyllodes tumor ***(E) Tubular carcinoma
17 The answer is B: Ductal carcinoma in situ, comedocarcinoma type. lmraductal carcinoma in situ of the comedo type is composed of ve ry large, pleomorphic cells that have abundant eosinophilic cytoplasm and irregular nuclei, commonly with prominent nucleoli, and typically grows in a solid panern. Cetm·al necrosis is a prominent faClor. The necrotic debris may undergo dystrophic calcification On gross examination, the cur surface shows distended ducts containing pasty necrotic debris resembling comedos, hence the tenu comedocarcinoma. Although the malignant cells do not mvade through the basement membrane of tl1e ducts, this fonu of carcinoma in situ commonly incites a chronic inflammatory and fibroblastic response in the surrounding stroma. The cancer may extend within the duct system beyond the clinkally detectable rumor growth. The consequent difficulties in obtaining complete excision of the primary tumor frequenLiy necessitate mastectomy rather than "lumpectomy" The chances of local recurrence as either in situ or invasive cancer are substantially greater tn the case of the comedo subLype than the noncomedo subtype. ColJoid carcinoma (choice A) features abundam mucin producuon. Meduilary carcinoma (choice C) is composed of sheets of invasive and pleomorphic cells. Phyllodes LUmor (choice D) demonstrates proliferation of spindly stromal cells. Tubular carcinoma (choice E) is an invasive well-di[feremiated carcinoma with well-formed smaU duct s tructures. ***Diagnosis: Comedocardnoma, ductal carcinoma in situ
18 A 50-year-old woman has been aware of a mass in her left breast for the past 6 months. A 4-cm mass is palpated on examination. The mass is hard, tender, and fixed to the overlying skin. A lumpecromy is performed. The surgical specimen is firm, has poorly defined margins, and ems with a griny sensation. The cut surface is gray, opaque, and slighLly depressed. Streaks of gray connective tissue extend imo the sunounding fibroadipose tissue. The rumor histology is shO\.vn in the image. Which of the following risk facrors has the strongest association with this patiems rumor? ***(A) Exposure to carclnogens ***(B) Family history ***(C) Fibrocystic change ***(D) Obesity ***(E) Smoking
18 The answer is B: Family history. The strongest association with an increased risk for breast cancer is a family history, specifically breast cancer in firs t-degree relatives (mother, sister, or daughter). The risk is greater when the relative is afnicted at a young age or with bilateral breast cancer. A woman who has two sisters with breast cancer, one of whom had bilateral ttunors, or a morher and sister who show the same pattern has a greater than 25% chance of developing breast cancer by age 70 years. Fibrocystic change {choice C) also has an increased risk ofbreast cancer (proliferative lesions), but the relative tisk does nm approach that of family histmy ***Diagnosis: Invasive ductal carcinoma of the breast
19 A 58-year-old woman presents INith an n regular nodularity that has developed in her right breast over the past 3 months. Mammography demonstrates irregular densities in both breasts. A needle biopsy of one breast lesion is shown. An excislonal biopsy of the contralateral breast shows similar hiswlogy. Which of the following is the most likely pathologic diagnosis? ***(A) Colloid carcinoma ***(B) Lobular carcinoma in situ ***(C) Malignant phyllodes tumor ***(D) Medullary carcinoma ***(E) Tubular carcinoma
19 The answer is B: Lobular carcinoma in situ. Lobular carcinoma in situ arises in the terminal duct lobular unit. Malignant celts appear as solid clusters that pack and distend the tenninal ducts but not LO the extent of ductal carcinoma in situ. The lesion does not usually incite the dense fibrosis and chronic inn.ammaLion so characteristic of intraductal carcinoma in situ and is, therefore, less likely to cause a detectable mass. It is not uncommon for lobular carcinoma in situ to be an "incidental" finding in a biopsy that was prompted by benign changes. As with intraductal carcinoma in situ, 20% to 30% of women with lobular carcinoma in situ receiving no further treatment after biopsy will develop invasive cancer within 20 years of diagnosis. However, about half of these invasive cancers will arise in the contralateral breast and may be either lobular or ductal cancers. Thus, lobular carcinoma in situ, more than ductal carcinoma in sim, serves as a marker for an enhanced risk of subsequent invasive cancer in both breaslS. The histologic appearance is not consistent with any of the other choices. ***Diagnosis: Lobular carcinoma in situ
20 A 2.2-year-old woman nursing her newborn develops a render erythematous area around the nipple of her left breast. On physical examination, a purulent exudate is observed to drain from an open fLSsure. CuiLure of thLs exudate will most likely grow which of the following microorganisms7 ***(A) Candida albicans ***(B) Escherichia coli ***(C) Haemophil LLS tnjluenzae ***(D) Lacwbacillus aadopf11lus ***(E) Staphylococm~ au reus
20 The an.swer is E: Staphylococcus aureus. This lactating patient has developed acute mastitis. The most common organisms isolated are StaehylococcLL~ and StreptococcLtS. Untreated, the infection may progress to abscess fom1ation, which is a complication that necessitates surgical imervention. A finn, walled-off, nomender abscess may be mistaken for cancer. Acute bactetial mastitis may be treated successfully by aggressive mechanical suction, wilh frequent emptying of the breasts, and b)' the administrnlion of amibiotics. None of the other pathogens are ordinarily seen in acute mastitis. ***Diagnosis: Acute mastitis
21 A 52-year-old woman presents with a 3-month history of a palpable breast mass. Physical exantination confirms a 1-cm nodule in the upper outer quadrant of the tight breast. A biopsy reveals small cuboidal cells, with round nuclei and prominent nucleoli. The cells are arranged in single cell columns, between strands of connective tissue (shown in the irnage). Which of the following is the appropriate diagnosis7 ***(A) Ductal carcinoma in siru ***(B) Invasive ducLal carcinoma, Lubular type ***(C) Invasive lobular carcinoma ***(D) Lobular carcinoma in situ ***(E) Medullary carcinoma
21 The answer is C: Invasive lobular carcinoma. Invasive lobular carcmoma is the second most common Conn o( invastve breast cancer. Because the amoum of fibrosis is variable, the clinical presemation or invasive lobular carcinoma varies fTom a discrete firm mass, similar to ductal carcinoma, to a more subtle, diffuse, indurated area. MicroscopicaUy, classic invasive lobular carcinoma consists of single strands of malignant cells infiltradng between stromal fibers, which is a feature tenned Indian filing (see photomicrograph). Despite the innocuous cytologic characteristics of this fonn of invasive carcinoma, it is biologically as aggressive as the invasive ductal type. Twenty-live percent of invasive carcinomas have features of both ductal and lobular carcinoma. Lobular carcinoma in situ (choice D) is confined to the lobule. Invasive ductal carcinoma may share feanues of invasive lobular carcinoma, but lt usually forms glands, particularly the tubular type (choice B). ***Diagnosis: Invasive lobular carcinoma
22 A 58~year-old \'loman has a screening mammography and is found to have a 4-cm circumscribed mass, withou t calcifica· dons, in her lefL breast. An e.xdsional biopsy shows solid nesLS and sheeLS of highly pleomorphic cells, with many mitotic figures, surrounded by a dense infiltrate of lymphocytes. 'vVI1ich of the following is Lhe most likely diagnosis7 ***(A) InvaslVC ductal carcinoma ***(B) Invastve lobular carcinoma ***(C) Medullary carcinoma ***(D) Paget diSease ***(E) Phyllocles tumor
22 The answer is C: Medullary carcinoma. Medullary carcinomas present as fleshy, bulky tumors measuting 5 to 10 em in diameter. They are generally larger at the Lime they are detected than infiltrating ductal carcinomas {average size, 2 to 3 em). This invasive rumor presents as a circumscribed mass that lacks calcifications. On gross examinadon, medullary carcinoma appears as a well-circumscribed, fleshy, pale gray mass. Microscopically, it is composed of sheets of cells that are highly pleomorphic and have a high mitotic index. The pathologic definition of medulla1y carcinoma includes a lymphoid infiltrate encompassing the periphery of the tumor. Despite the highly malignam histologic appearance of this neoplasm, it has a distinctly better prognosis than infiltrating ductal or lobular carcinoma. A dense lymphoid infiltrate is nm charactelistic of the other choices. ***Diagnosis: Medullary carcinoma of the breast
23 A 45-year-old woman presenLc; \vilh an oozing, reddish patch on her left nipple (pal1ent shown m the image)_ The patiem has a history of skin rashes and food allerg~es and believes this condition is due to an allergic reaction ro her bra. Cytologic exarmnation of fiuid oozing from the skin lesion reveals neoplasuc cells. Exctstonal btopsy shows large clear malignant cells in the eptdcrmiS of the areola. WhiCh of the followmg IS the most likely dtagnosts? ***(A) Chr011lc de1matitis ***(B) Colloid carcinoma ***(C) Intraductal papllloma ***(D) Paget disease ***(E) Phyllodes tumor
23 The answer is D: Paget disease. Paget disease of the nipple refers to an uncommon variant of ductal carcinoma, ei ther in situ or invasive, that extends w involve the epidermis of the nipple and areola. This condition usually comes to medical attention because of an eczematous change in the skin of the nipple and areola. Microscopically, large cells with clear cytoplasm (Paget cells) are found singly or in groups within the epidermis. The prognosis of PageL disease is related Lo that of the underlying ductal cancer. Eczematous change in the skin of the nipple and areola are not fearures of tl1e other choices. ***Diagnosis: Paget disease of the breast
24 A 60-year-old woman presents with a large breast mass that she first detected .3 months ago. Mammography reveals a wellcircumscribed mass measuring 8 em in diameter. A breast biopsy shows loose fibroconnective tissue with a sarcomaLOus stroma , abundant mlt.oses, and nodules and ridges lined by cuboidal epithelial cells. Which of the foUowin g is the appropriate diagnosis? ***(A) Fibroadenoma ***(B) Medullary carcinoma ***(C) Paget disease ***(D) Phyllodes twnor ***(E) Tubular carcinoma
24 The answer is D: Phy11odes tumor. Phyllodes tumor of the breast is a proliferation of srromal elements accompanied by a benign growth of ductal strucmres. These mmors usually occur in women be £Ween 30 and 70 years of age. Phyllodes rumors resemble fibroadenomas in their overall architecture and the presence of glandular and s tromal elements. Uke fibroadenoma, benign phyllodes tumor is sharply circumscribed, and the cut surface is firm , glistenin g, and grayish while. Microscopically, the stroma of a benign phyllodes rumor is hypercellutar and has mitotic activity. The distinction from fibroadenoma is made not on the size, but on the histologic and cytologic characteristics of the stromal component Malignam phyllodes tumors have an obviously sarcomatous stroma with abm1dant mitotic activity, and the stromal component is increased oUL of proponton to the bentgn duct elements. They are usually poorly circumscribed, witl1 invasion into the sun-oundi.ng breast tissue. Sarcomatous elements are noL features of Lhe other choices. ***Diagnosis: Phyllodes tumor of the breast
25 A 65-year-old woman presen ts with a palpable breast mass that she palpated 1 month earlie r. Physical examination reveals a soft, jelly-like tumor measuring 5 em in diameter. Hiswlogic examiJ1ation of a breast biopsy is shown in the image. What is Lhe appropliate diagnosis? ***(A) Colloid carcinoma ***(B) Lobular carcinoma ***(C) Medullary carcinoma ***(D) Paget disease ***(E) Phyllodes mmor
25 The answer is A: Colloid carcinoma. Colloid (mucinous) carcinoma is an invasive variant that tends Lo occur in older women. On cut section colloid carcinoma has a glistening surface and mucoid consistency. Histologically, it is composed of small clusters of epnhelial cells, occasionally fomting glands, floating in pools or extracellular mucin. 1n its pure form, colloid carcinoma has a considerably better prognosis than infiltrating ductal or lobular carcinoma. However, it is often admixed with infiltrating ductal carcinoma, ill which circumstance the prognosis is derem1ined by the ductal component. Abundant mucin producrion is not a feam re of the other choices. ***Diagnosis: Mucinous carcinoma of the breast
26 A 55-year-old man presents with a solitary breast mass and biopsy reveals malignant cells. Immunohistochemical staining e:~qJerimems show that the tumor cel1s are positive for HER2/ neu and cyLOke racins 4 and 11 and negative for estrogen recepLOrs. What is the most likely diagnosis? ***(A) Basal cell carci noma ***(B) Invasive ductal carcinoma ***(C) Invasive lobular carcinoma ***(D) MeduJJary carcinoma ***(E) Tubular adenoma
26 The answer is B: Invasive ductal carcinoma. Cancer in the male breast is uncommon and accoums for less than 1% of all cases of breast cancer. The most common subtype is infiltrating (invasive) ducral carcinoma. Because there is less fat in the male breast, invasion of chest wall muscles is more frequent at the time of diagnosis. For tumors of the same srage, however, !he pmgnosis for male breast cancer is similar to that of female breast cancer. Choice A is a skin tumor and the o!.her choices (C, D, and c) are rare in the male breast. ***Diagnosis: Male breast cancer, invasive ductal carcinoma of the breast
27 Which of the following is though t to play a role in the developmem of cancer in the padem described in Question 26? ***(A) BRCA2 mmation ***(B) Chronic alcoholism ***(C) Gynecomastia ***(D) Hyperestrinism ***(E) PTEN mutation
27 The answer is A: BRCAZ mutation. Predisposing facwrs for !he development of breast cancer in men are largely unknO\vn, although mmaLions in the BRCA2 gene increase the risk of !his mmor. Choices B, C, and D are not risk factors for breast cancer in men. PTE.J.V mutations (choice c) are associated with endometrial imraepithelial neoplasia and endometrial adenocarcmoma. ***Diagnosis: Male breast cancer
1 An 18-year-old man moves from sea level to an elevation of 2,400 m to train as a skier. The increased requirement for oxygen delivery to tissues at the higher elevation stimulates the synthesis of a renal hormone (erythropoietin), which targets hemaLOpoietic stem cells in the bone marrow. Erythropoietin promotes the survival of early erythroid progenitor cells primarily through which of the following mechanisms? ***(A) Altered cell -matri" adhesion ***(B) Downregulation of p53 ***(C) Enhanced glucose uptake ***(D) lnhibnion of apopwsis ***(E) Stimulation of globin biosynthesis
1 The answer is D: Inhibition of apoptosis. Recent smdies indicate that erythropoietin promotes the survival of early erythroid progenitor cells through an inhibition of the default apoptosis pathway. Thus, this hom1one rescues stem cells that are otherwise fated w undergo programmed cell death. None of the other choices are known to control the e:-q)ansion of hemawpoietic s tem cell colonies in the bone marrow. ***Diagnosis: E1ythropoiesis
2 A 6-year-old girl is brought imo the emergency room after an automobHe accident. Physical examination shows bleeding from multiple wounds, and a CBC reveals a nonnocytic, normochromic anemia. Which of the following indices is most helpful in defining this padem's anemia as normocytic? ***(A) Hematocrit ***(B) Hemoglobin ***(C) Mean corpuscular hemoglobin concentration ***(D) Mean corpuscular volume ***(E) Red blood cell coum
2 The answer is D: Mean corpuscular volume {MCV) . MCV is the index used to measure the volume of a red blood cell. lt categorizes RBCs by size. Cells with normal size are called normocytic, smaller cells are termed microcytic, and larger cells are referred to as macrocytic. Mean corpuscular hemoglobin concentration (choice C) measmes hemoglobin content. ***Diagnosis: Anemia
3 A 60-year-old man presents ·with a 6-month h istory of increasing fa tigue Physical examination reveals marked pallor, and a CBC shows a macrocytic anemia. Vv'hich of the following is the most likely cause of anemia in this patient? ***(A) Alcoholism ***(B) Chronic disease ***(C) Iron deficiency ***(D) Renal disease ***(E) Thalassem1a
3 The answer is A: Alcoholism. Macrocytic anemia may be caused by impaired DNA synthesis due to a deficiency of folic acid or vitamin B12 • This results in abnonnal nuclear development, which, ln tum, leads to ineffective erythrocyte maturation and macrocytic anem1a. Folic acid deficiency is most commonly due to inadequate dietary intake, which often develops in patients with poorly balanced diets (e.g .• alcoholics). Other possible causes of macrocytic anemia include liver di<;ease, hypothyroidi'>m, and pnmary b one marrow disease . None of the other chmces are associated wilh macrocytic anemia. Chron ic disease (choice B) and renal disease {choice D) cause nonnoduomic, nonnocytic an emia. lron deficiency (choice C) and thalassemia (choice E) are microcytic anemias. ***Diagnosis: Macrocytic anemia
...._4_ A 4 3-year-old woman of Scandinavian descent complains of constant tiredness, Ught-headedness, and occasional palpitations and shonness of breath while ascending the stairs. Physical examination shows pallor of the oral mucosa and a raspberry-red wngue (glossitis). Neurologic examination reveals paresthesias, numbness, decreased vibration sensation , and loss of deep tendon refle-xes. The results of laborato ry studies include hemoglobin of 7.2 gldL, 'NBC of 4 ,500htL, platelets of 140,000/~t l, e rythrocyte folate of 220 ng/ml, serum vitamin B12 of 40 pgfml (no rmal >200 pg/ml), serum ami-intrinsic factor of 1:128, and semm anti-parietal cell antibody o f 1:6+. Examination of peripheral blood shows macrocytic anemia, '-"ilh poikiloc)rtosis of RBCs and hypersegmenred nelllrophils. Atrophic gastritis is diagnosed by gastric biopsy Bone marrow examination in this patient will reveal whkh of the following pathologic findings? ***(A) Absent stainable bone manow iron ***(B) Atypical megakaryocyres wilh fibrosis ***(C) Hypercellularity with megaloblasuc e1ythroid maturation ***(D) Hypocellularity with absence of erythroid precursors ***(E) Myeloid hyperplasia ·with increased basophils
4 The answer is C: Hypercellularity with megaloblastic erythroid maturation. Pernicious anemia is an au toimmune disorder in which patients develop antibodies directed against gastric parietal cells and inninsic factor. Parietal cell antibodies lead lO atrophic gastritis with achlorhydria. Deficiency of vitamin B11 or folic acid results m megaloblastic anemia. The peripheral blood smear shows macrocytosis and hypersegmemation of nemrophils. Megaloblastic maturation , charactetized by cellular enlargement with asynchronous mamration between d1e nucleus and cytoplasm, is noted in bone marrow precmsors £rom all lineages. Although the bone manow tends to be hypercellular, me blood demonsn·ares pancytOpenia because of ineffective hematopoiesis. Neurologic symptoms develop m vitamin B12 deficienC)~ secondaty lO degeneration of the posterior and lateral columns of the spinal cord. The OLher choices are not seen in pernicious anemia. ***Diagnosis: Megaloblastic anemia, pernicious anemia
5 Which of the following mechanisms of disease best describes th e pathogenesis of anemia in the pat1em described in Question 47 ***(A) Bone marrow fibrosis ***(B) Clonal stem cell abnormality ***(C) Defective heme synthesis ***(D) lmmune destruction of circulating erythrocytes ***(E) Impaired DNA synthesis
5 The answer is E: Impaired DNA synthesis. Megaloblastic anemias are caused by impaired DNA synthesis, usually due to a deficiency of either vitamin B12 or Colic acid. ln the face of defective DNA synthesis, nuclear development is impaired, whereas cytoplasmic maturation proceeds nom1ally. This simation, termed nuclear to cytoplasmic asynchrony, results in the formation of megaloblasLS. Because the megaloblastic precursors do not marure enough w be released into the blood, they undergo intramedullary destruction. Pernicious anemia is not related to any of me other choices. ***Diagnosis: Megaloblastic anemia
6 A 30-year-old woman complains of recent easy fatigability, bruising, and recurrent throat in fections. Physical examination reveals numerous petech iae over her body and mouth. Abnormal laboratory findings include hemoglobin of 6 gldl , 'vVBC of 1 ,5 00/~tl, and platelets of 20 ,000/~tl. The bone marrow is hypocellular and displays increased fat. What is the appropriate diagnosis? ***(A) Aplastic anemia ***(B) Iron-deficiency anemia ***(C) Megaloblastic anemia ***(D) Myelofibrosis with myeloid metaplasia ***(E) Pure red cell aplasia
6 The answer is A: Aplastic anemia. Aplastic anemia is a disorder of pluripotential stem cells that leads to bone marrow failure. The disorder features hypocellular bone man-ow and pancytopenia (decreased circulating levels o[ all fonned elements in the blood). Most cases are idiopathic. The bone marrow in aplastic anemia sho·ws variably reduced cellularity, depending on the clinical stage of the disease. There is a decrease in the number of ceUs of myeloid, erythroid, and megakaryocytic lineages, \Vith a relative increase in lymphocytes and plasma cells. As the cellulariry decreases, there is a corresponding increase in bone marrow faL Anemia, leukopenia {primarily granulocytopenia), and thrombocytopenia characterize aplastic anemia. Patients with aplastic anemla present with wealmess, fatigue, infection, and bleeding lron-deficiency anemia (choice B) and megaloblastic anemia (choice C) are not characterized by a hypoplastic bone man-ow. Myelofibrosis (choice D) shows increased connective tissue. Megakaryocytes and myeloid cells are nm decreased in pure red cell aplasia (choice E). ***Diagnosis: Aplastic anemia
7 A 20-year-old thin fashion model complains that she cannot concentrate and is always tired. She has heavy menstrual bleeding every month but is otherwise healthy. The peripheral blood smear is shown in the image. Which of the following laboratory findings would be e)..'Pected in this patient? ***(A) Hyperbilirubinemia ***(B) Increased serum ferri tin ***(C) Low plasma iron saturation ***(D) Positive direct Coombs test ***(E) Vitamin B 12 deficiency
1 The answer is(: Low plasma iron saturation. The blood smear ·'-- reveals microcytic, hypochromic erythrocytes, characteristic of iron deficiency anemia caused by inadequate uptake or, more often, excessive loss of iron. Women who have menorrhagia, especially those \:Vho consume restricted diets, are especia1ly prone to iron deficiency anemia. Iron stores of the body are reduced, as evidenced by reduced levels of serum fetTitin (not mcreased fe rriiin, choice B) and low imn samration (iron/tmal iron binding capacity). None of the other laborawry findings would be expected tn a patient with iron deficiency anemia. ***Diagnosis: Iron deficiency anemia
8 A 39-year-old woman presents with a 2-momh history of upper abdominal pain, weakness, and fatigue. Physical examination reveals marked pallor Laboratory studies show microcytic, hypochromic anemia (hemoglobin= 8.5 gldL) and mild thrombocytosis. Gastroscopy discloses a mucosal defect in the antrum measuring L5 em in diameter. Which of the following best describes the pathogenesis of anemia in this patient? ***(A) Defective globin chain synthesis ***(B) lmprured heme synthesis ***(C) Poor utilization of iron stores ***(D) Synthesis of suucrura!Jy abnormal hemoglobin molecules ***(E) Toxic damage to bone marrow stem cells
8 The answer is 8: Impaired heme synthesis. The p resence of a peptic ulcer incriminates gastrointestinal bleeding as the cause of anemia_ The resulting iron deficiency imerferes with heme symhesis and thus leads to impaired hemoglobin production and anemia. Defective globm chain symhesis (choice A) and synthesis of structurally abnormal hemoglobin molecules (choice D) are hemoglobinopathies. Poor miljzation of iron stores (ch01ce C) reflects sideroblastic anemia and anemia of chronic disease. ***Diagnosis: Iron deficiency anemia, peptic ulcer disease
9 A 10-year-old boy presents with chronic fatigue. Physical ..____ examination reveals slight jaundice and splenomegaly. The resuhs of laboratory studies include hemoglobin of 1L7 gld l , hemawcrit of 32%, total billrubin of 2.6 mgldL, and conjugated billrubin of 0.8 mgldL. The peripheral blood smear is shown in the image. The osmotic fragility of the patients RBCs is increased, but the Coombs test is negative. Defects in which o f the following are involved in the pathogenesis of this disorder? ***(A) DNA S}11thesis ***(B) Erythrocyte cytoskeleton ***(C) Erytlu-ocyte maturation ***(D) Glucose-6-phosphate dehydrogenase (G6PD) ***(E) Hemoglobin synthesis
9 The answer is 8: Erythrocyte cyt~skeleton. The smear shows many RBCs to be spherocytes, with decreased diameter and no ceno·aJ pallor. Hereditary spherocytosis {HS) represents a heterogeneous group of inherited disorders of the erythrocyte cytoskeleton, characterized by a deficiency of spectrin or other cytoskeletal components (ankytin, protein 4 2, band 3). Most fonus of HS are inherited as autosomal dominant traits, and most patients have a moderate normocytic anemia. The bone marrow demonstrates erythroid hyperplasia (erythroid matu ration is not affected). The deficiency of a cytoskeletal protein in HS leads to uncoupling o( the lipid bilayer from the lmderlying cy£oskeleton. The defect results in progressive loss of membrane surface area and fom1ation of spherocytes, which show increased osmmic fragility and are susceptible to chronic extravascular hemolysis. The osmotic fragility tesl is not abnormal in G6PD deficiency (choice D). ***Diagnosis: HerediLary spherocytosis
10 The patiem desc1ibed in Question 9 is at increased risk for development of which of the following conditions? ***(A) Acu te renal tubular necrosis ***(B) CholeUrJ1jasis ***(C) Cirrhosis ***(D) Nephrolithiasis ***(E) Portal hypertension
10 The answer is 8: Cholelithiasis . While circulating through the spleen , spherocytes lose additional surface membrane before they ultimately succumb to extravascular hemolysis and produce hyperbilirubinemia. Up Lo 50% of patients with spherocywsis develop cholelithiasis, with. pigmented (bilirubin) gallstones due to the increased supply of bilirubin. Kidney srones (choice D) do nor comain bilirubin. The liver (choices C and E) and kidney (choice A) are not affected by hereditary spherocytosis. ***Diagnosis: Hereditary spherocytosis
11 A 27-year-old pregnant woman comes to the obstetrician for a prenatal check-up. Routine laboratory testing reveals a mild normocytic anemia. The peripheral blood smear is shown in the image. Which of the following best explains the pathogenesis of anemia seen in lhis patient? ***(A) Abnom1al membrane lipoprotein molecules ***(B) Abnom1al polyme rization of spectrin molecules ***(C) Decreased iron release in the bone marrow ***(D) Destabilization of Lhe lipid bilayer of lhe RBC membrane ***(E) Oxidative denamration of hemoglobin
11 The answer is 8: Abnormal polymerization of spedrin molecules. The smear displays elliptical erythrocytes. Hereditary elliptocytosis (HE) rders to a heterogeneous group of inherited disorders involving the etythrocyte cytoske leton, all of which feaLU re a horizontal abnormality ·within the cytoskeleton. Variams of HE include defects in self-assembly of spectrin, speco ·in -ankyrin binding, protein 4.1, and gtycophorin C. ***Diagnosis: Hereditary elliptocytosis
12 A 10-momh-old boy of Arabic extraction is broughl to the physician by his parents who complain that Lheir child is failing to thrive. Physical examination reveals splenomegaly and jaundice. A CBC shows a microcytic, hypochromic anemia (hemoglobin= 7.4 fidL). Fetal hemoglobin accounts for most of the hemoglobin A peripheral blood smear is shown in the image. Which of the following is the appropriate diagnosis? ***(A) G6PD deficiency ***(B) Hereditary elliptocytosis ***(C) Hereditary sphera<::ytosis ***(D) Iron deficiency anemia ***(E) f3-Thalassemia
12 The answer is E: P· Thalassemia. The 13-thalassemias are a heterogeneous group of disorders that most often arise secondary Lo poim mmarions affecting the ~-globin gene. Accordingly, hemoglobin A (a2 P2) is not r armed. Unpaired a.-chains precipitate in red blood cells, accounting for ineffective er>rlhropoiesis and increased hemolysis. The blood smear shows features characteristic of thalassemia, including hypochromic and micro<:ytic RBCs, with anisocytosis, poikilocytosis, and target cells. tn homozygous ~-thalassemia, fetal hemoglobin (hemoglobin F) accounts for most of the hemoglobin, although increased levels of hemoglobin A2 are also present. Symptoms of the disease appear early in life, and affected children require constant transfusions. A heterozygous state for thalassemia may provide a protective effeCl against malatia and increase the reproductive pmemial of heterozygotes, thereby e;oq)laining the persistence of thalassemic disorders. Hemoglobin f is not increased in choices A, B, or C. ***Diagnosis: HomozygoLLS ~ -thalassemia
13 Which of the followi ng best describes the pathogenesis of splenomegaly seen in the patiem desclibed in Question 12? ***(A) Amyloidosis ***(B) Chronic malaria ***(C) Extramedullary hematopoiesis ***(D) Infectious mononucleosis ***(E) Splenic vein thrombosis
13 The answer is (: Extramedullary hematopoiesis. Increased oxygen affinity of hemoglobin F and Lhe underlying anemia impair m:ygen delivety and lead to marked bone marrow et)•throid hyperplasia. The marrow space is expanded, causing facial and cranial bone deformities. Extramedullary hematopoiesis contributes w sple·nomegaly and the formation of soft tissue masses. Excess erythropoiesis leads to i11creased iron absorption, which, together with repeated transf1.1sions, creates iron overload. Excess iron deposition in tissues is a major cause of morbtdity and monality in cl1alassemic patiems. The other choices may cause splenomegaly, but they are not related to ~-thalassemia. ***Diagnosis: Homozygous j3~thalassemia, splenomegaly
14 A 22-year-old woman from a large Italian family is screened for a familial blood disorder. The results of laboratory studies include a hemoglobin of 9.5 gtdL and a smear displaying mild m.lcrocytosis, hypochromia, and a few target cells. Hemoglo· bin electrophoresis shows a mild increase in hemoglobin A7 (7.5%). vVhat is the appropriate diagnosis? ***(A) Anemia of chronic disease ***(B) G6PD deficiency ***(C) Heterozygous P-thalassemia ***(D) Homozygous ~-thalassemia ***(E) Silem canier a -thalassemia
14 The answer is C: Heterozygous P·thalassemia. A normal hemoglobin molecule contains four globin chains, consisting of two ex.- and two non-a.-chains. Three normal variants of hemoglobin are encoumered, based on the nature of the non-a -chains. Hemoglobin A (cx.2 P2) accounts for 95% to 98% of the total hemoglobin in adults; only minor amoun ts of hemoglobin f {a2 y2) and hemoglobin A2 (<X2 52) are present. Heterozygous P-thalassemia is associated with microcytosis and h}1Jochromia, and the degree of microcytosis is disproportionate to the severity of the anemia, which is gener· ally mild. Target cel1s, basophilic stippling, and a mild increase in hemoglobin A2 are present. \'lost patients are as)mlplomatic. Choice D (homozygous P-thalassemia) is a more serious disease and choice E (silent canier for <X-thalassemia) is asymptomatic. ***Diagnosis: Heterozygous P- thalassemia
15 A 28-year-old woman delivers a male neonate at 36 weeks of gestation. The mother has a history of poor prenatal care and several previous miscaniages. Examination of the n eonate reveals marked pallor and generalized edema (anasarca), and the peripheral blood smear is shown in the image The nucleated cells in this blood smear are which of the following? ***(A) B lymphocytes ***(B) EosinophiJs ***(C) Erythrob lasts ***(D) Monocytes ***(E) T lymphocytes
15 The answer is(: Erytbroblasts. The peripheral blood smear displays erythroid precursors. which are nom1ally confined to the bone marrow. Hemolytic disease of the newborn reflects a hiswincompadbiliry between the mother and the developing fetus. The mother lacks an amigen that is expressed by the fetus. Maternal lgG alloantibodies cross the placema, causing complement-mediated hemolysis of feral erythrocytes and resulting in the release of numerous erythroid precursors (erythroblasts). The other choices represent nom1al immune cells. ***Diagnosis: Hemolytic disease of newborn, erythroblastosis fetalis
16 A 60-year-old man presents with headaches and pruntts. Physical examination reveals splenomegaly but no lymphadenopathy. A CBC demonstrates elevated hemoglobin or 19.5 gldL, WBC of 12,800/!!L, and platelets of 550,000/~tL. The bone marrow displays hypercellularity or aU lineages and depletion of marrow iron s tores. Which of the following is the most likely diagnosis? ***(A) Acme myelogenous leukemia ***(B) Essential thrombocythemia ***(C) Idiopathic myelofibrosis ***(D) Occult infection ***(E) Polycythemia ve ra
16 The answer is E: Polycythemia vera (PV). PV is a myeloproliferative disease that arises from a single donal hematopoietic stem cell and results in uncontrolled production ofRBCs. The increase in erythrocytes in PV is autonomous and is not regulated by erythropoietin. PV derives from the malignam rransfonnation of a single, hematopoietic sLem cell with primary comminnemto the erythroid lineage. Proliferation of the neoplastic clone occurs predominamlylnthe bone matTowbut may involve extramedutlary sites including the spleen, lymph nodes, and liver {myeloid metaplasia) The bone marrow is hypercellular with hyperplasia of all elements. The spleen is moderately enlarged, and its cut surface is unifonnly dark red, with expansion of the red pulp and obliteration of the whi te pulp. Acute myelogenous leukemia (choice A) and essential thrombocythemia (choice B) involve the myeloid and megakaryocytic lines, respectively. Idiopathic myelofibrosis (choice C) fearures manow collagen deposition (librosLs). ***Diagnosis: Polycythemia vera
17 The patient described in Question 16 is at increased risk of developing which of the following condidons7 ***(A) Cerebral aneurysm ***(B) Cerebrovascular accident ***(C) Cholelithiasis ***(D) Osteogenic sarcoma ***(E) Raynaud phenomenon
17 The answer is 8: Cerebrovascular accident. The patient has polycythemia vera (PV). Hyperviscosity associated with PV increases me risk for thrombotic stroke. The other choices are not associated with PV ***Diagnosis: Polycythemia vera
18 A 10-year-old black girl is brought to the emergency room. She complains of severe pain in her chest, abdomen, and bones. Physical examination reveals jaundice and anemia. Her parems state that she has been anemic since birth. A CBC shows normocytic anemia with marked poiki locytosis A peripheral blood smear is shown in the image. Hemoglobin electrophoresis demonstrates hemoglobin $. This childs chest and bone pain is most likely caused b y wh ich of the following mechanisms? ***(A) Amyloidosis ***(B) Coagulopathy ***(C) Infection ***(D) Ischemia ***(E) Vascubtis
18 The answer is D: Ischemia . Sickle cell disease is characterized by lhe presence of an abnonnal hemoglobin (hemoglobin$). Erythrocyte sickling is initially reversible with reoxygenation, but after several cycles of sickling and unsickling, the process becomes ineversible. Irreversibly sickLed cells display a reanangement of phospholipids between the outer and inner mono layers of the cell membrane. The erythrocytes are no longer deformable and are more adherent to endothelial cells, which are properties that predispose to thrombosis of small blood vessels. The resulting vascular occlusions lead to 'videspread ischemic complications, which are associated with severe pain, especially in the chesr, abdomen, and bones. ***Diagnosis: Sickle cetl disease
19 Over the next 6 years, the patient described in Question 18 develops multiple splenic infarcts. Which of the following is a common complication of auwsplenectomy in this patiem? ***(A) Autoimmune gastritis ***(B) Cholelithiasis ***(C) Megaloblastic anemia ***(D) Membranous nephropathy ***(E) Pneumonia
19 The answer is E: Pneumonia. The asplenic sr.ate associated with sickle cell anemia renders the patient susceptible to infections by encapsulated bacteria, especially Strepwcoccus pneumo11iae. ln addtion to splenic infarcts, patients with sickle cell disease frequently develop renal papillary necrosis due to conditions of low oxygen, low pH, and high osmolality in the renal medulla. None of the other choices represent complications of splenectomy. ***Diagnosis: Sickle cell disease
ZO A 24-year-old woman with sickle cell disease is seen in the emergency room for an acme upper respiratory tract infection. Laboratory findings include a severe, nonnocytic anetnia. The patient develops a rapid drop in the hemoglobin level. However, the reticulocyte coLmt is ve1y low (<0.01 %). This finding most bkely reflects which of the following conditions? ***(A) Bone man-ow failure due to repeated infarction ***(B) Expected result for the patient's underlying anemia ***(C) Parvovirus Bl9 infection ***(D) Ren·operironeal hemonhage ***(E) Vitamin B12 deficiency
20 The answer is C: Parvovirus 819 infection. Patients with sickle cell anemia may unde rgo an aplasoc crisis because of infection of the bone marrow by parvovirus Bl9, which suppresses erythrocyte production. None of the other choices are complications of sickle cell anemia. ***Diagnosis: Sickle cell anemia, aplastic crisis
21 A 36-year-old man from China presems with increastng fatigue. He has a 3-year history of tuberculosis, and CBC shows a mild microcytic anemia. Blood work-up demonstrates low serum iron, low iron-binding capacity, and increased serum femdn. The pathogenesis of anemia in this patient is most likely caused by which of the following mechanisms? ***(A) Clonal stem cell defect ***(B) Hypoxemia ***(C) Impaired utilization of iron from storage sites ***(D) Synthesis of structurally abnormal globin chains ***(E) Toxic damage to bone marrow stem cells
21 The answer is C: Impaired utilization of iron from storage sites. Anemia of chronic disease arises in association with chronic inOammatory diseases (e.g., tuberculosis and rheumatoid arthritis) and malignam conditions. Clu-onic disease leads to ineffective use of iron from macrophage stores in the bone marrow, resulting in a functional iron deficiency, although swrage iron is normal or even increased. The anemia of chronic disease is mild to moderate, and the red cells are often microcytic. Serum iron levels tend Lo be reduced. However, in comrast w iron deficiency anemia, total ironbinding capacity also tends to be decreased (as is the semm albumin level). The other choices are nm related to anemia of chronic disease. ***Diagnosis: Anemia of chronic disease
22 A 45-year-old chmnlc alcoholic man presents wilh memal confusion. The peripheral blood smear is shown in the image. The morphologic abnormalities demonstrated in this blood smear are most likely associated wilh which of the following conditions? ***(A) Abnom1al specuin in red ceU membranes ***(B) Chronic liver disease ***(C) Chronic renal fail ure ***(D) Microthrombi in capillaries ***(E) Vitamin B12 deficiency
22 The answer is B: Chronic liver disease. Acanthocytosis (shown in the phowmicrograph) results from a defect with in the lipid biJayer of the red cell membrane and features spiny projections of the surface, which may be associated with hemolysis. The most common cause o[ acanthoc)rtosis is chronic liver disease, in which increased free cholesterol is deposited within Lhe ceU membrane. Abnormalities in the lipid membrane cause eryth· rocytes to become deformed and develop in egular spiny surface proj ections and centrally dense cytoplasm (acamhocytes or spur cells). Chronic renal failure (choice C) features burr cells. Abnormal specnin (choke A) causes hereditary spherocytosis. ***Diagnosis: Acamh ocywsis
23 A 78-year-old man presents with increasing fatigue. A CBC shows pancywpenia, with moderate anemia (hemoglobin = 10.5 gldl) and nmmochromic, hypochromic RBCs. Mild neun ·openia and Lhromb ocytopen1a are noted. A bone marrow evaluation reveals erythroid hyperplasia with increased iron. A ?russian blue-stained bone marrow aspirate is shown in Lhe in1age. Which of the following is the appropriate diagnosis? ***(A) Hairy cell leukemia ***(B) Mulnple myeloma ***(C) Myelodysplastic syndrome ***(D) Polycythemia vera ***(E) Promyelocytic leukemia
23 TheanswerisC: Myelodysplasticsynd·rome {MDS) . MD$ exhibits dysplastic morphologic fearu res in one or more hematopoietic lineages and is accompanied by ineffective hematopoiesis. The disease is most common in the elderly and presents with anemia, neutropenia, and thrombocytopenia. The morphologic classification of MD$ is based on the presence of abnormally shaped hematopoietic cells and an increased proportion of myeloblasts. Dysplastic features may be present in one or more hematopoietic lineages. Ringed sideroblasts are common. In this case, a smear of a bone marrow aspirate stained \vith Prussian blue shows erythroid precursor cells containing iron-laden mitochondria thal encircle the nuclei. Ringed sideroblasts are not a fearure of the other choices. ***Diagnosis: Myelodysplastic syndrome
24 Which of the following best desCJibes the pathogenesis of the hematologic disorder seen in the patient desoibed in Quesdon 23? ***(A) Clonal stem cell defect ***(B) C-myc translocation ***(C) Deletion of a ponion of the ~-globin gene ***(D) Functional asplenia ***(E) Mutation of the T-cell recepwr gene
24 The answer is A: Clonal stem cell defect. Myelodysplasdc syndromes (MD$) are hematopoietic stem cell disorders mat are characterized by a discrepancy between the paucity of perlpheral blood elements and marked hyperplasia in the bone marrow. MDS may be either primary (de novo) or secondary (therapy related). Patients wlLh secondary myelodysplasia usually have a history of ch emOlherapy, especially alkylating agents, or radiation therapy for the treatment of can cer. Oth er 1isk factors for MDS include vimses, benzene e.'\l)Osure, cigarette smoking, and Fanconi anemia. ***Diagnosis: Myelodysplasia
25 A 32-year-old man presems v.rith mild fever and increasing fatigue. He is an immigrant from Russia and worked in a benzene factory Physical examination does not reveal lymphadenopathy or splenomegaly, but petechial skin lesions are noted. A CBC demonstrates severe pancytopenia, with normocytic red cell indices. A bone marrow biopsy is shown in the image. Which of the following is the most likely underlying mechanism in the development of this patient's anemia? ***(A) Damage to stem cells ***(B) Decreased erythropoietin production by the kidneys ***(C) Folate deficiency ***(D) Impaired globin chain synthesis ***(E) Neoplastic proliferation of committed stem cells
25 The answer is A: Damage to stem cells. The bone manow is aplastic , consisting largely of fat cells and lacking nonnal hematopoietic activity Patients wlth aplastic anemia presem with severe pancytopenia and clinical sym pwms related to the various cytopenias, including fatigue (anemia), fever (neu trop enia), and petechiae (thrombocytopenia). The lack of an approprlate reticulocyte response Lo the anemia indicates decreased or ineffective hematopoiesis as the mechanism [or the pancytopenia. lnjury to bone marrow stem cells is idjopathic (two thirds of cases), toxic (as in this case), inununologic, or hereditary (Fanconi anemia). ***Diagnosis: Aplastic anemia
26 A patiem with a hiswry of chronic alcoholism presents with a macrocytic anemia and thrombocytopenia. Blood smear exanlinmion demonstrates numerous oval macrocytes and hypersegrnemed neutrophils (results shown in the image). A Schilling test is nonnaL Which of the following is the most lil<ely diagnosis? ***(A) Anemla of chronic disease ***(B) Folic acid deficiency ***(C) GoPD deficiency ***(D) iron deficiency anemia ***(E) Sickle cell anemia
26 The answer is B: Folic acid deficiency. Folic acid deficiency commonly occurs in alcoholics who have poor nunition. Macrocytosis, hypersegmented neutTophlls, and a normal Schilling lest (vitamin B12 absorption) point to folic acid deficiency. Folic acid and vitamin B12 are required for synthesis of DNA, and deficiency of either fac tor leads to megaloblastic transformation of hematopoietic cells. Macrocytosis and hypersegmented neutrophils are not features of the other choices. ***Diagnosis: Megaloblastic anemia
27 A 23-year-old, previously healthy man of llalian origin develops moderate to severe hemolytic anemia. The previous evening he had celebrated a Saints day \vith a feast of beans and pasta. Ulinalysis shows free hemoglobin, and the direct Coombs test is negative. Supravital staining of the blood smear demonstrates numerous membrane-bound inclusions (Heinz bodies) within erythrocytes. Which of the follo\ving is the most likely diagnosis? ***(A) G6PD deficiency ***(B) Paroxysmal nocturnal hemoglobinuria ***(C) Sickle cell anemia ***(D) P-Thalassemia minor ***(E) Warm antibody autoimmune hemolytic anemia
27 The answer is A: G&PD deficiency. G6PD deficiency is an X-linked disorder that causes a hemolytic anemia characterized by abnormal sensitivity of red cells to oxidative stress. The highest prevalence is in Africa and the Mediten-anean region. Because of the role of G6PD in recycling reduced glutathione, red cells deficient in this enzyme are susceptible to oxidative stress, which, in this case, is fava bean ingestion {favism). In quiescent periods, the erythrocytes of G6PD deficiency appear normal. However, during a hemolytic episode precipitated by oxidative stress, Heinz bodies can be demonstrated by supravital staining. Full expression of G6PD deficiency is seen only in males, with females being asymptomatic can-iers. Heinz bodies are not characteristic of the other choices. ***Diagnosis: Glucose-6-phosphate dehydrogenase deficiency
28 A 60-year-old man complains of night sweats, weight loss, easy fatigability, and discomfon in the left upper abdominal quadrant. Physical examination reveals splenomegaly. Laborawry studies show leukocywsis (40,000/~LL). A peripheral blood smear demonstrates mature and marming granulocytes, myelocytes, basophils, and occasional myeloblaslS. The bone man ow is hypercellular and dominated by 'NBC precursors. Megakaryocytes are numerous, and RBC precursors are less prominent. A smear of the bone marrow aspi rate is shown in the image. Cytogenetic studies disclose a monoclonal popula· tion of abnormal cells with a t(9;22)(q34;q ll) chromosomal translocation. \11/hat is the appropriate diagnosis? ***(A) Acme lymphoblastic leukemia ***(B) Acute myeloid leukemia ***(C) Chronic lymphocytic leukemia ***(D) Chronlc myelogenous leukemia ***(E) Myelodysplastic syndrome
28 TheanswerisD: Chronic myelogenous leukemia (CML). Chronic myeloproliferative diseases are defined as clonal hematogenous stem cell dLo;;orders with increased proliferation of one or more myeloid lineages. CML is derived from an abnormal pluripotem bone marrow stem cell and results in prominent neutrophilic leukocytosis over the f·ull range of myeloid maturation. CML is the most common myeloproliferative disease and accounts for 15% to 20% of all cases ofleukemia. It affects middle-aged or older adults. Replacement of the bone marrow by neoplastic cells causes anemia and thrombocytopenia and a predisposition to infections. ln 95% of all CML cases, the Philadelphia chromosome can be demonstrated by conventional cytogenetics. The initial symptoms are nonspecific and include weakness, malaise, fever, and splenomegaly Acme lymphoblastic leukemia {choice A) and acute myeloid leukemia (choice B) feature clonal expansion of lymphoblasts and myeloblasts, respectively. Although myelodysplasric syndrome (choice E) features hyperplastic bone marrow, the Philadelphia chromosome does not occur, and there is peripheral cytopenia in va1ious cell lines. ***Diagnosis: Chronic myelogenous leukemia
29 Which oncogene is located aL the t(9 ;22) chromosomal breakpoint in the patiem described in Question 28? ***(A) abl ***(B) erb ***(C) myb ***(D) myc ***(E) neu
29 The answer is A: ab/. Presence of the Philadelphia chromosome or molecular demonstration of the bcr!abl fusion gene is required to establish the diagnosis of chronic myelogenous leukemia (CML). The bcr/abl gene encodes a fusion protein, pliO, which acts as a constitutively activated tyrosine kinase_ The other choices may be involved in malignant transfonnations bm they are not related to CML ***Diagnosis: Chronic myelogenous leukemia
30 A 40-year-old woman complains of fatigue and nausea of 3 months in duration. Ph)1Sical examination reveals numerous pusmles on the face, as well as splenomegaly and hepatomegaly. Laboratory studies show hemoglobin of 6.3 gldl and platelets of 50,000/~-tL A pe1ipheral smear shows malignant cells with Auer rods (anow). The patient develops diffuse purpura, bleeding from the gums, and laboratory features of disseminated intravascular coagulation (DlC). Which of the following is the appropriate diagnosis? ***(A) Acute lymphoblastic leul<emia ***(B) Acme megakaryocytic leukemia (C} Acme promyelocytic leukemia ***(D) Chronic myelogenous leukemia ***(E) Monocytic leukemia
30 The answer is C: Acute promyelocytic leukemia (APL) . ln APL, the bone marrow is packed wilh tumor cells that have promydocytic features, wilh abundant Auer rods. Patients with APL frequently presem wiLh D£C. Senescent leukemic ceUs degranulate and acrivaLe the coagulation cascade. The presence of Auer rods excludes acme lymphoblasLic leu kemia (choice A) , acute megakaryocytic leukemia (choice B), and chronic myelogenous leukemia (choice D). DIC is not characteristic of the other choices. ***Diagnosis: Acute promyelocytic leukemia
31 CyLOgenetic studies in malignant cells from the patient described in Question 30 demonstrate a chromosomal translocation .. Which of the following genes is most likely found at the translocation site' ***(A) abl ***(B) bel-l ***(C) bcl-2 ***(D) myc ***(E) Rednoic acid receptor
31 The answer is E: Retinoic acid receptor (RAR) . The underlying genetic defect in acute promyelocydc leukemia is a translocation involving the PML gene on chromosome 15 and the RAR (RARa) gene on chromosome 17. The resulting PMURARa fusion gene encodes a functional RAR. The receptor can be targeted by all-trans-retinoic acid, which mediates maturation of the tumor cells. Complete remissions have been obtained in some patients. The other choices may be involved i11 other translocations. ***Diagnosis: Acute promyelocytic leukemia
32 A 60-year-old man presents with a 3-week history of lymph node enlargement in his neck and axillae. A CBC reveals mild anemia, with a leukocytosis of 20,000/)lL The peripheral blood smear is shown in the image. More than 80% of 'vVBCs are small lymphocytes, blll there are also prominent "smudge cells." Examination of a bone maJTow biopsy shows nodular and interstitial infiltrates of lymphocytes, which demonstrate donal rearrangement of the IgG light-chain gene. Which of the following is the appropriate diagnosis? ***(A) Acute lymphoblastic lymphoma ***(B) Chronic lymphocytic leukemia ***(C) Chronic myelogenous leukemia with lymphoid blast crisis ***(D) M1.lltiple myeloma ***(E) Waldensuom macroglobulinemia
32 The an-swer is B: Chronic lymphocytic leukemia (CLL). CLL is charac terized by clonal proliferation of small, mamreappearing lymphocytes in du~ bone marrow, lymph nodes, and spleen, with an expression in the peripheral blood. ln most instances, the leukemic cells belong lOB-cell lineage and show clonal lg gene rearrangements and activation of the bel prowoncogene. Most patients are over 50 years of age. The sympwms tend to be nonspecific, bm 80% of patients have lymph node enlargement, and 50% show splenomegaly CLL usually has an indolent and protracted course. Acme lymphoblastic lymphoma (choice A} is principally a leukemia of childhood. Multiple myeloma (choice D) is a neoplasm of plasma cells. Waldenstrom macroglobulinemia (choice E) is a neoplasm of small lymphocytes and a variable number of lgM-secredng plasma cells of t.he same malignant clon e. ***Diagnosis: Chronic lymphocytic leukemia
33 A 6-year-old boy presents with fatigue, fever, and night sweats. Physical e.,xamination reveals marked pallor. Palpation of his stemum demonstrates diffuse tenderness. Laboratory studies disclose anemia, thrombocytopenia, and leukocytosis. The 'NBC differential coum shows that 90% blasts. A bone marrow biopsy stained immunohistochemically for terminal deoxynucleotidyl transferase (TdT) is shown in the image. Which of the following is the appropriate diagnosis' ***(A) Acme lymphoblastic leukemia ***(B) Acute myelogenous leukemia ***(C) Acme promyelocytic leukemia ***(D) Chronic lymphocytic leukemia ***(E) Chronic myelogenous leukemia
33 The answer is A: Acute lymphoblastic leukemia (ALL). Most precursor B-cell mallgnancies involve primarily bone marrow and peripheral blood and are te rmed B-ee U acme lymphoblastic leukemias (B-ALL). Only 15% of childhood All s) are de1ived from T cells, and 75% of B-ALL cases occur in children under the age of 6 years . B-ALL features numerical aberrations and chromosomal translocations, including the Philadelphia clu·omosome. ln childhood ALL, a balabl fusion protein, Pl90, is produced. B-ALLs are positive for nuclear expression of TdT The demonstration of TdT activity suggests that a leukemic blast is of lymphoid rather than myeloid lineage The orher choices are rarely, if ever, encountered in this age group. ***Diagnosis: Acute lymphocytic leukemia
34 A 27-year-old man presents with an 8-week history of fevers, chills, pruritis, and night sweats. Two months ago, he experienced a nu-like illness. A nagging cough wi th occasional hemoptysis persisted for seve ral weeks following resolution of his other symptoms Physical examination reveals moderately enlarged, firm, nomender lymph nodes located in Lhe right supraclavicular region. A lymph node biopsy is shown in the image. What is the appropriate diagnosis? r ***(A) Acllle myelogenous leukemia ***(B) Btn·kitt lymphoma ***(C) Hodgkin lymphoma ***(D) Infectious mononucleosis ***(E) Lymphoblastic lymphoma
34 The answer is C: Hodgkin lymphoma (HL) . The lymph node biopsy shows a Reed-Sternberg cell. These large atypical mononuclear or multinucleated tumor cells in an inOammatory background are the diagnostic hallmark of HL. HL is the most common malignant neoplasm of Ameri.cans between the ages of 10 and 30 years. There is a distinctive bimodal age disnibmion in developed countries. Most patients with HL present with lymphadenopathy. Constitutional symptoms include night sweats, fever, and weight loss exceeding 10% of body weight. Pmrims may occur with disease progression. Reed-Sternberg cells do not occur in the other choices. ***Diagnosis: Hodgkin lymphoma
35 'vVhich of the following is the most common hiswlogic subtype of the disease in the patient described in Question 34? ***(A) Lymphocyte depleted ***(B) Lymphocyte predominam ***(C) Lymphocyte rich ***(D) Mixed cellularity ***(E) Nodular sclerosis
35 The answer is E: Nodular sclerosis. Nodular sclerosis accounts for 70% of classical Hodgkin lymphoma, with most cases occurring between the ages of 20 and 30 years. Histologic examination shows dense, band-like collagenous fib rosis that envelops cellular aggregates of lymphoid and inflammator y cells, as well as the specific lacunar cell var ian t of the Reed-Sternberg cell ***Diagnosis: Hodgkin lymphoma, nodular sclerosis subtype
36 A 55-year-old man complains of pain in his back, fatigue and occasional conf·usion. He admits w polyUJia and polydipsia. An X-ray examination reveals numerous lytic lesions in the lumbar vertebral bodies. Laboratory studies disclose hypoalbuminemia, mHd anemia, and thrombocytopenia A monoclonallgK peak is demonstrated by serum electrophoresis. U1inal)'Sis shows 4+ proteinuria. A bone manow biopsy discloses foci o[ plasma cells, which account for 18% of alJ hematopoietic cells \"'hat is the approp1iate diagnosis? ***(A) AcUle lymphoblastic lymphoma ***(B) Chronic lymphocytic leukemia ***(C) Extramedullary plasmacytoma ***(D) Multiple myeloma ***(E) Waldenstrom macroglobulinemia
36 The answer is D: Multiple myeloma. Plasma cell myeloma (multiple myeloma) is characterized by a mul ufocal infiltration of malignant plasma cells in the bone manow. There are typically multiple destntctive lytic lesions or diffuse demineralization of bone. Major diagnostic crite1ia for plasma cell myeloma include manow plasmacytosis (>30%), plasmacytoma on biopsy, and immunoglobulin paraprotein (M-componem). Neoplastic cells typically secrete a homogeneous complete or partial immunoglobulin chain, which can be seen in serum or urine electrophoresis. ln this patiem, the neoplastic done secretes excess kappa light chains. Waldenstrom macroglobulinemia (choice E) is a neoplastic disorder of small lymphocytes that secrete lgM. ***Diagnosis: Multiple myeloma
37 For the patient described in Question 36, which of the following is the most common, and ultimately lethal, extramedullary complkation? ***(A) Dementia ***(B) Hepatic failure ***(C) Peticarditis ***(D) Petitonitis ***(E) Renal failure
37 The answer is E: Renal failure. The most common and important extramedullary complication of multiple myeloma is amyloid nephropathy, which accounts for more than half of aU deaths. Other complications include bone fractures and infection. ***Diagnosis: MulLiple myeloma
38 A 62-year-old man presems vtith a hiswry of several momhs of vague abdomb1al pain and fatigue. Physical examination reveals marked splenomegaly but no evidence of lymphadenopathy. The patient subsequently develops bacterial sepsis and e}.-pires. A bone marrow biopsy at autopsy shows n umerous atypical megakaryocytes and marked marrow fibrosis ( results shown in the image). Which of the following is the most likely djagnosis? ***(A) Acme myelogenous leukemia ***(B) Acme promyelocytic leukemia ***(C) Chronic idiopalhic myelofibrosis ***(D) Chronic lymphocytic leukemia ***(E) Chronic myelogenous leukemia
38 The ancSwer is C: Chronic idiopathic myelofibrosis. Chronic idiopad1ic myelofibrosis is a clonal myeloproliferative disease in which manow fibrosis is accompanjed by prominem megakaryopoiesis and granulopoiesis. A prefibroric stage is recognized wherein the bone marrow is hypercellular, with predominant neutrophilic and megakaryocytic proLiferation. In the fibrotic stage, the peripheral blood shows eiLher leukopenia or marked leukocytosis, and myeloid p recursors and nucleated RBCs are usually presem. Conspicuous reticu]jn or collagen fibrosis in the marrow defines this stage. Transformation to acute myelogenous leukemia (choice A) occurs in 15% of cases. The otl1er choices do nol feature marrow fibrosis. ***Diagnosis: Myelofibrosis
39 A 57 -year-old man is admilted to the hospiml with inguinal and cervical lymphadenopathy. He had noticed the first palpable nodule about 6 months ago. Upon physical examination, more palpable lymph nodes are found in the axillary and supraclavicular regions. laboratmy data show the se rum p roteins to be '.viU1in normal limits, whereas Lhe vVBC coum is 25,000/~-tl vvith many small abnormal lymphocytes. A cervical lymph node biopsy is shown in the image. The h istologic features are most consistent with which of the following hematologic disorders? ***(A) Burldtt lymphoma ***(B) Hodgkln lymphoma ***(C) Plasmacywma ***(D) Reactive follicular hyperplasia ***(E) Small lymphocytic lymphoma
39 The answer is E: Small lymphocytic lymphoma (SUs). Chronic lymphocytic leukemias (Cll5) as well as SLl.s are malignant B-cell proli ferations of small, mature-appearing lymphocytes and a variable number of large cells. A diagnosis of CLL is made if bone manow and peripheral blood are primatily involved. If the tumor cells give rise to lymphadenopathy or solid tumor masses , the term small lymphocy1ic lymphoma is used. Reactive follicular hyperplasia (choice D) is excluded in this case by the peripheral lymphocytosis and by Lhe Lack of reactive follicles in the lymph node biopsy. Hodgkin lymphoma (choice B) femures Reed-Stemberg cells . ***Diagnosis: Small lymphocytic lymphoma
40 A 50-year-old man presems with fever and diffuse lymphadenopaLhy. A lymph node biopsy reveals non-Hodgkin follicular lymphoma. lmmunohiswchemical staining of neoplastic lymphoid cells withJn tl1e nodular areas of the lymph node would be expected to stain positively for which of the following protein markers? ***(A) Abl ***(B) Bax ***(C) Bcl-2 ***(D) Myc ***(E) Rednoic acid recepwr
40 The answer is C: Bd·Z . Follicular lymphoma is a clonal lymphoid proliferation derived from germinal-center B cells. The most common cytogenetic translocation in follicu lar lymphoma is L(l4; 18)(q32 ;q2l), with Igli and bcl-2 as parmer genes. The bc\-2. protein, expressed in follicular lymphoma, is localized in the mitochondrial membrane and functions as an apoptosis inhibitor. Choices A, D, and E (abl, myc, and retinoic acld receptor) represent translocations i11 other disorders. Bax (choice B) is a pro-apoptotic protein. ***Diagnosis: follicular lymphoma
4t A 12-year-old girl presenLS \Vilh a feve r of 38°C (l0l°F) and a swollen lymph node of 6 days in duration. The CBC is normaL Biopsy of the swollen lymph node shows benign follicular hyperplasia (shown in the image). Tl1is pathologic finding is best imerpreted as which of the following? ***(A) Proliferation in B-cell areas ***(B) Prolife ralion of dendritic cells ***(C) Proliferation of plasma cells ***(D) Proliferation in T-cell areas ***(E) Proliferation of marginal-zone lymphocytes
41 The answer is A: Proliferation in B-cell areas. FolHcular h)~::Jerplasia refers to enlarged lymph follicles (principally in the cortex of the lymph node) , which consist of B lymphocytes. Reactive follicular hyperplasia of lymph nodes represents a response to infections, i nnanm1ation, or n.tmo rs. Hyperplasia of the secondary follicles, germinal centers, and plasmacytosis of the medullary cords indicate B-lymphocyte inununoreactivily. Hyperplasia of the deep conex or paracortex (imerfolhcular of diffuse hyperplasia) is charactenstic of T-lymphocyr.e immunoreactivity (choice D). ***Diagnosis: Follicular hyperplasia
42 A 4-year-old boy from Kenya presents with a 3-week history of a rapidly e>-rpanding jaw. A biopsy of an enlarged cervical lymph node is shown in the image. Histologic examination reveals numerous micotic figures and many macrophages containing nuclear and cytoplasmic debr is. The cells eArpress surface lgM and are positive for common B-cell antigens. Wl1ich of the following is lhe appropriate diagnosis? ***(A) Acute myelogenous leukemia ***(B) Burkiu lymphoma ***(C) Plasmacytoma ***(D) Reactive follicular hypeq)lasia ***(E) Small lymphocytic lymphoma
42 The answer is B: Burkitt lymphoma (BL) . BL, one of the most rapidly growing malignancies, is defined by a chromosomal translocation involving 8q24, which harbors the myc oncogene. Endemic BL is the most common childhood malignancy in Central Africa. Sporadic BL affects mainly children and young adults in the vVestern world , where ir accoums for 1% lO 2% of all lymphomas. Immunodeficiency-associated BL mainly occurs in HlV-i ruected persons. The cellular debri.c; of apoprotic mmor cells is cleared by macrophages, whose scattered appearance is termed "starry sky macroph age." Most patients present with extranodal tumors that emerge in a short period of time and respond w aggressive chemotherapy. Choices A, C, and E are not endemic in Africa, and choice D (reactive follicular hyperplasia) is a nonmalignant disorder. Djagnosis: Burkitt lymphoma
43 A 24-year-old woman presents wi th an earache of 4 days in duration. She also repons increased urine production, a skin rash, and bone pain on her scalp. Physical examination reveals otitis media, dermatitis, and exophthalmos. An X-ray of the scalp shows calvarial bone defects. A fine-needle aspirate displays numerous cosinophils. Which of the following is the most likely diagnosis? ***(A) Hodgkin lymphoma ***(B) Langerhans cell histiocytosis ******(C) Malignant melanoma ***(D) Metastauc breast carcinoma ***(E) Multiple myeloma
43 The answer is 8: Langerhans cell histiocytosis (L(H) . LCH refers to a spectrum of uncommon proliferative disorders of langerhans cells. The disease ranges from asymptomatic involvement of a single site, such as bone or lymph nodes, to an aggressive systemic disorder that involves multiple organs. There is clinical heterogeneity of LCH; eosinophilic granuloma (15% of aU cases) is a localized, usually self-limited disorder of older children and young adults; Hand-Schuller-Christian disease is a multifocal and typically indolent disorder, usually in children bet\veen 2 and 5 years of age; and LetLerer-Siwe disease (fewer than 10% of cases) is an acute, disseminated va riant of LCH in infants and children younger t.han 2 years of age . Organs involved by LCH include 1:he skin (seborrheic or eczematoid dermatitis), lymph nodes , spleen, liver, lungs, and bone numow. Otitis media is a common finding. Painful Lyric lesions of bone are conun on. Proptosis may complicate infiltration of Lhe orbit. The classic rriad of diabetes insipidus, propLDsi. s, and defects in membranous bones characterizes HandSchuller- Christian disease. Hodgkin lymphonca (choice A) often feam res eoslnophils bUL does not have this clinical presentation. ***Diagnosis: Langerhans cell histiocytosis
44 A +2.-year-old woman presents with an enlarged supraclavicu· lar lymph node. The patient is H lV positive and takes antiviral medications. A lymph node biopsy is shown ln the image. The tumor cells express B cell antigens and are posltive for Epstein-Ba rr virus (EBV) Which of the fo11owing is the most likely diagnosis? ***(A) Follicular lymphoma ***(B) Hodgkin lymphoma ***(C) Large B-celllyrnphoma ***(D) MALT lymphoma ***(E) Mamie cell lymphoma
44 The answer is C: large B·cell lymphoma. Most lymphomas in patients who have AIDS are high-grade B-cdllymphomas. These aggressive, but potemially curable neoplasms include Burkitt lymphoma and diffuse large-cell B-cell lymphoma. As in follicular lymphoma, bd2 gene rearrangements are often seen, suggesting a potential germinal cemer origin. Diffuse large-cell B-cell lymphomas associated with immunodeficiency are usually positive for EBV The other choices are not complications of HTV infectwn. ***Diagnosis: Large B-celllymphoma
45 A 55-year-old man presents wi th a 3-weekhisto ryofabdominal discomfon. Physical examination demonstrates splenomegaly bm no lymphadenopathy. A CBC shows pancytopenia. Examination of a peripheral blood smear reveals atyptcallymphoid cells that exhibit tartrate-resistant add phosphatase activity: Which of the following is the appropriate diagnosl<>? ***(A) Acute lymphocytic leukemia ***(B) Chronic lymphocytic leukemia ***(C) Chronic myelogenous leukemia ***(D) Hairy cell leukemia ***(E) Hodgkin disease
45 The answer is D: Hairy cell leukemia. Hairy cell leukemia is a clonal B-cell proliferation of small- to medium-sized lymphocytes that display abundanL cytoplasm and hairlike protmsions of the cell membrane. The disease involves primatily the monocyte/macrophage system of the bone marrow, spleen, and liver. Hairy cell leukemia is rare and affects mainly middle-aged to elderly persons, with a markedly increased male-to-female ratio of 5:1. Unlike the other choices, hairy cell leukemia e.xpresses tartraLe-resistam acid phosphatase. ***Diagnosis: Hairy cell leukemia
46 A 55-year-old man presenlS with recurrent epigac;tric pain. Upper Gl endoscopy and gastric biopsy reveal a neoplastic, lymphocytic infiltrate invading glandular tissue. Giemsa staining is positive for HelJcobacter pylon. Which of the following is the most likely diagnosis? ***(A) B·urkitt lymphoma ***(B) Follicular lymphoma ***(C) Hodgkin lymphoma ***(D) Mamle cell lymphoma ***(E) Marginal zone lymphoma
46 The answer is E: Marginal zone lymphoma. Extranodal marginal-zone B-ceJ l lymphomas that originate in mucosaassociated lymphoid tissue are referred to as MALT lympho· mas. MALT lymphomas are indolent, malignant lymphocyte proliferations that consist or small- to medium-sized lymphocytes with frequem monOC)'LOid features and variable admLxrun~ s of plasma cells. The malignant lymphocytes appear £O miginate from marginal-zone B cells. Most p1imary gastric lymphomas are MALT lymphomas. l'v1ALT lymphomas occur either in granular organs or along mucosal surfaces. They commonly arise in the context of a chronic inflammatory process or autoimmune disease. The other choices may mrely affect organs such as the stomach, but unlike MALT lymphoma, they are unrelated to H pylori infection ***Diagnosis: Marginal zone lymphoma, MALT lymphoma
47 A 58-year-old man presents with a 2-month history of erythematous, scaly plaques over his trunk and upper extremities. Biopsy of these lesions reveals an atypical lymphocytic infiltrate in the dermis, which extends into the overlying epidemlis. Immunohistochemical staining demonstrates positive staining for CD4. Which of the following is the most like!>' djagnosis? ***(A) Acme lymphoblasoc lymphoma ***(B) Chronic lymphoid leukemia ***(C) Extramedullary plasmacytoma ***(D) Hairy cell leukemia ***(E) Mycosis f-tmgoides
47 The answer is E: Mycosis fungoides. Mycosts fungoides represents a cutaneous T-cell lymphoma, composed of marure, posuhymic T-helper (CD4') lymphocytes. Mycosis fungoides displays lymphocytic infiltrates at the dermal-epidem1al junction and, in some cases, inrraepidermal accumuladons of tumor ceUs (Pautrier microabscesses) The other choices are not characteristically seen as skin lesions. ***Diagnosis: Mycosis fungoides
48 A 56-year-old man presenlS "With enlarged lymph nodes. Physical examination reveals mild hepatosplenomegaly. A lymph node biopsy shows small lymphocytes, as well as plasmacytoid lymphocytes containing Dutcher and Russell bodies. Immunohistochemical studies demonstrate cytoplasmic accumulation oflgM. \Vhat is the appropriate diagnosis? ***(A) Bmk:itt lymphoma ***(B) Hodgkin lymphoma ***(C) Richter syndrome ***(D) Sezary syndrome ***(E) Waldenstrom disease
48 The answer is E: Waldenstrom disease. Waldenstrom disease (lymphoplasmacyric lymphoma) is a neoplastic proliferation of small lymphocytes and a variable number of lgM-secrering plasma cells of the same malignam clone. Waldensrrom disease is not a variant of multiple myeloma, bm rather, is an indolem ma1ignam lymphoma that mainly affects the elderly. Eigh ty percem of patients with Waldenstrom macroglobulinemia presem with a monoclonallgM spike on serum electrophoresis {>3 gldL). Many of the clinical symptoms are associated with hyperviscosity of the blood. The other choices do not feature lgM-producing lymphocyres. ***Diagnosis: Waldenstrom macroglobulinemia
49 A 9-year-olcl girl develops widespread pinpoint ski.t1 hemorrhages. She recovered from a flu -like illness 1 week earlier. Laboratory findings reveal a platelet count of 20,000/f.lL bur no other abnormalities, Her bone marrow shows an increased number of megakmyocytes. The platelet count is nmmal after 2 momhs. Wh ich of the following is the approp1iate diagnosis? ***(A) Antiphospholipid antibody syndrome ***(B) Disseminated intravascular coagulation ***(C) Hemolytic-uremic syndrome ***(D) Idiopathic thrombocytopenic purpura ***(E) Thrombotic thrombocytopen ic purpura
49 The answer is D: Idiopathic thrombocytopenic purpura (ITP) . liP is a quantitative disorder of platelets caused by antibodies directed against platelet or megakaryocytic antigens. Similar to autoimmune hemolytic anemia, the etiology of liP is related to antibody-mediated immune desnuction of platelets or their precursors. In adults with acute liP, the platelet count is typically less than 20,000/J.l L. ln chronic adultlTP, the platelet count varies from a few thousand to 100 ,000/~L. The peripheral blood smear in lTP exhibits numerous large platelets, and the bone manow shows a compensatmy increase in megakaryocytes. Acute liP in children typically appears after a viral ilh1ess and presents with sudden onset of petechiae and purpura without other symptoms. Spontaneous recovery can be expected ln more than 80% o( cases within 6 months. Thrombocytopenia may be observed in the other choices but is usuaUy associated with other systemic signs and symptoms. ***Diagnosis: ldiopathlc Lhrombocytopenic purpura
SO A 25-year-old woman with a history of systemic lupus erythematosus presents with diffuse petechiae and fatigue. Physical examination demonstrates lymphadenopathy and splenomegaly Laboratory findings include normocytic anemia (hemoglobin= 6.1 g!dl ) and thrombocytopenia (30,000/J.ll). 'vVhich of the following is the most likely underlying mechanism in the development of thrombocytopenia in this patient? ***(A) Antibody-mediated platelet destmcrion ***(B) Clonal plasma cell circulating paraprotein ***(C) Decreased susceptibility to complemem-mecUared lysis ***(D) Ddecl in the platelet cytoskeleton ***(E) Increased activity of an enzyme in the glycolytic pathway
50 The answer is A: Antibody-mediated platelet destruction. ln common with idiopathic thrombocytopenic purpura and certain drug-induced thrombocywpentas, systemic lupus erythematosus is associated vvith increased platelet desrmction due to immune-mediated damage and removal of circulating platelets (antibody-mediated platelet desrmction). The other choices do not represem immune desuuction of platelets. ***Diagnosis: IdiopaLhic thrombocytopenic purpura, systemic lupus erythematosus
51 For the patient desc1ibed in Question 50, a periphe ral blood smear shows polychromasia with 10% re ticuloc:ytes. This patient most likely has which of the followmg hematOlogic diseases? ***(A) Anemia of chronic renal failure ***(B) Aplastic anemia ***(C) Hemolytic anemia ***(D) Iron deficiency anemia ***(E) Myelophthisic anemia
51 The answer is C: Hemolytic anemia. Reticulocytes are nonnucleated cells that represent the last stage before mature erythrocytes. The nucleus is extruded fTOm the orthochromatic erythroblast, leaving mitochondria and hemoglobinproducing polyribosomes in the re ticulocyte. After release from the bone marrow, the reticulocyte loses its capacity for aerobic metabolism and hemoglobin synthesis, and after l to 2 days, it becomes a mature erythrocyte. Hemolytic anemia<; are charactetized by a compensatoty increase in production and release of red cells by the bone marrow, manifested in the blood by polychromasia of red ceUs and an increased reticulocyte counr. Increased peripheral reliculocytes are not observed in the other choices. ***Diagnosis: Hemolytic anemia, idiopathic thrombocytopenic purpura
52 A 22-year-old man presents with a 6-day history of sore throat, fever, and general malaise. Physical examination reveals generalized lymphadenopathy; whlch is most prominent in the cervical lymph n odes. A CBC demonstrates atyp ical lymphocytes. The monospot test is positive. Two weeks later, the patient complains of interm]ltent pain and tingling m the tips of his fingers. A CBC discloses a mifd, macrocytic anemia. The peripheral blood smear is shown in the image. Which of the following is the most likely cause of anemia in this palient? ***(A) Autoamibodies directed agalnst the erythrocyte membrane ***(B) Clonal plasma cell dyscrasia •vi!.h circulating paraprotein ***(C) Decreased activity of an enzyme in the glycolytic path· way ***(D) De fect in the erythrocyte cytoskeleton ***(E) Increased susceptibility to complement-mediated hemolysis
52 The answer is A: Autoantibodies directed against the erythro· cyte membrane. The peripheral blood smear from this patiem shows clumped red cel1s caused by cold agglutinins (autoantibodies). Cold agglutinins are mostly IgM, are directed against the Ill antigen system, and act optimally at 4°C. Cold aggltllinins may be idiopathic or develop seconda1y to an underlying condition, most frequently infections (Epstein-Barr virus) or lymphoproliferative disorders. However, most autoimmune hemolytic anemias are mediated by lgG amibodies that exert Lheir maximal effect at body 1.emperarure (warm agglminins). Choices D and E are associated vvith hemolysis rather than clumping of red blood cells. ***Diagnosis: Autoimmune hemolytic anemia
53 A 25-year-old previously healthy woman develops spontaneous vaginal bleeding. The following day. she experiences a LOnic-clonic seizure. On physical examination, she is febrile and jaundiced and has widespread purpura. Laboratory findings include a hemoglobin of 5.3 gldl with 8% reticulocytes, a platelet count of 10,000/~LL, and a BUN of 48 mgldl. The pe1ipheral blood smear is shown in the image. Which of the following is the appropriate diagnosis? ***(A) Drug-lnduced thrombocytopenia ***(B) Henoch-Schonlein purpura ***(C) Idiopathic thrombocytopenic purpura ***(D) Thrombotic thrombocytopenic purpura ***(E) von Willebrand disease
53 The answer is D: Thrombotic thrombocytopenic purpura (TTP) . TIP occurs in acute or chronic form and presems with a pentad that includes fever, thrombocytopenia, nricroangiopathic hemolytic anemia, renal impairment, and neurologic symptoms. The morphologic hallmark of TTP is Lhe deposition throughout the body of PAS-positive hyaline microthrombi in arterioles and capill.aties, principally in the heart, brain, and kidneys. The microthrombi contain plaLelet aggregaLes, fibrin, and a few erythrocyLes and leukocytes. The peripheral blood smear displays microangiopathic hemolytic anemia. The pe1ipheral blood smear shows numerous schisLocytes TTP resembles hemolytic uremic syndrome, which occurs more often in chUdren than adull5. The other choices are not associated with microangiopathic changes (e.g., schistocytes). ***Diagnosis: Thrombotic thrombocytopenic purpura
54 A 45-year-old man suffers severe third-degree burns in an industrial accident. During his hospital stay, the patient develops anemia, thrombocytopenia, and widespread purpura. Blood oozes from venipuncture sites. Laboratory studies show that fibrin split products are elevated. The peripheral blood smear is shown in the image. What is the appropriate diagnosis? ***(A) AcamhocylOsis ***(B) Henoch-Schon1ein purpura ***(C) Idiopathic thrombocytopenic purpura ***(D) Microangiopathic hemolytic anemia ***(E) Paroxysmal nocturnal hemoglobinuria
54 The answer is D: Microangiopathic hemolytic anemia. MicroangiopaLhic hemolytic anemia resuli.S from abnormalides in the microcirculation that cause turbulent blood flow patterns. The classic examples of this type of hemolytic anemia are DlC (this case) and TTP; bot.h of which fearure generalJzed thrombosis of capillary vessels. Long-distance running or walking or prolonged vigorous exercise can cause repetitive trauma to red blood cells in the microcirculation leading to hemolysis. Alterations in blood now, as are encoumered in malignant h)1)ertension or vasculitis syndromes, may also produce mechanical fragmentation of erythrocytes. Microangiopal.hic features are not observed in the other choices. ***Diagnosis: Microangiopathic hemoly1.ic anemia
55 A 67-year-old woman with a prosthetic ao1t ic valve develops progressive anemia. Examination of a peripheral blood smear reveals reticulocytosis and schistocytes. Vvhal is the appropriate diagnosis? ***(A) Acamhocytosis ***(B) Henoch-Schonlein purpura ***(C) Idiopathic thrombocytopenic purpura ***(D) Macroangiopathic hemol ytic anemia ***(E) Mkroangiopathic hemolytic anemia
55 The answer is D: Macroangiopathic hemolytic anemia. Macroangiopathic hemolytic anemia most commonly results from direct erythrocyte trauma due ro an abnormal vascular surface (e.g., prosthetic hean valve, synthetic vascular graft). Anem. ia is m.ild to moderate and is accompanied by an approprime reticulocyte response. Blood smear examination reveals fngmemed red blood cells (schistocytes) and polychromasia. Although choice E (microangioparhic hemolytic anemia) resuhs b1 morphologically-similar red blood cells, iL reflects changes in small blood vessels. ***Diagnosis: Macroangiopathic hemolytic anemia
56 A 14-year-old boy presents \Vith acULe onset of right flank pain, which developed after he helped his father paint the ceiling of his bedroom. Physical examination demonstrates an area of ecchymosis in the right flank that is tender to palpation. The patient has a lifelong history of easy bruising. His brO[her shows the same tendency. The semm level of clotting factor Vlll is less than 2% of normal. Which of the following is the most likely underlying mechanism for bleedjng tendency in this patient? ***(A) Circulating antibodies directed against factor Vlll ***(B) Decreased hepatic synthesis of multiple coagulation facwrs ***(C) Deficiency ofvit.amin K ***(D) Genetic defect involving the factor Vlll gene ***(E) Nonlmmune peripheral consumption of coagulation proteins
56 The answer is D: Genetic defect involving the factor VIII gene. Hemophilia A is an X-linked recessive disorder of blood douing rhat results in spontaneous bleeding, particularly into jobm, muscles, and internal organs. Classic hemophilia results from mutations in the gene encoding factor Vll1 (hemophilia A). Hemophilia A is the most frequently encountered sex-linked inherited bleeding disorder (1 per 5.000 w 10,000 males). Choices C and E represent acquired disorders. ***Diagnosis: Hemophilia A
57 A 4-year-old boy develops severe bleeding into the knee joint. Laboratory studies show that serum levels of factor IX are reduced, but levels of factor VHI are normal. What is the appropriate diagnosis? ***(A) HemophHia A ***(B) Hemophilia B ***(C) Henoch-Schonlei11 purpura ***(D) Idiopathic thrombocytopenic purpura ***(E) von Willebrand disease
57 The answer is B: Hemophilia B. Hemophilia B is an X-linked recessive dL.:;ease caused by mutations in the gene encoding factor IX. lt accounts for only 10% of all cases of hemophilia One third of all cases represent new mutations. h is clinically indistinguishable from hemophiba A (facmr V1II deficiency) (choice A) ln both fonns of hemophilia, the panial thromboplastin time (PTT) is prolonged. Mixing of a patient's blood with that of a normal donor normalizes the PIT. ***Diagnosis: Hemophilia B
58 A 39-year~old man reports seeing red-colored urine in the morning. The CBC reveals anemia. low semm iron, and an elevated reticulocyte count Laboratory studies show increased lysis of erythrocytes when incubated with either sucrose or acidified semm. Which of the following is the appropriate diagnosis? ***(A) Anemia of chronic renal failure ***(B) Hereditary spherocytosis ***(C) Microangiopathic hemolytic anemia ***(D) Paroxysmal nocturnal hemoglobinuria ***(E) Vitamin B12 deficiency
58 The answer is D: Paroxysmal nocturnal hemoglobinuria (PNH) . Despite its name, the disorder is nocLUrnal in only a minority of cases. PNH is a clonal stem cell disorder characterized by episodic intravascular hemolytic anemia that is secondary to increased sensitivity of eryLhrocytes to complemem- mediated lysis. The underlying defect involves somatic mmation of the phosphatidylinositol glycan-class A (PIG-A) gene. PNH may develop as a primary disorder or evolve from preexisting cases of aplastic anemia. During hemolytic episodes, patiems develop nonnocytic or macrocytic anemia, accompanied by an appropliate reticulocyte response. The tmditional djagnostic tests for PN H, hemolysis in sucrose (sucrose hemolysis test) or acidified semm (Ham test), are now more e.aslly diagnosed by demonstrating loss of GPl-anchored proteins on blood cells b)' flow cyrofluoromeny. Choices B and C, which are hemolytic conditions, do not show increased lysis in the described laboratory studies. ***Diagnosis: Paroxysmal nocturnal hemoglobinuria
59 A 40-year-old man presen(S with at<L'...-i1l. MRI shows a cerebellar infarcL The platelet count is discovered to be 955 ,000/~L. The bone marrow biopsy reveals increased megaka.ryocytes with absent fibrosis (shown in Lhe image). Cytogenetic srudies are normaL Which of the following is the most likely diagnosis' ***(A) Chronic myelogenous leukemia ***(B) Essential thrombocythenua ***(C) Myelofibrosis with myeloid metaplasia ***(D) Polycythenua ve ra ***(E) Thrombotic lhrombocytopenic purpura
59 The answer is B: Essential thrombocythemia. Essential thrombocythemia is an uncommon neoplastic disorder of hematopoietic stem cells that is characterized by uncontrolled proliferation of megakaryocytes. A marked increase in circulating platelets is accompanied by recurrem episodes of thrombosis and hemorrhage. The VvHO criteria require a sustained platelet count above 600,000/).lL and prominent megakaryocytic proliferation in the bone marrow. Essemial thrombocythemia is believed to derive from the malignant rransfonnation of a hematopoietic stem cell with principal commitment to the megakaryocytic lineage. Increased megakaryocytes are a fearure of chronic myelogenous leukemia (choice A) but in the comext of multili.neage e.><pansion. Megakaryocytosis is also seen in myelofibrosis (choice C), but the marrow is fibrotic. ***Diagnosis: Essential thrombocythemia
60 An 18-year-old man is rushed to the emergency room in shock following a mmor vehicle accidenr. He is transh.1sed with 5 U of blood. Following the transfusion !he paliem complains of fever. nausea, vomiLing, and chest pain. Laboratory data show elevated indirect serum bilirubin, decreased serum hapwglohin, and a positive Coombs tesr. Which of tl1e followi ng is the most likely diagnosis? ***(A) Autoimmune hemolytic anem1a ***(B) Disseminated imravascular coagulation ***(C) Hemolytic transfusion reaction ***(D) Hemolytic uremic syndrome ***(E) Microangiopathic hemolysis
60 The answer is C: Hemolytic transfusion reaction. An immediate hemolytic transfusion reaction occurs when grossly incompatible blood is administered to patients with prefom1ed alloamibodies, usually because of clerical errors. Massive hemolysis of the transfused blood may be associated \~rith severe complications, including hypotension, renal failure, and even death. Choices B, D, and E are not characterized by a positive Coombs test. Choice A is not related LO blood transfusions. ***Diagnosis: Acute hemolytic transfusion reaction
61 A 60-year-old woman complains of weakness and h ematuria. Physical examination shows marked pallor, hepatosplenomegaly, and numerous ecchymoses of the upper and lower extremities. A CBC reveals a n on nocytic normochromic anenua, Lhrombocyropenia, neutropenia, and a marked leukocywsis, whlch is composed mainly o£ myeloblasts. The major clinical problems associated with this patient's condition are most directly related to which of the fo llowing? ***(A) Avascular necrosis of bone ***(B) Disseminated intravascular coagulation ***(C) Hypersplerusm ***(D) Microangiopathic hemolytic anemia ***(E) Suppression of h ematopoiesis
61 The answer is E: Suppression -of hematopoiesis. The presence of myeloblasts in the peripheral blood is indicative of acute myelogenous leukemia (AML). In AML, there is an accumuladon in the marrow of immarme myeloid cells that lack the potential for further differentiation and maturation, which leads w suppression of nonnal hematopoiesis. As a consequence, the major clinical problems associated with AML are granulocywpenia, thrombocytopenia, and anemia. Promyelocyl: ic leukemia causes dissemmated inn·avascular coagulation (choice B). ***Diagnosis: Acute myelogenous leukemia
62 A 69-year-old man is scheduled fo r surgery, but the procedure is canceled because or abnonnal findings in the preoperative blood work. A CBCshows leukocytosis (WBC = l24,000htL), consisting mainly of maturing neutrophils. Basophilia and eosinophilia are also observed. The platelercoumis820 ,000/~tL. A t(9;22)(q34;qll) translocation is documented. A bone marrow biopsy is shown in the image. Which of the following best characterizes the pathogenesis of the hematologic condition encountered in this patient? ***(A) Blocked differentiation of pre~B cells ***(B) Blocked differemiation of pre-T cells ***(C) Maturational anest oflymphoid progenitor stem cells ***(D) Transfom1ation of a lymphoplasmacytic ceU ***(E) Transformation of a pluripotent bone marrow s~em cell
&Z The answer is E: Tran:sformation of a pluripotent bone marrow stem cell. The patient exhibits the Philadelphia chromosome and clinicopathologic features of chronic myelogenous leukemia (CML) The neoplasuc cells in CML are derived from an abnonnal plutipotem bone marrow stem cell, which resuhs in prominent neutrophilic leukocytosis over the f·ull range of myeloid mamration. The other choices relate to cells in the lymphoid Lineage. ***Diagnosis: Chronic myelogenous leukemia
63 A 48-yea:r-old alcoholic man presents with a 6-day histmy of productive cough and fever. The temperarure is 38. 7°C (10.3°F), respirations are 32 per minute, and blood pressure is 125/85 mm Hg. The patiem:S cough worsens, and he begins e:~-q}ectorating large amounts of foul-smelling sputum. A chest X-ray shows a right upper and middle lobe infiltrate. A CBC demonsu·ares leukocytosis (WBC = 38,000h tL) , wilh 80% slightly immaune neutrophlls and toxic granulation. Laboratory studies reveal elevated leukocyte alkaline phosphatase. 'vVhicl1 of the following best describes this patients hemato· loglc condition? ***(A) Acute myelogenous leukemia ***(B) Chronic lymphocytic leukemia ***(C) Chronic myelogenous leukemia ***(D) Leukemoid reaction ***(E) Rkhter syndrome
63 The answer is D: Leukemoid reaction. Neutrophllia is an absolute neutrophil count above 7 ,000/~t L ln acute infections , neuuophilia may be so pronounced that it may be mistaken for leukemia, especially, chronic myelogenous leukemia (CML), in which case it is tem1ed a leukemoid n :action. Clues to the benign (or reactive) nature of a leukemoid reaction include the following: (l) the cells in the peripheral blood smear are more mature than myelocytes; (2) leukocyte alkaline phosphatase activity is high in benign conditions and low 111 patiems with CML; and (3) benign neutrophils often contain large blue cytoplasmic in clusions referred to as "Dahle bodies" or toxic granulation. The other choices are incorrect because the neutrophils in these disorders do not display Lhese morphologic fealures , and they are usually associated wilh other hematologic abnormali ties. ***Diagnosis: Leukemmd reaction
64 A 20-year-old carpenter with a wound infection on his ldt rJmmb presents with an enlarged and tender lymph node in tJ1e axilla. A lymph node biopsy shows follicular enlargement and hyperemia. The sinuses are filled with nemrophHs. Which of the following is the most likely diagnosis' ***(A) Castleman disease ***(B) Histiocytosis X ***(C) lmer£o1Hcular hyperplasia ***(D) Sinus histiocytosis ***(E) Suppurative lymphadenitis
64 The answer is E: Suppurative lymphadenitis. Acute suppurative lymphadenitis occurs in the lymph nodes that drain a site of acute bacterial infection. Suppurative lymph nodes enJarge rapidly because of edema and hyperemia and are tender due w distention of the capsule. Microscopically, infiltration of the lymph node sinuses and stroma by polymorphonuclear leukocytes and prominent folJicular hyperplasia are noted. Neutrophils are not a morphologic featu re of d1e other choices. ***Diagnosis: Suppurative lymphadenitis
65 A 56-year-old man \~lh .3-year hisloty of B-cell chronic lymphocytic leukemia complains of the recent onset of fever, weight loss, abdominal pain, and enlargemenL of lymph nodes. Physical examination reveals hepatosplenomegaly and generalized lymphadenopathy A lymph node biopsy shows a high-grade, large-ce1J lymphoma. This patient has which of tlle following diseases? ***(A) Acme lymphoblastic lymphoma ***(B) Chronic myelogenous leukemia ***(C) Hodgkln lymphoma ***(D) Leukemoid reaction ***(E) Richter syndrome
65 The answer is E: Richter syndrome. Five percent of patienlS with B-cell chronic lymphocytic leukemia (B-CLL) develop a large-cell lymphoma. Patients with this complication present with a rapid onset of fever, abdominal pain, and progressive lymphadenopathy and hepawsplenomegaly. Richter syndrome is aggressive and refractory to therapy, with a mean survival of 2 months. The mher choices are nm associated with B-CLL. ***Diagnosis: Richter syndrome, B-cell chronic lymphocytic leukemia
66 A 21-year-old woman complains of generalized weakness, blun-ed vision, and difficulty swallowing. Physical examination shows bil ateral ptosis and facial muscle weakness. A CT scan of lhe chest reveals a mass in the ame1ior mediastinum. The pariem's mass is surgically removed (shown in the image). A microscopic examination demonstrates epilhelial cells and nonnal lymphocytes. What is the most likely diagnosis? ***(A) Carcinoid mmor ***(B) Hodgkin lymphoma ***(C) Lymphocytic lymphoma ***(D) Squamous cell carcinoma ***(E) Thymoma
66 The answer i,s E: Thymoma. Thymoma 1s a neoplasm of thymic epithe1ial cells, without regard LO the number oflymphocytes. This tumor of adults is usua11y benign, bm malignant examples occur. Fifteen percent of patients with myasthen ia gravis have thymoma, as in this case. Conversely, one third to one half of patients with thymoma develop myasthenia gravis. lvlyasthenia gravis is not associated with the other choices, although mediastinal presentations occur. ***Diagnosis: Thymoma, myasthenia gra"is
67 A 4 7-year-old man with a history of a heart-lung transplant 3 yea rs ago complains of fever, malaise, and abdominal pain_ The patient has been taking cyclosporine for immunosuppression _ Physical examination reveals an abdominal mass. A CT-guided biopsy of the mass shows atypical lymphocytes that are positive for latent membrane proteins of Epstein-Barr virus (EBV)_ What is the most likely diagnosis? ***(A) AcUle suppurative lymphadenitis ***(B) Burkitt lymphoma ***(C) Graft-versus-host disease ***(D) Infectious mononucleosis ***(E) Posuransplant lymphoproliferative disorder
67 The answer is E: Posttransplant lymphoproliferative disorder (PTLD) . PTLD results from immunosuppression and is often associated wi.th EBV infection. ln most cases, the disease is an EBV-driven, monoclonal, lymphocyte proliferation with variable morphology. The incidence of PTLD parallels the extent of immunosuppression. In this cmmection, liver transplant recipients have a higher incidence of PTLD than do patients who receive kidney nansplams. Burkiu lymphoma (choice B) has been related to EBV infection but is not a compllcation of immunosupp ression. Infectious mononucleosis (choice D) does not present with an abdominal mass. ***Diagnosis: Posuransplant lymphoproliferative ctisorder
68 A 56-year-old man presents with a 2-week history of fatigue . The patients past medical history is significam for aonic and mitral valve replacement 5 months ago. A CBC shows moderate anemia with an increased reticulocyte coum_ vVhich of the following best explains the pathogenesis of anemia in this patient? ***(A) Complement-mediated hemolysis ***(B) Decreased blood Oow ***(C) Direct red cell trauma ***(D) Sludging of erythrocytes ***(E) Thrombin activation
68 The answer is C: Direct red cell trauma. Red cell fragmentation syndromes are disorders in which erythrocytes are subjected to mechanical dismpdon as they circulate in the blood (intravascular hemolysis). These disorders are classified as either macroangiopathic Oarge vessels), as in this case, or m.icroangiopathic (capillaries), according w the site of hemolysis. Mechanical fragmemation of red cells is primruily due either w al£eration of the endothelial surface of blood vessels or disturbances in blood flow patterns that lead to turbulence and increased shear stress. Macroangiopathic hemolytic anemia most conunonly resulLS from direct red cell trauma due ro an abnormal vascular surface {e.g., prosthetic hean valve). The other choices are not an expected complication of a prosthetic valve. ***Diagnosis: Macroangiopathic hemolysis
69 A 46-year-old man is rushed w the hospital after suffeti ng massive trauma in an automoblle accident. Two days later the patient suffers a clonic-tonic seizure. Blood cultures are posidve for Gram-negative bacteJia, and the patient is starred on intravenous amibiotics. Laborawry smdies show prolonged prothrombin time (PT) and partial thromboplastin time (PTT), low levels of fibrinogen, a positive o-dimer test, and thrombocytopenia. The patient develops renal failure and expires. A secdon of the kidney at autopsy is stained with phosphomngstic acid hematoxylin (shown in the image). The dark purple objects ·within the glomeruli are besL identified as which of the following? ***(A) Arteriovenous malfom1auons ***(B) Fat emboli ***(C) Fibtin thrombi ***(D) Psammoma bodies ***(E) Vascular calcifications
69 The answer is C: Fibrin thrombi. Disseminated inu·avascular coagulation (DlC) refers w widespread ischemic changes secondary to microvascular fibrin thrombi. which are accompanied by the consumption of plateleLS and coagulation facto rs and a hemorrhagic diathesis. DIC typically occurs as a complication of massive trauma, septicemia, and obstetric emergencies. ll is also associated with metastatic cancer, hematopoietic malignancies, cardiovascular and liver diseases, and numerous other conditions. The other choices are not directly associated with DIC. ***Diagnosis: Disseminated intravascular coagulation
1 A 14-year-old boy presents \vith 3 months of lethargy, headaches, and muscle weakness. His parents note Lhat he d1inks water excessively. His vital signs are normal. A H-hour urine collection shows polyuria. The fasting blood sugar is normal. An X-ray film of the brain reveals suprasellar calcification. An autopsy specimen of a similar case is shown in the image. Whicl1 of the following neoplasms is the most likely cause of polyuria in this patient? ***(A) Craniopharyngioma ***(B) Glioblastoma multiforme ***(C) Pheochromocytoma ***(D) Pimitary adenoma ***(E) Retinoblastoma
1 The answer is A: Craniopharyngioma. Cemral diabetes .....__ insipidus is characterized by an inability w concemrate the urine, with consequent chrome water diuresis, thirst, and polydipsia. The disease reflects a deficiency of antidiuretic hormone (ADH, arginine vasopressin), which is secreted by the posterior pimitary lmder the influence of the hypoLhalamus. One fourth of cases of cemral diabetes insipidus are associated with brain tumors, particularly craniopharyngioma. This mmor arises above the sella turcica from remnants of Rathke pouch and invades and compresses adjacem tissues. Destruction of the posterior lobe o[ the pituitary. the source of ADH, causes pol>ruria and diabNes insipidus. This coronal section of the brain shows a large cystic rumor mass replacing the midline structures in the region of the hypothalamus. The other choices do not occur in this location and are rarely associated with cemral diabetes insipidus. ***Diagnosis: Craniopha1yngioma, diabetes insipidus
2 A 60-year-old man with small cell carcinoma of the lung is rushed to the emergency room in a coma after suffering a clonic-tonic seizure. The patiems tempemrure is37°C (98.6°F), blood pressure 100/50 nun Hg, and pulse 88 per minute. Laborato ry s tudies show a serum sodium of 103 mmoVL, normal serum levels of BUN and creatinine, and a dilute bm otherwise normal UJine. A CT scan of the head is normaL Which of the following ts the most likely cause of seizures in this padem? ***(A) Central diabetes insipidus ***(B) Diabetes mellims ***(C) Renal metastases ***(D) Sheehan syndrome ***(E) Syndrome of inappropriate ADH secretion
2 The answer i.s E: Syndrome of inappropriate ADH sec-retion. The inappropriate production of ADH by a mmor may cause a paraneoplastic syndrome that features wmer retention, which is manifested as water intoxication ·with hyponau-em.ia. resulting in altered mental status, seizures, coma, and sometimes death. This syndrome is most often related to small cen carcinoma of the lung. lt is also reported with carcinomas of the prostate, gastrointestinal tract, and pancreas and with thymomas, lymphomas, and Hodgkin disease. Diabetes mellitus (choice B) may cause polytuia but features glucosuria and does not cause severe hyponatremia. Sheehan syndrome (choice D) refers to hypopituitarism caused by infarcdon of the anterior pituitary ***Diagnosis: ~yndrome of inappropriate ADH secretion
3 A 60-year-old woman with small cell carcinoma of the lung notes rounding of her face , upper truncal obesity, and muscle weakness. Physical examination reveals thin. wrinkled skin, abdominal striae, and multiple purpmic skin lesions. The patients blood pressure is 175/9'5 mm Hg. Laborawty studies wiJl likely show elevated serum levels of which of the following hormones? ***(A) Aldoste rone ***(B) Corticotropin ***(C) Epineplui ne ***(D) Pro 1 actin ***(E) Thyrmropin
3 The answer i.s 8: Corticotropin. This patient most likely has a paraneoplasric condidon associated with a conicotropin (ACTH)-producing small cell carcinoma of the lung. The major clinkal manifestations ol' hyperconisolism in patients with Cushing syndrome include emotional disturbances, moon facies, osteoporosis, hypenension, upper truncal obesity (buffalo hump), amenorrhea, muscle weakness, thin wrinkled skin, purpura, glucose intolerance, poor wound healing and skin ulcers. None or the other hormones are associated ''Vith Cushing syndrome. ***Diagnosis: Cushing syndrome, paraneoplastic syndrome
4 A 2.1-year-old woman e.xperiences abruptio placentae with ....__ severe bleeding during the delivery of a lerm fetus. five months later, she presents with profound lethargy, pallor, muscle weakness, failure of lactation, and amenorrhea. Vvhich of the following pathologic findings is expected in this patient? ***(A) Atrophy of lhe endocrine pancreas ***(B) Autoimmune destruction of !he adrenal conex ***(C) lnfarction of the pituitaly ***(D) Pituitary prolactinoma ***(E) Polycystic ovaJies
._4 The answer is C: Infarction of the pituitary. This patient expe- 1ienced postpartum ischemic necrosis of Lhe pituitny (Sll eehan syndrome). This commonly, but not exclusively, occurs after severe hypotension induced by postparmm hemonhage. The pituitary is particularly susceptible at this time because its enlargement during pregnancy renders it vulnerable to a reduction in blood flow Major clinical manifestations of panhypopituitatism include pallor (decreased MSH), h ypoth yroidism (decreased TSH) , failure of lactation (decreased prolactin), adrenal insuffic iency, and ovatian failure (decreased FSH and LH). None of the other choices lead to this constellation of hormone deficiencies. ***Diagnosis: Sheehan syndrome
...._5_ A 25-year-old man presents \Vith 3 months of polyuria and increased thirsL The patient suffered trauma to Lhe base of the skull in a motorcycle accident 4 months ago. A 24-hour urine collection shows polyuria bm no evidence of hemamtia, glucosuria, or proteinuria The pathogenesis of polymia ill this patient is most likely cau..c;ed by a lesion in which of the following areas of the brain? ***(A) Aden ohypophysis ***(B) Bram srem ***(C) Mammillothalamic LracL ***(D) Neurohypophysis ***(E) Subthalamic fasciculus
5 The an-swer is D: Neurohypophysis. Central diabetes insipidus may be caused by head trauma that damages the posterior lobe of the pituitary (neurohypophysis) and imerrupts t.he secredon of ADH. One fourth of cases of central diabetes insipidus are associated with brain tumors, particularly craniopharyn· gioma. The other choices do not involve areas of the b rain that secrete ADH. The anterior pituitary (adenohypophysis, choice A) does not secrete ADH. ***Diagnosis: Diabetes insipidus
6 A 30-year-old woman complains of headache, visual disturbances, deepening of the voice, and generalized weakness. She reports amenorrhea for the past year and states that she recently required a larger shoe size. Laborawry studies show impaired glucose tolerance. What other procedure would be useful for establishing your diagnosis? ***(A) CBC with differential count ***(B) CT scan of the abdomen ***(C) MRJ of the sella rurcica ***(D) Test [or serum 21-hydroxylase ***(E) Test for serum androstenedione
6 The answer is C: MRI of the sella turcica. This patient has clinical features of a pituitaJy adenoma with acromegaly. Pim· itary adenomas range from small lesions that do not enlarge r.he gland 1.0 expansive tumors that erode the sella turcica and impinge on adjacent cranial suucrures. Adenomas smaller than 10 mm in diameter are referred 1.0 as microadenomas, and larger ones are r.em1ed macroadenomas. Microadenomas do not produce symptoms unless they secrete hormones. The mass effects of pituitary macroadenomas include impingement on the optic chiasm, often with bitemporal hemianopsia and loss of ccmral \'i.sion, oculomotor palsies when the rumor invades the cavernous sinuses, and severe headaches. Large adenomas may invade the hypothalamus and lead to loss of temperature regutadon, hyperphagia, and hormonal syndromes because they interfere with the nmmal hypothalamic input to the pirujtaiy Symptoms of acromegaly include characteristic facial changes, goiter, barre I chest, abnmmal glucose tolerance, male sexual dysfunction, menstrual disorders in women, degenerative arthritis, peripheral n europathy, and thickened skin. The OLhe r choices represent tests that will not provide the diagnosis or pituitary tumor. ***Diagnosis: Acromegaly, pituitruy adenoma
7 A 35-year-old woman 'Nith a histOJy of schizophrenia com- .__ plains of headaches, visual disturbances, and irregular menses for 9 months. On physical examination the breasts are firm and tender. MRI shows enlargement of the anterior pituitary (arrm.v)_ Which of the following is the most likely cause of pituitary enlargement in this patient? ***(A) Corticotrope adenoma ***(B) Gonadotrope adenoma ***(C) Lactou·ope adenoma ***(D) NuU cell adenoma ***(E) Somatorrope adenoma
7 The answer is C: Lactotrope adenoma. Pimitary adenomas are ,_ benign neoplasms of the anterior lobe of the pituitary and arc often associated with excess secretion of pituitary hormones and evidence of corresponding endocrine hyperfunction. They occur in both sexes at almost any age bm are more common in men between the ages of 20 to 50 years. HyperprolacLincmia is the most con1111on endocrinopathy associated with pituitmy adenoma. Almost half of all piruitary microadenomas synthesize prolactin. Functional lactotropc adenomas lead to amenorrhea, galactorrhea, and infertility. None of the mher piLUita ry rumors produce prolactin or lead to inappropriate lactation. ***Diagnosis: Galactorrhea, pituitary adenoma
8 A 55-year-old man complains of severe muscle weakness and drooping eyelids. He states that his symptoms worsen with repetitive movements but then resolve after a shon resL A chest X-ray reveals an ante1ior mediastinal mass. A biopsy of this mass would most likely reveal which of the following pathologic changes? ***(A) B-celllymphoma ***(B) Hodgkin lymphoma ***(C) Metastatic carcinoma ***(D) Small cell carcinoma of the lung ***(E) Thymic hyperplasia
8 The an.swer i.s E: Thymic hyperplasia. Myasthenia gravis is an acquired auroimmune disease characterized by abnonna] muscular raogability and is caused by circulating antibodies w the acetylcholine recepwr at the myoneural junction. ln two thirds of patients, thymic hyperplasia is closely associated wilh myasthenia gravis, and 15% have thymoma. Conversely, one third to one half of patients vvith thymoma develop myasthenia gravis. Thymectomy is often an effective treatment for these patients. None of the other choices are associated with myasthenia gravis, although paraneoplastic muscle weakness can occur. ***Diagnosis: Myasthenia gravis
9 A 45-year-old woman complains of lingUng in her hands and feet, 24 hours after surgery lO remove a thyroid follicular carcinoma. Her symptoms rapidly progress to severe muscle cramps, laryngeal snidor, and convulsions. Which of the following laborawry findings would be expected in this patient prior to treatmem? ***(A) Decreased serum calcium and decreased PTH ***(B) Decreased serum calcium and increased PTH ***(C) lncreased serum calcium and decreased PTH ***(D) Increased serum calcium and increased PTH ***(E) Normal serum calcium and decreased PTH
9 The answer is A: Decreased serum calcium and decreased PTH. The most common cause of hypoparathyroidism is surgical resection of the parathyroids as a complication of thyroidecwmy. Of patien[S undergoing surgery for pnmary hyperparathyroidism, 1% develop Irreversible hypoparathyroidism. The symptoms of h)1)0parathyroiclism relate to hypocalcemia, which causes increased neuromuscular excitability. This is reflected in symptoms that range from mild tingling in the hands and feet to severe muscle cramps, laryngeal stridor, and convulsions. Neuropsychianic manifestations include depression, paranoia, and psychoses. lncreased PTH in Lhe setting of parathyroid adenoma or a paraneoplastic syndrome is associated with hypercalcemia (choice D). ***Diagnosis: Hypoparathyroidism
10 A 55-year-old woman presents wi th a large anterior neck mass (patient shown in the image). She also complains of dysphagia and hoarseness. Physical examination reveals inspiratmy suidor. Laborawry evaluation of this patiem would most likely demonsnate which of the following? ***(A) Emhyroidism ***(B) Hypercalcemia ***(C) Hyperthyroidism ***(D) Hypocalcemia ***(E) Hypothyroidism
10 The answer is A: Euthyroidism. Nomoxic goiter (also termed simple, colloid, or mulLinodular goiter) refers to an enlargement of (he thyroid that is not associated with functional, inflammatory, or neoplastic alterations. Patients with nontoxic goiter are neither hyperthyroid nor hypothyroid and do not suffer from any form o[ thyroiditis. Large goiters may cause dysphagia or inspiratory stridor by compressing the esophagus or trachea. Hoarseness may result from recurrem laryngeal nerve compression. The d isease is far more common in women than in men (8:1). A number of dmgs and naturally occuning chemicals in foods are goitrogenic due w their suppression of thyroid ho rmone synthesis. The most commonly used goitrogen ic dmg is lithimn, which is used in the managemem of manic-depressive states. Endemic goiter refers w the goiu·ous hypothyroidism of dietary iodine deficiency in locales with a lligh prevalence of the disease. Without sufficiem iodine, thyroid hormones are not produced and lhe pituitary continuously secretes thyroid stimulating honnone. ***Diagnosis: Nontoxic goiter
11 A 48-year-old man presents ·with recurrent headaches and anhrilic pain in his knees of 9 months in dura lion. He notes that his hat size has recemly increased. He also states that he suffers from erectile dysfunction. His past medical history is significant for kidney stones 2 years ago . Physical examination reveals a blood pressure of 170/100 nun Hg. The patient is observed w have coarse facial features and a goiter Urinalysis reveals glucosuria and hypercalciuria. vVhich of the follovving is the most likely explanation for this patient's clinical presentation? ***(A) Excess growth hormone secretion ***(B) Excess parathyroid hormone secretion ***(C) Excess prolactin secretion ***(D) Hypersecretion ofbone morphogenetic protein ***(E) Insufficiem growth hormone production
11 The answer is A: Excess growth hormone secretion. Acromegaly refers to enlargement of the terminal portions of lhe extremities and the jaw and is caused by growth horrnone-secretil1g tumors of the pituita1y (somatotrope adenomas). The pituitary n1mor may lead lO headaches and may compress the optic chiasm. These patients also have other honnonal problems, such as menstrual inegularities in women and diabetes mellitus. The excess secretion of growlh hormone produces dramatic bodily changes. A somatotrope adenoma that arises in a child or adolescent before the epiphyses dose results in gigantism. By contrast, after lhe epiphyses of lhe long bones have fused and adult height has been achieved, the same tumor produces enlargement of the skull (acromegaly) Most acromegalies suffer from neurologic and musculoskeletal symptoms. One third have hypertension. Diabetes occurs mas many as 20% of these patients, and hypercalciuria and renal stones are present in anmher fi.f1.h of the patients. None of the other choices p resem this wide variety of signs and symptoms. ***Diagnosis: Acromegaly
12Why does the patient desCJibed in Question 11 suffer from erectile dysfunction? ***(A) Excess cortisol secretion b)' the adrenal cortex ***(B) Excess growth hormone secretion by the pituitary ***(C) Excess prolactin secretion by the pituitary ***(D) Insufficient cortisol secretion by the adrenal cortex ***(E) lnsuffidem FSH secretion by the pituitary
12 The answer is C: Excess prolactin secretion by the pituitary. In mixed somatOLroph-lactotroph adenomas the two cells types elaborate growth hormone and prolactin, respectively. [n half of patients with acromegaly, hyperprolactinemia is severe enough w be symptomatic, causing loss of libido and erectile dysfunction in men. Functionallactorroph m.icroadenomas are successfully treated with dopamine agonLsts (bromocliptine) to inhib1t prolactin secretion. lnsufficiem FSH secretion by the piruitary (choice E) would inhibit spennarogenesis, but would not affec t testosterone production by Leydig cells. The otJ1er choices do not halt the production of androgens that are involved in male se}.'Ual function. ***Diagnosis: Acromegaly
13 Physical examination of a neona te shows peculiar genitalia (show·n in the image). Cytogenetic studies reveal a 46, XX ka1yorype. Laboratmy studies will most lli<ely reveal a deficiency o[ which of Lhe following? ***(A) Androstenedione ***(B) Corticotropin ***(C) 21-Hydrm.-. ylase ***(D) Progesterone ***(E) Prolactin
13 The answer is C: 21-Hydroxylase. Congenital deficiency of 21-hydroxylase results in adrenogenital syndrome, which is associated with virilization of external geni talia in female infants (pseudohennaphroditism). The photograph shows a markedly virilized and hypernophic clitoris and partial fusion of labioscrotal folds in a genetic female. The extem of the biochemkal defect is highJy variable, ranging from mild to complete deficiency. Male infants show normal external genitalia. Levels o£ adrenal androgens (choice A) and progesterone (choice D) increase in this disorder. ***Diagnosis: Con genital ad renal h yperplasia, adrenogenital syndrome
14 The infam described in QuesLion 13 is shown to have an autosomal recessive genetic disorder. The infam is expected to manifest which of rh e followi ng developmental anomalies' ***(A) Adrenal hyperplasia ***(B) Bladder diverticulum ***(C) Cystic renal dysplasia ***(D) Empty sella turcica ***(E) Polycystic ovaries
14 The answer is A: Adrenal hyperplasia. Congeni tal adrenal hn>erplasia (CAH) is a syndrome that results from a munber of autosomaJ recessive, enzymatic defects m the biosynthesis of cortisol from cholesteroL CAH is the most common cause of ambiguous genitalia in newborn girls. Most cases of CAH (>90%) represent an inborn deficiency of 21-hydroxylase, more specLfically termed P450C2L Deficiency in the synthesis of corticosteroids in the adrenal conex results in the continuous secretion of ACTH by the antetior pituitary, resulting in congenital adrenal hyperplasia. The adrenal glands are greatly enlarged, weighing as much as 30 g (normal= 4 g). Polycystic ovary syndrome (choice E) occurs in aduiL women. ***Diagnosis: Con genital ad renal hyperplasia, adrenogenital syndrome
15 A 7 -week-old infant develops seve re dehydration and hypotension and expires. The lddneys and adrenal glands at amopsy are shown in the image. Hypovolemic shock in this infant was most likely caused by inadequate symhesis of which o [ the following h ormones? ***(A) Aldoslerone ***(B) Angiotensin ***(C) Antidiuretic honnone ***(D) Atrial nauiuredc factor ***(E) Renin
15 The answer is A: Aldosterone. The autopsy specimen shows massive bilateral adrenal enlargemem charactetistic of congeni· tal adrenal hyperplasia (CAH). Congenita12l-hydroxylase defi· ciencies may be associated with impaired aldosterone synthesis (salt-wasring CAH). Hypoaldosteronism develops within the fi rst Fe''' weeks of hfe ln two Lhirds of newborns with congenital adrenal h yperplasia, wbo suffer dehydration and hypotension. Laboratmy studies in these neonates show hyponatremia, hyperkalemia, and increased renin sec retion (choice E). ***Diagnosis: Congenital21-hydroxylase deficiency
16 A 6-momh-old girl with Wiskott-Aldrich syndrome is brought to the emergency room shortly after spiking a fever of 38.J<'C (103°F). The infant has a hiswry of chronic respiratory infecdons, gastrointestinal infections, petechiae, and eczema. This infant likely has which of the following associated birth defects? ***(A) Adrenal cortical hyperplasia ***(B) Cystic renal dysplasia ***(C) Hypoplasia of thymus ***(D) Meckel diverticulum ***(E) Pituitary hypoplasia
16 The answer is C: Hypoplasia of thymus. Wiskott-Aldrich syndrome is a sex-linked , hereditary disease in whlch severe immunodeficiency is associated with a hypoplastic thymus, eczema, and thrombocytopenia. Alterations in the U1YJTIUS vary from complete absence (agenesis) or severe hypoplasia w a situation in which the thymus is small but exhibits a normal architecture. Some small glands exhibit thymic dysplasia, characterlzed by an absence of thyn10cytes, few. if any, Hassall corpuscles, and only epitheli al components. The Olher choices do not involve the immune S)'Stem. ***Diagnosis: vViskott-Aldtich synd rome
17 A female neonate with DiGeorge syndrome develops severe muscle cramps and convulsions soon after birth. Which of the following is the cause of convulsions in this neonate? ***(A) Acute hemonhagic adrenalitis ***(B) Hypocalcemia ***(C) Hypoglycemia ***(D) Hypokalemia ***(E) H)1)onatremia
17 The answer is B: Hypocalcemia. OiGeorge syndrome is caused by a failure in the devclopmem of the Lhird and fOLmh branchial pouches, resulting in agenesis or hypoplasia of the thymus and parathyroid glands, congenital hcan defects , dysmorphic facies, and a vanety of other congenital anomalies. As a result of parathyroid agenesis, patients 'Nith DiGeorge syndrome exhibit hypocalcemia, which manifests as increased neuromuscular excitability. Symptoms range from mild tingling in the hands and [cellO severe muscle cramps and convulsions. DiGeorge syndrome does not feature any of tbe other choices. ***Diagnosis: DiGeorge syndrome
18 ln addition to parathyroid agenesis, the neonate described in Question 17 would be expected to have which of the following conditions? ***(A) Adrenal hyperplasia ***(B) Congenital goiter ***(C) Extralobar sequestration of the lung ***(D) Immune deficiency ***(E) Pituitary hnoplasia
18 The answer is D: Immune deficiency. Thymic aplasia in patients with OiGeorge syndrome results in a congen:ilal immune deficiency syndrome characterized by the loss of T cells. As a result, patients exhibit a deficiency of cell-mediated immunity, with a particular susceptibility to Candida sp. infections. None of the other choices are associated with DiGeorge syndrome. ***Diagnosis: DiGeorge syndrome
19 A 15-year-old boy with Albright herediLa ry osteodysn·ophy is rushed to the emergency room with severe muscle cramps and convulsions. The child has a hiswry of mental retardation. Laborawry smdies reveal hypocalcemia and elevated blood levels of PTI-L Which of the following distinguishes this patients endocrinopathy from hypoparathyroidism seen in DiGeorge syndrome? ***(A) Abnormalities in cardiac conduction and contractility ***(B) Accelerated degradation of PTH ***(C) Decreased neuromuscular excitability ***(D) End-organ unresponsiveness to PTH ***(E) Increased neuromuscular excitability
19 The answer is D: End ·organ unresponsiveness to PTH. Pseudohypoparathyroidism designates a group of hereditary conditions in which hypocalcemia is caused by target organ ill5cnsitivity to PTH. The defect in these padems has been traced to mutations in a gene whose product couples hom1one recepto rs lO the stimulation of adcnylyl cyclase Consequently, in the renal tubular epithelium, the produCLion of cAMP in response to PTH is impaired, and inadequate resorption of calcium from the glomerular filtrate ensues. These patients demonstrate a characteristic phenotype (Albright hereditary osteodystrophy), including shan stature, obesity, mental retardation, subcutaneous calcification, and a number of congenital anomalies of bone. Abnormalities in cardiac conduction (choice A) and increased neuromuscular excitability (choice E) are related to hypocalcemia. ***Diagnosis: Pseudohypoparathyroidism, Albright hereditary osteodystrophy
20 A 50-year-old woman presents with acute right flank pain of 72 hours in duration. Physical examination is unremarkable. Her temperamre is 37°C (98.6°F), blood pressure 140/85 mm Hg, and pulse 85 per minute. A CBC is normaL Urinalysis reveals hematuria and urine cultures are negative. lmaging studies show stones in the 1i.ght renal pelvis and ureter. This patiems condition may be associated with which of the following endocrine disorders? ***(A) Conn syndrome ***(B) Cushing syndrome ***(C) Hyperparathyroidism ***(D) Hyperthyroidism ***(E) Hypoparathyroidism
20 The answer is ( : Hyperparathyroidism. P1imary hyperparathyroidism refers to the syndrome caused by excessive secretion of PTH by a parathyroid adenoma, primary hyperplasia of all parathyroids, or parathyroid carcinoma. Excessive PTH leads to excessive loss of calcium from the bones and enhanced calcium resorption by the renal rubules. The clinical manifestations of primary hyperparathyrmdism range from asymptomatic hypercalcemia detected on routine blood analysis to florid systemic renal and skeletal disease. Hypercalcemia and hypophosphatemia lead to an increased risk of urolithiasis. Hyperparathyroidism is often accompanied by mental changes, including depression, emotional liability, poor mentation, and memmy defects. The other choices are not assoctated with hypercalcemia or the formation of renal calcuU . ***Diagnosis: Urolithiasis, hyperparathyroidism
21 Laboratory evaluation of Lhe patient described in Question 20 shows markedly elevated serum levels of calcium and PTH. A CT scan of the neck reveals a 3-cm mass on the posterior surface of rhe right lobe of d1e thyroid gland . External and cross-sectional views of the surgical specimen are shown in the image. Microscopic examination of rl1is neck mass would most likely reveal a benign neoplasm derived from which of the follmving cells? ***(A) Chief ***(B) Clear ***(C) Follicular ***(D) Parafollicu]ar ***(E) Oxyphil
21 The answer is A: Chief. Parathyroid adenoma is the cause of 85% of all cases of primary hyperparathyroidism. The tumor arises sporadically or in the contexl or multiple endocrine neoplasia (MEN-1 and MEN-2A, 20% of cases). ln a small minor· ity of cases of sporadic adenoma, genetic analysis has identi fied reanangemen t and overexpression of the cyclin D protooncogene. On gross examination, a parathyroid adenoma appears as a circumscribed, reddish brown, solitary mass, measuring 1 to 3 em in diameler. Microscopically, these tumors show sheets of neoplastic chief cells in a rich capillary n etwork. A rim of nonnal parathyroid tissue is usually evidem outside the tumor capsule and distinguishes adenoma from paraLhyroid hyperplasia. None of the other cells secre tes PTH. ***Diagnosis: Hyperparathyroidism, parathyroid adenoma
22 A 72-year-old woman wi1h a long histmy of ruabetes rype 2 presents '>Vith abdominal pam. Physical examination reveals neuromuscular weakness and hypertension. Laboratory smdies show markedly elevated levels of serum calcium and PTH. A surgical exploration of the patients neck demonstrates four symmenically enlarged parathyroid glands. This patients endocrinopathy may be caused by which of the fol lowing underlying disorders? ***(A) Adrenal insufficiency ***(B) Chronic liver disease ***(C) Insulin deficiency ***(D) Pituitary adenoma ***(E) Renal insufficiency
22 The answer is E: Renal insufficiency. Hyperparathyroidism can be primary as a result of autonomous proliferation of chief cells or may be secondary, in which case il is a com· pensatory mechanism. Secondary paralhyroid hyperplasia is encountered principally in patients with chronic renal fail ure, although the disorder also occurs in association with vitan~ n D denciency, intestinal malabsorption, Fanconi syndrome, and renal tubular acidosis. Diabetic glomemlosclerosis is a major cause of renal insufficiency. Chronic hypocalcemia due to renal retemion of phosphate leads to compensarory hypersecretion of PTH. As a result, secondary hyperplasia of aU parathyroid glands occurs. Enlarged parathyroid glands occasionally become independently hyperfunctional. None of the other choices are associated with l1ypocalcemia or secondary parathyroid hyper plasia. ***Diagnosis: Hyperparathyroidism
23 A 20-year-old woman with Hirschsprung disease presents with acute leg pain. The patiem had a glioma resected 3 years ago. An X-ray film of the leg reveals a fracture of the left tibia. Laborar.ory sLUdies show elevated serum levels of calcium and PTH. A CT scan of the patient's neck demonstrates a solitary parathyroid mass. Two years later, the patiem presents with hyven ension, and a CT scan of the abdomen displays a 4-cm mass in the right adrenaL Genetic sturues conducted on this patient would likely reveal germline mutations in which of the following prowoncogenes7 ***(A) BRCAl ***(B) Rb ***(C) RET ***(D) VHL ***(E) Vfl-1
23 The answer is C: REl Patiems with muldple endocrine neoplasia (MEN) syndromes types 1 and 2 have gene mutalions lhat make them susceptible to neoplasia or hyperplasia in multiple organs. Patients with MEN-2A (Sipple syndrome) have C-ecil-derived medullary thyroid carcinoma and chromaffin cell-derived pheochromocytoma. One third of patients exhibit hyperparathyroidism as a result of parathyroid hyper· plasia or adenoma. Hirschsprung disease (congenital megacolon) and a variety of neura] crest tumors (e.g., ghoma) are also seen in patients •Nith MEN-2A. Mutations of the RET protooncogene, a transmembri}ne receptor of the Lyrosine kinase family, are responsible for MEN-2 syndromes. MEN-1 (Wenner syndrome) comprises adenoma of the plluit.ary, parathyroid hyperplasia or adenoma, and islet cell tumors of the pancreas (insulinoma and gasrrinoma). MEN-1 is caused by mutation of the MENl tumor suppressor gene. The mher choices are not associated with endocrine syndromes. ***Diagnosis: Parathyroid adenoma, multiple endocrine neoplasia
24 A 40-year-old woman with a histoty of hyperparathyroidism presents with a 2-month history of bunting epigastric pain. The pain can be relieved with antacids or food. The patient also repons a recent history of ta n y stools. She denies taking aspirin or NSAIDs. Laboratmy smdies show a microcytic, hypochromic anemia (hemoglobin = 8.5 gldL). Gastroscopy reveals a bleeding mucosal defecL in the antrum. Which of the following best characterizes the pathogenesis of epigastric pain in this panem? ***(A) Decreased calcium resorption by renal tubules ***(B) Decreased serum levels of PTH ***(C) Gastnc nonresponsiveness to PTH ***(D) Immunologic tolerance to H. pylori gasuicts ***(E) Increased secretion of gasuin
24 The answer is E: Increased secretion of gastrin. The incidence of peptic ulcer disease is increased in patients with hyperparatl1yroidism, possibly because hypercalcemia increases serum gasttin, thereby stimulaling gastric acid secre tion. Peptic ulcers in the context o£ MEN-1 may be secondary to ZollingerEllison syndrome (i.e , gastrinon1a of the endocrine pancreas). None of the other choices are associated with gasuic ulcers. ***Diagnosis: PepLlc ulcer disease, hyperparathyroidism
25 The parents of a 4-week-old girl complain that their baby is apathetic and sluggish. On physical examination, the childs abdomen is large and exhjbits an umbilical herrua. The skin is pale and cold, and the temperature is 35°C (95"F). Which of the following provides a plausible explanation for the signs and symptoms of this child7 ***(A) Cystic fib rosis ***(B) Muscular dystrophy ***(C) Parathyroid hyperplasia ***(D) TI1yro1d agenesis ***(E) Vitamin D defidency
25 The answer is D: Thyroid agenesis. Cretinism denotes physical and mental insufficiency that is secondary w congenital hypothyroidism. Cretinism may be endemk, sporadic, or familial and is twice as frequem in girls as in boys. Iodination of salt has reduced the incidence of cretinism in the United States and Olher coumries. The most common cause of neonatal hypothyroidism wday is agenesis of the thyroid, which occurs at a rate of l i11 4,000 newborns. Hypothyroidism in pregnant women also has grave neurologic consequences for the fetus, expressed after birth as cretinism. Symptoms of congenital hY1Jothyroidism appear in the early weeks of life and include sluggishness, a large abdomen often with umbilical hemiation, low body temperature, and refractmy anemia. Mental retardation, stunted growth, and characteristic facies become evident. lf thyroid hormone replacemem therapy is not promptly provided, congenital hypothyroidism results in mentally retarded dwarfs. None of the other choices produces the listed signs and symptoms at such an early age. ***Diagnosis: Congenital hypothyroidism
26 A 55-year-old man who is on dialysis because of end-stage renal disease complains of pain in his jaw and left arm for 6 months. An X-ray of the left am1 reveals multiple, small bone cysts and pathologic fractures. What is the appropriate diagnosis for this patient's bone lesions? ***(A) Chron ic osteomyelitis ***(B) Marble bone disease ***(C) Osteitis fibrosa cysLica ***(D) Osteoid osteoma ***(E) Osteoporosis
26 The answer is C: Osteitis fibrosa cystica. Secondary hyperparathyroidism is a complication of chronic renal insufficiency due to renal retention of phosphate and resulting hypocalcemia. Excess PTH causes renal osteodystrophy or, in severe cases, osteitis fibrosa qrstica. The Lauer is charac terized by severe bone defonnilies and the formation of "brown rumors" of hyperparathyroidism. Patients present with bone pain, bone cysts, pathologic fractures, and localized bone swellings (brown mmors). The other choices are not related to hypocalcemia or hyperparathyroidism ***Diagnosis: Osteitis q rstica fibrosa, hyperparathyroidism
27 A 46-year-old woman complains of increasing fatigue and muscle weakness over the past6 months. She repons an inability to concentrate at work and speaks with a husky voice. The patient denies drug or alcohol abuse. Physical examination reveals cold and clammy skin, coarse and brittle hair, boggy face with puffy eyelids, and peripheral edema. There is no evidence of goiter or exophthalmos. Laboratory studies show reduced serum levels ofT3 and T4. Which of the fo llowing is the most likely underlying cause of these signs and symptoms? ***(A) Amyloidosis of the thyroid ***(B) Autoimmune thyroiditis ***(C) Thyroid follicular adenoma ***(D) Muldnodular goiter ***(E) Papillary carcinoma of the thyroid
27 The answer is B: Autoimmune thyroiditis. Hypothyroidism refers to the clinical manifestations of thyroid hormone deficiency. lt can be the consequence of three general processes: (l) defective symhesis of thyroid hormone; (2) inadequate function of thyroid parenchyma; and (3) inadequate secretion of TSH. Dominam clinical manifestations of hypothyroidism include muscular weakness, peripheral edema, "myxedema madness," pallor, and enlarged tongue. Women with hypothyroidism suffer ovulatory failure , progesterone deficiency, and irregular and excessive menstrual bleeding. Erectile dysfunction and oligospermia are common symptoms of hypothyroidism in men. Primary (idiopathic) hypothyroidism is often autoimmune. Three founhs of patients with prima1y hypothyroidism have circulating antibodies to thyroid amigens , suggesting that these cases represent the end stage of amoimmune thyroiditis. Nongoitrous hypothyroidism may also result from antibodies that block TSH itself or the TSH recepwr, \vithout activating the thyroid. Some cases of primary hypothyroidism are pan of multiglandular amoimmune syndrome. The other choices present with either an enlarged rbyroicl or a mass and rarely present with hypothyroidism Hypothyroidism secondary LO amyloidosis of the thyroid (choice A) is rare. ***Diagnosis: Hypothyroidism
28 A 65-year-olcl woman wJLh a history of multinodular goiter complains or increasing nervousness, insomnia, and heart palpilations. She has lost 9 kg (20 lb) over the past 6 months. Physical examination reveals a diffusely enlarged thyroid. There is no evidence or exophthalmos. Labormmy smdies show elevated serum levels of T3 and T4. Sero logic tests for antithyroid antibodies are negmive. Which of the following is an important complication of this patiems endocrinopathy7 ***(A) Autoimmune hepatitis ***(B) Cardiac anhythmia ***(C) Follicular carcinoma or the thyroid ***(D) Medullary carcinoma of the thyroid ***(E) Myxedema madness
28 The answer is B: (ardiac arrhythmia. Hypenhyroidism refers to the clinical consequences of an excessive amount of circulating thyroid hormone. The p1incipal merabolic produC£s of the thyroid gland are ntwdothyronine (T3) and tetraiodorhyronine (lhyroxine; T4) T4 is ptincipally a prohormone; the major effector of thyroid funnion is T3. These molecules are formed by the iodination of tyrosine residues of thyroglobulin wilhin the follicular cells. Iodinated thyroglobulin is then secreted imo the lumen of the follicle. Many patients with nomox'ic goiter, usually over the age of 50 years, eventually develop a to)l.ic form of the disease. Since patients -..vith toxic goiter tend to be older, cardiac complications, including atrial fibtillation and congestive heart failure, dominate the clinical presemation. ***Diagnosis: Hypenhyroidism, wxic goiter
2t A 40-year-old woman complains of chronic constipation and anovulatory cycles for the last 8 months. Her vital signs are normal. Physical examination reveals pe1ipheral edema and a firm, diffmely enlarged thyroid gland. Semm levels of T3 and T4 are abnormally low. A thyroid biopsy is shown in the image. What is the appropriate diagnosis7 ***(A) Acute necrotizing thyroiditis ***(B) Autoimmune thyroiditis ***(C) Multinodular goiter ***(D) Reidelthyroiditis ***(E) Subacute (DeQuervain) thyroiditis
29 The answer I•S B: Autoimmune thyroiditis. Chronic autoimmune thyroidins (Hashjmoto thyroiditis) is a common cause of goitrous hypothyroidism. The disease is characte1ized by the presence of circulating antibodies to thyroid amigens and features of cell-meruated immuruty to 1.hyroid tissue. The disorder a1ises tnost commonly in the founh and fi fth decades, and women are six times more likely to be affected than men. On gross examination, the gland in patients with Hashimoto thyroiditis is diffusely enlarged and fim1, weighing 60 w 200 g. The cut surface is pale tan and Oeshy and exhibits a vaguely nodular panern . Microscopically, the thyroid displays (1) a conspicuous infiltrate of lymphocytes and plasma cells, (2) destruction and atrophy of the fo Uicl es, and (3) oxyphilic metaplasia or the folllcular epithelial cells (Hunhle or Askanazy cells) The inflammatmy infiltrates are focally arranged in lymphoid follicles, often 1.vith germinal cemers (see photomicrograph) Choice A (acute necrotizing rhyroirutis) has the appearance of an infection, whereas d 1oice C (multinodular goiter) is characterized by a nodular gland \:vithom significam inflammation. Choice D (Reidel thyroiditis) is a 6brosing condition and choice E (subacute IDeQuervain l thyroiditis) features muldnucleated giant cells. ***Diagnosis: Hashimoto thyroiditis, au toimmune thyroiditis
30 A 52-year-old woman complains of swelling in the anterior portion of her n eck, which she fi rst noticed 6 months ago. Except for some discomfon during swallowing, the patient does not repon any significant sympwms. Physical exami· nation reveals a symmetrically enlarged thyroid. A thyroid biopsy is shown in the image. vVhich of the following is the most likely diagnosis? ***(A) Follicular adenoma ***(B) Graves disease ***(C) Hashimom thyroiditis ***(D) Nontoxic goiter ***(E) Non-Hodgkin lymphoma
30 The answer is D: Nontoxic goiter. Nontoxic goiters range from doubte the size of a normal gland ( 40 g) to massive thyroid weighing hundreds of gram.'>. Microscopically, nomoxic goiler exhibits hypenrophy and hyperplasia There is marked variadon in size of the follicles (see photomicrograph). fibrosis, and evidence of old hemorrhage. The diffuse fom1 is frequem in adolescence and during pregnancy, whereas the multinodular type usually occurs in persons older than 50 years of age. Hashimow thyroiditis (choice C) feamres lymphocytic infiltrates. Graves disease (choice B) demonstrates a l1yperplasric and vascular thyroid. ***Diagnosis: Nontoxic goiter, multinodular goiter
31 Five years later, the patient described in Question 30 remrns with sympwms of hyperthyroidism Which of the following best summarizes the clinical symptoms expected in this patient? ***(A) Dry skin, hypogonadism, fatigability ***(B) Hyperpigmemation, weakness, hypotension ***(C) Nervousness, initability, paresthesias, tetany ***(D) Pale complexion, cold imolerance, lethargy ***(E) Tremor, tachycardia, weigl1lloss
31 The an.swer is E: Tremor, tachycardia, weight loss. Some patients with nontoxic goiter, usually over the age of 50 years, eventually develop hyperthyroidism , m which case the term toxic multinodular goiter is applied. Hypenhyroidism gives rise w tremors, tachycardia, heat intolerance, and weight loss. Women may experience oligomen orrhea. The sympwms of wxic goiter are less severe than those associated with Graves disease, and patients do not develop exophthalmos. Because patients with roxie goiter tend to be older, cardiac complications are common. The other choices, whlch include sympLOms such as hypogonadism (choice A), hyperpigmemation (choice B), tetany (choice C), and lethargy (choice D), are encountered in other endocrinopathies. ***Diagnosis: Toxic goiter
32 A 32.-year-old woman presents with a solitary, nontender, firm nodule on the left side of her neck. Thyroid function tests are within normal limits. A fine-needle biopsy reveals malignant cells. The tumor is excised and examined by light microscopy (shown in the image). Whal is the appropriate pathologic diagnosis? ***(A) Anaplastic carcinoma ***(B) Follicular carcinoma ***(C) Lymphoma ***(D) Medullary carcinoma ***(E) PapiJlary carcinoma
32 The answer is E: Papillary carcinoma. Although thyroid nodules are found in up to 10% of the population, malignant tumors of the thyTOid account for only about 1% of al1 cancers. Papillary carcinoma of the thyroid is the most common thyroid tumor in younger women. It has a tendency to metastasize to regional lymph nodes, but distant metastases are rare. The tumor is usually cured by surgery. As illustrated in this case, papillar)' carcinoma of the thyroid shows branching papillae lined by epithelial cells with clear (ground glass or Orphan Am1ic) nuclei and fibrovascular cores. A calcospherites (psammoma body) is also evident These structures are virtua11y diagnostic of papillary thyroid carcinoma. Psammoma bodies are not a feature of the other choices. ***Diagnosis: Papillary carcinoma of the thyroid
33 A 4 3-year-old woman complains of low-grade fever and has a J-day hist01y of pain in her neck. Physical examination reveals a slightly enlarged thyroid. A CBC is nom1al. A biopsy of the tl1yroid reveals granulomawus inflammation and the presence of giant cells (shown in the image). What is the appropriate diagnosis? ***(A) Graves disease ***(B) Hashimoto thyroiditis ***(C) Lymphadenoid thyroiditis ***(D) Nontoxic multinodular goiter ***(E) Subacute (deQuervain) thyroiditis
33 The answer is E: Subacute (DeQuervain) thyroiditis. Subacute thyroiditis (deQuervain , granulomatous, or giant-cell thy· roidil:is) is caused by a viral infection. lt is an tnfTequem, selfUmited disorder of the thyroid characterized by granulomatous in [lammation. The disease typically occurs after upper respiratory tract infections, including those caused by in Ouenza vims, adenovirus, echovims, and coxsackiev;rus. The thyroid gland is enlarged to -1-0 to 60 g, and the cut surface is firm and pale Initially, microscopic examination reveals an acme innammation, often with microabscesses. This is followed by the appearance of a patchy infiltrate of lymphocytes, plasma cells, and macrophages throughomtne thyroid. Destruction of follicles allows the release of coUoid, which elicits a conspicuous granulomatous reaction. The orher choices do not fearure a granulomatous reaction. ***Diagnosis: Subacute (DeQuervain) thyroiditis
34 A 33-year-old woman complains of swelling in the anterior ponion of her neck, which she first noticed 8 montJLS ago. Except for some discomfort during swallowing and hoarseness, the patient does not report any symptoms. Physical examination reveals a stony, hard thyroid gland that is adherent LO Olher neck structures. A thyroid biopsy is shown in the image. The pad1ologist reports that the thyroid parenchyma ls replaced by dense, hyalinized fibrous tissue and a chronic inflammatory infiltrate. What is the appropriate diagnosis7 ***(A) Follicular adenoma ***(B) Graves hypen.hyroidism ***(C) Multinodular goiter ***(D) Papilla1y thyroid carcinoma ***(E) Riedelthyroiditis
34 The answer is E: Riedel thyroiditis. The term thyroiditis in Riedel thyroiditis is something of a misnomer because this rare disease also affects soft tissues of the neck and i.s often associated with progressive fibrosis in other locations, including the retroperitoneum, mediastinum. and orbit. On gross examination, pan or all of the thyroid is stony hard and is described as ''woody." In most instances, the process is asymmetric and often affects only one lobe. Patients with Riedel t11>•roiditis notice the gradual onset of a painless goiter. Subsequently, mey may suffer from the consequences of compression of the trachea (st!idor), esophagus (dysphagia), and recurrent laryngeal nerve (hoarseness). The omer choices do not fearure extensive fibrosis. ***Diagnosis: Riedel thyroiditis
35 A 29-year-old woman complains of nervousness and muscle weakness of 6 months in duration. She is intolerant of heat and swealS excessively. She has lost 9 kg (20 lb) pounds over the past 6 months, despite increased caloric intake. She frequently finds her heart racing and can feel it pounding in her chest. She also states that she has missed several menstrual periods over the past few months. Physical examjnation reveals warm and moist skin and bulging eyes (exophthalmos). Laboratory studies will likely reveal which of the following endocrine abnom1alities in this patient? ***(A) Anti-thyroid DNA antibodies ***(B) Anti-TSH receptor antibodies ***(C) Decreased uptake of radioactive iodine in the thyroid ***(D) Increased serum T$):-[ ***(E) Low serum T3
35 The answer is 8: Anti· TSH receptor antibodies. Graves disease is the most frequent cause of l1ypenhyroidism in young adults. lt is an autoimmune disorder characterized by diffuse goiter, hyperthyroidism, and exophthalmos. The disorder is the most prevalent autoimmune disease in the United States, affecting 0.5% to 1% of the population under 40 years of age, and is seven to ten times more frequent in women than in men_ Patients with Graves disease are hyperthyroid due to the presence of stimulaLing lgG ami bodies that bind to the TSH receptor eA.'Pressed on the plasma membrane of thyrocytes. Patients note the gradual onset of nonspecific sympwms, such as nervousness, emotional lability; n·emor, weakness, and weight loss. They are imoleram of he<U, seek cooler envirmm1ems, tend to sweat profusely; and may report hean palpitations. Ami-thyrmd DNA antibodies (chotce A) are not common in patients with Graves disease, and the thyroid shows increased uptake of radioactive iodine (see choice C). Serum levels of TSH are low (see choice D) and serum levels of T3 and T4 are iligh (see choice E). ***Diagnosis: Graves disease
36 A thyroid biopsy obtamed from r.he patient desCiibed in Question 35 is shov.rn in the image. Which of the following best describes the pathologic findings? ***(A) Atrophy and fibrosis ***(B) Dense lymphoid infihrate with genninal cemers ***(C) FoUicular hyperplasia with scalloping of colloid ***(D) Necrotizing parenchymal granulomas ***(E) PapiJlaty hyperplasia vvith psammoma bodies
36 The answer is C: Follicular hyperplasia with scalloping of col· loid. ln Graves disease, the follicles are lined by hyperplastic, tall columnar cells. Colloid is pink and scalloped at the periphery adjacent to the follicuLar cells. None of the mher choices would appear in biopsy ***Diagnosis: Graves disease
37 A 33-year-old woman presents w;th a swelling in her neck, which she first noticed 2 months ago. Physical examination reveals a solitary, nomender nodule of the thyroid gland measuring 2 em i_n diameter. Thyroid function tests are within nonnal limits. The nodule does nol accumulate 125lodine on thyroid scintiscan_ A biopsy of the nodule is shown in the image. Which of the following is the most hkely diagnosis? ***(A) Follicular adenoma ***(B) Medullary thyroid carcinoma ***(C) Metastatic carcinoma ***(D) Multinodular goiter ***(E) Papillary thyroid carcinoma
37 The answer is A: Follicular adenoma. A single, wellcircumscribed. thyroid nodule in a young patient most likely represents a follicular adenoma, which refers to a benign neoplasm that exhibits [ollicular differentiation. It is the most common tumor of the thyroid and typically presents in euthyroid persons as a solitary "cold" nodule (l.e., a tumor that does not take up radiolabded iodine). Follicular adenoma is an encapsulated neoplasm in which the celJs are arranged in follicles resembling normal thyroid tissue. The other choices do 110 t typically present as an isolated small nodule, but a biopsy is necessary w rule om O£her causes. ln this case, the biopsy demonstrates a benign proliferation of thyroid follicles. ***Diagnosis: Follicular adenoma of the thyroid
38 A 36-year-old woman presents with swelling in her neck that she first noticed 3 months ago. She also complains of intermittent watery diarrhea over the same time period. Physical examination reveals a nontender nodule in the left lobe of the thyroid. The patients mother died o[ thyroid cancer 8 years ago. The thyroid nodule is found to be "cold" by radioiodine scintiscan. A needle biopsy of the nodule reveals malignant cells and homogeneous eosinophilic material (shown in the image). Laboratory studies would likely show elevated blood levels of which of the following hormones in thjs patient? -- ~ ***(A) Calcitonin ***(B) Cortisol ***(C) PTH ***(D) 14 ***(E) TSH
38 The answer i.s A: Calcitonin. Medullary thyroid carcinoma (MTC) is derived from C cells of the thyroid, which secrete the calcium-lowering hormone calcitonin. The disease rep resents fewer than 5% of all thyroid cancers, although the incidence is con..'iide rably higher in familial forms (e.g., McN-2). MTC is characteristically solid and composed of polygonal, granular cells that are separated by a distinctly vascular su·oma. A conspicuous feature is the presence of stromal amyloid, representing the deposition of procalcitonin This material is eosinophilic with the hematoxylin and eosin stain and rakes up the Congo red srain. MTC extends by direct invasion imo soft tissues and metastasizes to the regional lymph nodes and distam organs. Patients \vith MTC often suffer a number of symptoms related w endocrine secretion , including carcinoid syndrome (serotonin) and Cushing syndrome (ACTH). Watery diarrhea in one third of patiems is caused by the secretion of vasoactive intestinal peptide. T4 (choice D) is incorrect because the tumor does not cause hyperthyroidism and TSH (choke E) is normal because the remaining thyroid produces adequate thyroid hormone. Chokes B and C (cortisol and PTH) are not tl1yroid hom10nes. ***Diagnosis: Medullary carcinoma of lhe thyro1d
39 The rumor in the patiem described in Question 38 is removed, and a section s tained with Congo red reveals birefringent amyloid stroma. Genetic studies show that this patient has a familial cancer syndrome. In addition to hyperparathyroidism, the patient is advised that she is at risk of developing which of the following neoplastic diseases? ***(A) Craniopharyngioma ***(B) Follicular adenoma of thyroid ***(C) Neuroblastoma ***(D) Pheochromocytoma ***(E) Pimitary adenoma
39 The answer is D: Pheochromocytoma. Patients with the familial fom1 of medullary carcinoma are often affected with MEN-2, wh1ch includes pheochromocytoma o[ the adrenal medulla and parathyroid hyperplasia or adenoma. Somatic mutations of the RET protooncogene are found in patients with MEN-2A, and RET mutations have been detec ted in 25% lO 70% of cases o[ sporadic medullary lhyroid carcinoma. None. of the other choices are encountered in patients with MEN-2. ***Diagnosis: Multiple endocrine neoplasia
40 A +5-year-old man presents •vith swelling in the anterior portion of his neck. Physical examination reveals an enlarged nodular thyroid. Thyroid function tests are within nonnal limits. A thyroid scintiscan shows a dominant "hot" nodule. A biopsy of rh.is nodule reveals neoplastic cells wi th evidence of vascular and capsular invasion (shown in the image). X-rays demmlSu·ate distant bony metastases. vVhat is the most likely diagnosis? ***(A) Anaplastic carcinoma ***(B) B-celllymphoma ***(C) Follicular carcmoma ***(D) Medullary carcmoma ***(E) Melastadc carcinoma
40 The answer is C: Follicular carcinoma. Follicular thyroid carcinoma (fTC) is purely follicular and does not contain any papillary or other elements. Minimally invasive fTC is seen grossly as a well-defined, encapsulated tumor, \Vhich on cut section is soft and pale tan and bulges from the confines of its capsule. Microscopically, most lesions resemble follicular adenoma, although they tend more to a microfollicular or trabecular pattern. Anaplastic carr:inoma of the thyroid (choice A) manifests as large masses of the gland that are poorly drr:umscribed and frequently e>-'tend into the soft tissues of the neck ***Diagnosis: Follicular carcinoma of the thyroid
41 A 4-year-old girl is brought to the pedjatric clinic by her mother who reports that her daughter has decreased appetite , lethargy, and an enlarging belly Physical examination reveals a large, finn, irregular, nontender mass in the child's abdomen. ACT-guided biopsy reveals neoplastic "small blue cells ." Th.e child's ma1ignam neoplasm is removed and th.e surgical specimen is shown in the image. vVhich of Lh.e following laboratory tests would be useful in monitoring this patient for recurrence of disease? ***(A) Serum acetylcholine ***(B) Serum a. 1-antiuypsin ***(C) Serum potassium ***(D) Urinary angiotensin ***(E) Urinary vanillylmandelic acid
41 The answer is E: Urinary vanillylmandelicacid. Neuroblastoma is a malignant tumor of neural crest origin that 1s composed of neoplastic neuroblasts, and originates in the adrenal medulla or sympathetic ganglia (note the suprarenal location of the mmor) The neuroblast is de rived from primitive sympathogonia and represents an intermediate stage in the development of the sympathetic ganglion neurons. On hiswlogic examination, pseudorose ttes, featuring tumor cells clustered radially around small vessels, are present. Tumor cells, like their more mature descendants in the adrenal medulla, may secrete catecholamines. These compounds are metabolized and excre ted as urinary vanillylmandelic acid in the u rine. Neuroblastoma is one of the most imponanr ' malignam mmors of childhood , accouming for up ro 10% of all childhood cancers and 15% of cancer deaths among children. The overall incidence is 1 in 7 ,000. The peak incidence is in the firsL 3 years of life. The other choices do not serve as markers for neuroblasroma or orher childhood malignancies. ***Diagnosis: Neuroblastoma
42 Genetic analysis of tumor ce Us taken from the patient described in Question 41 may reveal which of the following mutations? ***(A) C-ras amplification ***(B) Deletion of c-myc ***(C) N-myc amplification ***(D) 8;21 chromosomal translocation ***(E) 9;22 chromosomal translocation
42 The answer is C: N-myc amplification. Neuroblastoma is characterized by extrachromosomal double minutes, and homogeneously staining regions (HSRs) are found on chromosome 2 in 30% of cases. These H$Rs represent amplification of N-myc, an abnonnalit:y that plays a key role in determining the aggressiveness of neuroblastoma. The other genetic abnormalities are not characteristic of neuroblastoma. ***Diagnosis: Neuroblastoma
43 A 45-year-old man with a recem history of bizarre behavior is seen by a psychiatrist, who recommends evaluation of his endocrine status. On ph ysical examination, the patient appears moderately obese (BMl = 31 kglm2 ), with mild hypertension, facial acne, fat accumulation in the supraclavicular fossae, and a protuberant abdomen. Laboratory studies demonstrate a neutrophilic leukocytosis, with a decrease in the percemage oflymphocytes and an absence of eosinoph]ls. The hematocrit and hemoglobin are normal. There is a mild hypokalemia and mild metabolic alkalosis. The fasting serum glucose is withu1 the reference range, but on a 2-hour glucose tolerance test, both the 60- and 120-minute samples had glucose concentrations greater than 200 mgldL. Laboratory studies show rree urinary cortisol of 156 mg per 24 hours (normal = 10 lO 100 pg per 24 hours). vVhich of the following questions would be of most help in es1.ablishmg a dtagnosis? ***(A) Arc you cxpcncncing muscle weakness? ***(B) Are )'OU experiencing shonness of breath? ***(C) Arc you rcccivmg corticosteroids for some other disease? ***(D) Do you have a family hiswry of endocrine neoplasia? ***(E) Have you rccel\'cd recent blood transfusions?
43 The answer is C: Are you receiving corticosteroids for some other disease? The cJiJ1ical fearures of hyperconisolism from any cause are referred w as Cushing syndrome, and the te rm Cushing disease is reserved for excessive secretion of conicouopin by pituimry conicotrope rumors. The most common cause of Cushing syndrome in the United States is chronic adnlinistration of corticosteroids in the treatmem of immunologic and inHammarory disorders. Major clinical manifestaLi011S of Cushing syndrome include emotional disturbance, moon facies , osteoporosis, upper truncal obesity, d1in and ~rrinkled skin, amenorrhea, muscle weakness. purpura, and skin ulcers . lialf of patients exhibit absolute lymphopenia. These cUnical manifestations depend on the degree and duradon of excessive conicosteroid levels , as well as on the levels of adrenal androgens and mineralocorticoids. Muscle weakness (choice A) is a feature of Cushing syndrome but is nonspecific. ***Diagnosis: Cushing syndrome
44 A -+2-rear-old woman presents w1th amenonhea and emotional disturbances. You note upper truncal obesity and suspect Cushmg syndrome. U.boratory studtes re\'eal elevated serum levels of comcosterotds that can be lowered by administrauon of dexamethasone. Whtch of the following is the most likely cause of hypcrcorusolism in this patient? ***(A) Adrenal conical adenoma ***(B) Adrenal corucal carcinoma ***(C) Adrenal conical hyperp lasia ***(D) Pheochromocytoma ***(E) Pituitary adenoma
44 The answer is E: Pituitary adenoma. Cushing disease is caused by pituirary tumors that secre te ACTH and result in adrenal hyperfunction. It is five times more frequent than the type of Cushing syndrome associated with adrenal tumors. The dexamethasone suppression test is used to distinguish between ACTH-dependent and ACTH-independem fonns of C1.tshing syndrome. Dexamethasone suppresses pituitary secretion of co rtkotropin and, hence, hyperconisolism, whereas it is without effect in cases of adrenal hyperplasia or functional adrenal tumors. Thus, the dexamethasone suppression test can be used to distinguish between ACTH-dependem and ACTH-independent forms of Cushing syndrome. Adrenal conical carcinoma (choice B) is often a functioning tumor but is rare. Adrenal cortical hyperplasia (choice C) usually occurs secondary to a corticotropinsecre ting pituitary tumor. ***Diagnosis: Cushing disease
45 A 40-year-old woman with a hiStory of diabetes complains of recent changes m her bodtly appearance. A photograph of Lhe pauem is shown m the image. Laboratory studies reveal elevated serum corucosteroids and low serum corticotropm. Admmtstrauon of dexamethasone does not lower serum le,·els of comcoster01ds. Thts pauem most hkel)' has a tumor !hat ongtnatcs m whtch of the followmg anaLOmic locanons1 ***(A) Adrenal cortex. zona fasciculata ***(B) Adrenal cortex, zona glomerulosa ***(C) Adrenal cortex, zona reticulari.s ***(D) Adrenal medulla ***(E) Anterior pituitary
45 The answer is A: Adrenal cortex, zona fasciculata. Cushing syndrome presents with an accumulation o[ subcutaneous fat on the posterior neck ("buffa lo htunp"), striae, and diabetes melliLus. In this case, the disorder was caused by an adrenal tumor, which could have been either a conical adenoma or carcinoma. The typical adenoma is an encapsulated, firm, yeliO\·V, slightly lobulated mass, measuring abom 4 em in diameter, which secretes glucocomcoid honnones. Tumors of the zona glomerulosa (choice B) secrete aldosterone (mineralocorticoid), and those of the adrenal medulla (choice D) are pheochromocytomas that secrete epineph1ine Lack of dexamethasone suppression rules out pituimry adenoma (choice E). ***Diagnosis: Cushing syndrome
46 A -+6-year-old woman with severe asLl1ma presents with increasing weight and back pain for 9 momhs. The patient i.s taklng corticosteroids ror her asthma. An X-ray or the vertebrae will likely reveal which of the rouowing pat11ologic find ings? ***(A) Bone infarct ***(B) Dislocation ***(C) Osteomalacia ***(D) Osteomyeli tis ***(E) Osteoporosis
46 The answer is E: Osteoporosis. Long-term administration of corticosteroids causes increased bone resmption and decreased bone fom1atlon, thereby leading LO osteoporosis. As many as 20% of patients \vith Cushing syndrome suffer compression fractures of the venebrae. Osteomalacia (choice C) usually renects abnormalities of vitamin D metabolism and features inadequate mineralization of newly formed bone. ***Diagnosis: Cushing syndrome
47 A 40-year-old man complains of nausea, vomiting, diarrhea, and cramping abdominal pain. His temperature is 38°( (101 °F), blood pressure 90/60 mm Hg, and pulse rate 90 per minute. On physical examination, the patienL appears dehydrated, \Vit.h sunken eyeballs, dry congue, and poor skin turgor. Hyperpigmentation is noted in the palmar creases and the gingival margins. Laboratory results include fasting serum glucose of 62 mgldL (nonnal = 70 to 115 mgldL), BUN of 27 n1g!dL (normal = ll to 23 mgldL), Na of 122 mEq/L (n01111al = 136 to 145 mEq/L), and K of 6.5 mEq!L (normal = 3.5 to 5.0 mEq/L). Which of the following is the most likely cause of this patients symptoms? ***(A) Amyloidosis ***(B) Autoimmunity ***(C) Metastatic cancer ***(D) Sarcoidosis ***(E) Tuberculosis
47 The answer is B: Autoimmunity. Primary chronic adrenal insuffi.ciency (Addison disease) most often reflects autoimmune destruction of the adrenal gland. It is a fatal wasting disorder caused by the failure of the adrenal glands to produce glucoconicoids, mineralocorti.coids, and androgens. 1[ untreated, Addison disease is characterized by weakness, weight loss, gastrointestinal symptoms, hypotension, electrolyte disturbances, and h}1Je rpigmemation. More than 90% of the adrenal gland mLLSt be destroyed before the symp· toms of chronic adrenal insufficiency surface. Less common. causes of Addison disease include tuberculosis (choice E) , metastatic carcinoma (choice C), amyloidosis (choice D) , sarcoidosis, adren.oleukodystrophy, and congenital adrenal hypoplasia. ***Diagnosis: Addison disease
48 A 50-year-old man complains of muscle weakness and dizziness of 3 months in duration. His blood p ressure is 185/100 mm Hg. Laboratory studies show hypernatTemia and hypokalemia. Endocrine studies reveal elevated serum aldosterone and low renin and angiotensin. BUN is 24 mgldL, and creatinine is 1.2 mgldl. Endocrinologic studies rule out Cushing syndrome. Which of the following is the most likely cause ofhypertension in this patiem7 ***(A) Adrenogenital syndrome ***(B) Chronic adrenal failure ***(C) Chronic renal failure ***(D) Conn syndrome ***(E) Pheochromocytoma
48 The answer is 0: Conn syndrome. Primary hyperaldosteronism (Conn syndrome) reflects inappropriate secretion of aldosterone by an adrenal adenorna (75%) or hype rplastic adrenal glands. Most patients with primary aldosteronism are diagnosed after the detection of asympromatic diastolic hypertension. Muscle weakness and fatigue are produced by the effec ts of potassium depletion on skeletal nmscle. Less Common syndrome is assodated with low or normal renin levels. Chronic renal failure (choice C) is excluded by normal BUN and creatinine. ***Diagnosis: Conn syndrome
49 A 34-year-old man complains of sudden attacks of dizziness, bluned vision, and excruciating headaches of 4 momhs in duration. During one of these attacks, his blood pressme was 180/120 mm Hg. The patients father had been treated for thyroid cancer about 15 years ago. Laboratory studies show nonnal serum levels of aldosterone, renin, and angiotensin. A 24-hour urinalysis reveals increased metanephril1es. Episodic hypertension in this patient is most likely caused by a tumor in which o[ the following endoCJine organs? ***(A) Adrenal ***(B) Kidney ***(C) Parathyroid ***(D) Pltuitary ***(E) Thyroid
49 The answer is A: Adrenal Pheochromocytoma is a rare tumor of chromaffin cells of the adrenal medulla that secretes catecholamines. Such tumors also originate in extra~adren al sites, in '<Vhich case they are te rmed paragangliomas. The clinical featu res associated with pheo· chromocytoma are caused by the release of catecholamines. Patients wlth pheochromocytoma come to medical atten· tion because of (1) asymptomatic hypertension discovered on a l'OUtinc physical examination, (2) symptomatic hypertension that is resistant to antihypertensive therapy; (3) malignam hypertension (e.g., encephalopathy, papilledema, proteinuria), (4) myocardial infarction or aortic dissection, or (5) convulsions. The other choices do not include tumors that lead to episodic hypertension. ***Diagnosis: Pheoch romocywma
1 A 14-year-old boy presents for a presummer camp physical examination. Routine urinalysis discloses 3+ glucosuria. He admits to thirst and frequent urination, accompanied by a 4-kg (9-lb) weight loss over the past few months. His parents note that he had a Ou-like illness 5 months ago. His blood glucose is 2.20 mgldL Which of the following best explains the pathogenesis of hyperglycemia in this patient? ***(A) Excess dietary glucose ***(B) Increased peripheral insulin uptake ***(C) Irregular insulin secretion ***(D) Islet cell desnuclion ***(E) Pe1iphe ral insulin resistance
1 The answer is D: Islet cell destruction. Type l diabetes mellitus (TlDM) is a lifelong disorder of glucose homeostasis that results from the autoimmune destn.lction of the ~-cells in the islets of Langerhans. The clinical onset o£ TlDM often coincides with another acute Ulness, su ch as a feb1ile viral or bacterial infection. The disease is characterized by few, if any, remaining functional cells in the islets of Langerhans and limited or absent insulin secretion. The most cha racteristic early lesion in the pancreas is a lymphocytic infiltrate in the islets (insulitis), sometimes accompanied by a few macrophages and neutrophils. As the disease becomes chronic, P-cells of the islets an~ progressively depleted. Choice E (peripheral insulin resistance) represents diabetes type 2 . The other choices do not lead m hyperglycemia in normal persons. ***Diagnosis: Diabetes mellitus, type l
2 A 10-year-old boy with a recent onset of diabetes mellims dies following an automobile accident. Hiswlogic examination of the child's pancreas at autopsy is shown in the image. Injury to pancreatic islet celJs in dus patient was most likely mediated by which of the following mechanisms of disease? ***(A) Antibody-medinted islet cell destruction ***(B) Cell-mediated Immunity ***(C) Duecl \ira! C}'lopathic effects ***(D) Hypo\'olemic shock ***(E) Phagoc}'lOSIS by acnvated macrophagcs
2 The answer is 8: Cell-mediated immunity. Cell-mediated immune mechanisms are fundamental to the pathogenesis of type l diabetes mellitus (TlDM), and cytotoxic T lymphocytes sensitized to P-cells in TlDM persist indefinitely. Circulating antibodies (choice A) against components of the ~-cells of the islets, including insulin itself, are identifted in most newly diagnosed patients with diabetes. However, these amibodies are regarded as a response to P-cell injmy, rather than the initial cause of ~-cell depletion. Evidence for viral causes of diabetes mellims type 1 (choice C) remains controversial. ***Diagnosis: Diabetes mellitus, type 1
3 A 55-year-old obese ''voman (body mass index = 33 kglm2) complains of declining visual acuity. Funduscopic examination shows peripheral retinal microaneurysms. Urinalysis reveals 3+ proteinuria and 3+ glucosuria. Serum albumin i.s 3 gldL, and serum cholesterol is 350 mgldL. These clinicopathologic findings are best explained by which of the following mechanisms or disease? ***(A) Ami-insulin amibodies ***(B) lncreased peripheral insulin up take ***(C) Irregular insulin secretion ***(D) Pe1ipheral insulin resistance ***(E) Secretion of insulin-like proteins
3 The answer is D: Peripheral insulin resistance. Type 2 diabetes mellln.ts results hom a complex interplay between underlying resistance to the anion of insulin in its metabolic target nssues (liver, skeletal muscle, and adipose Lissue) and a reduction in glucose-stimulated insulln secreUon, which fails to compensate for the lno·eased demand for insulin. ln obese persons, tl1e release of inhibitory mediawrs from adipose tissue interferes Virith intracellular signaling by insulin. Hyperinsulinemia secondary lO insulin resistance also dov.rmegulates the munber of insuljn receptors on the plasma membrane. The other choices have not been related to the pathogenesis of type 2 diabetes. ***Diagnosis: Diabetes mellims, type 2
4 A 61-year-old man presems >vith a 5-year histmy of pain i.n both legs during exercise. He has been treated for diabetes for 8 years. His fasting blood glucose is 280 mgldL Which of the following best explains the pathogenesis of leg pain in this patiem? ***(A) Atherosclerosis ***(B) Malignam hypertension ***(C) Microaneurysms ***(D) Pe1ipheral neuropathy ***(E) VascuJi tis
4 The answer is A: Atherosclerosis. The extent and seve1ity of atherosclerotic lesions in medium-sized and large arteries are increased in patients with long-standing diabetes. leg pain during walking or exercise, which forces the patient to stop or limp (intermittent claudication) is typically a complkation of atherosclerosis involving the major arteries of the lower extremi ties. Peripheral neuropathy (choice D) is a complication of diabetes but is an unlikely cause of claudication. Diabetes does not cause vasculitis (choice E). ***Diagnosis: Atherosclerosis
A 60-year-old man with diabetes mellitus complains of deep burning pain and sensitivity to touch over his hands and finge rs. Nerve conduction studies show slow transmission of impulses and diminished muscle stretch reflexes in the ankles and knees. Sensati011s to vibrations and light wuch are also markedly diminished. The development of polyneuropatll )' in this patient cor relates best with which of the following conditions? ***(A) Anti-insulin amibody liler ***(B) Hyperglycemia ***(C) lnsulin deficiency ***(D) lmermiuem h)rpoglycemia ***(E) Ketoacidosis
5 The answer is B: Hyperglycemia. The severity and chronic- L...- ity of hyperglycemia in both TlDM and T2DM are the major pathogenetic facto rs leading to the microvascular complications of diabetes, including retinopathy, nephropathy, and neuropathy. Thus, control of blood glucose remains the major means by which Lhe development of microvascular diabelic complkaLions can be mil)jmized . Glucose binds nonenzymatically by aLtacl)jng to a variery of prOLeins. This process, tem1ed glycosylation, occurs roughly in proportion to the severity of hyperglycemia. Unfonunately, trials in which blood glucose levels were carefully connolled did not necessarily prevem all complications of di.abetes. ***Diagnosis: Diabetic neuropathy
6 A 56-year-old man with a 14-year history of diabetes mellims presents with poor vision, peripheral vascular disease, and mild proteinuria. Whlch of the following is the best moniwr of the control of blood sugar levels in this patiem7 ***(A) Glycosylated hemoglobin ***(B) Islet cell autoantibody ***(C) Serum myoinositol ***(D) Serum sorbitol ***(E) Serum triglycerides
6 The answer is A: Glycosylated hemoglobin. A specific frac tion of glycosylated hemoglobin in circulating red blood cells (hemoglobin Ale) is measured routinely to monitor the overall degree of hyperglycemia that occurred during the preceding 6 to 8 weeks. Nonenzymatic glycosylation of hemoglobin is irreversible, and the level ofhemoglobm Ale, therefore, serves as a marker for glyceJmc control. None of the other choices are quantitative measures of glucose levels. ***Diagnosis: Diabetes mellims
7 A 65-year-old obese man (body mass index = 32 kglm2) presents with a 2-year history of difficulty walking. Physical examination reveals chronic ukers in the lower extremities. Fun duscopic examination reveals proliferative retinopathy. Which of the follo·wing best describes the pathogenesis of chronic ulcers on the legs of this patient? ***(A) Abnormal glycosyladon o[ hemoglobin ***(B) Inadequate leukocytic respon..se to infection ***(C) Low concemrations of insulln in tissues ***(D) Microvascular disease ***(E) Varicose veins
7 The answer is D: Microvascular disease. Microvascular disease, a characteristic complication of diabetes, causes ischemia and is, in pan , responsible for the slow hea]jng of wounds in the diabetic patient. It also results in other complkations of diabetes such as renal disease. ln addition to microvascular disease, aggregation of platelets in the smaller blood vessels and impaired 6blinolytic mechanisms have also been suggested to play a role in the pathogenesis of diabetic microvascular disease. The susceptibility of djabetics to infection is a complex problem, but u does not seem that the functions of polymorphonuclear leukocytes are directly affected (choice B) The tissue concentra(ion of insulin {choice C) does not mOuence the healing process. DiabeLes mellitus does not predispose to varicose veins (choice E). ***Diagnosis: Diabetic n)jcrovascular disease
8 Thickening of small vessel basement membranes in the patiem described in Question 7 is most likely related to abnonnalir.ies in which of the following cellular and biochemical processes? ***(A) Amyloidosis ***(B) Collagenous fibrosis ***(C) Glycosylation ***(D) Immunoglobulin deposition ***(E) lnsudation of fib1in
8 The answer is C: Glycosylation . Increased deposition and glycosylation of basement membrane proteins contribute to the pathogenesis of diabetic microvascular disease. Thus, control of b lood glucose remains the major means by which the development of microvascular diabetic complications can be minimized. The ot11er choices do not preferentially accumulate in small vessels affected by diabetes. ***Diagnosis: Diabetic microvascular disease
9 A 58-year-old man with a long-standing histo1y of type 2 diabetes mellitus suffers a massive hemorrhagic stroke and e.\.1)ires. Examination of the pancreas shows hyalinization of many islets of Langerhans. Which of the following characterlzes the material within the lslets of Langerhans? ***(A) Amyloid ***(B) Collagen rype IV ***(C) Fibrin ***(D) Fibronectin ***(E) Proteoglycan
9 The answer is A: ,. _ Amyloid. In T2DM, amyloid is often p resent within the islets of Langerhans , panicularly in patients over 60 years of age. This type of amyloid derives from a polypeptide molecule known as amy lin, which is secreted with l11Sulin by the ~-cell. As many as 20% of aged nondiabetic persons also have amyloid deposits in the pancreas, which is a findi ng that has been attributed to the agi ng process itself. None of the other choices show Congo red staining and apple-green bi refringence under pola1ized light. ***Diagnosis: Amyloidosis
10 A 50-year-old man with diabetes mellitus develops swelling in his lower eJ<:tremlties. Ulinalysis shows 3+ protelnuria and 3+ glucosuria. Se.mm albumin is 3 gtdL and serum cholesterol is 350 mgl dL A kidney biopsy is shown in the image. Which of the following glomemlar changes is evident in this biopsy specimen? ***(A) Amyloidosis ***(B) Deposition of baseme:m membrane-like material ***(C) Endothelial cell hyperplasia ***(D) Fibrinoid necrosis ***(E) Mesangial hyperplasia
10 The answer is B: Deposition of basement membrane-like mate· rial. Microvascular disease is the major cause of renal failure and blindness in diabetics. Hyaline arteriolosclerosis and caplllaiy basement membrane thlckening are characteristic vascular changes in those with diabetes. The frequent occurrence of hypertension comribUles [0 the development or the arteriolar lesions. In addition, deposition of basement membrane proteins, which may also become glycosylated, increases in diabetes. increased mesangium results in glomerulosclerosis and, evemually, in renal failure. Eventually, the glomemU in the diabetic kidney exhibit a unique lesion termed KimmelstieiWilson disease or nodular glomerulosclerosis. Of patients with Tl DM, 30% to 40% uJtimately develop renal failure. A somewhat smaller proportion (up to 20%) of patients with T2DM are similarly a[fected. The deposited basement membrane material has a similar morphologic appearance w amyloid (choice A) but does not stain with Congo red. ***Diagnosis: Diabetic glomerulopathy
11 A 75-year-old woman with well-controlled diabetes complains of poor eyesight. A grayish-white opacification of the lens is found during a comprehensive eye examination. Which of the follov.ring metabolic pathways is most ljkeJy involved in this lens abnonnality? ***(A) Aldose reductase pathway ***(B) Amino acid degradation cycle ***(C) Citric acid cycle ***(D) Oxidative phosphorylation ***(E) Penrose-phosphate shunt
11 The answer is A: Ald.ose reductase pathway. The aldose reductase pathway has been implicated in the pathogenesis of diabe tic compllcadons in some tissues, including the formation of cataracts. Glucose is converted ro sorbiwl (sugar alcohol) , which can be cytOtoxic. It is suspeCLed to play a role in diabetic complications in a variety or tissues, including pe1ipheral nerves, retina, lens and kidney None or the other chokes have been implicated in the patJ1ogenesis or cataracts. ***Diagnosis: Cataract
12 A 40-year~old diaberic woman complains of flank pain and fever. J-ler temperature is 38.rC (l03°F), respirations are 25 per m]nme, and blood pressure is 150/90 mm Hg. Urinalysis reveals pyuria vvith WBC casts. Whlch of the following features of diabetes is the most important comributing factor in the development of nank pain and fever in this patient? ***(A) Anti-insuJin amibomes ***(B) Glycosylation of hemoglobin ***(C) Hyperglycemia ***(D) Pe1ipheral insulin resistance ***(E) Sorbitol accumulation
12 The an-swer is C: Hyperglycemia. Flank pain, fever, and pyuri.a are indicative of acme pyelonephritis, a common complication of cUabetes. Glucose in the urine provides an enriched culture medium. ln addition, patients with amonomic neuropathy often have a dystonic bladder that retains urine. Pyelonephritis is a constant uuear for parienrs with diabetes, and necrotizjng papillitis may be a devastating complication of renal infection. The other choices are not related ro renal infection. ***Diagnosis: Pyelonephritis, papillary necrosis
13 A 37-year-olcl woman with cUabetes mellitus delivers a child after 38 weeks of gestation Which of the following is the most hkely abnormality that might be encou ntered in this child at birth? ***(A) Cataracts ***(B) Hyperbihrubinemia ***(C) Hypoglycemia ***(D) Low binh weight ***(E) Mental retardation
13 The answer is C: Hypoglycemia. Tight glucose comrol in the diabetic mother is necessary to prevent overstimulation of the fetal pancreas dming gestation. Fetuses exposed to hyperglycemia in utero may develop hyperplasia or the pancreatic P-cells, which may secrete insulin autonomously and cause hypoglycemia at birth and in the early neonatal period. Infants of diabetic mmhers show a 5% to 10% incidence of major developmem abnonnahties. Increased birth weight is commonly encountered in offspring of diabetic mothers (see choice D). The incidence of mental retardation (choice E) is not specifically increased. ***Diagnosis: Gestational diabetes
14 An obese woman (body mass index [BMII ""32 kglm2 ) presenr.s for a routine physical examination. ln revle'wing your patients health status, you memion that obesity is associated with an increased incidence of which of the following diseases? ***(A) Cardiomyopathy ***(B) Cervical carcinoma ***(C) Chronic obstmctive pulmonary disease ***(D) Degenerative joim disease ***(E) Diabetes mellitus type 1
14 The answer is D: Degenerative joint disease. Degenerative joint disease (osteoarthritis) of weight-bearing join ts is a common complication of obesity. The hips and knees are most commonly affected. Obesity is determined according to BMI, which is calculated as weight (kg}/heigh t (m2). A BMI of 2.5 to 30 kglm2 is classed as overweight. Obesity is a risk factor for the development of adult-onset (type 2.) diabetes mellitus, but not for j uvenile-onset (type l) diabetes mellitus (choice E) . Obesity by itself does not cause cardiomyopathy (choice A). ***Diagnosis: Obesity
1 A 57-year-oldwoman p resents with a 1-year history of diarrhea, increasing fatigue, and weight loss. Physical examination reveals a large tongue and Lnduration of the anterior abdominal '<VaiL Laboratory tests are all within nonnal ranges. A o-:x-ylose absorption test is abnormal, indicating small Lntestinal malabsorption A CT scan of the pelvis shows diffuse thickening of the rectal wall, retroperltoneum, and mesentery. UrLnalysis documents proteinu tia (0.5 g per day), witJ1out evidence of Bence-jones protein. The plasma cell count in the bone marrow biopsy is increased by 10%, but there is no evidence of multiple myeloma. Which of the following tests would be most useful for establishing a diagnosis of primary amyloidosis? ***(A) Magnetic resonance imaging ***(B) Polymerase chain reaction (PCR) ***(C) Protein electrophoresis ***(D) Technetium scan ***(E) Tissue biopsy
.___1__ The answer is E: Tissue biopsy. No single set of clmical signs or symproms points unequivocally w amyloidosis. The symptoms are governed by bmh the underlying disease and the type of protein deposited. Even when one suspects amyloidosis, the diagnosis ultimately rests on its hiswlogic demonstration in biopsy specimens. The staining and stmctural properties of amyloid allow a general definition, based primarily on morphologic characteristics. vVhen routine stains are used, amyloid is amorphous, glassy, and has almost cartUa geHke properties, which are responsible for its so-called hyaline appearance. PCR (choice B) is used to amplify DNA, generating thousands to millions of copies of a particular sequence. PCR is currently used in biomedical research laboratories for the diagnosis of hered.ita1y and infectious diseases. Protein electrophoresis (choice C) may disclose a paraproteinemia but by itself does not inform about tissue amyloid formation. ***Diagnosis: Lymphoma
2 Which of the following histoch emi.cal s tains is most useful for identifying amyloid in a tissue biopsy taken from the patient described in Question 1? ***(A) Congo red ***(B) Masson trichrome ***(C) MucicarmLne ***(D) Petiodlc acid-SchilT (PAS) ***(E) Rl1odanine
2 The answer is A: Congo red.. All fonm of amyloid stain positively with Congo red and show red-green birefringence when viewed under pola1ized light. The fibrillary deposits organized in one plane have one color, and those organized perpendicular to that plane have the other color. Ultrastructurally, all forms of amyloid consist of imerlacing bundles of parallel arrays of fibrils. which have a diameter of 7 to 13 nm. The protein in the amyloid fibrils contains a large proportion of crossed P-pleated sheet stmcture. Masson trichrome stain (choice B) highlights collagen. Mucicam1ine (choice C) stains mucin. The PAS stain (choice D) identifies glycogen and basement membranes. Rhodanine stain (choice E) demonstrales copper. ***Diagnosis: Lymphoma
3 A biopsy taken from the rectum of Lhe patient described il1 Questions land 2 is app ropriately stained and observed under polarizing light (show11 in the image). Immunohistochemical assays show that this birefringent material contains kappa chains. This palient'S amyloidosis belongs w which of lhe fo Uowing caLegories? ***(A) AA ***(B) AJ32M ***(C) AL ***(D) APrP ***(E) ATTR
3 The answer is ( : AL. Primary amyloidosis refe rs to the presentation of amyloid without any preceding disease. ln one third of these cases, primary amyloidosis is the harbinger of frank plasma cell neoplasia, such as multiple myeloma or other B-cell lymphomas. AL amyloid usually consists of the variable region of immunoglobulin light chains (L) and can be derived fTom either the kappa ( K:) or lambda (A.) polypeptides. The other choices do not involve immunoglobulins. ***Diagnosis: AL amyloid, lymphoma
4 A 65-year-old woman with longstanding rheurnatoid arthritis complains of swelling of her eyelids , abdomen, and ankles. A chest X-ray shows bilateral pleural effusions, without evidence of pulmonary infiltration or consolidation. Utinalysis reveals heavy proteinuria (8 g per 24 hours). A percmaneous renal biopsy demonst rates glomemlar deposits that are PAS-negative (shown iJ1 the image). The acellular material obstructing the glomerular capillaries in this patient is most likely derived from which of the following serum proteins7 ***(A) Apo serum amyloid A ***(B) Calciwnin ***(C) Immunoglobuli n light chain ***(D) f3-Protein precursor ***(E) Transthyredn
4 The an.swer is A: Apo serum amyloid A. AA amyloid is common to a host of seemingly unrelated, persistent, inflammatory, neoplastic, and hereditary disorders that lead to so-called secondary amyloidosis. for example, patients with rheumatoid anhritJ.s, ankylosing spondylitis, and tuberculosis may develop secondary amyloidosis. There is a spectrum of AA peptides of differing size in the deposits, all of which have the same aminotenninal sequence. This includes the intact precursor of M serum amyloid A (SAA). SAA is an acute-phase reactan t protein that increases up to 1,000- fold during inOammatory processes. Denatl.uing of SAA releases a subunit termed apoSAA, which renders it amyloidogenic. ln contrast to AL protein derived from immunoglobulin light chain {choice C), the amino acid sequence of AA proteins is identical in all patients, regardless of the underlying disorder. Progressive glomerular obliteration may ultimately lead to renal failure. ***Diagnosis: Rheumatoid arthritis
5 The patiem described in Question 4 asks for funher infom1adon regarding her medical condition. You e>q>lain that "amyloid" refers to a group of extracellular prmein deposits that are best defined as which of the following? ***(A) Breakdo\vu products of fibrinogen ***(B) ImmunoglobuUn light chains ***(C) Polypeptides with alpha-helical stmcmre ***(D) Proteins with common morphologic properties ***(E) Substances visualized as dense deposits on elecuon microscopy
5 The answer is D: Proteins with common morphologic proper- .__ ties. Amyloid refers Loa group of diverse extracellular protein deposits that have (1) common morphologic proprieties, (2) affinities for specific dyes, and (3) a characteristic appearance under polarized Ught. Ahhough the differem types of amyloid vary in amino acid sequence, all amyloid proteins are folded in such as way as to share common uhrasuuctural and physical propenies (e.g., P-pleated sheets) Thus, all amyloids have a similar uhrasnuctural appearance, regardless or which protein is responsible for the fibrillary components. By electron microscopy, amyloid consists of interlacing bundles of parallel arrays of fibrils, which have a diameter of 7 to 13 nm (not dense deposits, choice E). Amyloid proteins rich in alphahelical strucrures (choice C) are encountered in diseases that generate AA amyloid. ***Diagnosis: AA amyloid, rheumawid arthritis
6 A 55-year-old man, who has smoked heavily for many years, presents with a 3-year hist01y of persistem cough. frequenl upper respiratory infections, and production of foul -smelling, mucopumlem sputum. An X-ray film of the chest shows bronchlcctasis. Five years later, the patiem complains of frequem diarrhea. ACT scan of the abdomen and pelvis shows diffuse thickening of the rectal wall, retroperitoneum, and mesentely. A rectal biopsy demonstrates amyloid that is most Ukely derived from which of the follo'Aring serum proteins? ***(A) Apo serum amyloid A ***(B) Calcitonin ***(C) Fibrinogen ***(D) Immunoglobulin light chain ***(E) f31 Microglobulin
6 The answer is A: Apo serum amyloid A. Chronic bronchitis and bronchiectasis are chronic inflammatory conditions that may lead to the deposition of AA amyloid. Termed secondary amyloidosis, this form of amyloidosis most often affects the kidneys, liver, adrenals, and spleen. Afih (derived from fibtinogen, choice C) is encountered in hereditary renal amyloidosis. None of the other choices are associated \vi.th chronic inflammation. ***Diagnosis: AA amyloid, bronchiectasis
7 For the patient described in Question 6 , progressive cell and organ dysfunction will develop primarily as a result of which of the following biological processes? ***(A) Accumulation of abnormal protein v,rithin lysosomes ***(B) Deposition of exiem;ive collagen scar tissue ***(C) Dimjnished blood supply to parenchymal cells ***(D) lmer ference with inflanunation and immuniry ***(E) Progressive DNA damage in parenchymal cells
7 The answer is C: Diminished blood supply to parenchymal cell.s. Regardless of type, amyloidosL'> leads evemually to cell atrophy and death. Amyloid adds interstitial material to sites of deposition, thereby increasing the size or affected organs. This increase may be counterbalanced by the deposilion of amyloid in blood vessels, an effect that impairs circulation and may lead lO organ atrophy and cell death. ***Diagnosis: AA amyloid, bronchiectasis
8 A 38-year-old woman complains of lingling sensations in her hands and fingers for the past year. Nemologic studies show loss of pain and temperature sensation in the patients extremities_ Laboratory analysis of cerebrospinal fluid shows no biochemical abnonnalilies_ A skin biopsy reveals amyloid deposits in nerves. Based on these findings and a stmng family history of a similar disorder, the pariem is diagnosed with familial amyloido[ic polyneuropathy Amyloid deposits in this patient's nerves are de1ived from which of the follovving serum prmeins7 ***(A) Apo semm amyloid A ***(B) Immunoglobulin light chain ***(C) P2-Microglobulin ***(D) f3 -Protein precursor ***(E) Transthyredn
8 The answer is E: Transthyretin. Familial amyloidotic polyneuropathy (FAP) is an autosomal dominant disorder that affects families in Sweden, Portugal, and Japan and is related to a mutation in the transthyretin gene, which encodes a thyroxine-binding protein. Amyloid in the peripheral and autonomic nerves is derived from the abnormal Lransthyretin. At least 60 different FAP mutations have been described, which give rise to clinical variants of the disease. The familial polyneuroparhic forms of amyloid usually manifest as paresthesias, with loss of temperaLUre and pain sensation of the extremities. ***Diagnosis: Familial amyloidotic polyneuropathy
9 A 74-year-old man with a history of progressive cogmuve impai rmem dies from complications of congestive heart fail ure. At the time of autopsy, his brain shows diffuse atrophy. Microscopic examination demonstrates numerous senile plaques and neurofibrillary tangles, as well as deposition of eosinophilic mate1ial in the walls of the cereb ral vasculature. A section is stained with Congo red and examined under polmized light (shown in the image)_ The birefringent material is derived from which of the following proteins? ***(A) Apo serum amyloid A ***(B) Fibrinogen ***(C) lmmunoglobuHn light chain ***(D) ~-Protein precursor ***(E) Transthyretin
9 The answer is D: P-Protein precursor. This patient suffers from Alzheimer disease, the pli11cipal cause of so-called senility. Amyloid deposits in the brain and cerebral vessels of patients suffering from Alzheimer disease contain fibrils of ~-amyloid (A~) . The deposi ted protein, namely A~ protein, is a fragment of a larger A~protein precursor (A~PP) , which is a normal cell membrane constituent. The larger part of A~PP is extracellular, with the remainder traversing the cell membrane and ending in a cytoplasmic portion. None of the other choices are found in Alzheimer disease. ***Diagnosis: Alzheimer disease
10 A 56-year-old woman dies of a chronic neurodegeneradve disease_ Autopsy reveals spongiform encephalopmhy with brain amyloidosis. This patients amyloidosis most likely belongs to which of the following categ01ies? ***(A) Aj:} ***(B) AA ***(C) AE ***(D) AH ***(E) APrP
10 The answer is E: APrP. APrP amyloid is found in spongiform encephalopathies. Prion proteins (PrPs) are namral plasma membrane constiruems found in a va1iety of cells, including the central nervous system. Their physiologic function is not yet apparent. PrP in an altered conformation serves as a template for the association of addition al PrP molecules and, in so doing, confers on them a new PrP conformation (PrPsc). PrPsc and its aggregates form fib1ils with the characteristic of amyloid and play a role in a group of human and animal central nervous system degenerative diseases, such as Kuru, Creutzfeldt-Jakob disease, Gerstman n-Straussler·Sheinker disease, scrapie, and bovine spongiform encephalopathy (mad cow disease). None of the other choices are seen in prion diseases. ***Diagnosis: Spongifonn encephalopathy
11 A 30-year-old woman with Down syndrome receives several rounds of aggressive chemotherapy for leukemia but subsequently dies of complications or septic shock. Immunohistochemical examination or this patients brain at autopsy would most likely show cerebral deposits of which of the following amyloid proteins? ***(A) A~ ***(B) AA ***(C) AL ***(D) ATTR ***(E) P2 -Microglobulin
11 The answer is A: A~ . Brains of middle-aged patien ts with ._____ Down syndrome show changes similar to those of Alzheimer disease. As in Alzheimer disease, the deposits of cerebral amyloid are derived from the A~ protein. The gene for the AP protein precursor resides on chromosome 21, uisomy of which causes Dow11 syndrome. None of the other ch oices are charactelistic of Down syndrome. ***Diagnosis: Down syndrome
12 A 52-year-old man complains of pain in his back and fatigue for 6 months. He admits w polymia and polydipsia. An X-ray film of the upper torso reveals numerous lytic lesions ln the lumbar vertebral bodies. Laboratory studies show hypoalbuminemia and mild anemia and thrombocytopenia. A monoclonal immunoglobulin peak is demonstrated by serum electrophoresis, and a bone marrow aspiration demonstrates numerous atypical plasma cell s. Urinalysis shows 4+ proteinuria. A renal biopsy in this patient would most likely show deposits of which of the following amyloid precursor proteins7 ***(A) Amylin ***(B) Apo serum amyloid A ***(C) Fibrinogen ***(D) Immunoglobulin light chain ***(E) f\-Microglobulin
12 The answer is D: Immunoglobulin light chajn. AL amyloid usually consists of the variable region of immunoglobulin light chains and can be derived from either the kappa (K) or lambda (A.) moieties. Since the light chains produced by the neoplastic cells in plasma cell dyscrasias are unique to each patiem, AL amyloid isolated from different persons differs in its amino acid sequence. Al protein is common w primary amyloidosis and amyloidosis associated with multiple myeloma, B-ceD lymphomas, or other plasma cell dyscrasias. Multiple myeloma js accompanied by amyloidosis in 10% w 15% of cases. The other choices do not involve inmmnoglobulins. ***Diagnosis: Multiple myeloma
13 A 56-year-old woman with a cold thyroid nodule is admitted for fine-need le aspiration, which results in a diagnosis of medullary carcinoma. Histologic examination of a subsequent surgical specimen reveals tumor cells surrounded by extracellular, amorphous eosinophilic material (shown in the image). These amyloid deposits are most closely related to which of the fo llo•ving serum proteins? ***(A) Apo semm amyloid A ***(B) Calcitonin ***(C) lmmunoglobuJjn light chain ***(D) ~1-Microglobulin ***(E) Transthyretin
13 The answer is 8: Calcitonin. lsolated amyloidosis has been described in the major arteries, lungs, hean, and various joints and in association with endocrine tumors that secrete polypeptide hormones. In endocrine tumors, the amyloid is usually part of a hormone or a prohormone. Medullary carcinoma or the thyroid originates from c cells of the thyroid, wh.ich normally secrete calcitonin. The amyloid found in this tumor is a fragment of procalcitonin. The other choices are not derived from endoc1ine cells. ***Diagnosis: Medullary carcinoma of the lhyroid
14 A 45-year-old man presems with flank pain and the passage of blood elms in his urine. Physical examination reveals bilateral flank and abdominal masses. Laboratory studies show elevated BUN and creatinine. U1inalysis discloses hematuria, proteinuria, and oliguria. A CT scan shows bilaterally enlarged, polycystic kidneys. The patient undergoes weekly hemodialysis for 5 years and develops severe joint pain. The deposition of which of the fo llowing amyloid proteins besl explains the pathogenesis of joim pain in this patient? ***(A) Amylin ***(B) Apo semm amyloid A ***(C) Fibtinogen ***(D) Immunoglobulin light chain ***(E) f31-Microglobulin
14 The answer is E: ~z·Microglobulin . The deposition of amyloid fanned from ~ 2-microglobulin is characterized by a destructive arthropathy due to amyloid deposition in the major joints of patients undergoing chronic renal dialysis. The other choices do not follow chronic renal dialysis. ***Diagnosis: Autosomal domi:nam polycysti.c kidney disease
15 A 65-year-old woman dies of trauma suffered in an automobile acddem. Examination of her pancreas at !he dme of autopsy reveals acellulaJ, eosinophilic matetial within the pancreatic islets. This material is congophilic and has a fibrillar appeamnce bye leco ·on microscopy. These findings suggest that p1ior LO her death, the patient suffered from which of the following conditions7 ***(A) Alcoholism ***(B) Chronic pancreatitis ***(C) Pancreatic cancer ***(D) Diabetes mellitus type l ***(E) Diabetes mellitus type 2
- - - - -./ - 15 The answer is E: Diabetes mellitus type 2. Type 2 diabetes is a heterogeneous disorder characterized by reduced tissue sensitivity to insulin and inadequate secretion of insulin from the pancreas. A variety of microscopic lesions are found in the islets of Langerhans of many patients with type 2 diabetes mellitus. Fibrous tissue may accumulate to such a degree that the islets are obHterated. Islet amyloid deposits are often presem, particularly in patients over 60 years of age. ***Diagnosis: Diabetes mellitus, type 2
16 The precursor of the pancreatic amyloid found in the patient descnbed in Question 15 helps regulate which of the following normal cellular funcdons7 ***(A) Apoptosis ***(B) DNA synthesis ***(C) Glucose uptake ***(D) Membrane fluidity ***(E) Protein glycosylation
16 The answer is C: GJucose uptake. The amyloid deposited in the islets of Langerhans in type 2 diabetes is derived from a larger precursor, a peptide related to a variant of calcitonin tem1ed islet amyloid polypeptide or arnylin. Like insulin, this honnone is produced by P-cells of the islets and seems to have a profound effect on glucose uptake by the liver and striated muscle. These observations suggest that islet amyloid may be involved in the pathogenesis of type 2 diabetes. Amy lin per se does not regulate any of the other choices. ***Diagnosis: Diabetes mellitus, type 2
1 A 55-year-old man from China presents with a 3-momh history of scales on his skin. Physical examination reveals numerous scaly, pigmented plaques. which rub o[ easily. Biopsy of a plaque shows anastomosing cords of marure and stratified squamous epithelium, associated with small keratln cysts. This paliem may have which of lhe following underlying conditions? ***(A) Acquired immunodeficiency ***(B) Basal cell nevus syndrome ***(C) Familial hypercholesterolemia ***(D) Human papillomavin.1s infection ***(E) Imernal malignancy
1 The answer is E: Internal malignancy. Seborrheic keratoses .._____ are scaly. frequently pigmented, elevated papules or plaques whose scales an~ easily mbbed off. Microscopically. the lesions are composed of broad anastomosing cords of mature stratified squamous epithelium associated with small cysts of keratin (hom cysts). The sudden appearance of numerous seborrheic keratoses has been associated \vith imernaJ malignancies (sign of Leser-Trelat), especially gasnic adenocarcinoma. Papillomavirus infection (choice D) induces papilloma formation. Choices B and D do not lead to acute plaque eruption. ***Diagnosis: Seborrheic keraroses
2 An 18-year-old woman notes that one ofher moles has increased ln size and become darker. The patient has a family history of melanoma, and she is seen by her dem1.atologist regularly to follow her moles. Physical examination reveals numerous, 5 to 10 mm, darkly pigmented, variegated lesions distributed p1imarily on her trunk but also involving non-sun-exposed skin. This patient may harbor a ge1mline mutation in a gene thaL regulates which of the following proteins? ***(A) Caspase ***(B) Cyclin-dependem kinase ***(C) Epidermal growth factor receptor ***(D) Glycogen phosphorylase ***(E) Sodium-potassium ATPase
2 The answer is 8: Cyclin-depenctent kinase. Germline mutations in the CDKN2A tumor suppressor gene (also known as p16) have been found in some patients who have dysplast ic nevus/melanoma and in their family members. This gene encodes an inhibitor of cyclin-dependent kinase that normally functions to suppress cell proliferation. Choices A, D, and E are not related to cell proliferation. Mmations in epidermal growth factor receptor (choice C) are involved in some malignant neoplasms but not dysplastic nevus/melanoma syndrome. ***Diagnosis: Dysplastic nevus syndrome
3 An 80-year-old fanner presents "Vith a 1-cm, red, slightly raised plaque on his face . A biopsy of d1e lesion shows cytologic atypia and dyskeratosis limited to the basal layers of tile sLramm spongiOsum, as well as hyperkeratosis and parakeratosis. This lesion is a precursor for which of the fo llm"ving dermawlogic diseases' ***(A) Basal cell carcinoma ***(B) Erythema multiforme ***(C) Lichen planus ***(D) Malignant melanoma ***(E) Squamous cell carcinoma
3 The answer is E: Squamous cell carcinoma. Actinic keratoses .....__ ("from the sun's rays") develop in sun-damaged skln as circumscribed keratotic patches or plaques, commonly on the backs of the hands or the face. Microscopically, the stratum corneum is replaced by a dense parakeratotic scale. The underlying basal keratinocytes display significant atypia. With time. actinic keratoses may evolve into squamous cell carcinoma in situ and, finally, into invasive squamous cell carcinoma. 1-lowever. most are stable and many regress. Basal cell carcinoma (choice A) and malignant melanoma (choice D) also arise frequemly m sun-exposed skin bm are unrelated to actinic kerawsis. ***Diagnosis: Actinic keratosis
4 A 45 ~year-old man presents with painful, purple nodules on the dorsal surface of his left hand that he first noticed 9 months ago. A biopsy (shown in the image) discloses a poorly demarcaled lesion composed of alypical spindle-shaped neoplastic cells and extravasated red cells. Si milar lesions are found in the lymph nodes and liver. Which of the following viruses is associated with the pathogenesis of these skin lesions? ***(A) Cytomegalovirus ***(B) Epstein-Ban virus ***(C) Herpes simplex virus lype 2 ***(D) Human herpesvirus type 8 ***(E) Human papilloma .. irus types 16/18
4 The answer is D: Human herpesvirus type 8. Kaposi sarcoma ......_ is a malignant tumor derived from endothelial cells. This vascular neoplasm is an imponant cutaneous sign in the AlDS pandemic. Human herpesvirus 8 (HHV-8) is thought 1.0 play a mle in the pathogenesis of Kaposi sarcoma, however only a small percentage of HliV-8-infected individuals develop Kaposi sarcoma. These malignant tumors most often appear on the hands or feet but may occur anywhere. The hiswlogic appearance of Kaposi sarcoma is highly variable. One form resembles a simple capillary hemangioma. Other fonns are highly cellular and vascular spaces are Jess promi.nenl. EpsteinBarr virus (choice B) is associated with Burkitt lymphoma in sub-Saharan Africa, and human papiJlomavirus types 16/18 (choice E) are associated with cervical ca11eer. ***Diagnosis: Kaposi sarcoma
5 A 36-year-old woman presents with a pigmented lesion on the .....__ posterior aspect of her left calf (shown in the image). An exci· sional biopsy demonstrates a superficial spreading type of melanoma. Which of the follo'wing histologic features has the most important prognostic value in your evaluation of this patient? ***(A) Degree of melanocytic atypia ***(B) Degree of vascularity ***(C) Deprh of dermal invasion ***(D) Extem of imraepide rmal invasion by melanoma cells ***(E) Presence of variable melanin pigmemarion
5 The answer is(: Depth of dermal invasion. Although choices A, B, and D may be factors in melanoma aggressiveness, tbe evaluation of tumor thickness is recognized as the single strongest prognostic variable for melanoma. Presence of variable melanin pigmemation (choice E) is not a predktor of melanoma growth or spread. ***Diagnosis: Melanoma
6 A 3-year-old child has numerous small, white scales coveting the extensor surfaces of 1he extremities, ttunk, and face (shmvn in the image). The child's mother and grandmother have the same condition. Biopsy of lesional skin shows hyperkerawsis and a thin stramm granulosum Whtch of the following is the most likely diagnosis? ***(A) Acute eczematous dermautis ***(B) Bullous pemphigoid ***(C) Eptdem1olys15 bullosa ***(D) Ichthyosis \'Ulgaris ***(E) Pemphtgus vulgaris
6 The answer is D: Ichthyosis vulgaris . Ichthyosis vulgaris is an autosomal dominant disorder of keratinization characteJized by mild hyperkeratosis and reduced or absent keratohyaline granules in tJ1e epidermis. Scaly skin results from increased cohesiveness of the stratum corneum. The anenuated stramm granulosum consists of a single layer with small defective kerawhyaline granules. Most patien ts are maintained free of scales with topical rreatments_ The mher choices do not feamre autosomal dominant inheritance. ***Diagnosis: Ichthyosis vulgaris
7 A 70-year-old woman presents with facial discoloration. The paucnt is observed to have a flat , pigmented lesion on the atrophtc, sun-damaged skin (shown in the image). Which of the fo llowing is the most likely diagnosis? ***(A) Acrallen tiginous melanoma ***(B) Alopecia ***(C) Lentigo maligna melanoma ***(D) Mucosallentiginous melanoma ***(E) Nodular melanoma
7 The answer is C: lentigo maligna melanoma. Lentigo maligna melanoma, also known as Hutchinson melanotic freckle, is a large, pigmented macule that occurs on stm-damaged skin. 1t develops almost exclusively in fair-s kinned, usually elderly, whites. Because it occurs on exposed body surfaces, it is probably related to chronic ultraviolet light exposure, without acute episodes of su nburn, and often occurs in outdoor worke rs. Acral lentiginous melanoma (choice A) is the most common fonn of melanoma in dark-skinned people and is generally llmited to the palms, soles, and subungual regions. A similar tl.tmor occurs on the mucous membranes and is called mucosal lentiginous melanoma (choice D). Alopecia (choice B) refers to hair loss. ***Diagnosis: Melanoma
8 A 25-year-old man presents with a 1-momh history of fa tigue, mild fever, and an ety Lhematous scaling rash. His major concem is related to the scaling plaques distribured on his knees, bmtocks, elbows, scalp, and fee t. He also nmes some joim pain and swelling, primari1y involving the small bones of his fingers. Physical examination reveals eJ)rthematous plaques with adherem silvery scales that induce punctate bleeding points when removed. Biopsy of Lesional skin would most Ukely show an accumulation of w~uch of the (oUowing cells in the epidemus? ***(A) B Lymphocytes ***(B) Mast cells ***(C) Melanocyres ***(D) Neutrophils ***(E) T Lymphocytes
8 The answer is D: Neutrophils. Psmiasis 1s a disease of the dermis and epidennis that is characterized by persistem epidermal hyperplasia. lt is a chronic, fTequemly familial disorder that featu res large, erythematous, scaly p laq ues, commonly on the dorsal extensor cutaneous surfaces. The nucleated layers of the epidermis are thickened several-fold in the rete pegs and are freq uemly thinner over the dennal papillae. The capillaries of the papillae are dtlated and ronu ous. Neutrophils emerge at their tips and migrate into the epidennis above the apices of the papillae Nemrophils may become localized in the epideJmal spinous layer or in small Munro microabscesses in the stratum corneum. The dem1is below the papillae exhibits a varying number of mononuclear inflammatory cells (choices A and E) around the super ficial vascular plexuses. ***Diagnosis: Psoriasis
9 Examination of a 2-day-old neonate reveals numerous blisters on the trunk and extremities. Skin biopsy discloses separation of the basal layer of the epidermis From its basement membrane and is devoid of inflammatory cells. No antibody depos· iLS are identified by immunofluorescence microscopy: vVhich of the following is the most likely diagnosis? ***(A) Bullous pemphigoid ***(B) Dermatitis herpetiformis ***(C) Epidermolysis bullosa ***(D) lchLhyosis vulgaris ***(E) Pemphigus vulgatis
.....9__ The answer is C: Epidermolysis bullosa (EB). EB comprises a heterogeneous group of disorders Loosely bound by their hereditmy narure and by a tendency ro fmm blisters at sites of minor trauma. EB simplex has been amibuted to mutations of genes encoding cytokeratin imermediate filamems. The clinical spectrum of the disease ranges from a minor annoyance to a widespread, life-threatening blistering clisease. These bUsters are almost always noted at binh or shortly thereafter. The classification of these disorders is based on the site of blister formation in the basement membrane zone. Although epidemlolytlc EB is cosmetically disturbing and sometimes debilitating, it is not life threatening. Blisters seen in bullous pemphigoid (choice A) and pemphigus w lgaris (choice E) are assodated with immunoglobulin deposits that are visualized by direct immunofluorescence microscopy. Dermatitis herpetiformis (choice B) occurs al a later age and is associated with gluten hypersensitivity. ln ich thyosis vulgaris (choice D), scaly skin results from increased cohesiveness of the stratum corneum. ***Diagnosis: Epidermolysis bullosa
tO A 25-year-old man complains of eruptions of blisters on his scalp and inner surface of the groin and in his mouth. The blisters rupture easily and leave large crusted areas. Histologically, the lesions show separation of the stratum spinosum from the basal layer. The results of direct immunofluorescence mlcroscopy for IgG are sho\\:n. Which of d1e rol!owing proteins is targeted by lgG autoantibody in the skin of this patient? ***(A) Collagen type rv ***(B) Desmoglein-3 ***(C) E-cadherin ***(D) FibronecLin ***(E) L-sclecun
10 The answer is B: Desmoglein-3. Pemphigus vulgatis is an autoimmune disease caused by autoantibodies to a keratinocyte antigen. The characteristic lesion is a large, easily mptured blister that leaves extensive denuded or cmsted areas. Suprabasal dyshesion resulrs in a blister that has an intact basal layer as a floor and Lhe remaining epidermis as a roof. The blister contains a moderate number of lymphocytes. macrophages, eosinophils, and neutrophils. Distinctive, rounded kermlnocytes (tenned acamholytic cells) are shed into the vesicle during the process of dyshesion. Circulating lgG antibodies in patients with pemphigus vulgaris react with an epidem1al surface andgen called desmoglein-3, a desmosomal protein. Antigen-antibody union results in dyshesion, which is augmemed by the release of plasminogen activawr and, hence , the activation of plasmin. This proteolytic enzyme acts on the imercellular substance and may be the dominant factor in dyshesion. None of the other choices are related to the pathogenesis of pemphigus vulga1is. ***Diagnosis: Pemphigus vulgaris
11 A 60-year-old fonner lifeguard presents with several small, pearly nodules on the back of her neck. A biopsy of one of the nodules (shown in the image) reveals buds of arypical, deeply-basophilic keratinocytes extending from the overlying epidermis imo the papillary dermis. Which of the following is the approptiate diagnosis' ***(A) Basal cell carcinoma ***(B) Fibroepithelial poln) ***(C) Kerawacamhoma ***(D) Squamous cell carcinoma ***(E) Xanthoma
11 The answer is A: Basal cell carcinoma (BCC). BCC is the most common malignant tumor in persons with pale skin. Although it may be locally aggressive, metastases are exceedingly rare. BCC usually develops on the sun-damaged skin of people >vith fair skin and freckles. The tumor is composed of nests of deeply basophilic epithelial cells with narrow 1ims of cytoplasm that are aLtached to tl1e epidermis and protmde into the subjacent papillary dermis. Basaloid keratlnocytes are rarely seen in squamous cell carcinoma (choice D) and are not encountered in the other choices. ***Diagnosis: Basal cell carcinoma
12 A 10-year-old girl presents with multiple papules on the back of her hand chat bleed easily. Histologic examination of a lesion reveals squamous epithelial-lined fronds with 6brovascular cores (shown in the image). Which of the following viruses is most likely responsible for the development of these skin lesions? ***(A) Cytomegalovirus ***(B) Epstein-Barr virus ***(C) Herpes simplex virus type 2 ***(D) Human herpesvirus type 8 ***(E) Human papillomavirus
12 The answer is E: Human papillomavirus. Verruca vulgaris, also known as the common wan , is an elevated papule with a vermcoLtS (papillomatous) surface. The wans may be single or multiple and are most frequent on the dorsal surfaces of t.he hands or on the [ace. Several human papiUomavuus types, including types 2 and 4, have been demortStrated in verruca vulgaris. No malignant potential is recognized . The other choices do not induce papillomas ***Diagnosis: Verruca vulga1is
13 A 28-year-old woman presents with a 2-day history of domeshaped erythemaLous nodules appeating on her skin. She recently had a minary tract infection for w·hich she was treated with trimethoprim sulfamethoxazole (Bactrim). Biopsy discloses focal hemonhage, neutrophllic infi.ltrates in the subcutaneous fibrous tissue septa, and giant cells at the interface between the septa and the adipose fat tissue. No infectious agems are identified. \Vhich of the following is the most Ukely diagnosis? ***(A) Allergic contact dermatitis ***(B) Dermatitis herpetifom1is ***(C) Bul lous pemphigoid ***(D) Erythema nodosum ***(E) Lupus erythematosus
13 The answer is D: Erythema nodosum (EN). EN is a cutaneous disorder that manifests as sel f-limited, nonsuppurative, tender nodules over the extensor surfaces or the lower extremuies. It is uiggered by exposure to a vatiery of agenrs, including drugs and microorganisms (bacteria, viruses, and fungi) , and occurs in association with a number of benign and malignant systemic diseases. The early neutrophilic innammation suggests that EN may he a response w the activation of complement , with resulting neutrophilic chemotaxis. The subsequent appearance of chronic infiammarion, foreign body giant cells, and fibrosis is secondary to necrosis of adipose tissue. The other choices do not feature this dis.linctive histology. ***Diagnosis: Erythema nodosum
14 A 65-year-old woman complains of having an itchy rash for the past few months. She said the lesions first appeared as red swollen plaques on her abdomen and flexor aspect of her foreanns. Physical examination reveals mticarial plaques, as well as large bullae on her abdomen and thighs (shown in the image). A skin biopsy shows a positive direct immunofluorescence test for IgG antibasement membrane antibody Which of the fo llovving is the appropriate diagnosis? ***(A) Allergic contact dermatitis ***(B) Bullous pemphigoid ***(C) Dem1alitis herpetiformis ***(D) Epidermolysis bullosa ***(E) Pemphigus vulgaris
14 The answer is B: Bullous pemphigoid (BP) . BP is a common, autoimmune, blis.tering disease with clinical similarities w pemphigus vuJgatis (rhus, the term pemphigoid) but in which acamholysis is absent. Complemem-fixing IgG antibodies are directed against two basement membrane proteins, BPAGl and BPAG2. In contrast ro pemphigus vulgaris (choice E), immunofiuorescent studies demonstrate linear deposition of C3 and lgG along the epidermal basemenr membrane zone. The other choices do not feam re antibasemem membrane antibodies. ***Diagnosis: Bullous pemphigoid
15 A 15-year-old girl complains of itchy skin lesions of 6 months in duration. Physical examination reveaJs numerous wheal-like lesions with small vesicles over her elbows and knees. A skin biopsy demonstrates inflammation in the tips of the dermal papillae and subepidermal vesicles. Which of the following histopathologic findings would provide the best evidence to support a diagnosis of dennatitis herpetiformis in this patiem? ***(A) Horn and pseudo-hom cysts ***(B) lgA deposits in dermal papiUae ***(C) Koilocytotic change ***(D) Microabscesses in the stratum corneum ***(E) Spongiosis
15 The answer is B: lgA deposits in dermal papillae. Dennaritis herpetiformis is an imensely pruritic cutaneous eruption related to gluten sensitivit)~ which is characterized by urticaria-like plaques and vesicles over the exrensor surfaces of the body. Genetically predisposed patients may develop lgA antibodies LO componems of glmen in the i.ruestines. The resulting lgA complexes then gain access to the circulation and are deposited in the skin. The release oflysosomal enzymes by inflammatory cells cleaves the epidermis from the demlis. The other choices are not typical histologic findings in dermatitis herpetiformis. ***Diagnosis: Dermatitis herpetiformis
16 A 17-year-old woman is brought to the physician by her parems because "she has been acting strangely" for a couple of days. Over the past 3 momhs, she has experienced malaise, joim pain, weight loss, and sporadic fever. The patient appears agitated, with a temperature of 38°C (101 on Other physical findings include malar rash, erythematous-pink plaques wah telangiectatic vessels, oral ulcers. and nonblanching purpmic papules on her legs. Laboratory studies show elevated levels of BUN and creatinine. The ami-double-stranded DNA antibody test is positive. Biopsy of sun-damaged lesional skin would most likely show which of d1e follmving histopathologic findings in this patient? ***(A) Acamhosis, parakeratosis, and neurrophils within the stratum con1eum ***(B) Early dennal-epidermal separalion medialed by eosinophits ***(C) Granular distribution of immune complexes in the basement membrane zone ***(D) Unear IgA deposits within dermal papillae ***(E) Stramm corneum microabscess (Munro)
16 The answer is C: Granular distribution of immune complexes in the basement membrane zone. The patiem exhibits signs and symptoms of systemic lupus erythematosus (SLE), a disorder characterized by a variety of aumantibodies and other i.mmune abnonnalities indicating B-cell hyperactivity. Epidermal cellular damage initiated by light or other exogenous agents causes the release o[ a targe number of antigens, some of which may retum to the skin in the form of Immune complexes. lmmune complexes are atso formed in the skin by a reaction of local DNA with antibody that may also be deposited beneath the epidermal basement membrane zone. The other choices are not feamres of SLE. ***Diagnosis: S)'stemk lupus erythematosus
17 A 14-year-old boy presents with a 6-month history of erythematous papules on his face. Physical exammation reveals numerous "blackheads" over the forehead and cheeks. Which of Lhe following bacteria is associated with the development of these lesions? ***(A) Closcndium sp. ***(B) Lacwbacl ll LL~ sp. ***(C) Propwnibactelittm sp. ***(D) SW[Jhylococcus sp. ***(E) Streptococws sp.
17 The answer is (: Propionibacterium sp. Acne vulgaris is a selflimited, innammatory disorder of the sebaceous follic les that typically afflicts adolescents, results in the intermittem formation of discrete papular or pustular lesions, and may Lead to scarring. The development of acne is related to excessive hotmonally induced production o[ sebum, abnom1al cornification of portions of the follicular epithelium, and an inflamn1atmy response to the anaerobic diphtheroid Propionibacterium (Knes. Choice B (Lactobacillus sp.) is not a pathogenic organism and choices A, D, and E usually represent acute bactetial infections. ***Diagnosis: Acne vulgaris
18 A 30-year-old man presents with rlaHopped papules that have appeared gradually on the flexor surfaces of his wrists. White streaks and patches are also found on the buccal mucosa of the patient's mouth. Histologically. the lesions showed hyperkeratosis, thickening of the stratum granulosum, and a band· like infiltrate of lymphocytes and macrophages in the upper dermis, disrupting the basal layer of the epidermis. Lymphocytes were mostly of the CD4· immunophenotype. Which of the following is the appropriate diagnosis? ***(A) Dennatilis herpetifomlis ***(B) Erythema multifom1e ***(C) Erythema nodosum ***(D) H}rpersensitivity angiitis ***(E) Uchen planus
18 The answer is E: Li(hen planus. Lichen planus is a hypersensitivity reaction \•lith Lymphocytic infiltrates at the dermal-epide rmal Junction. The disease is apparently initiated by epidermal inju ry. This injury causes some epidermal cells to be treated as "foreign.'' The antigens of such cells are processed by Langerhans cells. The processed antigen induces local macrophage activation and lymphocytic proliferation. Macrophages and T lymphocytes disrupt the strarum basalis resulting in reactive epidennal proliferation (hyperkeratosis). The skin displays multiple, flat-topped, violaceous, polygonal papules. The site of pat.hologic injury is at the dem1al-epidermal junedon, where there is a striking infiltrate of lymphocytes, many of which surround apopwtic keratinocytes. These histologic features are not observed in the other choices. ***Diagnosis: Lichen planus
19 A 66-year-old woman presents with a 5-year history of erythematous, scaly patches on her buttocks. Physical examination reveals plaques with telangiectases, atrophy, and pigmentation. Biopsy of lesional skin shows that the epide1mis and papillary dermis a re expanded by an extensive infiltrate of atypical lymphocytes. These infiltrating lymphocytes most likely express which of the following "cluster of differentiadon" cell surface markers? ***(A) CD4 ***(B) CD9 ***(C) CDlS ***(D) CD20 ***(E) CD3l
19 The answer is A: CD4. Mycosis fungoides (MF) is a variant of cutaneous T-celllymphoma. The most important histologic feature of :MF i.s Lhe presence of lymphocytes in the epidermis (epidennotropism). In late stages, the dermal infiltrat.e becomes dense to the point of fmming tumor nodules. Sezary syndrome refers to the systemic dissemination of MF Cluster of differenLiation (CD) antigens are cell surface molecules that serve as useful markers of cellular identity Cunently some 300 differem molecules have been assigned CD numbers. CD4 is a useful marker of helper T lyn1phocytes. None of the other choices are T-cell markers. ***Diagnosis: Mycosis fungoides
20 A 30-year-old woman with chronic hepatitis B presents wiLh numerous red sldn lesions thaL she has had for 5 days. Physical examination reveals multiple, purpuric, 2- to 4-mm papules on the sldn (shown in the image). The papules did noL blanch under pressure. Biopsy of ]esional skin shows necroLizing leukocytodasLic venulltis. Immunofluorescence sLUdies disclose immune complex deposition in vascular walls . Which of the fo llowing is the most likely diagnosis? ***(A) Allergic contacl dcnnamls ***(B) Dennamis hcrpcufonnis ***(C) Erythema multifom1e ***(D) Er)'thema nodosum ***(E) Hypersensill\'11)' angiitiS
20 The answer is E: Hypersensitivity angiitis. Cutaneous necrotizing vasculitis (CNV) presents as "palpable purpura" and has also been called allergic cutaneous vasculitis, leukocytoclastic vasculitis, and hypersensitivity angiitis. Circulating immune complexes are deposited in vascular walls. The elaborated C5a complement component amacts neunophils, which degranulate and release lysosomal enzymes, resulting in endothelial damage and fibrin deposition. CNV may be primary, without a known precipitating event in abom half of the cases, or associated with a specific infectious agent (e.g., hepatitis B virus) It may also be a secondary process in a variety of chronic diseases (e.g .. ulcerative colitis). Allergic contact dermatitis (choice A) is associated with external comaCL with an allergen (e.g., poison ivy) and dermatitis herpetifmmis (choice B) is secondaty LO gluten sensitivity. Erythema multifonne (choice C) is an immune complex disease associated with a drug reaction that is histologically different than CNV E1ythema nodoswn (choice D) does not feature the deposition of immune complexes. ***Diagnosis: Hypersensitivity angiitis, cutaneous necroliz]ng vasculitis
Z1 A 20-)'ear-old man presents 10 his family physician for treatment of ttchmg af1er exposure 10 poison hT The pauents hands and arms appeared red and were covered with oozmg blisters and crusts (shown m the Image). \Vhich of the following represents the most tmponant step m the pathogenesis of the sens1t1zanon phase of mjury in this patient? ***(A) Development of spongiosis ***(B) Infiltradon of the epidennis by neutrophj]s ***(C) Migration of Langerhans cells imo dennallymphatics ***(D) Rapid increase in vascular permeability ***(E) $eparaLion of the epidermis from Lhe dermis mediated by eosinophils
21 The answer is C: Migration of Langerhans cells into dermal lymphatics. Allergic contact dermatitis is a model of spongiotic dermatitis, a reaction pattern in which there ts edema in the epidermis. ln the initial 2 4 hours following ree.>rposure w the offending plam (elicitation phase), numerous lymphocytes and macrophages accumulate abom the superficial venular bed and extend into the epidermis. The epidermal keratinocytes are partially separated by the edema fluid, creating a spongelike appearance (spongiosis). During the sensili.zation phase, low molecular weight haptens combine with canier proteins at the cell membrane of Langerhans cells. These inflammawry cells carry processed antigen through the lymphatics to regional lymph nodes and present it to CD4• T lymphocytes. Sensitized T cells then migrate back into the epidermis. CyEOkine production leads to the accumulation of more T cells and macrophages and to epidennal cell injury. ***Diagnosis: Allergic contact dermatitis
22 A 12-year-old girl presents for a routine physical examination. The patient has numerous freckles over her upper tnmk and face . Which 0 r the [o Uov.,ring Lenns best describes the morphologic appearance of her fTeckles? ***(A) Comedo ***(B) Macule ***(C) Nodule ***(D) Papule ***(E) Plaque
22 The answer is B: Macule. Freckles, or ephelides, are small, brown macules that occur on sun-exposed skin, especiaJly in people with fair skin. Freckles usually appear at about age 5 years. The pigmentation of a freckle deeperts with exposure to sunlight and fades when light exposure ceases. Freckles show hyperpigmentation of the basal keradnocytes without a concomitant increase in the number of melanocytes. The other choices represent raised lesions of the skin. ***Diagnosis: Freckles, ephelides
23 A 40-year-old woman complains that the skin on her fi.nge rs feels stiff. The skin of her face appears tense, and radial furrows are evidem around the momh. A skin biopsy shows loss of dermal appendages and abundant coHagen bundles aligned parallel to an atrophic epidermis. Which of Lhe following clinical symptoms is commonly seen in patiems with this dem1atologic condition? ***(A) Facial acne ***(B) Dysphagia ***(C) Fever and malaise ***(D) Polyu1ia and polydipsia ***(E) Urolithiasis
23 The answer is 8: Dysphagia. Scleroderma is marked by fibrosis and tightening of t.he skin. The disease also displays variable strucrural and functional involvemem of internal organs, including the kidneys, lungs, heart, esophagus, and small imestine. Dysphagia due 1.0 lnvolvemem of the esophagus is common. Patients with early scleroderma usually present with Raynaud phenomenon or non pitting edema of the hands or fingers. The affected areas become hard and tense. ln lale s tages of the disease, the skin over large parts of the body is thickened, densely fibrotic, and fixed to the underlying tissue. Although scleroderma can affect the kidney, polyuria/polydipsia (choice D) is most. commonly a complication of diabetes. The other choices arc not related to scleroderma. ***Diagnosis: Sclerodenna
1 A 6-year-old boy presems with painful sores on his upper lip. He was seen for a flu 1 week ago. The lesion appears as 0.2- to 0.4-cm vesicles w1th focal ulceration. Which of the following is the most likely histologic feature of this skin lesion' ***(A) Acute arte ritis ***(B) Caseactng granulomas ***(C) Fun gal hyphae ***(D) Multinucleated epithelial cells ***(E) Noncaseating granulomas
1 The answer is D: Multinucleated epitheljal cells. Herpes labialis .____ (cold sores, fever bliste rs) and herpetic stomatitis are caused by hetves virus type 1. They are among the most common viral infections of the lips and oral mucosa in both children and }'OLmg adults. The disease starts -.vith painful inflammation of the affected mucosa, followed shortly by the fom1ation of vesicles. These vesicles rupture and form shallow, painful ulcers. Microscopically, the herpetic vesicle forms as a result of ballooning degeneration of the epithelial cells. Some epithelial cells show intranuclear inclusion bodies. At the edge of the ulcer are large, multinuc1eated, epithelial cells with "ground glass" homogenized nuclei, often exhibiting nuclear molding. The ulcers heal spontaneously without scar fonnadon. Acute aneritis (choice A) does not cause the described lesions. Choices B, C, and E do not represent acute vesicular lesions. ***Diagnosis: Herpes labialis
2 A 74-year-old man with a 15-year history of diabetes is hospitalized because of end-stage kidney disease. His tongue, inner side of the lips, and b1.1ecal mucosa are covered 'W'i Lh white , slightly elevated, soft patches (shown in the image). What is Lhe most likely diagnosis? ***(A) Aphthous SlOtnatitis ***(B) Candidiasis ***(C) Herpes labialis ***(D) Pyogenic granuloma ***(E) Xerostomia
Z The answer is B: Candidiasis. Also termed thrush or moniliasis, candidiasis is caused by a yeast-like fungus, Candida albrcam;, which is a common surface inhabitant of the oral cavity, gasu·oimestinal tract, and vagina. Oral candidiasis is most common in people with immunocompromised systems or with diabetes, and the incidence in patients with AlDS is 40% to 90%. The oral lesions rypically appear as white, slightly elevated, soft patches that consist mainly of fungal hyphae. Choices A, C, and D are generally focal lesions. ***Diagnosis: Candidiasis
3 A 40-year-old woman is hospitalized because of a massive neck infection that developed over a period of 3 days after exttacdon of an impacted wisdom toot h. She has a high fever, and her lower jaw and entire neck are S\•vollen, red, and paLnful. Throat cuiLure reveals a mi...xed bacterial flora, containing both aerobic and an aerobic microorganisms. Wl1ich of the following is the most likely diagnosis? ***(A) Actinomycosis ***(B) Acme necrotizing ulcerative gingivitis ***(C) Ludwig angina ***(D) Pyogenic granuloma ***(E) Scarlet fever
3 The answer is C: Ludwig angina. Ludwig angina is a rapidly spreading cellulitis, or phlegmon, which otiginates in the submaxillary or sublingual space but e:-:tends locally to involve both. The bacteria responsible for lhis infection originate from the oral flo ra, butt his potentially life-threatening inOammatory process is uncommon in developed countries. After extraction of a tooth, hairUne fTactures may occur in the lingual conex of the mandible, providing microorganisms ready access to the submaxillary space. By following the fascial planes, the infection may dissect into the parapharyngeal space and, from there, into the carotid sheath. Actinomycosis (choice A) features branched, filamentous baCLeria. Acute necrotizing ulcerative gingivitis (Vincent angina, choke B) does not e:<tend to the neck Pyogenic granuloma (choice D) is a focal, reactive vascular lesion. ***Diagnosis: Ludwig angina
4 A 45-year-old woman presents with a 1-year history of dry mouth and eyes. Physical examination reveals bilateral enlargement of the parotid glands . A biopsy (shown in the image) discloses infiltrates o£ lymphocytes fanning focal gemrinal centers. Which of the following best describes changes that would be expected in this patients parotid glands, late in the course o£ her disease? ***(A) Atrophy ***(B) Dysplasia ***(C) Hyperplasia ***(D) Hypenrophy ***(E) Metaplasia
4 The answer is A: Atrophy. Sjogren syndrome is a chronic inDammatory disease of the salivary and lacrimal glands; it may be restricted to these sites or may be associated with a systemic collagen vascular disease. Involvement of the salivaty glands leads to dry mouth (xerostomia), and disease of the lactimal glands results in dry eyes (keratoconjunCLivitis sicca). Late in the course of the disease , the affected glands become atrophic, with fibrosis and fatty infiltration of the parenchyma. Dysplasia (choice B) is not feamred in Sjogren syndrome_ ***Diagnosis: Sjogren syndrome
5 A 16-year-old girl presents with a sore throat of 4 months in duration. She describes fee ling a lump in her throaL Physical examination reveals a l-cm cystic lesion at the base of the tongue. This developmemal Lesion most likely arises as a remnam of which of the fo llowing anatomic structures7 ***(A) Auditory tube ***(B) Branchial arches ***(C) Facial fus ion lines ***(D) Rudimemary thymus ***(E) Thyroglossal duct
5 The answer is E: Thyroglossal duct. During its normal devel- .__ opmem, the thyroid gland descends from the base of the tongue to its final position in the neck. Heterotopic h.mctioning thyroid tissue or a developmental cyst (thyroglossal duct cyst) may occur anywhere along the path of descent. The most common location is at the foramen cecum of the tongue. Symptoms such as dysphonia, sore throat, and awareness of a mass in the throat often become evident dming adolescence and pregnancy. The other choices do not present in this anatomic location. ln particular, branched cleft cysts that miginate from remnams of the branchial arches (choice B) occur in the lateral ame1ior aspect of the neck or in the parotid gland. ***Diagnosis: Thyroglossal dueL cysL
6 A 2-year-old girl "vas withdrawn from a day care center for excessive irritability. On physical examination, she has multiple , small superficial ulcers of the oral mucosa. The ulcerations heal spontaneously over the next 5 days. Which of the following is the most likely cUagnosis7 ***(A) Aphthous sLOmatitis ***(B) Candidiasis ***(C) Gingivitis ***(D) Ludwig angina ***(E) Pyogenic granuloma
6 The answer is A: Aphthous stom·atitis. Aphthous stomatitis describes a common disease that is charactetized by painful, recurrent , solitary or mulliple, small ulcers of the oral mucosa. The causative agent is unknovvn. Microscopically, the lesion consists of a shallow ulcer covered by a fibrinopumlent e.,'-'ttdate. Candidiasis (choice B) features white plaques. ***Diagnosis: Aphthous stomatitis
7 A 6-year-old boy presents with a painful sore in his momb. Physical examination reveals a small, elevated, and focal ly ulcerated red-purple gingival lesion. A soft red mass measuring 1 em in diameter is surgically removed. Hisrologic examination discloses highly vascular granulation tissue , with marked acute and clu-onic lnnammation . What is the most likely diagnosis? ***(A) Acute necrotizing gingivitis ***(B) Aphthous stomatitis ***(C) Herpes labialis ***(D) Pyogenic granuloma ***(E) Tuberculosis
.....7__ The answer is D: Pyogenic granuloma. Pyogenic granuloma ls a reactive vascular lesion that commonly occurs in the oral cavity Usually some minor trauma to the tissues permits invasion of nonspecific microorganisms. In the oral cavity, pyogenic granulomas, ranging from a few millimeters to a centimeter, are most frequent on the gingiva. The lesion is seen as an elevated, red or purple, soft mass, wiLh a smooth, lobulated, ulcerated surface. Microscopically, the nodule consists of highly vascular granulation Lissue that shows varying degrees of acute and chronic inflammation . Vv1Lh time , pyogenic granuloma becomes less vascular and comes to resemble a fibroma. Choices A, B, and Care ulcerating lesions. Tuberculosis (choice E) features granulomatous inflammation. ***Diagnosis: Pyogenic granuloma
8 A 60-year-old man, who is a chronic alcoholic, is refe rred by a homeless shelter for facial ulcers. On physical examination, there are large ulcerations in the mouth and facial tissues with focal exposure of bone. A culture grows Borrelia vincenlii Which of the following is the most likely diagnosis? ***(A) Acute necrotizing ulcerative gingivitis ***(B) Apical granuloma ***(C) Ludwig angina ***(D) Periodontitis ***(E) Pyogenic granuloma
8 The answer is A: Acute necrotizing. ulcerative gingivitis. Acute necrotizing ulcerative gingivitis (Vincent angina) represents an infection by t\Vo symbiotic organisms; one is a fusiform bacillus, and the other is a spirochete (B. VII1W1tii). The fact that these organisms are found in the mouths o[ 1nany healthy persons suggests that predLsposing factors are important in the development of acute necrotizing ulcerative gingivitis. The most imponam element appears to be decreased resistance to infection as a result of inadequate nutrition, immunodeficiency, or poor oral hygiene. Vincem infection is charaCLeJized by punched-om erosions of the interdental papillae The ulceration tends to spread and eventually to involve all gingival margins, which become covered by a necrotic pseudomembrane. None of the other choices are desn-uctive, ulcerating lesions. ***Diagnosis: Necrotizing ulcerative gingivitis
9 A 30-year-old man was told by his dentist that his left upper wisdom tooth showed some decay. Even though the decay was still reparable, tooth extraction was recommended. A whole mount section through the extracted tooth is shovm in the image. What is the approp riate diagnosis7 ***(A) Apical granuloma ***(B) Dental caries ***(C) Petiapical abscess ***(D) Petiodomitis ***(E) Pulpitis
9 The answer is 8: Dental caries. Caries is the most prevalent chronic disease of the calcified tissues of the teetl1. Caries begins with the disintegration of the enamel prisms after decalcification of the imerprismatic substance; these evems lead w the accumulation of debris and microorganisms. These changes produce a small pit or fissure in the enamel. 'vVhen the process reaches the dentinoenamel junction, iL spreads laterally and also penetrates the dentin along the dentinal tubules. A substantial cavity then forms in the dentin, producing a flaskshaped lesion with a narrow orifice. Only when the vascular pulp of the woth is invaded does an inflammatory reaction (pulpitis) appear (choice E). Apical granuloma (choice A), the most common sequel of pulpitis, is the (onnation of chronically inflamed periapical granulation tissue. Periapical abscess (choice C) is also a result of pulpitis. Pe1iodomitis (choice D) is an inflammatory condition of the marginal gingiva. ***Diagnosis: Dental caries
10 A 4-year-old boy from Uganda presents for pre-school physical examination_ His parems indicaLe that hearing loss in his right ear developed 3 yea rs ago following an acute illness. If this childs hearing loss was caused by a viral infection, which of the following was the most likely etiology? ***(A) Chickenpox ***(B) Epstein- Barr virus ***(C) Mumps ***(D) Rubella ***(E) Yellow fever
10 The answer is C: Mumps. Mumps is the most common cause of deafness among the postnatal viral infenions. The infection can cause rapid hearing loss, which is unilateral in 80% of cases. By contrast, prenatal infection of the labyrinth with mbella (choice D) is usually bllateral, wtth pem1anem loss of cochlear and vestibular function. A number of other vimses are suspected w cause labyrimhitis, including influenza and parainfluenza viruses, Epstein-Barr virus (choice B), herpesviruses, and adenoviruses. Temporal bone specimens of such cases reveal severe damage w the organ of Coni, with almost wtalloss of both inner and outer hair cells. ***Diagnosis: Mumps
11 An 82-year-old man presents wiLh painless swelling below the right ear that has been growing s lowly for about a year. Physical examination con fim15 a cystic lesion. Fine-needle aspiration returns scanty brown fluid , which microscopically consists of a mLxture of lymphoid cells and epithelial cells with abundant granular eosinophilic cytoplasm. A biopsy of the mass is shown in the image. Which of the following is the most likely diagnosis? ***(A) Acinic cell carcinoma ***(B) Adenoid cystic carcinoma ***(C) Chronic abscess ***(D) Pleomorphic adenoma ***(E) Watthin tumor
1 t The answer is E: Warth in tumor. Wanhin tumor is a benign neoplasm of the parotid gland, composed of cystic glandular spaces embedded in dense lymphoid tissue. This tumor is the most common monomorphic adenoma. Although the neoplasm is clearly benign, it can be bilateral (15% of cases) or mulrifocal within the same gland. Warthin tumor is the only rumor of the salivary glm1ds that is more common in men than in women. These lesions generally occur after the age of 30, with most arising after age 50 years. Pleomorphic adenoma (choice D) has a biphasic appearance, which represents an admixru re of epithelia] and stromal elements. Acinic cell cardnoma (choice A) is an uncommon malignancy. Adenoid cystic carcinoma (choice B) is a slmv-growing maUgnam neoplasm of the salival)' gland, which invades locaUy and tends w recur after surgery. ***Diagnosis: Wanhin tumor, adenolymphoma
12 A 64-year-old man presents with sores on the gums. Oral examination shows muhiple, gingival abscesses adjacenr to the teeth and excessive mobility of several teeth. Which of the following is the most likely diagnosis? ***(A) Dental caries ***(B) Hyperparathyroidism ***(C) Ostei[is deformans ***(D) Petiapical granuloma ***(E) Petiodomal disease
12 The answer is E: Periodontal disease. Periodontal disease refers to acute and chronic disorders of the soft tissues surrounding the teeth, whlch eventually lead to the loss of supporting bone. Chronic periodontal disease typically occurs in adults, particularly in persons with poor oral hygiene. However, many persons \Vith apparemly impeccable habits, but a strong family history of periodontal disease, man.lfesL the disorder. Chronic penodontitis causes loss of more teeth in adullS than does any other disease, including caties. Periapical granuloma (choice D) is the most common sequel to pL1lpitis and represents chronically inO.arned petiaplcal granulation tissue. ***Diagnosis: Periodontal disease
13 A 33-year-old woman presenlS with a painful gingival lesion. Oral examination shows a l -cm swelling on the lower right gingiva. A biopsy is taken (shown in the image). Which of the following is the most likely diagnosis7 ***(A) Ameloblastoma ***(B) Odontogenic q•st ***(C) Petipheral giant-cell granuloma ***(D) Radicular cyst ***(E) Squamo1..1s ce II carcinoma
13 The answer is C: Peripheral giant-cell granuloma. Peripheral giant-cell granuloma is an unusual proliferative reaction to local injury that is seen as a mass on the gingiva or the alveolar process. The adjective "peripheral'' denotes the superficial, extraosseous location of the lesion, as opposed LO the "central" giant-cell granulomas that occur within the jawbones. Pe1ipheral giant-cell granuloma is seen as a mass covered by mucous membrane, which can be ulcerated. Hiswlogical examination reveals a nonencapsulated lesion with numerous multmudeated giant cells embedded ln a fibrous stroma that also comains ovoid or spindle-shaped mesenchymal cells. The other choices do not typically feature multinucleated giant cells. ***Diagnosis: Petipheral giam-cell granuloma
14 A 26-year-old football player complains of persistent nasal blockage, nmny nose, and headache. Multiple nasal polyps are identified and resected (shown in the image). Hismlogical examination of the polyps shows that they are benign. Which of the following is the most likely cause of polyps in this patient? ***(A) Acute suppurative otitis media ***(B) Acute wnsillitis ***(C) Acute viral rhin itis ***(D) Chronic allergic rhinitis ***(E) Chronic otitis media
14 The answer is D: Chronic allergic rhinitis. Sinonasal inOammatory polyps are nonneoplastic lesions of the mucosa. Most pol· yps arise from the lateral nasal wall or the ethmoid recess. They may be unilateral or bilate ral and single or multiple. Symptoms include nasal obstruction, rhinorrhea, and h eadaches. The etiology involves multiple factors, including allergy, cystic fibrosis, infections, diabetes mellitus, and aspirin intolerance. Microscopically, sinonasal allergic polyps are lined externa11y by respiratory epith elium and con tain mucous glands withi11 a loose mucoid stroma, which is infiltrated by plasma cells, lymphocytes, and numerous eosinophils. Neither acute tonsillitis (choice B) nor acute viral rhinitis (choice C) leads to nasal polyps. ***Diagnosis: Nasal polyps
15 A 55-year-old man presents with recurrent nosebleeds. He complains of persistent rum1y nose, fever, malaise, and a 20-lb (9-kg) weight loss. Physical examination reveals a "saddle nose" ddom1ity (patient shown in the image). An X-ray film of the chest shmvs patchy infiltrates. Urinalysis discloses 2+ hematuria. Antibodies directed w which of the following cellular sn·uctures would be e."pec ted in the serum of this patient? ***(A) Epithelial nuclei ***(B) MitOchondria ***(C) Nemrophil cywplasm ***(D) Skeletal muscle ***(E) SmoOLh muscle
15 The answer is C: Neutrophil cytoplasm. Wegener granulomatosis is a disease of unknown origin that shares some features with lethal midline granuloma. Both diseases are characterized by necrotizing, ulcerated, mucosal lesions. Lethal midline granuloma is a sign of an underlying lymphoid malignanc)~ whereas Wegener granulomawsis is an inflanm1awry disease. Evidence points to an autoimmune etiology for Wegener granulomatOsis. In most instances the lesions are not limited w the upper respiratory tract; they also involve the lungs and the kidneys. More than 90% of patients with Wegener granulomawsis exhibit amineutrophil cytoplasrnic antibody (ANCA); of these patients, 75% have C-ANCA. Antibodies directed against the other choices are not associated with the clinical syndrome described. ***Diagnosis: Wegener granulomatosis
16 A 60-year-old woman presents with a 5-month history of "sinus pressure." A 2-cm firm lesion is identified in the tight lateral nasal wall. A CT scan reveals involvement of the adjacent paranasal sinus. A biopsy (shown in the image) displays epithelial nests protruding into the submucosa with unifo rm cellular proliferation and no atypia. Vvhich of the following etiologic agents is most likely assodated with the development of this lesion? ***(A) Cytomegalovirus ***(B) .Epstein-Ban virus ***(C) Herpes simplex virus Lype 1 ***(D) Human papillomavirus !:)'pes 6 and ll ***(E) Toxoplasma gondii
16 The answer is D: Human papillomavirus types 6 and 11. Papillomas are the most common rumors of the nasal cavity. Human papillomavims types 6 and 11 cause papillomas, which may be hiswlogically diagnosed as either squamous or invened. Epstein-Ban virus infection (choice B) is related w nasopharyngeal carcinoma. ***Diagnosis: Sinonasal inverted papilloma
17 An 8-year-old boy presents with a 3-day history of fever and sore throat. His temperature is 38"C (1 Ol °F), pulse is 88 per minute, and respirations a re 33 per mil1ute. On physicaJ examination, the tonsUs are enlarged, boggy, and coated with a purulent exudate. A tonsillar swab is most likely to grow whkh of the follO\ving microorganisms? ***(A) Sraplly lococcus at~reus ***(B) Sraplly lococcus epidenmdiS ***(C) Streptococcus pncumoniae ***(D) Screptococws pyogenes ***(E) Streptococcus vrtidattS
17 The answer is D: Streptococcus pyogenes. Acute tonsillitis may be caused by bacterial or viral infections. Streptococcus pyogenes (group A f3-hemolytic streptococci, choice D) is the most common etiologic agent in acute suppurative wnsillitis. ***Diagnosis: Tonsillitis, acute
18 A 70-year-old woman complains of gradual hearing loss. Which of the following conditions is the most likely cause of conducting hearing loss in this patiem? ***(A) Acoustic trauma ***(B) Chronic otitis media ***(C) Labyrinthine LOXkity ***(D) Mastoiditis ***(E) Owsclerosis
18 The answer is E: Otosclerosis. Otosclerosis refers to the forn'lation of new spongy bone aboUL the stapes and the oval window, which results m progressive deafness. Thts condition is an auwsomal dominam hereditary defect and is the most conm1on cause of conductive hearing loss in young and middle-aged adults in the United States. Ten percent of white and 1% of black adult Americans have some otosclerosis, allhough 90% of cases are asymptomatic The other chokes are much less common causes of hearing loss. ***Diagnosis: Owsclerosis
19 A 24-year-old man presents with right ear pain that he has had for 3 days. He recemly returned from a scuba diving trip to the Caribbean. The patient is afebrile. Otoscopic examination reveals a bulging, right tympanic membrane. What is the most likely diagnosis? ***(A) Acute mastol<litis ***(B) Acme rlunitis ***(C) Acute serous otitis media ***(D) Acute sinusitis ***(E) Acltle suppmative OLitis media
19 The answer is C: Acute serous otitis media. Obstmction of the eustachian tube may result from sudden changes in atmospheric pressure (e.g., during Gying in an aircraft or deepsea diving). This effect is particularly severe in the presence of an upper respiratory tract infection, an acme anergic reaction, or viral or bacterial infection at the otifice of the eustachian rube. lnflammation may also occur without bacterial invasion of the middle ear. More than hal [ of children in the United States have had at least one episode of serous otitis media before their third birthday. lt has become increasingly evident that repeated boLUs of otitis media in early chtldhood often conmbure w unsuspected hearing loss, which ts due ro residual (usually sterile) Quid in the middle ear. Acme suppurative otitis media (choice E) is unlikely without fever. ***Diagnosis: Serous otitis media, acute
20 A 33-year-old man from China presents with a lump in his neck Physical examination reveals painless, anterior cervical adenopathy. A large necroti.zi.ng mass is identified in the posterior nasopharynx , with obstruction of both eustaduan mbes. A biopsy of the mass reveals sheets of malignam cells with la rge nuclei (sho~'n in the image). Which of the following pathogens has been associated with the development of this patients neoplasm? 'I ..... -:IIi· . ..,. ,. " ..II''.-·~··· .,. ...... ,.. ***(A) Cywmegalovirus ***(B) Epstem-Barr virus ***(C) Herpes simple.." v1rus type 2 ***(D) Human immunodefiCJtmcy VJrus ***(E) Human T-cell lymphoma/leukemia virus
20 The answer is 8: Epstein-Barr virus. Nasopharyngeal carcinoma is an epithelial cancer of the nasopharynx that is dassiJ1ed into keratinizing and nonkeratinizing subtypes. Nortkeratinizing nasophmyngeal carcinoma is associated with EBV infection. Most patiems have and-EBV lgA in their serum. Both differentiated and undifferentiated nonkeratinizing nasopharyngeal carcinomas are immunoreactive with antibodies to cytokeratins. Choices A, C, and D are noncardnogenic vimses. Choice E does not cause malignamtransformation of epithelial cells. ***Diagnosis: Nasopharyngeal carcinoma
21 A 50-year-old woman presenls witJ1 a slowly enlarging, painful mass at the angle of the right Jaw. A biopsy of the mass is shown in the image. Which of the following is the most likely diagnosis? ***(A) Adnic cell carcinoma ***(B) Adenoid cystk carcinoma ***(C) Mahgnam mixed rumor ***(D) Malignant ODCO{:ytOma ***(E) Mucoepidermoid carcinoma
21 The answer is B: Adenoid cystic carcinoma. Adenoid cystic carcinoma is a slowly growing malignant neoplasm of the salivary gland that is notorious for its tendency to invade locally and to recur after surgical resection. The tumor cells are small, have scant cytoplasm, and grow in solid sheets or as small groups, strands, or columns. \.vlthin these structures, Lhe tumor cells interconnect to enclose cystic spaces, resulting in a solid, tubular or cribriform (sie\re-like) arrangement. The other choices do not exhibit the typical cribrifonn pattern ***Diagnosis: Adenoid cystic carcinoma
22 A 34-year-old man complains of hearing loss. He has had multiple bouts of ear infections over the last 2.0 years and was recently diagnosed with chronic suppurative otitis media. Which of the following is the most likely complication of this condition in lhis patient? ***(A) Acoustlc neuroma ***(B) Cholesteatoma ***(C) Chron ic rhinitis ***(D) Chronic sinusitis ***(E) Squamous ce 1l carcinoma
22 The answer is B: Cholesteatoma. Cholesteatoma is a complication or chronic suppurative otitis and a rupture or the ear· drum. Cholesteatoma is a mass of accumulated keratin and squamous mucosa that results from the growth of squamous epithelium fTom the external ear canal through the perfonued eardrum into the m1ddle ear. Microscopically, cholesteatomas are identical to epidermal inclusion cysts and are surrounded by granulation tissue and fibrosis. The keratin mass frequently becomes infected and shields the bacteria from antibiotics. Squamous cell carcinoma (choice E) rarely occurs in the ear. ***Diagnosis: Cholesteatoma
23 A 40-year-old man presents with a 2-momh history of a painless lump in his left jaw A radiograph of the mandible shows a 2-cm multilocular lesion with smooth cyst-like appearance , smooth pe1iphery, expansion of the bone, and thinning of the cortex. A biopsy of rhe mass is shown b1 the image. This patients ameloblastoma is thought to arise from which of the following cells or structures? ***(A) Developmemnlrests ***(B) Odomogemc cyst ***(C) Osteoblasts ***(D) Osteoclasts ***(E) Radicular cysL
23 The answer is A: Developmental rests. Ameloblastomas (adamantinomas) are tumors o[ epithelial odontogenic origin and represent the most common clinically significam odontogenic tumor. They are slow-growing, locally invasive rumors that generally follow a benign clinical course. Microscopically, ameloblastoma resembles !.he enamel organ in its various stages of differentiation, and a single mmor may show various histologic patterns. The centers of these cell nests consist of loosely arranged, large polyhedral cells that resemble the stellate reticulum of the developing tooth. Choices B and E are acquired nonneoplastic lesions of the oral cavity. Cells of nom1al bone (choices C and D) do not give rise LO this charactetistic odomogen:ic mmor. ***Diagnosis: Ameloblastoma
24 A 67-year-old woman complains of a white discoloration in her mouth. She has a 60-pack-ycar history of smoking. Physical examination reveals white plaques on the buccal mucosa, tongue, and floor of the mouth. A biopsy (shown in the image) demonstrates severe epithelial dysplasia. What is the appropriate clinical diagnosis? ***(A) Actinic keratosis ***(B) Candidiasis ***(C) Leukoplakia ***(D) Malakoplakia ***(E) Papillomatosis
24 The answer is C: Leukoplakia. Leukoplakia is a descriptive te1m for many reactive, preneoplastic, and neoplastic lesions of the oral mucosa. Leukoplakic lesions are not necessarily premalignant and demonstrate a spectrum of histopathologic changes, ranging from increased surface keratinization \vithout dysplasia to invasive keratinizing squamous carcinoma. Candidiasis (choice B) also presents with whitish plaques but does not induce dysplasia. Actinic keratosis (choice A) mvolves sun-e...\.11osed skin and malakoplakia (choice D) occurs i.n the bladder. Papillomatosis (choice E) does not presem as a Oat white lesion. ***Diagnosis: Leukoplakia
25 The patient dcsc tibed in Question 24 is at higher risk of developing which of the follmving neoplasms? ***(A) Adenoid cystic carcinoma ***(B) Mucoepidermoid carcinoma ***(C) Nasopharyngeal carcinoma ***(D) Papilloma ***(E) Squamous cell carcinoma
25 The answer is E: Squamous cell urcinoma. Although the probability of squamous cell carcinoma developing in a patient with oral leukoplakia is low, there is still a 1isk {10% to 12%) of malignant transformation. Carcinogenic factors that lead tO the induction of cancer usually affect more than one site in the oral mucosa, and the rumors may therefore be multiple. Choices A, B, and C do nor mise from the oral epithelium and choice D is nor a complication of epithelial dysplasia ***Diagnosis: Squamous cell carcinoma
26 A 45-year-old man p resents with a painless mass in the neck (shown in the image). A 4-cm finn, movable rumor is idenli- 6ed at the angle of the left jaw. A biopsy or the mmor reveals myoepithelial cells intermingled with myxoid, mucoid, and cartilaginous areas. The tumor is removed surgicaUy. Which of Lhe fo llowing is the most likely prognosis' ***(A) Comralareral spread ***(B) Invasion ofbone ***(C) Local recurrence ***(D) Malignant lransforn1ati on ***(E) Pyogenic abscess
26 The answer is C: Local recurrence. Pleomorphic adenoma, the most common tumor of the salivary glands, is a benign neoplasm characterized by a biphasic appearance which rep resents an admixture of epithelial and stromal elements. At surgery, tumor projections can be missed if the tumor is not carefully dissected. Tumor implanted during surgery or tumor nodules left behind continue to grow as recurrences in the scar tissue of the p revious operation. Recurrence of pleomorphic adenoma represents local growth and does not renect malignancy. Malignant transformation (choice D) is exceedingly rare. ***Diagnosis: Pleomorphic adenoma
1 A 30-year-old man \'lith dwarfism is admitted to the hospital fo r hip replacemem due to severe osreoanh1itis. He has short anns and legs and a relatively large head. His parems do not show sigm of this congenital disease. Tills patiem most Hkely has a spomaneous mutation in the gene encoding which of the following prmems? ***(A) Collagen type l ***(B) Dystrophin ***(C) Fibroblast growth factor receptor ***(D) Growth hormone receptor ***(E) Insulin-like grovnh factor
1 The answer is C: Fibroblast growth factor receptor. Achondroplasia refers to a syndrome of shon-Umbed dwarfism and macrocephaly and represents a failure of normal epiphyseal cartilage formation. It is the most common genetic form of dwarfism and is inherited as an autosomal dominant o·ail. However, most cases represent new mutations. Achondroplasia is caused by an activating mutation in the fibroblast growth factor-3 receptor. This mutation negatively regulates chondrocyte proliferation and differentiation and anests the development of the gro•vth plate. A defective growth honnone receptor (choice D) is responsible [or rare cases of dwarfism (Laron dwarfism) Mutations in dystrophin (choice B) are encountered in cases of Duchenne muscular dystrophy. Congenital deficiency of insulin-like growth factor (choice E) has not been reported as a cause of achondroplasia. ***Diagnosis: Achondroplasia
2 A 2-year-old boy is treated for recurrem fracrures of his long bon es. Physical examination reveals blue sclerae, loose joints, abnormal teeth, and poor heanng. Molecular diagnostic studies will most likely demonstrate a mmation in the gene encoding which of d1e following proteins? ***(A) Collagen ***(B) Dystrophin ***(C) Lysyl hydroxylase ***(D) Fibrillin ***(E) Fibroblast growth faclOr receptor
2 The anosweris A: Collagen. Osteogenesis imperfecta (01) refe rs LO a group of mainly auwsmnal dominant, heritable disorders of connective tissue, caused by mutations in the gene for type I collagen~ this affects the skeleton,joims, ears, ligaments, teeth, sclerae, and skin. The pathogenesis of 01 involves mutations of COLlAl and COL1A2 genes, which encode the a.l and a.2 chains of type 1 procollagen, the major structural protein of bon e. Mmations in lysyl hydroxylase gene (choice C) are seen in patients wiLh Ehlers-Danlos syndrome, and mutations in the fibrillin gene (choice D) account for Marfan syndrome. Mutations in the d}'Strophin gene (choice B) cause Duchenne muscular dystrophy. Mutations in the fibroblast growth factor receptor gene (choice E) may result in achondroplasia. ***Diagnosis: Osteogenesis imperfecta
3 A 9-yea.r-old boy is evaluated for signs of precocious puberty. Laboratory tests demonstrate a 21-hydrorylase deficiency and increased serum levels of androgens. A CT scan reveals enlargement of the adrenal glands bilaterally. If this patient is untreated, short stature will result as a consequence of which of the following mechanisms of disease? ***(A) Decreased growth hormone production ***(B) End-organ resistance to androgens ***(C) Impai red osteoblast activity ***(D) Premature epiphyseal plate closure ***(E) Unresponsiveness to bone morphogenetic p rotein
3 The answer is D: Premature epiphyseal plate closure. Congenital deficiency of2l-hydroxy1ase results in adrenogenital syndrome, which is associated with virilization of extemaJ genitalia in female infants (pseudohermaphroditism). Males exhibit precodous puberty. Evenwally, high levels of adrenal androgens lead to premaLure closure of the epiphyses and stunted growth. The other choices are not related to the adrenogenital syndrome. ***Diagnosis: Adrenogenital syndrome, adrenal cortical hyperplasia
A 17 -year-old girl suffers a spiral fracture of her tight tibia, and the leg is casted. Unfortunately, the fracture does not heal correctly due to excessive motion and interposition of soft tissue at the fracmre site. vVhich of the following represents Lhe most likely complication of nonunion in this patient? ***(A) Cadman triangle fom1adon ***(B) Cup-shaped epiphysis ***(C) lnvolucmm formation ***(D) Osteomyelitis ***(E) Pseudoarthrosis
..__4_ The answer is E: Pseudoarthrosis. l f a fracture site does not heat, the condition is tenned nonunion. Causes of nonunion include interposition of soft tissues at the fracture site, excessive motion, infection, poor blood supply, and other factors mentioned in the question. Continued movement at the unhealed fracture site may also lead to pseudoanhrosis, a condition in which joint-like tissue is formed. ln such cases, the fracture never heals, and the joint-like material must be removed surgically for the fracture to heal properly: Codman tTiangle (choice A) is an X-ray finding of a bone involved in osteosarcoma, where, an incomplete rim of reactive bone adjacem Lo tumor is lifted from the conical surface. Involucrum (choice C) is the viable bone that surrounds necrotic bone (sequestmm) in osteomyelitis. Osteomyelitis (choice D) is an uncommon complication of a closed fracmre. ***Diagnosis: Nonunion of healing fracture
5 A 50-year-old woman presents with lower back pain of 3 weeks in duration. Radiologic studies reveal several dis· crete lytic lesions in the lumbar back and pelvis. laboratory studies show elevated serum levels of alkaline phosphatase. Serum calcium, serum protein, and peripheral blood smears are normal. Aspiration biopsy of a pelvic lesion shows keratinpositive cells. 'vVhich of the follo•ving is the most likely diagnosis? ***(A) Chondrosarcoma ***(B) Metastatic carcinoma ***(C) Osteochondroma ***(D) Osteosarcoma ***(E) PlasmacytOma
5 The answer is 8: Metastatic carcinoma. Multiple lytic lesions ..._ associated with keratin-positive cells strongly suggest metastatic bone cancer. Metasmuc carcinoma is the mosL common tumor of bone, and skeletal metastases are found in at least 85% of cancer cases that have run their full clinical course . The verrebral colunm is the most commonly affected bony stmcture. Tumor cells usually arrive in the bone by way of the bloodstream. Some tumors (thyroid, gastrointestinal nact, kidney, neuroblaswma) produce mosdy lytic lesions. A few neoplasms (prosLate, breast, lung, swmach) s timulate osteoblastic components w make bone. The other choices are not keratin positive. ***Diagnosis: MetasLatic bone cancer
6 A 6-year-old child with mild hydrocephalus suffers chronic infections and dies of intractable chronic anemia. At autopsy, his bones are dense and misshapen. The femur, tn particular, shows obliteration of the n1anow space. Histologically, the bones dem· onstrate disorganization of bony trabeculae by retention of primary spongiosa and further obliteration of the marrow spaces by secondaty spongiosa (shown in the image). Hematopoietic bone marrow cells are sparse. The disorder is caused by muta· tions in genes that regulate which of the following cell types? ***(A) Fibroblasts ***(B) Myofibroblasts ***(C) Notmoblasts ***(D) Ostcoblasts ***(E) Osteoclasts
6 The answer is E: Osteodasts. Osteopetrosis, also known as "marble bonen disease or Albers-Schonberg disease, is a group of rare, inherited disorders. The most common aurosomal recessive form is a severe, sometimes fatal disease affecting infams and children. The sclerotic skelewn of osteopetrosis is the result of failed osteoclastic bone resorption. The disease is caused by mmations in genes that govem osteoclast fo rmation or function. Because osteoclast function is arrested, osteopetrosis is characterized by (1) the retention of the primary spongiosum with its cartilage cores, (2) lack of funnelization of the metaphysis, and (3) a thickened conex. The result is shan, block-like, radiodense bones, and hence the te rm marble bone disease. Choices A, B, and C do not regulate bone organization. Increased osteoblast activity (choice D) has not been demonstrated in patients with osteopetrosis. ***Diagnosis: Osteopetrosis. Albers-Schonbe rg disease
7 A 33-year-old woman presents with a spontaneous fracture ,___ of her femoral head. She has suffered from Crohn disease for 20 years. Multiple surgical procedures have resulted in the removal of much of her small boweL She has had profound weight loss over the last 10 years. The bone is pinned. Hiswlogically, the resected femoral head shows bony trabeculae that are cove red by a thicker-than-normal layer of osteoid (shown in the image). In this section, the osteoid is stained red, and m.ineralized bone is stained black. Which of the following best describes the pathogenesis of this lesion? ***(A) Degenerative changes in !he subchondral bone ***(B) Enhanced osteoblast activity ***(C) Impaired mineralization of osteoid ***(D) Infianunatory synovium with pannus formarion ***(E) Subpe1iosteal bone resorption
._7_ The answer is(: Impaired mineralization of osteoid. Osteoma!- acia (so ft bones) is a disorder of adults characterized by inadequate mineralization of newly fom1ed bone matrLx. Diverse conditions associated with osteomalacia and rickets include abnormalities in vitamin D metabolism, phosphate deficiency states, and defects in the mineralization process itself. In osteomata cia, the bony trabeculae are rimmed by broad layers of osteoid, whereas the bone spicules in osteoporosis are thin but normally mineralized. Intrinsic diseases of the small intestine, cholestatic disorders of the Uver, biliary obstruction, and chronic pancreatic insufficiency are Lhe most frequent causes of osteomalacia in the United States. Malabsorption of vitamin D and calcium complicates a number of small intestinal diseases, including celiac disease , Crohn disease, and scleroderma. Enhanced osteoblast activity (choice B) is encoumered in new bone fom1ation. lnflanunatory synovium with pannus formation {choice D) is a feature of rheumatoid arthritis. ***Diagnosis: Osteomalacia
8 An 18-year-old man presents with bone pain about his knee rhat he has had for 6 weeks. Radiologic studies reveal a lytic lesion of the distal end of the femur, which arises in the metaphysis, e:-:tends into the proximal diaphysis, and elevates rhe periosreum. Serum levels of alkaline phosphatase are markedly elevared. The lesion is removed, and rhe cut surface of the surgical specimen is shown in the image. Molecular studies of this tumor would most likely reveal a mutadon in the gene encoding which of the following proteins? ***(A) CydinA ***(B) Cyclin D ***(C) Fibroblast growth facror receptor ***(D) Rb tumor suppressor protein ***(E) Stimulatory guanine nucleotide-binding protein
8 The answer is D: Rb tumor suppressor protein. Osteosarcoma is a highly malignam bone rumor characterized by fonnation of bone tissue by mmor cells_ lt is most frequem in adolescents between the ages of 10 and 20 years_ Almost two thirds of cases of osteosarcoma exhlbit mutations in the retinoblastoma {Rb) gene, ru1d many rumors also comain mutations in p53. Often, the periosteum produces an incomplete rim of reactive bone adjacent to the site where it is lifted from the cortical surface by the rumor. When this appears on an X-ray as a shell of bone intersecting the cortex at one end and open at the other end, it is referred to as Cadman tr iangle. A "sunburst" periosteal reaction is often superimposed. Mutations in the fibroblast growth factor receptor gene (choice C) are a cause of achondroplasia. Deregulation of cyclins (choices A and B) and cyclin-dependem kinases are associated with several neoplasms. ***Diagnosis: Osteosarcoma
9 A 10-year-old boy complains of increasing pain in his left hip. He began limping shonly after playing a baseball game at schooL He is afebrile. An X-ray of the femoral head shows a fractu re and inegular densities of the cancellous bone. You make a diagnosis of Legg-Calve-Penhes disease. Which of the following best describes the pathologic findings in this patient? ***(A) Avascular osteonecrosis ***(B) Chondroma ***(C) Fibrous dysplasia ***(D) Osteitis fibrosa cystica ***(E) Osteopetrosis
._9_ The an.swer is A: Avascular osteonecrosis. Osteonecrosis, also known as avascular necrosis, refers to the death of bone and marrow in the absence of infection. Such bone infarcts may be caused by a variety of conditions, such as trauma, thrombi , emboli, and corticosteroids. Growing bones of children and adolescents are often affected, and in most instances, the cause of such infarctions is not evident. Legg-Calve-Penhes disease refers to osteonecrosLs in the femoral head in ch tldren. Collapse of the femoral head may lead to joim incongmity and seve re osLeoanhtitis . Chondroma (choice B) is a benign, inrraosseous mmor composed of well-differentiated hyaline cartilage. Fibrous dysplasia (choice C) is a developmema] abnonnality of the skelewn, characterized by a disorganized n~xture of fibrous and osseous elements in the imetior of the affected bones. Osteitis fibrosa cyslica (choice D) occurs in primary hyperparathyroidism. ***Diagnosis: Avascular osteonecrosis
10 A 50-year-old man presems with a 2-day history or left leg pain. His temperature is 38.7"C {lOY'F). He has a harsh systolic murmur and echocardiographic evidence of bac terial endocarditis. lf this patient has developed hematogenous osteomyelius, his bone infection would most likely be found in which of the following anawmic lo ca tions? ***(A) Body of a nat bone ***(B) Diaphysis of a long bone ***(C) Epiphysis or a long bone ***(D) Metaphysis of a long bone ***(E) Peti osteum of a 1 ong bone
10 The answer is D: Metaphysis of a long bone. Hematogenous osteomyelitis pnmatily affects the metaphyseal area of the long bones (knee, ankle, hjp) because oft he unique vascular supply in this region Nonnally, anerioles enter the calcified porlion of Lhe growth plate, form a loop, and then drain into the medullary cavity without esLabllshing a capillary bed. Tllli vascular loop pem1its slowing and sludging of blood now, allowing bacteria time to penetrate the walls of the blood vessels and establish an infective focus wiLhin the bone marrow. Osteomyelitis may break imo the pe1iosteum (choice E) bm does not Oliginate there. Vascular loops do nor reach the epiphysis (choice C). Choices A and B would be distinctly uncommon. ***Diagnosis: Osteomyelitis
11 A 74-year-old, obese \:voman (BMI"" 33 kglm2 ) complains of chronic pain in her back, knees, and fingers. The pain typically subsides at rest. On physical examination, the distal iJllerphalangeal joint.s are enlarged and tender. Which of the follo\ving best describes the pathogenesis of joim pain in this patient? ***(A) Acme inflammation of the ligaments ***(B) Degeneration of articular cartilage ***(C) Degenerative changes of cortical bone ***(D) Inflammatory synovium \>vith pannus fonnation ***(E) Reduction of the volume of synovial fluid
11 The answer is B: Degeneration of articular cartilage. Osteoar· Lh rit:is is a slow! y progressive destruction of the articular cartilage Lhat is manifested in the weight-bearing joints and lingers of older persons or in the joints of younger persons subjected to trauma. Osteoarthritis ts the single most common form of joint disease. The disorder is nor a single nosologic emity but rather a group of conditions that have in common the mechanical desu·uc£ion of a joim. In flammation of synovium with pannus formation (choice D) occurs in patients with rheumawid arthritis. ***Diagnosis: Osreoarth1itis
12 A 60-year-old woman with arthritis suffers a massive stroke and expires. At autopsy, rhe proximal phalangeal joint tissue shows pannus, synovial cell hyperplasia, and lymphoid follkles. Which of the following best describes the pathogen· esis of pannus formation in this patient? ***(A) Calcification of the synovium ***(B) Chronic inflanunation of synovium ***(C) Degeneration of cartilage ***(D) Dislocation o[ a portion o[ bone ***(E) Necrosis of fibroadipose tissue
12 The answer is 8: Chronic inflammation of synovium. Rheumatoid arthritis (RA) is a systemic, chronic in nammatory disease in which chronic polyarthlitis involves diarthrodial joints symmeuically and bilaterally. Synovial lining cells undergo hyperplasia. The result is a synovial lining thrown imo numerous villi and frond -llke folds that .6H the peripheral recesses of the joint. As the synovium undergoes hyperplasia and hypertTOphy, it creeps over the surface of the articular cartilage and adjacent strucLures. This inflammatory synovium is tenned a pannus (cloak). The pannus covers the articular cartilage and isolates it from the synovial nuid. Synovial calcification (choice A) does not occur in RA. Pannus may desLroy cartilage (choice C) by depriving it of noutishmem. ***Diagnosis: Rheumatoid arthritis
13 A 9-year-old boy complains of 2 weeks of pain in the hip. His temperature is 38°C (101 °F). Laboratory studies show an elevated erythrocyte sedimentation rate. An X-ray reveals a mottled radiolucent defect in the upper femur, with abundant periosteal new bone formation. Fine-needle aspiration retums numerous neutrophtls and cocci. Staphylococcus aureus is cultured from the bone lesion. A biopsy shows a fragment of necrotic bone embedded in fibrinopumlent exudate. ·which of the following terms best describes the necrotic bone? ***(A) Brodie abscess ***(B) Cloaca ***(C) Involucmm ***(D) Osteophyte ***(E) Sequestrum
13 The answer is E: Sequestrum. lnfeClious organisms may reach the bone through the bloodstream. lf the infection is not contained, pus and bacteria exlend imo the endosLeal vascular channels that supply the conex and spread throughom the Volkmann and Haversian canals of the cortex. EvenLUally, pus forms underneath Lhe pe riosteum, shearing off Lhe perforating aneries of the periosteum and further devitalizing d1e conex. This expansion may shear oiT the perforating arteries that supply the conex \Vilh blood, leading to necrosis of Lhe conex. The necmtic bone ts called a sequestrum. Brodie abscess (choice A) consists of reacrive bone from the periosteum and the endosteum that surrounds and contains the infection Cloaca (choice B) is the hole found in the bone during formation of a draining sinus. Involucrum (choice C) refers to a lesion in which periosteal new bone fo rmation fo rms a sheath around the necrotic sequestmm. Osteophytes (choice D) are bone nodules appearing on the peripheral portion of the jolm surface that are complications of osteoarthritis. ***Diagnosis: Osteomyelitis
14 A 40-year-old woman complains of morning stiffness in her hands. On physical examination, her finger joints are painful, swollen, and wann. X-ray examination of the hands shows narrowing of the joint spaces and erosion of joim surfaces of the metacarpal/phalangeal joints. The adjacent bones show osteoporosis. A synovial biopsy reveals prominent lymphoid follicles. synovial hyperplasia, and villous folds (shown in the image). Laboratory studies conducted on a blood sample from this patient will most Ukely show polyclonal antibodies directed against which of the following proteins? ***(A) Double-stranded DNA ***(B) Fab2 portion of IgM ***(C) Fe portion of lgG ***(D) Ribonucleoprotein ***(E) Topoisomerase l
14 The answer is C: Fe portion of lg(i . Immunologic mechanisms play an important role in the pathogenesis of rheumatoid arthritis (RA). Lymphocytes and plasma cells accumulate in the synovium, where they produce immunoglobulins, mainly of the lgG class. Some 80% of patients with classic RA are positive for rheumatoid factor {RF). This factor actually repre· sents multiple antibodies, principally IgM, but sometimes TgG or lgA, directed against the Fe fragment of lgG. Significant titers of RF are also found in patients ·with related collagen vascular diseases, such as systemic lupus erythematosus, sclerode rma, and dermatomyositis. The presence of RF in high titer i'> associated with severe and unremitting disease, many systemic complications, and a serious prognosis. Antibodies against choices A, D, and E are seen in patienLS vvith other collagen vascular/systemic autoimmune d1seases. ***Diagnosis: Rheumatoid arthritis
15 A 55-year-old man presents with pain in the left arm. LaboratOry studies show elevated serum levels of calcium and parathyroid hormone. An X-ray of the left ann reveals multiple small bone cysts and patJ1ologic fractures. Biopsy of the affected bone discloses numerous giant cells in a cellular and fibrous stroma. The patient undergoes removal of a parathyroid adenoma. Wl1ich of the following best describes the pathogenesis of bone pain and pathologic fractures in this patient? ***(A) Enhanced osteoblast activity ***(B) Impaired mineralization of osteoid ***(C) Increased bone resorption ***(D) Increased mineralization of bone ***(E) Osteoporosis
15 The answer is C: Increased bone resorption. rn patients with primary hyperparathyroidism, osteoclast.s are stimulated ro resorb bone_ From the subperiosteal and endosteal smfaces, osteoclasrs bore their way into the cortex as cutting cones. This process is te rmed dissecting osteitis. As the disease progresses, the trabecular bone is resorbed, and the manow is replaced by loose fibrosis. Cystic degeneration ultimately occurs, leading w areas of fibrosis that contain reac tive woven bone, and hemosiderin-laden macrophages ofLen illsplay many giant cells, which are actually osteoclasts. Because of its macroscopic appearance, this lesion has been rem1ed a brown mmor. This is not a tme tumor, but raLher a repair reaction. impaired mineralization of osteoid (choice B) is a feature of osteomalacia. Osteoporosis (choice E) is characterized by decreased bm othe1wise normally mineralized bone. ***Diagnosis: Hyperparathyroidism, osteitis fibrosa cystica
16 A 67-year-old man from England develops bow-legs and leg pain over a period of 5 years. He also complains of progressive hearing loss over the last 2 years. A bone biopsy shows a mosaic pattern with prominent cement lines and increased osLeoblasnc and osteoclasLic activity. 5.emm elecn·olyte levels are normal. This patient is at increased risk for developing which o[ the following diseases? ***(A) Amyloidosis ***(B) Mulnple myeloma ***(C) Osteogenic sarcoma ***(D) Pulmonary embolism ***(E) Renal failure
16 The answer is C: Osteogenic sarcoma. Paget disease is a chronic condition characterized by lesions of bone resulting from disordered remodeling, in which excessive bone resorp· tion initially results in lytic lesions, folJowed by disorganized and excessive bone fotmation. The diagnostic hallmark in late disease is an abnom1al arrangement oflamellar bone, in which islands of irregular bone formation, resembling pieces of a jig· saw puzzle, are separated by prominent cement lines. Persons of English descent have a high incidence of this dtsease. Neoplastic transformation may occur in a focus of Paget disease, usually in the femm, humeJUs, or pelvis. This complication occurs in less than l% of all cases; however, the incidence of osteogenic sarcoma is stilll,OOO times higher than that in the general population The other choices are not associated ·with disorganized bone. ***Diagnosis: Paget disease, osteogenic sarcoma
17 A 60-year-old man with a rustory of gout presents \Vith muldple mbbery nodules on his hands (shown in the image). Which of the following best explains the pathogenesis of this patients underlying condi tion? ***(A) Autoimmune relapsing polychondritis ***(B) High dietary intake of purine-rich foods ***(C) Hypercalcemia and chondrocalcinosis ***(D) Impaired renal excretion of uric acid ***(E) Increased calcium hydro.xyapatite deposition
17 The answer is D: Impaired renal excretion of uric acid. Gout is a heterogeneous group of diseases in which the common denon'linawr is an increased senun uric acid level and deposition of urate crystals in the joints and kidneys. A tophus (shown in the phmograph) is an exTracellular soft tissue deposit of urate crystals surrounded by foreign-body giant cells and mononuclear celts. Most cases (85%) of idiopathic gom result from an as yet unexplained impairment of uric acid e."cretion by the kidneys. When sodium urate crystals precipiuue from supersaturated body Dltids, they absorb fibronectin, complement, and a number of other proteins on their surfaces. Neutrophils that have ingested urate crystals release activated m:ygen species and lysosomal enzymes, which mediate tissue injury and promote an inflammawry response. A high dieta1y intake of purine-rich foods (choice B) does not lead to gout, although endogenous overproduction of purines is associated with this condition. ***Diagnosis: Gout
18 A 23-year-old man complains of stiffness and pain in his lower back that causes him to awaken at night. He first noticed morning stiffness in his Lower back during his college years. He also dese1ibes occasional pain in his right eye and sensitivity w light. An X-ray of me sacroiliac region shows fusion of me small joim spaces in the postetior spine and ossification of the intervertebral discs. Serologic tests for rheumawid facwr and antinuclear antibodies are negative. This patient most likely expresses which of the following human leukocyte antigen (HLA) haplorypes? ***(A) BlS ***(B) Bl9 ***(C) B27 ***(D) B31 ***(E) B9
18 The answer is C: 827. Ankylosing spondylitis is an inflammatory anhropathy of the vertebral column and sacroiliac joints It may be accompanied by asymmeuic, peripheral arthritis (30% of patients) and systemic manifestations. Ankylosing spondylitis is most common in young men, and the peak incidence is at age 20 years. More than 90% of patiems arc positive for human leukocyte antigcn-B27 {HLA-B27), although the disorder affects only 1% of persons with this haplotype. None of the other haplotypes are related to the pathogenesis of ankylosing spondylitis. ***Diagnosis: Ankylosing spondylitis
19 A 28-year-old man complains of burning pain on urination, as well as pain in his fingers and left eye. He also relates a recent episode of bacillary diarrhea contracted during a \>isiL to Me,.'ico. Physical C-"<amination confirms arthritis and conjunctivitis. The patient responds well Lo ueatmem with NSAIDs. Which of the following is the most likely diagnosis? ***(A) Ankylosing spondylitis ***(B) Infectious arthritis ***(C) Osteomyelitis ***(D) Reiter syndrome ***(E) RheumaLOid arthritis
19 The answer is D: Reiter syndrome. Reiter syndrome is a triad that includes (l) seronegative polyarthtitis, (2) conjunctivitis, and (3) nonspecific urethritis. The disorder is almost exclusively encountered in men and usually follows venereal exposure or an episode of bacillary dysemery. As in ankylosing spondylitis, Reiter syndrome is associated with HLA-B27 amigen in up to 90% of patients In fact, after an attack of dysentery, 20% ofHLA-B27-posi tive men develop Reiter syndrome. The pathologic feaLures of Reiter anhtilis are comparable to those of rheumatoid arthritis. The other choices do nm typically affect the eye. ***Diagnosis: Reiter syndrome
20 An 85-year-old man presents with a 3-week history of painful swelling of his right knee. Aspiration of joim fluid returns numerous neutrophils and c1ystals, which are described as rhomboid and "coffin-like." Chemical analysis shows that these crystals are composed o[ calcium pyrophosphate. Which of the following is the most likely diagnosis? ***(A) Ankylosing spondyliLis ***(B) Gout ***(C) Infectious anhritis ***(D) Pseudogout ***(E) Rheumawid anhriLis
20 The answer is D: Pseudogout. Calcium pyrophosphate dihydrate ( CPPD)-depositiondisease refers to the accumulation of this compound in synovial membranes (pseudogout), joint cartilage (chondrocalcinosis), ligaments, and tendons. CPPDdeposition disease is ptincipalty a condition of 0 ld age, with hal r of the population older than 85 years being afClicted. Pseudogout refers to sel f-lim.ited attacks of acute arthtitis lasting from 1 day to 4 weeks and involving one or two Joints. Some 25% of patients \Vilh CPPD-deposition disease have an acme onset of gom-like symptoms, manifesting as inflammation and swelling of the knees, ankles, wrists , elbows, hips, or shoulders. The synovial fluid exhibits abundam leukocytes comaining CPPD crystals. Gom (choice B) feamres deposition of urate crystals. Crystal deposi tion does not occur in rheumatoid arlhtitjs (choice E). ***Diagnosis: Chondrocalcinosis, pseudogoul
21 A 10-)rear-old boy complains of pain in his hands and feet. His Lemperature is 38°C (l0l°F). Physical examination reveals a faint pericardjal friction rub. His spleen, liver, and axillaty lymph nodes are enlarged. Which of the following is the most likely diagnosis? ***(A) Gaud1er dlsease ***(B) Gom ***(C) juvenile arthritis ***(D) Psoriatic anhritis ***(E) Reiter syndrome
21 The answer is C: Juvenile arthritis. juvenile anhtius (Still disease) refers w a number of different chronic arth ri tic conditions in ch ildren. Twenty percent of children with polyarticular juvenile anluitis have pronunent systemic sympwms that include fever, rash, hepatosplenomegaly, lymphadenopathy, pleuritis, and anemia. Many children with juvenHe arthritis develop rheumatoid anh1itis, ankylosing spondylitis, psmiatic arthritis, and other connective tissue diseases. Gout (choice B) is a disease mainly of adult men and does not cause organomegaly. Reiter syndrome (choice E) occurs almost exclusively in men and 1.1sually follows venereal exposure. Psoriatic anhri· tis (choice D) is excluded by lack of ps01iasis ***Diagnosis: Juvenile arthritis
22 A 58-year-old woman fractures her hip after slipping on an icy sidewalk. An X- ray shows generalized osteopenia. A bone biopsy reveals attenuated bony trabeculae and a nom1al ratio of mineral-LO-matrbc Serum calcium and phosphorus levels are normal. Which of the following best explains the pathogenesis of osteopenia in this postmenopausal woman? ***(A) Impaired mineralization of osteoid ***(B) Increased osteoblast activity ***(C) Increased mineralization of bone ***(D) Increased osteoclast activity ***(E) Mosaic bone [ormation
22 The answer is D: Increased osteoclast activity. Osteoporosis is a metabolic bone disease characterized by diffuse skeletal lesions in which n ormally mineralized bone is decreased in mass w the point that it no longer provides adequate mechanical support. The remaining bone exhibits a normal ratio of mineralized to nonmineralized {osteoid} matrL'< (therefore, not choices A and C). Bone loss and eventually fractures are tl1e hallmarks of osteoporosis. Primary osteoporosis occurs principally in postmenopausal women (type l) and elderly persons of both sexes (type 2). Type l primary osteoporosis is due to an absolute increase in osteoclast activity. The increased number of os teoclasts that appear in the early postmenopausal skeleton is the direct result of estrogen \'lithdrawa1. Type 2 osteoporosis reflects decreased osteoblast activity (therefore, not choice B). Mosaic bone formation (choice E) is a feature of Paget disease. ***Diagnosis: Osteoporosis, osteopenia
23 A 16-year-old boy presents with a swelling on his left tibia. An X- ray or the leg shows a destructive process, with indistinct borders and an "onion-skin" pattern of pe1iosteal bone. Histologic examination of a biopsy reveals unifom1 small cells with round, dark blue nuclei (shO\llfrl in the image). A PAS stain demonstrates abundant intracellular glycogen. lmmunohistochemisny for leukoqrte common antigen is negative Which of the following is the most likely chromosomal abnom1ality in tllis tumor? ***(A) t(l l ;22) ***(B) t(l4;18) ***(C) t(8;14) ***(D) t(9;22) ***(E) t(3;l6)
23 The answer is A: t(11 ;22) . Ewing sarcoma (EW$) is an uncommon malignant bone mmor composed o[ small, uniform, round cells. It represems only 5% of all bone tumors and is found in children and adolescems. EVvS is d1ought to arise from primitive manow elemen rs or immature mesenchymal cells. Virmally all of these mmors l1ave a reciprocal u·ans]ocation between chromosomes 11 and 22, which results in the fusion of the amino tem1inus of the EWSl gene ro d1e FLl-1 gene, wl1ich encodes a nuclear transcription facwr Chromosomal rranslocation tC14;18) (choice B) is found in follicular lymphomas; t(8) 4) (choice C) is present in Burkitt lymphoma; and t(9;22) (choice D) occurs in clu-onic D1)'elogenous leukemia. ***Diagnosis: Ewing sarcoma
24 A 2-year-old boy presents with a rash. On physical examination, he has a crusty, red skin lesion at the hairline and on the extensor surfaces of his extremities and abdomen. Exophthalmos is noted. An X-ray film of the head shows multiple, radiolucem lesions in the skulL A biopsy of one of lhe skull lesions shows large, plump cells with pale, eosinophilic, foamy cytoplasm and convoluted nuclei (shown in the ilnage). vVhat is the most likely diagnosis? ***(A) Ewing sarcoma ***(B) Giam cell tumor of bone ***(C) Hand-Schuller-Christian disease ***(D) Large B-celllymphoma ***(E) Multiple myeloma
24 The answer is C: Hand-Schiiller-Christian disease. Langerhans ceU histiocytosis is a generic term for three emities characterized by the proliferation of Langerhans cells in various tissues: (1) eosinophilic granuloma, a localized form; (2) Hand-Schuller-Christian disease, a disseminated variant; and (3) Letterer-Siwe disease, a fulminant and often fatal generalized disease. Hand-SchnUer-Christian disease occurs in yow1ger children (age 2 to S years). Radiolucent bony lesions characterize the disorder and occur most frequently in the calvaria, ribs, pelvis, and scapulae. A lesion may infiltrate the retro-orbital space, producing exophmalmos. Infiltration of the stalk of the hypothalamus by the proliferated Langerhans cells leads to diabetes insipidus. Crusty, red, weepy skin lesions occur at the hairline and on the extensor surfaces of the extremities, abdomen, and occasionally soles of the feet. Ewing sarcoma (choice A) is composed of small, uniJonn, round cells. ***Diagnosis: Langerhans cell histiocytosis, Hand-Schuller-Christian disease
25 A 17-year-old boy fractures his lefL tibia in a skiing accident. One year later, an X-ray of the leg discloses reactive bone formad on in the calf muscle at the site of injury. Which of the following is the most likely diagnosis? ***(A) Myositis ossificans ***(B) Fibrous dysplasia ***(C) Malignant fibrous histiocytoma ***(D) Nodular fasctitis ***(E) Synovial sarcoma
25 The answer is A: Myositis ossificans. Myositis ossificans affects young persons an d, although it i.s emiiely benign, often mimics a malignant neoplasm. The lesion typically results from blunt trauma to the muscle and soft tissues, usually of the lower limb. Per ipheral neovasculaiization of the resulting hemawma leads in a short time to the formation of bone spicules in the soft tissue because the local environment is similar lO that of an initial hematoma in a healing fracture. Because myositis ossificans often occurs near a bone, on radiography, it may be misdiagnosed as a malignant bone-fanning tumor. The other choices are unrelated to prior trauma. ***Diagnosis: Myositis ossificans
26 A 16-year-old boy presents with a 2-week history of pain in his righ t leg He says that he has been taking aspirin to relieve the pain. An X-ray of the leg shows a 1-cm sharply demarcated, radiolucent lesion in the diaphysis of the tibia surrounded by dense, sclerotic bone. The lesion is surgically removed , and the gross specimen is show'tl in the image. Microscopically, the rumor shows irregular trabeculae of woven bone surrounded by osteoblasts, osteoclasts, and fibrovascular marrow. 'vVhat is the approp1i ate diagnosis? ***(A) Chondroblastoma ***(B) Giam cell tumor of bone ***(C) Osteoblastoma ***(D) Osteoid osteoma ***(E) Solitary chondroma
26 The answer is D: Osteoid osteoma. Osteoid osteoma is a small, painful, benign lesion of bone composed of osseous tissue (the nidus) and surrounded by a halo of reactive bone formation. The tumor typically occurs in young persons ranging in age from 5 to 25 years. Osteoid osteoma frequently alises jn the cortex of the diaphysis of the tubular bones of the lower extremity. Osteo1d osteoma is a spherical, hyperemic tumor of about 1 em in diamecer that is considerably sofLer than the surrounding bone and easily enucleated m surgery Reactive , sclerotic bone surrounds the nidus. Chondroblaswma (choice A) features primitive chondroblasrs and cartilage ma£rix. Giant cell tumor (choice B) of bone is a locally aggressive neoplasm composed of multinucleated, osteoclastic g1am ceUs. Osreoblasroma (choice C) is a benign neoplasm that is hiswlogically similar lO osteoid osteoma buL larger and not accompanied by nocturnal pain relieved by aspiliJ1 Solitary chondroma (choice E) is a benign, imraosseous tumor composed of welldifferentiated hyaline cartilage. ***Diagnosis: Osteoid osteoma
27 A 68-year-old woman presems \Viih a lump in the soft tissue ofherneck. Physical examination reveals a 0.'5-cm subcutaneous tumor. Biopsy of the mass shows a benign neoplasm. The patient is told that she has the most common soft tissue tumor. What is the appropriate diagnosis? ***(A) Fibroma ***(B) Leiomyoma ***(C) Lipoma ***(D) Pleomorphic adenoma ***(E) Rhabdomyosarcoma
27 The answer is C: Lipoma. Lipoma is composed of welldifferentiated adipocytes and is the most common soft tissue mass. This benign, circumscribed tumor can originate at any site in the body that contains adipose tissue, but most appear in the subcutaneous tissues of the upper half of the body. especially on the trunk and neck. lipomas are encountered mainly in adults. Histologically, a lipoma is often indistinguishable from normal adipose tissue. fibroma (choice A) and leiomyoma (choice B) are benign neoplasms of fibroblasts and smooth muscle cells, respectively Pleomorphic adenoma (choice D) is a mixed neoplasm of the salivary gland. ***Diagnosis: Lipoma
28 A 56-year-old woman receiVes high-dose radiation therapy for thyro1d carcinoma. One year later, the patient presents with a subcutaneous mass at the site of irradiation. A photomicrograph of the b1opsy is shown in the image. Which of the following is the most likely diagnosis? ***(A) Hodgkin lymphoma ***(B) Leiomyosarcoma ***(C) Malignant fibrous histiocytoma ***(D) Rhabdomyosarcorna ***(E) Synovial sarcoma
28 The answer is C: Malignant fibrous histiocytoma. Malignant fibrous histiocywma (MFH) is a soft tissue mmor that conLains foci of histiocytic (macrophage) differend ation and is the most frequent sarcoma encountered after radiation therapy. Hiswlogically. MFH displays a highly variable morphologic pattem, with areas of spindle-shaped tumor cells arrayed ill an irregularly whorled (sto riform) pattem adjacem to pleomorphic fields. The spindle cells tend to be well differentiated and resemble fibroblasts. The other choices do not typically arise as a consequence of radiation treatment. ***Diagnosis: Malignant fibrous histiocytoma
29 A 40-year-old woman presems with pain and swelling Ln her left elbow that has lasted 6 momhs. Physical examination reveals a 0.5-cm soft tissue mass. Biopsy of the mass discloses a biphasic histologic pauern consisting of cuboidal epithelial and spindle-shaped mesenchymal cells. Which of lhe following is the most likely diagnosis7 ***(A) Uposarcoma ***(B) Malignam fibrous l11Suocytoma ***(C) Nodular fasciids ***(D) Rhabdomyosarcoma ***(E) Synovial sarcoma
29 The answer is E: Synovial sartoma. Synovial sarcoma is a highly malignant soft d.ssue tumor that arises in the region of a joim. Synovial sarcoma occurs principally in adolescents and young adults as a painful or tender mass in the vicinity of a large joim, panicularly the knee. The neoplasm consists of spindle-shaped mesenchymal cells and cuboidal epithelial-like cells. The latter stain with antibodies w keratin, form glands and clefts, and are presumabl)' epitheliaL The other tumors listed exhibit some histologic polymorphism, but their cells do not show any epithelial features or markers of differentiation. ***Diagnosis: Synovial sarcoma
30 A 50-year-old man complains of fever and severe pain in his great toe of 24 hours in duration. The pain developed in the morning and became so severe that he could not walk. Laboratory findings include leukocytosis, hyperuricemia, and hyperlipidemia. An X-ray of the affected joint reveals punched-out lesions in the juxta-an icular bone. An aspirate of joint Ouid returns urate crystals and neUlrophiJs. Which of the fo llowing would be lhe most likely pathologic finding witl1in the periarticular soft tissue of this patient? ***(A) Osteophyte ***(B) Pannus ***(C) Reactive bone ***(D) Rheumatoid nodule ***(E) Tophus
30 The answer is E: Tophus. Chronic accumulation of uric acid crystals leads co the formation of nodules (cophl) that contain granuloma-like aggregates o[ macrophages. These granuloIna- like areas are found in canllage, in any of the soft tissues around joints, and even in the subchondral bone marrow adjacent to joints. Osteophytes (choice A) are a complication of osteoarthritis. Pannus (choice B) is feamred in rheumatoid arthritis. Rheumatoid nodules (choice D) are found in extraanicular locations. ***Diagnosis: Gom
31 A 35-year-old \Voman has multiple canilaginous lesions in her long and shon bones. A radiograph of Lhe hand (shown in the image) reveals bulbous swellings. A biopsy shows abnom1ally arranged hyaline cartilage, with scattered zones of proliferation. This paLiem is at risk for which of the following bone di<Jeases? ***(A) Chondrosarcoma ***(B) Gtam cell mmor of bone ***(C) Osteosarcoma ***(D) HtSlOC>'lic lymphoma ***(E) Srno,•1al sarcoma
31 The answer is A: Chondrosarcoma. Enchondromatosts, also termed Oilier disease, is a bone disorder characterized by the development of numerous cartilaginous masses that lead to bony deformities. The condition is not strictly a disease of delayed maturation of bone, but one in which residual hya1ine cartilage. anlage cartilage, or cartilage from the growth plate does not undergo endochondral ossification and remains in the bones. As a consequence, the bones show multiple, tumor-like masses of abnormally arranged hyaline cartilage (enchondromas). with zones of proliferative and hypenrophied cartilage. Enchondromas exhibit a strong tendency to undergo malignant change into chondrosarcomas in adult life. None of the other choices are related to cartilaginous tumors. ***Diagnosis: Enchondromatosis, Oilier disease
32 A 2-t-ycar-old man on chronic corncosteroid therapy for severe asLhma presents with a 6-momh history of increasing hip rain . This patiem most likely exhibiLs symptoms of which of the following metabolic bone diseases? ***(A) Gaucher disease ***(B) Osteomalacia ***(C) Osteopetrosis ***(D) Osteoporosis ***(E) Paget disease
32 The answer is D: Osteoporosis. Risk factors [or osteoporosis include smoking, vitamin D deficiency, low body mass index, hypogonadism, a sedentary lifestyle, and glucocorti· coid therapy (seen in this patient). Bone loss and fractures are the hallmarks of osteoporosis, regardless o[ the underlying cause. Choices A and Care congenital disorders that are not related to conicosLeroid therapy. Choices B and E are acquired conditions blll they are not related to conicosteroid therapy. ***Diagnosis: Osteoporosis
1 A 4-year-old boy is brought to the physician by his parents because he falls a lot, cannot jump, and tires easily. Physical examination reveals weakness in the pelvic and shoulder girdles and enlargemem of the child's calf muscles. The serum level of creatine kinase is elevated. A biopsy of calf muscle reveals marked variation in size and shape of muscle fibers. There are foci of muscle fiber necrosis, myophagocytosis, regenerating fibers, and fibrosis. Which of the following is the most likely cause of death expected in this patient? ***(A) Dissecting aortic aneurysm ***(B) Disseminated intravascular coagularion ***(C) Pulmonary embolism ***(D) Respiratory insufficiency ***(E) Rhabdomyosarcoma
1 The answer is D: Respirat.ory insufficiency. Duchenne muscular dystrophy is a severe, progressive, X-linked, inherited condition characterized b)' progressive degeneration of muscles, particularly those of 1.he pelvic and shoulder girdles. The weakness is noted mainly around the pelvic and shoulder girdles (proximal muscle weakness) and is relemlessly progressive. "Pseudohypenrophy'· (enlargemem of a muscle due w abundant replacement of muscle fibers by fibroadipose tissue) of the calf muscles eventually develops. Patients are usually wheelchair bound by the age of 10 years and bedridden by age 15 years. The most common causes o[ death are complications of respiratory insufficiency caused by muscular weakness or cardiac arrhythmia due 1.0 myocardial involvement. The other choices are n01. complications of Duche1me muscular dystrophy. ***Diagnosis: Duche1me muscular dystrophy
2 Molecular diagnostic assays performed on muscle biopsy from tl1e patient described in Question l would show alterationsin the length of the primary transcript for which of the following muscle-associated proteins? ***(A) Creatine kinase ***(B) Desmin ***(C) Dystrophin ***(D) Glycogen phosphorylase ***(E) Myosin
2 The answer is C: Dystrophin. Duchenne muscular dystrophy is caused by mutations of a large gene on the short arm of the X chromosome (Xp21). This gene codes for dystrophin, a protein localized to tl1e inner surface of the sarcolemma. Dystrophin links the subsarcolemmal cytoskeleton to the exterior of the cell through a transmembrane complex of proteins and glycoproteins that binds ro laminin. Dystrophin-deficiem muscle fibers thus lack the normal interaction between the sarcolemma and the extracellular matrix. This dismption may be responsible for the observed increased osmotic fragility of dystrophic muscle, £he excessive influx of calcium ions, and the release of soluble muscle enzymes such as creadne kinase into the serum. The other proteins are not altered in pauems wilh Duchenne nmscular dysuophy. ***Diagnosis: Duchenne muscular dystrophy
3 A 10-year-old girl complains of persistent redness of the skin over her knuckles and around the nail beds. She describes easy fatigability and can rise only with difficulty from a squauing position. Physical examination reveals erythema over the knuckles and a heliotropic rash. A muscle biopsy shows infiltrates of B and T lytnphocytes around blood vessels and in connective tissue of the perimysium. Elevated serum levels of which of the fol lowing would be expected in this patient? ***(A) AlkaBne phosphatase ***(B) Alpha-femprotein ***(C) Carcinoembryonic antigen ***(D) Creatine kinase ***(E) Urea-nitrogen
3 The answer is D: Creatine kinase. The inflammatory myopathies represent a heterogeneous group of acquired disorders, aiJ of which feature symmeu·ic pro:-.:imal muscle weakness, increased serum levels of muscle-derived enzymes, and nonsuppurative mOammation of skeletal muscle. They are thought tO have an autoimmune migin The mosL common morphologic characteristics in the inflammatory myopathies are (1) inflammatory cells, (2) necrosis and phagocytosis of muscle fibers, (3) a mixtme o[ regenerating and atrophic fibe rs, and (4) fibrosis. Dermaromyosms is distinguished from the other myopathies (i.e., polymyositis and inclusion body myositis) by the presence o[ a characteristic heliotropic rash on the upper eyelids, face, and Lrunk. PaLiems with inflammatory myopathies have increased serum levels of creatine kinase and mher muscle enzymes. Elevated serum alkaline phosphatase (choice A) is associated with liver and bone disease. Alphafetoprotein and carcinoembryonic antigen (choices B and C) are markers of neoplasia. Elevated blood urea-nitrogen (choice E) is associated with renal disease. ***Diagnosis: Dennatomyositis
4 A 25-year-old woman complains of weakness and easy ,___ fatigability, which is most pronounced in the late aftemoon. She describes difficulty reading and tiredness while watching television. She has problems che'.\ring and swallowing and loses her voice while talking Physical examination reveals ptosis and diplopia. Laboratory studies would most likely demonstrate semm autoantibodies directed against which of the follo\\ring proteins? ***(A) Acetylcholine receptor ***(B) Phosphodiesterase ***(C) Desmin ***(D) Dystrophin ***(E) Troponln
4 The answer is A: Acetylcholine receptor. Myasthenia gravis is an acqui red autoimmune disease characterized by abnonnal muscular fatigabiJity. It is caused by ci rculating antibodies to the acetylchollne receptor at the myoneural junction (moror endplate) Antibodies to the acetylcholine receptor can be demonstrated in the serum of most patients \Vith myasthenia gravis and localized in muscle biopsies by immunohistochemistry. The clinical severity of the condition is variable, and symptoms tend to wax and wane. The other choices are not related to myasthenia gravis. ***Diagnosis: Myasthenia gravis
5 Which of the following may be of therapeutic benefit to the patient described in Question 47 ***(A) Adrenalectomy ***(B) Parathyroidectomy ***(C) Thymectomy ***(D) Thyroidecwmy ***(E) Vagotomy
5 The answer is C: Thymectomy. The thymus clearly plays an important role in the pathogenesis o[ myasthenia gravis. Up to 40% of patients with thymoma develop myasthenia gravis, and surgical removal of the tumor is often curative. Other patients \vith myasthenia gravis have thymic hyperplasia, and in such cases, thymectomy is often an effective treatment. Acetylcholine receptors have been demonstrated on the surface of some thymic cells in both thymoma and th}rmic hyperplasia. None of the other choices are curative [or myasthenia gravis. Abnormalities of the other organs are not associated with myasthenia gravis. ***Diagnosis: Myasthenia gravis
6 A 60-year-old man recovenng from a "flu " complains of marked fatigability. He reports that he cannot climb stairs two at a time as he used to. He also desc1ibes pain in his thighs. A muscle biopsy (shown in the image) demonstrates a mono· nuclear inflammatory cell in filtrate chiefly ln the endomysium. lmmunostaining shows that most of these inflanunatory cells are CDS• T lymphocytes. \Vhat is the appropriate diagnosis? ***(A) Dermawmyositis ***(B) Myasthenia gravis ***(C) Myownic dystrophy ***(D) Polymyositis ***(E) Werdnig-Hoffrnan disease
6 The answer is D: Polymyositis. Polymyositis is related to di rect muscle ce ll damage produced by cytotoxic T cells. Healthy muscle fibers are initially surrounded by CD8' T lymphocytes and macrophages (see photomkrograph), after which muscle fibe rs degenerate. There is a frequent associadon berween polymyositis and ami-jo-1, an antibody against hisndyl-tRNA synthetase, with the concomitant presence of interstitial lung disease, Raynaud phenomenon, and nonerosive arthritis. Although viral infections (e.g., influenza) may n·igger polymyositis, muscle tissue has not yielded a \'ims on culrure. An inflammatory myopathy indistinguishable from polymyositis occurs in many cases of HlV infection, but the role of the lemivirus is unclear. None of the other choices are associated with an infiltrate of cytowxic T lymphocytes. ***Diagnosis: Polymyositis
7 A 60-year-old man who had been treated for lung cancer com- L-- plains of a rash on his chest and pain in his upper arms and calves. He cannot raise his anus and climbs lhe stairs only with difficulty. A muscle biopsy shows perivascular infiltrates of lymphocytes and plasma cells extending in between the muscle fibers. lmmunofluorescence reveals inunune complexes in the walls of intramuscular blood vessels. Which of the fo llo\.ving is the most likely diagnosis? ***(A) Becker muscular dystrophy ***(B) Dermawmyosids ***(C) Lambcn-Eaton myasthenic syndrome ***(D) Myasthenia gravis ***(E) To>..'ic myopathy
7 The answer is 8: Dermatomyositis. Dermatomyositis is an immune-mediated microangiopathy that leads to obliteration of capillaries, ischemic injury, and muscle damage. lmnmnoOuorescence demonstrates that the walls of many capillaries contain C5b-9 proteins (Le . membrane attack complex)_ When dermaromyosilis occurs in a middle-aged man, it is associated with an increased risk of epithelial cancer, most commonly carcinoma of the lung. By comrast, polymyositis and inclusion body myositis have only a chance association with malignancy. Lamben-Eawn myasthenic syndrome (choice C) is also seen in patients ·with lung cancer, bm as with the other inconect choices, it is not associated with a skin rash or muscle inflammation. ***Diagnosis: Dermatomyositis
8 A 70-year-old man who had been treated for small cell carcinoma of the lung develops marked weakness of his legs and anns. He also complained of momh dryness, double vision, and drooping upper eyelids. On physical examination there is diffuse muscle weakness and wasting. A muscle biopsy is nonnaL Laborawry studies demonstrate semm lgG autoantibodies that recognize voltage-sensitive calcium chatmels in motor nerve tenninals. Which of the following is the most likely diagnosis? ***(A) Becker muscular dystrophy ***(B) Dermawmyositis ***(C) Lamben -Eaton myasthenic syndrome ***(D) Myasthenia gravis ***(E) Toxic myopathy
8 The answer is C: Lambert-Eaton myasthenic syndrome. Lamben- Eaton syndrome is a paraneoplastic disorder that manifests as muscular weakness. wasting, and fatigability of the proximal limbs and trunk Also tenned myasthenk-myopathic syndrome, the disease is usually associated with small cell carcinoma of the lung, although it may also occur in patients 'Nith other malignant diseases. Like myasthenia gravis, the disease seems to have an autoimmune basis because it can be transferred to mice by lgG from patients and it responds to treatment with corticosteroids. The pathogenic IgG autoantibodies recognize voltage-sensitive calcium channels that are expressed both in motor nerve terminals and in the cells of the lung cancer. The calcium channels, which are necessary for release of acetylchoJ]ne, are greatly reduced in the presynaptic membrane in these patients, thereby interfering with neuromuscular transmission. Amichannel antibodies are not encountered in the other choices. ***Diagnosis: Lambert-Eaton myasthenic syndrome
9 A +O-year-old man presems with muscle weakness. He catmot open his hand for a handshake and cannot extend hls arm after flexing it. On physical examination, he has marked atrophy of leg and arm muscles, ptosis, and a fixed facial expression. There is testicular atrophy. laboratory studies demonstrate mild diabetes. A muscle biopsy reveals atrophy of L ype l fibers, hype11rophy oft)'J>e 11 fibers, and numerous fibe rs \vith centrally located nuclei. Which of the following is the most likely diagnosis? ***(A) Denuawmyositis ***(B) Duchenne muscular dystrophy ***(C) Limb-girdle muscular dystrophy ***(D) MyasthenLa gravis ***(E) Myotonic dystrophy
9 The answer is E: Myotonic dystrophy. Myownic dystrophy, the most common form of adult muscular dystrophy, ls an autosomal dominant disorder characterized by slowing muscle rela..'<ation (myotonia) and progressive muscle weakness and wasting. ln addition to skeletal muscle, myotonic dystrophy affects many systems, including the heart, smooth muscle, cemral nervous system, endocrine glands , and eye. Myotonic dystrophy can be separated into two clinical groups: adulr onset and congenital. Most adult patients display atrophy of type I fibers and hypertrophy of type II fibers. Unlike the other choices, internally situated nuclei are a constant feature. Necrosis and regeneration, although occasionally present, are not as prominent as they are in Duchenne muscular dystrophy (choice B). ***Diagnosis: Mymonic dystrophy
10 A 20-year-old man reports muscle cramping after vigorous exercise. He noted darkening of his urine after recently running a half-marathon. A Ulinalysis at that time showed myoglobinuria. A muscle biopsy displays no morphologic abnormalities. Which of the following is the most likely diagnosis? ***(A) Carnitine palmityl transferase deficiency ***(B) lambert-Eaton myasthenic syndrome ***(C) Miwchondrial encephalomyopathy ***(D) Pompe disease ***(E) Toxic myopathy
10 The answer is A: Carnitine palmityl transferase deficiency. Patients with carnitine palmityl transferase deficiency cannot metabolize long-chain fatty acids because of an inability to transport lhese lipids into the mitochondria, where !.hey undergo ~-oxida tion. After prolonged exercise, these patients have muscular pain, which may progress to myoglobinuria. After such an episode, fibers regenerate and restore muscle structure. Biopsy specimens show no microscopic abnormalities, and the diagnosis depends on the biochemical assay for carniline pabnityl transferase activity The other choices do not feature exercise-induced myoglobinuria. ***Diagnosis: Carnitine palmityl transferase deficiency
1 t A 3-month-old boy p resents with seve re hypotonia and areflexia. His tongue and hean are enlarged. A muscle biopsy displays massive accu mulation of membrane-bound glycogen and disappearance of the myofilaments and other sarcoplasnlic organelles. The patiem dies after l year. Which of Lhe following is the most likely diagnosis? ***(A) Carnitine deficiency ***(B) Duchenne muscular dystrophy ***(C) Hurler syndrome ***(D) Niemann-Pick disease ***(E) Pompe disease
1 t The answer is E: Pompe disease. Glycogen-storage diseases are autosomal recessive, inherited, metabolic diso rders characterized by an inability to degrade glycogen. The fi rst acid malrnse deficiency to be recognized, described by Pompe, is the most severe form and occurs in !.he neonatal or early infantile stage. These patients have severe hypotonia and areflexia. Sometimes, the patients have an enlarged wngue and cardiomegaly and die of cardiac failure, usually within the first 2 years of life. Many tissues arc affected, but the most significant involvement is in skeletal and cardiac muscle, the central nervous system, and the Uver. The serum creatine kinase leve.J is only shghtly to moderately increased. Hurler syndrome (choice C) is an inheJited defect in mucopolysaccharide metabolism. Glycogen accumulation is not a featu re of the other choices. ***Diagnosis: Pompe disease
12 A 42-year-old woman presents with muscle weakness. She has difficulty climbing stairs and lately ti res while combing her hair. A muscle biopsy demonstrates lyn1phocytic infiltration with single-fiber necrosis. In addition, muscle fibers exhibit basophilic granular material. An electron micrograph of the cytoplasmic material is shovm. Which of the following is the most likely diagnosis? ***(A) Amyotrophic lateral sclerosis ***(B) Becker muscular dysrrophy ***(C) Inclusion body myositis ***(D) Myasthenia gravis ***(E) Myotonic dystrophy
12 The answer is C: Inclusion body myositis. The pathologic features of inclusion body myositis resemble those of poly· myositis and consist of single-fiber necrosis and regeneration with predominamly endomysia! cytoto>..ic T cells. The inclusions are stained by Congo red and represent a form of intracelllllar amyloid that can be demonstrated by electron microscopy. The fibers in thee lectron micrograph shmvn here represent amyloid filamems. These fi laments are immunoreactive for ~-amyloid protein-same type of amyloid present in the senile plaques of Alzheimer disease. The pathogenic significance of these inclusions is not underswod. None of the mher disorders feature amyloid deposition or inclusions. ***Diagnosis: Inclusion body myositis
13 A healthy 2 8~year-old woman collapses near the end of a summer marathon. ln the emergency room, the patiem is noted to have hot dry skin with little sweating. Her temperatu re is 40.4°C (l04.8°F). Laboratory findings reveal high levels of creatine kinase. The urine is dark red and contains myoglobin. Which of the following would be most expected in the muscle tissue o[ this patient? ***(A) Amyloid deposition ***(B) Gangrene ***(C) Lymphocytic infiltration ***(D) Microabscesses ***(E) R11abdomyolysis
13 The answer is E: Rhabdomyolysis. Rhabdomyolysis refers to the dissolution of skeletal muscle fibers and the release of myoglobin imo the circulation, an event that may result in myoglobinuria and acute renal faiture. The disorder may be acute, subacute, or chronic. During acute rhabdomyolysis, the muscles are swollen, tender, and profoundly weak Rhabdomyolysis may complicate heat stroke or malignant hyperthennia after administration or an anesthetic such as halothane .. Pathologic changes in rhabdomyolysis correspond to an active, noninfl ammatory myopathy. with scattered necrosis of muscle fibers and varying degrees or degeneration and regeneration. None of the other choices are seen in acute rhabdomyolysis. ***Diagnosis: Heat stroke, rhabdomyolysis
14 A 22-year-old woman injured her leg in a motor vehicle accident and subsequently su!Ters from weakness of her left lower leg A biopsy of rhe gastrocnemius muscle is obtained after 4 months (shown in the image). Which of the following best characterizes this pathology? ***(A) Autoimmune myopathy ***(B) Compensatory hypemophy ***(C) Denervation ***(D) Mitochondrial depletion ***(E) Target fibers
14 The answer is C: Denervation. A muscle biopsy is a highly sensitive test for detecting a lesion of the low·er motor neuron, but the pattern of denervation does not identify the cause of the lesion. When a skeletal muscle fiber becomes separated from comact with iLo; lower moLOr neuron, it invatiably atrophies due to the progressive loss of myofibrtls. On cross section, the atrophic fiber has a characteristic angular configuration, seemingly compressed by sunounding normal musde fibers. If the fi ber is nor reinnervated, the atrophy proceeds to complete loss of myofibrils, and the nuclei condense into aggregates. The photomicrograph in this case shows small groups of angular arrophic fibers reflecting advanced denervation. In the end stage, the muscle fibers disappear and are replaced chiefly by adipose tissue. The other choices are nor related to denervation. ***Diagnosis: Skeletal muscle atrophy, denervation
15 A 28-year-old man begins a program of vigorous body building. After 6 months, his biceps would be expected to exhibit which of the follo,ving adaptive cellular changes? ***(A) Hyperplasia of type l fibers ***(B) Hyperplasia of t)l">e 11 fibers ***(C) Hypemophy of type 1 fibers ***(D) Hypertrophy of rype ll fibers ***(E) Hypoplasia of type l and 11 fibers "viLh hyperplasia of myeloblasts
15 The answer is D: Hypertrophy of type II fibers. Stimulation of type II fibers elicits a fas ter, shoner, and more powerful con· u·action than occurs in type I fibers. T)rpe Il muscle fibe rs are suitable for rapid contractions of brief duration and react to strength training with hypertrophy. Androgenic s teroids also induce hyperrrophy of type ll fibe rs, and disuse of Lhe muscle resu lts in their selective atrophy. Skeletal muscle does not respond to an increased workload by increasing the number of fibe rs (hyperplasia). ***Diagnosis: Skeletal muscle hypertrophy
1 A female neonate is noted at binh to have a gross defonniry of her lower back. Examination of the subcutaneous lesion reveals disorganized neural tissue wilh entrapmem of nerve roots . What is the appropriate diagnosis7 ***(A) Meningocele ***(B) Meningomyelocele ***(C) Rachischisis ***(D) Spina bi fida occulta ***(E) Sytingomyelia
1 The answer is 8: Meningomyelocele. Neural rube defects (dysraphic anomalies) reflect impaired closure of the dorsal aspect of the vertebral colunm. These abnormalities are classified according the extent of the defect, ranging in severity from spina bifida occulta to meningocele, meningomyelocele, and rachischisis. Spina bifida (choice D) is restricted to the vertebral arches and is usually asymptomatic. Meningocele (choice A} pem1its protrusion of the meninges as a Duid-fiJled sac.. Meningomyelocele (choice B) e:li.1)0ses the spinal canal and causes the nerve roots to be entrapped. Rachischisis (choice C) is an extreme defect, often vvithout a recognizable spinal cord. Syringomyelia (choice E) is a congenital malformation, in which a tubular cavitation (S)•rim:) extends for variable distances along the entire length of the spinal cord. ***Diagnosis: Neural rube defect, meningomyelocele
2 The parems of the neonate described in Question 1 ask about the risks for simllar birth defects in their future offspring. You mention that supplementation of the maternal diet can reduce the incidence of neural tube defects. What is this important dietary supplement? ***(A) Folic acid ***(B) Niacin ***(C) Thiamine ***(D) Vitamin B~ ***(E) Vitamin 812
2 The answer is A: Folic acid. Maternal folic acid deficiency has been associated with an increased incidence of neural tube defects, and foHc acid has, therefore, been approved for inclusian as a food supplemem in commercial flour. Deficiencies of vitamins in the other choices are nor associated Wlth congenital dysraphic anomalies. Niacin deficiency results in pellagra. Thiamine deficiency (choice C) causes Wernicke syndrome and beriberi. Vitamin B12 deficiency (choice E) leads to pernicious anemla. ***Diagnosis: Neural tube defeCL
3 A male neonate is noted at birth to have paralysis of the lower .___ limbs. The infam fails w thtive and e:x:pires. The brainstem and cerebellum are examined at autopsy (shown in the image). What is the diagnosis? ***(A) Anencephaly ***(B) Arnold-Chiari malformation ***(C) Holoprosencephaly ***(D) Hydromyelia ***(E) Lissencephaly
3 The answer is B: Arnold·Chiari malformation. Arnold-Chiati malformation is a condition in which the brainstem and cerebellum are compacted imo a shallow, bowl-shaped posterior fossa with a low-positioned tentorium. The cerebellar vennis is herniated below the level of the foramen magnum in the photograph shown. Anencephaly (choice A) refe rs to the congenital absence of all or part of the brain. Holoprosencephaly (choice C) is a microcephalic brain in which the interhemi· spheric fissure is absent. Hydromyelia (choice D) is the term for dilation or the central canal of the spinal cord. ***Diagnosis: Arnold-Chiari malformation
4 A female neonate is noted to have a pronounced enlargement of her head (shown in the image). She develops convulsions. MRI reveals excessive accumulallon of cerebrospinal Ouid, ventricular enlargemem, and atrophy of the cerebral cortex. Thts developmental birth defect was most hkely caused by which of the followmg mechamsms of disease? ***(A) Atresia of the aqueducl of Sylvius ***(B) Birth trauma ***(C) Congenital brain tumor ***(D) Maternal folate deficiency ***(E) Oligohydramnios
4 The answer is A: Atresia of the aqueduct of Sylvius. Congenital hydrocephalus refers to an excessive amount of CSF and ventricular enlargement. Congenital atresia of the aqueduct of Sylvius is the most common cause of congenital hydrocephalus , occurring in 1 in 1,000 live births. Histologic examination of ll1e midbrain may disclose multiple atretic channels or an aqueduct narrowed by gliosis. Congenital brain tumors (choice C) are rare. The other choices are not associated with congenital hydrocephalus. Because the infantile cranium expands easily, symptoms of increased intracranial pressure are generally absent. However, convulsions are common, and optic atrophy with blindness can occur. ***Diagnosis: Hydrocephalus
5 Which anatomic structure/region produced the CSF that ..____ accumulated in the brain of the neonate described in Question 4? ***(A) Arachnoid ***(B) Choroid plexus ***(C) Corpus callosum ***(D) Pia mater ***(E) Subependymal areas of the cerebral hemispheres
,_5_ The answer is B: Choroid plexus . Cerebrospinal Ouid (CSF) constirutes an accessory circulatory system adapted to the needs of the CNS. CSF is formed principally by the choroid plexus at a rate of appro:ximately 500 ml per day. The choroid ple>..'tls stretches along the roof of the third ventricle and then angles posteriorly to span the lateral vemrkles. It Oows from its intraventricular origin to sites of reabsorption, principally through the arachnoid villi, into the dural sinuses. The 11uid transports metabolites, serves as a medium for clearing metabolic waste, and protects or ''cushion_c;" the structures contained within it. CSF is not produced in any of the other snuctures. ***Diagnosis: Hydrocephalus
6 A 3-year-old girl who has been memally retarded since birth is killed in a drownil1g accident. The superior surface of the brain at autopsy is shown in the image. What is the diagnosis for this gross defonnity of the brain? ***(A) Arrhinencephaly ***(B) Holoprosencephaly ***(C) Llssencephaly ***(D) Pachygyria ***(E) PolymJcrogyrta
6 The answer is E: Polymicrogyria. Abnonnalities of the cerebral gyri are frequently associated with mental retardation. Polymicrogyria is a congenital disorder in which lhe surface of the brain exhibits an excessive number of small, irregularly sized, randomly distributed gyral folds. Arrhinencephaly (choice A) refers to absence of the olfactory tracts. Ussencephaly (choice C) is a congenital disorder in which the conical surface of the cerebral hemispheres is smooth or has imperfectly formed gyri. Pachygyria (choice D) is a condition in which the gyri are reduced in number and usually broad. ***Diagnosis: Polymicrogyria
7 A 2-week-old male neona1e has frequent generalized seizures and fails to thrive. Physical examination reveals facial dysmorphology, mcluding cleft palate and low-set ears. A coronal section of the brain at autopsy is shown in the image. Which of the followmg tS the most likely cause of this congenital binh defect? ***(A) Bacterial meningitis ***(B) Birth trauma ***(C) Chromosomal abnormality ***(D) Fetal alcohol synd rome ***(E) TORCH syndrome
7 The answer is C: Chromosomal abnormality. Conge:njtal L...- defects of the CNS are often associated with chromosomal abnormalities, which are best exemplified by trisomies of chromosomes 13 to 15 (holoprosencephaly) and chromosome 21 (Down syndrome). Holoprosencephaly refers to a mic rocephalic brain in which the interhemispheric fissure is absent. The horseshoe-shaped cerebral hemispheres have fused frontal poles, across wruch the gyri show an irregular hmizomal orientation. Holoprosencephaly is rarely compatible with life beyond a few weeks or months. All of the other choices are acquired disorders that do not affect gross brain morphology. ***Diagnosis: Holoprosencephaly
8 A 50-year-old man presents \Vlth a "staggering" gait and "lightning pain" in his hands and legs. His past medical history is significant for an aortic aneurysm and aortic insufficiency. Neurologic examination reveals impaired senses of vibration, as well as touch and pain in the lower extremities. The patient subsequently dies of pneumonia. Amopsy discloses obliterative endarteritis of meningeal blood vessels and atrophy of the posterior columns of the spinal cord. What is the appropriate diagnosis1 ***(A) Amyotrophic lateral sclerosis ***(B) flied reich ataxia ***(C) Huntington disease ***(D) Tabes dorsalis ***(E) Vitamin B12 deficiency (subacute combined degeneration)
8 The answer is D: Tabes dorsalis. Tabes dorsalis is a feature of tertiary syphilis and is characterized by ch ronic fibrosing menin gitis, which constricts the posterior root of rhe spinal cord. The posterior roots contain senso ry nerves that ori ginate in the spinal ganglia and form the posterior columns of the spinal cord. Compression of sensory nerves that originate in the posterior roots causes lancinating pain in extremities. lt also damages the transmission of proprioceptive impu lses, causing gait disturbances (ataxia). Amyotrophic lateral sclerosis (choice A) is a mmor neuron disease that does not affect the posterior columns. Friedreich ataxia (choice B) is an autosomal dominant trait that involves the spinal cord in a complex way It affects not only the cemripetal pathways (spinocerebellar and posterior colunms), but also the efferent corticospinal tracts. Subacute combined degeneration (choice E) is due w vitanlin B11 deficiency and involves not only the posterior colunms, but also the ame1ior horn cells and the spinocerebellar and corticospin al tracts. ***Diagnosis: Syphilis
9 A 76-year-old man is admitted to the hospital for evaluation of progressive memory loss and disorientation. The pupils are small bm react nom1ally to light. Muscle LOne is normaL A lumbar puncture returns clear, colorless CSF under nonnaJ pressure. An electroencephalogram shows diffuse slowing. ACT scan of the brain reveals moderate atrophy. Which of the following is the most likely diagnosis? ***(A) Alzheimer disease ***(B) CreutzfeldHakob disease ***(C) Glioblastoma mulliforme ***(D) Huntington disease ***(E) Pick disease
9 The answer i.s A: AJzheimer disease. Alzheimer disease is the most common cause of dementia in the elderly, accounting for more than half of all cases. Alzheimer's original paLients were younger than 65 years of age, but the disease now refe rs to dementias that display characteristic pathologic findings. Alzheimer disease is an insidious and progressive neurologic disorder, characterized clinically by loss of memory, cognitive impairment, difficulty \'lith language, and eventual dementia. 1t features atrophy of the brain, which can be recognized by CT scan as widening of the sulci and bilateral atrophy of the gyri, particu larly the frontal and hippocampal conex TI1ese clinical and morphologic feamres are not typically obsetved in the Dlher choices. Bronchopneumonia is the usual lethal outcome of Alzheimer disease. ***Diagnosis: AJzheimer disease:
10 A 66-year-old woman vocalist complains of difficulty remembeJing her favorite songs. This problem continues to worsen over the next several months, and the paliem becomes increasingly withdra\.vn from her ramUy. When examined, she evidences dememia and gait disturbance. MRl demonstrates mild cerebral atrophy. Analysis of CSF shows no inOammalOry cells and norn1allevels of glucose and proLein. An elecnoencepha- 1ogram reveals periodic spike-wave complexes. One momh larer, Lhe patient is bedridden and nonresponsive , and subsequemly dies. A section of brain tissue obtained at autopsy is shown in the image. This patient mosr likely has whkh of the following categmies of organ-specific amyloidosis? ***(A) A~ ***(B) AA ***(C) AE ***(D) AL ***(E) APrP
10 The answer is E: APrP. The brain biopsy shows spongiform degeneration of the gray mauer, characterized by indiYidual and clustered vacuoles, with no evidence of inHammation. Prion diseases are recognized clinically by slowly progressive ataxia and dementia and pathologically by the accumulation of fib tillar or insoluble prton proteins, degeneration of neurons, and vacuolization (spongiform degeneration). Spongiform encephalopathies are transmissible, and inadvertent human transmission of Cremzfeldt-Jakob disease (CJD) has followed the administration of comaminated human piruitary growth honnone, corneal n·ansplamation from a diseased donor, insufficiently sterilized neurosurgical instruments, and surgical implantation of comaminated dura. APrP amyloid is found in patiems with spong~fonn encephalopathies. P1ion proteins (PrPs) are natural plasma membrane constiruems found in a variety of cells, including the central nervous system The residual PrP, now in an altered conformation, may serve as a template for the association of addilional PrP molecules. In so doing, they confer the new PrP confonnadon (PrP""). Such altered PrP and its aggregates form fibrils with the characteristic of amyloid and are believed to play a role in a group of human and animal degenerative diseases of the cennal nervous system, such as kuru, CjD, Gerstmann-StrausslerSheinker disease , scrapie, and bovine spongiform encephalopathy (mad cow disease). The other fom1S of amyloid are not associated w1ch spongiform encephalopathies. ***Diagnosis: Creutzfeldt-]akob disease, spongifonn encephalopathy
11 What key feature distinguishes nom1al proteins from their pathologic fonns in the brain of the patient desc1ibed in Ques· tion 10? ***(A) ATPase activity ***(B) Glycolipid anchor ***(C) Three-dimensional confom1ation ***(D) Tyrosine kinase activil:)' ***(E) Zinc finger domain
11 The answer is (: Three-dimensional conformation. The normal prion gene product (P rP) is a constitutively expressed, cell-surface glycoprotein that is bound to the plasma membrane by a glycolipid anchor. The highest levels of PrP mRNA are found in CNS neurons. Remarkably, du~ nonnal cellular prion protein (PrPC) and the pathogenic (infectious) p rion protein (scrapie PrP or PrP5c) do not differ in amino acid sequence. However, they represent different three-dimensional conformations. Specifically, PTJ>C· is rich in a -helix configuration, whereas the f3·pleated sheet configuration is predominam in PrPsc_ This 1s an epigenetic phenomenon. Newly convened proteins then change other PrPc proteins into pathogenic PrPsc_ The result is an autocatalytic, e.xponentially ex-panding accrual of abnormal PrP5c About 15% of prion diseases are familial and reflect mutations that predispose to abnotmal pro teLn folding. ***Diagnosis: Cremzfeldt-]akob disease, spongi[orm encephalopathy
12 An 88-year-old woman with Alzheimer disease dies of congestive hean failure. Examination of the brain at autopsy shows bilateral atrophy of the gyri, particularly in the frontal and hippocampal cortex. What additional finding mighL be expected in the brain of this patient? ***(A) Cerebritis ***(B) Hydrocephalus ex vacuo ***(C) Ussencephaly ***(D) Pachyg)'1·ia ***(E) Periventricular patches of demyelination
12 The answer is 8: Hydrocephalus ex vacuo.. The brain of patients with Alzheimer disease loses approximately 200 gin an interval of J to 8 years. The gy1i narrow, the sulci widen, and cortical atrophy becomes apparent. In turn, these changes lead to v;idening of the lateral ventricles (hydrocephalus ex vacuo). Hydrocephalus ex vacuo refers to enlargement of the ventricular system as a compensatory response to severe brain atrophy and is unrelated to obsnuctive lesions. The atrophy is bilateral and symmen·ic, and targets the frontal and hippocampal cortex. Choices C and D are congenital disorders. Choice E is common in patients with multiple sclerosis. ***Diagnosis: Alzheimer disease, hydrocephalus ex vacuo
13 A 35-year-old man presents with a history of behavioral and personality changes and unusual involuntary movements. Physical examination reveals chorea and dystorua. The patient's mother and maternal grandfather had similar clinical symptoms. His mother died in a psychiatric institute, and his maternal grandfather commined suicide. MRl shows bilateral cerebral atrophy and enlargement of the lateral ventricles. Marked atrophy would also be expected in •vhich of the following regions of this patients brain? ***(A) Anterior horn of the spinal cord ***(B) Caudate nuclei ***(C) Cerebellum ***(D) Hypmhalamus ***(E) Substamia nigra
13 The answer is 8: Caudate nuclei. Huntington disease (HD) is an autosomal dominant genetic disorder characterized by involuntary movements of all parts of the body, deterioration of cognitive function, and often severe emotional disn1rbance. The huntingtin gene product is widely expressed in tissues tl1roughout the body and in all regions of the CNS by neurons and glia. On gross examination of brains from patients who died ofHD, the frontal conex is symmetrically and moderately atrophic, whereas the lateral ventricles appear disproportionately enlarged, owing to the loss of the nom1al convex curvamre of the caudate nuclei. There is symmetric atrophy of the caudate nuclei, with lesser involvement of the putamen. Biochemical assays at the tetmination of the disease show a marked decreased in "(-aminobutyric acid and glmamic acid decarboxylase_ Choices A, C, and D are not involved in Huntingmn disease. Pathology of the substamia nigra (choice E) characterizes Parkin5on disease. ***Diagnosis: Huntington disease
14 'vVhich of the following gene abnom1alities would be expected in the patient described in Question 13? ***(A) Deletion of an exon in the gene for presenllin ***(B) Expansion of a trinucleotide repeat ***(C) Frame shift mutation in the gene for superoxide dismutase ***(D) Nondisjunction dllling meiosis of chromosome 21 ***(E) Point mutadon in the gene for the prion protein
14 The answer is B: Expansion of a trinucleotide repeat. A large group of herediuuy neurodegenerative diseases are classified on a genetic basis as trinucleotide repeat expansion syndromes. The triplet repeat mutation disorders include Huntington disease (tiD), fragile X syndrome, and Friedreich ataxia. Trinucleotide repeats are a nom1aJ feature of many genes, but e>..'Pansion of the number of repeats confers pathogenicity. In HD, the expansion lies within the coding region of a gene segment and results in the production of a toxic protein, namely huntingtin. The other choices reflect genetic abnonnalities that are not related to Huntington disease. ***Diagnosis: Huntington disease
15 A 20-year-old woman with mild scoliosis complains of a 3-momh history of difficulty walking. Physical and neurologic examinations reveal clysanhria, lmver-limb areOexia, extensor plantar reflexes, and sensory loss. Genetic studies show evidence of a trinucleotide repeat e:l\.'])ansion syndrome. The fami ly asks for information regarding their daughter's prognosis. You are cognizant of the fan that the length of this childs trinucleotide repeat is directly related LO the rate of clinical progression, as w·e ll as the probability that she will develop which of the following life- Lhreatenlng camp lications? ***(A) Aplastic anemia ***(B) Astrocytoma ***(C) Cardiomyopathy ***(D) Cerebral amyloidosis ***(E) Tuberous sclerosis
15 The answer is C: (ardiomyopathy. Friedreich ataxia is the most common inherited at.axia. Although the inheritance pattern ls autosomal recessive, many cases arise sporadically as new mmations without a family histOT)'· The onset of symptoms is usually before age 25 years, followed by an unremitting and progressive course of abom 30 years before death. The hallmark of Friedreich ata;da is a combined ataxia of both the upper and lower limbs. Dysarthria, lower-limb areflexia, extensor plantar reflexes, and sen5ory loss also occur in most patienLS. Frequently associated systemic abnormalities are deformities of the skeleton system (scoliosis), diabetes mellitus, and hypertrophic cardiomyopathy (which commonly causes death)_ The candidate gene encodes a mitochondrial protein (frataxin) involved in iron rranspon. Friedrelch ataxia is associated with an unstable expansion of a trinucleotide repeat thm presumably imerferes with transcription or RNA processing The highest levels of fTa ta,'<in gene expression are found in the hean and spinal cord. The other choices are not associated wilh trinuclemide repeat expansion syndromes. ***Diagnosis: F1iedreicll ataxia
16 A 35-year-old woman complains of urinary incontinence and blurred vision for 2 months. A funduscopic examinadon shmvs no abnormalilies. T'NO months later, the patient develops double vision and numbness in the fingers or her left hand. MRI shows scattered plaques in the pariems brain and spilull cord. Over the next several momhs, some of these plaques diminish in size, while others appear in new locations. These plaques would most Ukely show selective loss of which of the following proteins? ***(A) f3 -Amy1oid ***(B) Glial fibrillary acidic prmein ***(C) Myelin ***(D) Synaptophysin ***(E) a.-Synuclein
16 The answer is C: Myelin. Multiple sclerosis (MS) is a chronic demyelinating disease that ls the most common chronic CNS disease o[ young adults in the United States. The disor· der affects sens01y and motor functions and is characte1ized by exacerbations and remissions over many years. MS may present \vlth a variety of symptoms, such as sensmy deficits, sphincter weak11ess, and tremors. Forty percent of cases are marked by eye problems, such as loss of visual fields, blindness in one eye, or diplopia. The demyelinated plaque is the hallmark of MS. Evolving plaques are marked by: selective loss of myelin in a region of axonal preservation; a few lymph ocytes that duster about small veirlS and arteries; an influx of macrophages; and considerable edema. Intracellular deposits of cx-synuclein (choice E) are seen in patients with Parkinson disease. ***Diagnosis: Multiple sclerosis
17 A 30-year-old woman presents with an 8-day history of mild tremor in her am15 and impaired balance when walking. Vital signs are nonnaL Her symptonlS disappear tl1e following week. About 18 momhs later, the patient experiences another episode of weakness and requires assistance when walking. Neurologic examination reveals ataxia, dysarthria, decreased Vlbratory sensation in her legs , absent abdominal reflexes, increased deep tendon re nexes, and a BabitlSki sign on the left. fifteen years after tl1e onset of symptoms, the pad em becomes bedridden and dies. A coronal section of Lhe pauents brain at autopsy is stained for myelin with luxol Cast blue (shovm in Lhe image). Which of the fo1lmv1ng histopathologic findlngs would he expected in these plaques? ***(A) Astrogliosis ***(B) Lewy bodies ***(C) Negri bodies ***(D) Neurofibrillary tangles ***(E) Myelm figures
17 The answer is A: Astrogliosis. Multiple sclerosis is punctuated by abrupt and brief episodes of clinical progression, imerspersed with periods of relative sLabUiry. Each exacerbation refiecrs the formation of additional demyelinated plaques. Plaques of demyelinated white maner are typically found around the lateral ventricles of the cerebrum, in the cerebellum, and in the spinal cord. End-stage lesions feature astrogliosis, thick-walled blood vessels, moderate perivascular inflammation, and a secondary loss of axons. Lev.ry bodies (choice B) are features of Parkinson disease. Negri bodies (choice C) are encountered in rabies. Neurofibrillary tangles (choice D) are Cearures of Alzheimer disease. Whorled "myelin figures" (choice E) represent lysosomal storage of unmetabolized gangliosides in the neurons of patients with Tay-Sachs disease. ***Diagnosis: Multiple sclerosis
18 A f5-year-old woman is rushed to the emergency room following an automobile accident. Ten hours after admission, the patient complains of a severe headache and blurred vision. An X-ray film of the cranium shows a fracture of the temporalparietal bone. Despite emergency craniotomy, the paliem dies. Which of the following pathologic findings would be ex-pected at amopsy? ***(A) Epidural hematoma ***(B) Intracerebral hemorrhage ***(C) Intraventricular hemorrhage ***(D) Subaraclmoid hemonhage ***(E) Subdural hematoma
18 The answer is A: Epidural hematoma. Epidural hematoma is the accumulation of blood berween the calvaria and the dura. It usually resulrs from a blow to t.he head, and unless treated promptly, it is generally fatal. The temporal bone is one of the thinnest bones of the skull and is particularly 'v'lllnerable to rracrure, so that seemingly minor trauma may fracture il. An epidural hematoma usually results from a traumatic bone fracture that severs the middle meningeal arter y. The other choices are not characteristic complications of temporal bone fracture. ***Diagnosis: Epidural hematoma
19 A 68~year-old woman complains of difficulty getting out of a chair. On examination, the patient shows reduced fac ial expression, a resting tremor, cogwheel rigidity, and bradykinesia (slowness of vol untary movements). The patient dies of congestive heart failure 10 years later. Microscopk examination of brain tissue at autopsy is shown in the image. The spherical, eosino philic inclusions in the cytoplasm of this pigmented neuron are composed of which of the follO\.ving proteins7 ***(A) P-Amyloid ***(B) Polyglut.amine ***(C) PrP"'- ***(D) a.-Synuclein ***(E) Tau
19 The answer is D: a.·Synudein. Parkinson disease (PO) is a common movement disorder characterized path ologically by the loss of neurons , p rimarily in the substantia nigra, and the accumulation of eosinoph ilic inclusions te nned Lewy bodies, formed by filamentous aggregates or a.-synuclein. Accumulating evidence suggests that oxidative stress produced by the auto-oxidation of catecholamines during melanin formation injures pigmented neurons in the substantia nigra by p romoting the misfolding of a.·synuclein. Clinically. PD features tremors at rest, muscular rigidiry, expressionless countenance, emotional liability, and, less common ly, cognitive impair· ments, including dementia late in the disease course. The other choices are not characterized by demarcated inclusions \Vithin neurons of the St1bstantia nigra. ***Diagnosis: Parkinson disease
20 A 50-year-old woman presents with a 3-momh histmy of easy fatigability, a smooth sore wngue, numbness and tingling of the feet, and weakness of the legs. The hemoglobin is 5.6 gldl, WBC count is 5 , 100/~LL , and platelets are 240,000/r.tl. A hematologic evaluation reveals a megaloblastic anemia that is not reversed by folate therapy. Peripheral neuropathy in this patient is most likely associated with which of the fol lowing pathologic findings? ***(A) Au·ophy of front.al conex ***(B) Atrophy of mammillary bodies ***(C) Degeneration of anterior horn cells in the spinal column ***(D) Degeneration of the posterior columns of Lhe spinal cord ***(E) Spongifonn degeneration of the cerebellum
20 The answer is D: Degeneration of the posterior columns of the spinal cord. Subacute combined degeneration results from a lack of vitamin B12 (pernicious anemia) and leads to lesions in the posterolateral portions of the spinal cord. Vitamin B 12 is required for DNA synthesis, and its deficiency results in la rge (megaloblastic) nuclei in all bone marrow lineages. A burning sensation in the soles of the feel and other paresthesias herald the onset of this rapidly progressive and poorly reversible neurologic disorder. Initially, there is symmetric myelin and axonal loss at the thoracic level of the spinal cord. With lime, the affected sp inal cord exhibits gliosis and atrophy, especially in the posterolateral areas of the cord. The other anatomic locations arc not adversely affected by vitamin B12 deficiency. ***Diagnosis: Subacme combined degeneration , pernicious anemia
21 A 60-year-old man with a history of smoking and chronic b ronchitis complains of difficulty walking. On examination, Lhe patient appears stiff and stooped, shows an expressionless face, and speaks in a monotonous voice. A tremor of his fingers is appa rent but ceases when he tries to reach for something. The patient dies 3 years later of metastatic lung cancer. At autopsy, the substantia nigra of the patient (righ t) diffe rs from that of a normal brain (left). This pathologic finding is assoctated with which of the fo llowing biochemical changes? ***(A) Increased j3-amyloid biosynthesis ***(B) Increased release o[ NMDA ***(C) Increased synthesis of GABA ***(D) Reduced levels of dopamine ***(E) Reduced t.au prmein self-assembly
21 The answer is D: Reduced levels of dopamine. The substantia nigra is a component of the extrapyramidal syslem that relays information to the basal ganglia through dopaminergic synapses. Normal aging is associated with a loss of neurons in the subsramia nigra and reduced leveLs of dopamine, but these feamres are more exaggerated in padems with Parkinson disease (P O). Degeneration in the pars compacta of the subs tantia nigra is characterized by macroscopic nigra! pallor, mkroscopic loss of pigmented neurons, pigment granules found extracellularly or \\ri thin macrophages, gliosis, and, in some surviving neurons , cytoplasmic inclusions that have an eosinophilic core surrounded by a clear halo (Lewy bodies) The vast majority of cases of Parkinson disease are idiopathic, but the disease has been recorded after viral encephalitis (von Economo encephalitis) and after Lntake of the toxic chemica] methyl-phenyl-tetrahydropytidine. None of the other choices reflects damage to the substantia nigra. ***Diagnosis: Parkinson disease
22 A 60-year-old chronic alcoholic was fm.md in a state of mental confusion. Physical and neurologic examinations reveal horizontal diplopia, strabismus, amblyopia, nystagmus, ataxia, and peripheral neuropathy The patient subsequently develops lobar pneumonia and expires. Examination of the brain at autopsy shows calcification and brownish discoloration of atrophic mammillary bodies. Petechiae in the quadligeminaJ plate and periaqueductal regions of the midbrain are also observed. Which of the foUowing best explains the pathogenesis of these clinical and pathologic findings? ***(A) AIDS-related encephalopathy ***(B) Amyotrophic lateral sclerosis ***(C) Hepatic encephalopalhy ***(D) Hepatorenal syndrome ***(E) Thiamine deficiency
22 The answer is E: Thiamine deficiency. Wernicke syndrome is secondary to thiamine (vitamin B1 ) deficiency It is characterized clinically by the rapid onset of a disturbance in thennal regulation, altered consciousness, ophthalmoplegia, nystagmus, and ataxia and pathologicalJy by lesions in the hypothalamus and mamillary bodies, the periaqueductal regions of the midbrain, and the tegmentum of the pons. Atrophy of the superior portion of cerebellar vermis also occurs. The syndrome arises most commonly in association with chronic alcoholism. Wernicke syt1drome may progress rapidly w deatll but is reversed by the adntinisrrarion of thiamine. WemickeKorsakoff syndrome refers to a state of disordered recent memory oflen compensated for by confabulation. Choices C and 0 are consequences of hepatic failure that are not related to thiamine deficiency. Choice B (amyotrophic lateral sclerosis) is a chronic neurological disorder unrelated to vitamins. ***Diagnosis: Wernicke encephalopathy
23 A 15-yea r-old boy is rushed to the emergency room after suffering a tonic-dank seizure 4 weeks after a spelunking e>.'])edidon. The boy appears initable and agitated, and his parents state that he has djfficull:y swallowing nu ids. Lumbar puncture shows numerous lymphocytes. The patiem becomes delirious , slips into a coma, and expires. Al autopsy, the brain stem shows infiltrates of lymphocytes around small blood vessels and evidence or neuronophagia. Some neurons contain eosinophilic inclusions. What is lhe proper name for these neuronal inclusion s? ***(A) Councilman bodies ***(B) Hirano bodies ***(C) Lewy bodies ***(D) Negti bodies ***(E) Psammoma bodies
23 The answer is D: Negri bodies. Rabies is an encephalitis caused by the rabies virus, which is transmiued w humans through contaminated saliva inuoduced by a bite. Dogs, wolves, foxes, and skunks are the principal reservoirs. bm the infection is also acquired from the bite of rabid bats. which often inhabit caves. Viral replication initiates at the site of the bite, and Lranspon w the CNS is mediated by viral entry into peripheral n erves. Destruction of the brainstem neurons by rabies virus initiates painful spasms of the throat, difficulty swallowing, and a tendency w aspirate fluids, which has prompted the designation "hydrophobia." Encephalopathy progresses LO death within 1 to several weeks. Pathologic features of rabies encephalitis include perivascular cuffing by lymphocytes, neuronophagia, microglial nodules, and "Negri bodies," which are distinctive eosinophilic, cytoplasmic inclusions in infected nerve cells. Councilman bodies (choice A) are remnants of apoptotic hepatocytes seen in acute viral hepatitis. Le\vy bodies (ch oice C) are features of Parkinson disease. ***Diagnosis: Rabies
24 A 10-month-old girl is brought tO the emergency room with severe, unremitting wate ry diarrhea. Her blood pressure is 80 mm Hg systolic, and the pulse is 120 per minute. Which of the following is a potentially lethal complication of systemic dehydration in this patiem? ***(A) Diffuse axonal sheming ***(B) Imravemricular hemorrhage ***(C) Midbrain hemonhage ***(D) Pontine hemorrhage ***(E) Venous sinus thrombosis
24 The answer is E: Venou-s sinus thrombosis. The cerebral veins empty into large venous sinuses, the most prominem of which is the sagittal sinus, because it accommodates the venous drainage from the supetior portions of the cerebral hemispheres. Venous sinus thrombosis in the brain is a potentially lethal compllcation of systemic dehydration, as occurs in infants with severe gastrointestinal fluid loss. Because venous obsnuction causes stagnation upstream, abrupt thrombosis of the sagittal sinus resuhs in bilateral hemorrhagic infarctions of the fronLal lobe regions. Dehydration is not a cause of hemorrhage (choices B, C, and D). Choice A (diffuse axonal shearing) re flects trauma. ***Diagnosis: Sagittal sinus thrombosis
25 A 54-year-old woman dies 48 hours after suffering severe head injuries in an automobile accident. Just before her death, her left pupiJ becomes fixed and dilated. An inferior view of the patients brain at autopsy is shown in the image. Which of the following was the most likely cause or death? ***(A) Diffuse axonal shea1ing ***(B) Laminar necrosis ***(C) Thrombosis or sagittal sin us ***(D) Transrentorial herniarion ***(E) vVatershed infarct
25 The answer is D: Tran1tentorial herniation. Head trauma can cause extensive tmracranial hemorrhage and cerebral edema. After compensawry mechanisms have been exhausted, the brain is shifted laterally away from the side of the lesion. The medial temporal lobe on the side of the l1ematoma is compressed against the midbrain to displace it downward through the opening cremed by the tentorium, a fatal event known as transtemorial herniation. Thus, the oculomotor nerve may be compressed against the edge or the tentorium, causing third· nerve palsy The pupil, generally on the side of the lesion, becomes fixed and dilated. The herniated uncus also compresses Lhe vasculature of the midbrain, especially the mes· encephalic veins. Venous stagnation in the midbrain causes further hypoxia and impairs neuronal function. Choices Band E are related to global anoxia. Diffuse axonal shearing (choice A) is a microscopic diagnosis. ***Diagnosis: Transtentorial herniation
26 Further examination of the brainstem at auwpsy of the patient described in Question 25 would most llkely reveal which of the following pathologic findings? ***(A) Cervical contusion ***(B) Duret hemorrhages ***(C) Encephalomalada ***(D) Pontine myelinolysis ***(E) Ruptured saccular anemysm
26 The answer is B: Duret hemorrhages. In patiems with transtemorial herniation, the uncus of the hippocampus is herniated downward to displace the midbrain, whlch is the site of secondary (Duret) hemorrhages. Duret hemorrhages in a case of lranstentotial herniation tend to be midline and occupy the brainstem from the upper midbrain to the midpons. Pontine myeUnolysis (choice D) is seen in chronic alcoholics. Encephalomalacia (choice C) occurs after brain infarcts. ***Diagnosis: Transtentorial herniation
27 An 18-year-old man suffers massive trauma in a motorcycle accident. ACT scan shows multiple intracerebral hemorrhages. The patient expires after 6 months in a coma. At autopsy, there are cystic cavities within the fronLal and temporal lobes, corresponding LO the areas of prior hemorrhage. These cavities were formed Ln large measure due to the phagocytic activity of which of the following cell types? ***(A) Astrocytes ***(B) Endothelial cells ***(C) Microglial cells ***(D) Nemrophils ***(E) Oligodendrocytes
27 The answer is C: Microglial cells. Microglia are phagocytic macrophage~de1ived cells of the CNS, accounting for 5% of all glial cells. ln response to necrosis, macroglia become phagocytic, accumulate lipids and other cellular debris, and are designated giuer cells. Some reactive microglia exhibit a prominent elongated nucleus, in which case they are referred to as rod cells. After microglial phagocytosis, astrocytosis (choice A) then leads to local scar formation, which persists as tell tale evidence or a prior inj ury. ***Diagnosis: Cerebral contusion
28 A 22-year-old boxer suffers a concussion during a boxing match and is mshed ro the emergency room. According LO his trainer, the blow deflected his head upward and posteriorly. Loss of consciousness in this patiem presumably occurred because of a functional paralysis of neurons in which of the following anatomic regions of his brain? ***(A) Brainstem reticular formation ***(B) Cerebellum ***(C) Hypothalamus ***(D) PeJivenllicular white matter ***(E) Temporopmielal area
28 The answer is A: Brainstem reticular formation. Concussion is defined as the transient loss or consciousness due to trauma. Consciousness is a positive neurologic activity that depends on the function or specific neurons, especially in the bratnstem reticular formation. In the current case, a blow that deflects the head upward and posteliorly, often with a rotal)' componem, impans quick torque on the brainstem and causes functional paralysis of the neurons of the redcular formation. By contrast, a blow tO the temporoparietal area (choice E) may lead w a skull fracture bm does not generally cause a concussion because lateral movemem of the cerebral hemispheres is prevemed by the falx Contusion of the cerebellum (choice B) is unlikely to cause loss of consciousness. ***Diagnosis: Subdural hematoma, concussion
29 The patient described in Question 28 persists in a vegetative coma for several months and then expires. A section of the temporal lobe shows massive proliferation of cells with a star-shaped appearance (shown in the image). Which of the following best accounts for this cellular response to injury? ***(A) Axonal regeneration ***(B) ChromaLolysis ***(C) Gliosis ***(D) Leukodystrophy ***(E) Neuron ophagia
29 The answer is(: Gliosis. Asrrocytes are star-shaped glial cells that far outnumber neurons throughout the CNS. Astmcytes proliferate locally in response w injuries (e.g .. trauma, abscess, rumors, infarcts, and hemorrhages). This process, referred to as astrocyrosis or gliosis, is readily demonstrated by immunostaining for glial fibrillary acidic protein (shown in photomicrograph). Astrocytosis evolves in hours w days and persists w an extem that is usually commensurate with the severity of the initiating injury. The result ts a uglial scar" composed of reactive astrocyt.es and their processes. Neuronophagia (choice E) is a li.mction of microglia. ***Diagnosis: Astroghosis
30 A 68-year-old obese woman (BMl = 34 kg!m2) suffers a stroke and expires. Histologic examination of the brain al autopsy reveals extensive arteriolar lipohyalinosis and numemus Charcot-Bouchard aneurysms. vVhich or the following best accounts for the pathogenesis of these autopsy findings? ***(A) Atherosclerosis ***(B) Autoimmunity ***(C) Diabetes ***(D) Hypertension ***(E) Vascuh tis
30 The answer is D: Hypertension. Hypertension compromises the imegrity or cerebral arterioles by causing the deposition of lipid in, and hyalinization of, the arterial walls, an alteration referred to as lipohyalinosis. Further weakening of the walJ leads to the formation of Charcot-Bouchard aneurysms, which are located mainly along the tmnk of a vessel rather than at its bifurcation. Although the other choices affect blood vessels, they do not typically cause cerebral microaneurysms. ***Diagnosis: Hypertensive stroke, cerebral hemorrhage
31 A 12-year-old boy is rushed to the emergency room in a coma after falling from an upper st01y window of his home. MRI shows a subdural hematoma over tl1e left hemisphere. vVhat is the most likely source of intracranial bleeding in this patient? ***(A) Bridging veins ***(B) Charcot-Bouchard aneurysm ***(C) Internal carotid anery ***(D) Middle meningeal artery ***(E) Sagittal sinus
31 The answer is A: Bridging veins. Subdural hematoma reflects torn bridging veins in the subdural space. Unlike the epidural space, the subdural space can expand. Because bleeding in this silUalion is from veins, it usually stops spontaneously after an accumulation of 25 to 50 ml because of a local tamponade effect. However, this effect also can compress severed bridging veins and cause thrombosis. Because the brain is symmetric and a force applied in the saginal plane similarly affects both cerebral hemispheres, lt is not surp1ising that subdural hematomas are frequently bilateraL Tearing of the middle meningeal arte ry (choice D) causes epidural hemorrhage. ***Diagnosis: Subdural hematoma
32 Two momhs later, the patient described in Question 31 expetiences increasing headaches and muscle weakness and suffe rs a tonic-clonic seizure. What is the most likely explanation for this patient's new clinical presentation? ***(A) Rebleeding from previous hemawma ***(B) Ruptured saccular aneurysm ***(C) Sagittal sinus thrombosis ***(D) Thromboembolism to middle meningeal arrety ***(E) Watershed infarcts
32 The an.swer is A: Rebleeding from previous hematoma. Subdural hemorrhages tend to stop spontaneously, owing to low pressure within the torn veins and external compression by the hematoma. The blood evemually coagulates, and granulation tissue grows into the wound. The organizing subdural hematoma may enlarge because of repeated trauma. Even minor trauma, such as shaking the head, may cause rebleeding. The other choices do not represent complications of subdural hemorrhage. ***Diagnosis: Subdural hemawma
33 A 5-year-old boy is brought to the emergency room with fever, vomiting, and co1wulsions. The patient is febtile to 39.5°C (l04°F). Physical examination reveals cervical rigidity and pain in the neck and knees. Acute inflammation most likely involves which anatomic region of the patiems brain? ***(A) Choroid plcx'Us ***(B) Ependyma ***(C) Hypothalamus ***(D) Lateral vennicles ***(E) Leptomeninges
33 The answer is E: Leptomeninges. Meningitis is a dangerous infection caused by a valiety of microorganisms. Leptomeningitis denotes an inflammato1y process localized to the pia/ arachnoid. This compartment houses the CSF, an excellent culture medium for most microorganisms. With few exceptions, all forms of meningitis are initiated by microorganisms, suppurative bacteria being the principal offenders. In untreated cases, deli1ium gives way to coma and death. The other choices are rarely affected by bacteria. ***Diagnosis: BaCleJial meningitis
34 A 32-year-old woman presents with a 2-day histmy of headache, vomiting, and fever. Physical examination reveals cervical rigidity and knee pain with hip flexion. Lumbar puncmre demons nates an abundance of neutrophils and decreased levels of glucose. 'vVhich of the following diseases is most likely associated with these clinical laboratory findings? ***(A) Meningococcal meningitis ***(B) Neurosarcoidosis ***(C) Staphylococcal meningitis ***(D) Tuberculous meningitis ***(E) Viral meningitis
34 The answer is A: Meningococcal meningitis. The presence of neurrophils in the CSF is the most definitive index of bacterial meningili.s By comrnst, lymphocytes are the hallmark of tuberculosis and the viral meningitides (choice E), as well as some chronic infenions. The classic signs of meningitis include cervical rigidity, knee pain with hip flexion (Kernig sign), and knee/hip fleJdon wh en the neck is flexed (Brudzinski sign). Sraphylococws (choke C) is rarely a cause of meningitis. CSF neutrophils are not predominam in ruberculous meningitis (choice D). ***Diagnosis: Meningococcal meningitis
35 A 3-day-old infant presents with a fever of 38. 7°C (l03°F) and convulsions. The infant is started on broad-spectrum antibiotics and antiviral medications but slips into a coma and expires. At autopsy; 1.he brain shows a purulem exudate in the subaraclmoid space at the base of the brain (shown in the image). What was the mosL likely cause of suppurative meningitis in this neonate? ***(A) Candida albicans ***(B) CrycJwcoccus neojorma11~ ***(C) EsdJelichia coli ***(D) Haemopltilus lliflttenzae ***(E) Nrti.<>seria meningit1dis
35 The answer is C: Escherichia coli. E. coil is the prime cause of meningitis il1 the newborn, whose resistance to Gram-negative bacteria has not fully developed. The rransplacemal transfer of matemallgG imparts protection to the newborn against many bacteria, but E. col1 requires TgM for neutral izadon. At autopsy, the brain shows a creamy e>.'Udate in the leptOmeninges (see photograph). Choices A and B do not lead to meningitis. Haemopl1iiLcs innucnzae (choice D) and Neissena meningitides (choice E) cause meningitis at a later age. ***Diagnosis: Banetial meningitis
36 A 1-year-old boy presents wilh a delay in motor developmenL Progressive muscle weakness and blindness ensue, and the patient dies within a year. The bmin at auwpsy shows swollen neurons that contain numerous lysosomes filled with lipid. Whkh of the foll o·wing is the most likely diagnosis? ***(A) AL amyloidosis ***(B) Hurler syndrome ***(C) Phenylketonuria ***(D) Tay-Sachs disease ***(E) Tuberous sclerosis
36 The answer is D: lay-Sachs disease. Tay-Sachs disease is a lethal, autosomal recessive disorder caused by an inbom deficiency ofhexosaminidase A, which permits the accumulation of ganglioside ~U in CNS neurons. The disease is fatal in infancy and early childhood. Retinal involvement increases macular transparency and is responsible for a cherry-red spot In the macula. On histologic examination, lipid droplets are seen in the cytoplasm of di~tended nerve cells of the CNS and per)pheral nervous system. Electron microscopy reveals the lipid within lysosomes in the fonn of whorled "myelin figures." Swollen neurons that exhibit marked vacu olization of the pe tikaryon and contain lysosomes filled with lipid can also occur in other lipidstorage diseases (e.g., Gaucher disease, Niemann-Pick disease). The other diseases do not produce such neuronal changes. ***Diagnosis: Tay-Sachs disease
37 A 59-year-old woman presents with headache and mild fever of 3 days in duration. On phys1cal examination, the patiem appears confused and inattentive. On the foliO\\Iing day, she is ntshed to the emergency room after suffering a generalized seizure. Lumbar puncmre shows increased levels of CSF protein, but cultures are negative , and the white cell count is not elevated. PCR analysis of the (Sf fluid shows evidence of herpes simplex type l. This infection most likely involves which of the following anatomic reg10ns of the patient's brain? ***(A) Basal ganglia ***(B) Brainstem nuclei ***(C) Cerebml hemispheres ***(D) Subependymal areas o[ the cerebral hemispheres ***(E) Tempomllobes
37 The answer is E: Temporal lobes. The manifestations of viral infections of CNS parenchyma are heterogeneous. owing to vi ral tropism ro specific regions of the brain. Herpes simplex targets the temporaJ lobes. The mechanisms of viral tropism in general are unknown bUL may reflect specific binding of vi ruses to sites on the plasma membranes of CNS cells, the ability of vimses to remain latent, or selective replication in distinct intracellular microenvironments. Brainstem nuclei (choice B) aTe affected by poliovirus. ***Diagnosis: Herpes encephalitis
38 The patient described in Question 37 is started on antiviral medication b ut becomes increasingly unresponsive and expires. Examination of affected brain tissue at amopsy would most likely reveal which of the following pathologic findings7 ***(A) Charcot-Bouchard aneurysms ***(B) Focal plaques of demyelination ***(C) Neurofibrillary tangles ***(D) Petivascular cu ffs oflymphocytes ***(E) Spongifonn degeneration
38 The answer is D: Perivascu lar cuffs of lymphocytes. The classic hallmark of most CNS viral infections is the p resence of perivascular lymphocytes around aneries and arterioles. The other ch oices are not features of acute viral infection. ***Diagnosis: Herpes encephalitis
39 A 2-day-old neonate exhibits convulsions. Imaging smdies disclose mild hydrocephaly, as well as areas of calcification in pe1ivencricular areas and in the brain s tem. The neonate is started on broad-specn·um antibiotics but dies within 2 days. Examination of the brain at autopsy reveals pink intranuclear inch.1sions in Purkinje cells. Which of the following is the most likely etiology of cerebral calcification and convulsions in this neonate? ***(A) Crypwcoccus neojorma11s ***(B) Cytomegalovims ***(C) Human immunodeficiency virus ***(D) Poliovirus ***(E) Toxoplasma gondii
39 The answer is B: Cytomegalovirus (CMV) . CMV is one of the agents of the so-called TORCH syndrome. CMV crosses the placenta to induce encephaHtis in utero. CMV panicles are demonstrated as ino·anuclear and intracytoplasmic inclusions in neurons and astrocytes. The lesions in the embryonic CNS predomjnate in the pe1iventricular areas and are characterized by necrosis and calcification. Because of the proximity of these lesions w the third ventricle and the aquedun, they are prone w induce hydrocephalus. Toxoplamosis (choice E) does not feature imranuclear inclusions. ***Diagnosis: Viral encephalitis
40 A 48-year-old man with AlDS is adm.iued to the hospital wi.t.h a headache, fever o[ 38.JOC (103"'F) , and persistent cough. His CD4 cell count is less than 500/~L l. Lumbar puncture returns cloudy fluid, and microscopic examination shows numerous encapsulated microorganisms (shown in the image). vVhich of the fo llowing pathogens is the most likely cause of meningitis in this patient? ***(A) A~pergillus flavus ***(B) Crypwcoccus neoformans ***(C) Mycobacterium tuberculosis ***(D) Neisse1ia meningitidic; ***(E) Toxoplasma gondii
40 The answer is B: Cryptococcus neoformans. Cryptococcal meningitis is an indolent infection in which Lhe virulence of Lhe. causative agent marginally exceeds the resistance o[ the host. ln most cases, it acts opponuni.stically in inmmnocompromised persons {e. g., panems who have AIDS). The organisms vary in size from 5 lO 15 pm in diameter and reproduce by budding. When a drop of comaminated CSF is mb,ed with India ink, microscopic examination shows a dear halo about the encapsulated organism. The tissue response lO C neo.formans in the meninges is rypicaUy sparse. The other choices are not typical CNS infections in patients with AIDS. ***Diagnosis: Cryptococcal meningitis
41 A .30-year-old woman presents w the emergency room complainlng that she has the "worst headache" of her life. Her temperature is 37°( (98.6°F), blood pressure 135/85 mm Hg, and pulse 90 per min me. The patiem shows no evidence of muscle weakness or am,'Cia. Imaging smdies reveal subarachnoid hemon hage, and an angiogram shows a saccular aneurysm. Which of the follmving best describes the pathogenesis of anemysm formation in this patient? ***(A) Atherosclerosis ***(B) Bacterial infection ***(C) Congemtal weakness ***(D) Diabetes mellitus ***(E) Systemic hypertension
41 The an.swer is C: Congenital weakneu. Rupture of a berry (saccular) aneurysm results in life-threatening subarachnoid hemorrhage, 'vVilh a 35% monallty during the initial hemorrhage. A sudden severe headache characteristically heralds the onset or subarachnoid hemorrhage and may be followed by coma. Berry aneurysms are the consequence of arterial defects that are presumed to a1ise during embryogenesis. The muscular layer of a blood vessel that bifurcates imo two branches may fail to interdigitate adequately across the branch point, thereby creating a point of congenital muscular weakness that is btidged onJy by endothelium, the internal elastic membrane, and a thin adventitia. Over time, the blood fiow from the parent vessel exerts pressure at the point of bifurcation and expands the congenital defecl. Atherosclerosis (choice A) results in aneurysms that produce a mass effect. Hypertension (choice E) is associated with arteriolar lipohyalinosis and induces Charcot-Bouchard aneurysms. Bacterial infection (choice B) leads to mycotic aneurysms. ***Diagnosis: Berry aneurysm
42 Which of the following is the most likely anatomic locatton of the ruptured aneurysm in the patient described in Question 417 ***(A) Circle of Willis ***(B) Internal carotid ane ty ***(C) Middle meningeal artery ***(D) Striate artery ***(E) Ven ebral artery
- - -- ... - --- - - - - - - - .f - --- - - ~ 42 The answer is A: (ircle of Willis. More than 90% of saccular aneurysms occur in the circle of Willis at b ranch points in the carotid system. They are equally distributed at the j1.mction of (1) the anteri or cerebral and anteJi or communicating arteries , (2) the internal carotid-posterior communi eating-anterior cerebral-an terior choroidal arteries, and (3) the tri furcation of the middle cerebral artery Lesions of the other arteries do not cause subarachnoid hemorrhage. ***Diagnosis: Berry aneurysm
43 A 62-year-old man witl1 a history of poorly controlled hypertension and diabetes presents with sudden onset of weakness. His blood pressure is 200/115 mm Hg, and his pulse is 80 per 1ninme. An X-ray film of the chest demonstrates cardiomegaly and pulmonary edema. A CT scan of the brain reveals intraparenchymal hemonhage. The patient becomes unresponsive and eventually expires. Which of the following was the most likely site for cerebral hemorrhage in this patient? ***(A) Basal ganglia/thalamic area ***(B) Frontal lobe cortex (conical layers IV through Vl) ***(C) Medulla ***(D) Midbrain ***(E) Sommers sector of the hippocampus
43 The answer is A: Basal ganglia/thalamic area. Cerebral hemorrhage causes stroke (apoplexy). Cerebral hemorrhages tl1at occur without trauma are referred to as "spontaneous," although most are re Ia ted to preexisting vascular lesions (Charcot-Bouchard aneurysms) or are l11e consequence of long-standing hypertension. Hypertensive intracereb ral hemorrhage occurs at preferential sites, which in order of frequency are Lhe basaJ ganglia-thalamus (65%), pons (15%), and cerebellum (8%). Necrosis of pyramidal neurons of Sommers sector in the h~ippocampus (choice E) occurs as a conseguence of global anoxia~ ***Diagnosis: Cerebral hemorrhage
44 A 42-year-old man ·with AIDS dementia complex dies of respiratory insufficiency secondary w PnewnocystLS carinU pneumonia. Examination of the brain at autopsy reveals mild cerebral atrophy, with dilation of the lateral ventricles. Which of the fo llowing best explains the pathogenesis of neuronaJ injury in this patient? ***(A) Accumulation of lysosomal storage material ***(B) Apopwsis of oligodendrocytcs ***(C) Lytic infection of neurons by HIV ***(D) Necrotizing vasculitis that results in multiple cerebral infarcts ***(E) Release of neurotoxic cytokines from macrophages
44 The answer is E: Release of neurotoxic cytokines from mac· rophages. In most patients with AIDS encephalopathy, the dLc;ease is auribmable Lo an active infection of the CNS by the vims ilSeU. Macrophages and microglial cells in the CNS arc productively infected by HIY-1. Although neurons and astrocyr.es may interact with the virus, they do nm seem w he infected but are injured indirectly by cytokines or other ncurowxic factors released by macrophages. Dementia is the most common clinical manifestation of AIDS encephalopathy, which ranges from mild Lo severe cognitive impairment, with paraJysis and loss of sensory functions. The other choices are not typical complications of AIDS. ***Diagnosis: AIDS dementia complex
45 A 27-year-old man presents wilh dementia, weakness, visual loss , and ataxia l year after receiving a cadaveric kidney transplant. Which of the follov.ling is 1.he most likely cause of this patient~ CNS disorder? ***(A) Adrenoleukodysrrophy ***(B) Gaucher disease ***(C) Metachromatic leukodysnophy ***(D) Progressive multifocalleukoencephalopathy ***(E) Subacute sclerosing panencephalitis
45 The answer is D: Progressive multifocal leukoencephalopathy (PML}. PML is a relentlessly destructive disease caused by JC virus, which principally affects the white matter in brain. Typical lesions appear as widely disseminated discrete foci of demyelinauon near rhe gray-white junction in the cerebral hemispheres and the brainstern. Most commonly, PML is a tenninal complication in immunosuppressed patients, such as those treated for cancer or lupus erythematosus, organ transplam patients, and persons with AIDS. Aclrenoleukodystrophy {choice A), Gaucher disease (choice B), and metachromatic leukodystrophy (choke C) are caused by inborn errors of metaboHsm. Subacute sclerosing panencephalilis (choice E) is a chronic, lethal, viral infection of the brain caused by measles virus. ***Diagnosis: Progressive multifocalleukoencephalopathy
A 45-year-old man with AIDS is admitted to the hospital with a productive cough, feve r, and night sweats. An X-ray film of the chest shows an ilJ-defined area of consolidation at the periphe ry of the right middle lobe and mediastinal lymphadenopathy. A spmum culture grows acid-fast baci lli. The patient develops severe headache and neck 1igidity. Which of the following brain areas is most likely affected by this patient~ in fection? ***(A) Base of the brain ***(B) Cerebellum ***(C) Hippocampus ***(D) Pe1ivemricular white matter ***(E) Temporal-occipital sulcus
46 The answer is A: Base of the brain. Tuberculous meningitis has a predilection for the base of the brain, and infarcts are often found in the distribution of the su·iate ane1ies. Inadequately treated tuberculous meningitis results in meningeal fibrosis, communicating hydrocephalus, and aneritis, with the last leading to infarclS. Less often, parenchymal tuberculosis produces tuberculomas (i.e. , soliwry masses with cemral caseous necrosis surrounded by granulomatous tissue; cholces B, C, D, and E). ***Diagnosis: Tuberculosis
47 A 65-year-old homeless man is found in the street and is brought to the emergency room with severe lethargy. He is treated with intravenous fluids bm lapses into a coma and e:ll:piTes. Examination of the brainstem at auwpsy shows a soft lesion in the tegmemum of the pons. A section stained for myelin with luxol fast blue is shown in the image. Which of the following is associated with this disorder? ***(A) AIDS ***(B) Alcoholism ***(C) Hypenension ***(D) Multiple sclerosis ***(E) Viral infection
47 The answer is B: Alcoholism. Cemral pomine myelinolysis is a rare demyelinating disorder that featu res selective demyelination in the pons. The lesions are often too small to have clinical manifestations and are discovered only at amops)~ However, some patiems develop quad1iparesis, pseudobulbar palsy, or severe depression of consciousness (pseudocoma) Central pontine myellnolysis is thought w arise from overly rapid correnion of hyponatremia in alcoholics or malnourished persons. Demyelination in patients ·with multiple sclemsis (choice D) is preferentially located in other pans of the brain. ***Diagnosis: Cemral pontine myelinosis
48 A /9-year-old man presems to the emergency room wilh severe righL-sided weakness. He has noticed increasing difficulty LLSing his right hand over the past several months and now walks •vith gTeat difficulty. His past medical history is significant for colon cancer that was resected 5 years ago. He has poorly controlled hypertension and adm.iLS to smoldng two packs of cigarenes a day for the past 50 years. A CT scan of the brain reveals a discrete globoid lesion in the frontal lobe \Vilh a prominent halo of edema. A CT-guided biopsy reveals neoplastic cells. Which of the following is the mosL likely d1. agnosL.S ?. ***(A) Craniopharyngioma ***(B) Glioblastoma multiforme ***(C) Medulloblastoma ***(D) Meningioma ***(E) Metastatic cancer
48 The answer is E: Metastatic cancer. Metastatic tumors read1 the intracranja] compartmen t through the bloodstream, generally in patients with advanced cancer. Tumors of differem organs vary in their incidence of inu·acranial metastases (e.g., melanoma-high, liver-low). Most metastatic lesions seed to the gray-vvhi te junction, reOecting the rich capillary bed in this area. A metastasis contrasts with a primary glioma (choice B) or meduUoblaswma (choice C) in its cUscrete appearance, globoid shape, and prom:inem halo of edema. ***Diagnosis: Metastatic cancer
49 A 34-year-old intravenous drug abuser presents to the emergency Toom wilh a 2.+-hour history of feve r and shaking chills. His temperature is 38.JCC (l03°F), pulse 92 per minute, and blood pressure 140/80 mm Hg. Card iac auscultation reveals a harsh systolic murmur. Blood cultures are positive for Stap11ylococcus aureuc;. The patient suddenly develops left-sided paralysis. lmaging studies would most likely reveal occlusion of which of the following arteries? ***(A) Amerior meningeal ***(B) Basilar ***(C) Cerebellar ***(D) Common carotid ***(E) Middle cerebral
49 The answer is E: Middle cerebral. Septicemia and card iac murmurs in an intravenous drug abuser suggest bacterial endocarditis. Infected thromboemboli from the heart valves cause infarcts and abscesses in va1ious organs, including brain, k-idneys, spleen, intestines, and upper and lower extremities. Because the trifu rcation of the middle cerebral an ery is a siLe of a major stepdown in vascular caliber, it is the predominant site occluded by emboli, most of which emanate from tl1e heart. The other choices are much less common sites of occlusion. ***Diagnosis: Bacterial endocarditis, cerebral infarction
50 A 56-year-old man is rushed w the emergency room after collapsing while shoveling snow. The patient has no pulse on admission but is resuscitated. Laboratory studies show elevated semm levels o[ cardiac-specific proteins, and ECG confinns a transmural infarct of the left ventricle. The patient expires 2 weeks later of cardiac tamponade. Examination of the pad em's brain at autopsy would most likely reveal necrosis of Purkinje cells and selective loss of neurons in which of the follO\ving regions? ***(A) Frontal lobes ***(B) Hippocampus ***(C) Hypothalamus ***(D) Occipital lobes ***(E) Thalamus
50 The answer is B: Hippocampus. Inadequate perf1.1sion of the brain results from generalized low blood flow due w exrracranial events that lead to global ischemia (cardiac anest, extemal hemonhage) or from occlusive cerebrovascular disease (cerebral anery thrombosis), which produces regional ischemia and often a localized infarct. Global ischemia also results from hypoxia (near-drmvning, carbon monoxjde poisoning, suffocation). The pattem of injury produced by global ischemia or hypoxia reflects the anatomy of the cerebral vasculature and the selective Vl.llnerability of individual neurons to oxygen deprivation. Although the other choices may be affected, selective neuronal sensitivity w a lack of oxygen is expressed most dramatically in the Purkinje cells of the cerebellum and the pyramJdal neurons of Sommers secwr in the hippocampus. ***Diagnosis: Global ischemia
51 A 30-year-old woman suffers massive trauma in an automobile accident and expires 4 days later of respirawry insufficiency. A horizontal section of the patient's bram at autopsy reveals numerous petechiae scattered throughout the white matter. Which of the followmg lS the most likely explanation for this pathologic findmg? ***(A) Fat embolism ***(B) Global ischemia ***(C) Occlusion of middle cerebral anery ***(D) Sepsis ***(E) Uremia
51 The answer is A: Fat embolism. Small emboli, notably those composed of fat or air, occlude capiLlaries. Fat emboli originating from bone fractures are carried downstream througl1the cerebral vessels umil the caliber of Lhe emboli exceeds that of the blood vessels, at which point they lodge and block blood flow The distal capillary endothelium becomes hypoxic and permeable, and petechiae develop, most cornmonly in the white mauer_ Although sepsis (choice D) sometimes leads to brain petechiae , the patients condition does not support this conclusion. None of the other choices are characterized by petechiae in the brain. ***Diagnosis: Fat embolism
52 A 67-year-old man with a history of ischemic hear£ disease is mshed to the emergency room after collapsing in h is garden. ACT scan demonstrates a la rge infarct of the lefl frontal lobe. The patient dies, and the brain is examined at autopsy (shown in the image). This lesion was caused by thrombosis of which of lhe following blood vessels? ***(A) External carotid artery ***(B) lnternal carotid arte1y ***(C) Middle cerebral artery ***(D) Sagittal venous sinus ***(E) Vertebral anery
52 The answer is C: Middle cerebral artery. The diversity of the neuro logic deficits caused by su-oke dLrectly rdlects the consequences of occluding different cerebral vessels. For example, the lengthy and slender striate arteries, which take migin from the proximal middle cerebral artery, are commonly occluded by atherosclerosis and thrombosis. The resultant infarct often o·ansecLS the internal capsule and produces hemiparesis or hemiplegia. The trifurcation of the middle cerebral anery is a favored site for lodgment of emboli and for thrombosis secondary to atherosclerotic damage. Occlusion of the middle cerebral anery at this si te deptives the parietal cortex of circulation and produces motor and sensory deficits. When the dominam hemisphere is involved, these lesions are commonly accompanied by aphasia. Occlusion of the vertebral artery (choice E) results in infarcts of the cerebellum. ***Diagnosis: Cerebral inJarction
53 A 70-year-old man with a history of senile dementia and a recent myocardial infarct dies of multiple organ system failure following occlusion of the superior mesenteric artery. Examination of Lhe patients brain at autopsy reveals aneurysmal dilation of the basilar artery (shown in the image). Which of the following is the most common complication of this pathologic finding? ***(A) Dissection ***(B) HemotThage ***(C) Infection ***(D) Tbrombosis ***(E) Transformation
53 The answer is D: Thrombosis. Aneurysms caused by atherosclerosis are localized mainly in major cerebral aneries (vertebral, basilar, and internal carotid), which are favored siLes of atherosclerosis. Fibrous replacement of the media and destruction of the internal elastic membrane weakens the arterial waU and causes aneurysmal dilation. Although hemorrhage (choice B) and d issection (chmce A) may occur, the major complication of an atherosclerotic aneurysm is thrombosis. ***Diagnosis: Atherosclerotic aneurysm
54 A 55-year-old man is brought to the emergency room after a near-drowning accident while boating. The patiem has no pulse when the paramedics anive, but he is resuscitated. The patient never regams consciousness and expires 3 days later. Examination of the brain at autopsy reveals a watershed zone of infarction in the left cerebral hemisphere. Which of the fol lo'Ning best desc1ibes the pathogenesis of this infarct? ***(A) Dissemmated imravascular coagulation ***(B) Prolonged hypotension ***(C) Sagittal sinus thrombosis ***(D) Spomaneous cerebral hemorrhage ***(E) Thromboembolism
54 The answer is 8: Prolonged hypotension. The an te tior, middle, and postelior ce rebral arteries perfuse partially overlapping territories, but there arc no anastomoses b etween their terminal branches. Because this overlap zone is not as rich ly perfused as the primary ten itories of the anLerior and the m.iddle cerebral arteries, reduced blood flow in these arteries will diminish perf-usion more severely in the partial overlap zone (watershed area), thereby causin g a parasagittal watershed infarct. None of the other choices are associated with watershed infarcts. ***Diagnosis: Watershed infarct
55 A 52-year-old man is brought to the emergency room 2 hours after being involved in an automobile accident. The patient denies striking his head, ahhough hLc; head >vas thrust forward and backward. His vital signs are normal. and he returns home. The follmving day, the patient's wife notices that he is lethargic. By the time the ambulance anives at the emergency room, the patient is comatose. Which of the following is the most li.kely cause of the decline in mental status in this patient? ***(A) Diffuse axonal injury ***(B) Duret hemorrhages ***(C) Ruptured saccular aneurysm ***(D) Spinal cord contusions ***(E) Watershed infarcts
55 The answer is A: Diffuse axonal injury. The consequences of traumatic brain inJury may be imernal and subtle. The parasagittal cortex is anchored to arachnoid villi, whereas the lateral aspects of the cerebrum move more free1>~ This anatomtca1 feature , together with the d.ifferential density of gray and wh.ite matter, pem1its generation of shearing forces between differem brain regions , leading to diffuse axonal shearing injmies, particularly in vehicular accidents. Shearing injuries can diston or disntpt axons, causing them to retract into "sphe roids ,~ as well as lose myelin. This type of injury ty-pically occurs in parasagitt.al white matter and may be accompanied by multiple small hemorrhages Duret hemorrhage (choice B) occurs in the uncus during transtentmial herniation. ***Diagnosis: Traumatic brain injury
56 A 45-year-old man presents with weakness and wasting of the muscles of his light hand for 8 momhs. Ph)'Sical examination shows fascicularjons of the hand. The patiem's speech is impaired, and 6 years later, he dies of respiratory insufficiency Autopsy shows atrophy of ventral roots in the spinal cord. Which of the following is lhe most likely diagnosis? ***(A) Amyotrophic lateral sclerosis ***(B) Gerstman Straussler-Scheinker diseas-e ***(C) Humington disease ***(D) Multiple infarct dementia ***(E) Pick disease
56 The answer is A: Amyotrophic lateral sclerosis (ALS}. ALS is a degenerative disease of motor neurons of the brain and spinal cord that resulls in progressive weakness and wasting of the extremities and evenruaUy impairment of respiratory muscles. The disease affects motor neurons in three locations: (1) the anterior horn cells of the spinal cord; (2) the motor nuclei of the brains tern, particularly the hypoglossal nuclei; and (3) the upper motor neurons of the cerebral cortex The injury w the n1otor neurons leads to the degeneration of their axons, visualized in striking alterations of the lateral pyramidal pathways in the spinal cord. The defining hiswlogic change m ALS is a loss of large motor neurons accompanied by mild gliosis. ALS begins as weakness and wasting of the muscles of the hand, often accompanied by painful cramps o£ the muscles of the ann. The other choices do not affect the moror neurons o£ the spinal cord . ***Diagnosis: Amyotrophic lateral sclerosis
57 A 6-month-old girl with metachromatic leukodystrophy (MLD) fails to thrive and rues of respiratory insufficiency. Histologic examination or the child's brain at autopsy shows marked accumulation of metachromatic material in the white Jnatter and prominent astrogliosis. Laboratory studies confirm an accumulation of ce rebroside. This patient most likely suffered from an inbom error of metabolism, characterized by a deficiency of which or the following enzymes? ***(A) Arylsul fatase ***(B) Galactosidase ***(C) Mannosidase ***(D) N-acetyl-hexosaminidase ***(E) Phenylalanine hydroxylase
57 The answer is A: Arylsulfatase. MLD, the most common leukodystrophy, is an autosomal recessive disorder of myelin metabolism that is characteJized by the accumulation of a cerebroside (galactosyl sulfatide) in the white matter or the brain and peripheral nerves. MLD is caused by a deficiency in the activity of arylsulfatase, a lysosomal enzyme involved in the degradation of myelin sulfatides. Accordingly, there is progressive accumulation of sulfatides \Vithin the lysosomes of myelin-Conning Schwann cells and oligodendrocytes. Although the other choices are involved in congenital metabolic disorders, their deficiencies do nm cause metachromatic leukodysu·ophy. ***Diagnosis: Metachromatic leukodystrophy
58 A 5-year-old girl exhibits a progressive decline in motor and sensory functions and is quickly reduced lO a vegetative state. MRI reveals con fluent, bilaterally synm1etric, demyelination. At autopsy, the brain shows gliosis and perivascular infi ltrates of lymphocytes. Laboratory studies disclose high levels of sanuared very long-chain fany acids in tissues and body fluids. This patient's leukodystrophy is most likely caused by an inborn error in the function of ·what ceU component? ***(A) Golgi apparams ***(B) Mitochond1ia ***(C) Nuclear envelope ***(D) Peroxisome ***(E) Plasma membrane
58 The answer is D: Peroxisome. Adrenoleukodystrophy (ALD) refers to an X·linked disorder in which dysfunction of the adrenal co rtex and demyelination of the nervous system are associated with high levels of saturated very long-chain fatty acids in tissues and body fluids. ALD occms in children between the ages of 3 and 10 yeaTs. The disease progresses rapidly, and the body is quickly reduced to a vegetative slate. A defect in the peroxisomal membrane prevents the normal activation of free fatty acids by the .addition of coenzyme A. As a result, these fatLy acids accumulate in gangliosides and myelin. Defects in the other organelles are not related to the accumulation of very long-chain fatty acids. ***Diagnosis: Adrenoleukodystrophy
59 An 8-month-old boy exhibns severe motor, sensory, and cognitive impainnems. Brain biopsy shows a disease of ,;vhite matter characterized by the accumulation of "globoid cells." Biochemical studies reveal an absence of galacwcerebroside ~-galactosidase activity. What is the appropriaLe diagnosis? ***(A) Alexander disease ***(B) Hurler disease ***(C) Krabbe disease ***(D) Metachromatic leukodystrophy ***(E) Glycogen swrage disease
59 The answer i.s C: Krabbe disease. Krabbe disease is a rapidly progressive, invariably fatal. autosomal recesstve neurologic disorder caused by a deficiency of galactocerebroside 13- galactosidases. The condition appears in young infants and is defined by the presence of perivascular aggregates of mononuclear and multinucleated cells (globoid cells) in the white matter. The globoid cells are macrophages that contain undigested galactosylceramide. Alexander disease (choke A) is a rare neurologic disorder, also of infants, characterized by mutations in Lhe gene encoding glial fibrillary acidic protein, which leads to aggregates of fibrous structures, known as Rosenthal fibers. Hurler disease (choice B) features the accumulation of muco· polysaccharides. ***Diagnosis: Krabbe disease
60 An 8-year-old girl is brought to the physician by her parents, who express concerns about their daughters' academic performance and recent changes in behavior. Her vital signs are nonnal. Neumlogic examination reveals motor and sensory impairments. Lumbar puncture shows no biochemical abnormalities. At autopsy, the brain shows prominent intranuclear inclusions in neurons and oligodendroglia, marked gliosis in gray and white matter, and demyelination. PCR analysis of the brain tissue shows the presence of a defective form of Lhe measles virus. Which of the following is the appropriate diagnosis? ***(A) Fatal familial insomnia ***(B) Granulomatous vasculitis of the central nervous system ***(C) Progressive multifocal leukoencephalopathy ***(D) Spongifonn encephalopathy ***(E) Subacute sclerosing panencephalitis
60 The answer is E: Subacute sclerosing panencephalitis (SSPE). SSPE is a chronic, lethal, viral i11rection of the b rain caused by a reactivation of a latem measles virus. Jn children, the course is protracted, and inflammation occurs primarily in cerebral gray matter. ln adulls, SSPE may follow a more rapid course. The infection is highlighted by prominent intranuclear inclusions in neurons and oUgodendroglia, marked gUosis in affected gray and white matter (hence scle rosing), patchy loss of myelin, and ubiquitous petivascular lymphocytes and macrophages. Choices A and D are prion diseases. Choice C is caused by activation of a latent] C virus. ***Diagnosis: Subacute sclerosing panencephalitis
61 A 36-year-old woman with inoperable brain cancer lapses into a coma and dies of cardiorespiratory faiJure. A horizontal secdon of the patients brain at autopsy is sh0\11,'11 in the image. Dilation of the ventricular system in this patient was most likely caused by obstruction at which of the following anatomic locations? ***(A) Aqueduct of Sylvius ***(B) Cemral spinal canal ***(C) Foramen of Luschka ***(D) Foramen of Magendie ***(E) Virchow-Robin spaces
61 The answer is A: Aqueduct of Sylvius. Hydrocephalus refers to dilation of the vemricles by accumulated CSF When obstrucdon to the flow of CSF in the brain is \.Vithin the vemrides, hydrocephalus is designated "noncommunicating.'' In noncommunicating hydrocephalus, the Oow of CSF through the venuicular system may be obstructed at the aqueduct of Sylvius by congenital malfonnadons, neoplasms (as in this case), inOammanon, or hemorrhage. The mass of an intracranial neoplasm, combined with edema or hydrocephalus, causes increased imracranial pressure, which leads w headaches and vomiting Choices C and D connect the 4th ventricle with the subarachnoid space and are the ptimary routes for escape of CSF into the posterior cerebelJomedullary cistem (cisterna magna). Virchmv-Robm spaces (choice E) surround blood vessels as they emer the brain. ***Diagnosis: Hydrocephalus
62 A 60-year-old man is brought to the emergency room in a coma. Physical examination reveals an emaciated man with a distended abdomen, jaundice, ascites, and a slightly enlarged liver and spleen. Laboratory studies show elevated levels of blood ammonia. A liver biopsy demonstrates cirrhosis. Histologic examinaLiOn of brain tissue rrom this patient would most likely shmv prominent changes in the nuclei of which of the following cell types? ***(A) Astmcytes ***(B) Ependymal lining cells ***(C) Microglia ***(D) Neurons ***(E) Ollgodendroglia
62 The answer is A: Astrocytes. Hepatic encephalopathy is a common clinical expression of liver failure, manifested as delirium, seizures , and coma. ln general, the clinical sympwms greatly exceed their morphologic conelates, which are restricted to the appearance of altered astroglia (tenned Alzheimer type ll astrocytes), which show enlarged nuclei and marginated chromatin, especially in the thalamus. The development or hepatic encepllalopathy is caused by increased serum concentrations of neurotoxic substances, including ammonia. None of the other choices include cells that are altered by hepatic encephalopathy. ***Diagnosis: Hepatic encephalopathy
63 A 2·year·old boy is brought to the physician by his parents, who complain that their son continually loses his balance. They also reponthat h is speech seems more slurred. Physical ~xamination confirms the truncal ataxia and wide-based gait The chiJd appears lethargic, and there is bobbing of the head \Vhlle he is silting. Muscle tone is normaL This patient may have a midline tumor in which anatomic portion of the brain? ***(A) Cerebellum ***(B) Corpus callosum ***(C) Frontal lobes ***(D) Hypothalamus ***(E) Meninges
63 The answer is A: Cerebellum. The neuroectoderm may (albeit infrequently) give rise to a neoplasm of neuronal heritage. These tumors occur most often in childhood, and their cellular composition is usually primitive. An example is meduUoblastoma, which arises in the cerebellum, generally in Lhe first decade of life. This entity is usually situated in the vern1is. lts growth is rapid and regional infiltration is ex:tensive. Children with medulloblastoma are first seen with cerebellar dysfunction or hydrocephalus. Tumors in the other areas do not ordinarily produce the symptoms desc1ibed. ***Diagnosis: Medulloblastoma
64 A lumbar puncture is perfmmed ill the patiem described in Question 63, and neoplastic cells are found. A suboccipital cran:ioLDmy is performed, and a tumor is resected. Microscopic examination of the surgical specimen would most likely reveal which or the following histologic pattems? ***(A) Malignant epithelial cells wi.th prominent tonofilamems ***(B) Perivascular collections of n eoplastic lymphocytes ***(C) Small, hyperchromatic cells and rare neuroblastic rosettes ***(D) Vacuolated tumor cells and an abundant capillary network ***(E) Whorled pattem of meningmhelial cells and psammoma bodies
64 The answer is C: Small, hyperchromaticcells and rare neuroblastic rosettes. Medulloblastomas and, less commonly ependymomas and pineal parenchymal tumors, have a propensity to disseminate by way of the CSF throughout the CNS. Since medullobk'l.Stomas are close to the fourth ventricle, they may disseminate downstream into the spinal cord. The detection of neuronal and progenitor cell markers (e .g., synapwphysi11 or nestin) may be diagnostically informative in distinguishing these rumors from metastatic epithelial tumors, lymphomas, or other neuroectodermal malignancies. Medulloblastomas are characterized by cells with hyperchromatic, round-to-oval nuclei and scant cytoplasm, wh ich often crowd together with no structural pattern. The neuroblastic character of the cells is occasionally expressed in rosette formation, a histologic feature of neuroblasts. The other choices do not describe midline, cerebellar tumors arising in children. ***Diagnosis: Medullohlaswma
65 A 2.4-year-old woman presents with a 4-month history of headaches and unsteadiness in walking. She reports a tendency to fall to the tight and a loss of coordinated movements in her right hand and leg. Her father and pmernal aum died of renal cell carcLnoma. A paternal grandfather had a history of spinal cord tumors, and her sister is seeing an ophthalmologist Cor retinal angiomas. On ~xamination , the woman has a widebased gaiL She is unable to stand on her right leg and has intentional tremor and dysdiadokinesia of the right upper extremity. MRI reveals a right-sided parasagi ttal n.1mor of the cerebellum. A biopsy of the lesion demonstrates a hemangioblastoma. Whkh of the following is Lhe most likely diagnosis? ***(A) Down syndrome ***(B) Multiple endocrine neoplasia type 2A (Sipple syndrome) ***(C) NeurofibromaLOsis lype 1 ***(D) Sturge-Weber syndrome ***(E) Von Hippel-lindau syndrome
65 The answer is E: Von Hippei·Undau syndrome. Von HippelUndau syndrome is a hereditary disorder in which cerebellar hemangioblastomas are associated with reti.nal angiomas and other tumors (e g., renal ce ll carcinoma) due to mutations in the VHltumor suppressor gene. Hemangioblastoma is a highly vascularized tumor that originates p redominantly in t11e cerebellum. lt features endotl1elium-lined canals interspersed with plump cells. ln 20% of cases, these cells secrete erythropoietin and induce polycythemia. Smrge-Weber syndrome (choice D) is a nonfamilial congenital disorder that feamres a facial pon-wine stain (nevus Jlammet~s) and angiomas of me leptomeninges. The mher choices do not feature vascular mmors. ***Diagnosis: Von Hippd-Lindau syndrome, hemangioblastoma
66 A 65-year-old woman \vith a history of breast cancer and a recent melanoma presents to the emergency room following a tonic-clonic seizure. Blood chemistry values are within nor· mal limits. There is no history of drug or alcohol use. MRI of the brain shows bilateral cerebral edema and a cysLic, frontal lobe lesion. Frozen section obtained from a CT-guicled biopsy reveals a hemonhagic nodule of neoplastic cells. Immunohistochemical stains for which of the following antigens would be most helpful in making your diagnosis definitive for melanoma? ***(A) Alpha-fetoprotein ***(B) HMB-45 ***(C) Human chorionic gonadotropin ***(D) Neuron-specific enolase ***(E) Synaptophysin
66 The answer is 8: HMB-45. Metasratic tumors are the most common intracrarual neoplasms. They reach me intracranial compartment through the bloodsrream, generally in patienLS vvith advanced cancer. Tumors of different organs vmy in their incidence of imracranial metastases. for example, a patiem wilh disseminated melanoma has a greater than 50% likelihood of acquiring intracranial metastases, whereas the incidence of such metastases in carcinoma of the breast and lung ts 35%, and the incidence for cancer of the kidney or colon is only 5%. I-lMB-45 is a useful tumor marker for melanoma. Neuron- specific enolase (choice D) and synap tophysin (choice E) are markers fo r neurons. ***Diagnosis: Melanoma, metastatic cancer
67 A 68-ycar-old man presents with a 2-week history of LOmc-clonJc seizures that imuall}' mvolve his left arm but have more recentlr progressed to involve h1s left leg. The seizures are accompamed by muscle weakness but no other neurologic signs. The cranial nerYcs arc intact, and the Babinski sign is present. A CT scan reveals a mass m the left cerebral hemisphere. A left frontopar ietal craniotomy is performed. Histologic exnmmation of the brain biopsy is shown in the image. Which of the following is the appropnate diagnosis? ***(A) Craniopharyngioma ***(B) Ependymoma ***(C) Ganglioglioma ***(D) Glioblaswma multiforme ***(E) Meningioma
67 The answer is D: Glioblastoma multiforme. Glioblastoma multifonne is a brainmmor composed of malignam astrogJja] cells, which show marked pleomorphism and are often rnulnnucleated. &cause of their invasive properties and vascular changes ("aneritic obliteration"), the tumors feature patchy yellow areas of necrosis and red zones of hemorrhage. The tem1 multifom1e derives from the variegated gross appearance of these mmors and the histologic pleomorphism of the tumor cells. The other choices do not display Lhe characteristic palisading of mmor cells around necrotic areas (see phowmkrograph). ***Diagnosis: Glioblastoma mulLiforme
68 A 65-year-old woman presents with a 3-week history of intractable headaches. Her vital signs and CBC are normaL Two weeks later, the patiem develops left-sided hemiparesis. MRl reveals a large, necrotic tumor in the right hemisphere of the cerebmm, extending across the corpus callosum into the left hemisphere. A coronal section of the patient's brain at autopsy is shown in the image. This tumor is most likely derived from which of the foUowing cell types? ***(A) Astrocytes ***(B) Ependymallming cells ***(C) Microglia ***(D) Neurons ***(E) Oligodendrogha
68 The answer is A: Astrocytes. Unlike the other choices, glioblastoma mulrifonne can invade the comTalateral hemisphere across the corpus callosum and present as a bilateral buuerny-like lesion. None of me other cells contribute w glioblastoma multiforme. ***Diagnosis: Glioblastoma mulrifom1e
69 A 50-year-old man presents with a 5-month hisrory of severe headaches. Vital signs and CBC are normal. Imaging studies demonstrale a mass in the fourth ventricle and hydrocephalus. The results of a CT-guided biopsy are shown in the image. ·what is the app ropriate diag11osis for this patients malignant neoplasm? ***(A) Craniopharyngioma ***(B) Ependymoma ***(C) Glioblastoma multiforme ***(D) Oligodendroglioma ***(E) Schwannoma
69 The answer is 8: Ependymoma. Ependymoma ong~nates in the lining of the cavities that contain cerebrospinal fiuid. The tumor is most common in the founh ventricle, where ir produces obstruction and results in hydrocephalus. The cells of an ependymoma characteiistically have an "epithelial" appearance, similar to that of no rmal ependymal cells. They may anange around blood vessels forming perivascular pseudoroseues. Ependymomas may also origb'late from the lining of the central canal of the spinal cord and the filum terminale. Together with astrocytomas (choice A), ependymomas are the most common neoplasms of the spinal cord. Because of their central location, they cannot be easily resected. ***Diagnosis: Ependymoma
70 A -+5 -year-old woman is brought to the emergency room after ex-periencing a generalized seizure. An X-ray film of the skuiJ reveals a lytic bone mass. A CBC is normal. A portion of the skull and the adherent mass are removed. Microscopic examination o[ lhe surgical specimen is shown in the image. 'vVhat Lo; the appropriate diagnosis? ***(A) Gltoblastoma muluforme ***(B) Hemangioblastoma ***(C) Mcdulloblaswma ***(D) Meningioma ***(E) Oligodendroglioma
70 The answer is D: Meningioma. Meningiomas are benign lmracranial tumors tJ1at arise from the arachnoid \illi and produce symptoms by compressing adjacent brain tissue. They account for almost 20% of all primary intracranial neoplasms. Meningiomas occur at almost any intracran1al sile bm are most common in parasagiual regions of the cerebral hemispheres, the olfactory groove, and the lateral sphenoid wing. On gross examination, most meningiomas appear as well-drcumscJibed, firm, bosselated masses or variable size. The hisrologic hallmark of meningiomas is a whorled pattern of "meningothelial" cells (see phoromicrograph). The indolent grow1.h of meningiomas enables them to enlarge slowly for years before becoming symptomatic, during 'vhich time they displace the brain bm do not infihrate it. Although benign, meningiomas have a propensity to erode contiguous bone. Choices A, C, and E are malignant bone tumors. Choice B (hemangioblastoma) is characterized by vascular proliferation . ***Diagnosis: Meningioma
71 A 26-year~ol d woman with a history of bitemporal hemianopsia and diabetes insipidus develops meningococcal infection and dies of septic shock. Examination of the brain at autopsy reveals a large cystic tumor mass replacing the midline structures in the region of the hypothalamus (shown in the image) This tumor is derived from epithelial cells originating in which of the following embryonic structures? • ***(A) Diencephalon ***(B) Mesencepl1alon ***(C) Nasopharynx ***(D) Prosencephalon ***(E) Th ird pharyngeal arch
71 The answer is C: Nasopharynx. Craniopharyngiomas are solid and cystic lesions located above the sella turcica that arise from the epithelium of Rathke pouch. This structure is a part of the embryonic nasopharyn"-x that migrates cephalad and gives origin to Lhe anterior lobe of the h)1Jophysis. Some cystic lesions are lined by squamous epithelium, whereas others, referred to as adamantinomas, are solid and resemble tumors of dentigerous origin. Neoplasms derived from ectopic tissues compress adjacent snuctures. Craniopharyngiomas generally become sympwmatic in the first two decades of life, creating visual deficits and headaches. They may cause pituitary failure, including diabetes insipidus. Choices A, B, and D represem neural structures. The third pharyngeal arch. (choice E) is distal to the nasopharynx and gives rise LO the thymus and parathyroid glands. ***Diagnosis: Craniopharyngioma
72 A 48-year-old man with AIDS is admiued to the hospital with a fever of 38. 7<>C (l039F), persistent cough, and diarrhea. His CD4 cell comu is less than 500/J.lL The patient is staned on broad-spectrum antibiotics. He has also expe1ienced a recent decline in cognitive function. He is at increased 1isk of developing which of the following CN$ neoplasms? ***(A) Ependymoma ***(B) GlioblasLOma ***(C) Lymphoma ***(D) Medulloblastoma ***(E) Oligodendroglioma
72 The answer is C: Lymphoma. Lymphomas may arise as a primary B-celJ lesion in the brain in a manner analogous lo its occunence in the stomach, small bowel, or testis, but the overwhelming majority of lymphomas are metastatic to the brain from other si tes. Primary lymphoma of the brain often originates deep in the cerebral hemispheres, commonly in bilateral pe1iventticular positions. A mLxture of small and large lymphocytes is angiocentric. Lymphomas often arise in the context of immunosuppression and in patients with AIDS. In some instances, they have been linked etiologically to infecrion with EBV AIDS does not give tise to tumors of neural origin (choices A, B, D, and E). ***Diagnosis: Lymphoma
73 A 14-year-old boy complains of a 4-momh history of fatigue, abdominal pain, and yellowing of his eyes and skin. Physical examination shows tremor of both hands, lack of coordination, and mild jaundice An ophthalmic examination reveals Kayser-Fleischer rings. Degenerative changes are present il1 which of the following anatomic regions of the CNS in this patient? ***(A) Cerebellum ***(B) Corpu..s striamm ***(C) Paravennicular white matter ***(D) Pons ***(E) Temporal-occipiml sulcus
73 The answer is B: Corpus striatum. Wilson disease is an autosomal recessive condition in which excess copper may deposit in the tiver and brain. Chronic hepatitis leads to cirrhosis in young people. Ocular lesions, so-called Kayser-Fleischer rings, represent deposition of copper in Descemet membrane in the iris. Lack of coordination and tremor (n'trapyramidal neurologic symptoms) are related to degenerative changes in the corpus sttiatum. Deposition of copper is not observed m the other anatomic regions of the brain. ***Diagnosis: Wllson disease
74 A 50-year-old man presents to the emergency room after suffering an epileptic seizure. Vltal signs are nonnal. An X-ray of the patient's head shows a mass in the left cerebral hemisphere \vith scattered foci of calcification. Histologic examination of a brain biopsy is shown in the image. Which of the following is the approp1i ate d iagnosis? ***(A) Ependymoma ***(B) GlioblastOma ***(C) Hemangioblastoma ***(D) Me1~ingioma ***(E) OHgodendroglioma
74 The answer is E: Oligodendroglioma. OHgodendrogliomas arise in the white matter and grow slowly They occur predominantly in the white maner of the cerebral hemispheres or adults. Histologically, the tumors have small rounded nuclei similar to normal oligodendrocytes, bm they also exhibit increased cell density and cellular pleomorphism. Cakospherites, which may be visualized radiographically, are occasionally scauered randomly throughout the lesion. The mher choices do not fearure unifonn monomorphic cells. ***Diagnosis: Oligodendroglioma
75 A 20-year-old man complains of increasing difficu lty in hearing over the past several years. Physical examination confirms bilateral sensor ineural hearing deficits_ MRl discloses bilateral cerebellopomine angle tumors, consistent with schwannomas. This patiem has a strong family hiswry for benign uunors, includmg low-grade gliomas and meningiomas on his mothers side of the family. v\lhich of the following is the probable diagnosis? ***(A) Neurofibromatosis type 1 ***(B) Neurofibromawsis type 2 ***(C) Sturge-Weber syndrome ***(D) Tuberous sclerosis ***(E) Von Hippel-Lindau syndrome
75 The answer is 8: Neurofibromatosis type 2. Neurofibromatosis type 2 refers to a syndrome defined by bilateral tumors of the eighth cranial nerve (acoustic neuromas), and commonly; by meningiomas and gliomas. Acoustic neuromas are intracranial schwannomas that are restricted to the eighth cranial nerve. Some schwannomas exhibit deletions or mutations of the Nf2 gene. Neurofibromatosis type 1 (choice A) exhibits neurofibromas but not acoustic neuromas. None of the other choices lead to acoustic Schwannomas. ***Diagnosis: Neurofibromatosis type 2
76 An 80-year-old woman wanders away from a mu·sing home and dies in a pedesn·ian-motor vehicle accident. A silverstained section of her brain at autopsy reveals numerous lesions in the cerebral cortex (shown in the image). Which of the following tenns best desctibes these pathologic findings? ***(A) Granulo\·acuolar degeneration ***(B) Lewy bodies ***(C) Ncurmc plaques ***(D) Neurofibrillary tangles ***(E) Spongifonn cncephalopalhy
76 The answer is C: Neuritic plaques. Microscopic changes in Alzhebner disease are dominated by the presence of senile (neuritic) plaques, neurofib1illary tangles, and neuron loss. In end-stage disease, neurili.c plaques converge to occupy large volumes of the cerebral gray matter. The plaques are positive for Congo red and exhib1t immunoreactivity for AP at the core and periphery. Neurofib1illary tangles are formed by large ino·acytoplasmic masses of tau filaments. Remarkably. most cases of Alzheimer disease are also associated with Lewy bodies. Thus, Alzheimer disease features a "LTiple '· brain amyloidosis, O\.ving to accumulations of filamentous tau, Ap, and a -synuclein. Granulovacuolar degeneration (choice A), Lewy bodies (choice B), and neurofibrillat)' tangles (choice D) are all seen in the brains of patients with Alzheimer disease, however, these lesions are not evident in the slide that accompanies this case (see phoromicrog, ·aph). Spongiform encephalopathy (clustered vacuoles within the gray numer, choice E) is feamred in prion disease. ***Diagnosis: Alzheimer disease
77 A 14-year-old girl is rushed to the emergency room after receiving a gunshot wound to the head and is pronounced dead on arrival. 1 [this child died due to an immediate "blast effec t" of the penetrating wound, the autopsy would IJkely show herniation of the tonsils of the cerebellum into whkh anatomic space? ***(A) Aqueduct of Sylvius ***(B) Cemral spinal canal ***(C) foramen magnum ***(D) foramen of Magendie ***(E) Posterior fossa
77 The answer is C: Foramen magnum. Peneuaring wounds produce hemonhage and blast effects. The "blast effect" of a high-velocity projectile causes an immediate increase in supratentorial pressure and results in deat]l because of impaction of Lhe cerebellum and medulla into the foramen magnum. A low-velociryprojectile iJ1creases the pressure at a more gradual rate through hemon hage and edema. The other choices do noL represent areas of hemiation. ***Diagnosis: Bullet wound
78 A 60-year-old man with a 15-year histmy of diabetes mellims t:ype 2 complains of deep burnmg pain and sensitivity w rouch over his hands and fingers. Nerve conducoon smdies show slow o·ansmission of impulses and diminished muse le stretch reflexes in both ankles and knees. Sensations w vibrations and light touch are also markedly diminished. Laboratory analysis of CSf shows no biochemical abnom1alities. Which of the following is the most likely type of peripheral nerve disease in this paLient? ***(A) AULonomic neuropathy ***(B) Distal polyneuropalhy ***(C) Inflamma tory neuropathy ***(D) Mononeuropathy ***(E) Paraprotelnemk neuropathy
78 The an:sweris B: Distal polyneuropathy. Peripheral neuropathy is a process that arfects the function of one or more peripheral nerves. Diabetic neuropathy is the most common neuropar.J•y in the United States. Other common causes include alcoholism, renal failure, neurotoxic dmgs, amoimmune diseases, monoclonal gammopathy, and HlV infection. The pathologic findings are limited to axonal degeneration or segmental demyelination, or a combination of both. Diabetes affects both the sensory and the mowr portions of the petipheral nervous system and most often presents as distal polyneuropathy. Autonomic neuropathy (choice A) and mononeuropathy (choice D) are less common. The pathogenesis of these disorders is nor f·uUy understood, but they may be related tO metabolic disturbances or lO disease o[ small blood vessels. Vasculitis and paraproteinemia (choices C and E) are not features of diabetes. Peti pheral sensory or sensorimotor neuropathy typically presems with cramps or deep burning pain, cutaneous hyperesthesia, or insensitivity to changes of temperature. ***Diagnosis: Diabetes mellitus
79 A 35-year-old man with Dovvn syndrome dies of acme lymphoblastic leukemia. Gross examination of the patient's brain at autopsy shows mild microcephaly and underdevelopment of the superior temporal gyri. Histologic examination would mosL lil<ely show which of the following neuropathologic cllanges? ***(A) AA amyloidosis ***(B) Lewy bodies ***(C) Negri bodies ***(D) NeurofibrillaLy tangles ***(E) Spongiform encephalopathy
79 The answer is D: Neurofibrillary tangles. One of the most intriguing neurologic features of Down syndrome (trisomy 21) is its association \Vilh Alzheimer disease. The morphologic lesions characteristic of Alzheimer disease progress in all patients with Down syndrome and are universally demonstrable by age 35 years. These changes include granulovacuolar degeneration, neurofibrillary tangles, senile (neUJitic) plaques, and loss of neurons. The gene for amyloid precursor protein (APP) is located on chromosome 21, and the additional dose of the gene product in patients witl1 trisomy 21 may predispose LO precocious accumulation of Af:l. Some patiems with the famllial form of Alzheimer disease harbor mutations in APP or prese nilin genes. These mutations lead to increased production of A ~-t11 e amyloidogenic fragmem of APP. None of the other choices are associated with the pathogenesis of Down syndrome or Alzheimer disease. ***Diagnosis: Down syndrome, amyloidosis
1 A 15-year-old boy who emigrated LO the United Stares from Sudan presents with bilateral eye initation and itching. Physical examination reveals photophobia, conjunctivitis, and fihrovascular opacity in the superior cornea (shown in the image). Giemsa staining of a conjunctival scrape demonstralf~s cytoplasmic inclusion bodies. What term describes the corneal lesion? ***(A) Glaucoma ***(B) Keratitis ***(C) Pannus ***(D) Pinguecula ***(E) Pterygium
1 The answer is C: Pannus. Trachoma is a chronic, coma- ..__ gious conjunctivitis caused by Ch1amydLa trac1wma£i5. When inoculated into the eye, the organism reproduces within the conjunctival epithelium, inciting a mixed acute and chronic inOammatOty infiltrate. As the disease progresses, lymphoid aggregates enlarge, and the conjunctiva becomes focal ly hypertrophic and scarred. The cornea is invaded by blood vessels, and fibroblasts form a trachomawus pannus. In children, the d]sease usually heals spontaneously. In adults, it may cause blindness, especially if combined with bacterial superinfection. Pinguecula (choice D) is a yellowish conjunctival lump usually located nasal to the corneoscleral limbus. Pterygium (choice E) is a fold of vascularized co mea not related to infec· tious agents, which grows horizontally onto the cornea in the shape of an insect wing. ***Diagnosis: Trachoma
2 A 29-year-old woman complains of nervousness and weakness of 6 months in duration. She says that she is increasingly intolerant of heat and frequently sweats. She has experienced a 9-kg (20-lb) weight loss over the past 2 momhs despite nonnal caloric intake and says that she frequemly fi.nds her heart racing Physical e-xamination reveals a symmetrically enlarged thyroid and exophthalmos. Which of the following best explains the pathogenesis of bilateral exophthalmos in this patient? ***(A) Acute orbital inflammation ***(B) Enlargemem of the eyeball ***(C) Orbital deposits of thyroid hormone ***(D) Orbital hemonhage ***(E) Swelling of the extraocular nmscles
2 The answer is E: Swelling of the extraocular muscles. The most common cause of proptosis (forward protrusion of the eye) is thyroid disease, followed by orbital dermoid cysts and hemangiomas. Exophthalmos may precede or follow other manifestations of thyroid dysfunction. Thi.s condition is caused by enlargement or the extraocular muscles within the orbit. n1e muscles themselves are normal, but they are swollen by mucinous edema, the accumu1ation of fibroblasts, and infilu ·ation by lymphocytes. Complications of severe exo phthalmos include comeal e>.rposure with subsequent ulceration and optic nerve compression. The other choices are not complications o[ hyperthyroidism. ***Diagnosis: Graves disease, exophthalmos
3 A 56-year-old obese, diabedc woman (BMI = 32. kglm2) complains of declining visual acuity. Funduscopic examination shows peripheral retinal capillary microaneurysms, deep and superficial retinal hemorrhages, "waxy'' hard exudates, areas of capillary closure, and viLreoretinal neovascularization. Urinalysis reveals 3+ proteinuria and 3+ glucosuria. Which of the fo llowing is the most imponant pathogenetic factor underlying this patiems retinopathy? ***(A) Hyperlipidemia ***(B) Hypoalbuminemia ***(C) Hypoglycemia ***(D) Retinal ischemia ***(E) Systemic hypertension
3 The answer is D: Retinal ischemia. The eye is freq uently involved in diabetes mellitus, and ocular symptoms occur in 20% to 40% of diabetics, even at the clinical onset of the disease. Virtually all patients with type l (insulin-dependent) diabetes and many of those \\rith type 2 (non-insulin-dependent) diabetes develop some background retinopathy within 5 to 15 years of the onset of diabetes. Retinal ischemia can account for most features of diabetic retinopathy, includtng cottonwool spots, capillary closure, microaneurysms, and retinal neovascularization. ischemia results from narrm.ving or occlusion of retinal arterioles (as from arteriolosclerosis or platelet and lipid thrombi) or from atherosclerosis of the central retinal or ophthalmic arteries. The frequency of proliferative retinopathy correlates with the degree of glycemic control (i.e., hyperglycemia, not hypoglycemia, see choice C)~ the better the conn ·ol, the lower is the rate of retinopathy Systemic hypenension (choice E) ma)' also be a contributing facLOr in diabetics. ***Diagnosis: Diabetic retinopathy
A 65-year-old man complains of gradual loss of peripheral vision over du~ past 2. years. Ophthalmic examination shows elevated intraocular pressure. Funduscopic examination reveals "optic cupping." Which of the following best explains lhe loss of visual field sensitivity in this patient7 ***(A) Comeal abrasion ***(B) Degeneration of the choroid ***(C) FLxed dilation of the pupil ***(D) Reti nal degeneration ***(E) Retinal ischemia
4 The answer is D: Retinal degeneration. Glaucoma refers to a collection of disorders that feature an optic neuropathy accompanied by a characteristic excavation of the optic nerve head and a progressive loss of visual field sensitivity In most cases, glaucoma is produced by increased intraocu lar p ressure (ocular hypertension). Increased intraocular pressure, however, does not necessarily cause glaucoma. ln certain pathologic states, aqueous humor accumulates within the eye, and the imraocular pressure increases. Temporary or permanent impainnem of vision results from pressure-induced degenerative changes in the retina and optic nerve head and from corneal edema and opacification. The other choices are nor complications of glaucoma. ***Diagnosis: Glaucoma
5 The patient described in Question 4 has no underlying ocular disease. Which of the following types of glaucoma is most likely present? ***(A) Closed-angle ***(B) Congenital ***(C) Low tension ***(D) Open-angle ***(E) Secondary
5 The answer is D: Open-angle. Primary glaucoma develops in a person with no apparent underlying eye disease. The disorder is subdivided into open-angle glaucoma, in which the anterior chamber angle is open and appears normal, and closed -angle glaucoma, in which the anterior chamber is sh allower than normal and the angle is abnormally narrow. Primary openangle glaucoma is the most frequent type of glaucoma and is a major cause of blindness in the United States. The angle of the anter ior chamber is open and appears normal, but increased resistance to the outflow of the aqueous humor is present within the vicinity of Schlemm canaL Primary closedangle glaucoma (choice A) afflicts persons whose iris is displaced ameriorly When the pupil (Ulates , the iris obstructs the amerior chamber angle, thereby impairing fluid drainage and raising imraocular pressure. Acute primary closed-angle glaucoma, low tension glaucoma (choice C), is an ocular emergency, and it is essential to stan ocular hypotensive treatment within the first 24 w 48 hours if vision is w be maintained. Primary closed-angle glaucoma affects both eyes, but it may become apparem in one eye 2 to 5 years before it is noted in the other. Congenital glaucoma (choice B) refers to glaucoma caused by obsnuction to the aqueous drainage by developmental anomalies. Causes of secondary glaucoma (choice E) include inflammation, hemonhage, neovascula1ization of the iris, and adhesions. ***Diagnosis: Glaucoma
6 A 28-year-old man consults his physician rega rding his vision. He describes a 6-month histmy o[ increasing nighL blindness. Two of his maternal uncles have the same condition. Visual examination reveals a marked constriction of the patients peripheral visual field. Funduscopic examination would likely show which of the follo•ving pathologic findings? ***(A) Cheny-red spot ***(B) Cotton-wool spots ***(C) Microaneurysms ***(D) Optic cupping ***(E) Pigmem accumulation
6 The answer is E: Pigment accumulation. Retinitis pigmenwsa (pigmentary retinopathy) is a gene1ic term that refers to a variety of bilateral, progressive. degenerative retinopathies. They are charactelized clinically by night blindness and consuiction of peripheral visual fields and pathologtcally by the loss of re tinal phowreceptors (mds and cones) and pigment accumulanon within d1e retina. ln reti11itis pigmemosa, the des trued on of rods and later cones is followed by migration of retinal pigmem epithelial cells into the sensory retina. Melanin appears within slender processes of spidery cells and accumulates mainly around small branching retinal blood vessels, like spicules of bone. The retinal blood vessels then gradually attenuate, and the optic nerve head acquires a cha racteristic waxy pal lor. Half of all patients with retinitis pigmentosa have a family history of the disease. Cherry-red spot (choice A) describes a bright central foveola that occurs in the setting of lysosomal sLOrage diseases and central retinal artery occlusion. Cotton-wool spots (choice B) are observed in diabetics and patients with hypertension. Microaneurysms (choice C) are encountered in patients with hypertension. ***Diagnosis: Retinitis pigmentosa
7__ A 3·year-old child presents with a red and painful left eye. OphLhalmic examination reveals swelling of the eyelid and a purulent exudate. \NhaL is the proper name for this condition? ***(A) Conjunctivltis ***(B) Glaucoma ***(C) Keratitis ***(D) Pinguecula ***(E) Retinitis
7 The answer is A: Conjunctivitis. Microorganisms lodging on the surface of t.he eye frequently cause conjunctivitis, alt.hough keratitis or corneal ulcer may also occur. At some stage in life, vinuaTiy everyone has viral or bacterial conjunctivitis. This most common of eye diseases is cha racterized by hyperemic conjunctival blood vessels (pink eye). The inflammato ry exudates that accumulate in Lhe conjunctival sac commonly crust, causing the eyelids to stick together in the morning. Choices B, C, and E affect other anatomic stmctures in the eye. Choice D (pinguecula) is a yellow lLm1p in the conjunctiva that is not. infected. ***Diagnosis: Conjunctivitis
8 A 54-year-old woman with rheumawid arthritis presems witl1 a 2-year histmy of dry eyes and dry mouth. Physical examination confmnsxerostomia (dry mouth) and xerophthalmia (dl)' eyes) and reveals enlarged lacrimal glands bil aterally Histologic examination of the lacrimal glands would likely show an infiltrate of which of the follov-ri.ng inflammatory cell types? ***(A) Fibroblasts ***(B) Lymphocytes ***(C) Macrophages ***(D) Mast cells ***(E) Neutrophits
8 The answer is 8: Lymphotytes. This patient has Sjogren (sicca) syndrome, an autoimmune disease characterized by an intense lymphocytic infiltrat.ion of the salivary and lacrimal glands. Patients develop xerostomia and xerophthalmia. The other ch oices are not characterisLic of autoimmune disease. ***Diagnosis: Sjogren syndrome
9 A 55-year-old obese woman (BMl = 34 kglm2) v-ri.th a history of hypertension and hypercholesterolemia complains of nodular lesions on the nasal aspect of both of her e)relids. The lesions are yellow and nonpainful. Biopsy of a lesion would likely show an accumulation of which of the following cells? ***(A) Fibroblasts ***(B) Lymphocytes ***(C) Macrophages ***(D) Mast cells ***(E) Neurrophits
9 The answer is C: Macrophages. Xamhelasmas are yellow plaques of lipid-containing macrophages that usually are located on the eyelids. They are frequently seen in patien ts with familial hypercholesterolemia but may occur in persons withom lipid disorders. The other cello:; do nm store hpid materiaL ***Diagnosis: Familial hypercholesterolemia, xanthelasma
._1_0 A 67-year-old man complains of a lesion in his left eye. Physical examination reveals a triangular fold of vascularized conjunctiva growing hmizomaUy into Lhe cornea in the shape of an insecl wing. Which of the following terms best describes Lhis patient's lesion? ***(A) Loiasis ***(B) Onchocerciasis ***(C) Pinguecula ***(D) Pte1ygium ***(E) Siderosis bulbi
10 The answer i.s D: Pterygium. Pterygium is a fold of vascularized conjunctiva that grows horizontally onto the cornea in the shape of an insect wing (hence the name). It is often associated with a pinguecula (conjunctival lump, choice C) and frequently recurs after excision. ***Diagnosis: Pterygium
11 A 70-year-old man who was bom prematurely and has been .___ blind since birth suffers a stroke and expl1·es. The eye at autopsy is shown in the image. Which of the following was the most likely cause of this mans blindness? ***(A) Autoimmune hemolytic disease ***(B) Birth trauma ***(C) Congenital hypothyroidism ***(D) DiabeLes ***(E) Oxygen coxicity
11 The answer is E: Oxygen toxicity. Retinopathy of prematurity. also termed retrolental fibroplasia, is a bilateral, iatrogenic, retinal disorder that occurs predominanlly in premature infants who have been treated after binh with oxygen. The entity was original ly called retrolental fibroplasia because or a mass of scaJTed tissue behind the lens in advanced cases. In premature infants exposed to excessive amounts of o;..:ygen (e.g., in an incubator), the developing retinal blood vessels become obliterated, and the periph eral retina, which is normally avascular until the end of fetal life, does not vascularize. The condition is unrelated to birth trauma (choice B). ***Diagnosis: Rennopathy of prematmity, retrolental fibroplasia
t2 A 65-year-old man complains of decreased visual acuity in one eye of 5 years in duration. Physical examination reveaJs a white discoloration in one pupil (shown in the image)_ Which of the following best explains the pathogenesis of this patients loss of vision? ***(A) Chronic conj unctivitis ***(B) Degeneration of the lens ***(C) Fibrovascular occlusion of the cornea ***(D) Increased intraocular pressure ***(E) Retinal ischemia
12 The answer is B: Degeneration of the len$. Cataracts represent opacifications in the crystalline lens. They are a major cause of visual impairment and blindness throughout the world and are the outcome of numerous conditions. Clefts appear between the lens fibers, and degenerated lens material accumulates in these open spaces. The most common cataract in the United States is associated with aging (age-related cataract), although the risk is increased with diabetes, heavy e>.:posure to uhraviolet light, certain genetic disorde rs, and many other conditions. The degenerated lens material exerts osmodc pressure, causing the damaged lens to increase in volume by imbibing water. Such a swollen lens may obstruct the pupil and cause glaucoma (phacomorphic glaucoma). ln a mature cataract, the entire lens degenerates, and its volume diminishes because lenticular debris escapes into the aqueous humor through a degenerated lens capsule (hypennature cataract). The other choices do not aCfect the ocular lens. ***Diagnosis: Cataract
13 A 58-year-old obese woman (BMI = 32 kglm1) complains of declining visual acuity of 6 momhs in duration. Physical examination reveals a pulse of 82 per minme, respirations 20 per minute, and blood pressure 195/110 nm1 Hg. Funduscopic examination shows "cotton-wool spots," retinal hemorrhage, macular star, edema of the optic nerve , and aneriovenious nicking of retinal anerioles. A photograph of the ocular fundus is shown. These findings are best e>.1)1ained by which of the following mechanisms? ***(A) Central retinal artery occluswn ***(B) Central retinal vem occlusion ***(C) Gangliosidosis ***(D) Hypcnensive retmopathy ***(E) Pigmenta1y retinopathy
13 The answer is D: Hypertensive retinopathy. Characte1isric feamres of hypertensive retinopathy include arteriolarnan owing, hemorrhages in the retinal nerve fiber layer (flame-shaped hemorrhages), e-x'lldates, including some that radiate from the center of the macula (macular star). Arteriolosclerosis accompanies longstanding h ypertension and commonly affects the retinal and choroidal vessels. The lumina or the thickened retinal arterioles become narrowed, increasingly wnuous, and of irregular caliber. At sites where the arterioles cross veins, the latter appear kinked (aneriovenous nicking). Small super· ficial or deep retinal hemorrhages often accompany retinal arteriolosclerosis. Impaired axoplasmic Oovv within the nerve fiber layer, caused by ischemia, results in swollen axons with cytoplasmic bodies. Such stmctu res resemble cotton on funduscopy (cotton-wool spots}. None of the other choices are associated with the ophthalmologic features shown in the photomicrograph. ***Diagnosis: Hypertensive retinopathy
14 A 67-year-old man with a history of ischemic heart disease presents ·with 4 days of blurred vision in hjs left eye. Whkh of the following finrungs on funduscopic examination would characterize the pathologic consequences of central retinal vein occlusion in thjs patient? ***(A) Cherry-red macula ***(B) Couon-wool patches ***(C) Flame-shaped hemorrhages ***(D) Macular star ***(E) Microaneurysms
14 The answer is C: Flame-shaped hemorrhages. Central retinal vein occlusion is more common than occlusion of the retinal ane1y. It typically evolves gradually and has a better prognosis_ Funduscopic examination typically reveals flame-shaped hemorrhages. The mLTaocular pressure tends to be elevated. Retinal hemorrhages are also a feature of oLher disorders, including hypertension and diabetes mellitus, as well as cemral retinal vein occlusion_ The appearance varies with the location. Hemorrhages in the nerve fiber layer spread between axons and cause a flame-shaped appearance on fLmduscopy, whereas deep retinal hemonhages tend w be round_ When located between the retinal pigmem epithelium and Bruch membrane, blood appears as a dark mass and clinically may resemble a melanoma. The other choices are not characteristic of cemral retinal vein occlusion. ***Diagnosis: Retinal occlusive vascular disease
15 If the patient described in Question 14 suffered fTom cenn·al retinal ane1y occlusion , which of the following pathologic findings would be e.-...'Pecred on funduscopic examination? ***(A) Cherry-red macula ***(B) Cotton-wool patches ***(C) Macular star ***(D) Microaneurysms ***(E) Neovasculatization
15 The answer is A: Cherry-red macula. Central retinal anery occlusion may follow thrombosis of the reth1al anery, as in atherosclerosis, giam cell aneritis, or embolization to that vesseL lmracel- 1ular edema, manifested by retinal pallor, is prominent, especiaily in the macula, where the ganglion cells are most numerous. The foveola stands out in sharp conu·ast as a prominent cheny-red spor. The lack of retinal circulation reduces the retinal anelioles to delicate tlu·eads_ Permanem blindness follows cemral retinal ane1y ohstmction unless ischemia is o£ shon duration. Cherryred macula is also seen in patients with lysosomal swrage diseases. In these patients, intracytoplasmic lysosomal inclusions within the multilayered ganglion cell layer of the macula irnpan a striking pallor to the affected redna. As a result, the central foveola appears b1ight red because of the underlying choroidal vasculature. Cotton-wool patches (choice B) and nem7ascularizadon (choice E) are noted in patients with diabetic retinopathy. Macular star (choice C) and microaneUJysms (choice D) occur in patients with hypertensive retinopathy ***Diagnosis: Reunal occlusive vascular disease
16 A 60-year-old woman with a history of aduh-onseL (type 2) diabetes suffers a massive stroke and expires. A flat preparadon of the retina at autopsy is shown in the image. Which of the following pathologic features of diabetic retinopathy is evident il1 this specimen? ***(A) Cotton-wool patches ***(B) Flame-shaped hemorrhages ***(C) lnllammatory exudate ***(D) Lacy vacuolization ***(E) Microaneurysms
16 The answer is E: Microaneurysms. This autopsy specimen '-- shows capiUary mkroaneurysms. Background diabetic retinopathy exhibits capillary microaneurysms, small hemorrhages (dot and blot hemorrhages), and exudates. These lesions usuaUy do not impair vision unless associated with macular edema. After many years, diabetic retinopathy becomes proliferative. Delicate new blood vessels grow along with fibrous and glial tissue toward the vitreous body. Neovasculmization of the retina is a prominent feature of diabetic retinopathy and of other conditions caused by retinal ischemia. Tortuous new vesse 1s first appear on Lhe surface of the retina and optic nerve head and then grow into the vitreous cavily. None of the other choices are shown in the photomicrograph. ***Diagnosis: Diabetic retinopathy
17 A 13-year-old girl presents because of increased thirst, hunger, and frequem utinarion over the past few momhs. Urinalysis discloses 3+ glucosuria. Tile blood sugar is 2 75 mgldL The patient receives counseling on the selfadministration of insulin. Six years later, she complains of blurred vision. Ophthalmic examination reveals "snowllake" cauuacrs. An accumulation of which of the following biologic compounds best accounts for the pathogenesis of cataracts in this patient? ***(A) Glycogen in the iris ***(B) Hemoglobin in the lens ***(C) Lipid in the lens ***(D) Lipid in the retina ***(E) Sorbiwl in the lens
17 The answer is E: Sorbitol in the lens. Patients \vith type 1 dia- .__ betes often develop bilateral "snowOake" cataracts. These consist of a blanket of white needle-shaped opacities in the lens immediately beneath the anterior and poster ior lens capsule. The opacities coalesce within a few weeks in adolescents and within days in children, until the whole lens become opaque. Snowflake cat.aracts can be produced experimentally in young animals and result from an osmotic effect caused by lhe accumulation o[ sorbitol, the alcohol de1ived (rom glucose. The increased sorbitol comem of the lens causes imbibition of water and enlargement of the lens. The accumulation of glycogen (choice A) in the iris causes lacy vacuollzation. Upemia rer.inalis (choice D) accounts for the wa>.'Y streaks seen by funduscopy. Bleeding (choice B) occurs in the cornea and vitreous body and can cause glaucoma. ***Diagnosis: Cataract, diabetic retinopathy
18 A 50-year-old man presents \Vith loss of vision in his left eye after a motorcycle accidenL Funduscopy reveals separation of the sensory retina from the retinal pigmemary epit. helmm. Which of the following is the likely cause of retinal detachment? ***(A) Autoimmune uveitis ***(B) Increased intraocular pressure ***(C) 1nt.raocular hemorrhage ***(D) Macular degeneration ***(E) Retinitis pigmentosa
18 The answer is C: Intraocular hemorrhage. Traumatic hemorrhages separate the senso1y retina from the retinal pigment epithelium. Factors predisposing to retinal detachment also include retinal de fects (e.g. , certain retinal degenerations), vitreous traction, and diminished pressure on the retina (e.g., after vi treous loss). After accidemal or surgical perforation of the globe, choroidal hemorrhages may detach the choroid and displace the retina, vitreous body, and lens through the wound. The other choices are not associated with retinal detachment. ***Diagnosis: Retinal detachment
19 A 15-year-old boy sees an optometrist w obmin his first pair of eye glasses. An eye examination reveals 20/80 vision in both eyes. Which of the following best explains the pathogenesLs of myopia in thts patiem? ***(A) Decreased anteroposterior diameter of the eye ***(B) Increased anteroposterior diameter of the eye ***(C) Increased thickness of the lens ***(D) Sensory rednal degeneration ***(E) Water retention in the aqueous humor
19 The answer is B: Increased anteroposterior diami!ter of the eye. Myopia (near-sightedness) is a refractive ocular abnormality in which light from Lhe visualized object focuses at a point in front of the retina because of a longer than usual anteroposterior diameter of the eye. Myopia affects more than 10 million persons in the United States, requiring correction wi th glasses, contact lenses, or laser treatment. Choice A causes the focal point to be behind the retina causing hyperopia (far-sightedness). The other choices do not involve the point of focus. ***Diagnosis: Myopia
20 A -+-year-old boy presents with a white pupil that his parents first nodced about 4 months ago. A history of an eye tumor in a first cousin is provided. Ophthalmic examination shows a tumor in the left eye (patient shown in the image), after which the eye is enucleated. A mutation in which of the following genes was mosL likely associated with the develop· ment of this tumor? ***(A) p53 ***(B) C-myc ***(C) N-myc ***(D) Rb ***(E) RET
20 The answer is D: Rb. Retinoblastoma is the most common intraocular malignant neoplasm of childhood, affecting 1:25,000 children. Most frequently the tumor is seen within the first 2 years of life and sometimes even at birth. The presenting signs include a while pupil (leukocoria), squint (strabismus), poor vision, spomaneous hyphema, and a red, painful eye. Secondary glaucoma is a frequent complication. Although most retinoblastomas occur sporadically, about 7% are inhe1ited and are related to a germline mutation in the retinoblastoma gene, which functions as a tumor suppressor. The Rb protein functions to regulate the cell cycle at a checkpoint before S phase. RET protooncogene mmations (choice E) are seen in patiems with multiple endocrine neoplasia type 2. ***Diagnosis: Retinoblastoma
21 A 55-year-old woman presents with a 3-month history of reduced visual acuity of her tight eye. Funduscopic examination reveals a dark mass located beneath the choroid vessels. Biopsy of the mass shows spindle-shaped cells and epithelioid cells, many of which contain pigment The enucleated eye is shown in the image. This mushroom-shaped rumor most likely miginated from which of the following anatomic Jocations7 ***(A) Cornea ***(B) Lens ***(C) Retina ***(D) Sclera ***(E) Uvea
21 The answer is E: Uvea. The eye and adjacent strucmres contain a variety of cell types, and malignant neoplasms arise from them. Malignant melanoma is the most conunon primary intraocular malignancy. The tumor atises within the eye from uveal melanocytes or nevi. Microscopically, uveal melanomas are generally composed of variable numbers of spindle-shaped cells and polygonal cells wi th distinct cell borders and prominem nucleoli (epithelioid cells). Melanocytes are found in boLh the uvea and the retina, but the retinal melanocyt.es do not unde rgo malignant transfom1ation as readily as those in the uvea. The choroid is the most common site of origin of ocular melanomas. Retinoblastoma a1ises from immature retinal neurons (choice C). ***Diagnosis: Melanoma
22 A 67-year-old woman complains that she has had poor vision for 4 years. Ophthalmic examination shows a selective loss of cenu·a] vision. Which of the following is the most likely diagnosis? ***(A) Cataract ***(B) Corneal dystrophy ***(C) Gaucher disease ***(D) Macular degeneration ***(E) Retinitis pigmentosa
22 The answer is D: Macular degeneration. The most common cause of amblyopia (impaired visual acuity) in the United States is age-related macular degeneration. The cemer of the macula (foveola) is the point of greatest visual acuity. In this area, a high concemration of cones rests on the retinal pigmem epithellum. Surrounding the macula, the retina has a multUayered concentration of ganglion ceUs_ Wi lh aging, the macula may degenerate, in which case cemral vision is impaired. Retinitis pigmemosa (choice E) begins in the periphery of the retina, leading to night blindness_ The other choices are not associated \vith central vision. ***Diagnosis: Macular degeneration
1 A 31-year-old woman complains of increased vaginal discharge of 3 months in duration. A cervical Pap smear is obtained. Two superficial squamous cells (shown in the image) display enlarged nuclei and perinuclear vacuoles (koilocywtic atypia). What is the appropriate diagnosis? ***(A) Chlamydia infection ***(B) Endometrial adenocarcinoma ***(C) Herpes simplex vims infection ***(D) Human papiUomavims (HPV) infection ***(E) Invasive squamous cell carcinoma
,__1_ The answer is D: Human papillomavirus (HPV) infection. HPV is a DNA virus that infects a variety of skin and mucosal surfaces to produce wart-like lesions {ven ucae and condylomata). In the female reproductive tract, HPV in£ections are linked to the pathogenesis of cervical cancer. The morphologic hallmark of HPV infection is koilocytotic atypia, a term that denotes the presence o[ sharply demarcated, large petinuclear vacuoles, combined with alte rations in the ch romatin pattem of squamous epithelial cells. These pe rinuclear vacuoles are filled wilh replicating virus panicles. Endomenial adenocarcinoma (choice B) demonstrates eccentric nuclei with irregular nuclear membranes and abnormally distributed chromatin. Herpes simplex virus infection (choice C) demonstrates multinucleation and "ground glass'" nuclei. lnvasive squamous cell carcinoma (choice E) is characterized by pleomorphic elongate squamous cells, with enlarged, irregular, and hyperchromatic nuclei. ***Diagnosis: Human papillomavirus
2 A 33-year-old woman complains of vaginal spotting after intercourse. A ce rvical Pap smear is taken, and the results are shovvn in the image. A normal squamous epimelial cell is in the 1ight upper comer. Which of the following is me most likely diagnosis? ***(A) Herpes simplex vims infection ***(B) High-grade cervical dysplasia ***(C) Huncan papillomavims infection ***(D) Low-grade cervical dysplasia ***(E) Vaginal clear cell adenocarcinoma
2 The answer is 8: High-grade cervical dysplasia. Cervical squamous intraepithelial lesions {SlLs) show a spectrum of changes from low-grade SIL with mild dysplasia (cervical intraepithelial neoplasia, CINl) to high-grade SIL with severe dysplasia (CIN3, carcinoma in situ). In low-grade CINl, the dysplastic cells have abundam cyLoplasm, and the nuclei are enlarged and hyperclu·omaLic. ln high-grade dysplasia, the dysplastic cells have a higher nuclear-cyLDplasmic ratio. l ow-grade dysplasia (choice D) is incorrect because the dysplastic cells shown here have minimal cytoplasm and a very high nucleus w cytoplasm raLio. Herpes simplex virus infection (choice A) shows multinucleation and "ground glass" nuclei. Human papillomaVlfUs infection (choice C) features perinuclear vacuoles. ***Diagnosis: Cervical inrraepilhelial neoplasia
3 For the patiem described in Question 2. which of the following cytologic features observed in the Pap smear would provide the best evidence to substantiate a diagnosis of invasive squamous cell carcinoma? ***(A) Atrophy ***(B) Dysplasia ***(C) Metaplasia ***(D) Nuclear inclusion bodjes ***(E) Pleomorphism
3 The answer is E: Pleomorphism. Squamous imraepithelial lesions must be distinguLshed from invasive lesions. In invasive squamous cell carcinoma, the neoplastic cells are usually pleomorphic, with caudate and elongate forms, and coarsely gnmular or pyknmic chromatin. Choices A. B. and C represem morphologic characteristics o£ cellular adaptation to chronic persistent ~mess . Nuclear inclusion bodies (choice D) likely represent viral in fection. ***Diagnosis: Squamous cell carcinoma of Lhe cervi'<
4 A 30-year-old woman presents for a routine physical examination. A cervical Pap smear is caken. A cluster of multinucleated squamous cells is shown in the image. A nomcal super ficial squamous cell is present at the wp of the field. This patiem most likely has which of the foUmving gynecologic diseases? ***(A) Endometrial adenocarcinoma ***(B) Herpes slmplex virus infection ***(C) High-grade cervical dysplasia ***(D) Human papiUomavims infection ***(E) Invasive squamous cell carcinoma
4 The answer is 8: Herpes simplex virus infection. Herpes simplex virus type 2 ls a DNA virus that is a common cause of sexually transmitted genital infections Muldnucleatecl giant cells with "ground glass" appearance of the nuclei are typical. The or her choices do not present with multinucleated giam cells. ***Diagnosis: Genital herpes
5 An 85-year-old woman presents with increasing fatigue and weight loss over the past 3 months. A CT scan of the abdomen reveals a 4-cm retroperitoneal mass. Fine-needle aspiration reveals numerous, large, noncleaved dissociated cells. By flow cytometry, the cells are positive for CDlO, CD19, CD20, IgM, and lambda light chain. Which of the following is the appropriate diagnosis? ***(A) Acure myelogenous leukemia ***(B) Adult T-cel llymphoma leukemia ***(C) B-celllymphoma ***(D) Hodgkln disease ***(E) T-celllymphoma
5 The answer is C: B-celllymphoma. Lymphomas are malignant L...- proliferallons of lymphocytes or lymphoblasts. The World Healrh Organization c.lassifica.uon distingu lshes be[ween Hodgkin lymphoma and B-cell and T-cell lymphomas (nonHodgkin Lymphomas). B-cell lymphomas are charactetized by Lheir expression of cell surface immunoglobulin heavy and light chains and other cell surface CD antigens (e.g., CD20). The other choices do not express cell surface immunoglobulin heavy or light chains. ***Diagnosis: B~cell lymphoma
6 A 59-year-old man presems with a 3-week history of painless hemamria. Physical examination is unremarkable. Urinalysis shows 2+ hematuria. A CT scan of the abdomen reveals an exophytic mass in the renal pelvis. Cytologic examination of a fine-needle aspiration of the renal pelvis is shown in the image. These cells show which of the following patterns of arrangement? ***(A) Follicular configuration ***(B) Glandular stmctures ***(C) Papillary configuration ***(D) Pearls ***(E) Rosettes
6 The answer is C: Papillary configuration. The relation between cells is a h elpful criterion for cytologic diagnosis. Cells may appear singly. in small groups, in monolayer sheets, in syncytia, or in th ree-dimensional clusters. Cell dusters may form papil· lary configurations (e.g., papillary transitional cell carcinoma), glandular structures (e g., adenocarcinoma), folli cles (e.g., follicular adenoma of thyroid), rosettes (e.g , neuroblastoma), or pearls (e.g , squamous cell carcinoma). Transitional cell carcinomas (this case) typically fonn papillary structures with a fibrevascular core. The nuclei are crowded and hyperchromatic. ***Diagnosis: Transitional cell carcinoma of the urothelium
7 A 2-momh-old girl is brought to the physician because her parents noticed an enlargement in her abdomen. Physical examination reveals a flank mass over the left kidney. Twentyfour- hour urinalysis shows elevated levels of metanephrines. A CT scan of the abdomen reveals an 8-cm suprarenal nutss. Fine-needle aspiration of the mass is performed. Cytologic examination of the aspirate would most likely show neoplastic cells with which of the following patterns of arrangemen[7 ***(A) Glandular stlucLures ***(B) Papillae ***(C) Pearls ***(D) Rosettes ***(E) Syncytia
7 The answer is D: Rosettes. The clinical features of this case are most consistent with neuroblastoma. This maUgnant neural crest mmor is composed of neuroblasts and originates in the adrenal medulla or sympathetic ganglia. Microscopically, the tumor is composed of dense sheets of small, round to fusifom1 cells with hyperchromatlc nuclei and scanty cytoplasm. Characteristic rosetles are defined by a rim of dark tumor cells in a circumferential anangement around a central pale fib1illar core. ***Diagnosis: Neuroblastoma
8 A 60-year-old man presenrs with a 4-month history of increas· ing weight loss and shortness of breath. He has smoked two packs of cigarenes a day for 20 years. His past medical history is significant for emphysema and chronic bronchitis. A chest X-ray shows a mass in the left main stem bronchus. Cytologic examinadon of a bronchial bmshing shows pleomorphic keratinized cells with large irregular and pyknotic nuclei. These cells would most likely show which of the following pauerns of anangement on cytologic examination? ***(A) Follicular configuration ***(B) Glandular stmctures ***(C) Papilla1y configuration ***(D) Pearls ***(E) Roseues
8 The answer is D: Pearls. Squamous cell carcinoma accoun ts for 30% of all invasive lung cancers in the United States. Following chronic injury (e .g., dgareue smokm.g), the metaplastic bronch.i.al epithelium follows a sequence of dysplasia, carcinoma in sitLt, and invasive carcinoma, similar to that in the cervi..'X. Well-differemlated tumors display keratin "pearls," whlch appear as small, rounded nests of brightly eosinophilic aggregates of keratin surrotmded by concentric layers of squamous epithelial cells. Glandular structures (choice B) would be indicative of adenocarclnoma. The other choices are unlikely LO exhibit keratinization. ***Diagnosis: Squamous cell carcinoma of the lung
9 A .36-year-old HIV-positive man presents with persistent d1y cough, fatigue, and low-grade fever. Physical examination shows marked pallor and respiratory distress. An X-ray film of the chest reveals diffuse, bilateral, interstitial infiltrates. Bronchoalveolar lavage demonsu-ates a foamy alveolar cast composed of small cysts (shown in the image) 'vVhat is the most likely diagnosi..s? ***(A) Adenocarcinoma ***(B) Herpes vi ms infection ***(C) Human papillomavirus infection ***(D) Pluumocyscis ca~ini1 pneumonia ***(E) Small cell carcinoma
9 The answer is D: Pneumotystis carinii pneumonia. P. ca1inii (now correctly termed P. Jlroveci) causes progressive, often fatal pneumonia in persons with severely impaired cell-med iated immunity and is one of Lhe most common opportunistic pathogens in persons with AIDS. About 80% of all untreated patients with AIDS develop P. cmi11ii pneumonia in the course of their illness. The morphologic hallmark of P. cmimi pneumonia is foamy alveolar casts composed of small cysts. None of the other choices display a foamy appearance shown in the photomicrograph. ***Diagnosis: P11eumocystis pneumonia
10 A 45-year-old woman complains or gradual swelling in her neck or 6 months in duration. Physical examination reveals an enlarged and nodular thyroid gland. A fine-needle aspiration of a n odule in the left lobe of rhe thyroid is performed. Cywlogic examination of rhe aspirate shows follicular cells, macrophages, and abundant colloid. What is the appropriate d1. agnost.s ?. ***(A) Chronic lymphocytic thyroiditis ***(B) Follicular adenoma ***(C) Hunhle cell carcinoma ***(D) Multinodular goiter ***(E) Papilla!) ' carcinoma of Lhyroid
10 The answer is D: Multinodular goiter. Multinodular (nontoxic) goiter refers to an enlargement of tl1e thyroid that is not associated with functional, mnammaLOry, or neoplastic alterations. Microscopically, thyroid nodules are lined by hyperplastic follicular epithelial cells and are distended with colloid. The other choices would show neoplastic cells (choices B, C, and E) or chronic innammatory cells (choice A). ***Diagnosis: Goiter
11 A 35-year-old woman comes to the physician after noticing a lump in her neck. Physical examination reveals a palpable nodule in an enlarged thyroid gland. A fine-needle aspiration of the left lobe of the thyroid is perfonned. Cytologic examination of the aspirate reveals malignam celts (shown in the image). These cells show which or tl1e following pauerns of arrangement, approp1iate for rumor cells arising in this location? ***(A) Apocrine glands ***(B) Follicular configuration ***(C) Keratin pearls ***(D) Mucin-secreting glands ***(E) Mttltinucleated giant cells
11 The answer is 8: Follicular configuration. Follicular Lhyroid carcinoma does nol contain papillae or other architectural elements. On cytologic examination, the tumor cells fom1 small follicles, with scant colloid and mild nuclear atypia. Keratin pearls (choice C.) or mucin secretion (choice D) are not observed. ***Diagnosis: Follicular carcinoma of the thyroid
12 A 35-year-old woman presents after nmicing a gradual enlargement ofher neck over the past 6months. Physical examination reveals a diHusely enlarged thyroid gland. Cytologic examination of a fine-needle aspirate of the thyroid shows follicular cells and abundam lymphoid cells. 'Which of the following is the approp1iate diagnosis? ***(A) Follicular adenoma ***(B) Goiter ***(C) Graves disease ***(D) Hashimoto thyroiditis ***(E) Himh le cell carcinoma
12 The answer is D: Hashimoto thyroiditis. Chronic, amoimmune thyroiditis displays a conspicuous infiltrate of lymphocyLes and plasma cells. The inflammatory infiltrates are focally arranged in lymphoid follicles, often with genninaJ centers. Lymphoid infiltrates are not features of the other choices. ***Diagnosis: Hashimoto thyroid1tis, chronic lymphocydc thyroiditis
13 A 67-year-old man presents wi.th chest wall pain and chronic nonproductive cough. A chest X-ray reveals a pleural-based mass and a large pleural effusion. A sputum sample is shown in the image. Fluid obtained from the left pleural cavity shows numerous single and clustered neoplastic cells. Which of the following is the most bkely diagnosis7 ***(A) Mahgnam lymphoma ***(B) Mahgnanl melanoma ***(C) Mesothelioma ***(D) Squamous cell carcinoma ***(E) Transitional cell carcmoma
13 The answer is C: Mesothelioma. The sputum sample shows a fermginous body, which appears as a yellow, beaded strucrures with clubbed ends. Also termed asbestos bodies, they are formed by the precipitation of iron and protein complexes on asbestos fibers. Asbestos e:Kposure is a well-known risk factor for the developmenr of mesothelioma. The other choices are not associated with asbestos exposure. ***Diagnosis: Mesothelioma, ferruginous bodies
14 A 54-year-old man presents with a 4-month history of progressive weight loss. He has smoked three packs of cigarettes a day for 30 years. An X-ray film of the chest shows a mass in the right upper lobe. The bronchial brush specimen is sho\\rn in the image. Special stains are positive for mucin. 'vVhich of the following is the most likely diagnosis? ***(A) Adenocarcinoma ***(B) Malignam lymphoma ***(C) Small cell carcinoma ***(D) Squamous cell carcinoma ***(E) Transitional cell carcinoma
14 The answer is A: Adenocarcinoma.. Adenocarcinoma accounts for a third of all invasive lung cancers. Cytologic e.'Camination of bronchial brush specimens shows clustered epithelial cells with highly at)rpical nuclei, prominent nucleoli, and cytoplasmic vacuoles. Mucin secretion is rarely, ii ever, encountered in the other choices. ***Diagnosis: Adenocarcinoma of the lung
15 A 55-year-old woman presents with a +-month history or increasing abdominal girth and pelvic discomfort. Physical examination reveals ascites, and pelVJc examination demonstrates an adnexal mass. A blood test for CA-125 is positive. A tap of the ascites returns numerotts epithelial cells with papmary fTonds. A small cluster of cells is shmvn in the image. Which or the following is the most likely diagnosis? ***(A) Adenocarcinoma of the ovary ***(B) Malignant melanoma ***(C) Mesmhelioma ***(D) Neuroblastoma ***(E) Transitional ce II carcinoma of bladder
15 The answer is A: Adenocarcinoma of the ovary. Ovarian cancer is most common in women between the ages of 40 and 60 years. By the time an ova1ian cancer has attained a size of 10 to 15 em, it has often spread beyond the ovary and seeded the peritoneum ("peritoneal carcinomatosis"). Monoclonal antibody to a tumor antigen (CA-125) detects about 90% of tumors that have already spread. Tile neoplastic cells shown here exhibit nuclear atypia, and their cytoplasm contains prominent secretory vacuoles. Although the other choices may metastasize to the ovary, they are unlikely to express CA-125 or demonstrate secretory vacuoles, ***Diagnosis: Adenocarcinoma of the ovary
16 A 6+-year-old man presents with a history of weight loss and progressive ye llo\.\ring of the skin and sclera. Ultrasound examination of the liver shows multiple lesions. A fineneedle aspiration i.s performed under ultrasound guidance. The smear discloses numerous round cells, wilh poorly defined C}'toplasm, eccentric nuclei, and p rominent nucleoli. The cytoplasm of these ce ils contains pigmented granules (shown in the image). Immunohistochemical stains are positive for the HMB-45 antigen. What is the appropriate diagnosis? ***(A) Hepaoc adenoma ***(B) Hepawcellular carcinoma ***(C) Melastadc adenocarcinoma ***(D) Melastalic melanoma ***(E) MelastaLic squamous cell carcinoma
16 The answer is D: Metastatic melanoma. The brown pigment in the cytoplasm is composed of fine melanin granules, which is characteristic of malignant melanoma. HMB-45 antigen is commonly eA')J ressed in melanoma cells. l mracellular pjg111em is nor encoumered in the other choices. ***Diagnosis: Melanoma